Download as pdf or txt
Download as pdf or txt
You are on page 1of 613

Chapter 1

1. The opening up of hospital visiting policies for children and families likely
resulted from the work of which individual?
A. Joseph Brennaman
B. John Bowlby
C. Marshal Klaus
D. John Kennell

Answer: B

Rationale: In 1951, John Bowlby received worldwide attention with his study that
revealed the negative results of the separation of child and mother because of
hospitalization. His work led to a re-evaluation and liberalization of hospital visiting
policies for children. Joseph Brennaman suggested that a lack of stimulation for
infants contributed to high infant mortality rates at the time. In the 1970s and
1980s, physicians Marshall Klaus and John Kennell carried out important studies on
the effect of the separation of newborns and parents. They established that early
separation may have long-term effects on family relationships and that offering the
new family an opportunity to be together at birth and for a significant period after
birth may provide benefits that last well into early childhood.
Question format: Multiple Choice
Chapter 1: The Nurse's Role in a Changing Maternal-Child Health Care Environment
Cognitive Level: Remember
Client Needs: Health Promotion and Maintenance
Integrated Process: Nursing Process
Reference: p. 4

2. An expectant mother states that she read that more black mothers die in
childbirth than do white mothers. When responding to her questions about the
reasons for this, the nurse accurately states that which is the major reason for the
high maternal mortality rate?
A. having formal education.
B. being unmarried.
C. income.
D. lack of prenatal care.

Answer: D

Rationale: Research shows that maternal mortality rate is directly related to lack of
prenatal care secondary to lack of access to services or insurance. Income as well
as educational level may play a role in the availability of health care, but they are
not directly responsible. Being unmarried has no bearing on infant mortality.
Question format: Multiple Choice
Chapter 1: The Nurse's Role in a Changing Maternal-Child Health Care Environment
Cognitive Level: Understand
Client Needs: Health Promotion and Maintenance
Integrated Process: Nursing Process
Reference: p. 9

3. Which statement correctly defines the term "infant death rate"?


A. number of deaths in utero of fetuses 500 g or more per 1,000 live births
B. number of deaths occurring in the first 28 days of life per 1,000 live births
C. number of deaths occurring at birth or in the first 12 months of life per 1,000
live births
D. death of a live-born child before his or her first birthday.

Answer: D

Rationale: The term infant death refers to the death of a live-born child before he
or she reaches age 1 year. It also includes neonatal mortality rate. Neonatal
mortality rate is the number of infant deaths during the first 28 days of life for
every 1,000 live births. Infant mortality rate is the number of deaths during the
first 12 months of life per 1,000 live births.
Question format: Multiple Choice
Chapter 1: The Nurse's Role in a Changing Maternal-Child Health Care Environment
Cognitive Level: Remember
Client Needs: Health Promotion and Maintenance
Integrated Process: Nursing Process
Reference: p. 9

4. The nursing instructor is preparing to teach a group of students about the history
of maternity care. What major development will the instructor emphasize as
greatly influencing the practice of maternity care in the United States over the past
century?
A. technologic advances and the use of forceps by primary care providers
B. development of anesthesia and acceptance of the germ theory
C. advent of birthing centers and the development of family-centered care
D. development of pediatric specialty and replacement of midwives as primary birth
attendants

Answer: B

Rationale: The emphasis should be placed on anesthesia and the germ theory. The
development of anesthesia allowed women a choice for pain management in birth;
the germ theory advanced the progress of general health care and decreased
infections in laboring women. Pediatrics as a specialty is an important step forward
but is not the greatest development, and midwives are still in practice. Maternity
care continues to evolve, and birthing centers are still under development. Forceps
are not considered an advance in maternity care.
Question format: Multiple Choice
Chapter 1: The Nurse's Role in a Changing Maternal-Child Health Care Environment
Cognitive Level: Analyze
Client Needs: Health Promotion and Maintenance
Integrated Process: Teaching/Learning
Reference: p. 3

5. When preparing a client for surgery, the nurse explains that the use of an
antiseptic during the surgery has greatly improved client survival rates and was
started by which physician?
A. Louis Pasteur
B. Ignaz Philip Semmelweis
C. Joseph Lister
D. Alexander Gordon

Answer: C

Rationale: Joseph Lister was a British surgeon who embraced Louis Pasteur's theory
of microorganisms as the cause of infection. Lister used carbolic acid as an
antiseptic during surgery and improved the survival rates of his patients. Alexander
Gordon proposed the theory of infection. Oliver Wendell Holmes and Ignaz Philip
Semmelweis confirmed his theory.
Question format: Multiple Choice
Chapter 1: The Nurse's Role in a Changing Maternal-Child Health Care Environment
Cognitive Level: Apply
Client Needs: Safe, Effective Care Environment: Safety and Infection Control
Integrated Process: Nursing Process
Reference: p. 4

6. A young couple are very excited to learn they are expecting their first child and
question the nurse about which test they need to discover its gender. Which
procedure will best provide this information to the couple?
A. Ultrasound
B. Amniocentesis
C. Chorionic villus sampling
D. HGP

Answer: A

Rationale: Ultrasound is a visual method for assessing the fetus in the uterus and
will provide information about the gender. Amniocentesis and chorionic villus
sampling provide the entire genetic code of the fetus. HGP refers to the Human
Genome Project, which can provide information regarding gene mutations and
variations.
Question format: Multiple Choice
Chapter 1: The Nurse's Role in a Changing Maternal-Child Health Care Environment
Cognitive Level: Apply
Client Needs: Health Promotion and Maintenance
Integrated Process: Caring
Reference: p. 6
7. A family is seeking sources for payment of health care costs. When talking with
the family, the nurse determines that the family makes too much money to qualify
for the federal program. Which source would the nurse likely recommend?
A. Medicaid
B. Special Supplemental Nutrition Program for Women, Infants, and Children (WIC)
C. Centers for Disease Control
D. State Child Health Insurance Program

Answer: D

Rationale: The State Child Health Insurance Program, first known by its acronym
"SCHIP" now referred to as "CHIP," was enacted in 1997. CHIP provides health
insurance to newborns and children in low-income families who do not otherwise
qualify for Medicaid and are uninsured. Based on the information given, the family
does not qualify for Medicaid, a federal program that supplies grants to states to
provide health care for individuals who have low incomes and meet other eligibility
criteria. The Special Supplemental Nutrition Program for Women, Infants, and
Children (WIC) provides nutritional servides to low-income, nutritionally at-risk
pregnant, breast-feeding, and postpartum women and their children (as old as 5
years). The Centers for Disease Control does not provide funding for health care
services.
Question format: Multiple Choice
Chapter 1: The Nurse's Role in a Changing Maternal-Child Health Care Environment
Cognitive Level: Apply
Client Needs: Safe, Effective Care Environment: Management of Care
Integrated Process: Caring
Reference: p. 8

8. After discussing the various options with a pregnant client and partner, they
have chosen to stay with their newborn and receive care from one nurse at the
time of birth. This is referred to as which type of care?
A. Regionalized care
B. Maternal-child care
C. Centralized care
D. Couplet care

Answer: D

Rationale: Couplet care is care in which the mother and child remain in the same
room after labor/delivery through the postpartum period. This has become the
standard of care. Regionalized or centralized care places the treatment centers in
centralized locations and transfers the patient to the facility. Maternal child care
allows the mother to be the primary provider of care.
Question format: Multiple Choice
Chapter 1: The Nurse's Role in a Changing Maternal-Child Health Care Environment
Cognitive Level: Apply
Client Needs: Safe, Effective Care Environment: Management of Care
Integrated Process: Caring
Reference: p. 5

9. A group of nursing students are examining the data of the local hospitals to
determine the potential maternal needs of the community. Which factor will be the
best statistical indicator of the adequacy of prenatal care?
A. Number of prenatal visits
B. Maternal mortality rate
C. Infant mortality rate
D. Infant measurements at birth

Answer: B

Rationale: Maternal mortality rate is the best indicator of a country's level of


prenatal care. Increases in prenatal care result in a decrease in maternal mortality.
Infant mortality is a reflection of postnatal care. Infant mortality includes all infant
deaths from birth to 12 months and can be the result of congenital anomalies,
genetic anomalies, or other problems. The number of prenatal visits and
measurements of the infant at birth are not the best indicators as they do not
accurately reflect the health of the mother.
Question format: Multiple Choice
Chapter 1: The Nurse's Role in a Changing Maternal-Child Health Care Environment
Cognitive Level: Apply
Client Needs: Health Promotion and Maintenance
Integrated Process: Nursing Process
Reference: p. 9

10. The nurse is teaching a client about lifestyle changes that could help the client
reach a higher level of wellness. The nurse is engaged in which aspect of care?
A. Infection prevention
B. Cost containment
C. Health promotion
D. Illness treatment

Answer: C

Rationale: By encouraging healthy lifestyle changes, the nurse is engaging in health


promotion activities. Health promotion involves helping people make lifestyle
changes to move them to higher levels of wellness. Health promotion includes all
aspects of health: physical, mental, emotional, social, and spiritual. Although health
promotion can help to contain costs over a longer span, this is not what the nurse is
doing. Healthy lifestyle changes are not part of infection prevention or illness
treatment.
Question format: Multiple Choice
Chapter 1: The Nurse's Role in a Changing Maternal-Child Health Care Environment
Cognitive Level: Apply
Client Needs: Health Promotion and Maintenance
Integrated Process: Teaching/Learning
Reference: p. 7
11. A young female client is pregnant for the first time and is uncertain who to seek
prenatal care from. The nurse should point out which health care provider as the
likely choice?
A. Perinatologist
B. Neonatologist
C. Family practitioner
D. Obstetrician

Answer: D

Rationale: The obstetrician is the common choice for prenatal care through labor
and delivery. Perinatologists may care for women who have a high-risk pregnancy,
and neonatologists provide care to infants. Family practitioner physicians may
provide care but are less likely to attend in labor and delivery.
Question format: Multiple Choice
Chapter 1: The Nurse's Role in a Changing Maternal-Child Health Care Environment
Cognitive Level: Understand
Client Needs: Health Promotion and Maintenance
Integrated Process: Caring
Reference: p. 3

12. While preparing to teach a group of nursing students the history of maternity
care, which factor will the instructor include to explain as the etiology of most
infections in females after birthing in the 1700s?
A. Reproductive tract infection
B. Breast infection
C. Kidney infection
D. Urinary tract infection

Answer: A

Rationale: Prior to the germ theory, women most often died of puerperal fever, an
illness marked by high fever caused by infection of the reproductive tract after
delivering infants. Women who delivered in hospitals were more likely to develop
this infection than women who delivered at home. Breast infections occurred during
breast feeding but were not usually fatal. There was no greater incidence of kidney
or urinary tract infections.
Question format: Multiple Choice
Chapter 1: The Nurse's Role in a Changing Maternal-Child Health Care Environment
Cognitive Level: Apply
Client Needs: Safe, Effective Care Environment: Safety and Infection Control
Integrated Process: Teaching/Learning
Reference: p. 3-4

13. A group of nursing students are comparing the various changes in maternity
care over the course of history and discover that the development of what medical
treatment was most influential in moving birth from the home into the hospital
setting?
A. Infection control and germ theory
B. Planned cesarean birth
C. Instruments to assist in birth of infants
D. Anesthesia and analgesic therapy

Answer: D

Rationale: Movement from the home to the hospital for the birth of infants began
with the use of medications to control pain during labor. This trend started with the
wealthy and followed to include more of society. Cesarean births are a more recent
development than the advent of anesthesia. Infection control could be maintained
at home, so it was not a driving force. The use of instruments developed along with
cesarean technology.
Question format: Multiple Choice
Chapter 1: The Nurse's Role in a Changing Maternal-Child Health Care Environment
Cognitive Level: Apply
Client Needs: Physiological Integrity: Basic Care and Comfort
Integrated Process: Teaching/Learning
Reference: p. 3

14. The nursing instructor is teaching a group of students about the history of
maternity and family care. The instructor determines the session is successful when
the students correctly choose which major change resulting from research by Klaus
and Kennell?
A. Limited family visits for children in the hospital
B. Family-centered care of today
C. Rooming-in for maternity patients
D. Isolation of children with infections

Answer: B

Rationale: Klaus and Kennell conducted studies and determined the optimal
outcomes for children occurred when parents had more contact and interaction with
the child in the hospital. Limiting visits has detrimental effects on infant
development. Rooming-in was not as well received, as patients were not
comfortable with the loss of privacy. Isolation of children with infections is still a
proper precaution.
Question format: Multiple Choice
Chapter 1: The Nurse's Role in a Changing Maternal-Child Health Care Environment
Cognitive Level: Apply
Client Needs: Health Promotion and Maintenance
Integrated Process: Teaching/Learning
Reference: p. 4

15. A client with a history of type 1 diabetes mellitus is confirmed to be pregnant.


The nurse determines this client will be best cared for under which practice model?
A. Case management
B. The nursing process
C. A clinical pathway
D. A health maintenance organization

Answer: A

Rationale: Case management is a system that integrates management and


coordination of care with financing in an attempt to improve cost-effectiveness,
use, quality, and outcomes. The nursing process involves the practice of nursing; it
is not a means of cost saving for the client. The clinical pathway is a treatment
regimen. HMOs are insurance organizations.
Question format: Multiple Choice
Chapter 1: The Nurse's Role in a Changing Maternal-Child Health Care Environment
Cognitive Level: Apply
Client Needs: Safe, Effective Care Environment: Management of Care
Integrated Process: Nursing Process
Reference: p. 7

16. A nursing student is preparing a presentation illustrating the Human Genome


Project. Which function will the student point out as being the primary focus?
A. Genetic testing in adults
B. Detection of genetic mutations in children
C. Identification of human genes and functions
D. Treatment of gene mutations

Answer: C

Rationale: The Human Genome Project was founded in 1990 and its primary
purpose was to identify and label all human genes and their functions. Information
from the HGP may eventually be used in genetic testing, the detection of mutations
or variations in children, and gene therapy.
Question format: Multiple Choice
Chapter 1: The Nurse's Role in a Changing Maternal-Child Health Care Environment
Cognitive Level: Apply
Client Needs: Health Promotion and Maintenance
Integrated Process: Teaching/Learning
Reference: p. 6

17. A healthy client without a primary care provider is exploring the options
available for a health care provider to assist with her pregnancy. Which health care
provider can the nurse point out as a best option?
A. Women's health nurse practitioner (NP)
B. Certified nurse midwife
C. Lay midwife
D. Clinical nurse specialist (CNS)

Answer: B
Rationale: A certified nurse midwife is a nurse with advanced practice training in
the care of women, specific to pregnancy and birth. A women's health NP would not
be able to deliver the infant. A CNS is an advanced practice role but not specific to
the care of pregnancy women. The lay midwife has no formal education.
Question format: Multiple Choice
Chapter 1: The Nurse's Role in a Changing Maternal-Child Health Care Environment
Cognitive Level: Apply
Client Needs: Health Promotion and Maintenance
Integrated Process: Teaching/Learning
Reference: p. 3-4

18. The nursing instructor is teaching a session comparing the various steps of the
nursing process. The instructor determines the session is successful when the
students correctly choose which step as the first one?
A. Nursing diagnosis
B. Planning
C. Assessment
D. Evaluation

Answer: C

Rationale: Assessment is the first step in the nursing process. The nurse must
complete the assessment and gather information to advance the nursing process.
Nursing diagnosis is based on actual or potential health problems that fall within the
range of nursing practice. Evaluation measures the success or failure of the nursing
plan of care. Planning is a stage of execution.
Question format: Multiple Choice
Chapter 1: The Nurse's Role in a Changing Maternal-Child Health Care Environment
Cognitive Level: Remember
Client Needs: Safe, Effective Care Environment: Management of Care
Integrated Process: Nursing Process
Reference: p. 13

19. After completing an assessment, the nurse examines the information to develop
a wellness diagnosis for the client to identify which potential goal?
A. Acute health problems
B. Potential problems
C. The potential for improvement in health
D. Chronic health problems

Answer: C

Rationale: Wellness diagnosis identifies the potential for a client to move from one
level of wellness to a higher level. The identification of potential, acute, and chronic
health problems is part of the diagnostic process.
Question format: Multiple Choice
Chapter 1: The Nurse's Role in a Changing Maternal-Child Health Care Environment
Cognitive Level: Apply
Client Needs: Safe, Effective Care Environment: Management of Care
Integrated Process: Nursing Process
Reference: p. 14

20. The nurse is organizing health information to teach a client basic principles that
will help maintain wellness in the family. Which actions should the nurse prioritize
in this teaching?
A. encouraging yearly checkups for all clients.
B. teaching insulin injection techniques
C. medication administration
D. assisting in understanding a treatment

Answer: A

Rationale: The current movement in health care is to focus on health promotion and
thereby prevent future illness and diseases. Encouraging yearly checkups would
assist with finding problems before they become serious. Prevention, treatment,
and rehabilitation are all processes of care. Teaching a client how to give injections,
administer medication, or understand a treatment would occur after a specific
health issue but not prevent these issues from occurring.
Question format: Multiple Choice
Chapter 1: The Nurse's Role in a Changing Maternal-Child Health Care Environment
Cognitive Level: Apply
Client Needs: Health Promotion and Maintenance
Integrated Process: Nursing Process
Reference: p. 14

21. Which nursing intervention would best demonstrate evidence-based practice in


maternal–child health care?
A. Family-centered pediatric care
B. Minimizing parental interaction with preterm infants
C. Placing adults and children with similar diseases on the same unit
D. Decentralizing care to allow clients to be closer to home

Answer: A

Rationale: Evidence-based practice has become the standard that nurses are to
strive for in caring for their clients. By involving the family in caring for ill children,
the child and the family are better served and have improved outcomes. Parental
interaction is encouraged for preterm infants to foster bonding. Children and adults
need to be separated on inpatient units to ensure that the caregivers have a clear
understanding of each client's needs, since children are not small adults.
Centralized care has proved to be most beneficial to client outcomes by providing
resources and specialists in one location.
Question format: Multiple Choice
Chapter 1: The Nurse's Role in a Changing Maternal-Child Health Care Environment
Cognitive Level: Apply
Client Needs: Health Promotion and Maintenance
Integrated Process: Teaching/Learning
Reference: p. 5

22. What societal issues greatly influence delivery of maternal and pediatric health
care? Select all that apply.
A. Cost of health care
B. Increase in surrogacy and adoption
C. Low income of families
D. Increased cultural and ethnic diversity of clients
E. Increased number of children born in the U.S.

Answer: A, C, D

Rationale: Demographic trends such as a decreased number children being born,


combined with an increase in a multicultural society seeking health care, are
affecting the delivery of maternal–child health care. Poverty and the cost of health
care also play a major role in influencing health care delivery for both women and
children.
Question format: Multiple Select
Chapter 1: The Nurse's Role in a Changing Maternal-Child Health Care Environment
Cognitive Level: Apply
Client Needs: Health Promotion and Maintenance
Integrated Process: Teaching/Learning
Reference: p. 7

23. The United States lags behind other industrialized countries regarding infant
mortality. The main factor that contributes to this is:
A. lack of available facilities for caring for the infants.
B. older mothers having babies.
C. the large number of preterm births in the U.S.
D. more congenital anomalies in children born in the U.S.

Answer: C

Rationale: Two factors that contribute to the fact that the United States lags behind
other industrialized countries are the large number of preterm births and the
differences in reporting live births in various countries.
Question format: Multiple Choice
Chapter 1: The Nurse's Role in a Changing Maternal-Child Health Care Environment
Cognitive Level: Apply
Client Needs: Health Promotion and Maintenance
Integrated Process: Teaching/Learning
Reference: p. 9

24. The nurse reviews the client's plan of care (above). Which nursing action(s)
does the nurse identify as independent? Select all that apply.
A. assisting out of bed
B. administering ibuprofen
C. giving IV normal saline
D. reinforcing breathing exercises
E. offering oral fluids

Answer: A, E

Rationale: Independent nursing actions are actions that may be performed based
on the nurse's own clinical judgment, for example, getting the client out of bed and
offering oral fluids. Dependent nursing actions, such as administering medications
or IV fluids, are actions that the nurse performs as a result of a health care
provider's prescription. Interdependent nursing actions are actions that the nurse
must accomplish in conjunction with other health team members, such as
reinforcing breathing exercises with the respiratory therapist.
Question format: Multiple Select
Chapter 1: The Nurse's Role in a Changing Maternal-Child Health Care Environment
Cognitive Level: Apply
Client Needs: Safe, Effective Care Environment: Management of Care
Integrated Process: Nursing Process
Reference: p. 14

25. A nurse manager is orienting a group of new nurses to the maternity unit.
During the orientation, the nurse manager emphasizes the need for accurate and
complete documentation based on which reason?
A. The facility could lose its standing in the community.
B. The facility might not keep its accreditation.
C. The records could be used in a legal action in the future.
D. The records act as the main source of communication among team members.

Answer: C

Rationale: One of the most important parts of nursing care is recording information
about the client on the permanent record. This record, the client's chart, is a legal
document and must be accurate and complete. In maternity and pediatric settings,
documentation is extremely important because records can be used in legal
situations many years after the fact. Although documentation is a form of
communication among team members, it is not the main source for doing so.
Incomplete or inaccurate documentation can affect a facility's accreditation and or
status in the community, but this is not the main reason for accurate and thorough
documentation.
Question format: Multiple Choice
Chapter 1: The Nurse's Role in a Changing Maternal-Child Health Care Environment
Cognitive Level: Apply
Client Needs: Safe, Effective Care Environment: Management of Care
Integrated Process: Communication and Documentation
Reference: p. 14
26. A community-based nurse is part of an agency sponsoring a booth at a local
health fair. An acute care nurse comes up to the booth and asks the community-
based nurse, "How is your practice different from practice in a hospital?" Which
response(s) by the community-based nurse would be appropriate? Select all that
apply.
A. "In the community, we rely on other disciplines for decision-making."
B. "We are more autonomous when providing client care."
C. "We tend to address the client's needs more holistically."
D. "Our major focus is on illness and treatment."
E. "We care for clients over a longer span of time."

Answer: B, C, E

Rationale: Unlike the hospital setting, community-based nursing practice is


autonomous and the nurse is often called on to be self-reliant. Community practice
tends to be more holistic and focuses on wellness rather than illness. In addition,
the nurse in a community-based setting sees the client over time.
Question format: Multiple Select
Chapter 1: The Nurse's Role in a Changing Maternal-Child Health Care Environment
Cognitive Level: Understand
Client Needs: Safe, Effective Care Environment: Management of Care
Integrated Process: Teaching/Learning
Reference: p. 28-29

27. A nurse is developing a program for families to promote spending more time
together. Which cause(s) of decreased family togetherness will the nurse
incorporate into the program? Select all that apply.
A. increase in families with one working parent
B. rise in children participating in extracurricular activities
C. increase in television use over discussion among family members
D. rise in time spent sitting down to meals as a family
E. greater focus on material things rather than relationships

Answer: B, C, E

Rationale: There is a trend for families to spend less time together. Many factors
contribute to this trend which may include two working parents, rather than just
one; the children participating in many extracurricular activities; family members
watching television, rather than talking together at mealtime, or eating fast food or
individual meals without sitting down together as a family; or an emphasis on the
acquisition of material goods, rather than the development of relationships.
Question format: Multiple Select
Chapter 1: The Nurse's Role in a Changing Maternal-Child Health Care Environment
Cognitive Level: Understand
Client Needs: Psychosocial Integrity
Integrated Process: Nursing Process
Reference: p. 22
28. A nurse will conduct an information session for a group of parents who have
children in elementary school. The session will focus on child growth and
development. Which information about the parents would be important for the
nurse to obtain before planning the session to promote the success of the session?
Select all that apply.
A. ages
B. educational levels
C. primary language(s) spoken
D. cultural attitudes
E. financial background

Answer: A, B, C, D

Rationale: A successful group experience relies on the nurse being prepared before
the session in addition to providing factual, unbiased information. It is important to
know the needs of the target population to provide information to a group
successfully. Important information about the group includes age, educational level,
ethnic and gender mix, language barriers, cultural attitudes regarding receiving and
acknowledging information, and any previous education the group has already had
on the subject.
Question format: Multiple Select
Chapter 1: The Nurse's Role in a Changing Maternal-Child Health Care Environment
Cognitive Level: Apply
Client Needs: Health Promotion and Maintenance
Integrated Process: Teaching/Learning
Reference: p. 28

29. A community-based nurse caring for a family gives the family information about
free transportation services to medical appointments. Which skill is the nurse is
using?
A. nursing process
B. data collection
C. education
D. client advocacy

Answer: D

Rationale: Client advocacy is speaking or acting on behalf of clients. This helps


them gain greater independence and to make the health care delivery system more
responsive and relevant to their needs. Giving the family information about free
transportation is an example. The nursing process is a proven form of problem-
solving based on the scientific method and involves data collection. Education is a
fundamental part of community-based care and involves the relaying of information
to improve outcomes.
Question format: Multiple Choice
Chapter 1: The Nurse's Role in a Changing Maternal-Child Health Care Environment
Cognitive Level: Apply
Client Needs: Safe, Effective Care Environment: Management of Care
Integrated Process: Caring
Reference: p. 28

30. A nurse working at a child health clinic is involved in primary and secondary
prevention activities. Which activity(ies) reflects secondary prevention? Select all
that apply.
A. vision screening at all well-child visits
B. safety education for bicycles and skateboards
C. administration of immunizations
D. instructions for treatment of head lice
E. drug education program

Answer: A, D

Rationale: Secondary prevention involves health screening activities that aid in


early diagnosis and encourage prompt treatment before long-term negative effects
arise. Therefore, vision screening and instructions for head lice treatment would be
appropriate. Safety education, immunizations, and drug education are examples of
primary prevention activities, which are health-promoting activities to prevent the
development of illness or injury.
Question format: Multiple Select
Chapter 1: The Nurse's Role in a Changing Maternal-Child Health Care Environment
Cognitive Level: Apply
Client Needs: Health Promotion and Maintenance
Integrated Process: Nursing Process
Reference: p. 23
Chapter 2
1. A young couple are asking the nurse for suggestions on the best way to help
their newborn develop properly. Which is the best response by the nurse?
A. Provide nutritional supplements.
B. Start reading to the child on first birthday.
C. Minimize distractions in the nursery.
D. Provide a variety of brightly colored toys.

Answer: D

Rationale: Children, from an early age, need stimulation and interaction to develop
the five senses optimally. Many parents buy brightly colored toys to facilitate
stimulation. You should encourage proper nutrition through diet rather than
supplementation. Reading should begin immediately, not on first birthday.
Question format: Multiple Choice
Chapter 2: Family-Centered and Community-Based Maternal and Pediatric Nursing
Cognitive Level: Apply
Client Needs: Health Promotion and Maintenance
Integrated Process: Teaching/Learning
Reference: p. 19

2. Which situation may occur in immediate families at a greater level than in some
other types of families?
A. financial difficulties
B. problems with allocation of resources
C. formation of a poor parent–child relationship
D. lack of support people in a crisis

Answer: D

Rationale: People in an immediate family may all be so involved in a crisis that


there is no objective support person. Financial difficulties, resource allocation
problems, and poor relationships may be formed in any family situation.
Question format: Multiple Choice
Chapter 2: Family-Centered and Community-Based Maternal and Pediatric Nursing
Cognitive Level: Understand
Client Needs: Health Promotion and Maintenance
Integrated Process: Caring
Reference: p. 21-22

3. Which of the following may be a strength of the single-parent type of family?


A. financial security
B. readily available support in a crisis
C. development of a special parent-child relationship
D. child wishing for no other siblings
Answer: C

Rationale: Because there is no competition for time from a spouse, single-parent


families can promote special parent-child relationships. Financial security, support
in a crisis, and the desire for no siblings can occur in any family situation.
Question format: Multiple Choice
Chapter 2: Family-Centered and Community-Based Maternal and Pediatric Nursing
Cognitive Level: Apply
Client Needs: Psychosocial Integrity
Integrated Process: Nursing Process
Reference: p. 21-22

4. The nurse is preparing to teach a drug education class at a local elementary


school. The nurse is focused on providing which type of care to the community?
A. Primary prevention
B. Secondary prevention
C. Tertiary prevention
D. Preventive care

Answer: A

Rationale: Primary prevention is promotion of healthy activities and includes


education concerning safety, diet, rest, exercise, and disease prevention.
Secondary prevention focuses on health screening activities that aid in early
diagnosis and encourage prompt treatment. Tertiary prevention focuses on
rehabilitation and instruction on ways to prevent further injury or illness.
"Preventive care" is not considered a specific category but is a general function that
encompasses all three levels.
Question format: Multiple Choice
Chapter 2: Family-Centered and Community-Based Maternal and Pediatric Nursing
Cognitive Level: Apply
Client Needs: Health Promotion and Maintenance
Integrated Process: Teaching/Learning
Reference: p. 23

5. The primary health care provider has recommended a client consult a nutritionist
for specialized care. The nurse, by providing a list of referrals to the client, is
providing which service?
A. Client advocacy
B. Community-based nursing
C. Primary care
D. Secondary prevention

Answer: A

Rationale: Client advocacy is speaking or acting on behalf of clients to help them


gain greater independence and to make the health care delivery system more
responsive and relevant to their needs. Community-based nursing focuses on
prevention and is directed toward persons and families within a community.
Primary care is the direct action of the primary care provider. Secondary prevention
involves health screening activities that aid in early diagnosis and encourage
prompt treatment before long-term negative effects occur.
Question format: Multiple Choice
Chapter 2: Family-Centered and Community-Based Maternal and Pediatric Nursing
Cognitive Level: Apply
Client Needs: Safe, Effective Care Environment: Management of Care
Integrated Process: Caring
Reference: p. 28

6. A community-based nurse is assessing the needs of a family of four, which


includes a physically challenged 9-year-old. Which activity would the nurse
prioritize to be an advocate for this family?
A. Ensure case history is complete for all family members.
B. Establish eligibility for assistive devices for child.
C. Ensure the client follows physical therapy recommendations.
D. Train the school nurse on the needs of the child.

Answer: B

Rationale: Client advocacy is acting on behalf of the client. Making calls to arrange
for special equipment is one role of a nurse who is acting on behalf of the client. A
case history would be taken at the initial visit to the treatment center by the
attending nurse. The client's progress in physical therapy is to be noted by the
therapist or PT assistant. The community-based nurse should not have to train the
school nurse but would share the needs of the child so that the school nurse is
prepared to provide appropriate care.
Question format: Multiple Choice
Chapter 2: Family-Centered and Community-Based Maternal and Pediatric Nursing
Cognitive Level: Apply
Client Needs: Safe, Effective Care Environment: Management of Care
Integrated Process: Caring
Reference: p. 28

7. The nurse is preparing a prenatal seminar for young mothers. Which type of
information should the nurse gather to ensure success of the program?
A. Who will be accompanying the mother
B. Approximate education level of the participants
C. Names of each of the participants
D. Incomes of each of the participants

Answer: B

Rationale: When teaching a group of clients, the nurse should seek demographic
information, such as the education level of the participants, to help develop an
appropriate teaching plan for the group. This is an impersonal group, so learning
who is accompanying the mothers, their names, or their incomes is not appropriate.
Question format: Multiple Choice
Chapter 2: Family-Centered and Community-Based Maternal and Pediatric Nursing
Cognitive Level: Apply
Client Needs: Health Promotion and Maintenance
Integrated Process: Teaching/Learning
Reference: p. 27-28

8. A nursing student is exploring community-based nursing as a career focus. The


nurse is prepared to prioritize which activity as a primary focus?
A. Wellness
B. Cost containment
C. Rehabilitation
D. Emergency care

Answer: A

Rationale: The community-based nurse is in a unique position of health promotion


for the community. Wellness is health promotion and aligns with the holistic
practice of the community-based nurse. Cost containment, rehabilitation, and
emergency care are the responsibility of direct care facilities.
Question format: Multiple Choice
Chapter 2: Family-Centered and Community-Based Maternal and Pediatric Nursing
Cognitive Level: Apply
Client Needs: Health Promotion and Maintenance
Integrated Process: Nursing Process
Reference: p. 23-24

9. The community health nurse is assessing a new client who reports having
recently moved to the area and living with an aunt and her parents. The nurse
determines this client resides in which type of family structure?
A. extended
B. immediate
C. communal
D. blended

Answer: A

Rationale: The extended family is an immediate family with other family members
in the same house. The immediate family is composed of an individual's smallest
family unit (commonly parents and their children), all of whom share a common
household. Members of a communal family share responsibility for homemaking
and child rearing; all children are the collective responsibility of adult members. In
a blended family, both partners in the marriage bring children from a previous
marriage into the household.
Question format: Multiple Choice
Chapter 2: Family-Centered and Community-Based Maternal and Pediatric Nursing
Cognitive Level: Understand
Client Needs: Psychosocial Integrity
Integrated Process: Nursing Process
Reference: p. 20

10. The nurse at a family planning clinic is meeting with a young couple who have
decided not to have any children after they marry since they are both bringing
children into the relationship. The nurse will document this as which type of family
structure?
A. Blended family
B. Immediate family
C. Extended family
D. Communal family

Answer: A

Rationale: The blended family consists of each parent bringing their own children
into the family when they marry. It may also consist of additional children if the
couple decide to have more. The immediate family is composed of a man, a
woman, and their children (either biological or adopted), who share a common
household. Members of a communal family share responsibility for homemaking
and child rearing; all children are the collective responsibility of adult members. An
extended family consists of one or more nuclear families plus other relatives, often
crossing generations to include grandparents, aunts, uncles, and cousins.
Question format: Multiple Choice
Chapter 2: Family-Centered and Community-Based Maternal and Pediatric Nursing
Cognitive Level: Apply
Client Needs: Psychosocial Integrity
Integrated Process: Nursing Process
Reference: p. 21

11. The nurse is assessing a child of a different cultural background. Which nursing
action should the nurse prioritize when providing nursing care?
A. Use personal cultural experiences with the family.
B. Use a standard nursing care plan for the family.
C. Expect the family to adopt the cultural beliefs of that facility.
D. Ask the family for input into their care based on their cultural beliefs.

Answer: D

Rationale: Cultural beliefs vary; care should be based on the individual beliefs of
each family. The imposition of personal or institutional beliefs, as well as the beliefs
of others, should never be placed on a client.
Question format: Multiple Choice
Chapter 2: Family-Centered and Community-Based Maternal and Pediatric Nursing
Cognitive Level: Apply
Client Needs: Psychosocial Integrity
Integrated Process: Nursing Process
Reference: p. 22

12. The nurse is explaining the differences of language development in children.


Which example would be appropriate for the nurse to include in the explanation?
A. Firstborn children are slower to develop language skills than their siblings.
B. Second-born children are quicker to develop language skills than their siblings.
C. Boys raised with older sisters are slower to develop language skills than their
siblings.
D. Girls raised with older brothers are quicker to develop language skills than their
siblings.

Answer: C

Rationale: Birth order has an impact on the development of children. Firstborn


children command a great deal of attention and tend to be higher achievers than
siblings; thus language skills develop sooner in the firstborn child. With second and
subsequent children, parents tend to be more relaxed and permissive. These
children are likely to be more relaxed and are slower to develop language skills.
Question format: Multiple Choice
Chapter 2: Family-Centered and Community-Based Maternal and Pediatric Nursing
Cognitive Level: Analyze
Client Needs: Health Promotion and Maintenance
Integrated Process: Teaching/Learning
Reference: p. 21

13. A nursing instructor is teaching the class about community-based nursing. The
instructor determines the session is successful when the students correctly choose
which activity as an example of tertiary prevention?
A. Caring for new mothers and infants in a maternity clinic
B. Identifying a child with pediculosis in an elementary school
C. Ensuring the client attends physical therapy after orthopedic surgery
D. Reviewing dietary habits with parents of a slightly anemic child

Answer: C

Rationale: Tertiary prevention focuses on rehabilitation activities and would be the


focus of a nurse in an orthopedic clinic, aligning with the needs of the clients in that
clinic. Orthopedic clients are typically recovering from injury or surgery and are in
need of rehabilitation such as physical therapy. Secondary prevention includes
health screening activities that aid in early diagnosis and encourage prompt
treatment, such as screening for head lice in schools and reviewing dietary habits in
individuals who are experiencing anemia. Primary prevention includes health
promoting activities to prevent the development of illness or injury, such as new
mothers bringing their infants to the clinic for follow-up well-child appointments.
Question format: Multiple Choice
Chapter 2: Family-Centered and Community-Based Maternal and Pediatric Nursing
Cognitive Level: Analyze
Client Needs: Health Promotion and Maintenance
Integrated Process: Teaching/Learning
Reference: p. 23

14. The nurse is preparing to discuss birthing options with a 25-year-old female
who is in a low-risk pregnancy with one older child. Which option will be best for
the nurse to recommend for this client?
A. Home setting
B. Birthing center
C. Hospital
D. Any birthing settings

Answer: D

Rationale: When a woman is low-risk and has no known medical needs, birth
location is the choice of the mother. Home births attended with certified nurse
midwives are less likely to have complications and require interventions. Birthing
centers are an option for an alternative birth setting, and hospitals are required
birth locations for women with potential needs or complications. The nurse should
present the advantages and disadvantages of each option and allow the client to
make the decision.
Question format: Multiple Choice
Chapter 2: Family-Centered and Community-Based Maternal and Pediatric Nursing
Cognitive Level: Apply
Client Needs: Health Promotion and Maintenance
Integrated Process: Teaching/Learning
Reference: p. 24

15. Which action by the nurse in a community clinic would best meet a family's
need of physical sustenance?
A. Enrolling the pregnant mother in a WIC program
B. Offering parenting classes to teenage mothers
C. Holding monthly educational sessions on nutrition
D. Providing weekly exercise classes for the neighborhood families

Answer: A

Rationale: Physical sustenance deals with meeting the basic needs of food, clothing,
shelter and protection from harm of each family member. By enrolling the pregnant
mother in WIC, her physical needs of nutrition are addressed. Parenting classes,
nutrition classes, and exercise classes are all good ideas but do not directly address
physical sustenance.
Question format: Multiple Choice
Chapter 2: Family-Centered and Community-Based Maternal and Pediatric Nursing
Cognitive Level: Apply
Client Needs: Safe, Effective Care Environment: Management of Care
Integrated Process: Teaching/Learning
Reference: p. 19
16. A mother voices her concerns to the nurse that her daughter is an "only child"
and she is worried that having no siblings may be detrimental to the child. The
nurse can reassure the mother that an "only child" tends to excel in what area(s)?
Select all that apply.
A. Being more relaxed around others
B. Advanced language development
C. Intellectual achievement
D. Less dependence upon the parent
E. Closer identification with peers instead of parents

Answer: B, C

Rationale: It is shown that "only children" tend to have more advanced language
skills and intellectual achievement than children from larger families. This is
thought to be from the fact that the parents have more one-on-one time with the
only child. "Only children" more closely identify with their parents, are more
dependent upon the parents and are not necessarily more relaxed around others.
Question format: Multiple Select
Chapter 2: Family-Centered and Community-Based Maternal and Pediatric Nursing
Cognitive Level: Apply
Client Needs: Health Promotion and Maintenance
Integrated Process: Teaching/Learning
Reference: p. 21

17. The nurse is providing tertiary care to a young, uninsured family who has a
child with frequent seizures. Which action by the nurse would demonstrate tertiary
care?
A. Performing a well-child checkup, noting weight gain
B. Educating the parents regarding appropriate play activities for the child
C. Finding a company to provide a helmet for the child to wear daily.
D. Nutritional guidance for healthy meals for the family.

Answer: C

Rationale: Tertiary care involves health promotion focused on rehabilitation and


prevention of further injury or illness, and it optimizes function. By providing a
safety helmet to the child with a history of seizures, the nurse is preventing further
injury to the child. Nutritional guidance is an example of primary prevention,
focusing on good nutrition to prevent risk factors that may cause impairment.
Performing the well-child checkup and educating about appropriate play activities
address secondary prevention, which reflects health screening and prompt
treatment of problems.
Question format: Multiple Choice
Chapter 2: Family-Centered and Community-Based Maternal and Pediatric Nursing
Cognitive Level: Apply
Client Needs: Health Promotion and Maintenance
Integrated Process: Caring
Reference: p. 23
18. A 7-year-old child who has recently immigrated with the family is brought to
the school nurse because the child refuses to eat lunch. Which response should the
nurse prioritize?
A. Eat lunch with the child.
B. Discuss the situation with the child.
C. Investigate for potential cultural issues.
D. Refer the family to a nutritionist.

Answer: C

Rationale: Culture influences the family's health beliefs. A newly immigrated family
may have attitudes toward food that are culturally founded. The nurse should seek
to clarify the cultural food influences of the family and the needs of this child.
Discussing the issue with the parents may be an option if the nurse is unable to
detect a possible cultural connection by talking with the child. Referring the family
to a nutritionist would be inappropriate.
Question format: Multiple Choice
Chapter 2: Family-Centered and Community-Based Maternal and Pediatric Nursing
Cognitive Level: Apply
Client Needs: Health Promotion and Maintenance
Integrated Process: Nursing Process
Reference: p. 22

19. The nurse is assessing an infant girl at her first well-baby visit. The nurse also
observes the actions of the 6-year-old brother and parents who share that she was
the best anniversary present they received this year. Which type of family will the
nurse conclude these individuals represent?
A. Cohabitation
B. Adoptive
C. Immediate
D. Extended

Answer: C

Rationale: An immediate family is defined as consisting of parents and children. An


extended family consists of one or more nuclear families plus other relatives, often
crossing generations to include grandparents, aunts, uncles, and cousins. In the
cohabitation family, couples live together but are not married. The children in this
family may be children of earlier unions, or they may be a result of this union. The
adoptive family is created when parents take in children who are not biologically
theirs but raise them as if they were.
Question format: Multiple Choice
Chapter 2: Family-Centered and Community-Based Maternal and Pediatric Nursing
Cognitive Level: Apply
Client Needs: Psychosocial Integrity
Integrated Process: Nursing Process
Reference: p. 20
20. A nursing instructor is teaching a group of nursing students about the various
options available to provide nursing care in a community. The instructor determines
the session is successful when the students correctly choose which action as the
primary focus of home care nursing?
A. Provide care based on insurance coverage.
B. Teach and supervise caregivers.
C. Provide direct client care.
D. Act as a liaison between health care provider and family.

Answer: C

Rationale: The primary focus of home care nursing is to provide direct care.
Teaching and supervising caregivers and acting as a liaison between the health care
provider and family are additional functions of the home care nurse that support
the direct care. The nurse should be aware of potential insurance restrictions so
that other options may be explored if insurance will not cover specific treatments or
medications that the health care provider has determined essential to the client. In
these instances, the nurse can then act as the advocate to help find the necessary
resources the client may need.
Question format: Multiple Choice
Chapter 2: Family-Centered and Community-Based Maternal and Pediatric Nursing
Cognitive Level: Remember
Client Needs: Safe, Effective Care Environment: Management of Care
Integrated Process: Nursing Process
Reference: p. 24
Chapter 3
1. A client who is trying not to get pregnant calls the nurse on Saturday at 11 a.m.
reporting that she had unprotected sex on Thursday at 10 p.m. She believes she
has just ovulated and wants to verify that she has no risk of pregnancy. What can
the nurse tell her?
A. Because she did not ovulate before the unprotected sex, she will not get
pregnant.
B. If she ovulated on Saturday around 11 a.m., the egg could be fertilized any time
before about 11 a.m. on Sunday.
C. If she ovulated on Saturday around 11 a.m., the egg could be fertilized any time
before about 10 p.m. on Sunday.
D. If she ovulated on Saturday around 11 a.m., the egg could be fertilized any time
before about 11 a.m. on Monday.

Answer: B

Rationale: A single ovum is released from the ovary 14 days before the next
menstrual period. It lives approximately 24 hours. The client's fertile period would
be from Saturday at 11 a.m. to Monday at 11 a.m. During any other time frame, it
would be unlikely that she would become pregnant.
Question format: Multiple Choice
Chapter 3: Structure and Function of the Reproductive System
Cognitive Level: Analyze
Client Needs: Health Promotion and Maintenance
Integrated Process: Nursing Process
Reference: p. 43

2. Male and female reproductive systems are complementary; for example, male
testes and female ovaries; male scrotum and female labia majora; and male glans
penis and female clitoris. What part of the female system is homologous to the
spermatic cord in the male?
A. cardinal ligaments
B. round ligaments
C. uterosacral ligaments
D. broad ligament

Answer: B

Rationale: Round ligaments and spermatic cord are both fibromuscular bands that
assist in holding specific reproductive structures in place in the male and female.
Cardinal ligaments anchor the walls of the cervix and vagina to the lateral pelvic
walls. The uterosacral ligaments anchor the lower posterior portion of the uterus to
the sacrum. The broad ligament is a sheet of peritoneum that attaches the lower
sides of the uterus to the sidewalls of the pelvis.
Question format: Multiple Choice
Chapter 3: Structure and Function of the Reproductive System
Cognitive Level: Apply
Client Needs: Health Promotion and Maintenance
Integrated Process: Nursing Process
Reference: p. 39

3. When reviewing normal menstruation with an early adolescent, the nurse would
teach that during the second half of a typical menstrual cycle, the endometrium of
the uterus becomes:
A. thin and transparent, due to progesterone stimulation.
B. thin and transparent, due to follicle-stimulating hormone.
C. thick and purple-hued, due to estrogen stimulation.
D. thick and purple-hued, due to progesterone stimulation.

Answer: D

Rationale: Progesterone is released following ovulation and thus is the dominating


hormone of the second half of the menstrual cycle; its effect is to increase
endometrium growth. Progesterone stimulates a growth of tissue, not a thinning of
it. FSH and estrogen are found in the earlier stages of menstruation, not in the
second half.
Question format: Multiple Choice
Chapter 3: Structure and Function of the Reproductive System
Cognitive Level: Understand
Client Needs: Health Promotion and Maintenance
Integrated Process: Teaching/Learning
Reference: p. 44

4. A woman tells the nurse that both she and her husband like to continue sexual
relations during her menstrual period. They have a monogamous relationship. She
asks if this will harm her. Which would be the best response?
A. "Avoid sexual relations because orgasm may be painful during your menses."
B. "The risk of infection is too great for sexual relations during this time."
C. "You will not be able to achieve orgasm during your menses."
D. "If this is satisfying for you and your partner, then there is no harm in it."

Answer: D

Rationale: Sexual relations may be continued through a menstrual flow if this is


satisfying for both partners. There is no danger to either party, and it is possible to
achieve orgasm.
Question format: Multiple Choice
Chapter 3: Structure and Function of the Reproductive System
Cognitive Level: Apply
Client Needs: Health Promotion and Maintenance
Integrated Process: Communication and Documentation
Reference: p. 42
5. A client asks the nurse questions about female hygiene and wants to know if she
should douche to clean her vagina. Which response by the nurse would be most
appropriate?
A. "Only douche if the vagina has an odor."
B. "You do not need to douche; the vagina is self-cleaning."
C. "Only douche if you have itching and burning."
D. "If you want, douching is ok."

Answer: B

Rationale: The vagina has an acidic environment, and this should not be disturbed.
It does not need to be cleaned with douching. Douching can change the
environment and lead to vaginal infection or irritation. Douching will only cover
odor for a short time and may make other issues worse. Itching and burning could
be signs of an infection, and the client should contact a health care provider.
Question format: Multiple Choice
Chapter 3: Structure and Function of the Reproductive System
Cognitive Level: Apply
Client Needs: Health Promotion and Maintenance
Integrated Process: Teaching/Learning
Reference: p. 39

6. A nurse is preparing a presentation on the menstrual cycle for a health fair.


Which phase will the nurse illustrate as producing progesterone?
A. menstrual
B. proliferative
C. secretory
D. ischemic

Answer: C

Rationale: The corpus luteum begins to produce progesterone during the secretory
phase of the menstrual cycle. The follicle develops during the menstrual phase and
begins to secrete estrogen. The proliferative phase continues secreting estrogen
and progesterone. There are no hormones secreted during the ischemic phase,
which will result in the endometrium sloughing off and leaving the uterus.
Question format: Multiple Choice
Chapter 3: Structure and Function of the Reproductive System
Cognitive Level: Apply
Client Needs: Health Promotion and Maintenance
Integrated Process: Teaching/Learning
Reference: p. 44

7. The nurse is preparing to assess a new client who has arrived for her initial visit
with a gynecologist specializing in menopause issues. The nurse anticipates the
client will be in which age group?
A. 45 to 50
B. 47 to 55
C. 53 to 58
D. 49 to 56

Answer: B

Rationale: Menopause refers to the time in a women's life when reproductive


capability ends. The average age at which menopause occurs is between 47 and 55
years. Although it is possible to start menopause earlier or later, 47 to 55 is the
typical age.
Question format: Multiple Choice
Chapter 3: Structure and Function of the Reproductive System
Cognitive Level: Understand
Client Needs: Health Promotion and Maintenance
Integrated Process: Nursing Process
Reference: p. 44

8. The nurse is preparing to teach a birthing class and intends to alert the couples
to potential difficulties, ensuring they have as much accurate information as
possible. Which area of the uterus will the nurse explain has the highest risk of
rupture during labor?
A. cervix
B. corpus
C. uterine isthmus
D. fundus

Answer: C

Rationale: The uterine isthmus is the lower uterine segment and is the thinnest
portion of the uterus; it does not participate in the muscular contractions of labor.
The cervix is the tubular structure that connects the vagina and the uterus. The
corpus is the main body of the uterus. The fundus is the top portion of the uterus.
The cervix, corpus, and fundus will all stretch and contract due to the muscular
action.
Question format: Multiple Choice
Chapter 3: Structure and Function of the Reproductive System
Cognitive Level: Apply
Client Needs: Health Promotion and Maintenance
Integrated Process: Nursing Process
Reference: p. 39

9. The nurse is teaching a group of college students about the four phases of the
sexual response. Which phase will the nurse point out as producing an elevation in
heart rate, blood pressure, and respirations?
A. excitement
B. plateau
C. orgasm
D. resolution
Answer: A

Rationale: During the excitement phase, the physiologic response of both male and
females results in an increase in heart rate, blood pressure, and respirations.
During the plateau phase, the physiologic changes that occurred during excitement
are maintained. In the female, the clitoris retracts and the uterus is fully elevated.
In the male, the penis engorges further and the testes remain elevated. Orgasm is
marked by muscular contractions. During resolution, the muscles relax and blood
pressure, heart rate, and breathing return to normal.
Question format: Multiple Choice
Chapter 3: Structure and Function of the Reproductive System
Cognitive Level: Apply
Client Needs: Health Promotion and Maintenance
Integrated Process: Teaching/Learning
Reference: p. 44-45

10. The nurse is answering questions from a newborn's parents concerning a


circumcision. Which structure will the nurse point out is removed during the
procedure?
A. tunica albuginea
B. corpus spongiosum
C. rugae
D. prepuce

Answer: D

Rationale: The foreskin or prepuce is a fold of skin which covers the glans of the
penis. This fold of skin is removed in a circumcision. The tunica albuginea is
connective tissue found inside the penile shaft. The corpus spongiosum is erectile
tissue that runs the full length of the penis. Rugae are folds that allow for stretching
during an erection.
Question format: Multiple Choice
Chapter 3: Structure and Function of the Reproductive System
Cognitive Level: Apply
Client Needs: Physiological Integrity: Reduction of Risk Potential
Integrated Process: Teaching/Learning
Reference: p. 35

11. The nurse at a health fair is teaching about the various changes of puberty.
Which sequence of events will be best for the nurse to present when illustrating
pubertal changes in females?
A. menarche, breast budding, appearance of pubic hair
B. Appearance of pubic hair, menarche, breast budding
C. breast budding, appearance of pubic hair, menarche
D. appearance of pubic hair, breast budding, menarche

Answer: C
Rationale: Secondary sexual characteristics develop in an orderly sequence with
variance in the timing for individuals. Breast budding in the female is usually the
first physical sign noted, and occurs between the ages of 10 and 12 years on
average. Appearance of pubic hair usually occurs just before menarche, the first
menstrual period. From the onset to the start of menarche is typically 2 years.
Question format: Multiple Choice
Chapter 3: Structure and Function of the Reproductive System
Cognitive Level: Apply
Client Needs: Health Promotion and Maintenance
Integrated Process: Teaching/Learning
Reference: p. 42

12. The nurse is assessing a 16-year-old female on a routine well-child visit. Which
assessment finding will the nurse predict this healthy female will report concerning
her menstrual cycles?
A. Flow usually lasts 4 to 6 days.
B. The usual cycle is 36 days.
C. There's abundant clear mucus at the beginning
D. Menstruation began at age 15.

Answer: A

Rationale: The average menstrual flow is 4 to 6 days in length. The cycle usually
lasts 28 days. There should be no mucus during the menstrual cycle, with clear
mucus being noted at the time of ovulation or approximately day 14. Menstruation
usually begins at the age of 12 to 14 years.
Question format: Multiple Choice
Chapter 3: Structure and Function of the Reproductive System
Cognitive Level: Apply
Client Needs: Health Promotion and Maintenance
Integrated Process: Nursing Process
Reference: p. 42

13. The nurse is teaching a family planning class at a health fair. Which physiologic
change during the plateau stage of sexual response will the nurse point out?
A. lengthening of the vagina
B. labia minor lightens in color
C. retraction of the clitoris
D. scrotal elevation

Answer: C

Rationale: Retraction of the clitoris occurs during the plateau stage prior to orgasm.
Lengthening of the vagina and scrotal elevation occur during excitement. The labia
minora deepens in color.
Question format: Multiple Choice
Chapter 3: Structure and Function of the Reproductive System
Cognitive Level: Apply
Client Needs: Health Promotion and Maintenance
Integrated Process: Teaching/Learning
Reference: p. 44-45

14. The nursing instructor is teaching a group of nursing students about the
menstrual cycle. The instructor determines the session is successful when the
students correctly choose which action as responsible for the increased thickness of
the endometrium?
A. the level of the FSH
B. the decreasing level of the progesterone
C. the dropping level of LH
D. the increasing level of estrogen

Answer: D

Rationale: Estrogen levels increase after menstruation. These levels promote a


thickening of the endometrial tissue. FSH and LH are responsible for ovarian
changes. Progesterone will be increasing not decreasing, and works with estrogen
in influencing the menstrual cycle.
Question format: Multiple Choice
Chapter 3: Structure and Function of the Reproductive System
Cognitive Level: Apply
Client Needs: Health Promotion and Maintenance
Integrated Process: Teaching/Learning
Reference: p. 44

15. The nurse is teaching a couple trying to conceive about the changes in the
cervical mucus near the time of ovulation. The nurse determines the session is
successful when the couple correctly choose mucus in which form as indicating
ovulation has occurred?
A. Thin and copious
B. Thick and tacky
C. Scant amount
D. Mucus is not visible at ovulation

Answer: A

Rationale: During ovulation, the mucus will be distensible and stretchable. After
ovulation, the mucus will be scant, thick and opaque in nature. During the
proliferative phase before ovulation, it is tacky, crumbly, and yellow or white in
color. During the menses, there will be no mucus noted.
Question format: Multiple Choice
Chapter 3: Structure and Function of the Reproductive System
Cognitive Level: Analyze
Client Needs: Health Promotion and Maintenance
Integrated Process: Teaching/Learning
Reference: p. 44
16. A group of nursing students are preparing a presentation for a health fair
illustrating what happens to the body during pregnancy. Which structure will the
students point out is influenced the most by hormones to prepare for conception?
A. myometrium
B. endometrium
C. perimetrium
D. fundus

Answer: B

Rationale: The endometrium is the inner layer of the uterus; it builds with
hormones during the month in potential preparation for pregnancy. The
myometrium is the muscular layer. The perimetrium is a protective layer, and the
fundus is the upper area of the uterus.
Question format: Multiple Choice
Chapter 3: Structure and Function of the Reproductive System
Cognitive Level: Apply
Client Needs: Health Promotion and Maintenance
Integrated Process: Nursing Process
Reference: p. 40

17. The school nurse is preparing to illustrate the menstrual cycle to a group of
high school students. Which change in the hormones will the nurse point out as
being responsible for the onset of menstruation?
A. Decrease in progesterone
B. Increase in estrogen
C. Increase in luteinizing hormone
D. Decrease in follicle-stimulating hormone

Answer: A

Rationale: A decrease in progesterone and estrogen during the menstrual phase is


responsible for the shedding of the uterine lining, resulting in menstruation.
Increases in estrogen and progesterone result in the growth of the endometrium in
preparation for implantation of the fertilized ovum. An increase in luteinizing
hormone will result in ovulation. An increase in follicle-stimulating hormone will
result in the formation of a follicle, which results in a mature ovum being released
at ovulation.
Question format: Multiple Choice
Chapter 3: Structure and Function of the Reproductive System
Cognitive Level: Apply
Client Needs: Health Promotion and Maintenance
Integrated Process: Teaching/Learning
Reference: p. 44

18. A group of nursing students are comparing male and female reproductive
organs. The instructor determines the class is successful when the students
correctly deduce the male glans penis is complementary to which female structure?
A. vulva
B. vagina
C. labia majora
D. clitoris

Answer: D

Rationale: The female clitoris is complementary (equitable) to the glans; both of


these are composed of erectile tissue and are the primary organs responsible for
orgasm. Other comparable structures include the testes and ovaries, and scrotum
and labia majora. The vagina does not have a comparable male structure.
Question format: Multiple Choice
Chapter 3: Structure and Function of the Reproductive System
Cognitive Level: Apply
Client Needs: Health Promotion and Maintenance
Integrated Process:
Reference: p. 34

19. A young male asks the nurse at the clinic about what he can do to maintain his
sperm production. The nurse would instruct him to take which action?
A. Wear snug fitting pants to support the scrotum.
B. Buy only cotton underwear for use.
C. Take long, hot baths to encourage sperm production.
D. It is recommended that men wear boxer-style underwear.

Answer: D

Rationale: It is recommended that men wear loose fitting pants and underwear in
order to keep the testes cooler than body temperature to best support sperm
production. Tight pants and hot baths are counterproductive to sperm production;
wearing cotton underwear does not affect it.
Question format: Multiple Choice
Chapter 3: Structure and Function of the Reproductive System
Cognitive Level: Apply
Client Needs: Health Promotion and Maintenance
Integrated Process: Teaching/Learning
Reference: p. 37

20. The alkaline fluids secreted by the prostate and Cowper glands serve what
functions related to sperm? Select all that apply.
A. Provide nourishment of the sperm
B. Protect the sperm from the acidic vaginal environment
C. Help mature the sperm
D. Enhance sperm motility
E. Cool the sperm prior to ejaculation

Answer: A, B, D
Rationale: Alkaline seminal fluids serve several purposes, including nourishing the
sperm by providing fructose, protecting the sperm from acidic vaginal fluids, and
enhancing motility. The alkaline fluids do not help in sperm maturation or cooling
the sperm, which would be ill advised.
Question format: Multiple Select
Chapter 3: Structure and Function of the Reproductive System
Cognitive Level: Apply
Client Needs: Health Promotion and Maintenance
Integrated Process: Teaching/Learning
Reference: p. 38

21. The nurse is assessing a female client who is having difficulty experiencing an
orgasm. Which action should the nurse point out as often being necessary for an
orgasm in females?
A. penile penetration
B. clitoral stimulation
C. uterine stimulation
D. sensory deprivation

Answer: B

Rationale: The clitoris is highly sensitive and allows the woman to experience
pleasure during sexual stimulation. It is thought to be the main stimulus for
initiating orgasm in women. Penetration may be pleasurable, but alone it is not
enough to stimulate orgasm. Uterine stimulation does not affect orgasm. Sensory
arousal also plays a major role in attaining female orgasm.
Question format: Multiple Choice
Chapter 3: Structure and Function of the Reproductive System
Cognitive Level: Apply
Client Needs: Health Promotion and Maintenance
Integrated Process: Teaching/Learning
Reference: p. 39

22. The school nurse is preparing a teaching session for a group of adolescent girls
explaining the menstrual cycle. The nurse determines the session is successful
when the young students correctly choose which fact?
A. The average menstrual flow lasts 4 to 6 days.
B. The average age for menarche is 11 to 13 years.
C. Menarche is the start of puberty.
D. Total blood loss each month averages 45 to 80 ml of blood.

Answer: A

Rationale: The average flow lasts 4 to 6 days and is caused by casting away of
blood, tissue, and debris from the uterus. The average age for menarche is from 12
to 14 years. Menarche is the end of puberty, which follows breast budding and the
appearance of pubic hair. The total blood loss each month is 25 to 60 ml.
Question format: Multiple Choice
Chapter 3: Structure and Function of the Reproductive System
Cognitive Level: Apply
Client Needs: Health Promotion and Maintenance
Integrated Process: Teaching/Learning
Reference: p. 42

23. The nurse is teaching an adolescent client about fertility and the various phases
of the menstrual cycle. The client has a 28-day menstrual cycle. Which statement is
a priority for the nurse to include in the teaching?
A. "Days 1 through 5 are the days you will typically experience menstrual flow."
B. "Day 10 is part of the proliferative phase of your menstrual cycle."
C. "Day 14 of your menstrual cycle is normally when you will ovulate."
D. "Days 27 and 28 are when you have declining hormonal secretions."

Answer: C

Rationale: It is a priority for the nurse to teach the client when ovulation will occur
to assist in pregnancy prevention at this age. While the other statements are all
true, not knowing the specific information for those days does not hold the same
consideration or life-altering potential as when pregnancy can occur. The menstrual
cycle is based on an average of 28 days and divided into four phases based on the
hormones secreted and their actions. Days 1 through 5 are the menstruation
phase. Days 6 through 14 are the proliferative phase, ending with ovulation on the
14th day. Days 15 through 26 are the secretory phase. Days 27 and 28 are the
ischemic phase when hormonal secretions decline.
Question format: Multiple Choice
Chapter 3: Structure and Function of the Reproductive System
Cognitive Level: Apply
Client Needs: Health Promotion and Maintenance
Integrated Process: Teaching/Learning
Reference: p. 43

24. A nurse is conducting a sexual health education class about the structures of
the female reproductive system. Which structure will the class identify as
responsible for contractions during labor if the teaching has been successful?
A. endometrium
B. myometrium
C. broad ligament
D. corpus

Answer: B

Rationale: The corpus is the main body of the uterus, and the fundus is the top-
most section resembling a dome. The walls of the corpus and fundus have three
layers. The perimetrium is the tough outer layer of connective tissue that supports
the uterus. The middle layer is the myometrium, a muscular layer that is
responsible for the contractions of labor. The muscle fibers of the myometrium
wrap around the uterus in three directions: obliquely, laterally, and longitudinally.
This muscle configuration allows for the strong contractions and expulsion of the
fetus during labor and birth. The endometrium is the vascular mucosal inner layer.
This layer changes under hormonal influence every month in preparation for
possible conception and pregnancy. Four ligaments provide support and hold the
uterus in position. These ligaments anchor the uterus at the base (cervical region),
leaving the upper portion (corpus) free in the pelvic cavity. The broad ligament
attaches the lower sides of the uterus to the sidewalls of the pelvis.
Question format: Multiple Choice
Chapter 3: Structure and Function of the Reproductive System
Cognitive Level: Understand
Client Needs: Health Promotion and Maintenance
Integrated Process: Teaching/Learning
Reference: p. 40

25. A nurse is conducting a sexual health education class about the structures and
events of semen production. Which component will the students identify as
responsible for semen production if the teaching has been successful?
A. accessory gland alkaline fluids
B. sperm
C. testes
D. vas deferens
E. seminiferous tubules

Answer: A, B

Rationale: The alkaline fluids from the accessory glands and sperm combine to form
a thick, whitish secretion termed semen or seminal fluid. The testes and
seminiferous tubules within the testes are where sperm are produced. The vas
deferens is the muscular tube in which sperm begin their journey out of the man's
body. It connects the epididymis with the ejaculatory duct.
Question format: Multiple Select
Chapter 3: Structure and Function of the Reproductive System
Cognitive Level: Understand
Client Needs: Health Promotion and Maintenance
Integrated Process: Teaching/Learning
Reference:
Chapter 4
1. A nurse is counseling women on birth control choices. Which woman is the best
candidate for an IUD?
A. a woman who has multiple sex partners
B. a woman who has a history of PID
C. a woman who is in her early twenties and wants children later in life
D. a woman who has one partner and three children

Answer: D

Rationale: A women who is in a stable monogamous relationship and has already


had one successful pregnancy is the best candidate for an IUD. An IUD is not
without risk to a woman. IUDs do not protect against STIs. PID may be caused by
IUDs. A woman in her twenties has several better, lower risk options.
Question format: Multiple Choice
Chapter 4: Special Issues of Women's Health Care and Reproduction
Cognitive Level: Apply
Client Needs: Health Promotion and Maintenance
Integrated Process: Nursing Process
Reference: p. 69

2. A married couple, both age 27, is in the reproductive clinic and have been trying
for a year to become pregnant. In a year, what percentage of couples usually do
conceive?
A. 40 percent
B. 60 percent
C. 90 percent
D. 100 percent

Answer: C

Rationale: Ninety percent of couples who engage in intercourse regularly will


conceive within a year.
Question format: Multiple Choice
Chapter 4: Special Issues of Women's Health Care and Reproduction
Cognitive Level: Remember
Client Needs: Health Promotion and Maintenance
Integrated Process: Caring
Reference: p. 73-74

3. A 20-year-old college student presents to the health clinic reporting fatigue,


nausea, vomiting, severe abdominal cramping radiating to the lower back, and
headaches. She reports these symptoms have started accompanying her menstrual
cycle, which started 2 days ago. The nurse anticipates the health care provider will
attempt to rule out which disorder first?
A. premenstrual syndrome
B. toxic shock syndrome
C. secondary dysmenorrhea
D. primary dysmenorrhea

Answer: C

Rationale: Dysmenorrhea is painful or difficult menses. Primary refers to painful


menstrual periods not associated with a disease process; secondary dysmenorrhea
is related to a pelvic pathology and should be ruled out first to ensure a serious
problem is not neglected but corrected as quickly as possible. PMS is a normal
change in mood and physical discomfort that occurs with menstruation. Toxic shock
syndrome is caused by bacterial infections.
Question format: Multiple Choice
Chapter 4: Special Issues of Women's Health Care and Reproduction
Cognitive Level: Apply
Client Needs: Physiological Integrity: Reduction of Risk Potential
Integrated Process: Nursing Process
Reference: p. 53

4. The nurse is preparing to teach a 22-year-old college student with PID how to
prevent a repeat episode in the future. Which suggestion should the nurse
prioritize?
A. Avoid sexual intercourse.
B. Use latex condoms.
C. Douche following intercourse.
D. Use diaphragms.

Answer: A

Rationale: PID is an inflammation of any portion of a woman's reproductive tract


often secondary to an infection, especially STIs. The patient should be encouraged
to avoid sexual intercourse until she is in a monogamous relationship. If the client
does not desire to do that, then a barrier such as a condom or diaphragm would be
the next suggestion. It will still be possible, however, to contract an infection and
develop PID again. Douching is never recommended due to its potential to push any
bacteria which may be present up into the uterus, increasing the chances of an
infection.
Question format: Multiple Choice
Chapter 4: Special Issues of Women's Health Care and Reproduction
Cognitive Level: Apply
Client Needs: Health Promotion and Maintenance
Integrated Process: Teaching/Learning
Reference: p. 55-56

5. The school nurse is preparing a health education session on the topic of birth
control for a college group. Which method should the nurse emphasize for this
group?
A. barrier
B. hormonal
C. rhythm
D. transdermal

Answer: A

Rationale: The nurse should emphasize a barrier method of birth control such as a
condom. The condom will help not only with birth control but with the spread of
STIs, which is common in this age group. The other methods, such as hormonal,
rhythm, and transdermal, will only prevent pregnancy and are more commonly
utilized in long-term monogamous relationships.
Question format: Multiple Choice
Chapter 4: Special Issues of Women's Health Care and Reproduction
Cognitive Level: Apply
Client Needs: Health Promotion and Maintenance
Integrated Process: Teaching/Learning
Reference: p. 64-65

6. The nurse is preparing a teaching session for a client considering tubal ligation.
Which factor should the nurse prioritize in this session?
A. Wait several months after birth, and schedule the surgery as an outpatient.
B. The procedure is easy to perform and will be painless.
C. This is a permanent and irreversible procedure for birth control.
D. She must have signed consent from her partner.

Answer: C

Rationale: The procedure is considered permanent and irreversible. This is a


procedure not for routine birth control but for permanent birth control. If the
women elects for this procedure, it can be done immediately following the birth of
the child, lessening the inconvenience or hospitalization of the client. The procedure
is not painless, nor easy. A consent form from the partner is not always required.
Question format: Multiple Choice
Chapter 4: Special Issues of Women's Health Care and Reproduction
Cognitive Level: Apply
Client Needs: Health Promotion and Maintenance
Integrated Process: Teaching/Learning
Reference: p. 70

7. The nurse is conducting a well-visit physical assessment on a 29-year-old


female. The nurse determines the client is probably ovulating based on which
condition of the cervical mucus?
A. Thin, slippery, and stretchy
B. Cloudy, thick, and watery
C. Clear and of large quantity
D. Thin and red-tinged
Answer: A

Rationale: The mucus at the time of ovulation is thin, slippery, and stretchy to allow
for the passage of sperm into the uterus. This is called spinnbarkeit fiber. If the
mucus is too thick, it will inhibit fertilization. Before ovulation, the mucus is thick
and does not stretch easily.
Question format: Multiple Choice
Chapter 4: Special Issues of Women's Health Care and Reproduction
Cognitive Level: Apply
Client Needs: Health Promotion and Maintenance
Integrated Process: Nursing Process
Reference: p. 62

8. A community health nurse is preparing a presentation for a health fair on the


topics of planning for a pregnancy. Which major goal has the nurse determined
should be accomplished with this presentation?
A. Ensure women are using the correct contraceptive method.
B. Ensure couples understand genetic risks.
C. Provide one-on-one counseling.
D. Decrease the number of unwanted pregnancies.

Answer: D

Rationale: Preconception counseling with general health care is a health promotion


activity to help prevent unwanted pregnancies. The nurse will be responsible for
presenting the basic information to everyone. In this environment it would not be
appropriate for the nurse to provide one-on-one counseling due to privacy
concerns. The nurse could present various risks but it would be beyond the scope of
this event to ensure every participant understands each point.
Question format: Multiple Choice
Chapter 4: Special Issues of Women's Health Care and Reproduction
Cognitive Level: Apply
Client Needs: Health Promotion and Maintenance
Integrated Process: Teaching/Learning
Reference: p. 58-59

9. A young couple are disappointed that they are not yet pregnant and are seeking
assistance at the health clinic. After assessing their medical history, the nurse
discovers the female has a history of several episodes of PID. The nurse predicts
this may be a source of the infertility related to which factor?
A. It causes anovulation due to interference with secretion of pituitary hormones.
B. It causes changes in cervical mucus that make it less receptive to penetration by
sperm.
C. It causes sperm-agglutinating antibodies to be produced in the vagina.
D. It interferes with the transport of ova due to tubal scarring.

Answer: D
Rationale: Pelvic inflammatory disease results in scarring and adhesions of the
tubes, leading to poor transport of ova. PID does not affect hormone metabolism,
nor does it affect the production of cervical mucus. Antibodies are present only in a
few cases and are unrelated to PID.
Question format: Multiple Choice
Chapter 4: Special Issues of Women's Health Care and Reproduction
Cognitive Level: Apply
Client Needs: Physiological Integrity: Physiological Adaptation
Integrated Process: Nursing Process
Reference: p. 55-56

10. The nurse is preparing to meet with a couple who have requested counseling
concerning family planning. The nurse will prioritize which topics for discussion?
A. Ensuring the couple is legally married
B. Ensuring the correct time frame is followed
C. Possible genetic abnormalities
D. Preventing STIs

Answer: C

Rationale: Family planning consists of two complementary components: planning


pregnancy and preventing pregnancy. The nurse may be involved with teaching the
families how to avoid unwanted pregnancies, bring about wanted births, and control
the intervals between births. Discussing potential genetic anomalies would be
involved as the family prepares for children. It would not be proper for the nurse to
ignore this couple based on their marriage status. The time frame would be
determined by the couple and is flexible. Preventing STIs would also not be a focus
of this session.
Question format: Multiple Choice
Chapter 4: Special Issues of Women's Health Care and Reproduction
Cognitive Level: Apply
Client Needs: Health Promotion and Maintenance
Integrated Process: Teaching/Learning
Reference: p. 58-60

11. A client is requesting information on the various available contraceptives. When


explaining a vaginal spermicide, which information should the nurse prioritize?
A. Wash with clean water and soap after intercourse.
B. Insert the product by applicator in the vagina prior to intercourse.
C. Apply the spermicide directly to the glans penis to kill the sperm when they exit
the penis.
D. Leave the product in place for 24 hours after intercourse.

Answer: B

Rationale: Vaginal spermicides provide a physical barrier that prevents sperm


penetration and a chemical barrier that kills sperm. It is designed to be inserted
vaginally immediately before or within a few hours before sexual intercourse.
Hygiene is always important and is unrelated to the use of spermicides. Applying
the spermicide to the penis would be ineffective as it would rub off. The product is
immediately effective and can be removed right after intercourse.
Question format: Multiple Choice
Chapter 4: Special Issues of Women's Health Care and Reproduction
Cognitive Level: Apply
Client Needs: Physiological Integrity: Pharmacological and Parenteral Therapies
Integrated Process: Teaching/Learning
Reference: p. 63

12. A 30-year-old woman reports that she has not had a menstrual period for the
last 4 or 5 months. Pregnancy is ruled out and she is experiencing no endocrine
symptoms. What factors could be contributing to her amenorrhea? Select all that
apply.
A. ovarian dysfunction
B. working out at the gym 3 hours per day
C. hypothyroidism
D. endometrial adhesions
E. history of chemotherapy

Answer: B, C, D

Rationale: Amenorrhea without endocrine symptoms often is caused by disorders


that interfere with hypothalamic function, such as excessive exercise, malnutrition,
and endocrine disorders of the thyroid gland. It may also be caused by outflow tract
problems such as adhesions of the endometrium, an imperforate hymen, or female
circumcision. The other two factors are associated with amenorrhea due to
endocrine problems.
Question format: Multiple Select
Chapter 4: Special Issues of Women's Health Care and Reproduction
Cognitive Level: Apply
Client Needs: Physiological Integrity: Reduction of Risk Potential
Integrated Process: Nursing Process
Reference: p. 52

13. The nurse is teaching a young couple who desire to start their family the
various methods for determining fertility. After discovering the woman regularly
travels internationally for work, deals with a lot of job anxiety, and frequently uses
an electric blanket at home, the nurse will discourage the use of which method?
A. calendar method
B. basal body temperature method
C. cervical mucus method
D. symptothermal method

Answer: B

Rationale: BBT is a method where the body temperature should be checked and
recorded first thing in the morning, immediately after waking and before getting out
of bed. It is important for the patient to maintain a normal bedtime routine. Use of
an electric blanket, stress, and anxiety can cause a false elevation in the BBT. The
calendar method would depend upon her schedule. Cervical mucus and
symptothermal methods would be viable options.
Question format: Multiple Choice
Chapter 4: Special Issues of Women's Health Care and Reproduction
Cognitive Level: Apply
Client Needs: Health Promotion and Maintenance
Integrated Process: Nursing Process
Reference: p. 62

14. The nurse is meeting with a 36-year-old client who wishes to begin using
contraceptives. The client reports being in a long-term, monogamous relationship,
runs 2 miles per day, and smokes a pack of cigarettes each day. Which method will
the nurse be least likely to suggest to the client?
A. oral contraceptive pills (OCPs)
B. condoms
C. coitus interruptus
D. spermicides

Answer: A

Rationale: Oral contraceptive pills (OCPs) are contraindicated for women who
smoke; these women would be at a higher risk for blood clots. Condoms, coitus
interruptus, and spermicides can be used at no risk.
Question format: Multiple Choice
Chapter 4: Special Issues of Women's Health Care and Reproduction
Cognitive Level: Apply
Client Needs: Health Promotion and Maintenance
Integrated Process: Nursing Process
Reference: p. 68

15. A client is desiring to start birth control and is questioning the nurse concerning
the best options. Which option should the nurse point out is the most reliable?
A. diaphragms
B. condoms
C. oral contraceptive pills (OCPs)
D. hormonal implant

Answer: D

Rationale: Hormonal implant is a small, flexible, plastic rod implanted into the arm
of the client. This method requires replacement only every 3 years and has a high
success rate for pregnancy prevention. The client does not have to remember to do
anything. Oral contraceptive pills, although highly effective, depend on the
individual remembering to take the pill every day at the same time without fail.
Diaphragms and condoms can have structural failures (a hole) and would be less
reliable.
Question format: Multiple Choice
Chapter 4: Special Issues of Women's Health Care and Reproduction
Cognitive Level: Apply
Client Needs: Health Promotion and Maintenance
Integrated Process: Nursing Process
Reference: p. 61

16. A client prescribed a combined oral contraceptive (COC) has presented for a
routine visit. Which finding if reported by the client upon assessment should the
nurse prioritize?
A. abdominal pain
B. small amount of breakthrough bleeding
C. light menstrual flow
D. cramping during menses

Answer: A

Rationale: The warning signs to report for a client on combined oral contraceptives
are severe abdominal or chest pain, dyspnea, headache, weakness, numbness,
blurred or double vision, speech disturbances, or severe leg pain and edema. Light
bleeding, light flow, and cramping are all normal.
Question format: Multiple Choice
Chapter 4: Special Issues of Women's Health Care and Reproduction
Cognitive Level: Apply
Client Needs: Physiological Integrity: Pharmacological and Parenteral Therapies
Integrated Process: Nursing Process
Reference: p. 68

17. The nurse has assessed several clients who have arrived for routine
appointments. The nurse predicts the health care provider will prioritize a bone
density scan for which client?
A. a 40-year-old black client, 5 ft 4 in (1.62 m) tall, 172 lbs (78 kg), inactive
lifestyle
B. a 25-year-old Asian client, 5 ft 7 in (1.7 m) tall, 129 lbs (58.5 kg), with two
children
C. a 55-year-old white client who smokes and has family history of osteoporosis
D. a 45-year-old Hispanic client with a vitamin D deficiency

Answer: C

Rationale: Risk factors for osteoporosis include female gender, white or Asian
ethnicity, slender build, advanced age, estrogen deficiency because of menopause,
low bone mass density, family history of osteoporosis, personal history of fracture
as an adult, smoking, excessive alcohol intake, low dietary intake of calcium,
vitamin D deficiency, inactive lifestyle, and use of glucocorticoids and
anticonvulsants. The more of these risk factors an individual has, the higher the risk
for osteoporosis. The 55-year-old white smoker with a family history of
osteoporosis should be referred for bone scan or density testing. The other
individuals have fewer overall risk factors.
Question format: Multiple Choice
Chapter 4: Special Issues of Women's Health Care and Reproduction
Cognitive Level: Apply
Client Needs: Health Promotion and Maintenance
Integrated Process: Nursing Process
Reference: p. 78

18. The nurse is assessing a young couple who desire to get pregnant. The 38-
year-old husband and 29-year-old wife report they had used oral contraceptive pills
(OCPs); however, they have now been trying unsuccessfully to conceive over the
past 4 months. What is the best response for the nurse to make?
A. Return in 9 months for further assessment if not pregnant.
B. Should seek fertility counseling from a specialist.
C. Increase intercourse frequency to four times a week around the time of
ovulation.
D. Should undergo comprehensive diagnostic testing.

Answer: C

Rationale: The most fertile time is the ovulation period of the woman's ovarian
cycle. Increasing the frequency of intercourse around ovulation will increase the
chance of conception. Individuals are not considered infertile until they have tried
for at least 1 year to get pregnant. If this couple is still not pregnant after 8
months, then they can be referred for a fertility workup.
Question format: Multiple Choice
Chapter 4: Special Issues of Women's Health Care and Reproduction
Cognitive Level: Apply
Client Needs: Health Promotion and Maintenance
Integrated Process: Nursing Process
Reference: p. 74-75

19. The nurse is assessing a 52-year-old perimenopausal female who is concerned


about the changes occurring in her body. When questioned about the most serious
changes, which effect should the nurse point out?
A. pelvic support muscles lose tone
B. dense breast tissue is replaced with adipose tissue
C. the uterus and ovaries decrease in size
D. bone mineral density decreases

Answer: D

Rationale: As women age, the decrease in hormone levels place women at


increased risk for osteoporosis. This is a potential concern that can impact life, and
bone mineral density should be monitored. There is no evidence of atrophy in the
pelvic muscles, nor in the uterus or ovaries as related to menopause. Breast tissue
changes at the same rate that all other tissue changes.
Question format: Multiple Choice
Chapter 4: Special Issues of Women's Health Care and Reproduction
Cognitive Level: Apply
Client Needs: Physiological Integrity: Physiological Adaptation
Integrated Process: Nursing Process
Reference: p. 78

20. The nurse is preparing to teach a client how to conduct the basal body
temperature method to determine her fertile window. Which instruction should the
nurse prioritize?
A. It depends on the average temperature taken each morning.
B. Temperature should be taken prior to any activity every morning.
C. Avoid intercourse only on the days the temperature drops.
D. It is the best method for predicting ovulation.

Answer: B

Rationale: BBT is the basal body temperature method and requires the woman to
take her temperature and record it every morning. This should be the first activity
of the day before exiting the bed or doing other activities. To prevent conception,
avoid unprotected intercourse from the day the BBT drops through the fourth day of
temperature elevation. The BBT alone is not a reliable method for predicting
ovulation. Use BBT along with calendar or cervical mucus methods to increase
effectiveness.
Question format: Multiple Choice
Chapter 4: Special Issues of Women's Health Care and Reproduction
Cognitive Level: Apply
Client Needs: Health Promotion and Maintenance
Integrated Process: Teaching/Learning
Reference: p. 62

21. The nurse was teaching a college student how to properly take the prescribed
combination oral contraceptive (COC). The nurse determines the session is
successful when the client correctly chooses which instruction to follow when taking
the OCP?
A. empty stomach, full glass of water
B. same time of day, each day
C. take with a meal
D. same day of the week, once a week

Answer: B

Rationale: OCPs are to be taken daily at the same time of day every day.
Absorption is not affected by the stomach contents. Oral contraceptive pills must be
taken every day to be effective.
Question format: Multiple Choice
Chapter 4: Special Issues of Women's Health Care and Reproduction
Cognitive Level: Analyze
Client Needs: Physiological Integrity: Pharmacological and Parenteral Therapies
Integrated Process: Teaching/Learning
Reference: p. 67

22. A young couple is having difficulty getting pregnant. The nurse is preparing the
couple for the initial tests to determine their fertility. When asked by the couple
why they need to start with a sperm analysis, what will be the nurse's best
response?
A. Asking the male to undergo diagnostic procedures first is the best way to assess
interest in treatment of infertility.
B. The man is found to be fertile in over 75 percent of couples with infertility
problems.
C. Male fertility testing is time-consuming and therefore should be initiated early.
D. Sperm analysis is one of the easiest tests to complete.

Answer: D

Rationale: Because sperm analysis is noninvasive, it is one of the easiest (and most
cost-effective) tests to complete. There is no bias in male vs. female with regard to
being the cause of infertility.
Question format: Multiple Choice
Chapter 4: Special Issues of Women's Health Care and Reproduction
Cognitive Level: Apply
Client Needs: Physiological Integrity: Reduction of Risk Potential
Integrated Process: Teaching/Learning
Reference: p. 76

23. The nurse is teaching a client about a vasectomy. The nurse determines the
session is successful when the client correctly chooses which fact concerning the
vasectomy?
A. Usually done as an office procedure
B. Will no longer ejaculate
C. May notice permanent increased scrotal swelling
D. Testes will no longer produce sperm.

Answer: A

Rationale: Vasectomy is an outpatient surgical procedure that is usually performed


in the office. It involves ligating and cutting the vas deferens, which stops the flow
of sperm from the testes to the outside of the body; however, it does not stop
sperm production. There may be short-term swelling immediately after the
procedure, but there should not be any permanent swelling. The client should still
have normal sexual function.
Question format: Multiple Choice
Chapter 4: Special Issues of Women's Health Care and Reproduction
Cognitive Level: Apply
Client Needs: Physiological Integrity: Reduction of Risk Potential
Integrated Process: Teaching/Learning
Reference: p. 70

24. The nurse is conducting a routine well-visit on a 43-year-old client who is


concerned about developing breast cancer, although no family members have
experienced it. What is the best advice for this client at this time?
A. Be aware of the normal appearance and feel of your breasts.
B. Inspect your nipples every day for discharge or bleeding.
C. Ask the health care provider for recommendations based on family history.
D. Have a yearly mammogram as part of a comprehensive screening.

Answer: A

Rationale: The American Cancer Society recommends that each woman know how
her breasts normally look and feel, and if she finds changes, she should
immediately report them to a health care provider. Discharge from the nipple is not
necessarily related to cancer. For women with an average risk for breast cancer,
yearly mammograms should start at age 45 and can change to every 2 years
beginning at age 55. Women who are at a higher risk for breast cancer due to
family history or other reasons may need to begin screening earlier and/or more
often.
Question format: Multiple Choice
Chapter 4: Special Issues of Women's Health Care and Reproduction
Cognitive Level: Apply
Client Needs: Health Promotion and Maintenance
Integrated Process: Teaching/Learning
Reference: p. 49

25. A 24-year-old female client states, "About a week before my period starts, I
have recently started craving certain sweets, having terrible mood swings, and
feeling fatigued and irritable." Which action will the nurse take next?
A. Document the findings in the client's medical record.
B. Educate the client on treatment for premenstrual syndrome.
C. Notify the client's primary health care provider.
D. Recommend the client's hormone levels be tested.

Answer: B

Rationale: The client is exhibiting symptoms of premenstrual syndrome and the


nurse would provide education on treatment options for the client's specific
symptoms, such as increased rest and exercise. The nurse would document the
findings and education provided in the client's medical record and notify the
primary health care provider; however, these are not a priority as the client is
stable and in need of treatment options. The client does not have symptoms
warranting hormone testing at this time. If the symptoms worsened or became
intolerable, then testing may be needed.
Question format: Multiple Choice
Chapter 4: Special Issues of Women's Health Care and Reproduction
Cognitive Level: Apply
Client Needs: Health Promotion and Maintenance
Integrated Process: Teaching/Learning
Reference: p. 53
Chapter 5
1. In caring for a fully immunized pregnant woman who is a nurse in a family health
practice, the obstetric nurse should remind the client that she must not come in
contact with clients who have symptoms that could indicate which infection?
A. measles
B. Chicken pox
C. smallpox
D. diphtheria

Answer: B

Rationale: Chicken pox can be teratogenic to the fetus and exposure may have a
significant impact on the fetus and may cause defects. The fetus receives passive
immunity from the mother for measles, diphtheria, and smallpox.
Question format: Multiple Choice
Chapter 5: Fetal Development
Cognitive Level: Apply
Client Needs: Health Promotion and Maintenance
Integrated Process: Nursing Process
Reference: p. 92

2. Assessment for surfactant level via lecithin/sphingomyelin (L/S) ratio in the


amniotic fluid is a primary estimation of fetal maturity. The purpose of surfactant is
to:
A. prevent alveoli from collapsing on expiration.
B. increase lung resistance on inspiration.
C. encourage immunologic competence of lung tissue.
D. promote maturation of lung alveoli.

Answer: A

Rationale: Surfactant is a phospholipid that reduces surface tension; it prevents


alveoli from collapsing on expiration. Resistance to airflow is an effect of tissue
elasticity and airway size. Immunologic competence is provided by antibodies in the
mucus layer. Fully matured alveoli contain squamous cells as well as type II
surfactant cells.
Question format: Multiple Choice
Chapter 5: Fetal Development
Cognitive Level: Remember
Client Needs: Health Promotion and Maintenance
Integrated Process: Nursing Process
Reference: p. 90

3. A 38-year-old client presents to the clinic desiring to get pregnant. She reports
she had a tubal ligation in her early 20s after two babies and a divorce. After
learning that the client recently underwent a reversal of the tubal ligation, the
nurse will warn the client of which potential risk?
A. ectopic pregnancy
B. Down syndrome
C. twins
D. exposure to teratogens

Answer: A

Rationale: Tubal ligation reversal is a difficult procedure and can place the woman
at higher risk for ectopic pregnancy. She needs to be aware of the possibility. Down
syndrome, multiple births, and exposure to teratogens are all issues that have
nothing to do with reversal of tubal ligation. These are issues that any pregnant
female should be made aware.
Question format: Multiple Choice
Chapter 5: Fetal Development
Cognitive Level: Apply
Client Needs: Health Promotion and Maintenance
Integrated Process: Nursing Process
Reference: p. 98

4. A client and partner are excited to discover they are expecting twins. The nurse
is prepared to monitor the twins for which potential situation after noting they
share an amniotic sac?
A. increased teratogenic effects
B. conjoined twins
C. twin-to-twin transfusion syndrome
D. cord entanglement

Answer: D

Rationale: The greatest risk for monoamniotic monochorionic twins is cord


entanglement as they share the same amnio and chorion and are contained in the
amniotic fluid together. Conjoined is a possibility but not the greatest risk. Twin-to-
twin transfusion syndrome is also a possibility but is a greater risk for diamniotic
monochorionic as they share either placenta or vessels. Fraternal (dizygotic) twins
with two placentas, two amnions, and two chorions are diamniotic dichorionic and
would not be at risk for the cord entanglement.
Question format: Multiple Choice
Chapter 5: Fetal Development
Cognitive Level: Apply
Client Needs: Physiological Integrity: Reduction of Risk Potential
Integrated Process: Nursing Process
Reference: p. 98

5. The nurse is assessing a 38-year-old Black client who has just discovered she is
pregnant. On assessment, the nurse documents the client is 5 ft 10 in (1.77 m) tall
and stopped using oral contraceptives 3 months ago hoping to become pregnant.
Which situation may accompany her pregnancy?
A. hypertension
B. sickle cell anemia
C. twins
D. ectopic pregnancy

Answer: C

Rationale: Women who have recently stopped using oral contraceptives, are tall or
large stature, or are Black race have an increased chance for multifetal pregnancy.
She is at no higher risk of hypertension or sickle cell anemia than she would be if
she had not gotten pregnant. Ectopic pregnancy is not influenced by age or race.
Question format: Multiple Choice
Chapter 5: Fetal Development
Cognitive Level: Apply
Client Needs: Health Promotion and Maintenance
Integrated Process: Nursing Process
Reference: p. 98

6. At a 12-week visit, the nurse assesses a pregnant woman who is questioning


what the baby may look like at this point. Based on the current stage, which
structures would the nurse point out to the client?
A. Budlike structures for arms and legs
B. Beating heart and differentiated fingers and toes
C. Audible heartbeat and identifiable sex characteristics
D. Fully shaped lungs and a defined skeletal system

Answer: C

Rationale: At the end of 12 weeks' gestation, most structures are fully developed
but need time to grow. The heartbeat is audible by Doppler, gender is discernible
by outward appearance, the placenta is complete, divisions of the brain begin to
develop, the face is well formed, the eyes are widely spaced and fused, kidney
secretion has begun, and spontaneous movements occur. The arms and legs are
budlike structures by the end of 4 weeks. By the end of 8 weeks, the heart with a
septum and valves is beating rhythmically and the fingers and toes are distinct and
separated. The skeletal structure is identifiable at the end of 16 weeks. The lungs
do not take on their final shape until after birth when the infant starts to oxygenate
its own blood by its own lungs. By the end of 32 weeks the lungs are not yet fully
developed, but the fetus usually does well if born at this time.
Question format: Multiple Choice
Chapter 5: Fetal Development
Cognitive Level: Apply
Client Needs: Health Promotion and Maintenance
Integrated Process: Nursing Process
Reference: p. 91
7. The parents are questioning why their newborn was born deaf when there are no
other deaf family members. The nurse could explore possible exposure to a
teratogenic agent at which stage of the pregnancy?
A. 6 weeks
B. at fertilization
C. 12 weeks
D. 18 weeks

Answer: A

Rationale: Teratogenic agents ingested during the embryonic stage (2 to 8 weeks)


can affect the neurologic system of the fetus, including the hearing. During weeks 6
through 8, the ear is most vulnerable to teratogenic agents. From the preembryonic
stage of fertilization to the beginning of week 2, there is decreased risk due to no
implantation or transfer of substances from the mother to the developing
blastocyte. During the fetal stage (9 weeks to birth), the fetus is fully formed and is
now concentrating on increasing in size. There is a decreased risk from teratogenics
during this time period.
Question format: Multiple Choice
Chapter 5: Fetal Development
Cognitive Level: Apply
Client Needs: Physiological Integrity: Physiological Adaptation
Integrated Process: Nursing Process
Reference: p. 97

8. A group of nursing students are preparing a presentation depicting the fetal


circulation. The instructor determines the presentation is successful when the
students correctly illustrate which route for the ductus arteriosus?
A. The left to right heart atria
B. The aorta to the pulmonary veins
C. The right ventricle to the aorta
D. The pulmonary artery to the aorta

Answer: D

Rationale: Because the fetal lungs are not inflated, blood must be diverted past
them. The ductus arteriosus helps to do this by shunting blood from the pulmonary
artery to the aorta. The foramen ovale diverts blood from the right atrium to the
left atrium, bypassing the lungs. The ductus venosus diverts a portion of blood from
the umbilicus to the inferior vena cava instead of passing through the liver first.
Question format: Multiple Choice
Chapter 5: Fetal Development
Cognitive Level: Analyze
Client Needs: Health Promotion and Maintenance
Integrated Process: Teaching/Learning
Reference: p. 94
9. The nurse is preparing a presentation for a health fair which will illustrate the
development of a baby. The nurse should point out the fertilized egg is implanted in
the endometrium by which day?
A. 4
B. 6
C. 8
D. 10

Answer: D

Rationale: By day 10 after fertilization, the blastocyst has completely buried itself in
the endometrial lining. Prior to day 10, the attachment is much looser.
Question format: Multiple Choice
Chapter 5: Fetal Development
Cognitive Level: Apply
Client Needs: Health Promotion and Maintenance
Integrated Process: Nursing Process
Reference: p. 88

10. The nurse is meeting with a young couple who desire to get pregnant to teach
them how to determine the best times for intercourse. During which time frame
should the nurse encourage them to engage in intercourse to increase their chances
of getting pregnant?
A. 2 days before to 1 day after ovulation
B. 1 day before ovulation to 2 days after ovulation
C. 3 days before or the day of ovulation
D. 3 days before to 2 days after ovulation

Answer: D

Rationale: Sperm are able to live for up to 72 hours after ejaculation and the ovum
remains fertile for a maximum of 48 hours after ovulation. The window of
opportunity for conception is 3 days before to 2 days after ovulation.
Question format: Multiple Choice
Chapter 5: Fetal Development
Cognitive Level: Apply
Client Needs: Health Promotion and Maintenance
Integrated Process: Nursing Process
Reference: p. 86

11. A group of nursing students are analyzing the fetal circulation. After the
session, the students correctly point out which fetal structure contains the highest
concentration of oxygen?
A. umbilical artery
B. umbilical vein
C. ductus arteriosus
D. pulmonary vein
Answer: B

Rationale: The umbilical vein carries oxygenated blood from the placenta to the
fetus; the umbilical artery carries deoxygenated blood from the fetus to the
placenta. The ductus arteriosus shunts blood from the right atrium to the left
atrium in order to bypass the deflated lungs. The pulmonary vein drains the
deflated lungs.
Question format: Multiple Choice
Chapter 5: Fetal Development
Cognitive Level: Apply
Client Needs: Health Promotion and Maintenance
Integrated Process: Teaching/Learning
Reference: p. 94

12. The nurse is assessing a young female who just found out she is pregnant. She
is now reporting vague abdominal discomfort. After noting the client has a history
of PID, the nurse predicts the health care provider will give priority to ruling out
which situation?
A. Ectopic pregnancy
B. Repeat PID
C. UTI
D. Endometriosis

Answer: A

Rationale: An ectopic pregnancy or tubal pregnancy can result when there is


blockage or scarring of the fallopian tubes due to infection (PID) or trauma (tubal
ligation reversal). Ectopic pregnancy may present with vague signs and symptoms
but is the leading cause of maternal death in the first trimester and should be given
priority when determining the cause of abdominal complaints. The other choices
would be ruled out after the ectopic pregnancy is ruled out.
Question format: Multiple Choice
Chapter 5: Fetal Development
Cognitive Level: Apply
Client Needs: Physiological Integrity: Reduction of Risk Potential
Integrated Process: Nursing Process
Reference: p. 98

13. The nurse is conducting a prenatal class for a group of first-time parents in the
first trimester. The nurse should point out that the mother should feel the baby
move by the end of which week of gestation?
A. 16 weeks
B. 18 weeks
C. 20 weeks
D. 22 weeks

Answer: C
Rationale: On average, the first time a mother can feel the fetus move is by the
end of 20 weeks' gestation.
Question format: Multiple Choice
Chapter 5: Fetal Development
Cognitive Level: Apply
Client Needs: Health Promotion and Maintenance
Integrated Process: Teaching/Learning
Reference: p. 91

14. The nursing instructor is illustrating the circulatory flow between the mother
and fetus. The instructor determines the session is successful when the class
correctly chooses which structure with which route?
A. The one umbilical artery carries oxygen-rich blood to the fetus from the placenta.
B. The two umbilical arteries carry waste products from the placenta to the fetus.
C. The one umbilical vein carries oxygen-rich blood to the fetus from the placenta.
D. The two umbilical veins carry waste products from the fetus to the placenta.

Answer: C

Rationale: There are two umbilical arteries and one umbilical vein. The arteries
carry waste from the fetus to the placenta; the vein carries oxygenated blood to the
fetus from the placenta.
Question format: Multiple Choice
Chapter 5: Fetal Development
Cognitive Level: Apply
Client Needs: Health Promotion and Maintenance
Integrated Process: Teaching/Learning
Reference: p. 95

15. A group of nursing students are preparing a presentation for a health fair
illustrating the structures found during a pregnancy. Which structures should the
students point out form a protective barrier around the developing fetus?
A. ectoderm and amnion
B. amnion and mesoderm
C. chorion and amnion
D. chorion and endoderm

Answer: C

Rationale: The chorion and amnion are the two fetal membranes. The ectoderm,
mesoderm, and endoderm are layers in the developing blastocyst.
Question format: Multiple Choice
Chapter 5: Fetal Development
Cognitive Level: Apply
Client Needs: Health Promotion and Maintenance
Integrated Process: Teaching/Learning
Reference: p. 90
16. A nurse is discussing the importance of good nutrition to a young pregnant
client. The nurse would point out that the growing fetus is getting nutrition from the
mother via which structure?
A. decidua
B. placenta
C. amniotic fluid
D. umbilical arteries

Answer: B

Rationale: The placenta is a flat, round structure which forms on the decidua and
attaches to the fetus by the umbilical cord. The placenta is the organ responsible
for supplying nutrients and oxygenated blood to the fetus. The amniotic fluid
surrounds the fetus and provides protection, temperature regulation, allows
movement, and symmetric growth. It collects urine and other waste products from
the fetus. The decidua is the name given to the endometrium after the pregnancy
starts. The umbilical arteries carry waste products away from the fetus to the
placenta, where they are filtered out into the maternal body for proper disposal.
Question format: Multiple Choice
Chapter 5: Fetal Development
Cognitive Level: Apply
Client Needs: Health Promotion and Maintenance
Integrated Process: Nursing Process
Reference: p. 92

17. The nurse is teaching a prenatal class on the functions of the various structures
involved with a pregnancy. The nurse determines the class is successful when the
class correctly chooses which function of amniotic fluid?
A. "This is how the baby is fed."
B. "It helps cushion the baby."
C. "It prevents viruses from passing to the baby."
D. "It provides oxygen to the fetus."

Answer: B

Rationale: The amniotic fluid has four functions: physical protection, temperature
regulation, provision of unrestricted movement, and symmetrical growth. Feeding
the baby, preventing infection, and providing oxygen are functions of the placenta.
Question format: Multiple Choice
Chapter 5: Fetal Development
Cognitive Level: Apply
Client Needs: Health Promotion and Maintenance
Integrated Process: Teaching/Learning
Reference: p. 90

18. The nursing instructor is preparing a class presentation covering the various
hormones and their functions during pregnancy. The instructor determines the class
is successful when the class correctly matches which function with hCG?
A. provides rich blood supply to decidua
B. maintains nutrient-rich decidua
C. continues progesterone production by corpus luteum
D. sustains life of placenta

Answer: C

Rationale: The corpus luteum is responsible for producing progesterone until this
function is assumed by the placenta. hCG is a fail-safe mechanism to prolong the
life of the corpus luteum and ensure progesterone production. Estrogen is
responsible for providing a rich blood supply to the decidua. Progesterone helps
maintain a nutrient-rich decidua.
Question format: Multiple Choice
Chapter 5: Fetal Development
Cognitive Level: Apply
Client Needs: Health Promotion and Maintenance
Integrated Process: Nursing Process
Reference: p. 88

19. The nurse is assessing a young couple who desire to start a family and are
questioning the nurse concerning various cautions to keep in mind. Which time
period should the nurse point out that teratogenics pose the greatest risk and
should be avoided?
A. weeks 3 to 8
B. week 9 to birth
C. fertilization through week 2
D. the entire pregnancy

Answer: A

Rationale: Teratogenic agents should be avoided throughout the entire pregnancy


to include 3 months prior to conception. The embryonic stage (weeks 3 through 8)
produces the greatest risk of damaging effects because the cells are rapidly dividing
and differentiating into specific body structures. Basic structures of all major body
organs and the main external features are completed during this time period,
including internal organs. Week 9 to birth is the fetal stage; fertilization through
week 2 is the pre-embryonic stage, and is not as concerning since the embryo is
not yet connected to the mother and the chance of receiving teratogenic agents is
decreased. The CNS is vulnerable throughout the entire pregnancy since it is
continuously developing, and is most vulnerable during the embryonic stage.
Question format: Multiple Choice
Chapter 5: Fetal Development
Cognitive Level: Apply
Client Needs: Health Promotion and Maintenance
Integrated Process: Nursing Process
Reference: p. 95
20. A student nurse is preparing an illustration for a health fair depicting the
various types of twins. Which format will the student use to depict identical twins?
A. two separate eggs and different sperm
B. two separate eggs and one sperm
C. one egg and two different sperm
D. one egg and one sperm

Answer: D

Rationale: Identical twins develop from the fertilization of one egg by one sperm,
and this divides into two zygotes shortly after fertilization, resulting in two babies
who share identical genetic material and are termed identical twins. With two
separate eggs and two separate sperm, the twins would be fraternal dizygotic. One
sperm cannot fertilize two eggs, and two sperm cannot fertilize one egg.
Question format: Multiple Choice
Chapter 5: Fetal Development
Cognitive Level: Apply
Client Needs: Health Promotion and Maintenance
Integrated Process: Teaching/Learning
Reference: p. 98

21. A client calls the nurse in a panic after a home pregnancy test indicates she is
pregnant. She reports that that she consumed a lot of alcohol on the night that she
thinks the pregnancy occurred. The next day she had taken several acetaminophen.
For the past 3 weeks, she has had her usual nightly glass of wine with dinner but
no other alcohol. What is an appropriate response for the nurse to make when the
client questions if she has caused irreversible damage to the fetus?
A. "Why did you have unprotected sex if you had been drinking? Exposure to
alcohol can cause facial deformities, low birth weight, and underdeveloped brains."
B. "The fetus is not exposed to the mother's blood until after it implants about 6
days after fertilization, so the first night is not an issue. But it is best to avoid
alcohol while you are pregnant."
C. "The wedding night is not an issue because the fetus is not exposed to the
mother's blood at first, but I hope this last week of drinking has not caused any
problems."
D. "Alcohol is very damaging to the growing fetus, so you had better be sure to
stop drinking. Do you need any support for that?"

Answer: B

Rationale: The fetus was not developing during the initial night of drinking. The
embryo would be exposed to the mother's blood following the sixth day after
implantation, at which time the alcohol will be circulating in the embryo's
circulation. Alcohol use should be stopped now that the client knows she is
pregnant, but the nurse should reassure her that the limited amounts she has
consumed will most likely not have an adverse effect on the fetus. A practitioner
should not question the client's choice of drinking and having unprotected sex.
There is no indication of alcohol use disorder, so support may not be warranted.
The nurse should maintain a professional and calm attitude and not criticize or
condemn the client.
Question format: Multiple Choice
Chapter 5: Fetal Development
Cognitive Level: Apply
Client Needs: Health Promotion and Maintenance
Integrated Process: Caring
Reference: p. 95

22. The nursing instructor is teaching a class on the structures vital to the
development of the fetus. The instructor determines the class is successful when
the class correctly chooses which facts concerning amniotic fluid?
A. Produced by fetal brain
B. Amniotic fluid is 90% water
C. Production stops in the eighth month
D. Helps the fetus regulate body temperature

Answer: D

Rationale: Amniotic fluid is formed by the fetal membranes, the amnion and
chorion, on a constant basis until birth. It is 98% to 99% water, with the remaining
1% to 2% composed of electrolytes, creatinine, urea, glucose, hormones, fetal
cells, lanugo, and vernix. It serves four main functions: physical protection,
temperature regulation, provision of unrestricted movement, and symmetrical
growth. The fetus is unable to regulate its own body temperature so the amniotic
fluid provides this function.
Question format: Multiple Choice
Chapter 5: Fetal Development
Cognitive Level: Apply
Client Needs: Health Promotion and Maintenance
Integrated Process: Nursing Process
Reference: p. 90

23. A young couple, 8 weeks' pregnant with their first child, are being assessed at
their first prenatal visit. They ask about scheduling an ultrasound to find out the
gender of the fetus. For when should the nurse recommend this ultrasound be
scheduled?
A. for today (8 weeks' gestation)
B. for 12 weeks' gestation
C. for 16 weeks' gestation
D. for 20 weeks' gestation

Answer: C

Rationale: The sex organs can be distinguished at 12 weeks' gestation but are
difficult to see on routine ultrasound. The gender of the fetus can be determined at
the end of 16 weeks' gestation on ultrasound.
Question format: Multiple Choice
Chapter 5: Fetal Development
Cognitive Level: Understand
Client Needs: Health Promotion and Maintenance
Integrated Process: Teaching/Learning
Reference: p. 91

24. The nurse is conducting an obstetrics assessment on a client at 20 weeks'


gestation who is questioning the nurse about the development of the fetus. Which
new occurring developments can the nurse point out to this client?
A. Eyelids are open.
B. Lungs are fully shaped.
C. Eyebrows and scalp hair are present.
D. A developed startle reflex is evident.

Answer: C

Rationale: At 20 weeks, the fetus is still developing. All structures are present, but
not in full size. The fetus will have limited amounts of eyebrows and scalp hair. At
20 weeks, the eyelids are not present; the lungs are present, but not developed.
The startle reflex will not be evident until after birth.
Question format: Multiple Choice
Chapter 5: Fetal Development
Cognitive Level: Apply
Client Needs: Health Promotion and Maintenance
Integrated Process: Nursing Process
Reference: p. 91

25. The nurse is preparing a pregnant client with severe hypertension for an
emergent amniocentesis for possible early delivery of the fetus. The nurse will
explain to the client that the health care provider is evaluating which parameter?
A. fetal renal output
B. fetal alimentary output
C. maternal blood makeup
D. level of fetal surfactants

Answer: D

Rationale: Amniocentesis is done to check the lung surfactant ratio of the fetus,
which will determine if the lungs are matured enough for delivery. Amniocentesis
can be used to determine fetal renal and alimentary output, but these factors are
not critical to birth. Maternal blood work will reveal information about the mother
and not the lung maturity of the fetus.
Question format: Multiple Choice
Chapter 5: Fetal Development
Cognitive Level: Apply
Client Needs: Physiological Integrity: Reduction of Risk Potential
Integrated Process: Nursing Process
Reference: p. 92
Chapter 6
1. The nurse is spending time with a client who has just learned, unexpectedly, that
she is pregnant. Which initial task should the nurse assist the client to focus on?
A. Accepting the pregnancy
B. Accepting a coming child
C. Making plans for the baby
D. Sharing time with a significant other

Answer: A

Rationale: The first task of pregnancy is to accept the pregnancy. This task is
usually met during the first trimester, although some women have difficulty fully
accepting the pregnancy until they can feel the baby move. Accepting the baby or
coming child usually occurs in the second trimester. It is also during the second
trimester when the woman spends more time with her partner as they prepare for
the coming infant. The third trimester is spent making plans for the baby, getting
the nursery ready, buying necessary supplies and preparing for the child.
Question format: Multiple Choice
Chapter 6: Maternal Adaptation during Pregnancy
Cognitive Level: Apply
Client Needs: Psychosocial Integrity
Integrated Process: Nursing Process
Reference: p. 111

2. A 17-year-old client arrives for an annual examination and reports no changes


since the last exam; however, the nurse assesses a positive Chadwick sign, slightly
enlarged uterus, and subsequent positive urine pregnancy test. Which task should
the nurse prioritize to assist this client who is denying any possibility that she is
pregnant?
A. Accepting the pregnancy
B. Accepting the baby
C. Preparing for parenthood
D. Telling her partner and family

Answer: A

Rationale: Acceptance of pregnancy is multi-factorial, and how the woman responds


to the pregnancy is certainly influenced by her age and if the pregnancy was
planned. As a teenager, she may not have been trying to get pregnant and may not
want to accept the role and experience. Baby and parenthood decisions should all
occur later.
Question format: Multiple Choice
Chapter 6: Maternal Adaptation during Pregnancy
Cognitive Level: Apply
Client Needs: Psychosocial Integrity
Integrated Process: Nursing Process
Reference: p. 111

3. The nurse is assessing a 37-year-old woman, pregnant with twins in her second
trimester, and notes the following over the past 3 visits: blood pressure 128/88,
134/90, and 130/86. Which nutritional supplement should the nurse suggest the
client take?
A. vitamin A
B. iron
C. calcium
D. lactase

Answer: C

Rationale: The elevated blood pressures indicate the client is possibly developing
gestational hypertension. This increases the risk of developing preeclampsia.
Current research has demonstrated that calcium supplementation during pregnancy
may reduce the risk of preeclampsia. Excessive levels of vitamin A may cause birth
defects. Iron supplementation is used to fortify blood cell formation and decrease
anemia. Lactase supplementation aids in the digestion of dairy.
Question format: Multiple Choice
Chapter 6: Maternal Adaptation during Pregnancy
Cognitive Level: Apply
Client Needs: Physiological Integrity: Pharmacological and Parenteral Therapies
Integrated Process: Nursing Process
Reference: p. 113

4. A pregnant client in her second trimester reports feeling tired all the time. The
nurse notes pale skin and low normal hemoglobin on assessment. Which
recommendation should the nurse prioritize for this client?
A. An iron supplement
B. A calcium supplement
C. More meat in her diet
D. More seafood and organ meats in her diet

Answer: A

Rationale: Iron is necessary for the formation of hemoglobin; therefore, it is


essential to the oxygen-carrying capacity of the blood. Women who have normal
hemoglobin may need increased iron to carry more oxygen. Calcium
supplementation is essential for normal fetal development. The use of measured
supplements would ensure a steady amount, whereas the use of meat and seafood
would not allow this.
Question format: Multiple Choice
Chapter 6: Maternal Adaptation during Pregnancy
Cognitive Level: Apply
Client Needs: Health Promotion and Maintenance
Integrated Process: Nursing Process
Reference: p. 113

5. A pregnant vegan reports eating lots of dark green leafy vegetables, legumes,
citrus fruits, and berries. To ensure that her infant's nervous system will develop
properly, what foods should the nurse recommend that she add to her diet?
A. Milk and cheese
B. Carrots, sweet potatoes, and mangoes
C. Nuts, seeds, and chocolate
D. Fortified cereals

Answer: D

Rationale: The best source to recommend are the fortified cereals to meet the
amino acid needs necessary for the development of her infant's nervous system
during pregnancy. She should be encouraged to include fortified cereals to meet
these needs. The carrots, sweet potatoes, mangoes, nuts, and seeds will add other
nutrients to her diet. A vegan will not eat milk and cheese, as they are animal
products.
Question format: Multiple Choice
Chapter 6: Maternal Adaptation during Pregnancy
Cognitive Level: Apply
Client Needs: Health Promotion and Maintenance
Integrated Process: Nursing Process
Reference: p. 116

6. The nurse is assessing a pregnant client at her 20-week visit. Which breast
assessment should the nurse anticipate documenting?
A. Slack, soft breast tissue
B. Deeply fissured nipples
C. Enlarged lymph nodes
D. Darkened breast areolae

Answer: D

Rationale: As part of the pigment changes that occur with pregnancy, breast
areolae become darker. The breast tissue should not be softer or slacker than
before. There should not yet be any lymph enlargement, and the nipples should not
have fissures.
Question format: Multiple Choice
Chapter 6: Maternal Adaptation during Pregnancy
Cognitive Level: Apply
Client Needs: Health Promotion and Maintenance
Integrated Process: Nursing Process
Reference: p. 105

7. A client calls to cancel an appointment for the first prenatal visit after reporting a
home pregnancy test is negative. Which instruction should the nurse prioritize?
A. Use a diluted urine specimen.
B. Wait until after two missed menstrual periods.
C. Keep the appointment.
D. Refrain from eating for 4 hours before testing.

Answer: C

Rationale: Although home pregnancy tests are accurate 95% of the time, they may
still have false positives or false negatives, and the client needs to seek prenatal
care and confirmation from her health care provider. Diluting the urine, waiting to
miss a second period, or eating before the test would have no effect. The tests look
for hCG, which is not affected.
Question format: Multiple Choice
Chapter 6: Maternal Adaptation during Pregnancy
Cognitive Level: Apply
Client Needs: Health Promotion and Maintenance
Integrated Process: Nursing Process
Reference: p. 104

8. A client in her first trimester reports frequent urination and asks the nurse for
suggestions. The nurse should teach the client that the urination is most likely
related to which cause?
A. Pressure on the bladder from the uterus
B. Increased concentration of urine
C. Addition of fetal urine to maternal urine
D. Decreased glomerular selectivity

Answer: A

Rationale: Early in pregnancy, the expanding uterus presses on the bladder. During
the second trimester there is some relief when the uterus lifts, but the pressure
returns again as the fetus continues to grow. Urine concentration does not affect
frequency. Fetal urine does not enter the mother's renal system, except through
increases in circulatory volume. The glomeruli should not be affected by pregnancy.
Question format: Multiple Choice
Chapter 6: Maternal Adaptation during Pregnancy
Cognitive Level: Apply
Client Needs: Health Promotion and Maintenance
Integrated Process: Nursing Process
Reference: p. 109

9. A pregnant client in her third trimester, lying supine on the examination table,
suddenly grows very short of breath and dizzy. Concerned, she asks the nurse what
is happening. Which response should the nurse prioritize?
A. Cerebral arteries are growing congested with blood.
B. The uterus requires more blood in a supine position.
C. Blood is trapped in the vena cava in a supine position.
D. Sympathetic nerve responses cause dyspnea when a woman lies supine.
Answer: C

Rationale: Supine hypotension syndrome, or an interference with blood return to


the heart, occurs when the weight of the fetus rests on the vena cava. Cerebral
arteries should not be affected. Mean arterial pressure is high enough to maintain
perfusion of the uterus in any orientation. The sympathetic nervous system will not
be affected by the supine position.
Question format: Multiple Choice
Chapter 6: Maternal Adaptation during Pregnancy
Cognitive Level: Apply
Client Needs: Physiological Integrity: Physiological Adaptation
Integrated Process: Nursing Process
Reference: p. 107

10. The nurse is explaining the latest laboratory results to a pregnant client who is
in her third trimester. After letting the client know she is anemic, which heme iron-
rich foods should the nurse encourage her to add to her diet?
A. Legumes
B. Dairy
C. Grains
D. Meats

Answer: D

Rationale: Meats are the best source of heme-rich iron and should be included in
her diet if she is not following a vegetarian diet. Grains and legumes are non-heme
iron sources. Dairy products will add various vitamins and calcium to the diet.
Question format: Multiple Choice
Chapter 6: Maternal Adaptation during Pregnancy
Cognitive Level: Apply
Client Needs: Physiological Integrity: Basic Care and Comfort
Integrated Process: Nursing Process
Reference: p. 113

11. The nurse is assessing a pregnant client in her third trimester who is reporting
a first-time occurrence of constipation. When asked why this is happening, what is
the best response from the nurse?
A. There is not enough fiber in your diet.
B. The intestines are displaced by the growing fetus.
C. This shouldn't be happening.
D. hCG is delaying peristalsis.

Answer: B

Rationale: The growing fetus is displacing the intestines and interfering with
peristalsis, delaying the passage of fecal matter and resulting in constipation. This
is common and expected; however, the client should take measures to prevent
hemorrhoids that can occur as the result of the pressure and straining.
Progesterone, not hCG, can delay gastric emptying and decrease peristalsis.
Question format: Multiple Choice
Chapter 6: Maternal Adaptation during Pregnancy
Cognitive Level: Apply
Client Needs: Health Promotion and Maintenance
Integrated Process: Nursing Process
Reference: p. 109

12. The nurse is assessing a client who believes she is pregnant. The nurse points
out a more definitive assessment is necessary due to which sign being considered a
probable sign of pregnancy?
A. Fatigue
B. Amenorrhea
C. Positive home pregnancy test
D. Nausea and vomiting

Answer: C

Rationale: A urine pregnancy test is considered a probable sign of pregnancy as the


hCG may be from another source other than pregnancy. Fatigue, amenorrhea, and
vomiting are presumptive or possible signs of pregnancy and can also have other
causes.
Question format: Multiple Choice
Chapter 6: Maternal Adaptation during Pregnancy
Cognitive Level: Apply
Client Needs: Health Promotion and Maintenance
Integrated Process: Nursing Process
Reference: p. 104

13. The nursing instructor is teaching a class on the various hormones necessary
for a successful pregnancy and birthing process. The instructor determines the
session is successful when the students correctly choose which hormone as being
necessary after birth to ensure growth of the newborn?
A. Oxytocin
B. Estrogen
C. Progesterone
D. Prolactin

Answer: D

Rationale: Prolactin is the hormone responsible for the initiation of lactation, the
production of breast milk. Oxytocin is responsible for the letdown of milk and
uterine contractions enabling the infant to be born, and estrogen and progesterone
are responsible for uterine and pregnancy maintenance.
Question format: Multiple Choice
Chapter 6: Maternal Adaptation during Pregnancy
Cognitive Level: Apply
Client Needs: Health Promotion and Maintenance
Integrated Process: Nursing Process
Reference: p. 107

14. The community nurse is preparing a presentation for a health fair illustrating
successful pregnancies. Which component should the nurse prioritize as the most
critical to ensure a positive psychological experience with the pregnancy by the
mother?
A. Early prenatal care
B. Age at the time of pregnancy
C. Having a planned pregnancy
D. Social support

Answer: D

Rationale: All options are correct and play a role pregnancy, but the most critical
for a positive psychological experience is for the woman to have a social support
system. Early care, maternal age, and planned pregnancy all affect fetal and
maternal health, but are not necessarily linked to positive psychological
experiences.
Question format: Multiple Choice
Chapter 6: Maternal Adaptation during Pregnancy
Cognitive Level: Apply
Client Needs: Health Promotion and Maintenance
Integrated Process: Nursing Process
Reference: p. 110

15. The nursing instructor is teaching a class on the nutritional needs of the
pregnant client. The instructor determines the session is successful when the
students correctly choose which supplement as being known to prevent up to 70%
of CNS birth defects?
A. iodine
B. zinc
C. folic acid
D. vitamin A

Answer: C

Rationale: Folic acid is noted to help prevent up to 70% of CNS birth defects;
however, the folic acid needs to be in the body prior to the pregnancy to be most
effective. Iodine affects thyroid development. Zinc is required for enzyme formation
and gene expression. Vitamin A helps develop vision.
Question format: Multiple Choice
Chapter 6: Maternal Adaptation during Pregnancy
Cognitive Level: Apply
Client Needs: Health Promotion and Maintenance
Integrated Process: Nursing Process
Reference: p. 114
16. A client in her third trimester reports sleeping poorly: sleeping on her back
results in lightheadedness and dizziness and lying on her side results in no sleep.
Which suggestion for sleeping should the nurse prioritize for this client?
A. without a pillow
B. with a pillow under her shoulders
C. with a pillow under her right hip
D. with a pillow under both hips

Answer: C

Rationale: Pregnancy places strain on the cardiovascular system with increased


fluid in the lungs and heart. The use of one pillow under the right hip will help
displace the uterus and fetus off the major blood vessels, allowing the circulation to
flow appropriately and provide relief to the client. When the woman lies flat on her
back the uterus and contents can compress the vena cava and aorta and reduce
blood flow, resulting in the light-headedness and dizzy spells. Removal of the pillow
would not alter the effects on the vena cava. A pillow under the shoulders would
hurt the neck, and a pillow under both hips would exacerbate the light-headedness.
Question format: Multiple Choice
Chapter 6: Maternal Adaptation during Pregnancy
Cognitive Level: Apply
Client Needs: Physiological Integrity: Basic Care and Comfort
Integrated Process: Teaching/Learning
Reference: p. 107

17. A client at 39 weeks' gestation calls the OB triage and questions the nurse
concerning a bloody mucus discharge noted in the toilet after an OB office visit
several hours earlier. What is the best response from the triage nurse?
A. "It might be nothing. If it happens again call your provider who is on-call."
B. "If the provider did an exam, it might be just normal vaginal secretions, so don't
worry about it."
C. "A one time discharge of bloody mucus in the toilet might have been your mucus
plug."
D. "Bloody mucus is a sign you are in labor. Please come to the hospital."

Answer: C

Rationale: Bloody mucus can either be a mucus plug or bloody show. The one-time
occurrence would be more likely to be the mucus plug. A bloody show would
continue if her cervix was changing, but this usually does not occur until after
contractions start. It is a sign that something is happening and should be reported
to the health care provider. The bloody mucus is not a sign of labor, but it can be
an early sign that labor is coming soon.
Question format: Multiple Choice
Chapter 6: Maternal Adaptation during Pregnancy
Cognitive Level: Apply
Client Needs: Health Promotion and Maintenance
Integrated Process: Nursing Process
Reference: p. 106

18. The nurse is assessing a pregnant client at 20 weeks' gestation and obtains a
hemoglobin level. Which result would be a cause for concern?
A. 12.8 g/dL
B. 11.9 g/dL
C. 11.2 g/dL
D. 10.6 g/dL

Answer: D

Rationale: The average hemoglobin level at term is 12.5 g/dl. The hemoglobin level
is considered normal until it falls below 11 g/dl.
Question format: Multiple Choice
Chapter 6: Maternal Adaptation during Pregnancy
Cognitive Level: Apply
Client Needs: Physiological Integrity: Reduction of Risk Potential
Integrated Process: Nursing Process
Reference: p. 107

19. A client at 40 weeks' gestation informs the nurse that she is tired of being
pregnant. What is the best response from the nurse?
A. "Do you need to speak with someone about your feelings?"
B. "That is a very normal feeling, especially at this point in pregnancy."
C. "Most woman would have asked to be induced by this point. Is that what you
want?"
D. "Are you getting enough rest? If you don't take time for rest, that is why you
might be tired."

Answer: B

Rationale: During the third trimester, the client is preparing for parenthood and is
often tired and ready for a break. The woman may feel large and unable to do any
normal activities, and may feel ready to have the baby in her arms rather than in
her uterus. This is not an abnormal statement, and the provider should not
overreact. Deciding to induce labor is something that should be done in consultation
with the health care provider and only when it is necessary for the health/safety of
the mother or baby.
Question format: Multiple Choice
Chapter 6: Maternal Adaptation during Pregnancy
Cognitive Level: Analyze
Client Needs: Psychosocial Integrity
Integrated Process: Caring
Reference: p. 112
20. The nurse is teaching a pregnant teenager the importance of proper nutrition
and adequate weight gain throughout the pregnancy. What is the best response
when the client refuses to eat due to fear of possible weight gain?
A. The infant will be small and could have problems.
B. There may be little impact on the infant, but the mother can suffer
complications.
C. It will just make the baby smaller, but there are no other problems associated.
D. The infant will be smaller but should quickly gain weight.

Answer: A

Rationale: Women who gain less than 16 pounds (7257 g) are at risk of giving birth
to small infants, which is associated with poor neonatal outcomes. The infant may
not quickly gain weight but continue to slowly put on weight.
Question format: Multiple Choice
Chapter 6: Maternal Adaptation during Pregnancy
Cognitive Level: Apply
Client Needs: Health Promotion and Maintenance
Integrated Process: Nursing Process
Reference: p. 112

21. The nurse cares for a pregnant client at the first prenatal visit and reviews
expected changes that will occur during pregnancy. Which information will the
nurse include in the education?
A. During pregnancy blood volume can increase by at least 40%.
B. Pregnancy typically causes a decrease in respiratory rate.
C. Hemoglobin levels rise significantly during pregnancy.
D. Blood pressure decreases in the third trimester.

Answer: A

Rationale: The pregnant woman can experience a blood volume increase by


approximately 40% to 50% above prepregnancy levels by the end of the third
trimester. Pregnancy results in an increased respiratory rate to provide oxygen to
both the mother and fetus. Hemoglobin levels are usually low during pregnancy
because of hemodilution of red blood cells, which is termed physiologic anemia of
pregnancy. Blood pressure usually reaches a low point mid-pregnancy and,
thereafter, increases to prepregnancy levels by the third trimester.
Question format: Multiple Choice
Chapter 6: Maternal Adaptation during Pregnancy
Cognitive Level: Understand
Client Needs: Health Promotion and Maintenance
Integrated Process: Teaching/Learning
Reference: p. 105

22. The nurse is conducting an annual examination on a young female who reports
her last menses was 2 months ago. The client insists she is not pregnant due to a
negative home pregnancy test. Which assessment should the nurse use to assess
confirm the pregnancy?
A. Chadwick sign
B. fetal heartbeat
C. positive urine human chorionic gonadotropin (hCG)
D. uterine size and shape changes

Answer: B

Rationale: The only positive sign of pregnancy is a sign or symptom that could only
be attributable to the fetus; thus, fetal heartbeat can have no other origin.
Chadwick sign is a color change in the cervix, vagina, and perineum; these could all
be the result of other causes. A positive urine hCG is a probable sign as it can be
related to causes other than pregnancy. A change in the size and shape of the
uterus can occur due to other causes.
Question format: Multiple Choice
Chapter 6: Maternal Adaptation during Pregnancy
Cognitive Level: Apply
Client Needs: Physiological Integrity: Reduction of Risk Potential
Integrated Process: Nursing Process
Reference: p. 104

23. The nurse is assessing a pregnant client at 12 weeks' gestation and the client
reports some new bumps on the dark part of her nipples. What is the best
response from the nurse when questioned by the client as to what they are?
A. Normal bumps of pregnancy; they do nothing
B. Might be sign of cancer; need to speak with health care provider
C. Montgomery glands (Montgomery tubercles); secrete lubricant for the nipples
D. Striae, stretching of the breast tissue

Answer: C

Rationale: All women have Montgomery glands (Montgomery tubercles). They


become more prominent during pregnancy and help to prepare the nipples for
breastfeeding. The bumps are not specific to pregnancy and are not a sign of
cancer. They are not the result of stretching.
Question format: Multiple Choice
Chapter 6: Maternal Adaptation during Pregnancy
Cognitive Level: Apply
Client Needs: Health Promotion and Maintenance
Integrated Process: Nursing Process
Reference: p. 106

24. A client arrives to the clinic very excited and reporting a positive home
pregnancy test. The nurse cautions that the home pregnancy test is considered a
probable sign and will assess the client for which sign to confirm pregnancy?
A. Positive office pregnancy test
B. Fetal movement felt by examiner
C. Hegar sign
D. Chadwick sign

Answer: B

Rationale: The positive signs of pregnancy are fetal image on a sonogram, hearing
a fetal heart rate, and examiner feeling fetal movement. A pregnancy test has 95%
accuracy; however, it may come back as a false positive. Hegar sign is a softening
of the uterine isthmus. Chadwick sign may have other causes besides pregnancy.
Question format: Multiple Choice
Chapter 6: Maternal Adaptation during Pregnancy
Cognitive Level: Apply
Client Needs: Health Promotion and Maintenance
Integrated Process: Nursing Process
Reference: p. 104

25. A client in her first trimester is concerned about how weight gain will affect her
appearance and questions the nurse concerning dietary restrictions. How much
weight gain should the nurse point out will be safe for this client with a low BMI?
A. 25 to 35 pounds (11 to 16 kilograms)
B. 28 to 40 pounds (13 to 18 kilograms)
C. 15 to 25 pounds (7 to 11 kilograms)
D. 16 to 30 pounds (7.25 to 14 kilograms)

Answer: B

Rationale: The recommendation for average weight gain is 25 to 35 lbs (11 to 16


kilograms). The woman who is underweight with a low BMI should gain 28 to 40
pounds (13 to 18 kilograms). Less than 28 pounds (13 kilograms) may hinder fetal
development, and weight gain over 40 pounds (18 kilograms) may be dangerous to
the mother. Individuals with a high BMI should gain 15 to 25 pounds (7 to 11
kilograms). A weight gain of less than 16 pounds (7.25 kilograms) may result in a
low-birth-weight infant and gains over 30 pounds (14 kilograms) may necessitate a
cesarean section.
Question format: Multiple Choice
Chapter 6: Maternal Adaptation during Pregnancy
Cognitive Level: Apply
Client Needs: Health Promotion and Maintenance
Integrated Process: Nursing Process
Reference: p. 113
Chapter 7
1. Charlene McCoy, who has several children already, reports for a first prenatal
visit. She seems preoccupied and withdrawn, and she makes consistently negative
remarks about the pregnancy. Reviewing her records, you note that she is receiving
a serotonin reuptake inhibitor. What should you do?
A. Reassure her that ambivalence is normal.
B. Refer her for drug and alcohol counseling.
C. Give her printed material to read at home.
D. Alert the RN or health care provider.

Answer: D

Rationale: A client on an SSRI or SRI (serotonin reuptake inhibitor) might be in


current treatment for a psychiatric disorder. The medication may also be one that is
not safe during pregnancy. The RN and the health care provider need to be alerted
to seek more information from the client. Reassurance is good practice, but not
enough in this case. You do not have enough information to refer her for drug and
alcohol counseling. She is under the care of another provider for her mental
disorder, so do not confuse her with more material to read.
Question format: Multiple Choice
Chapter 7: Prenatal Care
Cognitive Level: Apply
Client Needs: Safe, Effective Care Environment: Management of Care
Integrated Process: Nursing Process
Reference: p. 122

2. As part of the first prenatal visit, the nurse is assessing a pregnant woman's
obstetrical history, which includes an 18-month-old daughter, born 2 days after her
estimated date of birth; a 3-year-old son born at 35 weeks' gestation; and two lost
pregnancies, one at 12 weeks and one at 21 weeks. How should the nurse
document this history?
A. G5 T1 P2 A1 L2
B. G4 T1 P1 A2 L2
C. G5 T2 P2 A1 L1
D. G4 T1 P2 A2 L2

Answer: A

Rationale: The G represents the total number of pregnancies, which is 5. The T


represents term deliveries that ended at or beyond 38 weeks' gestation, which is 1.
The P refers to preterm deliveries (ended after 20 weeks and before end of 37
weeks), which is 2. The A refers to abortions or the number of pregnancies that
ended before 20 weeks' gestation, which is 1. The L refers to living children, which
is 2. Thus the nurse will document G5 T1 P2 A1 L2 for this client.
Question format: Multiple Choice
Chapter 7: Prenatal Care
Cognitive Level: Apply
Client Needs: Safe, Effective Care Environment: Management of Care
Integrated Process: Communication and Documentation
Reference: p. 122

3. A client who is uncertain when her LMP occurred is given an EDD of April 23 after
the first ultrasound. Based on this information, the nurse determines the client's
LMP was probably which day?
A. July 13
B. July 16
C. July 19
D. July 21

Answer: B

Rationale: According to Naegele rule, the last menstrual period was July 16th. Take
the LMP and add 7 days and subtract 3 months; if finding the LMP from the EDD,
subtract 7 days and add 3 months.
Question format: Multiple Choice
Chapter 7: Prenatal Care
Cognitive Level: Analyze
Client Needs: Health Promotion and Maintenance
Integrated Process: Nursing Process
Reference: p. 123

4. The nurse has just received the results of a pregnant client's MSAFP screening
and notes the levels are elevated. The nurse should prioritize which discussion
with the client?
A. risk for Down syndrome
B. risk for neural tube defects
C. test needs to be repeated
D. further testing is required

Answer: D

Rationale: The maternal serum alpha-fetoprotein (MSAFP) measures the levels of


alpha-fetoprotein, which is a protein manufactured by the fetus. The woman's blood
contains small amounts of this protein during pregnancy. The blood test is run
between 16 and 20 weeks' gestation; an abnormal level indicates a need for further
testing to determine the risks her fetus may face. Higher levels can indicate
multiple fetuses, death of the fetus, the presence of neural tube defects, and
possibly Down syndrome; however, further testing such as ultrasound or
amniocentesis is required to determine the exact cause of the elevation.
Question format: Multiple Choice
Chapter 7: Prenatal Care
Cognitive Level: Apply
Client Needs: Physiological Integrity: Reduction of Risk Potential
Integrated Process: Nursing Process
Reference: p. 127

5. A 38-year-old client and partner are carriers of the Tay-Sachs gene, have one
child with Tay-Sachs, and are concerned to learn she is pregnant again. The nurse
predicts the health care provider will order which test if the couple wants to know if
this baby will also be born with Tay-Sachs?
A. a multiple marker screening test
B. amniocentesis
C. chorionic villus sampling
D. percutaneous umbilical blood sampling

Answer: C

Rationale: Chorionic villus sampling (CVS) is a procedure that can provide


information on fetal chromosomal studies similar to an amniocentesis, but earlier in
pregnancy. The CVS is typically performed between 8 and 12 weeks gestation.
Multiple marker screen tests are done later in the pregnancy, as is amniocentesis.
Percutaneous umbilical blood sampling examines the blood and is not the best
source for chromosomal studies.
Question format: Multiple Choice
Chapter 7: Prenatal Care
Cognitive Level: Apply
Client Needs: Physiological Integrity: Reduction of Risk Potential
Integrated Process: Nursing Process
Reference: p. 128

6. The nurse is assessing a 24-year-old pregnant client who reports excessive


vaginal discharge that is messy and unpleasant but without a strong odor, itching,
or irritation present. Which response should the nurse prioritize?
A. Douche frequently with mild soap and water.
B. See her primary care provider.
C. Use sanitary pads.
D. Decrease her fluid intake.

Answer: C

Rationale: Vaginal discharge increases during pregnancy and is a concern for many
women. Encourage the client to keep clean and wear sanitary pads as needed.
Douching may be dangerous for the mother. STIs are not indicated simply by
discharge. Pregnant women should not decrease fluid intake.
Question format: Multiple Choice
Chapter 7: Prenatal Care
Cognitive Level: Apply
Client Needs: Health Promotion and Maintenance
Integrated Process: Nursing Process
Reference: p. 133
7. A client at 27 weeks' gestation still walks daily but reports "terrible" heartburn at
night. Which action should the nurse point out will best address this situation?
A. Stop or severely curtail her exercise.
B. Take sodium bicarbonate.
C. Seek emergency medical care.
D. Elevate the head of the bed.

Answer: D

Rationale: Heartburn is a common problem worsening as the pregnancy progresses.


The pregnancy hormones relax the lower esophageal sphincter, resulting in
increased heartburn. Elevation of the head of the bed will help prevent the acid
from refluxing. Exercise does not negatively impact heartburn and should be
continued. The pregnant mother should not take any medication that is not
prescribed by her primary care provider. Heartburn is not a medical emergency.
Question format: Multiple Choice
Chapter 7: Prenatal Care
Cognitive Level: Apply
Client Needs: Physiological Integrity: Basic Care and Comfort
Integrated Process: Teaching/Learning
Reference: p. 131

8. The health care provider has prescribed an over-the-counter antacid for a


pregnant client in her first trimester who is having ongoing nausea, vomiting, and
heartburn. Which instruction concerning the antacid should the nurse prioritize
after noting the client is also prescribed a multivitamin supplement?
A. Avoid caffeinated beverages.
B. Take only at bedtime.
C. Take antacid 1 hour after the multivitamin.
D. Take with dairy products.

Answer: C

Rationale: Antacids interfere with the uptake of the vitamin contents so the client
should take the antacid 1 hour after taking the multivitamin. Caffeine should be
avoided due to increases in blood pressure and diuretic effects. Antacids can be
taken more often than solely at bedtime, and some clients need them after each
meal. Antacids do not have to be taken with dairy products. The priority is to avoid
allowing the antacid to cancel out the multivitamin.
Question format: Multiple Choice
Chapter 7: Prenatal Care
Cognitive Level: Apply
Client Needs: Physiological Integrity: Pharmacological and Parenteral Therapies
Integrated Process: Teaching/Learning
Reference: p. 131
9. The nurse advises a pregnant client to keep a small high-carbohydrate, low-fat
snack at the bedside. The nurse should point out this will assist with which
condition?
A. heartburn
B. faintness
C. slowed GI transit time
D. nausea and vomiting

Answer: D

Rationale: Women will commonly experience nausea and vomiting upon awakening
first thing in the morning. Clients who experience this should be encouraged to
have small snacks at their bedside for eating prior to moving from the bed.
Heartburn is a result of pressure and hormone action. Faintness is due to pressure
on the vena cava, not blood sugar. GI transit time is not affected.
Question format: Multiple Choice
Chapter 7: Prenatal Care
Cognitive Level: Apply
Client Needs: Health Promotion and Maintenance
Integrated Process: Teaching/Learning
Reference: p. 131

10. The nurse is teaching a pregnant client some nonpharmacologic ways to handle
common situations encountered during pregnancy. The nurse determines the
session is successful when the client correctly chooses which condition that can be
minimized if she avoids drinking fluids with her meals?
A. nosebleeds
B. heartburn
C. blood clots
D. constipation

Answer: B

Rationale: Filling the stomach with heavy food and fluid can cause overfill and place
pressure on the stomach, increasing gastric reflux. Avoid excess fluids with meals
and eat small frequent meals to avoid heartburn. Nosebleeds result from increased
estrogen. Blood clots can result from sitting still for too long. Constipation can
result from increased progesterone.
Question format: Multiple Choice
Chapter 7: Prenatal Care
Cognitive Level: Apply
Client Needs: Health Promotion and Maintenance
Integrated Process: Teaching/Learning
Reference: p. 131

11. The nurse is meeting with a client at 28 weeks' gestation. To prepare her for
the final trimester, which factor should the nurse prioritize in the teaching
session?
A. preventing anemia
B. decreasing shortness of breath
C. decreasing bleeding gums
D. preventing varicosities

Answer: B

Rationale: As the fetus grows inside the mother, there is more pressure on the
diaphragm, more difficulty breathing, and episodes of dyspnea may occur. This
tends to decrease with "lightening," when the fetus drops. Preventing anemia,
decreasing bleeding gums, and preventing varicosities are situations that should be
addressed throughout the entire pregnancy.
Question format: Multiple Choice
Chapter 7: Prenatal Care
Cognitive Level: Apply
Client Needs: Health Promotion and Maintenance
Integrated Process: Teaching/Learning
Reference: p. 131

12. A pregnant client reports difficulty sleeping well. Which suggestion for sleeping
should the nurse prioritize to assist this client?
A. on her stomach with a pillow under her breasts
B. on her side with the weight of the uterus on the bed
C. on her back with a pillow under her knees and hips
D. on her back with a pillow under her head

Answer: B

Rationale: Resting on the side prevents pressure from the uterus against the vena
cava and therefore allows blood to return to the uterus. Other positions may be
more uncomfortable or may exacerbate the problems associated with pressure on
the vena cava.
Question format: Multiple Choice
Chapter 7: Prenatal Care
Cognitive Level: Apply
Client Needs: Health Promotion and Maintenance
Integrated Process: Teaching/Learning
Reference: p. 131

13. A pregnant client is planning a vacation to a different state and questions the
nurse concerning precautions. Which suggestion should the nurse prioritize for this
client who will be traveling by automobile?
A. Travel no more than 120 miles daily.
B. Sit in the back seat with feet elevated.
C. Stop and walk every 2 hours.
D. Limit trips away from home, greater than 200 miles.

Answer: C
Rationale: Walking increases venous return and reduces the possibility of
thrombophlebitis, a risk for pregnant women who sit for extended periods of time.
Limiting mileage, sitting in the back with the feet elevated, and limiting trips may
help, but they are not enough to prevent phlebitis.
Question format: Multiple Choice
Chapter 7: Prenatal Care
Cognitive Level: Apply
Client Needs: Health Promotion and Maintenance
Integrated Process: Teaching/Learning
Reference: p. 137

14. The nurse is assessing a client at 30 weeks' gestation who reports increased
constipation. Which suggestion should the nurse prioritize for this client?
A. taking mineral oil
B. increasing fluid intake
C. reducing iron supplement
D. increasing intake of meat

Answer: B

Rationale: Increasing fluid content by drinking at least 8 glasses of noncaffeinated


beverages helps relieve constipation in both pregnant and nonpregnant women.
Reducing an iron supplement could lead to anemia; mineral oil can reduce
absorption of fat-soluble vitamins. The client should add foods rich in fiber, which
would include grains, vegetables, and fruits (instead of meat).
Question format: Multiple Choice
Chapter 7: Prenatal Care
Cognitive Level: Apply
Client Needs: Health Promotion and Maintenance
Integrated Process: Nursing Process
Reference: p. 132

15. The nurse is assessing a client at 12 weeks' gestation who reports enjoying her
usual slow, long daily walk. The nurse should point out which recommendation to
this client?
A. Reduce walking to half a block daily.
B. Continue this as long as she enjoys it.
C. Stop and rest every block.
D. Engage in aerobics for greater benefits.

Answer: B

Rationale: Walking is an excellent exercise during pregnancy because it is low


impact and increases venous circulation. Exercise should be maintained as long as
it is comfortable, but intensity should not increase over what is normally performed.
Question format: Multiple Choice
Chapter 7: Prenatal Care
Cognitive Level: Apply
Client Needs: Health Promotion and Maintenance
Integrated Process: Teaching/Learning
Reference: p. 135

16. The nurse is assessing a client at her first prenatal visit and notes that she is
exposed to various chemicals at her place of employment. Which statement by the
client would indicate she needs additional health education to protect her and her
fetus?
A. "I only work four hours a day so I don't get exposed too much."
B. "The gloves they provide irritate my hands, so I don't use them."
C. "There hasn't been a chemical spill in three years."
D. "I have an assistant helping me now to handle the chemicals."

Answer: B

Rationale: There are various chemicals which are recognized for their teratogenic
effects and must be avoided during pregnancy. The nurse should find out which
chemicals the client is exposed to and determine the risk factor. The greatest
danger is the client handling chemicals without a barrier protection such as gloves.
The other issues may also be dangers depending on the chemicals and the
environment in which the client is working and should also be evaluated.
Question format: Multiple Choice
Chapter 7: Prenatal Care
Cognitive Level: Apply
Client Needs: Health Promotion and Maintenance
Integrated Process: Nursing Process
Reference: p. 136

17. The nurse is assisting a pregnant client who underwent a nonstress test that
was ruled reactive. Which factor will the nurse point out when questioned by the
client about the results?
A. There is no evidence of congenital anomalies or deformities.
B. The fetal heart rate increases with activity and indicates fetal well-being.
C. The fetus is developing at a fast rate but doing fine.
D. The results indicate a stress test is needed for further evaluation.

Answer: B

Rationale: A nonstress test is a noninvasive way to monitor fetal well-being. A


reactive NST is a positive sign the fetus is tolerating pregnancy well by
demonstrating heart rate increase with activity, and this indicates fetal well-being.
This test is not used to determine congenital anomalies or deformities. It does not
determine the speed by which fetus is developing. Further evaluation would be
necessary if the results were nonreactive.
Question format: Multiple Choice
Chapter 7: Prenatal Care
Cognitive Level: Analyze
Client Needs: Health Promotion and Maintenance
Integrated Process: Nursing Process
Reference: p. 129

18. A 31-year-old client at 28 weeks' gestation reports frequent low back pain and
ankle edema by the end of the day. Which suggestion should the nurse prioritize
for this client?
A. Soak feet every night and perform pelvic rocks.
B. Lie on right side with feet elevated and a heating pad on the back.
C. Take breaks at work and sit in a semi-Fowler position with feet below.
D. Rest when possible with feet elevated at or above the heart.

Answer: D

Rationale: Resting in the recumbent position helps alleviate stress on the back, and
elevating the legs will help relieve the edema. Soaking the feet or lying on the right
side will not alleviate the edema. Sitting semi-Fowler is not enough to alleviate the
edema.
Question format: Multiple Choice
Chapter 7: Prenatal Care
Cognitive Level: Apply
Client Needs: Physiological Integrity: Basic Care and Comfort
Integrated Process: Nursing Process
Reference: p. 131-132

19. A 25-year-old client at 27 weeks' gestation reports waking up with leg cramps.
Which suggestion should the nurse point out to the client to help relieve this
discomfort?
A. Use plantar flexion exercises three times every day.
B. Dorsiflex the foot while extending her leg during the cramp.
C. Encourage her to drink more fluids, 10 glasses a day.
D. Avoid any supplementation of vitamins or minerals.

Answer: B

Rationale: Plantar flexion can make cramps worse, so dorsiflexion while extending
the leg can relieve the cramp; excess fluid and lack of supplementation with
vitamins or minerals may worsen cramps. Performing plantar flexion exercise does
not prevent the cramp. Increasing fluids may help, but has never proven to
eliminate cramping.
Question format: Multiple Choice
Chapter 7: Prenatal Care
Cognitive Level: Apply
Client Needs: Physiological Integrity: Basic Care and Comfort
Integrated Process: Nursing Process
Reference: p. 131-132
20. The nurse is preparing to administer a prescribed medication to the pregnant
client. Which order should the nurse question?
A. penicillin
B. rubella
C. acetaminophen
D. folic acid

Answer: B

Rationale: Most vaccines are contraindicated during pregnancy and are considered
teratogenic, such as rubella. Penicillin and acetaminophen may be taken under
provider supervision. Folic acid supplementation should be encouraged.
Question format: Multiple Choice
Chapter 7: Prenatal Care
Cognitive Level: Apply
Client Needs: Physiological Integrity: Pharmacological and Parenteral Therapies
Integrated Process: Nursing Process
Reference: p. 123

21. The nurse is assessing a client at her first prenatal visit and reports her LMP
started December 1. Which date will the nurse predict for the EDD?
A. October 7
B. September 8
C. July 7
D. August 8

Answer: B

Rationale: According to Naegele rule, the estimated date of birth is September 8.


Add 7 days and subtract 3 months to the LMP to determine the estimated date of
birth.
Question format: Multiple Choice
Chapter 7: Prenatal Care
Cognitive Level: Analyze
Client Needs: Health Promotion and Maintenance
Integrated Process: Nursing Process
Reference: p. 123

22. A young couple are very excited to discover they are pregnant and ask the
nurse when to expect the baby. Based on a July 20 LMP, which day will the nurse
predict for delivery?
A. April 27
B. March 13
C. April 13
D. May 20

Answer: A
Rationale: Naegele rule is to subtract 3 months and add 7 days from the first day of
the last menstrual period to determine an expected due date, making the client's
due date April 27.
Question format: Multiple Choice
Chapter 7: Prenatal Care
Cognitive Level: Analyze
Client Needs: Health Promotion and Maintenance
Integrated Process: Nursing Process
Reference: p. 123

23. The nurse will be assisting a client during an amniocentesis. Which nursing
intervention should the nurse prioritize?
A. Caution about the opioid premedication.
B. Be certain she is aware of potential complications.
C. Ensure she understands the need for 2 days of bed rest.
D. Expect test results within 1 week.

Answer: B

Rationale: The client should be aware of the potential complications and risks, and
should sign an informed consent. Opioids are contraindicated for pregnant woman
due to side effects. She should maintain bed rest for the remainder of the day, with
light housework the following day and a return to normal activities on the third day.
It may take 2 or 3 weeks before the test results come back from the laboratory.
Question format: Multiple Choice
Chapter 7: Prenatal Care
Cognitive Level: Apply
Client Needs: Physiological Integrity: Reduction of Risk Potential
Integrated Process: Nursing Process
Reference: p. 128

24. The nurse is assessing a client at her first prenatal visit and notes the fundal
height is palpable at the level of the umbilicus. The nurse predicts the client is at
which gestational age?
A. 18 weeks
B. 20 weeks
C. 24 weeks
D. 22 weeks

Answer: B

Rationale: Some clients will not seek early prenatal care, especially if it is not their
first pregnancy. The uterus expands to reach the height of the umbilicus by week
20. Before week 20 it is too low to be palpated, and after week 20 it may be
beyond the umbilicus.
Question format: Multiple Choice
Chapter 7: Prenatal Care
Cognitive Level: Apply
Client Needs: Health Promotion and Maintenance
Integrated Process: Nursing Process
Reference: p. 121

25. A pregnant client arrives for her first prenatal appointment. She reports her
previous pregnancy ended at 19 weeks, and she has 3-year-old twins born at 30
weeks' gestation. How will the nurse document this in her records?
A. G2 T2 P1 A0 L2
B. G2 T1 P1 A1 L1
C. G3 T0 P1 A1 L2
D. G3 T2 P2 A0 L1

Answer: C

Rationale: G indicates the total number of pregnancies (2 prior, now pregnant = 3);
T indicates term deliveries at or beyond 38 weeks' gestation (none = 0); P is for
preterm deliveries (at 20 to 37 weeks = 1; multiple fetus delivery are scored as 1);
A is for abortions or pregnancies ending before 20 weeks' gestation (1); and L
refers to living children which is 2. Thus, G3 T0 P1 A1 L2 is what the nurse should
note in the client's record.
Question format: Multiple Choice
Chapter 7: Prenatal Care
Cognitive Level: Analyze
Client Needs: Safe, Effective Care Environment: Management of Care
Integrated Process: Communication and Documentation
Reference: p. 122

26. The nurse takes a call from a worried client who was seen several hours earlier
for her 35-weeks' gestation visit, which included a pelvic examination. Which
instruction should the nurse prioritize if the client is reporting a small amount of
vaginal spotting?
A. Return right away.
B. Watch it and report if heavy increase in bleeding.
C. The bleeding, called Chadwick sign, is a normal part of pregnancy.
D. The cervical mucus plug may have been expelled.

Answer: B

Rationale: During the third trimester, if the provider completes a vaginal exam it
can be normal to have a small amount of spotting. If the bleeding becomes active
or increases, the client needs to be seen immediately. Chadwick sign is a change of
color in the vaginal area. The loss of the mucus plug would lead to a much greater
amount of blood.
Question format: Multiple Choice
Chapter 7: Prenatal Care
Cognitive Level: Apply
Client Needs: Physiological Integrity: Reduction of Risk Potential
Integrated Process: Teaching/Learning
Reference: p. 125

27. The nurse discovers a new prescription for Rho(D) immune globulin for a client
who is about to undergo a diagnostic procedure. The nurse will administer the
Rho(D) immune globulin after which procedure?
A. contraction stress test
B. amniocentesis
C. nonstress test
D. biophysical profile

Answer: B

Rationale: Amniocentesis is an invasive procedure whereby a needle is inserted into


the amniotic sac to obtain a small amount of fluid. This places the pregnancy at risk
for a woman with Rh(D)-negative blood, since the puncture can allow the seepage
of blood and amniotic fluid into the woman's system. She should receive Rho(D)
immune globulin after the procedure to protect her and future babies. The CST,
NST, and a biophysical profile are noninvasive tests.
Question format: Multiple Choice
Chapter 7: Prenatal Care
Cognitive Level: Apply
Client Needs: Physiological Integrity: Pharmacological and Parenteral Therapies
Integrated Process: Nursing Process
Reference: p. 128
Chapter 8
1. A 28-year-old primigravida client presents to the unit in early labor. The record
reveals the client is 5 ft (1.5 m) tall, 95 lb (43 kg), and has gained 25 lb (11.3 kg)
over a normal, uneventful pregnancy. The nurse predicts this client will have which
type of pelvis upon assessment?
A. gynecoid
B. platypelloid
C. android
D. cannot be determined

Answer: D

Rationale: Pelvis shape cannot be determined by the information included in the


statement. The gynecoid is the most common; however, there is no way to predict
it. Early in the pregnancy, particularly if a woman has never given birth to a baby
vaginally, the practitioner may take pelvic measurements to estimate the size of
the true pelvis. This helps to determine if the size is adequate for vaginal birth.
However, these measurements do not consistently predict which women will have
difficulty giving birth vaginally, so most practitioners allow the woman to labor and
attempt a vaginal birth.
Question format: Multiple Choice
Chapter 8: The Labor Process
Cognitive Level: Apply
Client Needs: Health Promotion and Maintenance
Integrated Process: Nursing Process
Reference: p. 151

2. The nursing instructor is teaching a session on the birth process. During which
stage does the woman's cardiac output increase 80% above the pre-labor level?
A. first stage
B. pushing
C. immediately after birth
D. transition stage

Answer: C

Rationale: Due to an increased demand for oxygen the cardiac output increases up
to 80% immediately after birth. During the first stage of labor there is a moderate
increase in the demand for oxygen. While pushing, cardiac output can increase by
40% to 50%. During transition, changes are more psychological than physiologic.
Question format: Multiple Choice
Chapter 8: The Labor Process
Cognitive Level: Apply
Client Needs: Health Promotion and Maintenance
Integrated Process: Nursing Process
Reference: p. 161-162

3. The 29-year-old client presents at 5:30 a.m. with labor pains. Her history reveals
G4, three previous vaginal births, and gynecoid pelvis. At 9 a.m. her assessment
reveals 80% effaced and dilated at 3 cm. What nourishment can the nurse provide
if the client mentions she hasn't eaten since 5 p.m. yesterday and is hungry?
A. solid food and fluids
B. nothing except for intravenous fluids
C. clear liquids but no solid food
D. cannot assess with the information given

Answer: C

Rationale: The nurse should offer clear liquids but no solid foods. She is moving
closer to active labor nearing the end of the latent phase. It would not be advisable
to offer her solid foods, but she needs to continue her nourishment with fluids to
her thirst. Solid foods may lead to nausea and vomiting. Intravenous fluids are too
extreme as long as she is able to drink.
Question format: Multiple Choice
Chapter 8: The Labor Process
Cognitive Level: Apply
Client Needs: Physiological Integrity: Basic Care and Comfort
Integrated Process: Nursing Process
Reference: p. 161-162

4. The nurse is preparing to teach a group of soon-to-be new parents about the
labor process. When detailing the differences between the various presentations,
which one should the nurse point out seldom happens?
A. Breech
B. Shoulder
C. Oblique lie
D. Transverse lie

Answer: B

Rationale: Shoulder presentations are the least likely to occur. They occur in less
than 0.3% of all births. Approximately 97% of fetuses are in a cephalic presentation
at the end of pregnancy. A longitudinal lie, in which the long axis of the fetus is
parallel to the long axis of the mother, is the most common. When the fetus is in a
transverse lie, the long axis of the fetus is perpendicular to the long axis of the
woman. An oblique lie is in between the two.
Question format: Multiple Choice
Chapter 8: The Labor Process
Cognitive Level: Apply
Client Needs: Health Promotion and Maintenance
Integrated Process: Teaching/Learning
Reference: p. 153
5. A 33-year-old client has been progressing slowly through an unusually long
labor. The nurse assesses the fetal scalp pH and determines it is 7.26. How should
the nurse explain this result to the client when asked what it means?
A. Reassuring; it is associated with normal acid-base balance.
B. Worrisome; it may be associated with metabolic acidosis.
C. Critical; it represents metabolic acidosis.
D. Damaging; it is frequently associated with fetal neurological damage.

Answer: A

Rationale: The fetal pH slowly decreases during labor as a result of the normal
stress of labor. Although 7.26 is low for an adult, it is not problematic during labor
for an emerging fetus.
Question format: Multiple Choice
Chapter 8: The Labor Process
Cognitive Level: Apply
Client Needs: Physiological Integrity: Reduction of Risk Potential
Integrated Process: Nursing Process
Reference: p. 162

6. A 24-year-old primigravida client at 39 weeks' gestation presents to the OB unit


concerned she is in labor. Which assessment findings will lead the nurse to
determine the client is in true labor?
A. The contraction pains are 2 minutes apart and 1 minute in duration.
B. The client reports back pain, and the cervix is effacing and dilating.
C. The contraction pains have been present for 5 hours, and the patterns are
regular.
D. After walking for an hour, the contractions have not fully subsided.

Answer: B

Rationale: True labor is indicated when the cervix is changing. Contractions occur
for weeks before true labor, and may occur close together. Contractions may also
occur for a long time before true labor begins.
Question format: Multiple Choice
Chapter 8: The Labor Process
Cognitive Level: Apply
Client Needs: Health Promotion and Maintenance
Integrated Process: Nursing Process
Reference: p. 158

7. The nurse is monitoring a client who is in active labor. The nurse will carefully
monitor which phase of the involuntary uterine contraction to ensure the fetus is
progressing adequately?
A. Increment
B. Acme
C. Decrement
D. Relaxation
Answer: D

Rationale: The relaxation phase of uterine contractions is the time in which the
fetus has a break. This time needs to be observed, and it is beneficial for the fetus
to have a break. The three phases of a uterine contraction are the increment
(building up in intensity), acme (peak intensity), and decrement (decreasing
intensity). These phases are followed by a relaxation phase.
Question format: Multiple Choice
Chapter 8: The Labor Process
Cognitive Level: Analyze
Client Needs: Health Promotion and Maintenance
Integrated Process: Nursing Process
Reference: p. 162

8. A primigravida client at 39 weeks' gestation calls the OB unit questioning the


nurse about being in labor. Which response should the nurse prioritize?
A. Tell the woman to stay home until her membranes rupture.
B. Emphasize that food and fluid should stop or be light.
C. Ask the woman to describe why she believes that she is in labor.
D. Arrange for the woman to come to the hospital for labor evaluation.

Answer: C

Rationale: The nurse needs further information to assist in determining if the


woman is in true or false labor. The nurse will need to ask the client questions to
seek further assessment and triage information. Having the client wait until
membranes rupture may be dangerous, as she may give birth before reaching the
hospital. The client should continue fluid intake until it is determined whether or not
she is in labor. The client may be in false labor, and more information should be
obtained before she is brought to the hospital.
Question format: Multiple Choice
Chapter 8: The Labor Process
Cognitive Level: Apply
Client Needs: Physiological Integrity: Reduction of Risk Potential
Integrated Process: Nursing Process
Reference: p. 157

9. A pregnant client arrives to the clinic for a prenatal visit appearing


uncomfortable. During the assessment, the nurse determines the client is
experiencing fairly strong contractions at 12:05 p.m., 12:10 p.m., 12:15 p.m., and
12:20 p.m. What can the nurse conclude from these findings?
A. The client is in active labor.
B. The duration of the contractions is every 5 minutes.
C. The frequency of the contractions is every 5 minutes.
D. The client can be sent home.

Answer: C
Rationale: Based on the information, the nurse knows the contractions are regular
and every 5 minutes apart. This is the only data gathered based on the information
given, but it is very useful to the provider. A change in the cervix is necessary for
active labor. This client will need further assessment to determine whether the
client can go home or should be prepared for active labor. There is no information
providing the duration of the contractions.
Question format: Multiple Choice
Chapter 8: The Labor Process
Cognitive Level: Analyze
Client Needs: Physiological Integrity: Reduction of Risk Potential
Integrated Process: Nursing Process
Reference: p. 161

10. The nurse is teaching a prenatal class on the difference between true and false
labor contractions. The nurse determines the session is successful when the class
correctly chooses which factor as an indication of true labor contraction?
A. increase even if relaxing and taking a shower
B. remain irregular with the same intensity
C. subside when walking around and use the lateral position
D. cause discomfort over the top of uterus

Answer: A

Rationale: True labor contractions do not stop; they continue and strengthen, as
well as increase in frequency. If the contractions subside while taking a shower or
relaxing, then they are not labor contractions. The discomfort over the top of the
uterus is normal for full term pregnancy.
Question format: Multiple Choice
Chapter 8: The Labor Process
Cognitive Level: Analyze
Client Needs: Health Promotion and Maintenance
Integrated Process: Teaching/Learning
Reference: p. 158

11. The client in active labor overhears the nurse state the fetus is ROA. The nurse
should explain this refers to which component when the client becomes concerned?
A. Fetal station
B. Fetal attitude
C. Fetal position
D. Fetal size

Answer: C

Rationale: When documenting the ROA, this is the right occiput anterior or the
relationship of the fetal position to the mother using the maternal pelvis as the
point of reference. Fetal station refers to the relationship of the presenting part of
the fetus to the ischial spines of the pelvis. Fetal attitude refers to the relationship
of the fetal parts to one another. Fetal size refers the actual size of the developing
fetus.
Question format: Multiple Choice
Chapter 8: The Labor Process
Cognitive Level: Apply
Client Needs: Health Promotion and Maintenance
Integrated Process: Nursing Process
Reference: p. 155

12. A primigravida client at 38 weeks' gestation calls the clinic and reports, "My
baby is lower and it is more difficult to walk." Which response should the nurse
prioritize?
A. "The baby has dropped into the pelvis; your body and baby are getting ready for
labor in the next few weeks."
B. "This is not normal unless you are in active labor; come to the hospital and be
checked."
C. "That is something we expect with a second or third baby, but because it is your
first, you need to be checked."
D. "The baby moved down into the pelvis; this means you will be in labor within 24
hours, so wait for contractions then come to the hospital."

Answer: A

Rationale: The baby can drop into the pelvis, an event termed lightening, and can
happen for up to 2 weeks before the woman goes into labor. This is normal and
does not require intervention.
Question format: Multiple Choice
Chapter 8: The Labor Process
Cognitive Level: Apply
Client Needs: Health Promotion and Maintenance
Integrated Process: Caring
Reference: p. 156-157

13. The new parents are spending time with their newborn. However, they are
concerned with the edema and ecchymosis on the baby's scalp. How should the
nurse explain this to the parents after noting the baby was ROA in labor?
A. The infant needs to be assessed by the health care provider.
B. Ecchymosis indicates a blood disorder and the infant will need testing.
C. Ecchymosis with edema on the scalp is where the infant was pushed out of the
canal.
D. Edema is swelling and caused by unusual trauma; the provider must have used
forceps.

Answer: C

Rationale: Ecchymosis and edema are normal findings when located on the part
that was presenting in labor. The infant was ROA, so the head was the presenting
part and has normal bruising and swelling. The effects of birth are normal, and are
not of concern. Communicate to the parents that it is similar to a bruise or a blister.
This sign is not indicative of the use of forceps.
Question format: Multiple Choice
Chapter 8: The Labor Process
Cognitive Level: Apply
Client Needs: Physiological Integrity: Physiological Adaptation
Integrated Process: Nursing Process
Reference: p. 162

14. The nurse determines a client is 7 cm dilated. What is the best response when
asked by the client's partner how long will she be in labor?
A. "She is in active labor; she is progressing at this point and we will keep you
posted."
B. "She is in the transition phase of labor, and it will be within 2 to 3 hours, though
it might be sooner."
C. "She is still in early latent labor and has much too long to go to tell when she will
give birth."
D. "She is doing well and is in the second stage; it could be anytime now."

Answer: A

Rationale: At 7 cm dilated, she is considered in the active phase of labor. There is


no science that can predict the length of labor. She is progressing in labor, and it is
best not to give the family a specific time frame.
Question format: Multiple Choice
Chapter 8: The Labor Process
Cognitive Level: Apply
Client Needs: Health Promotion and Maintenance
Integrated Process: Communication and Documentation
Reference: p. 161

15. When documenting the fetus is at "zero station", the nurse knows this is where
in relation to the pelvic structure?
A. Pelvic inlet
B. Pelvic outlet
C. Ischial spines
D. Pelvic crest

Answer: C

Rationale: Zero station is the engagement of the fetus at the level of the ischial
spines of the pelvis. The ischial spines are a landmark that is used mark the
passage of the fetus. The pelvic crest is a landmark location on the pelvis for
documenting fetal station. The pelvic inlet must be shaped accordingly to allow for
passage of the fetus. The pelvic outlet is associated with internal rotation of the
fetal head.
Question format: Multiple Choice
Chapter 8: The Labor Process
Cognitive Level: Understand
Client Needs: Safe, Effective Care Environment: Management of Care
Integrated Process: Communication and Documentation
Reference: p. 154

16. A 37-year-old primigravida client at 40 weeks' gestation is seen in the clinic for
a scheduled prenatal visit. What report by the client would lead the nurse to predict
the woman is close to labor?
A. Nesting
B. Dilation
C. Effacement
D. Ripening of the cervix

Answer: A

Rationale: Nesting is the activity or burst of energy women often experience prior
to the onset of labor. The client could express feeling energetic or through a report
of her activities at home. Dilation, effacement, and cervical ripening are all
observed by the provider during assessment.
Question format: Multiple Choice
Chapter 8: The Labor Process
Cognitive Level: Apply
Client Needs: Health Promotion and Maintenance
Integrated Process: Nursing Process
Reference: p. 156-157

17. The community health nurse is conducting a presentation on labor and delivery.
When illustrating the birth process, the nurse should point out "0 station" refers to
which sign?
A. "This is just a way of determining your progress in labor."
B. "This indicates that you start labor within the next 24 hours."
C. "This means +1 and the baby is entering the true pelvis."
D. "The presenting part is at the true pelvis and is engaged."

Answer: D

Rationale: 0 station is when the fetus is engaged in the pelvis, or has dropped. This
is an encouraging sign for the client. This sign is indicative that labor may be
beginning, but there is no set time frame regarding when it will start. Labor has not
started yet, and the fetus has not begun to move out of the uterus.
Question format: Multiple Choice
Chapter 8: The Labor Process
Cognitive Level: Apply
Client Needs: Health Promotion and Maintenance
Integrated Process: Teaching/Learning
Reference: p. 154
18. The nurse has been monitoring a multipara client for several hours. She cries
out that her contractions are getting harder and that she cannot do this. The nurse
notes the client is very irritable, nauseated, annoyed, and doesn't want to be left
alone. Based on the assessment the nurse predicts the cervix to be dilated how
many centimeters?
A. 0 to 2
B. 5 to 7
C. 3 to 4
D. 8 to 10

Answer: D

Rationale: The reaction of the client is indicative of entering or being in the


transition phase of labor, stage 1. The dilation (dilatation) would be 8 cm to 10 cm.
Before that, when dilation is 0 to 7 cm, the client has an easier time using positive
coping skills.
Question format: Multiple Choice
Chapter 8: The Labor Process
Cognitive Level: Analyze
Client Needs: Health Promotion and Maintenance
Integrated Process: Nursing Process
Reference: p. 160

19. A group of nursing students are preparing a presentation that will illustrate
various components of the birthing process. When discussing the pelvis, the
students should point out that the pelvis is often referred to as which term?
A. passenger
B. passageway
C. powers
D. psyche

Answer: B

Rationale: The passageway is one of the 4 Ps and involves the pelvis, both bony
pelvis and the soft tissues, cervix, and vagina. The passenger refers to the fetus.
The primary powers are the involuntary contractions of the uterus, whereas the
secondary powers come from the maternal abdominal muscles. The psyche refers
to the mother's mental state.
Question format: Multiple Choice
Chapter 8: The Labor Process
Cognitive Level: Understand
Client Needs: Health Promotion and Maintenance
Integrated Process: Teaching/Learning
Reference: p. 151

20. A nulliparous client at 37 weeks' gestation calls the labor and delivery unit
stating she thinks she is in labor. The nurse predicts she is in true labor based on
which answer to her assessment questions?
A. contractions, irregular, lasting 15 to 20 seconds
B. bloody mucus in the toilet once earlier in the day
C. contraction, regular and lasting longer and stronger
D. scant amount of thick, white vaginal discharge, no odor

Answer: C

Rationale: True labor contraction will progressively get worse and last longer. The
pain will come to a point where the woman will not be able to walk or talk through
the contractions. Irregular contractions, bloody show, and white vaginal discharge
are normal for pregnancy but do not indicate true labor.
Question format: Multiple Choice
Chapter 8: The Labor Process
Cognitive Level: Apply
Client Needs: Physiological Integrity: Reduction of Risk Potential
Integrated Process: Nursing Process
Reference: p. 158

21. The nurse is assessing a pregnant client at 37 weeks' gestation and notes the
fetus is at 0 station. When questioned by the client as to what has happened, the
nurse should point out which event has occurred?
A. flexion
B. engagement
C. extension
D. expulsion

Answer: B

Rationale: The movement of the fetus into the pelvis from the upper uterus is
engagement. This is the first cardinal movement of the fetus in preparation for the
spontaneous vaginal delivery. Flexion occurs as the fetus encounters resistance
from the soft tissues and muscles of the pelvic floor. Extension is the state in which
the fetal head is well flexed with the chin on the chest as the fetus travels through
the birth canal. Expulsion occurs after emergence of the anterior and posterior
shoulders.
Question format: Multiple Choice
Chapter 8: The Labor Process
Cognitive Level: Apply
Client Needs: Health Promotion and Maintenance
Integrated Process: Nursing Process
Reference: p. 157-158

22. The nurse is monitoring a client who is in labor and notes the client is happy,
cheerful, and "ready to see the baby." The nurse interprets this to mean the client
is in which stage or phase of labor?
A. transition phase
B. stage two
C. latent phase
D. stage three

Answer: C

Rationale: The woman in labor undergoes numerous psychological adaptations


during labor. During the latent phase, she is often talkative and happy, and yet
anxious. During transition, the client may show fear and anger. During stage 2 she
may remain positive, but the work of labor is very intense.
Question format: Multiple Choice
Chapter 8: The Labor Process
Cognitive Level: Analyze
Client Needs: Health Promotion and Maintenance
Integrated Process: Nursing Process
Reference: p. 158

23. The nurse is appraising the post-birth laboratory results of a client and
discovers the WBC is 22,000 cells/μL (22 x 109/L). Which action should be
prioritized in response?
A. none, a normal variation due to labor
B. an abnormal finding, needs antibiotics
C. occurs in clients after a cesarean birth
D. further testing is required to determine source.

Answer: A

Rationale: An elevation of WBC up to 30,000 cells/μL (30 x 109/L) can be a normal


variation for any woman after birth. This is related to the stress on her body from
labor and birth. Antibiotics are not indicated as this is a normal response to intense
stress. The increase in WBC is not related to cesarean birth. Further testing would
be wasteful as it is known that this is a normal response to any stress.
Question format: Multiple Choice
Chapter 8: The Labor Process
Cognitive Level: Apply
Client Needs: Physiological Integrity: Reduction of Risk Potential
Integrated Process: Nursing Process
Reference: p. 162

24. The nurse cares for a pregnant client in labor and determines the fetus is in the
right occiput anterior (ROA) position. Which action by the nurse is best?
A. Continue to monitor the progress of labor.
B. Auscultate fetal heart rate (FHR) in the left upper quadrant.
C. Prepare the client for cesarean birth of the fetus.
D. Educate the client this fetal position may result in a longer labor.

Answer: A

Rationale: ROA (right occiput anterior) means the occiput of the fetal head points
toward the mother's right anterior pelvis; the head is the presenting part. This is a
common and favorable position for vaginal birth. Based on the ROA location, the
nurse will auscultate FHR at the right lower quadrant of the client's abdomen (FHR
will be loudest where the fetus' back is located). Occiput posterior (not anterior)
positions are associated with longer, more difficult births.
Question format: Multiple Choice
Chapter 8: The Labor Process
Cognitive Level: Apply
Client Needs: Health Promotion and Maintenance
Integrated Process: Nursing Process
Reference: p. 153

25. The nurse is monitoring a client and notes: contractions causing urge to push,
strong intensity, cervix 10 cm, 100% effaced, fetal head crowns when client
pushes. The nurse determines the client is currently in which stage or phase of
labor?
A. transition
B. second
C. third
D. active

Answer: B

Rationale: The second stage of labor is between full dilation (dilatation) and birth of
the infant. This woman has completed transition and is in the second stage of labor.
The third stage begins with the birth of the baby and ends with delivery of the
placenta. The active phase begins at 4 cm cervical dilation and ends when the
cervix is dilated 8 cm. During the transition phase, contractions reach their peak of
intensity, occurring every 2 to 3 minutes with a duration of 60 to 70 seconds, and a
maximum cervical dilatation of 8 to 10 cm occurs. Both the active and transistion
phases ocur during the first stage of labor.
Question format: Multiple Choice
Chapter 8: The Labor Process
Cognitive Level: Apply
Client Needs: Health Promotion and Maintenance
Integrated Process: Nursing Process
Reference: p. 161
Chapter 9
1. Erin, a 19-year-old first-time mother in the second stage of labor who has been
given an epidural, reports severe, unrelenting abdominal pain and rates it as 10 on
a scale of 0 to 10. What should the nurse do?
A. Proceed with standard care; the nurse knows that this is typical in the second
stage of labor and that younger women and those who are first-time mothers are
more likely to report severe pain.
B. Call the obstetrician; severe unrelenting abdominal pain could indicate placental
abruption, uterine rupture, or other undiagnosed complication.
C. Call the anesthetist, who is responsible for managing the epidural and should be
monitoring Erin's pain.
D. Discuss with Erin the additional pain medication options available to her at this
stage in her labor so that she can choose which option she prefers.

Answer: B

Rationale: After an epidural is in place, if the woman reports unrelenting pain, the
provider must assess the situation for a complication. This is not a standard
reaction; pain should come in waves, and should not be unrelenting. Wait for the
obstetrician to page the anesthetist. Do not leave the decision up to the mother;
rather, trust the experts.
Question format: Multiple Choice
Chapter 9: Pain Management during Labor and Birth
Cognitive Level: Apply
Client Needs: Physiological Integrity: Reduction of Risk Potential
Integrated Process: Nursing Process
Reference: p. 179

2. Which nursing intervention offered in labor would probably be the most effective
in applying the gate control theory for relief of labor pain?
A. Encourage the woman to rest between contractions.
B. Change the woman's position.
C. Give the prescribed medication.
D. Massage the woman's back.

Answer: D

Rationale: Gate-control is based on the idea of distraction or redirection of the


conduction of impulses up the neural pathways. Massage redirects the paths of
sensation away from the pain to the other area. Encouragement is a form of
psychological support. Position change will only distract the client. Medication
should be withheld until all nonpharmacologic treatments have been exhausted.
Question format: Multiple Choice
Chapter 9: Pain Management during Labor and Birth
Cognitive Level: Apply
Client Needs: Health Promotion and Maintenance
Integrated Process: Nursing Process
Reference: p. 171

3. The nurse instructs the client about skin massage and the gate control theory of
pain. Which statement would be appropriate for the nurse to include for client
understanding of the nonpharmacologic pain relief methods?
A. The gating mechanism is located at the pain site.
B. Pain perception is decreased if anxiety is present.
C. The gating mechanism opens so all the stimuli pass through to the brain.
D. These methods are a technique to prevent the painful stimuli from entering the
brain.

Answer: D

Rationale: Gate-control diverts the pain stimuli from the pain site by replacing with
a comfort stimuli in a new location. Gate control does not need to be applied
directly to the site of the pain. Anxiety heightens the painful feelings. Gating blocks
the flow of painful stimuli to the sensory centers in the brain.
Question format: Multiple Choice
Chapter 9: Pain Management during Labor and Birth
Cognitive Level: Apply
Client Needs: Physiological Integrity: Basic Care and Comfort
Integrated Process: Teaching/Learning
Reference: p. 171

4. A client and her husband have prepared for a natural birth; however, as the
client progresses to 8 cm dilation, she can no longer endure the pain and begs the
nurse for an epidural. What is the nurse's best response?
A. Suggest a less extreme alternative such as a sedative.
B. Support the client's decision and call the provider.
C. Gently remind the client of her goal of a natural birth and encourage and help
her.
D. Ask the husband to gently remind her of their goal of natural birth and to
encourage and help her.

Answer: B

Rationale: Pain is subjective and its level is only what the client experiences. The
nurse should support the desire of the client. Sedatives would be counterproductive
as they may slow the labor process. It would be inappropriate to negate her
feelings and remind her of earlier goals; that is the job of the support person and
should be left up him or her to decide what to say and when to say it.
Question format: Multiple Choice
Chapter 9: Pain Management during Labor and Birth
Cognitive Level: Apply
Client Needs: Safe, Effective Care Environment: Management of Care
Integrated Process: Caring
Reference: p. 169

5. A primigravida client in the second stage of labor has been moaning, screaming,
and generally vocal throughout her labor. Her husband is distraught seeing his wife
this way and asks the nurse for more pain medication for her. What is the nurse's
best response?
A. Assist the client with breathing and imagery techniques in an attempt to calm
her down.
B. Reassure the first-time father that his wife will be fine, and offer to stay with her
while he takes a walk.
C. Page the obstetrician to evaluate the client's pain, and administer an appropriate
increase in her pain medication.
D. Ask the client to describe the intensity of her pain on a scale of 0 to 10.

Answer: D

Rationale: The nurse should first assess the client's pain by asking her to describe
the pain on a scale of 0 to 10, as well as evaluate the client's actions. After the
assessment, further actions can then be taken, whether that be calling the
obstetrician or suggesting nonpharmacologic techniques to help the client calm
down. The client should be the one to request the medication, not the spouse. The
nurse should not encourage the mother's support person to leave; he or she is
necessary for the psychological well-being of the mother.
Question format: Multiple Choice
Chapter 9: Pain Management during Labor and Birth
Cognitive Level: Apply
Client Needs: Physiological Integrity: Basic Care and Comfort
Integrated Process: Nursing Process
Reference: p. 178

6. The nurse has just applied a sterile pressure dressing to an epidural site after
removing the epidural catheter in a client who is now recovering from a standard
delivery. Which action should the nurse now prioritize?
A. Assess return of sensory and motor functions to the lower extremities.
B. Help the client get up and walk around immediately.
C. Let the client rest and recover while keeping her legs slightly elevated.
D. Make sure the client receives plenty of fluids.

Answer: A

Rationale: After removal of the epidural catheter and medication is terminated, the
nurse needs to assess for return of motor function to ambulate the mother. The
mother will not be able to walk for some time (at least until the medication wears
off). Do not elevate the legs; the goal is to maintain normal circulation. Fluids are
important, but they are not related to the epidural or to the metabolism of the
medication.
Question format: Multiple Choice
Chapter 9: Pain Management during Labor and Birth
Cognitive Level: Apply
Client Needs: Physiological Integrity: Pharmacological and Parenteral Therapies
Integrated Process: Nursing Process
Reference: p. 175

7. A client has presented in the early phase of labor, experiencing abdominal pain
and signs of growing anxiety about the pain. Which pain management technique
should the nurse prioritize at this stage?
A. Immersing the client in warm water in a pool or hot tub
B. Practicing effleurage on the abdomen
C. Administering a sedative such as secobarbital or pentobarbital
D. Administering an opioid such as meperidine or fentanyl

Answer: B

Rationale: In early labor, the less medication use the better; allow use of
nonpharmacologic management and control the pain with effleurage. Sitting in a
warm pool of water is relaxing and may lessen the pain, but it does not control the
pain. Sedatives are not indicated as they may slow the birthing process. Opioids
should be limited as they too may slow the progression of labor.
Question format: Multiple Choice
Chapter 9: Pain Management during Labor and Birth
Cognitive Level: Analyze
Client Needs: Physiological Integrity: Basic Care and Comfort
Integrated Process: Nursing Process
Reference: p. 170

8. The nurse is assisting a client through labor, monitoring her closely now that she
has received an epidural. Which finding should the nurse prioritize to the
anesthesiologist?
A. Dry, cracked lips
B. Urinary retention
C. Rapid progress of labor
D. Inability to push

Answer: D

Rationale: If the client is not able to push, her epidural dose may need to be
adjusted to decrease the impact on the sensory system. Dry lips indicate that she
may need fluids, so the nurse should give her some ice chips or a drink of water.
Urinary retention and rapidly progressing labor should be directly reported to the
obstetrician, not the anesthesiologist.
Question format: Multiple Choice
Chapter 9: Pain Management during Labor and Birth
Cognitive Level: Analyze
Client Needs: Physiological Integrity: Pharmacological and Parenteral Therapies
Integrated Process: Nursing Process
Reference: p. 175
9. The nurse is monitoring a client who just received IV sedation. Which instruction
should the nurse prioritize with the client and her partner?
A. Ambulate only with assistance from the nurse or caregiver.
B. Ambulate within 15 minutes to prevent spinal headache.
C. Sit on the edge of the bed with her feet dangling before ambulating.
D. Remain in bed for at least 30 minutes.

Answer: A

Rationale: The client may have decreased sensory ability from the medication. She
needs assistance to ambulate for safety. She will be largely unable to move, so she
should remain in bed unless absolutely necessary.
Question format: Multiple Choice
Chapter 9: Pain Management during Labor and Birth
Cognitive Level: Apply
Client Needs: Safe, Effective Care Environment: Safety and Infection Control
Integrated Process: Nursing Process
Reference: p. 175

10. The nurse is monitoring a client who is in the second stage of labor, at +2
station, and anticipating birth within the hour. The client is now reporting the
epidural has stopped working and is begging for something for pain. Which action
should the nurse prioritize?
A. Call the anesthetist from the nurse's station to retry the epidural.
B. Call the primary care provider, and obtain a reduced dose of meperidine.
C. Give the meperidine because she needs pain relief now.
D. Encourage her through the contractions, explaining why she cannot receive any
pain medication.

Answer: D

Rationale: At this point, any medication would be contraindicated as it would pass


to the fetus and may cause respiratory depression. The nurse will have to work with
the mother through the contractions and pushing. The client has progressed too far
to retry the epidural medication. No meperidine should be given due to the risk to
the fetus.
Question format: Multiple Choice
Chapter 9: Pain Management during Labor and Birth
Cognitive Level: Apply
Client Needs: Physiological Integrity: Reduction of Risk Potential
Integrated Process: Nursing Process
Reference: p. 174

11. The nurse is admitting a primigravida client who has just presented to the unit
in early labor. Which response should the nurse prioritize to assist the client in
remaining calm and cooperative during birth?
A. "The baby is coming. Relax and everything will turn out fine."
B. "Do you want me to call in your family?"
C. "Even though the baby is coming, the health care provider will be here soon."
D. "The baby is coming. I'll explain what's happening and guide you."

Answer: D

Rationale: Continuous labor support with a trained nurse or doula has been shown
to be effective in increasing coping ability of laboring woman. To keep her calm, the
nurse needs to explain all procedures and discuss all events to the mother. The
nurse cannot know the final outcome and should be careful of making general
statements indicating everything will be OK. It is the nurse's responsibility to calm
the client down and not wait for the health care provider. While calling the family
may help, there is no guarantee and the nurse needs to work to calm the client
down.
Question format: Multiple Choice
Chapter 9: Pain Management during Labor and Birth
Cognitive Level: Apply
Client Needs: Psychosocial Integrity
Integrated Process: Nursing Process
Reference: p. 168

12. The client is progressing into the second stage of labor and coping well with the
natural birth method. Which instructions should the nurse prioritize at this point in
the process?
A. Use a birthing ball and find a position of comfort.
B. Stay low on her back to ease the back pain.
C. Use the Valsalva maneuver for effective pushing.
D. Ask for privacy, and have just the partner present.

Answer: A

Rationale: The position is very important during labor. Allowing the woman to
assume the most comfortable position will facilitate natural birth. The birthing ball
allows the woman to move and adjust her position so that she can remain
comfortable. The Valsalva maneuver may result in dangerous increases in blood
pressure, so the nurse should be sure to instruct the mother to breathe as she
pushes. The nurse should not intervene with who comes in or what family members
are present unless she is asked, or unless the visitation is upsetting the mother.
Question format: Multiple Choice
Chapter 9: Pain Management during Labor and Birth
Cognitive Level: Apply
Client Needs: Health Promotion and Maintenance
Integrated Process: Nursing Process
Reference: p. 171

13. The nurse is preparing a birthing care plan for a pregnant client. Which factor
should the nurse prioritize to achieve adequate pain relief during the birthing
process?
A. The client has the baby without any analgesic or anesthetic.
B. The health care provider decides the best pain relief for the mother and family.
C. Client priorities and preferences are incorporated into the plan.
D. The nurse suggests alternative methods of pain relief.

Answer: C

Rationale: The nurse and the client should work together with the nurse seeking
information on the desires of the client and work to achieve the desired level of
pain control for the labor and birth experience. The nurse can suggest various
options for the client to choose from, but that is not the priority. The health care
provider will also work with the client and prescribe medications as necessary
and/or desired. The health care professionals should respect the client's wishes for
pain control and not insist the client follow any specific format.
Question format: Multiple Choice
Chapter 9: Pain Management during Labor and Birth
Cognitive Level: Apply
Client Needs: Safe, Effective Care Environment: Management of Care
Integrated Process: Nursing Process
Reference: p. 169

14. The nurse is preparing a client for an epidural block. Which intervention is a
priority before the epidural anesthesia is started?
A. Increase oral fluids
B. IV fluid bolus
C. Monitor temperature
D. Monitor maternal apical pulse

Answer: B

Rationale: The client will need to have a bolus of IV fluids prior to the epidural to
prevent hypotension. The hypotensive event is transitory, and increasing oral
hydration is unnecessary and may lead to nausea later. Monitor the mother's body
temperature as per routine. The nurse should monitor the radial pulse not the
apical pulse.
Question format: Multiple Choice
Chapter 9: Pain Management during Labor and Birth
Cognitive Level: Apply
Client Needs: Physiological Integrity: Pharmacological and Parenteral Therapies
Integrated Process: Nursing Process
Reference: p. 175-176

15. The client in labor at 3 cm dilation and 25% effaced is asking the nurse for
analgesia. Which explanation should the nurse provide when explaining why it is
too early to administer an analgesic?
A. This would cause fetal depression in utero.
B. This may prolong labor and increase complications.
C. The effects would wear off before delivery.
D. This can lead to maternal hypertension.

Answer: B

Rationale: Administration of pharmacologic agents too early in labor can stall the
labor and lengthen the entire labor. The client should be offered nonpharmacologic
options at this point until she is in active labor. At this point in labor, the fetus
would not be affected by analgesia. The effects would wear off and the drug would
need to be re-administered, which would increase the risk to the fetus. There is no
link between maternal hypertension and analgesia.
Question format: Multiple Choice
Chapter 9: Pain Management during Labor and Birth
Cognitive Level: Apply
Client Needs: Health Promotion and Maintenance
Integrated Process: Nursing Process
Reference: p. 175

16. The nursing instructor is teaching a group of nursing students about the
uniqueness of pain involved with the birthing process. The instructor determines the
session is successful when the students correctly choose which pain factor to be
related to psychosocial influences?
A. Stretching of cervix
B. Descent of fetus into birth canal
C. Pressure in the perineum
D. Fear of pain during labor

Answer: D

Rationale: Fear of pain during labor is a psychosocial factor. The stretching of the
cervix, descent of the fetus into the birth canal, and pressure on the perineum are
physical factors.
Question format: Multiple Choice
Chapter 9: Pain Management during Labor and Birth
Cognitive Level: Analyze
Client Needs: Psychosocial Integrity
Integrated Process: Teaching/Learning
Reference: p. 168

17. The nursing instructor has finished leading a general discussion with a group of
nursing students exploring the labor and birthing process. The instructor
determines the session is successful when the students correctly state which goal
should be a priority?
A. Provide any medication the client requests.
B. Dictate the pain management during labor for the best outcome.
C. Work with the labor client to plan pain management options.
D. Monitor the client for active labor, and assess for pain as needed.

Answer: C
Rationale: The role of the nurse is to work with the client and plan with the nurse
the pain management technique desired by the client to meet the level of
expectation for the client. Pain is subjective, and each woman has a right to her
own labor plan. As the client progresses through the labor and delivery process, the
nurse should constantly monitor and assess the client's pain and provide the
necessary care accordingly.
Question format: Multiple Choice
Chapter 9: Pain Management during Labor and Birth
Cognitive Level: Analyze
Client Needs: Health Promotion and Maintenance
Integrated Process: Teaching/Learning
Reference: p. 167

18. The client and her partner have prepared for a natural birth and bring a picture
of a sunset over the ocean with them. The nurse predicts they will be using which
technique during labor?
A. Patterned birthing
B. Water therapy
C. Attention focusing
D. Hypnosis

Answer: C

Rationale: Attention focusing is the use of an object or picture or image for the
woman to reflect and focus on (internally or externally) during labor to distract her
from the labor pain. Hypnosis is a psychological state. Water therapy involves the
woman sitting in water to relax. Patterned breathing involves the woman controlling
her breathing patterns during contractions and "breathing through" them to help
control the pain. The attention focusing, patterned breathing, water therapy, and
hypnosis are all variations of relaxation which may be used by the client during the
birthing process.
Question format: Multiple Choice
Chapter 9: Pain Management during Labor and Birth
Cognitive Level: Apply
Client Needs: Health Promotion and Maintenance
Integrated Process: Nursing Process
Reference: p. 170

19. The client is asking the nurse about the various options for pain relief and
requests more information about the intradermal water injection her sister had.
When explaining the procedure, the nurse should point out these injections are
given at which location?
A. upper back
B. lower back
C. lumbar area
D. iliac crest
Answer: B

Rationale: This method of pain relief is considered nonpharmacologic in nature; it


does not use medication to relieve the pain. Sterile water is injected under the skin
to activate the gate theory of pain control. Injections are placed in the lower back
in the sacral area.
Question format: Multiple Choice
Chapter 9: Pain Management during Labor and Birth
Cognitive Level: Apply
Client Needs: Physiological Integrity: Reduction of Risk Potential
Integrated Process: Teaching/Learning
Reference: p. 172

20. A client in latent labor for the past 12 hours is requesting medication to help
her rest. The nurse predicts the health care provider will prescribe which
medication?
A. secobarbital
B. meperidine
C. fentanyl
D. morphine

Answer: A

Rationale: In the latent phase of labor, sedatives can be prescribed to assist a client
to rest. The use of analgesics, such as opioids (meperidine, fentanyl, and morphine)
in early labor may stop labor and are not recommended.
Question format: Multiple Choice
Chapter 9: Pain Management during Labor and Birth
Cognitive Level: Apply
Client Needs: Physiological Integrity: Pharmacological and Parenteral Therapies
Integrated Process: Nursing Process
Reference: p. 175

21. The multigravida client is moving into the transition phase and asks for an
opioid, stating she does not remember the pain being this bad before. Which
response from the nurse will be best?
A. "I will page the provider and ask for your pain medication."
B. "You are so close to birth; do you not want to have natural birth?"
C. Pain medication can affect the baby's breathing; let's try to focus and breathe."
D. "Rather than use an opioid, let's ask for a different type of pain medication."

Answer: C

Rationale: Once the woman has entered into the transition phase of labor, she is
considered to be imminent for birth. Any opioid medication might pass to the fetus
and is not recommended due to the effects of respiratory compromise. The nurse
will need to encourage nonpharmacologic methods at this point and should not
consult the provider. The nurse should also remain supportive of the mother.
Question format: Multiple Choice
Chapter 9: Pain Management during Labor and Birth
Cognitive Level: Apply
Client Needs: Health Promotion and Maintenance
Integrated Process: Nursing Process
Reference: p. 174

22. A client at 41 weeks' gestation has been in labor for 18 hours and the fetus is
now showing signs of distress. Due to prior back surgery, the client is to receive
general anesthesia instead of an epidural. Which medication will the nurse prepare
to give the client first?
A. Pain medication
B. Nonsteroidal anti-inflammatory (NSAID)
C. Antacid
D. Sedative

Answer: C

Rationale: Prior to intubation for a general surgery, the client should receive a dose
of antacid to decrease the risk of aspiration of acidic stomach contents. Pain
medication, NSAIDS, and sedative medications should not be given prior to a
general anesthetic.
Question format: Multiple Choice
Chapter 9: Pain Management during Labor and Birth
Cognitive Level: Apply
Client Needs: Physiological Integrity: Pharmacological and Parenteral Therapies
Integrated Process: Nursing Process
Reference: p. 177-178

23. The nursing instructor is preparing a presentation which will explore the various
sources of pain during the labor process. Which source should the instructor
emphasize as the main source of pain during the first stage?
A. perineum
B. cervix
C. back
D. birth canal

Answer: B

Rationale: The first stage of labor is when the cervix is dilating. The woman in labor
will have pain from the stretching and dilation (dilatation) of the cervix. Stretching
of the perineum and birth canal is associated with the second stage of labor. Not all
women have back pain, and typically when they do it is referred pain from the
lower abdomen and can happen in both the first and second stage of labor.
Question format: Multiple Choice
Chapter 9: Pain Management during Labor and Birth
Cognitive Level: Apply
Client Needs: Health Promotion and Maintenance
Integrated Process: Nursing Process
Reference: p. 168

24. The nurse is monitoring the client's vital signs and notes: 100.2oF (37.9oC),
heart rate 82, respiratory rate 17, and blood pressure 124/78. The client has
recently had an epidural. What is the best response when the client's partner asks
if she is getting sick?
A. "We will continue to monitor the situation."
B. "The fever may be due to the epidural."
C. "Have you been exposed to any illnesses recently?"
D. "She's dehydrated and needs something to drink."

Answer: B

Rationale: A common side effect of epidural anesthesia is elevated temperature


during labor. The client needs frequent assessment and to be observed for any
other signs or symptoms of an infection, but it is premature to state it is related to
an infection. If the mother has been exposed to any illness, it would be in the
history. Oral fluids would not be advisable as they may result in nausea later.
Question format: Multiple Choice
Chapter 9: Pain Management during Labor and Birth
Cognitive Level: Analyze
Client Needs: Physiological Integrity: Reduction of Risk Potential
Integrated Process: Nursing Process
Reference: p. 175-176

25. A primigravida client has just arrived in early labor and is showing signs of
extreme anxiety over the birthing process. Why should the nurse prioritize helping
the client relax?
A. Anxiety will increase blood pressure, increasing risk with an epidural.
B. Decreased anxiety will increase trust in the nurse.
C. Anxiety can slow down labor and decrease oxygen to the fetus.
D. Increased anxiety will increase the risk for needing anesthesia.

Answer: C

Rationale: Out-of-control anxiety can decrease the oxygen of the mother by


increasing her respiratory rate and increasing the demand on her body, and can
have a negative impact on the fetus by decreasing the amount of oxygen reaching
the fetus. Encourage control of the anxiety. Anxiety will not negatively affect the
action of the epidural or the need for anesthesia. Trust in the nurse is not
determined by the amount of anxiety the client experiences.
Question format: Multiple Choice
Chapter 9: Pain Management during Labor and Birth
Cognitive Level: Apply
Client Needs: Health Promotion and Maintenance
Integrated Process: Nursing Process
Reference: p. 168
26. The nurse is preparing to assist with a pudendal block. The nurse predicts the
client is at which point in the labor process?
A. early-stage labor
B. before dilation (dilatation) only
C. just before birth
D. just after birth

Answer: C

Rationale: Pudendal block is a local block in the perineal area and is used to numb
for birth. Giving a pudendal block before labor begins or while labor is in its early
stages would be counterproductive, as the client would not have proper feeling and
would have a harder time pushing. After birth it is pointless; the most painful part
is over.
Question format: Multiple Choice
Chapter 9: Pain Management during Labor and Birth
Cognitive Level: Apply
Client Needs: Health Promotion and Maintenance
Integrated Process: Nursing Process
Reference: p. 175

27. The client is experiencing back labor and reporting intense pain in the lower
back. The nurse should point out which intervention will be effective at this point?
A. effleurage of the abdomen during the contraction
B. conscious relaxation/guided imagery in low Fowler position
C. counterpressure against the sacrum
D. pant-blow (breaths and puffs breathing techniques)

Answer: C

Rationale: Counterpressure against the sacrum is a way to provide support and


comfort for a women having intense back labor. Effleurage is ineffective for true
back labor, as it is conscious relaxation. Breathing will not diminish the pain of back
labor.
Question format: Multiple Choice
Chapter 9: Pain Management during Labor and Birth
Cognitive Level: Apply
Client Needs: Health Promotion and Maintenance
Integrated Process: Nursing Process
Reference: p. 171

28. A woman dilated to 10 cm and feeling the urge to "have a bowel movement" is
refusing to push and is screaming, "It hurts down there too much to push." What
option should the nurse suggest at this point for pain management to facilitate
pushing?
A. epidural anesthesia
B. pudendal block
C. paracervical block
D. parenteral medication

Answer: B

Rationale: The client is too far dilated to have any parenteral medication or an
epidural block. The best option is a local block or a pudendal block that will numb
the vaginal wall to block the pain sensation to the pudendal nerve. The paracervical
block is only used in the first stage of labor, and this client is in the second stage.
Question format: Multiple Choice
Chapter 9: Pain Management during Labor and Birth
Cognitive Level: Apply
Client Needs: Physiological Integrity: Pharmacological and Parenteral Therapies
Integrated Process: Nursing Process
Reference: p. 175
Chapter 10
1. The student nurse is preparing to assess the fetal heart rate (FHR) and has
determined that the fetal back is located toward the client's left side, the small
parts toward the right side, and there is a vertex (occiput) presentation. The nurse
should initially begin auscultation of the fetal heart rate in the mother's:
A. right upper quadrant.
B. right lower quadrant.
C. left upper quadrant.
D. left lower quadrant.

Answer: D

Rationale: The best position to auscultate fetal heart tones in on the fetus back. In
this position, the best place for the FHR monitor is on the left lower quadrant.
Question format: Multiple Choice
Chapter 10: Nursing Care during Labor and Birth
Cognitive Level: Apply
Client Needs: Health Promotion and Maintenance
Integrated Process: Nursing Process
Reference: p. 192

2. A primigravida client admitted with signs of labor is evaluated with external


electronic fetal monitoring that shows baseline FHR of 136 to 150 and two instances
of FHR at 165 for 15 to 20 seconds. Which response should the nurse prioritize?
A. Immediately report to the RN that the FHR shows tachycardia.
B. Immediately report to the RN that the FHR shows no variability.
C. Before reporting to the RN, determine the short term variability (STV).
D. Before reporting to the RN, determine the uterine contraction pattern.

Answer: D

Rationale: The nurse needs to assess and determine if the changes are related to
accelerations secondary to contractions. Assess the contraction pattern with the
fetal heart rate and provide information to the RN. If the accelerations are not due
to uterine contractions, notify the RN immediately. Until then, the nurse should do
the assessment before reacting.
Question format: Multiple Choice
Chapter 10: Nursing Care during Labor and Birth
Cognitive Level: Apply
Client Needs: Health Promotion and Maintenance
Integrated Process: Nursing Process
Reference: p. 194

3. The client presents in the early stage of labor with mild contractions 7 to 9
minutes apart and blood pressure 130/80 mm Hg. The client changes from happy,
excited, and confident to introverted and restless. Assessment reveals heart rate
100, blood pressure 137/85 mm Hg, and hyperventilation. EFM reveals no
variability for almost 20 minutes, then evident variability with no late decelerations.
Which action should the nurse prioritize?
A. Notify the RN that client's blood pressure has increased.
B. Notify the RN about the lack of FHR variability.
C. Help the client regain control of her breathing technique.
D. Assist the client into a hands-and-knees position.

Answer: C

Rationale: The primary focus is to regain her breathing to a normal rhythm; focus
her on breathing and relaxation and relief from the hyperventilation. If there is no
improvement, notify the RN. Putting the client in the hands-and-knees position
should be avoided until later in labor.
Question format: Multiple Choice
Chapter 10: Nursing Care during Labor and Birth
Cognitive Level: Analyze
Client Needs: Safe, Effective Care Environment: Management of Care
Integrated Process: Nursing Process
Reference: p. 199

4. A client has been showing a gradual increase in FHR baseline with variables;
however, after 5 hours of labor and several position changes by the client, the fetus
no longer shows signs of hypoxia. The client's cervix is almost completely effaced
and dilated to 8 cm. Which action should the nurse prioritize if it appears the fetus
has stopped descending?
A. Alert the team that internal fetal monitoring may be needed.
B. Palpate the area just above the symphysis pubis.
C. Institute effleurage and apply pressure to the client's lower back during
contractions.
D. Encourage the client to push.

Answer: B

Rationale: Palpate just above the symphysis pubis to determine if the infant is
engaged and to determine the presenting part of the infant; it is possible for infants
to rotate and change position during labor. The nurse should assess the situation
and act further if necessary, but until there is more information on the fetal
position, the nurse should assume all is going well.
Question format: Multiple Choice
Chapter 10: Nursing Care during Labor and Birth
Cognitive Level: Apply
Client Needs: Health Promotion and Maintenance
Integrated Process: Nursing Process
Reference: p. 199
5. The nursing instructor is teaching the students the basics of the labor and
delivery process. The instructor determines the session is successful when the
students correctly choose which action will best help to prevent infections in their
clients?
A. Clean the woman's perineum with a Betadine scrub.
B. Strictly follow universal precautions.
C. Replace soiled drapes and linen as needed.
D. Thoroughly wash the hands before and after client contact.

Answer: D

Rationale: The most important infection control technique in any health care setting
is thoroughly washing hands on a routine basis. Keeping the area clean is
secondary but also important.
Question format: Multiple Choice
Chapter 10: Nursing Care during Labor and Birth
Cognitive Level: Analyze
Client Needs: Safe, Effective Care Environment: Safety and Infection Control
Integrated Process: Teaching/Learning
Reference: p. 201

6. The nurse is discussing the various positions for birth with a client and her
partner. The client mentions she would like a position that speeds up the process,
decreases stress to her baby, and reduces the possibility of needing an episiotomy.
Which position should the nurse point out will best meet the client's desires?
A. lithotomy
B. modified dorsal recumbent
C. side-lying
D. hands and knees

Answer: D

Rationale: The hands and knees position is documented to be one of the best
delivery positions for easing delivery and improving outcomes. Lithotomy (feet in
stirrups), modified dorsal recumbent (feet on foot pedals), and side-lying are all
potential positions, but not statistically the best. They also do not meet all the goals
of the client.
Question format: Multiple Choice
Chapter 10: Nursing Care during Labor and Birth
Cognitive Level: Apply
Client Needs: Health Promotion and Maintenance
Integrated Process: Teaching/Learning
Reference: p. 204

7. The nurse is admitting a client in early labor and notes: FHR 120 bpm, blood
pressure 126/84 mm Hg, temperature 98.8°F (37.1°C), contractions every 4 to 5
minutes lasting 30 seconds, and greenish-color fluid in the vaginal vault. Which
finding should the nurse prioritize?
A. Fetal heart rate
B. Possible maternal infection
C. Green-colored fluid in the vagina
D. Irregular contractions

Answer: C

Rationale: Green-tinted fluid with ROM is indicative of meconium in the amniotic


sac, or the infant having a bowel movement in utero. Infection would be shown by
pus or cloudy fluid and possibly an elevated temperature. The FHR is within normal
range. Irregular contractions are expected at this stage of labor.
Question format: Multiple Choice
Chapter 10: Nursing Care during Labor and Birth
Cognitive Level: Apply
Client Needs: Physiological Integrity: Physiological Adaptation
Integrated Process: Nursing Process
Reference: p. 199

8. The nurse is monitoring a client who has given birth and is now bonding with her
infant. Which finding should the nurse prioritize and report immediately for
intervention?
A. The mother is unable to void after 4 hours.
B. Maternal tachycardia and falling blood pressure
C. Placental separation 15 minutes after birth
D. Dark red lochia

Answer: B

Rationale: Maternal tachycardia and falling blood pressure may indicate fluid
volume deficit or hemorrhage and require immediate assistance. The client needs
further assessment and intervention. The final stages of labor may include uterine
contractions, the delivery of the placenta, and the discharge of lochia as the uterus
heals. These are normal and not an emergency.
Question format: Multiple Choice
Chapter 10: Nursing Care during Labor and Birth
Cognitive Level: Analyze
Client Needs: Physiological Integrity: Physiological Adaptation
Integrated Process: Nursing Process
Reference: p. 209

9. The nurse discovers that the FHM is now recording late decelerations in a client
who is in labor. The nurse predicts this is most likely related to which event?
A. Cord compression
B. Maternal hypotension
C. Maternal fatigue
D. Uteroplacental insufficiency

Answer: D
Rationale: Late decelerations are associated with uteroplacental insufficiency. They
typically indicate decreased blood flow to the uterus during the contractions.
Maternal hypotension and fatigue would not be observed on the fetal heart monitor.
Cord compression would be marked by fetal tachycardia.
Question format: Multiple Choice
Chapter 10: Nursing Care during Labor and Birth
Cognitive Level: Apply
Client Needs: Physiological Integrity: Physiological Adaptation
Integrated Process: Nursing Process
Reference: p. 193-194

10. A client in the latent phase of the first stage of labor is noted to be
uncomfortable with intact membranes and mild contractions on assessment. The
nurse should encourage the client to pursue which action?
A. Complete bed rest
B. Ambulation ad lib
C. Bathroom privileges
D. Up in chair TID

Answer: B

Rationale: To facilitate the first stage of labor, ambulation and movement will allow
better fetal descent and help to speed the labor process. Bed rest will slow or stop
the labor process. The client may use the bathroom as needed, but this does not
affect labor rate. The client should remain mobile.
Question format: Multiple Choice
Chapter 10: Nursing Care during Labor and Birth
Cognitive Level: Apply
Client Needs: Health Promotion and Maintenance
Integrated Process: Nursing Process
Reference: p. 198

11. The nurse is monitoring a client in the first stage of labor. The nurse determines
the client's uterine contractions are effective and progressing well based on which
finding?
A. Engagement of fetus
B. Dilation (dilatation) of cervix
C. Rupture of amniotic membranes
D. Bloody show

Answer: B

Rationale: The best determination of effective contractions is dilation (dilatation) of


the cervix. Engagement, membrane rupture, and bloody show may all occur before
the cervix has dilated.
Question format: Multiple Choice
Chapter 10: Nursing Care during Labor and Birth
Cognitive Level: Apply
Client Needs: Health Promotion and Maintenance
Integrated Process: Nursing Process
Reference: p. 196-197

12. The nurse is admitting an obstetric client in early labor. As the nurse assists the
client into the bed, which assessment should the nurse prioritize?
A. Past obstetrical history
B. Fetal status
C. Signs that birth is imminent
D. Client's temperature

Answer: C

Rationale: The priority is to establish the imminence of the birth, then the fetal
status. The obstetrical history can wait until after the birth of the baby, if
necessary. The maternal blood pressure is a higher priority over the temperature to
rule out possible preeclampsia.
Question format: Multiple Choice
Chapter 10: Nursing Care during Labor and Birth
Cognitive Level: Apply
Client Needs: Health Promotion and Maintenance
Integrated Process: Nursing Process
Reference: p. 185

13. A gravida 1 client is admitted in the active phase of stage 1 labor with the fetus
in the LOA position. The nurse anticipates noting which finding when the
membranes rupture?
A. Bloody fluid
B. Clear to straw-colored fluid
C. Greenish fluid
D. Cloudy white fluid

Answer: B

Rationale: The infant is in the correct position, and the client has been in labor.
Expectation would be for normal amniotic fluid presentation of clear to straw-
colored fluid. If there is blood, then the uterus is bleeding and there is an extreme
emergency. If the fluid is greenish, there is meconium in the fluid. Cloudy, white
fluid may indicate an infection is present.
Question format: Multiple Choice
Chapter 10: Nursing Care during Labor and Birth
Cognitive Level: Apply
Client Needs: Health Promotion and Maintenance
Integrated Process: Nursing Process
Reference: p. 187
14. The nurse is caring for a client who has been in labor for the past 8 hours. The
nurse determines that the client has transitioned into the second stage of labor
based on which sign?
A. Emotions are calm and happy.
B. Frequency of contractions are 5 to 6 minutes.
C. Fetus is at -1 station.
D. The urge to push occurs.

Answer: D

Rationale: Second stage of labor is the pushing stage; this is typically identified by
the woman's urge to push or a feeling of needing to have a bowel movement. In
the second stage the cervix can be 10 cm, dilated 100% and effaced. The station is
usually 0 to +2. The emotional state may be altered due to pain and pressure.
Contraction frequency is variable and not clearly indicative of a particular stage.
The fetus can be at stage -1 for any length of time.
Question format: Multiple Choice
Chapter 10: Nursing Care during Labor and Birth
Cognitive Level: Apply
Client Needs: Health Promotion and Maintenance
Integrated Process: Nursing Process
Reference: p. 202-203

15. The nursing instructor is teaching a group of nursing students about the various
responsibilities of the labor and delivery medical team. The instructor determines
the session is successful when the students correctly choose which function as the
primary role of the LPN/LVN members of the team?
A. Provide direct independent care to the client.
B. Assist the providers in the delivery room.
C. Provide care under the supervision of an RN.
D. Observatory to assist the RN.

Answer: C

Rationale: The LPN may provide care within the appropriate scope of practice under
the direct supervision of an RN. The RN is responsible for providing direct
independent care of the client. Both LPN/LVNs and RNs assist health care providers
in the delivery room. The LPN/LVNs provide more than just observatory functions
for the RN.
Question format: Multiple Choice
Chapter 10: Nursing Care during Labor and Birth
Cognitive Level: Understand
Client Needs: Safe, Effective Care Environment: Management of Care
Integrated Process: Teaching/Learning
Reference: p. 184-185

16. A multigravida woman arrives in the emergency department panting and


screaming, "The baby's coming!" Which action should the nurse prioritize?
A. Assess maternal and fetal vital signs.
B. Ask medical and obstetrical history.
C. Escort to Labor and Delivery.
D. Quickly evaluate the perineum.

Answer: D

Rationale: The woman is showing signs of advanced labor, possibly in transition or


stage 2. She needs to be managed as an imminent birth and a vaginal assessment
performed, as there may not be time to get to Labor and Delivery. Vital signs would
be assessed next. Medical/obstetrical history and her room assignment can be
taken care of later in the process.
Question format: Multiple Choice
Chapter 10: Nursing Care during Labor and Birth
Cognitive Level: Apply
Client Needs: Safe, Effective Care Environment: Management of Care
Integrated Process: Nursing Process
Reference: p. 203

17. The nurse is admitting a client who is in early labor. After determining that the
birth is not imminent, which assessment should the nurse perform next?
A. Risk factors
B. Maternal status
C. Fetal status
D. Maternal obstetrical history

Answer: C

Rationale: The woman may present to the birthing suite at any phase of the first
stage of labor. Therefore, it is important to assess birth imminence, fetal status,
risk factors, and maternal status immediately. If birth is not imminent and the fetal
and maternal conditions are stable, perform additional data collection, including the
full admission health history, a complete maternal physical assessment, the status
of labor and any labor, birth, and cultural preferences the woman may have.
Question format: Multiple Choice
Chapter 10: Nursing Care during Labor and Birth
Cognitive Level: Analyze
Client Needs: Safe, Effective Care Environment: Management of Care
Integrated Process: Nursing Process
Reference: p. 185

18. A multigravida client at 39 weeks' gestation has been in labor for 8 hours
without much change. The last vaginal exam revealed cervix 8 cm dilated and 0
station. Which is the best response if the client asks the nurse how far the fetus
has advanced in the past half hour?
A. "I can arrange for a cervix check, if you want."
B. "The health care provider will have to check you. I'll call him."
C. "Once your labor signs change, we can find out."
D. "Checking your cervix will not speed up labor; let's wait."

Answer: C

Rationale: The cervix must be assessed with a vaginal exam. The frequency of
vaginal exams is based on the signs of changes in labor. The client has not
demonstrated any changes in her labor pattern; the nurse should provide education
on the reason for not checking her. Frequent exams can interfere with the labor
process as well as increase the risk of infection.
Question format: Multiple Choice
Chapter 10: Nursing Care during Labor and Birth
Cognitive Level: Apply
Client Needs: Health Promotion and Maintenance
Integrated Process: Communication and Documentation
Reference: p. 199

19. The nurse is analyzing the readout on the EFM and determines the FHR pattern
is normal based on which recording?
A. Acceleration of at least 15 bpm for 15 seconds
B. Increase in variability by 27 bpm
C. Deceleration followed by acceleration of 15 bpm
D. Decrease in variability for 15 seconds

Answer: A

Rationale: A normal active fetal heart rate is a change in baseline by increase of 15


bpm for 15 seconds. This is a positive and normal periodic change in fetal heart
rates as a response to fetal movement. Normal variability is noted to occur within 6
to 25 bpm from the baseline FHR. There should be no decelerations.
Question format: Multiple Choice
Chapter 10: Nursing Care during Labor and Birth
Cognitive Level: Apply
Client Needs: Health Promotion and Maintenance
Integrated Process: Nursing Process
Reference: p. 193

20. The nurse is monitoring a laboring client with continuous fetal monitoring and
notes a decrease in FHR with variable deceleration to 75 bpm. Which intervention
should the nurse prioritize?
A. Administer oxygen.
B. Increase her IV fluids.
C. Change the position of the client.
D. Notify the primary care provider.

Answer: C

Rationale: Variable decelerations often indicate a type of cord compression. The


initial response is to change the position and try to release the cord compression. If
this does not work, apply oxygen while using the call light to alert others. If this
continues, her fluid status needs to be assessed before increasing her IV rate.
Question format: Multiple Choice
Chapter 10: Nursing Care during Labor and Birth
Cognitive Level: Apply
Client Needs: Physiological Integrity: Physiological Adaptation
Integrated Process: Nursing Process
Reference: p. 194

21. The nurse is assessing a new client who presents in early labor. The nurse
determines the fetus has an acceptable heart rate if found within which range?
A. 90 to 140 bpm
B. 100 to 150 bpm
C. 110 to 160 bpm
D. 120 to 170 bpm

Answer: C

Rationale: The standard acceptable fetal heart rate baseline is the range of 110 to
160 beats per minute. Sustained heart rates above or below the norm are cause for
concern.
Question format: Multiple Choice
Chapter 10: Nursing Care during Labor and Birth
Cognitive Level: Apply
Client Needs: Health Promotion and Maintenance
Integrated Process:
Reference: p. 193

22. The nursing instructor is preparing a class discussing the role of the nurse
during the labor and birthing process. Which intervention should the instructor point
out has the greatest effect on relieving anxiety for the client?
A. Massage therapy
B. Continuous labor support
C. Pharmacologic pain management
D. Prenatal classes

Answer: B

Rationale: Continuous labor support by a caring nurse or doula can help decrease a
woman's anxiety during labor. Anxiety causes the release of catecholamines, which
slow down the labor process. The continuous support helps keep the woman
focused on what is important as well as provide necessary guidance and education
as needed. The massage therapy, prenatal classes, and pharmacologic pain
management are all tools that the nurse can use to help the woman.
Question format: Multiple Choice
Chapter 10: Nursing Care during Labor and Birth
Cognitive Level: Apply
Client Needs: Health Promotion and Maintenance
Integrated Process: Teaching/Learning
Reference: p. 198

23. A client has moved into the active phase of labor and is now at 6 cm dilated and
+1 station. The nurse is prepared to monitor the contraction pattern how often?
A. Every 10 minutes
B. Every 15 minutes
C. Every 30 minutes
D. Every hour

Answer: C

Rationale: Active labor is a phase in the first stage of labor when the cervix dilates
from 4 to 8 cm. The contractions are progressing and occur every 2 to 5 minutes
and last 45 to 60 seconds. The nurse needs to evaluate the labor pattern every 30
minutes. During the latent phase of the first stage, the labor pattern should be
evaluated every hour. During the transition phase of the first stage, the contraction
pattern should also be evaluated every 30 minutes. During the second stage of
labor, the contraction pattern should be evaluated every 15 minutes.
Question format: Multiple Choice
Chapter 10: Nursing Care during Labor and Birth
Cognitive Level: Apply
Client Needs: Health Promotion and Maintenance
Integrated Process: Nursing Process
Reference: p. 192

24. The nurse is assessing a client in active labor and notes a small, rounded mass
above the symphysis pubis that is distended but nontender. Which action should
the nurse prioritize?
A. Check the chart for the last void.
B. Notify the health care provider about the mass.
C. Ask the client if the mass has always been present.
D. Assume this is part of the uterus.

Answer: A

Rationale: The most probable explanation of the mass is a full bladder. The nurse
should determine the last void by the client and offer to assist the client to void or
prepare to catheterize the client to empty the bladder. This can be handled by the
nurse. The client would not likely know if the mass was always present or not,
given its location. If it were the uterus, it would be tender to the touch.
Question format: Multiple Choice
Chapter 10: Nursing Care during Labor and Birth
Cognitive Level: Apply
Client Needs: Physiological Integrity: Reduction of Risk Potential
Integrated Process: Nursing Process
Reference: p. 199
25. The nurse is admitting a client who appears to be in advanced labor with
imminent birth. Which action should the nurse prioritize?
A. Obtain a comprehensive obstetric history.
B. Determine plans for labor and the newborn.
C. Take blood pressure and determine if clonus or edema are present.
D. Assess use of drugs, alcohol, and tobacco during pregnancy.

Answer: C

Rationale: In advanced labor, the most important assessments must be completed


first. The assessment for signs or symptoms of preeclampsia must be assessed
first, which would include the assessment of her blood pressure and presence of
clonus or edema. The history can be obtained after the birth of the baby or if labor
slows down. Plans for the newborn can be figured out later. Blood tests can be run
as soon as a sample can be taken from the mother.
Question format: Multiple Choice
Chapter 10: Nursing Care during Labor and Birth
Cognitive Level: Apply
Client Needs: Safe, Effective Care Environment: Management of Care
Integrated Process: Nursing Process
Reference: p. 187

26. A woman required an episiotomy and it was repaired by the birth attendant.
Which instruction will the nurse give to the woman?
A. "The stitches used will absorb on their own."
B. "Try applying some warm compresses to the area for pain relief."
C. "You will need an injection of a local anesthetic periodically until it heals."
D. "You will have to keep a sterile dressing on the area for 48 hours."

Answer: A

Rationale: The nurse should reassure the woman that the sutures are absorbable
and do not need to be removed. The birth attendant uses local anesthesia to numb
the perineum for repair, but it is not used periodically after the repair. If the woman
had epidural anesthesia, she may not need additional anesthesia for repair. A
sterile ice pack, not a warm compress, applied to the perineum after repair can help
decrease swelling and pain. There is no need for a sterile dressing.
Question format: Multiple Choice
Chapter 10: Nursing Care during Labor and Birth
Cognitive Level: Apply
Client Needs: Physiological Integrity: Reduction of Risk Potential
Integrated Process: Teaching/Learning
Reference: p. 220
Chapter 11
1. A woman who has already had one successful vaginal birth after prior cesarean
(VBAC) has elected to deliver her third child vaginally. She reports a sudden
sensation of "something snapping inside," followed by chest pain. What should the
nurse do?
A. Remind her that this stage of labor always seems longer than it really is.
B. Assist her into the squatting or hands-and-knees position.
C. Monitor the EFM tracing for non-reassuring patterns.
D. Prepare to assist the RN as scrub nurse.

Answer: D

Rationale: With VBAC there is a chance of rupture of the uterus. The report by the
client is indicative of a ruptured uterus, which is an obstetrical emergency and
would require immediate surgery. Reminding the woman that this stage of labor
always seems longer than it really is, or assisting her into the squatting or hands-
and-knees position, or monitoring the EFM tracing for nonreassuring patterns could
lead to the death of both mother and fetus.
Question format: Multiple Choice
Chapter 11: Assisted Delivery and Cesarean Birth
Cognitive Level: Analyze
Client Needs: Physiological Integrity: Physiological Adaptation
Integrated Process: Nursing Process
Reference: p. 231

2. A woman arrives at labor and delivery unit with contractions every 2–3 minutes
lasting 30–45 seconds. After several hours of labor have passed and not making
progress, the health care provider prepares to perform an amniotomy. The woman
asks the nurse, "How will this help my labor?" Which response by the nurse would
be appropriate?
A. "This procedure helps to push your baby through the birth canal."
B. "With this procedure, your body sends out hormones that make labor more
effective."
C. "By doing this, the passageway for your baby isn't blocked anymore."
D. "The fluid that comes out helps flushes away any debris in the birth canal."

Answer: B

Rationale: With an amniotomy, the health care provider guides the instrument
through the cervix and uses the hook to create a hole in the membranes. At this
point, amniotic fluid is usually expelled. This process causes the body to release
prostaglandins, which enhances labor. An amniotomy does not open up the
passageway, push the baby through the birth canal, or flush away debris.
Question format: Multiple Choice
Chapter 11: Assisted Delivery and Cesarean Birth
Cognitive Level: Apply
Client Needs: Health Promotion and Maintenance
Integrated Process: Nursing Process
Reference: p. 219-220

3. A client at 42 weeks' gestation presents for induction of labor. Which assessment


should the nurse prioritize as the best indicator for the induction to proceed?
A. Bishop score of 7
B. L/S ratio of 1.5
C. cervical presence of fetal fibronectin
D. cervical length of 28 millimeters

Answer: A

Rationale: Currently, the Bishop score of greater than 7 remains a reliable predictor
of cervical readiness that is both cost- and time-effective. Measuring the cervical
length by endovaginal ultrasound is another method that has been studied for
predicting labor readiness; however, at this time it has not shown to be a reliable
indicator of the cervix being ready for induction. At one time, it was considered to
be an indicator if the cervix was 27 millimeters or less. Evaluating the fetal
fibronectin is another test that has been conducted to determine cervical readiness.
The presence of this protein in the cervical secretions was considered a positive
sign; however, drawbacks include cost of the test and time to run the test. More
studies are needed to evaluate the predictive ability of the test. The L/S
(lecithin/sphingomyelin) ratio is a test conducted via amniocentesis to assess lung
maturity of the fetus.
Question format: Multiple Choice
Chapter 11: Assisted Delivery and Cesarean Birth
Cognitive Level: Apply
Client Needs: Health Promotion and Maintenance
Integrated Process: Nursing Process
Reference: p. 218

4. The nurse is preparing a client for treatment to ripen her cervix in anticipation of
labor. When comparing the various options for the client, which one will the nurse
point out possibly provides an oral option?
A. misoprostol
B. prostaglandin E2 gel
C. dinoprostone
D. prostaglandin E2 vaginal inserts

Answer: A

Rationale: The synthetic prostaglandin E1 misoprostol is used off-label as a cervical


ripening agent that has been successfully documented and approved by the FDA. It
is administered either orally or vaginally. When given via the oral route, misoprostol
causes less uterine hyperstimulation than via the vaginal route. Dinoprostone is
prostaglandin E2, which can be found in a gel or vaginal inserts. The health care
provider applies the gel to the cervix during the pelvic examination; this will work
to ripen the cervix. The vaginal insert is a time-release product that is inserted into
the vagina and will also gradually ripen the cervix. An advantage to the vaginal
insert is the ability to remove the insert should adverse reactions to the
prostaglandin occur.
Question format: Multiple Choice
Chapter 11: Assisted Delivery and Cesarean Birth
Cognitive Level: Apply
Client Needs: Physiological Integrity: Pharmacological and Parenteral Therapies
Integrated Process: Nursing Process
Reference: p. 219

5. The nurse is monitoring a client at 41 weeks' gestation receiving IV oxytocin.


Which action should the nurse prioritize if noticeable contractions are occurring
every 2 minutes, lasting 60 to 90 seconds on the fetal monitor?
A. Administer oxygen 10L via face mask.
B. Change the client's position.
C. Stop the IV oxytocin infusion.
D. Notify the health care provider.

Answer: C

Rationale: The first action is to stop the IV infusion. This contraction pattern is
noted to be hyperstimulation of the uterus, and the uterine stimulant should be
stopped first. Administering oxygen assists in the prevention of hypoxia to both the
mother and the fetus, but it does not take priority over stopping the uterine
stimulant. Changing the client's position will have no effect on the hyperstimulation
of the uterus as long as the stimulant is still being infused. The nurse would notify
the health care provider after stopping the stimulant.
Question format: Multiple Choice
Chapter 11: Assisted Delivery and Cesarean Birth
Cognitive Level: Analyze
Client Needs: Physiological Integrity: Physiological Adaptation
Integrated Process: Nursing Process
Reference: p. 219-220

6. The nursing instructor is teaching a group of student nurses about the current
use of episiotomies during the labor process. The instructor determines the session
is successful when the students correctly choose which situation that may require
the health care provider to perform an episiotomy?
A. persistent occiput anterior position
B. VBAC delivery
C. shoulder dystocia
D. multifetal births

Answer: C
Rationale: An episiotomy is appropriate in certain situations. These include shoulder
dystocia (baby's shoulders are stuck in the birth canal after the head is born); the
head will not rotate from an occiput posterior position (persistent occiput posterior,
not anterior); the fetus is in the breech presentation; or instruments (forceps or
vacuum) are being used to shorten the second stage of labor. A VBAC delivery
would not necessarily require an episiotomy unless one of the above situations was
present. A multifetus birth is also not an automatic indication unless one of the
above situations is present.
Question format: Multiple Choice
Chapter 11: Assisted Delivery and Cesarean Birth
Cognitive Level: Analyze
Client Needs: Health Promotion and Maintenance
Integrated Process: Teaching/Learning
Reference: p. 220

7. The LPN/LVN is caring for a client who is scheduled for a cesarean birth. The
nurse points out to the client that during which stage of the process a different
nurse will be the primary caregiver?
A. preoperative
B. intraoperative
C. immediate postoperative
D. late postoperative

Answer: C

Rationale: In the immediate postoperative phase, the PACU RN will be the primary
manager of her care. The LPN/LVN may be involved in some of the preoperative
care to prepare the woman for surgery, and may also be a part of the
intraoperative care by functioning in the role of scrub nurse. The LPN/LVN may
assume care of the woman during the postoperative phase, after she has
sufficiently recovered from anesthesia.
Question format: Multiple Choice
Chapter 11: Assisted Delivery and Cesarean Birth
Cognitive Level: Apply
Client Needs: Safe, Effective Care Environment: Management of Care
Integrated Process: Nursing Process
Reference: p. 226

8. The pregnant client and her partner have arrived for a scheduled cesarean birth.
Which action should the nurse prioritize?
A. Obtain laboratory specimens.
B. Give prescribed oral antacid.
C. Prep abdomen for surgery.
D. Teach turn, cough, and deep breathing.

Answer: A
Rationale: The client must have blood work drawn before any procedures can be
completed. Once the blood work has been completed, the health care provider will
admit the client and provide orders to the RN. The client may be given IV antacid
prior to surgery if receiving general anesthesia. Shaving the abdomen and learning
how to turn, cough, and deep breathe after surgery is included in preoperative
teaching, which will occur after the client is admitted.
Question format: Multiple Choice
Chapter 11: Assisted Delivery and Cesarean Birth
Cognitive Level: Apply
Client Needs: Safe, Effective Care Environment: Management of Care
Integrated Process: Nursing Process
Reference: p. 229

9. The health care provider has ordered a cesarean birth for an exhausted client
who has been in labor for many hours with the fetus now showing increasing signs
of distress. As the client and partner express disappointment in not having a
natural birth (and anxiety in not knowing what will now happen), which response
will the nurse prioritize?
A. Compare the risks of cesarean birth with those of continuing to attempt vaginal
birth.
B. Calmly and confidently describe the qualifications of the surgical team.
C. Tell them firmly that the client must be prepped for surgery and place an IV and
catheter.
D. Briefly describe what will be experienced, explain each procedure, and
encourage the partner to participate.

Answer: D

Rationale: The client and her partner were not prepared for the possibility of an
operative delivery. The nurse should explain as much as possible and provide
support to them. This is not the time to compare the risks to the mother and the
fetus of continuing to attempt a vaginal birth as compared to a cesarean birth. This
is not the time to talk about the qualifications of the surgical team. A client always
has the right to refuse a procedure. If the nurse places an IV and/or a catheter
without the client's consent, he or she could be arrested for assaulting the client.
Question format: Multiple Choice
Chapter 11: Assisted Delivery and Cesarean Birth
Cognitive Level: Apply
Client Needs: Psychosocial Integrity
Integrated Process: Nursing Process
Reference: p. 230

10. The nurse is preparing a client for an emergent cesarean delivery. Which action
should the nurse prioritize?
A. Sign informed consent.
B. Prep her abdomen.
C. Ensure that a urinary catheter is in place.
D. Record EFM tracing.
Answer: A

Rationale: Before the client can be transported and the cesarean can begin, the
client must sign a consent form. The client should have an EFM from labor, and the
urinary catheter is usually placed once the spinal is completed. The abdomen is
clipped or shaved once the consent has been signed.
Question format: Multiple Choice
Chapter 11: Assisted Delivery and Cesarean Birth
Cognitive Level: Apply
Client Needs: Safe, Effective Care Environment: Management of Care
Integrated Process: Nursing Process
Reference: p. 229-230

11. For the past 18 hours, the client has been progressing slowly in labor in spite of
various attempts to encourage the labor. The health care provider decides a
cesarean birth is necessary to ensure the well-being of both the mother and fetus.
The nurse should point out to the client that this is due to which situation?
A. dystocia
B. time factor
C. prevent uterine rupture
D. development of infection

Answer: A

Rationale: Dystocia is a general term used to describe difficult or abnormal labor.


Dystocia can lead to uterine rupture or the development of an infection of the
uterine membranes. The time factor is one of the criteria for the physician to
determine that the client is having dystocia.
Question format: Multiple Choice
Chapter 11: Assisted Delivery and Cesarean Birth
Cognitive Level: Apply
Client Needs: Health Promotion and Maintenance
Integrated Process: Nursing Process
Reference: p. 218

12. The nursing instructor has completed a session on the induction of labor and
how it is occurring more frequently. The instructor determines the session is
successful when the students correctly choose which factor to be contributing to the
increased induction rates?
A. increase in clients with advanced maternal age
B. elective inductions by choice of both physician and client
C. number of pregnancies at risk for complications
D. increase in high number of multiple gestation pregnancies

Answer: B
Rationale: The number of elective inductions is on the rise and current research
indicates the reason is due to physician and client preference. An elective induction
is one in which the physician and woman decide to induce labor in the absence of a
medical reason to do so. Neither the increase in advanced maternal age, the
number of pregnancies at risk for complications, nor the increase in the number of
multiple gestation pregnancies has impacted the increasing number of elective
inductions.
Question format: Multiple Choice
Chapter 11: Assisted Delivery and Cesarean Birth
Cognitive Level: Apply
Client Needs: Physiological Integrity: Reduction of Risk Potential
Integrated Process: Teaching/Learning
Reference: p. 218

13. The nurse is assessing a new client who is being admitted for induction of labor.
The client is very upset and keeps mentioning that she has an infection. The nurse
interprets this to indicate the client most likely has which condition?
A. chorioamnionitis
B. endometritis
C. peritonitis
D. mastitis

Answer: A

Rationale: Chorioamnionitis is an infection of the fetal membranes and places both


the fetus and mother at risk for morbidity or mortality if the pregnancy continues; it
is one indication for induction of labor. Endometritis is an inflammation of the lining
of the uterus. Cesarean birth, prolonged rupture of membranes, and long labor with
multiple vaginal examinations are important risk factors. Peritonitis is inflammation
of the peritoneum (the serous membrane which lines part of the abdominal cavity
and some of the viscera it contains). Mastitis is an inflammation of the breast.
Question format: Multiple Choice
Chapter 11: Assisted Delivery and Cesarean Birth
Cognitive Level: Apply
Client Needs: Physiological Integrity: Physiological Adaptation
Integrated Process: Nursing Process
Reference: p. 218

14. A client with limited prenatal care presents in labor and is requesting a VBAC.
What is the nurse's response after verifying the client had a classical incision with
the last cesarean birth?
A. "There is no reason to stop you at this point; we will admit you to the unit."
B. "Based on your history, a vaginal delivery is not recommended, it might cause
your uterus to rupture."
C. "We can do a trial of monitored labor, but may have to do a cesarean anyway."
D. "This will be up to your health care provider."

Answer: B
Rationale: The risk for uterine rupture during VBAC is much higher when the
woman has a classical uterine incision from previous cesarean births. VBAC is
contraindicated when this type of scar is present. She should not be allowed to
labor. A client who is in labor will be admitted whether she is having a cesarean
birth or is planning a vaginal birth. However, in the scenario described, the client
should not be encouraged to attempt a vaginal birth. A trial of labor is not
indicated.
Question format: Multiple Choice
Chapter 11: Assisted Delivery and Cesarean Birth
Cognitive Level: Apply
Client Needs: Physiological Integrity: Reduction of Risk Potential
Integrated Process: Nursing Process
Reference: p. 218

15. The health care provider has determined a client should be admitted for
induction of labor and begins the process with cervical ripening overnight. Which
teaching should the nurse prioritize for the client and her partner when describing
this procedure?
A. "The doctor thinks your cervix is not ready for labor and we need to do some
procedures to prepare it for labor."
B. "The cervix needs to be soft and thinning to be induced for labor; this helps
soften the cervix."
C. "Cervical ripening will cause your membranes to rupture and start the labor
process."
D. "Cervical ripening is using medications to start labor until the cervix dilates."

Answer: B

Rationale: The cervix is in need of ripening, or softening for labor, if the provider
feels the woman may need an induction and her cervix is not favorable. The idea of
cervical ripening is to use pharmacologic or nonpharmacologic methods to prepare
the cervix for labor. It will not cause the membranes to rupture, nor will it start the
labor process. The LPN will not be doing any procedures; they will be performed by
the health care provider. The LPN will monitor the client overnight.
Question format: Multiple Choice
Chapter 11: Assisted Delivery and Cesarean Birth
Cognitive Level: Apply
Client Needs: Physiological Integrity: Pharmacological and Parenteral Therapies
Integrated Process: Teaching/Learning
Reference: p. 219

16. The nurse is preparing the client so that the health care provider can ripen the
client's cervix for induction of labor. The nurse predicts the health care provider will
probably use which most commonly used mechanical method?
A. membrane stripping
B. insertion of laminaria
C. PROM
D. Foley bulb with extra-amniotic infusion

Answer: A

Rationale: The most common method of mechanical induction of labor is the use of
membrane stripping. In this instance, the provider will insert a finger between the
sac of waters and the edge of the cervix and sweep 360 degrees to separate the
membranes for the lower uterine segment. The theory is that this procedure
irritates and stimulates the uterus to contract and enter labor. Laminaria, or
cervical dilators, are sometimes used to soften and dilate the cervix. Laminaria are
made from the root of seaweed. A Foley bulb with an extra-amniotic infusion is not
a mechanical method of inducing labor. The premature rupture of membranes
(PROM) may be used to help induce labor but not cervical ripening.
Question format: Multiple Choice
Chapter 11: Assisted Delivery and Cesarean Birth
Cognitive Level: Apply
Client Needs: Physiological Integrity: Pharmacological and Parenteral Therapies
Integrated Process: Nursing Process
Reference: p. 219

17. The client has been progressing well through the labor process and the health
care provider prepares to deliver the infant and performs an episiotomy. The nurse
predicts which situation is the reason for this procedure?
A. prior episiotomy with last child
B. shoulder dystocia
C. client request so she does not tear
D. previous perineal laceration with her last child

Answer: B

Rationale: There is no reason to perform an episiotomy because of having had a


prior one, nor because of a prior laceration with a child. Each delivery is different
and the same may not be needed with this birth. The provider should not perform
one on client request. The common indication is a "stuck" baby, one with shoulder
dystocia or difficulty exiting due to a tight vaginal and perineal area.
Question format: Multiple Choice
Chapter 11: Assisted Delivery and Cesarean Birth
Cognitive Level: Apply
Client Needs: Health Promotion and Maintenance
Integrated Process: Nursing Process
Reference: p. 220-221

18. The nursing instructor is leading a group discussion on the various types of
birth. The instructor determines the session is successful when the students
correctly categorize forceps or vacuum-assisted birth as what type?
A. spontaneous vaginal birth
B. failed vaginal birth
C. surgical birth
D. operative vaginal birth

Answer: D

Rationale: The use of instruments during birth to remove the fetus is invasive and
with multiple risks. This type of birth is termed an operative vaginal birth (a baby
was born vaginally with assistance). A spontaneous vaginal birth is one in which the
baby is born without the assistance of the health care provider. A failed vaginal
birth is one in which cesarean birth becomes necessary. A surgical birth is a
cesarean birth.
Question format: Multiple Choice
Chapter 11: Assisted Delivery and Cesarean Birth
Cognitive Level: Apply
Client Needs: Health Promotion and Maintenance
Integrated Process: Teaching/Learning
Reference: p. 222

19. The nurse is conducting a prenatal class for a group of pregnant women and
their partners. When illustrating the various potential complications that can
necessitate a cesarean birth, which primary reason should the nurse point out?
A. placenta previa
B. ruptured uterus
C. nonreassuring fetal status
D. preeclampsia

Answer: C

Rationale: The main reasons for performing cesarean births include a history of
previous cesarean or other uterine incision; labor dystocia; nonreassuring fetal
status; and fetal malpresentation. Less common indications include placenta previa,
ruptured uterus, and preeclampsia.
Question format: Multiple Choice
Chapter 11: Assisted Delivery and Cesarean Birth
Cognitive Level: Apply
Client Needs: Health Promotion and Maintenance
Integrated Process: Teaching/Learning
Reference: p. 223-224

20. A client has been admitted for induction of labor and is asking the nurse what
happens next. The nurse should point out that the health care provider will conduct
which evaluation to determine what happens next?
A. fetal fibronectin
B. Bishop score system
C. endovaginal ultrasound
D. validate gestational age

Answer: B
Rationale: The Bishop score is a method used to determine cervical readiness for
labor. Five factors are evaluated: cervical consistency, position, dilation (dilatation),
effacement, and fetal station. The fetal fibronectin and an endovaginal ultrasound
to determine the cervical size have not been found to be reliable indications of
cervical readiness. Determining the gestational age will also not provide information
concerning the readiness of the cervix for delivery.
Question format: Multiple Choice
Chapter 11: Assisted Delivery and Cesarean Birth
Cognitive Level: Apply
Client Needs: Health Promotion and Maintenance
Integrated Process: Nursing Process
Reference: p. 218

21. At 0500 hrs, a client was started on oxytocin. The nurse notes on assessment
the client is dilated to 4 cm with contractions every 1 minute and increased signs of
fetal distress. What action should the nurse prioritize after noting the time is now
1200 hrs?
A. Administer pain medication.
B. Increase IV fluids.
C. Stop the oxytocin infusion.
D. Notify the health care provider.

Answer: C

Rationale: The drug oxytocin is used for induction of labor. It is a uterine stimulant.
In this client, the uterus is showing signs of overstimulation. The first intervention
is to stop the infusion. The nurse should then contact the health care provider for
further instructions. It would be inappropriate to administer pain medications or
increase the IV fluids.
Question format: Multiple Choice
Chapter 11: Assisted Delivery and Cesarean Birth
Cognitive Level: Apply
Client Needs: Physiological Integrity: Pharmacological and Parenteral Therapies
Integrated Process: Nursing Process
Reference: p. 219-220

22. The nurse is admitting a client who is to have labor induced at 37 weeks'
gestation. When asked by the client what the fibronectin test is for, what is the
nurse's best response?
A. to view the cells of the cervix for changes
B. to look for amniotic fluid ferning patterns
C. to check cervical secretions for the substance associated with readiness
D. to assess the lung maturity of the fetus before inducing at 37 weeks

Answer: C

Rationale: A fetal fibronectin test checks cervical secretions for fibronectin and if
present, is associated with cervical readiness for labor. The L/S test is performed to
assess for surfactant, which will reveal the maturity of the lungs of the fetus. The
ferning test is performed to determine if the amniotic membranes have ruptured.
The cells are not usually viewed to determine their readiness for delivery.
Question format: Multiple Choice
Chapter 11: Assisted Delivery and Cesarean Birth
Cognitive Level: Apply
Client Needs: Health Promotion and Maintenance
Client Needs: Physiological Integrity: Physiological Adaptation
Integrated Process: Nursing Process
Reference: p. 218

23. The nurse is preparing a client for administration of prostaglandin E2 to prepare


for induction of labor. The nurse should explain this is administered via which route
to the client?
A. rectal
B. venous
C. oral
D. vaginal

Answer: D

Rationale: Prostaglandin E2 is a medication placed in the vagina against the cervix


in either a gel form or via a vaginal insert. The preparation stays in place until
spontaneous labor ensues. Prostaglandin E1 can be administered either orally or
vaginally. Neither of these medications is administered rectally or via an IV.
Question format: Multiple Choice
Chapter 11: Assisted Delivery and Cesarean Birth
Cognitive Level: Apply
Client Needs: Physiological Integrity: Pharmacological and Parenteral Therapies
Integrated Process: Nursing Process
Reference: p. 219

24. A health care provider plans to use membrane stripping to ripen the cervix of a
pregnant woman who is undergoing labor induction. The nurse determines that the
client understands what will happen with this procedure based on which statement?
A. "My provider will use a catheter with a balloon filled with salt water to dilate my
cervix."
B. "A seaweed-like substance put into my cervix will expand and cause my cervix to
dilate."
C. "I will have a gel-type medicine put directly on my cervix."
D. "My provider will separate the thin layers of tissue from my lower uterus."

Answer: D

Rationale: The client understands what will happen by the statement about the
provider separating the thin tissues from the uterus. Membrane stripping is one of
the most common mechanical methods used to hasten cervical readiness. With this
technique, the health care provider inserts a gloved finger through the internal
cervical os and sweeps the finger 360 degrees to separate the membranes from the
lower uterine segment. Other methods to ripen the cervix may be used. These
include dilating the cervix using a catheter with the balloon filled with sterile saline,
inserting laminaria (a seaweed-like substance) that absorbs moisture and expands
to dilate the cervix, and using pharmacologic agents such as prostaglandins applied
locally to the cervix.
Question format: Multiple Choice
Chapter 11: Assisted Delivery and Cesarean Birth
Cognitive Level: Understand
Client Needs: Health Promotion and Maintenance
Integrated Process: Nursing Process
Reference: p. 219
Chapter 12
1. The nurse is caring for several postpartum clients on the unit. Which client's
reaction should the nurse prioritize for possible intervention?
A. Hesitates to hold newborn, expressing disappointment with baby's appearance.
B. Neglects to engage or provide care or show interest in infant.
C. Tearful for several days, difficulty eating and sleeping.
D. Express doubt in ability to care for newborn.

Answer: B

Rationale: A mother not bonding with the infant or showing disinterest is a cause
for concern and requires a referral or notification of the primary health care
provider. Some mothers hesitate to take their newborn and express disappointment
in the way the baby looks, especially if they want a child of one sex and have a
child of the opposite sex. Expressing doubt about the ability to care for the baby is
not unusual, and being tearful for several days with difficulty eating and sleeping is
common with postpartum blues.
Question format: Multiple Choice
Chapter 12: The Postpartum Woman
Cognitive Level: Apply
Client Needs: Psychosocial Integrity
Integrated Process: Nursing Process
Reference: p. 244-245

2. The nurse is monitoring a client who is recovering from a cesarean birth with
spinal anesthesia. Which sign or symptom should the nurse prioritize if noted on
assessment after the administration of morphine sulfate, simethicone, and
diphenhydramine?
A. intense itching manifested by scratching
B. abdominal distention and pain
C. difficulty coughing and turning
D. slow respiration, less than 12 breaths per minute

Answer: D

Rationale: Based on the history of medications, this client may be in respiratory


compromise secondary to medications, most likely the morphine sulfate. The CRNA
or anesthesiologist needs to be notified immediately and the nurse should prepare
to administer naloxone. If the client demonstrates intense itching manifested by
scratching the nurse would give the ordered diphenhydramine. The simethicone has
been ordered for abdominal distension and pain. Difficulty coughing and turning are
usually because of pain at the surgical site, and a client with these symptoms would
need support and encouragement from the nurse.
Question format: Multiple Choice
Chapter 12: The Postpartum Woman
Cognitive Level: Apply
Client Needs: Physiological Integrity: Pharmacological and Parenteral Therapies
Integrated Process: Nursing Process
Reference: p. 256-257

3. The nurse is preparing a client for discharge and notes an order for rubella
vaccine. Which teaching should the nurse prioritize?
A. Advise the client that the vaccine is excreted in breast milk.
B. Do not to attempt another pregnancy for at least 3 months.
C. May experience rash, sore throat, headache, or general malaise within 2 to 4
weeks of the injection.
D. Will prevent hemolytic disease of the infant in next pregnancy.

Answer: B

Rationale: The nurse should prioritize the fact that after the immunization, she
needs to wait for at least 3 months before attempting to get pregnant again, if
desired, so the fetus will not be exposed to the rubella vaccination. The rubella
vaccine is a live virus and is considered teratogenic. The other choices are not
priorities. Inform the breastfeeding woman that the rubella vaccine crosses over
into the breast milk. The newborn benefits from short-term immunity but may
become flushed, fussy, or develop a slight rash. Suggest that the woman speak to
the pediatrician if she has concerns. The client may also experience a rash, sore
throat, headache, and general malaise within 2 to 4 weeks after the injection. The
nurse would not tell the new mother that the immunization will prevent hemolytic
disease of the infant in her next pregnancy; this is incorrect information.
Question format: Multiple Choice
Chapter 12: The Postpartum Woman
Cognitive Level: Apply
Client Needs: Physiological Integrity: Pharmacological and Parenteral Therapies
Integrated Process: Nursing Process
Reference: p. 259

4. The nurse is conducting a postpartum examination on a client who reports pain


and is unable to sit comfortably. The perineal exam reveals an episiotomy without
signs of a hematoma. Which action should the nurse prioritize?
A. Notify a health care provider.
B. Apply a warm washcloth.
C. Place an ice pack.
D. Put on a witch hazel pad.

Answer: C

Rationale: The labia and perineum may be bruised and edematous after birth; the
use of ice would assist in decreasing the pain and swelling. Applying a warm
washcloth would bring more blood as well as fluid to the sore area, thereby
increasing the edema and the soreness. Applying a witch hazel pad needs the order
of the health care provider. Notifying a health care provider is not necessary at this
time as this is considered a normal finding.
Question format: Multiple Choice
Chapter 12: The Postpartum Woman
Cognitive Level: Apply
Client Needs: Physiological Integrity: Physiological Adaptation
Integrated Process: Nursing Process
Reference: p. 251

5. The nursing instructor is leading a discussion on the physical changes to a


woman's body after the birth of the baby. The instructor determines the session is
successful after the students correctly point out which process results in the return
of nonpregnant size and function of the female organs?
A. Evolution
B. Involution
C. Decrement
D. Progression

Answer: B

Rationale: Involution is the term used to describe the process of the return to
nonpregnancy size and function of reproductive organs. Evolution is change in the
genetic material of a population of organisms from one generation to the next.
Decrement is the act or process of decreasing. Progression is defined as movement
through stages such as the progression of labor.
Question format: Multiple Choice
Chapter 12: The Postpartum Woman
Cognitive Level: Analyze
Client Needs: Physiological Integrity: Physiological Adaptation
Integrated Process: Teaching/Learning
Reference: p. 241

6. The primigravida client is surprised by the continued uterine contractions while


holding her newborn. Which explanation by the nurse explains the primary reason
the contractions occur?
A. Returns the uterus to normal size
B. Seals off the blood vessels at the site of the placenta
C. Stops the flow of blood
D. Closes the cervix

Answer: B

Rationale: The contractions of the uterus help to constrict the vessels where the
placenta was located. This does decrease the flow of blood but is secondary in
occurrence to the constriction of the blood vessels. Uterine contraction also leads to
uterine involution, which normally occurs at a predictable rate. Uterine involution
assists in closing the cervix. Again, the other options are secondary to the
constriction of blood vessels at the placental site.
Question format: Multiple Choice
Chapter 12: The Postpartum Woman
Cognitive Level: Apply
Client Needs: Health Promotion and Maintenance
Integrated Process: Nursing Process
Reference: p. 241

7. The nurse is teaching a discharge session to a group of postpartum clients. When


asked how long to expect the bleeding, which time frame should the nurse point
out?
A. For 6 weeks
B. On and off for 2 to 3 weeks
C. Stops in 1 to 2 weeks
D. In approximately 10 days

Answer: D

Rationale: The lochia may persist for 6 weeks after delivery; however, it should
change in color and amount. For 3 to 4 days, lochia rubra is present in small to
moderate amounts, mostly blood and dark red in color. Lochia serosa is present on
days 4 to 10, decreasing to small amounts, brownish to pinkish in color. The lochia
alba is present after 10 days and becomes white or pale yellow because the
bleeding has stopped. This may continue until the end of the 6 weeks.
Question format: Multiple Choice
Chapter 12: The Postpartum Woman
Cognitive Level: Apply
Client Needs: Health Promotion and Maintenance
Integrated Process: Teaching/Learning
Reference: p. 241

8. The postpartum client and her husband are excited about their new baby.
However, they are also concerned about getting pregnant again too soon and ask
about using birth control. Which instruction should the nurse include in their
discharge education to address this issue?
A. "You may have intercourse until next month with no fear of pregnancy."
B. "Ovulation may return as soon as 3 weeks after birth."
C. "You will not ovulate until your menstrual cycle returns."
D. "Ovulation does not return for 6 months after birth."

Answer: B

Rationale: Ovulation may start at soon as 3 weeks after birth. The client needs to
be aware and use a form of birth control. She needs to be cleared by her health
care provider prior to intercourse if she has a vaginal birth, but in the event that
she has intercourse, needs to be prepared for the possibility of pregnancy.
Ovulation can occur without the return of the menstrual cycle, and ovulation does
return sooner than 6 months after birth.
Question format: Multiple Choice
Chapter 12: The Postpartum Woman
Cognitive Level: Apply
Client Needs: Health Promotion and Maintenance
Integrated Process: Nursing Process
Reference: p. 241-242

9. The nurse is preparing to assess a client who is 1 day postpartum. The nurse
predicts the client's fundus will be at which location on assessment?
A. At level of umbilicus
B. 1 cm above the umbilicus
C. 1 cm below the umbilicus
D. At the symphysis pubis

Answer: C

Rationale: The fundus of the uterus should be at the umbilicus after birth. Every
day after birth it should decrease 1 cm until it is descended below the pubic bone.
Question format: Multiple Choice
Chapter 12: The Postpartum Woman
Cognitive Level: Apply
Client Needs: Health Promotion and Maintenance
Integrated Process: Nursing Process
Reference: p. 241

10. The nurse has obtained the vital signs of a postpartum client 24 hours after
delivery and records: 99°F (37.2°C), RR 18, HR 78, and BP 140/90 mm Hg. Which
action should the nurse prioritize?
A. Notify the RN of the slight elevation in BP.
B. Nothing; the vital signs are within normal limits.
C. Recheck all vital signs in 30 minutes.
D. Recheck only the BP in 30 minutes.

Answer: A

Rationale: The LPN should prioritize notifying the RN immediately for further
assessment of this client. An elevated BP may be a sign the client is developing
preeclampsia after delivery. The other vital signs are within normal limits. Further
assessment of the vital signs should be conducted depending on the client's
situation.
Question format: Multiple Choice
Chapter 12: The Postpartum Woman
Cognitive Level: Apply
Client Needs: Physiological Integrity: Reduction of Risk Potential
Integrated Process: Nursing Process
Reference: p. 243
11. The nurse has completed assessing the vital signs of several clients who are
from 36 to 48 hours postpartum. For which set of vital signs should the nurse
prioritize for interaction?
A. Temp: 99.4° F (37.4° C), HR 90, RR 18, BP 112/67
B. Temp: 97.0° F (36.1° C), HR 80, RR 20, BP 120/72
C. Temp: 100.2° F (38° C), HR 65, RR 22, BP 130/78
D. Temp: 98.6° F (37° C), HR 74, RR 16, BP 150/85

Answer: D

Rationale: Postpartum women may have an elevated temp to 100.4° F (38° C) for
24 hours after birth; they may also have decreased pulse a few weeks after birth.
The elevated BP of 150/85 is a concern, as a postpartum woman is still at risk of
developing preeclampsia even after birth. The other choices are within normal
limits.
Question format: Multiple Choice
Chapter 12: The Postpartum Woman
Cognitive Level: Analyze
Client Needs: Physiological Integrity: Physiological Adaptation
Integrated Process: Nursing Process
Reference: p. 243

12. The nurse is performing an assessment on a 2-day postpartum client and


discovers a boggy fundus at the umbilicus and slightly to the right. The nurse
determines that this is most likely related to which situation?
A. Uterine atony
B. Full bowel
C. Bladder distention
D. Poor bladder tone

Answer: C

Rationale: Most often the cause of a displaced uterus is a distended bladder. Ask
the client to void and then reassess the uterus. According to the scenario described,
the most likely cause of the uterine findings would not be uterine atony. A full
bowel or poor bladder tone would not cause a boggy and displaced fundus.
Question format: Multiple Choice
Chapter 12: The Postpartum Woman
Cognitive Level: Apply
Client Needs: Physiological Integrity: Reduction of Risk Potential
Integrated Process: Nursing Process
Reference: p. 243

13. A woman who gave birth to her infant 1 week ago calls the clinic to report pain
with urination and increased frequency. What response should the nurse
prioritize?
A. "This is normal; give it a few days and then call back."
B. "After birth it is easier to develop an infection in the urinary system; we need to
see you today."
C. "Are you washing and providing good perineal hygiene? If not, this may be the
reason for the irritation."
D. "It is common for women to have yeast problems; try an over the counter cream
and let us know if this continues."

Answer: B

Rationale: The urinary system is more susceptible to infection during the


postpartum period. The woman needs to be checked to rule out a urinary infection.
The other responses are incorrect because they do not acknowledge her in an
appropriate manner.
Question format: Multiple Choice
Chapter 12: The Postpartum Woman
Cognitive Level: Apply
Client Needs: Physiological Integrity: Physiological Adaptation
Integrated Process: Nursing Process
Reference: p. 254-256

14. A client who is 3 days' postpartum calls the office and reports excessive night
sweats. Which explanation should the nurse provide for the client?
A. Change in pregnancy hormone
B. Body secreting the excess fluids from pregnancy
C. The patient may be drinking too much fluid.
D. The body is trying to get rid of the extra blood made during pregnancy.

Answer: B

Rationale: Copious diaphoresis occurs in the first few days after childbirth as the
body rids itself of excess water and waste via the skin. The excessive diaphoresis is
not caused by changes in hormones, nor because of the client drinking too much
fluid, nor because of the body trying to rid itself of the excess blood made during
pregnancy.
Question format: Multiple Choice
Chapter 12: The Postpartum Woman
Cognitive Level: Apply
Client Needs: Health Promotion and Maintenance
Integrated Process: Nursing Process
Reference: p. 243-244

15. The nurse is monitoring a client who is 3 hours postpartum. On assessment, the
nurse notes a temperature of 102.4°F (39.1°C). Which action should the LPN
prioritize?
A. Notify the RN who will then notify the health care provider.
B. Administer an antipyretic.
C. Assist the client in ambulation.
D. Continue to monitor for another hour.
Answer: A

Rationale: A temperature elevated above 100.4°F (38°C) is a sign of possible


infection. The LPN should notify the RN. The RN will then notify the provider and
receive further care orders for the client. Administering an antipyretic can only be
done at the physician's order. Assisting in ambulation and continuing to monitor the
client for another hour are not indicated interventions for this client.
Question format: Multiple Choice
Chapter 12: The Postpartum Woman
Cognitive Level: Apply
Client Needs: Safe, Effective Care Environment: Management of Care
Integrated Process: Nursing Process
Reference: p. 259-260

16. The nurse is assisting a new mother who just transferred from the PACU. The
nurse determines the client has already been adapting to her role as a mother by
performing which actions of the first stage of adaptation?
A. achieving a maternal identity
B. physical restoration and learning to care for infant
C. shift in normal life to "new normal"
D. beginning attachment and preparation for family

Answer: D

Rationale: The first stage is the beginning attachment to the fetus and idea of a
family. This occurs during pregnancy. The four stages include: 1) beginning
attachment and preparation for the infant during pregnancy; 2) increasing
attachment, learning to care for the infant, and physical restoration during the early
postpartum period; 3) moving toward a new normal in the first 4 months; and 4)
achieving a maternal identity around 4 months.
Question format: Multiple Choice
Chapter 12: The Postpartum Woman
Cognitive Level: Apply
Client Needs: Psychosocial Integrity
Integrated Process: Nursing Process
Reference: p. 244

17. The nursing instructor is conducting a class exploring the various changes that
occur in the early postpartum period. The instructor determines the session is
successful when the students correctly point out which definition of bonding?
A. A process of developing an attachment and becoming acquainted with each other
B. The skin-to-skin contact that occurs in the birth room
C. An ongoing process in the year after birth
D. Family growing closer together after the birth of a new baby

Answer: A
Rationale: Bonding in the maternal-newborn world is the attachment process that
occurs between a mother and her newborn infant. This is how the mother and
infant become engaged with each other and is the foundation for the relationship.
Bonding is a process and not a single event. The process of bonding is not a
yearlong process, and the family growing closer together after the birth of a new
baby is not bonding.
Question format: Multiple Choice
Chapter 12: The Postpartum Woman
Cognitive Level: Analyze
Client Needs: Psychosocial Integrity
Integrated Process: Teaching/Learning
Reference: p. 244

18. The nurse is questioning the effective bonding of a client and her 2-day-old
infant after noting signs of impaired bonding and attachment. Which actions does
the nurse find concerning?
A. Making eye contact with the baby
B. Breastfeeding the infant on demand
C. Calling the baby "it" or "they"
D. Asking for assistance changing a diaper

Answer: C

Rationale: Many new parents will need assistance with diaper changes; this is not a
flag for concern. Making eye contact and breastfeeding are positive interaction
behaviors. If the mother calls the baby "it" and does not use the child's name, this
is a sign that further information needs to be gathered and assessments should be
completed.
Question format: Multiple Choice
Chapter 12: The Postpartum Woman
Cognitive Level: Apply
Client Needs: Psychosocial Integrity
Integrated Process: Nursing Process
Reference: p. 244-245

19. The nurse is performing an assessment for a client in the immediate


postpartum period. Which assessment finding should the nurse prioritize?
A. infection
B. dehydration
C. hemorrhage
D. bladder distention

Answer: C

Rationale: If a woman is going to hemorrhage, she is most likely to do so within the


first postpartum hour. The nurse must assess a postpartum client every 15 minutes
for the first hour. Infection, dehydration, and bladder distention are all assessment
parameters but do not take precedence over hemorrhage in the first postpartum
hour.
Question format: Multiple Choice
Chapter 12: The Postpartum Woman
Cognitive Level: Apply
Client Needs: Health Promotion and Maintenance
Integrated Process: Nursing Process
Reference: p. 247-248

20. The nurse is conducting the initial postpartum assessment on a client. The
nurse will assist the client into which position to properly assess the postpartum
uterus?
A. semi-Fowler
B. high Fowler
C. supine
D. left-lateral side lying

Answer: C

Rationale: The best position for a complete assessment of the uterus is lying flat,
supine. The other positions will not allow for a true assessment of the location of
the uterus in relation to the umbilicus.
Question format: Multiple Choice
Chapter 12: The Postpartum Woman
Cognitive Level: Apply
Client Needs: Health Promotion and Maintenance
Integrated Process: Nursing Process
Reference: p. 247-248

21. The nurse is preparing a nursing care plan for an immediate postpartum client.
Which nursing diagnosis should the nurse prioritize?
A. Acute pain related to afterpains or episiotomy discomfort
B. Risk for infection related to multiple portals of entry for pathogens
C. Risk for injury: postpartum hemorrhage related to uterine atony
D. Risk for injury: falls related to postural hypotension and fainting

Answer: C

Rationale: The highest priority is the risk for injury related to postpartum
hemorrhage. The client needs close observation and assessment for hemorrhage.
All of the options presented are appropriate nursing diagnoses for a postpartum
client. However, the other options do not take precedence over the risk for
postpartum hemorrhage.
Question format: Multiple Choice
Chapter 12: The Postpartum Woman
Cognitive Level: Apply
Client Needs: Safe, Effective Care Environment: Management of Care
Integrated Process:
Reference: p. 254

22. The nurse is preparing a postpartum client for discharge 72 hours after birth.
The client reports bilateral breast pain around the entire breast on assessment. The
nurse predicts this is related to which cause after noting the skin is intact and
normal coloration?
A. Mastitis
B. Blocked milk duct
C. Engorgement
D. Excessive oxytocin

Answer: C

Rationale: The client is only 72 hours postbirth and is reporting bilateral breast
tenderness. Milk typically comes in at 72 hours after birth, and with the production
of the milk comes engorgement. Mastitis or blocked milk ducts do not typically
develop until there is fully established breastfeeding. Oxytocin would not be
responsible for this.
Question format: Multiple Choice
Chapter 12: The Postpartum Woman
Cognitive Level: Apply
Client Needs: Health Promotion and Maintenance
Integrated Process: Nursing Process
Reference: p. 248

23. The nurse is preparing discharge for a client who plans to bottle-feed her infant.
Which instruction should the nurse prioritize for this client in the discharge
teaching?
A. Run warm water over the breast in the shower.
B. Massage the breasts when they are painful.
C. Wear a tight, supportive bra.
D. Express small amounts of milk when they are too full.

Answer: C

Rationale: The client trying to dry up her milk supply should do as little stimulation
to the breast as possible. She needs to wear a tight, supportive bra and use ice.
Running warm water over the breasts in the shower will only stimulate the
secretion, and therefore the production, of milk. Massaging the breasts will
stimulate them to expel the milk and therefore produce more milk, as will
expressing small amounts of milk when the breasts are full.
Question format: Multiple Choice
Chapter 12: The Postpartum Woman
Cognitive Level: Apply
Client Needs: Health Promotion and Maintenance
Integrated Process: Teaching/Learning
Reference: p. 251
24. The nurse is preparing discharge teaching for a client who is 2 days
postpartum. Which action should the nurse prioritize to encourage prevention of
constipation?
A. Use a stimulant laxative.
B. Encourage fiber-rich foods.
C. Increase coffee intake.
D. Get plenty of rest.

Answer: B

Rationale: Encouraging fiber-rich foods will help with prevention of constipation.


The client needs plenty of water, needs to ambulate, and should take stool
softeners (not a stimulant laxative) if ordered by the health care provider.
Increasing the coffee intake will not assist with preventing constipation. The client
should get plenty of rest but it should be balanced with activity to help stimulate
peristalsis of the GI tract.
Question format: Multiple Choice
Chapter 12: The Postpartum Woman
Cognitive Level: Apply
Client Needs: Health Promotion and Maintenance
Integrated Process: Teaching/Learning
Reference: p. 262

25. A woman who had a cesarean birth about 12 hours ago reports feeling bloated
and "having gas pains." Which intervention would be helpful at this time?
A. Notify the RN that the woman needs an analgesic.
B. Encourage the woman to get up and walk around.
C. Offer the woman a cup of ice-cold soda.
D. Urge the woman to lie on the right side.

Answer: B

Rationale: After cesarean birth, the woman is at increased risk for gas formation
resulting from decreased peristalsis, the lingering effects of anesthesia and
analgesia, and manipulation of the intestines during surgery. Gas pain is usually not
relieved by analgesics. Many surgeons prescribe simethicone around the clock or
PRN. Frequent and early ambulation stimulates peristalsis and the passing of flatus.
The nurse will instruct the woman to avoid very hot or very cold beverages,
carbonated beverages, chewing gum, and drinking through straws. All of these
actions can increase the formation and discomfort of gas. Other medical
interventions may become necessary, such as rectal suppositories or enemas. The
nurse will encourage the woman to lie on the left side. This position facilitates the
release of gas. Having the woman rock in a rocking chair may also provide comfort
from gas pains.
Question format: Multiple Choice
Chapter 12: The Postpartum Woman
Cognitive Level: Apply
Client Needs: Health Promotion and Maintenance
Integrated Process: Caring
Reference: p. 252
Chapter 13
1. Why are newborns born to diabetic mothers prone to hypoglycemia?
A. Excess subcutaneous fat reduces blood flow to the tissues.
B. Metabolic stress is increased due to the stress on the mother's body.
C. Elevated insulin production metabolized glucose faster.
D. The liver is immature and cannot convert glycogen to glucose.

Answer: C

Rationale: When the mother is diabetic, she has levels of insulin and blood sugars
different from a pregnant woman without diabetes. Therefore the infant/fetus
develops elevated levels of insulin to combat the elevated sugars. The infant is then
at risk of low blood sugar once he or she is born. Infants born to diabetic mothers
do not have excess subcutaneous fat that reduces blood flow to the tissues; they do
not have increased metabolic stress because of stress on the mother's body; and
their immature liver is not the reason the newborn is prone to hypoglycemia.
Question format: Multiple Choice
Chapter 13: Nursing Care During Newborn Transition
Cognitive Level: Remember
Client Needs: Physiological Integrity: Physiological Adaptation
Integrated Process: Nursing Process
Reference: p. 269

2. When providing care for a newborn to a mother who was a smoker during her
pregnancy, the nurse will anticipate the size of the infant to be what?
A. larger than average
B. smaller than average
C. average

Answer: B

Rationale: Smoking decreases the blood flow to the fetus. This is one cause for the
infant to be smaller than average at birth. Mothers who smoke do not generally
have infants who are larger than average; diabetic mothers generally have infants
who are larger than average. An average size infant is not generally born to a
mother who smokes.
Question format: Multiple Choice
Chapter 13: Nursing Care During Newborn Transition
Cognitive Level: Understand
Client Needs: Health Promotion and Maintenance
Integrated Process: Nursing Process
Reference: p. 273
3. When taking the history of her postpartum client and infant couplet, the nurse is
aware which prenatal factor might have resulted in the infant being larger than
average?
A. smoking
B. hypertension
C. gestational diabetes
D. poor maternal weight gain

Answer: C

Rationale: When a mother has gestational diabetes and it's not well controlled, this
can result in having a baby who is large-for- gestational-age or larger than
average. Smoking does not generally result in an infant who is larger than average.
Hypertension and poor weight gain in the mother do not generally result in an
infant who is born larger than average.
Question format: Multiple Choice
Chapter 13: Nursing Care During Newborn Transition
Cognitive Level: Understand
Client Needs: Health Promotion and Maintenance
Integrated Process: Nursing Process
Reference: p. 273

4. The nurse is monitoring an infant who was born at 0515 hrs. At 1315 hrs, the
same day, the nurse determines the infant is starting to show yellowish staining on
the head and face. Which action should the LPN prioritize?
A. Start phototherapy.
B. Document and report to RN.
C. Continue monitoring, report if spreads.
D. Repeat bilirubin levels.

Answer: B

Rationale: Jaundice that appears in the first 24 hours may be a sign of excessive
bilirubin in the blood and is now seeping into the tissues. This needs to be further
evaluated and should be reported to the RN immediately so further assessments,
including lab work, can be ordered. Jaundice in the first 24 hours is considered
pathologic and needs to be evaluated immediately. Physiologic jaundice usually
occurs on the second or third day after birth and is considered a normal event as
the bilirubin levels rise. It should clear up with the use of phototherapy.
Question format: Multiple Choice
Chapter 13: Nursing Care During Newborn Transition
Cognitive Level: Apply
Client Needs: Physiological Integrity: Reduction of Risk Potential
Integrated Process: Nursing Process
Reference: p. 269

5. The nurse is assisting new parents adjust to the birth of their first child. The
parents appear hesitant to pick up the baby, stating they are afraid they will make
the baby cry. What is the best response if the nurse discovers the infant is lying
relatively still with eyes wide open, looking at the parents?
A. Suggest they rock the baby to sleep.
B. Encourage the mother to breastfeed.
C. Commend the parents for making the right choice.
D. Encourage the parents to pick up the baby.

Answer: D

Rationale: Dr. T. Berry Brazelton's Neonatal Behavioral Scale is often used to note
the state of reactivity in newborns. This infant is in the quiet alert state with the
eyes open and attentive to people. There is movement, but limited. This is a good
time for the parents to interact with the infant, such as picking up the infant,
touching, talking, and bonding with the infant. Other states of reactivity include:
• Active alert: eyes are open and active body movements are present,
newborn responds to stimuli with activity
• Deep sleep: quiet, nonrestless sleep state, newborn is hard to awaken
• Light sleep: eyes are closed but more activity is noted, newborn moves
actively and may show sucking behavior
• Drowsy: eyes open and close and the eyelids look heavy, body activity is
present with intermittent periods of fussiness
• Crying: eyes may be tightly closed, thrashing movements are made in
conjunction with active crying.
This would not be the time for the parents to avoid interacting with the infant.
There is also no indication the infant is hungry or tired, so feeding or trying to get
the infant to go to sleep would also be inappropriate at this time.
Question format: Multiple Choice
Chapter 13: Nursing Care During Newborn Transition
Cognitive Level: Apply
Client Needs: Psychosocial Integrity
Integrated Process: Nursing Process
Reference: p. 270

6. A young mother is concerned for her baby and asks the LPN if her baby is okay.
What is the best response if the nurse notes RR 66, nostrils flaring, and grunting
sounds during respiration?
A. "Your baby is fine, just learning how to breathe."
B. "Let's put a blanket around the baby; the baby is cold."
C. "Your baby is having a little trouble breathing. I'll let the RN know."
D. "Your baby is too warm. Let's take the blanket off."

Answer: C

Rationale: The assessment findings discussed are signs of respiratory distress. An


infant with a respiratory rate of greater than 60 with noise requires further
assessment. This does not indicate the infant is either too cold or too warm, so
using or not using a blanket would not be a factor in this scenario.
Question format: Multiple Choice
Chapter 13: Nursing Care During Newborn Transition
Cognitive Level: Apply
Client Needs: Physiological Integrity: Reduction of Risk Potential
Integrated Process: Nursing Process
Reference: p. 272

7. The nurse walks into a client's room and notes a small fan blowing on the mother
as she holds her infant. The nurse should explain this can result in the infant losing
body heat based on which mechanism?
A. Conduction
B. Convection
C. Radiation
D. Evaporation

Answer: B

Rationale: There are four main ways that a newborn loses heat; convection is one
of the four and occurs when cold air blows over the body of the infant resulting in a
cooling to the infant. Conductive heat loss occurs when the newborn's skin touches
a cold surface, causing body heat to transfer to the colder object. Heat loss occurs
by radiation to a cold object that is close to, but not touching, the newborn.
Evaporative heat loss happens when the newborn's skin is wet. As the moisture
evaporates from the body surface, the newborn loses body heat along with the
moisture. The cold air blowing on the infant's skin will cause heat loss.
Question format: Multiple Choice
Chapter 13: Nursing Care During Newborn Transition
Cognitive Level: Apply
Client Needs: Health Promotion and Maintenance
Integrated Process: Nursing Process
Reference: p. 268

8. The nurse is teaching a prenatal class and illustrating some of the basic events
that will happen right after the birth. The nurse should point out which action will
best help the infant maintain an adequate body temperature?
A. Bathe the infant immediately after birth.
B. Place the infant on the mother's abdomen after birth.
C. Wrap the infant in a warm, dry blanket.
D. Turn the temperature up in the birth room.

Answer: C

Rationale: Evaporation is one of the four ways a newborn can lose heat. As
moisture evaporates from the body surface of the infant, the newborn loses heat.
Wrapping the infant in a warm, dry blanket will allow the moisture to be absorbed,
limiting heat loss from evaporation. Bathing the infant will only add to the
evaporative heat loss. The newborn's skin is wet, so placing him on the mother'
abdomen will not prevent evaporation and heat loss. Increasing the ambient
temperature in the birth room does not address the evaporation problem.
Question format: Multiple Choice
Chapter 13: Nursing Care During Newborn Transition
Cognitive Level: Apply
Client Needs: Health Promotion and Maintenance
Integrated Process: Teaching/Learning
Reference: p. 268

9. An infant born via a cesarean birth appears to be transitioning well; however, the
nurse predicts that she will note which common assessment finding in this infant?
A. tachypnea
B. cardiac murmur
C. hypoglycemia
D. hyperthermia

Answer: A

Rationale: The infant born from a cesarean birth has not had the opportunity to exit
the birth canal and experience the squeezing of fluid from the lungs. The lungs
have more amniotic fluid than the lungs of a baby from a vaginal birth and are at
greater risk for respiratory complications, such as tachypnea. An infant born by
cesarean birth is not at increased risk for hyperthermia, hypoglycemia, or a cardiac
murmur.
Question format: Multiple Choice
Chapter 13: Nursing Care During Newborn Transition
Cognitive Level: Apply
Client Needs: Health Promotion and Maintenance
Integrated Process: Nursing Process
Reference: p. 267

10. The nurse is conducting an assessment on a newborn male and the parents
question why the nurse is using a penlight to examine the scrotal sac. The nurse
should point out this helps to eliminate which potential disorder?
A. cryptorchidism
B. hydrocele
C. epispadias
D. phimosis

Answer: B

Rationale: Hydrocele occurs when there is a buildup of fluid in the scrotal sac and
should be noted on assessment. If there is fluid in the scrotal sac, it will be
translucent when the penlight is placed against it. If there isn't fluid, the sac will
remain dark. Cryptorchidism results when the testes do not descend into the scrotal
sac during fetal life. These are checked by putting slight pressure on the scrotal sac
to feel the testes. The urinary meatus should be positioned at the tip of the penis. If
the opening is located abnormally on the dorsal (upper) surface of the glans penis,
the condition is called epispadias. Phimosis, or tightly adherent foreskin, is a normal
condition in the term newborn.
Question format: Multiple Choice
Chapter 13: Nursing Care During Newborn Transition
Cognitive Level: Apply
Client Needs: Health Promotion and Maintenance
Integrated Process: Nursing Process
Reference: p. 279

11. A new mother is learning how to change the diaper on her newborn and
becomes concerned after observing a rash on the trunk of the infant. Which
response should the nurse prioritize?
A. Immediately call the RN or health care provider.
B. Change and bathe the infant.
C. Check all of the baby's vital signs before calling the doctor.
D. Explain this is normal.

Answer: D

Rationale: Erythema toxicum is otherwise known as normal newborn rash. The rash
will resolve without intervention. There is no need to call the RN or health care
provider, change and bathe the infant, or check the vital signs.
Question format: Multiple Choice
Chapter 13: Nursing Care During Newborn Transition
Cognitive Level: Apply
Client Needs: Health Promotion and Maintenance
Integrated Process: Nursing Process
Reference: p. 274

12. The mother calls the nurse to check her baby after noting the right side of the
body is dark red while the left side of the baby is pale. Which question to the
mother should the nurse prioritize when assessing the situation?
A. "How long has it been since you last breastfed?"
B. "Was the baby recently crying?"
C. "Was the baby sleeping on her back?"
D. "Did you hold the baby while she was sleeping?"

Answer: B

Rationale: This is termed Harlequin sign and is related to dilation (dilatation) of


blood vessels often following vigorous crying. This can also happen if the baby is
sleeping on its side instead of its back. The condition will resolve without
intervention. This is not caused by breastfeeding or holding the baby while the
infant is sleeping.
Question format: Multiple Choice
Chapter 13: Nursing Care During Newborn Transition
Cognitive Level: Apply
Client Needs: Health Promotion and Maintenance
Integrated Process: Nursing Process
Reference: p. 274
13. The nurse prepares to give the first bath to a newborn and notes a white
cheese-like substance on the skin. The nurse should document this as which
substance?
A. lanugo
B. milia
C. vernix
D. amniotic fluid

Answer: C

Rationale: Vernix is the coating on the infant that was covering fetal skin to prevent
the skin from the drying effects of amniotic fluid. Lanugo is fine, downy hair that is
present in abundance on the preterm infant but is found in thinning patches on the
shoulders, arms, and back of the term newborn. Milia are frequently found on the
infant's face. These tiny white papules resemble pimples in appearance. Normal
amniotic fluid is not thick and white; it should be clear and give the baby a wet
appearance.
Question format: Multiple Choice
Chapter 13: Nursing Care During Newborn Transition
Cognitive Level: Apply
Client Needs: Health Promotion and Maintenance
Integrated Process: Nursing Process
Reference: p. 274

14. The newborn weighing 6 lb 6 oz (2856 g) now weighs 5 lbs 14 oz (2632 g), 2
days later. Which response should the nurse prioritize to address the mother's
concerns about the weight loss?
A. "We need to do a more in-depth assessment."
B. "This is a normal response."
C. "How often are you feeding your baby?"
D. "You may need to supplement breastfeedings for a while."

Answer: B

Rationale: The infant has a 5% to 10% loss of birth weight during the first few days
of life as the body loses excess fluid and has limited food intake. This physiologic
weight loss amounts to a total loss of 6 to 10 oz. There would be no need to assess
for other problems. It is also not related to feeding, nor would a breastfeeding
mother need to offer supplementary formula feedings.
Question format: Multiple Choice
Chapter 13: Nursing Care During Newborn Transition
Cognitive Level: Apply
Client Needs: Health Promotion and Maintenance
Integrated Process: Nursing Process
Reference: p. 273
15. The nursing instructor is teaching a class on the physiologic prosperities
involved with the birthing process. The instructor determines the session is
successful when the students correctly match surfactant with which function?
A. It expands the lungs with breaths.
B. It keeps alveoli from collapsing with breaths.
C. It removes fluid from the lungs.
D. It allows oxygen to move in the lungs.

Answer: B

Rationale: The role of surfactant is to act on surface tension and assist in keeping
the alveoli open in the lungs so the lungs do not collapse with the respiratory effort
of the newborn. Surfactant does not expand the lungs, remove fluid from the lungs,
or allow oxygen to move in the lungs.
Question format: Multiple Choice
Chapter 13: Nursing Care During Newborn Transition
Cognitive Level: Apply
Client Needs: Physiological Integrity: Physiological Adaptation
Integrated Process: Teaching/Learning
Reference: p. 267

16. The nurse is assessing a newborn by auscultating the heart and lungs. Which
natural phenomenon will the nurse explain to the parents is happening in the
cardiovascular system?
A. Oxygen is exchanged in the lungs.
B. Fluid is removed from the alveoli and replaced with air.
C. Pressure changes occur and result in closure of the ductus arteriosus.
D. The oxygen in the blood decreases.

Answer: C

Rationale: The ductus arteriosus is one of the openings through which there was
fetal circulation. At birth, or within the first few days, this closes and the heart
becomes the source of movement of blood to and from the lungs. The exchange of
oxygen in the lungs and increasing oxygen content in the blood are respiratory
functions. The removal of the fluid from the alveoli occurs mainly during the
birthing process and is completed by the lungs after birth.
Question format: Multiple Choice
Chapter 13: Nursing Care During Newborn Transition
Cognitive Level: Understand
Client Needs: Health Promotion and Maintenance
Integrated Process: Nursing Process
Reference: p. 268

17. The nurse notes a newborn has a temperature of 97.0oF (36.1oC) on


assessment. The nurse acts to prevent which complication first?
A. Seizure
B. Respiratory distress
C. Cardiovascular distress
D. Hypoglycemia

Answer: B

Rationale: It takes oxygen to produce heat and an infant who has an episode of
cold stress is at risk for respiratory distress. The infant needs to be warmed. The
temperature should be in the range of 97.7°F to 98.6°F (36.5°C to 37°C). After
respiratory distress sets in, it can be followed by seizures, cardiovascular distress,
or hypoglycemia.
Question format: Multiple Choice
Chapter 13: Nursing Care During Newborn Transition
Cognitive Level: Apply
Client Needs: Physiological Integrity: Physiological Adaptation
Integrated Process: Nursing Process
Reference: p. 272

18. The parents are concerned their newborn appears to be cold all the time. The
nurse should point out the infant is best helped by which primary method in the
first few days?
A. External blankets put on the infant by the nursing staff
B. Skin-to-skin (kangaroo) care with mother
C. Brown fat store usage
D. Shivering and increased metabolic rate

Answer: C

Rationale: Brown fat stores are used by the newborn infant to maintain warmth
until feeding begins and the infant is able to maintain temperature without
assistance. The infant's thermoregulatory system is not fully functional at birth.
Infants cannot shiver to warm themselves. The use of external blankets as well as
skin-to-skin (kangaroo) care with the mother assists in keeping the baby's
temperature within the normal range, but they are not the primary mechanism for
temperature regulation in the newborn infant.
Question format: Multiple Choice
Chapter 13: Nursing Care During Newborn Transition
Cognitive Level: Apply
Client Needs: Health Promotion and Maintenance
Integrated Process: Nursing Process
Reference: p. 268

19. The nurse notes the following on a newborn's assessment: poor muscle tone,
jitteriness, and temperature 97.0oF (36.1oC), HR 120 bpm, RR 26 breathes per
minute, and blood pressure 60/40 mm Hg. Which nursing action should the nurse
prioritize?
A. Check the infant's temperature again.
B. Complete an entire set of vital signs.
C. Assess the infant's blood sugar.
D. Check oxygen saturation of the blood.

Answer: C

Rationale: The poor muscle tone, low temperature, and jitteriness are signs and
symptoms indicative of hypoglycemia. The nurse should assess the blood glucose
first. Assessing the vital signs and oxygen saturation would be assessed again at
the appropriate time. The main concern at the moment is assessing for
hypoglycemia to prevent further complications.
Question format: Multiple Choice
Chapter 13: Nursing Care During Newborn Transition
Cognitive Level: Apply
Client Needs: Physiological Integrity: Physiological Adaptation
Integrated Process: Nursing Process
Reference: p. 271

20. The nurse receives a report from labor and delivery on an infant and mother
couplet. Which reported Apgar score will the nurse prioritize for close observation
for the entire transition period?
A. 8 at 1 minute; 9 at 5 minutes
B. 7 at 1minute; 8 at 5 minutes
C. 6 at 1 minute; 7 at 5 minutes
D. 5 at 1 minute; 6 at 5 minutes

Answer: D

Rationale: APGAR scores between 4 and 6 at 5 minutes of life indicate a newborn is


having difficulty in adjusting to life outside the womb and needs close observation.
The infant would transition into the nursery under close observation. The other
choices would indicate that the newborn is transitioning with minimal difficulty and
will not need close monitoring for the entire transition period.
Question format: Multiple Choice
Chapter 13: Nursing Care During Newborn Transition
Cognitive Level: Apply
Client Needs: Safe, Effective Care Environment: Management of Care
Integrated Process: Nursing Process
Reference: p. 270

21. The nurse is evaluating the morning blood glucose results from the laboratory
of several 1-day-old infants. Which result should the nurse prioritize for further
action?
A. Infant A - 52 mg/dl
B. Infant B - 56 mg/dl
C. Infant C - 48 mg/dl
D. Infant D - 60 mg/dl

Answer: C
Rationale: Blood glucose levels between 50 and 60 mg/dl during the first 24 hours
of life are considered normal. Levels less than 50 are indicative of hypoglycemia in
the newborn. Infant C is showing potential hypoglycemia. Infants A, B, and D have
values within the normal range.
Question format: Multiple Choice
Chapter 13: Nursing Care During Newborn Transition
Cognitive Level: Analyze
Client Needs: Physiological Integrity: Reduction of Risk Potential
Integrated Process: Nursing Process
Reference: p. 271

22. The LPN is preparing to assist the RN with the initial admissions assessment of
the newborn. The nurse should explain to the new mother that this will be
completed in what time frame after birth?
A. 30 minutes
B. 1 hour
C. 2 hours
D. 4 hours

Answer: C

Rationale: The infant and mother need time for bonding after delivery. While the
nurse is monitoring and may take vital signs, the initial full exam must be
completed within 2 hours of birth. The options of 30 minutes or 1 hour are options
which would be based on the individual situation. Waiting for 4 hours is too long
and may result in danger signs of potential complications being missed.
Question format: Multiple Choice
Chapter 13: Nursing Care During Newborn Transition
Cognitive Level: Understand
Client Needs: Health Promotion and Maintenance
Integrated Process: Nursing Process
Reference: p. 271

23. A new mother of a newborn girl calls the clinic in a panic, concerned about the
blood-tinged soiled diaper. What is the best response from the nurse?
A. "The baby may have a problem; let's schedule an appointment."
B. "This can be related to cleaning her perineal area; be more careful."
C. "This can be from the sudden withdrawal of your hormones. It is not a cause for
alarm."
D. "If this continues, call us back; for now, just watch her."

Answer: C

Rationale: The mother is describing pseudomenstruation and is usually the result of


the infant no longer having the mother's hormones in the body. This is not a cause
for alarm. It is always appropriate to offer to schedule an appointment if the
mother continues to be upset. The nurse should know that the infant's "bleeding" is
not indicative of a pathologic process and should be careful to not upset the mother
further. The statement of it being related to the way the mother is cleaning the
perineum is incorrect for it places the blame on the mother for the infant's problem.
The instruction to call back if it continues does not meet the mother's need to know
why this is happening to her baby, and it negates her concern for her infant.
Question format: Multiple Choice
Chapter 13: Nursing Care During Newborn Transition
Cognitive Level: Apply
Client Needs: Health Promotion and Maintenance
Integrated Process: Nursing Process
Reference: p. 279

24. The LPN assists the RN while performing the Ortolani maneuver on a newborn.
When asked by the mother the reason for this maneuver, which is the best
response from the nurse?
A. spinal column movement
B. shoulder movement
C. clavicles for dislocation
D. hip for dislocation

Answer: D

Rationale: Ortolani maneuver is used to assess the possibility of a dislocated hip in


an infant. Ortolani maneuver does not assess for spinal column movement,
shoulder movement, nor does it assess the clavicles for dislocation. There is no
specific movement to assess for spinal column movement, shoulder movement, or
clavicle dislocation.
Question format: Multiple Choice
Chapter 13: Nursing Care During Newborn Transition
Cognitive Level: Apply
Client Needs: Health Promotion and Maintenance
Integrated Process: Nursing Process
Reference: p. 280

25. The nurse is responding to an infant crying and notes it is very high-pitched and
shrill. The nurse predicts this is most likely related to which situation?
A. normal cry from pain
B. tired and stress from delivery
C. neurologic dysfunction
D. cold stress cry

Answer: C

Rationale: A high-pitched cry that is shrill is associated with a neurologic disorder.


The nurse will need to inform the RN and provider to assess the infant further. A
high-pitched, shrill cry in a newborn is not a normal cry from pain; it does not
indicate the infant is tired and stressed from delivery, and it is not a cry indicating
cold stress.
Question format: Multiple Choice
Chapter 13: Nursing Care During Newborn Transition
Cognitive Level: Apply
Client Needs: Physiological Integrity: Physiological Adaptation
Integrated Process: Nursing Process
Reference: p. 280-281

26. The nurse is conducting an assessment on a newborn and witnesses a startled


response with the extension of the arms and legs. The nurse should document this
as which response?
A. fencing
B. Moro
C. tonic neck
D. rooting

Answer: B

Rationale: The Moro reflex is also known as the startle reflex. When the infant is
startled, he/she extends the arms and legs away from the body. The fencing reflex
is also called the tonic neck reflex and is a total body assessment. The rooting
reflex assesses the infant's ability to "look" for food.
Question format: Multiple Choice
Chapter 13: Nursing Care During Newborn Transition
Cognitive Level: Apply
Client Needs: Health Promotion and Maintenance
Integrated Process: Nursing Process
Reference: p. 281

27. The new mother is holding her infant, speaking softly and gently stroking the
baby's face. She giggles and asks the nurse why the baby turns toward her finger
when she strokes the cheeks. The nurse should explain that this is which common
newborn reflex?
A. Moro
B. tonic neck
C. rooting
D. sucking

Answer: C

Rationale: This is the rooting reflex and is used to encourage the infant to feed.
This reflex and the sucking reflex work together to assist the infant with cues for
feeding at the breast. The tonic neck (or fencing) reflex and the Moro (or startle)
reflex are total body reflexes and assess neurologic function in the newborn.
Question format: Multiple Choice
Chapter 13: Nursing Care During Newborn Transition
Cognitive Level: Apply
Client Needs: Health Promotion and Maintenance
Integrated Process: Nursing Process
Reference: p. 281
28. The nurse is caring for a newborn who was delivered via a planned cesarean
delivery. The nurse determines the infant requires closer monitoring than a vaginal
delivery infant based on which factor?
A. Oxygen was cut off when the umbilical cord was clamped, resulting in decreased
oxygen and increased carbon dioxide.
B. Excessive fluid in its lungs, making respiratory adaptation more challenging.
C. Fetal lungs are uninflated and full of amniotic fluid that must be absorbed.
D. Much of the fetal lung fluid is squeezed out in cesarean delivery.

Answer: B

Rationale: During a vaginal birth the infant is squeezed by uterine contractions,


which squeeze fluid out of the lungs and prepare them for breathing. The infant
who is born via cesarean delivery without labor first does not have the mechanical
removal of the fluid from the lungs. This places the infant at increased risk for
respiratory compromise, so there is a need to more closely assess a newborn after
birth. The lungs should inflate once the baby is delivered and not wait until the
amniotic fluid is absorbed. The umbilical cord is not clamped until the infant is out
of the womb and starts to take its first breaths.
Question format: Multiple Choice
Chapter 13: Nursing Care During Newborn Transition
Cognitive Level: Apply
Client Needs: Health Promotion and Maintenance
Integrated Process: Nursing Process
Reference: p. 267

29. The nurse is assessing a newborn's vital signs and notes the following: HR 138,
RR 42, temperature 98.7oF (37.1oC), and blood pressure 70/40 mm Hg. Which
action should the nurse prioritize?
A. Report tachypnea.
B. Recheck blood pressure in 15 minutes.
C. Put warming blanket over infant.
D. Document normal findings.

Answer: D

Rationale: These vital signs are within normal limits and should be documented.
The heart rate should be 110 to 160 bpm; RR should be 30 to 60 breaths per
minute. The axillary temperature can range from 97.7°F to 99.6 & 176;F (36.5°C to
37.5°C). Blood pressure should be 60 to 80/40 to 45 mm Hg. There is no need to
contact the health care provider, recheck the blood pressure in 15 minutes, or place
a blanket on the infant.
Question format: Multiple Choice
Chapter 13: Nursing Care During Newborn Transition
Cognitive Level: Apply
Client Needs: Health Promotion and Maintenance
Integrated Process: Nursing Process
Reference: p. 272
Chapter 14
1. A newborn infant at 36 hours of age is jaundiced. The mother is breastfeeding.
What intervention is appropriate to increase the excretion of bilirubin?
A. Stop breastfeeding and administer glucose water for 24 hours.
B. Instruct the mom to feed every two to three hours.
C. Restrict feedings and give glucose water every 4-6 hours for hydration.
D. Keep the skin protected by preventing light onto the baby's skin.

Answer: B

Rationale: Bilirubin is excreted in the urine and feces. Encouraging the mother to
breastfeed at least every two to three hours will increase the waste and help
decrease the bilirubin level. Stopping breastfeeding and administering glucose
water for 24 hours would not be appropriate for the mother. Restricting feedings
and giving glucose water every 4 to 6 hours is not an appropriate nursing
intervention for an infant showing signs of jaundice. Keeping light away from the
baby's skin does not help to clear jaundice; it could only make it worse.
Question format: Multiple Choice
Chapter 14: Nursing Care of the Normal Newborn
Cognitive Level: Apply
Client Needs: Health Promotion and Maintenance
Integrated Process: Teaching/Learning
Reference: p. 291

2. A 30-minute-old newborn starts crying in a high-pitched manner and cannot be


consoled by the mother. Which action should the nurse prioritize if jitteriness is
also noted and the infant is unable to breastfeed?
A. Check blood glucose.
B. Place child in a radiant warmer.
C. Assess for pain source.
D. Assess the baby's temperature.

Answer: A

Rationale: One of the primary signs/symptoms of hypoglycemia in newborn infants


is jitteriness and irritability. Anytime an infant is suspected of having hypoglycemia,
the nurse needs to check the blood glucose level. Cold stress and pain are potential
considerations to rule out if hypoglycemia is not the cause; however, jitteriness is
not a recognized sign of these.
Question format: Multiple Choice
Chapter 14: Nursing Care of the Normal Newborn
Cognitive Level: Apply
Client Needs: Physiological Integrity: Reduction of Risk Potential
Integrated Process: Nursing Process
Reference: p. 288-289
3. The nurse is caring for a newborn after the parents have spent time bonding. As
the nurse performs the assessment and evidence-based care, which eye care will
the nurse prioritize?
A. Instill 0.5% ophthalmic silver nitrate.
B. Instill 0.5% ophthalmic tetracycline.
C. Instill 0.5% ophthalmic erythromycin.
D. Watch for signs of eye irritation.

Answer: C

Rationale: The standard eye care to prevent ophthalmia neonatorum is 0.5%


erythromycin ointment or 1% tetracycline eye drops. Although 1% silver nitrate
drops were once used, it has been discontinued due to its ineffectiveness. The
nurse would not wait to see if the eyes show signs of irritation before administering
the medication. Delaying could lead to preventable blindness.
Question format: Multiple Choice
Chapter 14: Nursing Care of the Normal Newborn
Cognitive Level: Apply
Client Needs: Physiological Integrity: Pharmacological and Parenteral Therapies
Integrated Process: Nursing Process
Reference: p. 289-290

4. The nurse is teaching a prenatal class illustrating the steps that are used to keep
families safe. The nurse determines the session is successful when the parents
correctly choose which precaution to follow after the birth of their infant?
A. Send a family member to accompany the infant when leaving the room.
B. Check the name on the baby's identification bracelet.
C. Provide a list of approved visitors who came spend time with the infant.
D. Check the identification badge of any health care worker before releasing baby
from room.

Answer: D

Rationale: Infant abduction is a concern, and all personnel should wear


identification badges and introduce themselves to the parents before they enter the
room or take the infant. If at any time the mother is suspicious, she has the right to
not allow an individual to take the infant. There may not always be a family
member around to accompany the infant and they may not be allowed admittance
to treatment rooms or other such areas. Checking the name on the baby's
identification bracelet will not stop abduction. Providing a list of approved visitors
may help prevent abduction from individuals outside the medical facility but will not
stop someone posing as an employee from abducting the infant.
Question format: Multiple Choice
Chapter 14: Nursing Care of the Normal Newborn
Cognitive Level: Apply
Client Needs: Safe, Effective Care Environment: Safety and Infection Control
Integrated Process: Nursing Process
Reference: p. 292

5. The nurse is preparing to assess the pulse on a newborn who has just arrived to
the nursery after being cleaned in the labor and birth suite and swaddled in a
blanket. Which action should the nurse prioritize?
A. Perform a 3-minute surgical-type scrub.
B. Wear clean gloves.
C. Use infection transmission precautions.
D. Clean hands with a betadine scrub.

Answer: B

Rationale: Infection control is a priority nursing intervention. Gloves need to be


worn when in contact with the infant who has not been bathed after birth. All
options are valid options; however, a three-minute surgical scrub is generally only
required at the beginning of a shift. The nurse should always wash the hands before
putting on gloves to care for an infant and after taking gloves off. Standard
precautions are used with every client.
Question format: Multiple Choice
Chapter 14: Nursing Care of the Normal Newborn
Cognitive Level: Apply
Client Needs: Safe, Effective Care Environment: Safety and Infection Control
Integrated Process: Nursing Process
Reference: p. 292

6. The nurse is giving a newborn his first bath. What should the nurse prioritize?
A. Give the sponge bath in the baby's bed.
B. Wash off all traces of blood and leave the vernix in place.
C. Use a soap such as hexachlorophene to prevent infection.
D. Apply talcum powder to the buttocks after washing.

Answer: B

Rationale: During the bath, all blood and products from the delivery need to be
washed off and the vernix should be left in place to allow it to gradually absorb into
the infant's body. The infant's first bath is given under a radiant warmer, not in the
infant's bed. If a radiant warmer is not used, the nurse should keep the infant
wrapped and expose only the part which is being washed to avoid cold stress. If
soap is used at all, it is a mild soap, not a soap like hexachlorophene. No special
ointments are necessary on the perineal area and buttocks after washing. Talcum
powder is contraindicated because of the possibility of respiratory distress.
Question format: Multiple Choice
Chapter 14: Nursing Care of the Normal Newborn
Cognitive Level: Apply
Client Needs: Health Promotion and Maintenance
Integrated Process: Nursing Process
Reference: p. 293
7. The nurse is teaching discharge instructions to the young parents of a healthy
newborn boy whose vital signs are stable and whose circumcision appears clean
and intact. The nurse should encourage the parents to call the health care provider
if which situation is discovered?
A. A yellowish crusty substance on the circumcision site
B. Crying for 2 hours or more each day
C. Redness at the base of the umbilical cord
D. Straining when he is passing stools

Answer: C

Rationale: The cord should dry and fall off in the 7 to 10 days after birth. If the cord
base changes color or develops drainage, the health care provider needs to be
notified as these could be signs of infection. A yellowish crusty substance on the
circumcision site indicates normal healing. Crying for 2 hours or more each day and
straining during bowel movements are normal in a newborn.
Question format: Multiple Choice
Chapter 14: Nursing Care of the Normal Newborn
Cognitive Level: Apply
Client Needs: Health Promotion and Maintenance
Integrated Process: Nursing Process
Reference: p. 292

8. The nurse is teaching new parents how to clear the secretions from their infant's
mouth and nose. The nurse determines they are prepared when they correctly
perform which initial step?
A. Position the newborn on side, and suction with a bulb syringe.
B. Position the newborn on side with head slightly below body; use a bulb syringe
to clear mouth.
C. Position the newborn on side with head slightly below body; use a small suction
catheter to clear nose.
D. Position the newborn on side with head slightly below body; use a bulb syringe
to clear nose.

Answer: B

Rationale: The infant needs to have bulb suction used to remove the secretions
from the mouth first; the head should be held slightly lower than the body to
facilitate use of gravity. A bulb syringe, not a small suction catheter, is used to
suction the mouth and nose of a newborn. The mouth should be cleared first to
prevent possible aspiration of secretions. Suctioning the nose first could cause the
infant to inhale the secretions in the mouth.
Question format: Multiple Choice
Chapter 14: Nursing Care of the Normal Newborn
Cognitive Level: Apply
Client Needs: Health Promotion and Maintenance
Integrated Process: Teaching/Learning
Reference: p. 299
9. The parents of a newborn are concerned that the different procedures are
causing pain for their newborn. Which action should the nurse prioritize to address
the parent's concerns?
A. Teach the infant's caregivers ways to soothe and comfort the child during any
episode of pain.
B. Adhere carefully to the plan for administration of any analgesics to the child after
procedures.
C. Advocate and use effective treatment methods that cause no pain or less pain.
D. Provide a soothing environment, swaddling and holding the newborn
experiencing pain.

Answer: C

Rationale: It is the ethical responsibility of the nurse to first prevent pain, and if
that is not possible, then to treat pain. The nurse should advocate to the provider
to provide the same. The other options are methods to treat the pain but would not
necessarily prevent pain.
Question format: Multiple Choice
Chapter 14: Nursing Care of the Normal Newborn
Cognitive Level: Apply
Client Needs: Health Promotion and Maintenance
Integrated Process: Caring
Reference: p. 295

10. The nurse is escorting the newborn to the transition nursery for the initial
assessment and care. The nurse is prepared to carefully monitor the infant during
the transition period, which occurs at which time interval?
A. First 12 to 24 hours
B. First 6 to 12 hours
C. First 1 to 6 hours
D. First 1 to 12 hours

Answer: B

Rationale: The first 6 to 12 hours after birth are the critical transition hours for a
newborn. The newborn may stay with the mother, but under close observation by a
nurse. The newborn requires close monitoring throughout the entire period but the
first 6 to 12 hours are the time when more complications may present and must be
handled early to prevent long-term complications.
Question format: Multiple Choice
Chapter 14: Nursing Care of the Normal Newborn
Cognitive Level: Apply
Client Needs: Health Promotion and Maintenance
Integrated Process: Nursing Process
Reference: p. 2587
11. The parents of a 1-day-old newborn are concerned the newborn is cold. Which
action should the nurse prioritize to best prevent heat loss?
A. Keep the newborn under the radiant heater when not with mother.
B. Cover the newborn with several blankets while under the warmer.
C. Warm all surfaces and objects that come in contact with the newborn.
D. Bathe and wash the newborn when temperature is 97.5° F (36.4° C).

Answer: C

Rationale: The 1-day-old newborn will have regulated body temperature at this
point in life, and the radiant heater is no longer used. Interventions are the best
way to prevent heat loss for this newborn; these would include making sure
surfaces such as scales, examination tables and instruments are warm. Keeping the
newborn under a radiant heater and covering the newborn with several blankets
while under the warmer could lead to hyperthermia, which can be just as
detrimental to the newborn as hypothermia. Newborns are bathed when their
temperatures are stable.
Question format: Multiple Choice
Chapter 14: Nursing Care of the Normal Newborn
Cognitive Level: Apply
Client Needs: Physiological Integrity: Reduction of Risk Potential
Integrated Process: Nursing Process
Reference: p. 288

12. The parents are bonding with their newborn when the nurse notes the infant's
axillary temperature is 97.2oF (36.2oC) an hour after birth. Which intervention
should the nurse prioritize for this family?
A. Help the mother provide skin-to-skin (kangaroo) care.
B. Place a second stockinette on the baby's head.
C. Administer a warm bath with temperature slightly higher than usual.
D. Place the infant under a radiant warmer.

Answer: A

Rationale: The nurse should encourage bonding to continue. One way to help the
infant get warm is to help the parents provide kangaroo care, which involves skin-
to-skin contact and parent/baby coverage with blankets. Once the infant is taken
for the initial assessment, placement under the radiant heater would then be
appropriate. Placing a second stockinette is a potential option; however, it would
not be as effective as the skin-to-skin contact. The bath would not be undertaken
until the infant's temperature is stabilized within the normal range.
Question format: Multiple Choice
Chapter 14: Nursing Care of the Normal Newborn
Cognitive Level: Apply
Client Needs: Physiological Integrity: Physiological Adaptation
Integrated Process: Nursing Process
Reference: p. 288
13. The nurse has completed an assessment on a 1-day-old newborn. Which finding
should the nurse prioritize?
A. temperature of 97.8°F (36.5°C)
B. heart rate 158 bpm
C. respiratory rate 42 breaths/min
D. Blood sugar 42 mg/dl

Answer: D

Rationale: Any blood sugar lower than 50 mg/dl is considered hypoglycemic and
should be further assessed. In the scenario described, the infant's temperature,
heart rate, and respiratory rate are all considered within normal limits.
Question format: Multiple Choice
Chapter 14: Nursing Care of the Normal Newborn
Cognitive Level: Apply
Client Needs: Physiological Integrity: Reduction of Risk Potential
Integrated Process: Nursing Process
Reference: p. 288

14. The nurse is preparing discharge instructions for the parents of a male newborn
who is to be circumcised before discharge. Which instruction should the nurse
prioritize?
A. Soak the penis daily in warm water.
B. Cover the glans generously with petroleum jelly.
C. Cleanse the glans daily with alcohol.
D. Notify the primary care provider if it appears red and sore.

Answer: B

Rationale: Covering the surgical site with an ointment such as petroleum jelly
prevents it from adhering to the diaper and being continually irritated. Normal
appearance is red and raw. Soaking the penis daily in warm water is not
recommended. Washing the penis with warm water, dribbled gently from a
washcloth at each diaper change, is the recommended way of keeping the penis
clean. The nurse would not tell the parents to use alcohol on the glans.
Question format: Multiple Choice
Chapter 14: Nursing Care of the Normal Newborn
Cognitive Level: Apply
Client Needs: Physiological Integrity: Reduction of Risk Potential
Integrated Process: Teaching/Learning
Reference: p. 300

15. The nurse is conducting a prenatal class explaining the various activities that
will occur within the first 4 hours after birth. The nurse determines the session is
successful when the couples correctly choose which reason for the use of an
antibiotic ointment?
A. Prevent infection of the umbilical cord
B. Prevent infection of the eyes from vaginal bacteria
C. Protect tear ducts from vaginal bacteria
D. Protect the urethra from fecal material

Answer: B

Rationale: Antibiotic ointment is used in the infant's eyes at birth to prevent


ophthalmia neonatorum, an infection which can lead to blindness. It is not an
acceptable practice to apply antibiotic ointment to the tear ducts, the umbilical
cord, or the perineum and urethra.
Question format: Multiple Choice
Chapter 14: Nursing Care of the Normal Newborn
Cognitive Level: Apply
Client Needs: Physiological Integrity: Pharmacological and Parenteral Therapies
Integrated Process: Teaching/Learning
Reference: p. 289

16. The nurse is explaining to the new parents the various substances which will be
administered to their newborn within a few hours of birth. Which explanation should
the nurse prioritize as the best rationale for administering vitamin K?
A. Provides blood clotting factors
B. Speeds up drying of umbilical stump
C. Stimulates population of normal gut flora
D. Reduces complications from possible circumcision

Answer: A

Rationale: Vitamin K is necessary in the formation of certain clotting factors. The


newborn male and female are both lacking in vitamin K, and the only method for
the infant to receive it in the early hours after birth is to administer the vitamin IM.
Vitamin K is manufactured by normal flora in the gut. Since the newborn has not
yet eaten, there is no normal flora in the gut, so the infant cannot manufacture
vitamin K. Vitamin K does not speed up drying of the umbilical stump or stimulate
population of normal gut flora. It can help reduce possible bleeding from
circumcision; however, that is not the only complication that can occur from a
circumcision, nor is a circumcision the only reason an infant might bleed.
Question format: Multiple Choice
Chapter 14: Nursing Care of the Normal Newborn
Cognitive Level: Apply
Client Needs: Physiological Integrity: Pharmacological and Parenteral Therapies
Integrated Process: Teaching/Learning
Reference: p. 290

17. The nurse is preparing new parents and their infant for discharge by answering
questions and presenting basic discharge instruction. Which explanation should the
nurse provide when questioned about the infant's yellow hue?
A. The tint is due to jaundice.
B. Yellow is the normal color for some newborns.
C. The infant needs to be in the sunlight to clear the skin.
D. It's a mild reaction to the vitamin K injection.

Answer: A

Rationale: Newborns often have a yellow tint to the skin if the newborn develops
jaundice. Any newborn developing jaundice needs to be assessed by the health care
provider. Jaundice in the first 24 hours is pathologic and must be reported
immediately to the health care provider. Jaundice after 2 days is considered
physiologic and is due to the liver's inability to adequately process bilirubin which
seeps into the tissues, giving the skin a yellowish color. It is not considered normal
and does require assessment and intervention. Phototherapy is the recommended
treatment of choice, not putting the child in sunlight. It is not a reaction to the
vitamin K injection.
Question format: Multiple Choice
Chapter 14: Nursing Care of the Normal Newborn
Cognitive Level: Apply
Client Needs: Physiological Integrity: Physiological Adaptation
Integrated Process: Teaching/Learning
Reference: p. 302

18. The young mother is nervous about discharge with her first child. The nurse
encourages the mother by pointing out various instructions, including to call her
health care provider if the newborn does not void within which time period?
A. 6 hours
B. 12 hours
C. 18 hours
D. 24 hours

Answer: B

Rationale: The mother should call the health care provider if her infant does not
void at all within a 12-hour period. The mother should not wait 18 to 24 hours
before calling. It is fine for the infant to go 6 hours without voiding; however, the
mother should be aware that this needs to be monitored.
Question format: Multiple Choice
Chapter 14: Nursing Care of the Normal Newborn
Cognitive Level: Apply
Client Needs: Health Promotion and Maintenance
Integrated Process: Nursing Process
Reference: p. 300

19. The nurse is caring for a new mother and newborn in a rooming-in unit and
watches the mother put the infant in the bed, lying on her side, propped up with a
pillow. The nurse should point out that this position can increase the risk of which
situation?
A. Gastroesophageal reflux
B. Sudden infant death syndrome
C. Apnea episodes
D. Sleeping for short intervals

Answer: B

Rationale: The 'back to sleep' campaign is a national campaign used to educate the
public concerning the fact that the proper position for sleep of infants is on their
backs to help decrease the risk of SIDS. Placing the infant on his or her back to
sleep does not reduce the risk for gastroesophageal reflux, apnea episodes, or
sleeping for short intervals.
Question format: Multiple Choice
Chapter 14: Nursing Care of the Normal Newborn
Cognitive Level: Apply
Client Needs: Health Promotion and Maintenance
Integrated Process: Teaching/Learning
Reference: p. 299

20. The nurse is preparing the nursing care plan for a newborn who was born via a
cesarean delivery. Which diagnosis should the nurse prioritize?
A. Ineffective thermoregulation related to heat loss to the environment
B. Altered nutrition less than body requirement related to limited formula intake
C. Altered urinary elimination related to postcircumcision status
D. Ineffective airway clearance related to mucus and secretions

Answer: D

Rationale: Any airway clearance or obstruction issue is the highest priority for
nursing interventions, whether the infant is born via vaginal or cesarean delivery.
The other options are valid nursing diagnoses for some newborns; however, they
would not take precedence over an airway problem.
Question format: Multiple Choice
Chapter 14: Nursing Care of the Normal Newborn
Cognitive Level: Apply
Client Needs: Safe, Effective Care Environment: Management of Care
Integrated Process: Nursing Process
Reference: p. 291

21. The nurse is conducting a preadmission class for a group of parents on the
safety features that are utilized to help prevent infant abduction. The nurse should
prioritize which factor as most essential to ensure the program's success?
A. Use of pass codes onto the unit
B. Use of monitor attached to babies
C. Use of cameras at all doors
D. Cooperation by the parents with the hospital policies

Answer: D

Rationale: The most essential piece to an effective infant abduction prevention plan
is the cooperation of the parents. If the parents are not willing to participate in the
unit policy, the unit is at risk. Using pass codes, placing cameras at each door, and
using monitors on the infants will all help, but only if the parents are cooperative.
Question format: Multiple Choice
Chapter 14: Nursing Care of the Normal Newborn
Cognitive Level: Apply
Client Needs: Safe, Effective Care Environment: Safety and Infection Control
Integrated Process: Teaching/Learning
Reference: p. 292

22. The parents of a newborn male are questioning the nurse concerning the pros
and cons of a circumcision. Which disadvantage should the nurse point out to these
parents?
A. Lower rate of urinary tract infections
B. Reduced risk of penile cancer
C. Fewer complications than if done later in life
D. Anesthetic may not be effective during the procedure

Answer: D

Rationale: The anesthetic block is not always effective when used and not all
providers will even use anesthetics prior to the procedure, thus the infant can feel
the pain of the circumcision. A lower rate of urinary tract infections, a reduced risk
of penile cancer, and fewer complications than if circumcised later in life are
advantages to the procedure.
Question format: Multiple Choice
Chapter 14: Nursing Care of the Normal Newborn
Cognitive Level: Apply
Client Needs: Physiological Integrity: Reduction of Risk Potential
Integrated Process: Teaching/Learning
Reference: p. 296

23. The nurse is explaining to new parents the various vaccinations their newborn
will receive before being discharged home. Which immunization should the nurse
teach the parents about that will help decrease the incidence of hepatic disease
later in life?
A. Vitamin K
B. Hep B
C. HBV immunoglobin
D. HiB

Answer: B

Rationale: Hep B is the vaccination against hepatitis B and recommended by the


CDC. It has been found to help prevent cirrhosis and liver cancer later in life. The
HBV immunoglobin may be given in conjunction with the hep B if the mother is
found to be HBV positive. The HiB is given later, usually at the 2-month visit.
VItamin K is given soon after birth to reduce the risk of bleeding.n
Question format: Multiple Choice
Chapter 14: Nursing Care of the Normal Newborn
Cognitive Level: Apply
Client Needs: Physiological Integrity: Pharmacological and Parenteral Therapies
Integrated Process: Teaching/Learning
Reference: p. 298

24. The nurse is preparing the delivery room before the birth occurs. What supplies
would the nurse have available to care for the newborn? Select all that apply.
A. Warmer bed
B. Glucose water
C. Suction equipment
D. Identification bands
E. Ophthalmoscope

Answer: A, C, D

Rationale: In preparing the delivery room, the nurse should preheat a warmer bed,
have suction equipment at bedside, and have the identification bands ready for
both the mother and newborn. Glucose water and an ophthalmoscope are not
needed immediately after delivery to stabilize the newborn.
Question format: Multiple Select
Chapter 14: Nursing Care of the Normal Newborn
Cognitive Level: Apply
Client Needs: Safe, Effective Care Environment: Management of Care
Integrated Process: Nursing Process
Reference: p. 287

25. A nurse is preparing to administer vitamin K to a newborn who was just birthed
vaginally. Which site would be appropriate for the nurse to select?
A. deltoid
B. ventrogluteal
C. vastus lateralis
D. dorsogluteal

Answer: C

Rationale: The nurse would select an injection site on the vastus lateralis (anterior
lateral aspect of the thigh) muscle. The deltoid, ventrogluteal, or dorsogluteal would
be inappropriate to use.
Question format: Multiple Choice
Chapter 14: Nursing Care of the Normal Newborn
Cognitive Level: Apply
Client Needs: Physiological Integrity: Pharmacological and Parenteral Therapies
Integrated Process: Nursing Process
Reference: p. 290
Chapter 15
1. A new mother asks when she can give her infant fruit juice. The nurse would
most likely instruct the mother to wait until the infant is how old?
A. 3 months old
B. 5 months old
C. 8 months old
D. 12 months old

Answer: C

Rationale: Fruit juice is not recommended for infants younger than 6 months
because it supplies no protein and, if not pasteurized, can carry infectious
organisms. The amount should be limited to 3 to 4 ounces.
Question format: Multiple Choice
Chapter 15: Newborn Nutrition
Cognitive Level: Apply
Client Needs: Health Promotion and Maintenance
Integrated Process: Teaching/Learning
Reference: p. 321

2. The nurse is preparing discharge instructions for a new mother who has been
learning to breastfeed. Which response should the nurse prioritize when the
mother questions her ability to produce enough milk for her infant?
A. Take a daily multivitamin.
B. Drink a lot of milk.
C. Drink a lot of fluids.
D. Consume a minimum of 3000 calories per day.

Answer: C

Rationale: The mother needs to be sure to drink enough fluids, especially water, to
handle her thirst. She only needs an additional 500 calories per day over her
normal diet. Too much milk may cause gas and if her diet is appropriate she would
not need a multivitamin.
Question format: Multiple Choice
Chapter 15: Newborn Nutrition
Cognitive Level: Apply
Client Needs: Health Promotion and Maintenance
Integrated Process: Teaching/Learning
Reference: p. 311-312

3. A first-time mother has been breastfeeding her baby for 3 days and is concerned
she is not making enough milk. Which signs can the nurse teach the mother to look
for that indicate the infant is getting enough milk?
A. should have at least 5 wet diapers and 1 bowel movement daily
B. The mouth is full of milk after feeding.
C. appears drowsy or asleep after feeding
D. gains approximately 5 to 10 oz (140 to 280 g) each day

Answer: C

Rationale: The infant who is getting enough milk during breastfeeding sessions will
be drowsy or asleep at the end of the feeding. There should be a small amount of
milk (not large amount) in the mouth when finished. There should be at least 6 wet
diapers and 3 bowel movements each day (not just 5 wet and 2 bowel
movements). The infant will lose approximately 10% of the birth weight in the first
few days and regain it within 7 to 10 days. Most families will not be prepared to
weigh the infant in ounces, so this would not be a valid measure.
Question format: Multiple Choice
Chapter 15: Newborn Nutrition
Cognitive Level: Apply
Client Needs: Health Promotion and Maintenance
Integrated Process: Nursing Process
Reference: p. 315

4. The nurse is assisting a new mother with breastfeeding her infant, and she is
concerned about producing enough milk. The nurse determines the mother needs
more teaching when she makes which comment?
A. "My husband is bringing bottles of water for our baby."
B. "I should wait to start pumping my breasts for a few weeks."
C. "My baby needs to feed every 2 hours."
D. "Maybe I should have started breastfeeding sooner."

Answer: A

Rationale: The mother should not offer her newborn supplemental water. The
mother should nurse her infant every 2 to 4 hours. To increase or maintain her milk
supply, the mother needs to continuously empty her breast to make more milk.
Offering supplementation would be a period where the breast would not be emptied
and the production of milk would be decreased at that point. There is no need for
the mother to start using a breast pump at this time. The time in which the mother
starts breastfeeding after birth is not as important as how well the breasts are
emptied to encourage the production of more milk.
Question format: Multiple Choice
Chapter 15: Newborn Nutrition
Cognitive Level: Analyze
Client Needs: Health Promotion and Maintenance
Integrated Process: Nursing Process
Reference: p. 315

5. The nurse is preparing to assist a new mother to breastfeed for the first time.
Which action should the nurse prioritize?
A. Ensure privacy by closing the door or pulling the curtain.
B. Assist the woman with holding the infant correctly.
C. Assist the woman into a comfortable position.
D. Check the newborn's and woman's ID bands to make sure they match.

Answer: D

Rationale: The first step before putting any infant to breast is confirmation of the
infant and mother matching by proper identification. The other options are
interventions that would follow ensuring the correct mother and infant are together.
Question format: Multiple Choice
Chapter 15: Newborn Nutrition
Cognitive Level: Apply
Client Needs: Health Promotion and Maintenance
Integrated Process: Nursing Process
Reference: p. 313

6. The nurse is helping a mother prepare to breastfeed her newborn for the first
time. Which position should the nurse point out will be most appropriate with her
cesarean incision site?
A. football hold
B. side-lying position
C. cradle hold
D. whichever is most comfortable

Answer: A

Rationale: The football hold is ideal for women who have undergone surgery, have
large breasts, or have twins. This position allows for the woman to see the infant
while maintaining control of the infant's head. It will also avoid potentially putting
pressure on the incision site. The side-lying position and the cradle hold are
positions that the mother may try at a later session, once her abdominal incision is
further healed.
Question format: Multiple Choice
Chapter 15: Newborn Nutrition
Cognitive Level: Apply
Client Needs: Health Promotion and Maintenance
Integrated Process: Nursing Process
Reference: p. 313

7. The nurse is monitoring a breastfeeding session and determines the infant is


nursing well. Which assessment finding is the nurse using to make this
determination?
A. The mother is free of painful engorgement.
B. The newborn has a dry mouth after feedings.
C. The mother has difficulty rousing the newborn for a feeding.
D. The mother is experiencing uterine contractions.

Answer: D
Rationale: When the infant is latched on and breastfeeding well, the body will
release oxytocin and the result is uterine contractions. This is a good sign that the
infant has a good latch. A lack of painful engorgement does not mean that the
infant is getting an adequate supply of milk. The mucous membranes of an infant's
mouth should be moist and pink. Being able to rouse an infant for feeding or having
a difficult time rousing the infant are not indicators that the breastfeeding is going
well. Indications that breastfeeding is going well include the following: a newborn
that is correctly latched onto the breast will resist being pulled off of the breast;
audible swallowing and rhythmic jaw gliding; leaking from the opposite breast or a
let-down reflex.
Question format: Multiple Choice
Chapter 15: Newborn Nutrition
Cognitive Level: Apply
Client Needs: Health Promotion and Maintenance
Integrated Process: Nursing Process
Reference: p. 313-314

8. The nurse is teaching young parents discharge instructions. Which form of birth
control should the nurse point out will be the best option for the breastfeeding mom
who hopes to have more children later?
A. tubal ligation
B. combination hormonal birth control pill
C. estrogen-based pill
D. condoms

Answer: D

Rationale: The breastfeeding client needs to avoid use of any type of hormonal
birth control or her milk supply could be changed. The most appropriate choice is
the condom. Tubal ligation is a surgical procedure that prevents the egg and the
sperm from meeting. It is a method of birth control for people who do not want any
more children.
Question format: Multiple Choice
Chapter 15: Newborn Nutrition
Cognitive Level: Apply
Client Needs: Health Promotion and Maintenance
Integrated Process: Nursing Process
Reference: p. 317

9. The new mother has decided to formula-feed her infant and is unsure when to
introduce soft foods. Which age should the nurse point out will be appropriate to
introduce her infant to mashed fruit and vegetables?
A. 4 to 6 months
B. 6 to 8 months
C. 8 to 10 months
D. after 12 months
Answer: B

Rationale: The AAP has recommended the introduction of solids at the age of 6 to 8
months. Starting before 6 months would be too early. Waiting until after 8 months
may lead to malnutrition issues for the infant, as an infant's requirement for
increased vitamins and minerals starts to increase and are not always met by
formulas.
Question format: Multiple Choice
Chapter 15: Newborn Nutrition
Cognitive Level: Apply
Client Needs: Health Promotion and Maintenance
Integrated Process: Teaching/Learning
Reference: p. 320

10. The new mother is determined to breastfeed her baby exclusively for 6 months
but will need to return to work before then. The nurse should encourage the mother
to wait until the baby is what age before using expressed milk?
A. 2 weeks
B. 4 weeks
C. 6 weeks
D. 8 weeks

Answer: C

Rationale: It is recommended to breastfeed for six weeks prior to introduction of


bottles and pumping to establish a solid breastfeeding relationship. If bottles are
introduced at much less than 6 weeks, the establishment of a solid breastfeeding
relationship may be impaired. Waiting 8 weeks to introduce bottles of breast milk
into the baby's routine is not necessary and should not be suggested by the nurse.
Question format: Multiple Choice
Chapter 15: Newborn Nutrition
Cognitive Level: Apply
Client Needs: Health Promotion and Maintenance
Integrated Process: Teaching/Learning
Reference: p. 317

11. The nurse is assisting the new mother who has chosen to feed her newborn
formula. The nurse should encourage the mother to burp her baby at which time?
A. after every half-ounce
B. after every ounce
C. after every feeding
D. only when the infant appears to be in discomfort

Answer: A

Rationale: The mother should stop to burp the newborn every half-ounce; as the
newborn grows, the infant will be able to tolerate more formula at one time and
burp less frequently. Waiting until the infant has consumed 1 oz of formula, waiting
until the end of the feeding, or waiting until the infant appears to be in distress are
incorrect choices.
Question format: Multiple Choice
Chapter 15: Newborn Nutrition
Cognitive Level: Apply
Client Needs: Health Promotion and Maintenance
Integrated Process: Nursing Process
Reference: p. 319

12. The nurse is preparing a dietary care plan for a newborn who currently weighs 7
lbs (3182 g). How many kcal per day will the nurse plan for to ensure the infant's
energy needs are met?
A. 300 kcal
B. 350 kcal
C. 400 kcal
D. 450 kcal

Answer: B

Rationale: The healthy term newborn needs 110 to 120 kcal/kg/day to meet the
energy needs for growth and development. To determine the kcal/kg/day, multiple
3.182 x 110 = 350 and 3.182 x 120 = 382. Therefore this infant should receive
approximately 350 to 382 kcal each day to ensure it is meeting the energy needs
for consistent growth and development. To give the infant less than 350 kcal could
lead to failure to thrive, growth stump, and related problems. To give the infant too
much can lead to the excess being converted to fat and create a potential obesity
problem for the infant.
Question format: Multiple Choice
Chapter 15: Newborn Nutrition
Cognitive Level: Apply
Client Needs: Health Promotion and Maintenance
Integrated Process: Nursing Process
Reference: p. 307-308

13. The nurse is meeting with a group of expectant parents to discuss the feeding
options available for their soon-to-be infant. The nurse predicts which factor will
exert the strongest influence on their choice?
A. sociodemographics
B. exposure to breastfeeding
C. culture
D. cost

Answer: C

Rationale: A woman's culture often strongly influences newborn feeding choices. If


the woman feels it is culturally acceptable to breastfeed, she is more likely to
breastfeed her infant. Sociodemographics are characteristics of a person or a group
of people; the mother's exposure to breastfeeding and the cost of bottle-feeding all
can influence the mother's decision to breastfeed. However, they will not have the
strongest influence on the mother's choice of feeding her infant.
Question format: Multiple Choice
Chapter 15: Newborn Nutrition
Cognitive Level: Apply
Client Needs: Health Promotion and Maintenance
Integrated Process: Culture and Spirituality
Reference: p. 308

14. A young couple are discussing how long the mother should breastfeed and
cannot agree on a time frame. What is the best response from the nurse when they
question the nurse about it?
A. "You should work toward a goal of 6 weeks and then you can probably switch to
formula."
B. "It is recommended to breastfeed until 18 months, and possibly as long as 2
years."
C. "The recommendation is to use only breast milk for the first 6 months, then add
other foods until 12 months."
D. "There are no recommended guidelines for breastfeeding."

Answer: C

Rationale: The AAP (2016) recommends infants be exclusively breastfed for the first
6 months and breastfed with complementary foods for the first year of life. The
mother should be encouraged to breastfeed for longer than six weeks to ensure the
infant gains all the benefits from proper nutrition provided by breast milk. It is the
mother's choice to continue breastfeeding after 12 months, but the child should be
weaned at 2 years of age.
Question format: Multiple Choice
Chapter 15: Newborn Nutrition
Cognitive Level: Apply
Client Needs: Health Promotion and Maintenance
Integrated Process: Nursing Process
Reference: p. 309

15. The nurse is preparing a presentation for a health fair on the benefits of
breastfeeding. Which noted long-term benefit should the nurse prioritize?
A. rapid uterine involution
B. decreased stress levels
C. increased immune system
D. decreased risk of ovarian cancer

Answer: D

Rationale: The long-term benefits of breastfeeding include a decreased incidence of


ovarian and premenopausal breast cancers. The other options (rapid uterine
involution, decreased stress levels, and increased immune system) are short-term
benefits.
Question format: Multiple Choice
Chapter 15: Newborn Nutrition
Cognitive Level: Apply
Client Needs: Health Promotion and Maintenance
Integrated Process: Teaching/Learning
Reference: p. 309

16. The nurse is assisting a primigravida woman make decisions and prepare for
her baby. The nurse should point out that breastfeeding is not an option for this
client based on which assessment finding?
A. breast implants
B. galactosemia
C. need to return to work quickly
D. HIV infection

Answer: D

Rationale: Contraindications for breastfeeding include women who have HIV, use
illegal drugs, have active untreated tuberculosis, are receiving chemotherapy, or
have herpetic lesions on the breast. Women with active infections can pass these
conditions on to their infants while breastfeeding. A woman with breast implants
should be referred to a lactation consultant to assist and support this mother.
Galactosemia is a newborn disorder in which the infant has an inborn error of
metabolism. The mother who needs to return to work should be taught how to
pump her milk so that she can still provide breast milk to her infant.
Question format: Multiple Choice
Chapter 15: Newborn Nutrition
Cognitive Level: Apply
Client Needs: Health Promotion and Maintenance
Integrated Process: Nursing Process
Reference: p. 309-310

17. The young mother has been breastfeeding her newborn; however, the nurse is
concerned the infant is not thriving and may need to be switched to a formula.
Which condition is a contraindication for the infant to continue breastfeeding?
A. cleft palate
B. preterm delivery
C. Down syndrome
D. galactosemia

Answer: D

Rationale: Most infants are able to breastfeed, but infants who have certain
metabolic disorders should not be breastfed. Galactosemia is an inborn error of
metabolism and requires a specialty formula. Infants with a cleft palate or Down
syndrome can be breastfed. An infant who is born preterm may have to take its
mother's milk from a bottle but can still have breast milk.
Question format: Multiple Choice
Chapter 15: Newborn Nutrition
Cognitive Level: Apply
Client Needs: Physiological Integrity: Physiological Adaptation
Integrated Process: Nursing Process
Reference: p. 309-310

18. The nurse is leading a group discussion of new mothers about breastfeeding.
Which instruction should the nurse prioritize as the best method to increase their
milk supply?
A. Massage breast while pumping or breastfeeding.
B. Double pump when away from the baby.
C. Only direct breastfeed.
D. Empty the breast as completely as possible.

Answer: D

Rationale: Milk production is based on supply and demand. To maintain or increase


breast milk supply, the mother should empty her breast as often and as completely
as possible. Massaging the breast, double-pumping, and only direct breastfeeding
are not ways to increase the mother's milk supply.
Question format: Multiple Choice
Chapter 15: Newborn Nutrition
Cognitive Level: Apply
Client Needs: Health Promotion and Maintenance
Integrated Process: Teaching/Learning
Reference: p. 310-311

19. The new mother is surprised to learn that oxytocin is helping her breastfeed her
baby, especially after her health care provider used oxytocin to induce her labor.
Which additional function of oxytocin should the nurse point out to the mother?
A. increases production of milk
B. stimulates the release of milk
C. contracts breast cells to move the milk out of ducts
D. interacts with lactogen to produce milk

Answer: C

Rationale: Oxytocin is a muscle stimulant; the role of oxytocin in breastfeeding is to


move the milk down the ducts and out of the breast for feeding. Oxytocin does not
increase the production of milk; only complete emptying of the breast stimulates an
increase in the production of milk. Prolactin is responsible for stimulating the
release of milk, which then moves into the ducts. The receptor sites in the breast
are responsible for the increased production of milk. Lactogen aids in milk
production by stimulating the receptor sites.
Question format: Multiple Choice
Chapter 15: Newborn Nutrition
Cognitive Level: Apply
Client Needs: Health Promotion and Maintenance
Integrated Process: Nursing Process
Reference: p. 311

20. A woman on the postpartum unit with her 1-day-old infant is asking when her
milk will "come in." The nurse should point out this normally occurs at which time
after delivery?
A. 1 to 2 days
B. 3 to 5 days
C. 6 to 9 days
D. 2 weeks

Answer: B

Rationale: The breast has colostrum during the first few days after delivery. The
breast begins to make milk between days 3 and 5 after delivery. At day 6 and
beyond, the mother should be producing milk and will gradually increase the
amount as she continues to breastfeed her infant.
Question format: Multiple Choice
Chapter 15: Newborn Nutrition
Cognitive Level: Apply
Client Needs: Health Promotion and Maintenance
Integrated Process: Nursing Process
Reference: p. 311

21. The nurse is assisting a new mother to develop a sample meal plan for her to
meet the nutritional needs of herself and her breastfed infant. How many additional
calories should the nurse encourage the mother to include?
A. 300 kcal/day
B. 500 kcal/day
C. 700 kcal/day
D. 1000 kcal/day

Answer: B

Rationale: In order to meet the increased energy requirements for breastfeeding,


women need an additional 500 kcal/day above prepregnancy diet needs. Less than
500 would not provide enough, and over 500 would be too much and could create
additional issues such as obesity.
Question format: Multiple Choice
Chapter 15: Newborn Nutrition
Cognitive Level: Apply
Client Needs: Health Promotion and Maintenance
Integrated Process: Nursing Process
Reference: p. 311-312

22. The nurse is preparing a care plan for a new mother who desires to breastfeed.
Which action should the nurse prioritize if the mother has a history of breast
implants?
A. Give the mother a trial of breastfeeding.
B. Encourage the mother to use formula.
C. Seek approval from health care provider for breastfeeding.
D. Consult a lactation specialist for assistance.

Answer: D

Rationale: Any woman with a history of breast surgery or abnormal nipples may
need additional assistance with breastfeeding by a lactation consultant. This
consultant will know the best plan of action to assist this mother in breastfeeding.
The health care provider's permission is not required. The nurse should not advise
the mother to just try breastfeeding to see if it works or to encourage formula
feeding, but instead should provide the mother with the proper support.
Question format: Multiple Choice
Chapter 15: Newborn Nutrition
Cognitive Level: Apply
Client Needs: Health Promotion and Maintenance
Integrated Process: Nursing Process
Reference: p. 312

23. The nurse is assisting a new couple with breastfeeding. The nurse points out
which common position will place the mother and baby "tummy-to-tummy?"
A. side-lying
B. football hold
C. cradle hold
D. cross-cradle hold

Answer: C

Rationale: The cradle hold is the most common hold used in breastfeeding. It places
the mommy and baby tummy-to-tummy, as the newborn's lower arm is tucked
between the mom's arm and breast. This position works well for most infants and
most sized breasts. A football hold is recommended for women with large breasts,
who have undergone surgery, or have twins. In the side-lying position, the woman
can rest while feeding her newborn. There is no hold referred to as the cross-cradle
hold.
Question format: Multiple Choice
Chapter 15: Newborn Nutrition
Cognitive Level: Understand
Client Needs: Health Promotion and Maintenance
Integrated Process: Nursing Process
Reference: p. 313

24. The nurse is monitoring a new mother as she breastfeeds her 1-day-old infant.
The nurse determines more instruction is necessary when the infant uses which
action to latch on?
A. wide open mouth
B. tongue down
C. part of areola
D. nipple tip only

Answer: D

Rationale: The infant needs to take the entire nipple and most of the areola to
compress the milk ducts for let-down of the milk. If the infant is not latched on the
breast, the most common problem is pain with breastfeeding. Instructions for
latching the infant on the breast would include the infant having a wide-open mouth
with the tongue down as if yawning.
Question format: Multiple Choice
Chapter 15: Newborn Nutrition
Cognitive Level: Apply
Client Needs: Health Promotion and Maintenance
Integrated Process: Nursing Process
Reference: p. 313

25. The nurse is assessing the breastfeeding sessions of a newborn by looking at a


log of the duration of these sessions. The nurse determines the mother is
successfully learning the process after noting which time frame for nursing?
A. 5 to 10 minutes each side
B. 10 to 20 minutes each side
C. 20 to 30 minutes each side
D. 30 to 40 minutes each side

Answer: B

Rationale: Nursing sessions should last approximately 10 to 20 minutes per breast;


this should allow the infant time to fully remove all milk from the breast in that
duration. Nursing less than 10 minutes would not allow the infant time to empty the
breast and will result in less milk production. Nursing for longer than 20 minutes
will result in the infant suckling on empty breasts and can result in sore breasts. It
will also frustrate the infant, as suckling will not provide the desired result of
getting fed.
Question format: Multiple Choice
Chapter 15: Newborn Nutrition
Cognitive Level: Apply
Client Needs: Health Promotion and Maintenance
Integrated Process: Nursing Process
Reference: p. 314-315

26. A new mother brings her 2-week-old infant to the clinic for a well-baby visit and
reports there are days when the baby wants to nurse all the time. Which instruction
should the nurse prioritize to this mother?
A. Increase the feeding to on-demand.
B. Offer formula in addition to breastfeeding.
C. Pump and offer the breast milk in a bottle.
D. Offer only formula during the growth spurt.
Answer: A

Rationale: As the infant enters growth spurts, the infant will demand more milk,
which in turn increases the milk production, ensuring there will be enough to meet
the needs. Nursing the infant on-demand and increasing the frequency of
breastfeeding and the duration of the feedings will increase the milk supply.
Offering formula in addition to breastfeeding, or offering exclusively formula during
the growth spurt, is not recommended. Pumping and offering the breast milk in a
bottle is not necessary and is not recommended.
Question format: Multiple Choice
Chapter 15: Newborn Nutrition
Cognitive Level: Apply
Client Needs: Health Promotion and Maintenance
Integrated Process: Nursing Process
Reference: p. 316

27. A breastfeeding mother, 1-month postpartum, calls the clinic and reports left
breast soreness, a temperature of 100.4°F (38°C), and feeling tired all the time.
The nurse suspects the mother is experiencing which situation after revealing she is
still trying to breastfeed on a regular schedule?
A. plugged milk duct
B. engorgement
C. mastitis
D. breast yeast infection

Answer: C

Rationale: Mastitis is a problem associated with breastfeeding and is an infection of


the breast tissue. The woman will need antibiotic therapy to resolve this infection. A
plugged milk duct is usually indicated by a sore, reddened, hard lump in one area of
her breast. Engorgement is the filling of the breast with milk when the mother's
milk comes in. Breast yeast infection is not indicated.
Question format: Multiple Choice
Chapter 15: Newborn Nutrition
Cognitive Level: Apply
Client Needs: Physiological Integrity: Physiological Adaptation
Integrated Process: Nursing Process
Reference: p. 315-316

28. The nurse is conducting a training session for breastfeeding mothers who are
planning to return to work. As the nurse allows the mothers to examine the various
pumps available, which instruction should the nurse prioritize to describe the best
method for pumping?
A. Single pump and alternate breast every 5 minutes for a total of 30 minutes.
B. Double pump for a full 20 minutes.
C. Double pump for 10 minutes then change to single for another 5 minutes each.
D. Pump, either double or single, until the last drop is out.
Answer: B

Rationale: The best method for pumping and return to work is to double pump with
an electric pump. The other options are not as effective in totally pumping the
breast to provide adequate milk; they also require more time to perform, which
may be a factor if the woman does not have a break that will allow this time frame.
Question format: Multiple Choice
Chapter 15: Newborn Nutrition
Cognitive Level: Apply
Client Needs: Health Promotion and Maintenance
Integrated Process: Teaching/Learning
Reference: p. 317

29. A primigravida is trying to decide whether to breastfeed or use a formula to


feed her infant. When asking the nurse about the disadvantages of using a formula,
which response should the nurse prioritize?
A. increased risk of otitis media
B. increased risk of allergies
C. inferior nutrition with no immunologic properties
D. harder to digest

Answer: C

Rationale: The most concerning disadvantage to formula over breast milk is the
inferior nutrition of formula. Formula lacks any of the immunologic properties of
breast milk. Formula feeding does increase the risk of otitis media and allergies,
and it is also harder for the infant to digest; however, these factors do not have the
most significant impact on the infant.
Question format: Multiple Choice
Chapter 15: Newborn Nutrition
Cognitive Level: Apply
Client Needs: Health Promotion and Maintenance
Integrated Process: Teaching/Learning
Reference: p. 318

30. The nurse is assigned to care for a new mother and her formula-fed 2-day-old
infant. Which action should the nurse prioritize for this mother after receiving the
report from the previous shift nurse?
A. Ask the mother what formula she wants to use.
B. Use the standard formula in the unit's supply room.
C. Check the provider order to confirm the type of formula ordered.
D. Offer the mother the opportunity to try breastfeeding.

Answer: C

Rationale: The first action by the nurse after receiving the report is to check
provider orders and confirm the information passed in the report. It would not be
appropriate for the nurse to ask the mother what formula she wants to use or to
use the formula that is in the unit's supply room. It is also inappropriate to discuss
the mother's choice of feeding in an attempt to sway her decision.
Question format: Multiple Choice
Chapter 15: Newborn Nutrition
Cognitive Level: Apply
Client Needs: Safe, Effective Care Environment: Management of Care
Integrated Process: Nursing Process
Reference: p. 318

31. The nurse is reviewing discharge instructions with a young couple. The nurse
determines they understand how to properly use prepared formula and will discard
any leftover refrigerated formula after which time frame?
A. 12 hours
B. 24 hours
C. 36 hours
D. 48 hours

Answer: B

Rationale: Once formula is mixed it should be stored in the refrigerator until ready
to use. Once a bottle is warmed and the infant has drank from it, any leftover
contents need to be discarded. Any pre-prepared formula not used in a 24-hour
period should be discarded as well.
Question format: Multiple Choice
Chapter 15: Newborn Nutrition
Cognitive Level: Apply
Client Needs: Health Promotion and Maintenance
Integrated Process: Teaching/Learning
Reference: p. 320
Chapter 16
1. Between her regularly scheduled visits, a woman in her first trimester of
pregnancy who is taking iron supplements for anemia calls the nurse at her
obstetrician's office reporting constipation. She reports that she has never had this
problem before and asks for some advice about how to get relief. What is the best
advice the nurse can give her?
A. Stop taking iron supplements for a few days, exercise more, drink more fluids,
eat high-fiber, low-iron foods until the constipation is relieved, then resume the iron
supplement.
B. Continue taking iron supplements but increase fluids and high-fiber foods;
exercise more.
C. Increase the iron supplements, fluid intake, and consumption of high-fiber foods;
exercise more.
D. Take the iron supplement every other day, increase fluid intake and consumption
of high-fiber foods; exercise more.

Answer: B

Rationale: Constipation is a common side effect of iron supplementation. The


diagnosis of anemia indicates a true need for the iron supplementation; she needs
to increase fluid and fiber to relieve the constipation associated with the iron
preparations. The nurse should not advise this client to stop taking her iron
supplements, even for a few days. The nurse should not advise the client to
increase her iron supplementation, nor take the supplements on an every other day
basis. These supplements are ordered by the primary care provider based on the
client's hematologic status.
Question format: Multiple Choice
Chapter 16: Pregnancy at Risk: Conditions that Complicate Pregnancy
Cognitive Level: Apply
Client Needs: Physiological Integrity: Pharmacological and Parenteral Therapies
Integrated Process: Teaching/Learning
Reference: p. 337

2. A 14-year-old client and her parents have presented at the obstetrician's office in
the second trimester; the teen had been hiding the pregnancy. The nurse is helping
them develop a plan of care. What is the best thing the nurse can say to the clearly
angry parents?
A. "Your daughter needs to make decisions about this pregnancy for herself."
B. "I know you must be very upset and angry about your daughter's pregnancy, but
because she's still an adolescent herself, she'll need your guidance in making
nutritional and health choices that will be good for the baby and for herself."
C. "Anger won't help this situation at all. You'll only push your daughter away, and
she'll be less likely to make good choices."
D. "I understand your anger, but if you had encouraged your daughter to use
condoms she would probably not be in this situation."
Answer: B

Rationale: The nurse needs to acknowledge the anger of the parents but remember
her role is as the client advocate. The nurse needs to encourage the relationship of
support between the parents and the client. The nurse should not attempt to lay
down ground rules between the client and her parents, and the nurse should
acknowledge the parents' feelings in this situation. It is inappropriate to lecture or
place blame on the parents.
Question format: Multiple Choice
Chapter 16: Pregnancy at Risk: Conditions that Complicate Pregnancy
Cognitive Level: Apply
Client Needs: Psychosocial Integrity
Integrated Process: Caring
Reference: p. 351

3. A 43-year-old, physically fit, healthy woman who is newly married tells the nurse
that she and her husband would like to have a child. What is an appropriate first
response?
A. "You must know that's pretty risky. Have you thought about adopting instead?"
B. "Well, I'm sure you know there are some risks involved so it's helpful that you've
been taking such good care of yourself."
C. "You're in great shape now, but are you sure that at your age you'll have enough
energy to care for a child?"
D. "If you got pregnant now you'd be at risk for multiple fetal pregnancies,
chromosomal abnormalities, spontaneous abortion (miscarriage), and hypertension
among other things. Are you ready to take that risk?"

Answer: B

Rationale: The nurse needs to be positive and supportive in client response and
relationship, but honest about the risk involved with advanced maternal age. Option
A is incorrect; the nurse should not suggest to the mother that she adopt instead of
having her own child. Option C is incorrect as it is not supportive of the mother and
it suggests that she could not care for a child if she had one. Option D is incorrect
as it challenges the mother's thinking and indicates that the risks outweigh the
desire for a child.
Question format: Multiple Choice
Chapter 16: Pregnancy at Risk: Conditions that Complicate Pregnancy
Cognitive Level: Apply
Client Needs: Health Promotion and Maintenance
Integrated Process: Caring
Reference: p. 352

4. A woman's baby is HIV positive at birth. She asks the nurse if this means the
baby will develop AIDS. Which statement would be the nurse's best answer?
A. "She already has AIDS. That's what being HIV positive means."
B. "The antibodies may be those transferred across the placenta; the baby may not
develop AIDS."
C. "HIV is transmitted at birth; having a cesarean birth prevented transmission."
D. "HIV antibodies do not cross the placenta; this means the baby will develop
AIDS."

Answer: B

Rationale: Infants born of HIV-positive women test positive for HIV antibodies at
birth because these have crossed the placenta. An accurate disease status cannot
be determined until the antibodies fade at about 18 months. Testing positive for
HIV antibodies does not mean the infant has AIDS. Having a cesarean birth does
decrease the risk of transmitting the virus to the infant at birth; it does not prevent
the transmission of the disease. HIV antibodies do cross the placenta, which is why
babies born of HIV positive mothers are HIV positive.
Question format: Multiple Choice
Chapter 16: Pregnancy at Risk: Conditions that Complicate Pregnancy
Cognitive Level: Apply
Client Needs: Physiological Integrity: Physiological Adaptation
Integrated Process: Caring
Reference: p. 334

5. A pregnant single mom living alone tells the nurse she is considering getting a
cat for her 2-year-old daughter. Which is the best response by the nurse?
A. "The exposure to the cat litter may cause you to need a C-section."
B. "This will cut down on the jealousy for your two-year-old when the baby comes."
C. "If you don't think caring for a cat is too much work, that would be great."
D. "You should wait until after you give birth to obtain the cat for your daughter."

Answer: D

Rationale: Toxoplasma gondii is a protozoan that can be transmitted via


undercooked meat and through cat litter. Having a cat is not an issue, but cleaning
the litter box may expose the mother to the infection and result in fetal anomalies.
Exposure to the cat litter will not necessitate a cesarean section, and having a cat
will not cut down on any jealousy the 2-year-old might feel when the new baby is
born. The nurse would discourage the mother from getting cat until after the baby
is born.
Question format: Multiple Choice
Chapter 16: Pregnancy at Risk: Conditions that Complicate Pregnancy
Cognitive Level: Apply
Client Needs: Safe, Effective Care Environment: Safety and Infection Control
Integrated Process: Nursing Process
Reference: p. 341

6. A postpartum mother has the following lab data recorded: a negative rubella
titer. What is the appropriate nursing intervention?
A. No action needed.
B. Administer rubella vaccine before discharge.
C. Assess the rubella titer of the baby.
D. Notify the health care provider.

Answer: B

Rationale: Rubella is a virus, which when contracted during pregnancy has


significant complications for the fetus. The illness is mild to the adult but can result
in the infant being born deaf and blind. There is no cure; the CDC recommends all
individuals be vaccinated against rubella. If the titer is negative, the mother does
not have protection against rubella, and the next pregnancy would be at risk. She
should receive the vaccination prior to discharge from the hospital. Assessing the
rubella titer of the baby would not mean anything. The baby has not had rubella
and has not received antibodies against rubella from the mother. Notifying the
health care provider is not a priority, as most institutions have standing orders to
administer the rubella vaccine if the mother's rubella titer is negative.
Question format: Multiple Choice
Chapter 16: Pregnancy at Risk: Conditions that Complicate Pregnancy
Cognitive Level: Apply
Client Needs: Physiological Integrity: Reduction of Risk Potential
Integrated Process: Nursing Process
Reference: p. 342

7. When providing education to a prenatal care class for teenagers, the nurse states
that infants born to teenage mothers are more likely to have which outcome?
A. genetic problems
B. postdate birth
C. low-birth-weight
D. lower mortality rates

Answer: C

Rationale: Infants born to adolescent mothers are more likely to have a low birth
weight and poor outcomes and higher mortality rates when compared to infants of
older mothers. Infants born to teenage mothers are not more likely to have genetic
problems; they are more likely to be born preterm rather than postterm.
Question format: Multiple Choice
Chapter 16: Pregnancy at Risk: Conditions that Complicate Pregnancy
Cognitive Level: Understand
Client Needs: Health Promotion and Maintenance
Integrated Process: Teaching/Learning
Reference: p. 351

8. A 21-year old client arrives at the prenatal clinical at 12 weeks' gestation and
has a positive STI. Which of the following should the nurse be aware of that would
indicate the client is at high risk for noncompliance with treatment for the STI?
A. has a job and health insurance
B. noted to have history of substance abuse
C. lives with parents who are supportive of pregnancy
D. monogamous relationship with father of the baby

Answer: B

Rationale: Clients with a history of substance abuse are less likely to follow the
treatment plan. Substance abuse is an indicator of risky behaviors and a
consideration for high-risk of developing other social problems such as multiple
sexual partners. Having a job and health insurance, living with parents who are
supportive of the pregnancy, and having a monogamous relationship with the father
are indications of the client being likely to follow the treatment plan.
Question format: Multiple Choice
Chapter 16: Pregnancy at Risk: Conditions that Complicate Pregnancy
Cognitive Level: Apply
Client Needs: Physiological Integrity: Reduction of Risk Potential
Integrated Process: Nursing Process
Reference: p. 345

9. A woman with class II heart disease is experiencing an uneventful pregnancy and


is now prescribed bed rest at 36 weeks' gestation by her health care provider. The
nurse should point out that this is best accomplished with which position?
A. Lie flat on her back.
B. Stay in high Fowler position.
C. Lie in a semi-recumbent position.
D. Use pillows and wedges to stay in a fully recumbent position.

Answer: C

Rationale: Semi-recumbent position is the best position for circulation of the mother
and fetus. Lying flat on the back can induce supine hypotensive syndrome and fully
recumbent impedes other circulation. The high Fowler position would not be
comfortable for sleeping, as well as possibly impede the blood flow through the hips
and lower abdomen.
Question format: Multiple Choice
Chapter 16: Pregnancy at Risk: Conditions that Complicate Pregnancy
Cognitive Level: Apply
Client Needs: Physiological Integrity: Reduction of Risk Potential
Integrated Process: Nursing Process
Reference: p. 335

10. A G4P3 client with a history of controlled asthma is upset her initial prenatal
appointment is taking too long, making her late for another appointment. What is
the nurse's best response when the client insists she knows how to handle her
asthma and needs to leave?
A. Remind her to continue taking asthma medications, to monitor peak flow daily,
and to monitor the baby's kicks in the second and third trimesters.
B. Acknowledge her need to leave but ask her to demonstrate the use of inhaler
and peak flow meter before she goes; remind her to take regular medications.
C. Schedule an appointment for her to return to discuss her asthma management.
D. Note in the chart that the woman was not counseled about her asthma.

Answer: B

Rationale: Management of asthma during pregnancy is very important; the nurse


must document that the client has the proper ability to manage her asthma for her
health and the health of the fetus. Reminding the client to continue taking her
prescribed medication and to monitor her peak flow daily is not enough. It is the
nurse's responsibility to know that the client knows how to take her medications.
Monitoring the baby's kicks in the second and third trimester is an appropriate
action. Scheduling a return appointment to discuss asthma management is not
appropriate. She could have an asthma attack between the time the nurse sees her
and the time of the return appointment. Noting in the chart that the woman was
not counseled does not relieve the nurse of his/her obligation to ensure that the
woman knows how to use her inhaler and her peak flow meter.
Question format: Multiple Choice
Chapter 16: Pregnancy at Risk: Conditions that Complicate Pregnancy
Cognitive Level: Apply
Client Needs: Physiological Integrity: Reduction of Risk Potential
Integrated Process: Nursing Process
Reference: p. 339

11. A 32-year-old woman with epilepsy mentions to the nurse during a routine well-
visit that she would like to have children and asks the nurse for advice. Which
response is most appropriate from the nurse?
A. "You should talk to the doctor about that; the medications you're on can damage
the fetus."
B. "Do you want to talk to a counselor who can help you weigh the pros and cons of
having your own child rather than adopting?"
C. "I'll let the doctor know so you can discuss your medications. In the meantime,
I'll give you a list of folate-rich foods you can add to your diet."
D. "That's great. I've got a 4-year-old and a 2-year-old myself."

Answer: C

Rationale: Any woman with epilepsy needs to discuss medication management with
her provider. The current research indicates the medications used for epileptic
management are the major cause of birth defects for these clients. The nurse
should be careful about mentioning that some epilepsy medications are teratogenic;
some women may stop taking their medications in order to get pregnant.
Suggesting adoption is inappropriate as the mother has given no indication she is
interested in adoption; also, the mother needs to discuss this with the physician so
that she can get accurate information about being on anti-seizure medications and
being pregnant. The nurse should not share personal information as it does not
assist this client in making a serious decision. The client should be referred to the
health care provider to help the client make the best decision.
Question format: Multiple Choice
Chapter 16: Pregnancy at Risk: Conditions that Complicate Pregnancy
Cognitive Level: Apply
Client Needs: Safe, Effective Care Environment: Management of Care
Integrated Process: Nursing Process
Reference: p. 340

12. The nurse is assessing a primigravida woman who reports vaginal itching, a
great deal of foamy yellow-green discharge, and pain during intercourse. The nurse
suspects the woman has contracted which disorder?
A. chlamydia
B. simple yeast infection
C. trichomoniasis
D. gonorrhea

Answer: C

Rationale: Trichomoniasis is caused by a one-celled protozoa. The symptoms


include large amounts of foamy, yellow-green vaginal discharge. Treatment is with
metronidazole, and her partner needs to be treated as well. A yeast infection
presents with a cottage cheese-like discharge. Chlamydia often has no symptoms.
If the woman does experience symptoms, these may include vaginal discharge,
abnormal vaginal bleeding, and abdominal or pelvic pain. Gonorrhea may have
symptoms so mild that they go unnoticed in the woman. The woman who contracts
gonorrhea may have vaginal bleeding during sexual intercourse, pain and burning
while urinating, and a yellow or bloody vaginal discharge.
Question format: Multiple Choice
Chapter 16: Pregnancy at Risk: Conditions that Complicate Pregnancy
Cognitive Level: Apply
Client Needs: Physiological Integrity: Physiological Adaptation
Integrated Process: Nursing Process
Reference: p. 345

13. The nurse is preparing a postpartum nursing care plan for a single HIV-positive
primigravida client. The nurse should prioritize in the plan how to acquire which
resource?
A. breast pump
B. diapers
C. car seat
D. formula

Answer: D

Rationale: It is possible to transmit HIV via breastfeeding, and formula is the only
option for feeding. The nurse needs to provide positive information and offer to
make a referral or get assistance for clients who may be in financial need. In this
case, acquiring adequate amounts of formula would be the priority. The diapers and
a car seat are also necessary but would follow the formula. The client would not
need a breast pump since she cannot give the milk to her baby.
Question format: Multiple Choice
Chapter 16: Pregnancy at Risk: Conditions that Complicate Pregnancy
Cognitive Level: Apply
Client Needs: Safe, Effective Care Environment: Safety and Infection Control
Integrated Process: Nursing Process
Reference: p. 345

14. The nurse is assessing a client with type I diabetes who has just been informed
she is 8 weeks' pregnant. The nurse should explain that her health care provider
will probably refer her to which specialist to assist with this high-risk pregnancy?
A. endocrinologist
B. obstetrician
C. neonatologist
D. perinatologist

Answer: D

Rationale: A perinatologist is a physician specializing in the care of women who


have high-risk pregnancies. The endocrinologist would assist the perinatologist with
controlling the diabetes but would not assume total care of the pregnancy. An
obstetrician is a physician who specializes in the care of pregnancies. A
neonatologist is a physician who specializes in the care of neonates (infants up to
28 days of age).
Question format: Multiple Choice
Chapter 16: Pregnancy at Risk: Conditions that Complicate Pregnancy
Cognitive Level: Apply
Client Needs: Safe, Effective Care Environment: Management of Care
Integrated Process: Nursing Process
Reference: p. 329

15. The nurse is leading a discussion with a group of pregnant women who have
diabetes. The nurse should point out which situation can potentially occur during
their pregnancy?
A. Small-for-gestational-age (SGA) infant
B. Polyhydramnios
C. Postterm birth
D. Hypotension of pregnancy

Answer: B

Rationale: Polyhydramnios is an increase, or excess, in amniotic fluid and is a


pregnancy-related complication associated with diabetes. An infant who is small-
for-gestational-age is not associated with a mother who had diabetes prior to
pregnancy. Other pregnancy-related complications associated with pregestational
diabetes mellitus include hypertensive disorders, preterm birth, and shoulder
dystocia.
Question format: Multiple Choice
Chapter 16: Pregnancy at Risk: Conditions that Complicate Pregnancy
Cognitive Level: Apply
Client Needs: Physiological Integrity: Physiological Adaptation
Integrated Process: Teaching/Learning
Reference: p. 334

16. The nurse is preparing information for a client who has just been diagnosed
with gestational diabetes. Which instruction should the nurse prioritize in this
information?
A. Maintain a daily blood glucose log
B. Report any signs of possible urinary tract infection
C. Plan daily menus with dietitian
D. Long-term therapy goals

Answer: A

Rationale: Control of the blood glucose throughout the pregnancy is the primary
goal to help decrease potential complications to both the mother and fetus. The
mother should keep a daily log of her blood glucose levels and bring this log to each
visit for the nurse to evaluate. The other choices of reporting possible signs of a UTI
and working with a dietitian to plan menus would also be important but would be
secondary to the blood glucose control. It would be inappropriate to discuss long-
term goals at this time. This would be handled at a later time and would depend on
the mother's situation.
Question format: Multiple Choice
Chapter 16: Pregnancy at Risk: Conditions that Complicate Pregnancy
Cognitive Level: Apply
Client Needs: Physiological Integrity: Physiological Adaptation
Integrated Process: Teaching/Learning
Reference: p. 333

17. A G2P1 woman with type 1 diabetes is determined to be at 8 weeks' gestation


by her health care provider. The nurse should point out which factor will help the
client maintain glycemic control?
A. Vitamin supplements
B. Oral hypoglycemic agents
C. Exercise
D. Plenty of rest

Answer: C

Rationale: The three main facets to glycemic control for the woman with
pregestational diabetes are diet, exercise, and insulin. An individual with type 1
diabetes uses insulin and not oral hypoglycemic agents. Vitamin supplements may
assist with helping to keep the woman healthy but not necessarily through glycemic
control. It will be important for the woman to get enough rest throughout the
pregnancy but this will not assist with glycemic control.
Question format: Multiple Choice
Chapter 16: Pregnancy at Risk: Conditions that Complicate Pregnancy
Cognitive Level: Apply
Client Needs: Health Promotion and Maintenance
Integrated Process: Nursing Process
Reference: p. 330

18. A client with asthma is confused by her primary care provider continuing her
medication while she is pregnant, since she read online the medications can cause
birth defects. What is the nurse's best response?
A. "Your primary care provider will order safe doses of your medication."
B. "It's OK to not use them if you would feel more comfortable."
C. "They won't cause any major defects."
D. "I'll let your primary care provider know how you feel about it."

Answer: A

Rationale: Women should take no medication during pregnancy except that


prescribed by their primary care provider. The PCP will work with the mother to
ensure the safest amount is given to adequately handle the mother's health issues
and not injure the fetus. The PCP must weigh the risks against the benefits for both
the mother and her fetus. The nurse should not encourage the client to stop her
asthma medication as that may result in the client having an asthma attack, which
could result in injury to the fetus or even miscarriage. The nurse should not tell the
client a drug will not cause any defects, especially if it is known that it can. That
could make the nurse liable for damages. The nurse should inform the PCP of the
client's concerns; however, it is more important for the nurse to calm the client's
anxiety and offer positive reinforcement that the PCP is working hard to protect the
mother and infant from harm.
Question format: Multiple Choice
Chapter 16: Pregnancy at Risk: Conditions that Complicate Pregnancy
Cognitive Level: Apply
Client Needs: Physiological Integrity: Pharmacological and Parenteral Therapies
Integrated Process: Nursing Process
Reference: p. 339

19. A pregnant woman with sickle cell anemia is very concerned her infant will also
develop the disease and questions the nurse about that possibility. Which is the
best response from the nurse?
A. Both parents have to carry the trait.
B. There is a good chance the infant will inherit the disease from the mother.
C. The infant inherits the disease from the father.
D. If the mother goes into a crisis while pregnant, the baby will develop sickle cell
anemia.

Answer: A

Rationale: Sickle cell anemia is an autosomal recessive disease requiring that the
person have two genes for the disease, one from each parent. If one parent has the
disease and the other is free of the disease and trait, the chances of the child
inheriting the disease is zero. The infant will not develop the disease just because
the mother has a crisis during the pregnancy.
Question format: Multiple Choice
Chapter 16: Pregnancy at Risk: Conditions that Complicate Pregnancy
Cognitive Level: Understand
Client Needs: Health Promotion and Maintenance
Integrated Process: Caring
Reference: p. 338

20. The nurse is preparing to teach a pregnant client with iron deficiency anemia
about the various iron-rich foods to include in her diet. Which food should the nurse
point out will help increase the absorption of her iron supplement?
A. Dried apples
B. Fortified grains
C. Dried beans
D. Orange juice

Answer: D

Rationale: Anemia is a condition in which the blood is deficient in red blood cells,
from an underlying cause. The woman needs to take iron to manufacture enough
red blood cells. Taking an iron supplement will help improve her iron levels, and
taking iron with foods containing ascorbic acid, such as orange juice, improves the
absorption of iron. Dried fruit (such as apples), fortified grains, and dried beans are
additional food choices that are rich in iron and should be included in her daily diet.
Question format: Multiple Choice
Chapter 16: Pregnancy at Risk: Conditions that Complicate Pregnancy
Cognitive Level: Apply
Client Needs: Physiological Integrity: Pharmacological and Parenteral Therapies
Integrated Process: Teaching/Learning
Reference: p. 337

21. A woman with a positive history of genital herpes is in active labor. Assessment
reveals vesicles in the perineum area, membranes are ruptured, dilated 5 cm, and
effaced 70%. The nurse should prepare the client for which type of birth?
A. Spontaneous vaginal
B. Vacuum-assisted
C. Cesarean
D. Forceps-assisted

Answer: C

Rationale: An active herpes infection can be passed to the fetus during labor or with
ruptured amniotic membranes. The nurse should anticipate the infant will be born
via a cesarean birth. The risk of transmitting herpes to the baby would be increased
if the baby were born by spontaneous vaginal birth, vacuum-assisted birth, or
forceps-assisted birth.
Question format: Multiple Choice
Chapter 16: Pregnancy at Risk: Conditions that Complicate Pregnancy
Cognitive Level: Apply
Client Needs: Safe, Effective Care Environment: Safety and Infection Control
Integrated Process: Nursing Process
Reference: p. 343

22. A woman with cardiac disease at 32 weeks' gestation reports she has been
having spells of light-headedness and dizziness every few days. Which instruction
should the nurse prioritize?
A. Decrease activity and rest more often.
B. Increase fluids and take more vitamins.
C. Bed rest and bathroom privileges only until birth.
D. Discuss induction of labor with the health care provider.

Answer: A

Rationale: If the client is developing symptoms associated with her heart condition,
the first intervention is to monitor activity levels, decrease activity, and treat the
symptoms. At 32 weeks' gestation, the suggestion to induce labor is not
appropriate, and without knowledge of the type of heart condition one would not
recommend an increase of fluids or vitamins. Total bed rest may be required if the
symptoms do not resolve with decreased activity.
Question format: Multiple Choice
Chapter 16: Pregnancy at Risk: Conditions that Complicate Pregnancy
Cognitive Level: Apply
Client Needs: Safe, Effective Care Environment: Management of Care
Integrated Process: Nursing Process
Reference: p. 335

23. The nurse is caring for a pregnant client who has a class II heart disease. The
nurse is prepared to intensify monitoring during which period of the pregnancy?
A. first trimester
B. second trimester
C. third trimester
D. All are at equal risk.

Answer: B

Rationale: The periods of greatest risk for the pregnant woman with a cardiac
disease are at the end of the second trimester (when blood volume peaks), during
labor, and in the early postpartum period secondary to fluid shifts.
Question format: Multiple Choice
Chapter 16: Pregnancy at Risk: Conditions that Complicate Pregnancy
Cognitive Level: Apply
Client Needs: Physiological Integrity: Physiological Adaptation
Integrated Process: Nursing Process
Reference: p. 335
24. A 17-year-old primigravida at 37 weeks' gestation has been unable to maintain
adequate control of her blood glucose throughout her pregnancy. The nurse should
prioritize which action after the health care provider suspects the infant has
macrosomia based on the recent ultrasound?
A. Schedule induction of labor today.
B. Allow her to continue without plans for delivery.
C. Schedule cesarean birth at 39 weeks.
D. Prepare for assessment of fetal lung maturity.

Answer: D

Rationale: If the infant has macrosomia, is large for gestational age, and the
mother has had poor blood sugar control, the provider will want further information
on the fetus and readiness for delivery before making any decisions on delivery.
This will best be accomplished by an amniocentesis to assess the fetal lung
maturity. Scheduling an induction of labor, allowing the client to continue without
plans for birth, or scheduling a cesarean birth at 39 weeks would not be appropriate
nursing actions. Scheduling an induction or a cesarean birth is not in the province
of a nurse without a physician's order.
Question format: Multiple Choice
Chapter 16: Pregnancy at Risk: Conditions that Complicate Pregnancy
Cognitive Level: Apply
Client Needs: Physiological Integrity: Reduction of Risk Potential
Integrated Process: Nursing Process
Reference: p. 330

25. A nurse working at the local health district clinic assists numerous adolescents
who become pregnant. Which factor will the nurse tell the teens is crucial for a
positive pregnancy outcome?
A. Cultural sensitivity
B. Support network
C. Involvement of the father
D. Acceptance by peers

Answer: B

Rationale: One crucial part of management of teenage adolescent pregnancy


includes helping the teens to develop an adequate support network. The network
may include parents, teachers, friends and the father of the baby, in addition to
resources needed to provide care for the infant and self. Cultural sensitivity,
involvement of the father, and acceptance by peers are important to the teenager
who is pregnant, but they are not considered crucial for a positive pregnancy and
outcome for the mother and fetus.
Question format: Multiple Choice
Chapter 16: Pregnancy at Risk: Conditions that Complicate Pregnancy
Cognitive Level: Apply
Client Needs: Health Promotion and Maintenance
Integrated Process: Nursing Process
Reference: p. 351

26. A 38-year-old client, G4P3, at 10 weeks' gestation with an unplanned


pregnancy, has concerns the fetus may have a genetic defect. The nurse should
point out which test would be the best current choice to investigate the possibility
of a chromosomal abnormality?
A. amniocentesis
B. chorionic villus sampling (CVS)
C. maternal serum alpha-fetoprotein
D. triple screening

Answer: B

Rationale: Chorionic villus sampling is the earliest method (8 to 10 weeks'


gestation) to test fetal genetics for anomalies. This testing might be offered if the
mother wants specific information on the genetics of the fetus as early as possible
in pregnancy. Amniocentesis is generally done between 14 and 18 weeks' gestation,
but can be done as early as 10 weeks' gestation. Maternal serum alpha-fetoprotein
test is usually done at 16 to 20 weeks' gestation, and triple screening is performed
between 15 and 20 weeks' gestation.
Question format: Multiple Choice
Chapter 16: Pregnancy at Risk: Conditions that Complicate Pregnancy
Cognitive Level: Apply
Client Needs: Health Promotion and Maintenance
Integrated Process: Nursing Process
Reference: p. 352

27. The nurse is educating a client with type 1 diabetes about the complications
associated with diabetes and pregnancy. Which problems would the nurse include in
her teaching? Select all that apply.
A. Decreased birth weight
B. Increased risk of spontaneous abortion (miscarriage)
C. Polyhydramnios
D. Hypertension
E. Cystic fibrosis

Answer: B, C, D

Rationale: Women with pregestational diabetes, which is type 1 diabetes, are at a


higher risk of having an infant with complications during the pregnancy and at the
birth. Spontaneous abortion (miscarriage) is higher in women who have
pregestational diabetes. Also, they run a higher risk of having a pregnancy with
polyhydramnios, and of developing maternal hypertension. The birth weight of an
infant born to a mother with diabetes is increased, not decreased. Cystic fibrosis is
not associated with maternal diabetes.
Question format: Multiple Select
Chapter 16: Pregnancy at Risk: Conditions that Complicate Pregnancy
Cognitive Level: Apply
Client Needs: Health Promotion and Maintenance
Integrated Process: Nursing Process
Reference: p. 329

28. An infant is born to a mother with gestational diabetes. Which long-term


maternal complication is associated with this diagnosis?
A. Increased risk of development of type 2 diabetes
B. Weight gain that is not lost after the pregnancy
C. Development of long-term hypertension
D. Heart disease

Answer: A

Rationale: A mother who had gestational diabetes is at a 30% to 50% higher risk of
developing type 2 diabetes mellitus than the general population. Long-term
hypertension and heart disease are not associated with gestational diabetes, nor is
weight gain following pregnancy. There is no data that validates long-term weight
gain as a complication of gestational diabetes.
Question format: Multiple Choice
Chapter 16: Pregnancy at Risk: Conditions that Complicate Pregnancy
Cognitive Level: Understand
Client Needs: Physiological Integrity: Physiological Adaptation
Integrated Process: Nursing Process
Reference: p. 329

29. A pregnant client with type I diabetes asks the nurse about how to best control
her blood sugar while she is pregnant. The best reply would be for the woman to:
A. limit weight gain to 15 pounds during the pregnancy.
B. check her blood sugars frequently and adjust insulin accordingly.
C. exercise for 1 to 2 hours each day to keep the blood glucose down.
D. begin oral hyperglycemic medications along with the insulin she is currently
taking.

Answer: B

Rationale: The goal for a mother who has type I diabetes mellitus is to keep tight
control over her blood sugars throughout the pregnancy. Therefore, she needs to
test her blood sugar frequently during the day and make adjustments in the insulin
doses she is receiving.
Question format: Multiple Choice
Chapter 16: Pregnancy at Risk: Conditions that Complicate Pregnancy
Cognitive Level: Apply
Client Needs: Physiological Integrity: Reduction of Risk Potential
Integrated Process: Nursing Process
Reference: p. 330

30. The nurse is conducting a class for pregnant women on problems associated
with infections seen in women during gestation. One mother tells the nurse that she
has never had chickenpox (varicella) and is worried she will contract it before she
delivers. What would the nurse explain to this mother to ease her anxiety?
A. She can receive her varicella vaccine immediately after delivery, followed by a
second dose at her 6-week postpartum visit.
B. The likelihood of her catching varicella is very slim, so tell her not to worry.
C. She needs to let her physician know that she has not had varicella so she can
get immunized at her next visit.
D. Even if she does get varicella, her baby will be fine since varicella is not passed
on to the fetus.

Answer: A

Rationale: Varicella zoster is spread by respiratory droplets and is easily contracted


if the mother is exposed. If the mother does contract varicella, especially from
weeks 12 to 20 of gestation, it can be devastating to the fetus. Birth defects,
central nervous system damage, low birth weight, cognitive delays and deafness
can all occur. However, a mother cannot receive a varicella immunization during
pregnancy since the immunization can cross the placenta and act like the actual
disease, causing the same effects. The mother is immunized prior to discharge after
delivery and at her 6-week postpartum visit.
Question format: Multiple Choice
Chapter 16: Pregnancy at Risk: Conditions that Complicate Pregnancy
Cognitive Level: Apply
Client Needs: Safe, Effective Care Environment: Safety and Infection Control
Integrated Process: Nursing Process
Reference: p. 342

31. The nurse is assessing a 35-year-old woman at 22 weeks' gestation who has
had recent laboratory work. The nurse notes fasting blood glucose 146 mg/dl (8.10
mmol/L), hemoglobin 13 g/dl (130 g/L), and hematocrit 37% (0.37). Based on
these results, which instruction should the nurse prioritize?
A. Check blood sugar levels daily.
B. the signs and symptoms of urinary tract infection
C. Include iron-enriched foods in the diet.
D. Take daily iron supplements.

Answer: A

Rationale: An elevated blood glucose is concerning for diabetes. A fasting blood


glucose level of greater than 140 mg/dl (7.77 mmol/L) or random level of greater
than 200 mg/dl (11.10 mmol/L) is concerning; this must be followed up to ensure
the client is not developing gestational diabetes. The hemoglobin and hematocrit
are within normal limits for this client. The values should be hemoglobin greater
than 11 g/dl (110 g/L) and hematocrit greater than 33% (0.33). Values lower than
that are possible indications of anemia and would necessitate further evaluation. An
individual with higher than normal blood glucose levels is at risk for developing
urinary tract infection. This will usually happen after the glucose levels are
elevated. Anemia can be treated by increasing the consumption of iron-enriched
foods and taking a daily iron supplement.
Question format: Multiple Choice
Chapter 16: Pregnancy at Risk: Conditions that Complicate Pregnancy
Cognitive Level: Apply
Client Needs: Physiological Integrity: Reduction of Risk Potential
Integrated Process: Nursing Process
Reference: p. 330
Chapter 17
1. A woman at 8 weeks' gestation is admitted for ectopic pregnancy. She is asking
why this has occurred. The nurse knows that which factor is a known risk factor for
ectopic pregnancy?
A. high number of pregnancies
B. multiple gestation pregnancy
C. use of oral contraceptives
D. history of endometriosis

Answer: D

Rationale: The nurse needs to complete a full history of the client to determine if
she had any other risk factors for an ectopic pregnancy. Adhesions, scarring, and
narrowing of the tubal lumen may block the zygote's progress to the uterus. Any
condition or surgical procedure that can injure a fallopian tube increases the risk.
Examples include salpingitis, infection of the fallopian tube, endometriosis, history
of prior ectopic pregnancy, any type of tubal surgery, congenital malformation of
the tube, and multiple abortions (elective terminations of pregnancy). Conditions
that inhibit peristalsis of the tube can result in tubal pregnancy. A high number of
pregnancies, multiple gestation pregnancy, and the use of oral contraceptives are
not known risk factors for ectopic pregnancy.
Question format: Multiple Choice
Chapter 17: Pregnancy at Risk: Pregnancy-Related Complications
Cognitive Level: Understand
Client Needs: Physiological Integrity: Physiological Adaptation
Integrated Process: Nursing Process
Reference: p. 366

2. An 18-year-old pregnant client is hospitalized as she recovers from hyperemesis


gravidarum. The client reveals she wanted to have an abortion (elective termination
of pregnancy) but her cultural background forbids it. She is very unhappy about
being pregnant and even expresses a wish for a miscarriage. Which action by the
nurse is most appropriate?
A. Encourage the client to be positive about the situation.
B. Continue to monitor the client's hyperemesis gravidarum.
C. Contact the health care provider to report the client's feelings.
D. Share the information with the client's family.

Answer: C

Rationale: The client may be experiencing a psychological situation that needs


intervention by a trained professional in the area of mental health. The hyperemesis
gravidarum may worsen her feelings toward the pregnancy, so reporting her
feelings to the health care provider is the best action at this time. Although the
nurse will continue to monitor the client's hyperemesis gravidarum, this is not the
only action needed at this time and there is a better action. Encouraging the client
to be positive about her situation may obstruct therapeutic communication. Sharing
the information with the client's family is not appropriate, because the scenario
described does not indicate that the nurse has the client's permission to share this
information with the family.
Question format: Multiple Choice
Chapter 17: Pregnancy at Risk: Pregnancy-Related Complications
Cognitive Level: Analyze
Client Needs: Psychosocial Integrity
Integrated Process: Nursing Process
Reference: p. 357

3. A 28-year-old client with a history of endometriosis presents to the emergency


department with severe abdominal pain and nausea and vomiting. The client also
reports her periods are irregular with the last one being 2 months ago. The nurse
prepares to assess for which possible cause for this client's complaints?
A. Healthy pregnancy
B. Ectopic pregnancy
C. Molar pregnancy
D. Placenta previa

Answer: B

Rationale: The most commonly reported symptoms of ectopic pregnancy are pelvic
pain and/or vaginal spotting. Other symptoms of early pregnancy, such as breast
tenderness, nausea, and vomiting, may also be present. The diagnosis is not
always immediately apparent because many women present with complaints of
diffuse abdominal pain and minimal to no vaginal bleeding. Steps are taken to
diagnose the disorder and rule out other causes of abdominal pain. Given the
history of the client and the amount of pain, the possibility of ectopic pregnancy
needs to be considered. A healthy pregnancy would not present with severe
abdominal pain unless the client were term and she was in labor. With a molar
pregnancy the woman typically presents between 8 to 16 weeks' gestation
reporting painless (usually) brown to bright red vaginal bleeding. Placenta previa
typically presents with painless, bright red bleeding that begins with no warning.
Question format: Multiple Choice
Chapter 17: Pregnancy at Risk: Pregnancy-Related Complications
Cognitive Level: Apply
Client Needs: Physiological Integrity: Reduction of Risk Potential
Integrated Process: Nursing Process
Reference: p. 360

4. The nurse is giving discharge instructions to a client who experienced a complete


spontaneous abortion (miscarriage). Which question should the nurse prioritize at
this time?
A. "Are you going to wait a while before you try to get pregnant again?"
B. "Do you have someone to talk to, or may I give you the names and numbers for
some possible grief counselors?"
C. "Did you know that 75% of women who are trying to get pregnant experience
this same thing?"
D. "May I give you some resources that you can use to try to prevent this from
happening again?"

Answer: B

Rationale: When a woman has a spontaneous abortion (miscarriage) one important


consideration is the emotional needs of the woman once she is home. She may not
want to talk about the loss for a period of time, but the nurse needs to determine
her support system for the future. Asking the woman if she is "going to try again" is
an inappropriate question for the nurse to ask and diminishes the experience of
having a spontaneous abortion (miscarriage). It would be inappropriate to point out
the woman is not the only one to have this experience or to offer ways to prevent it
from happening in the future. The woman needs to deal with this situation first
before moving on to a possible "next" time.
Question format: Multiple Choice
Chapter 17: Pregnancy at Risk: Pregnancy-Related Complications
Cognitive Level: Apply
Client Needs: Psychosocial Integrity
Integrated Process: Caring
Reference: p. 361

5. The nurse is preparing discharge instructions for several clients after their
admission for emergent care of a pregnancy complication. The nurse will stress the
importance of frequent and continuous office visits to the client with:
A. a molar pregnancy.
B. an ectopic pregnancy.
C. a complete spontaneous abortion (miscarriage).
D. Rh negative blood.

Answer: A

Rationale: Molar pregnancies can indicate the possibility of developing malignancy.


The woman will need close observation and follow-up for a year, every 1 to 2 weeks
for hCG levels to detect cancer. A follow-up visit after an ectopic pregnancy or a
complete spontaneous abortion (miscarriage) are typically scheduled at 6 weeks,
not monthly. A woman who is Rh negative does not need a follow-up visit because
of her Rh status, but would be scheduled as per routine postpartum visits.
Question format: Multiple Choice
Chapter 17: Pregnancy at Risk: Pregnancy-Related Complications
Cognitive Level: Apply
Client Needs: Safe, Effective Care Environment: Management of Care
Integrated Process: Teaching/Learning
Reference: p. 364
6. A woman at 31 weeks' gestation presents to the emergency department with
bright red vaginal bleeding, reporting that the onset of the bleeding was sudden
and without pain. Which diagnostic test should the nurse prioritize?
A. A digital cervical exam
B. Nonstress test
C. Electronic fetal monitoring
D. A transvaginal ultrasound

Answer: D

Rationale: For any pregnant woman who presents with painless bleeding, placenta
previa needs to be ruled out by either transvaginal or abdominal ultrasound. A
digital cervical exam is contraindicated until placenta previa has been ruled out, as
digital manipulation of placental tissue through the cervical os can cause
uncontrollable bleeding. The nonstress test and electronic fetal monitoring would be
utilized after the woman is stabilized and placenta previa has either been diagnosed
or ruled out.
Question format: Multiple Choice
Chapter 17: Pregnancy at Risk: Pregnancy-Related Complications
Cognitive Level: Apply
Client Needs: Physiological Integrity: Reduction of Risk Potential
Integrated Process: Nursing Process
Reference: p. 366

7. A 24-year-old client is brought to the emergency department complaining of


severe abdominal pain, vaginal bleeding, and fatigue. On assessment, the nurse
notes cool, clammy skin; confusion; and vital signs as the following: HR 130, RR
28, and BP 98/60 mm Hg. Which action should the nurse prioritize?
A. Rule out shock.
B. Rule out pregnancy.
C. Attach EFM.
D. Establish IV access.

Answer: A

Rationale: Any time a client presents with hemorrhage, the initial nursing
consideration is assessment and evaluation of shock. The next step would be to
treat the shock, which could include establishing IV access and providing the client
with fluids. After the client is more stable, then the source of the bleeding would be
determined. In this case, that would include performing a pregnancy test to
determine if the client is pregnant to rule out a placenta complication. If it is
determined that the client is pregnant, the fetus would be assessed, which could
include using an EFM depending on the approximate age of the fetus.
Question format: Multiple Choice
Chapter 17: Pregnancy at Risk: Pregnancy-Related Complications
Cognitive Level: Apply
Client Needs: Physiological Integrity: Physiological Adaptation
Integrated Process: Nursing Process
Reference: p. 370

8. A 28-year-old client and her current partner present for the first prenatal
appointment with the ob/gyn. The client has no children but does question a
possible miscarriage 2 years ago; however, she never sought medical attention
because she felt fine. Labs reveal both client and partner are Rh negative. Which
action should the nurse prioritize?
A. Perform direct Coombs test.
B. Assess client for anti-D antibodies.
C. Arrange for an amniocentesis.
D. Continue with routine procedures and tasks.

Answer: B

Rationale: The client should be checked for sensitization to Rh-positive blood. It is


unknown if the client did have a miscarriage earlier, and if so, what the blood type
was of the fetus. The risk is high for the current fetus to be affected with hemolytic
disease, and this can be easily ruled out by assessing the mother for sensitization.
If this screening is negative, then no further testing is required. If the father were
Rh positive, then the mother be given Rho(D) immune globulin to prevent the
woman from developing antibodies to the Rho(D) factor. However, if it is positive,
the health care provider may order an amniocentesis to evaluate the fetus for
hemolytic disease so proper treatment and monitoring may be given. It is too early
to perform a direct Coombs test. It would be improper to ignore the potential of
serious complications and simply continue with routine tasks and procedures at this
time.
Question format: Multiple Choice
Chapter 17: Pregnancy at Risk: Pregnancy-Related Complications
Cognitive Level: Apply
Client Needs: Safe, Effective Care Environment: Management of Care
Integrated Process: Nursing Process
Reference: p. 358

9. The nurse is assisting a client who has just undergone an amniocentesis. Blood
results indicate the mother has type O blood and the fetus has type AB blood. The
nurse should point out the mother and fetus are at an increased risk for which
situation related to this procedure?
A. Placental abruption (abruptio placentae)
B. Preterm birth
C. Baby developing hemolytic anemia
D. Baby developing postbirth jaundice

Answer: D

Rationale: The infant and mother have ABO incompatibility. The result is a
development of antibodies and breaking down of the blood, resulting in jaundice in
the infant after delivery. The mixing of some fetal blood with maternal blood during
the amniocentesis would not cause placental abruption or preterm birth. Hemolytic
anemia is caused by Rh incompatibility. ABO incompatibility will cause hemolytic
disease.
Question format: Multiple Choice
Chapter 17: Pregnancy at Risk: Pregnancy-Related Complications
Cognitive Level: Apply
Client Needs: Physiological Integrity: Reduction of Risk Potential
Integrated Process: Nursing Process
Reference: p. 359

10. The nurse is assessing a 37-year-old woman who has presented in active labor
and notes the client has an increased risk for placental abruption (abruptio
placentae). Which assessment finding should the nurse prioritize?
A. Sharp fundal pain and discomfort between contractions
B. Painless vaginal bleeding and a fall in blood pressure
C. Pain in a lower quadrant and increased pulse rate
D. An increased blood pressure and oliguria

Answer: A

Rationale: A placental abruption (abruptio placentae) refers to premature


separation of the placenta from the uterus. As the placenta loosens, it causes sharp
pain. Labor begins with a continuing nagging sensation. Painless vaginal bleeding
and a fall in blood pressure are indicative of placenta previa. Pain in a lower
quadrant and increased pulse rate are indicative of an ectopic pregnancy.
Hypertension and oliguria are indicative of preeclampsia.
Question format: Multiple Choice
Chapter 17: Pregnancy at Risk: Pregnancy-Related Complications
Cognitive Level: Apply
Client Needs: Physiological Integrity: Physiological Adaptation
Integrated Process: Nursing Process
Reference: p. 367-368

11. A woman in labor suddenly reports sharp fundal pain accompanied by slight
dark red vaginal bleeding. The nurse should prepare to assist with which situation?
A. Premature separation of the placenta
B. Preterm labor that was undiagnosed
C. Placenta previa obstructing the cervix
D. Possible fetal death or injury

Answer: A

Rationale: Premature separation of the placenta begins with sharp fundal pain,
usually followed by dark red vaginal bleeding. Placenta previa usually produces
painless bright red bleeding. Preterm labor contractions are more often described
as cramping. Possible fetal death or injury does not present with sharp fundal pain.
It is usually painless.
Question format: Multiple Choice
Chapter 17: Pregnancy at Risk: Pregnancy-Related Complications
Cognitive Level: Apply
Client Needs: Physiological Integrity: Physiological Adaptation
Integrated Process: Nursing Process
Reference: p. 367-368

12. A 24-year-old client presents in labor. The nurse notes there is an order to
administer Rho(D) immune globulin after the birth of her infant. When asked by the
client the reason for this injection, which reason should the nurse point out?
A. promote maternal D antibody formation.
B. prevent maternal D antibody formation.
C. stimulate maternal D immune antigens.
D. prevent fetal Rh blood formation.

Answer: B

Rationale: Because Rho(D) immune globulin contains passive antibodies, the


solution will prevent the woman from forming long-lasting antibodies which may
harm a future fetus. The administration of Rho(D) immune globulin does not
promote the formation of maternal D antibodies; it does not stimulate maternal D
immune antigens or prevent fetal Rh blood formation.
Question format: Multiple Choice
Chapter 17: Pregnancy at Risk: Pregnancy-Related Complications
Cognitive Level: Apply
Client Needs: Physiological Integrity: Pharmacological and Parenteral Therapies
Integrated Process: Communication and Documentation
Reference: p. 358

13. A 24-year-old woman presents with vague abdominal pains, nausea, and
vomiting. An urine hCG is positive after the client mentioned that her last menstrual
period was 2 months ago. The nurse should prepare the client for which
intervention if the transvaginal ultrasound indicates a gestation sac is found in the
right lower quadrant?
A. Bed rest for the next 4 weeks
B. Intravenous administration of a tocolytic
C. Immediate surgery
D. Internal uterine monitoring

Answer: C

Rationale: The client presents with the signs and symptoms of an ectopic
pregnancy, which is confirmed by the transvaginal ultrasound. Ectopic pregnancy
means an embryo has implanted outside the uterus. Surgery is necessary to
remove the growing structure before damage can occur to the woman's internal
organs. Bed rest, a tocolytic, and internal uterine monitoring will not correct the
situation. The growing structure must be removed surgically.
Question format: Multiple Choice
Chapter 17: Pregnancy at Risk: Pregnancy-Related Complications
Cognitive Level: Apply
Client Needs: Safe, Effective Care Environment: Management of Care
Integrated Process: Nursing Process
Reference: p. 360

14. A client suffering a miscarriage at 12 weeks' gestation is very upset that the
health care provider has ordered a dilatation and curettage (D&C). How should the
nurse respond after the client states she didn't have a D&C the time she lost a
previous baby at 5 weeks' gestation?
A. "This is the procedure ordered by the doctor."
B. "You have the option to refuse the surgery."
C. "This procedure is needed to adequately remove all the fetal tissue."
D. "Having the D&C will make it easier to get pregnant next time."

Answer: C

Rationale: By this stage of pregnancy, the placenta has adhered to the uterine wall.
This was not the case when the client was only 5 weeks' pregnant. The total
removal of the products of conception will be needed to ensure healing and prevent
excessive bleeding and infection. To inform the client that the D&C is ordered by
the health care provider is unprofessional and does not adequately explain the
reason for the procedure. It also does not recognize the client's autonomy in
choosing her health care. Informing the client that she can refuse the procedure
would be unethical and could result in serious injury (including death if the client
were to bleed to death). It would also be inappropriate for the nurse to imply the
client will have an easier time getting pregnant again by having this procedure.
Question format: Multiple Choice
Chapter 17: Pregnancy at Risk: Pregnancy-Related Complications
Cognitive Level: Apply
Client Needs: Safe, Effective Care Environment: Management of Care
Integrated Process: Nursing Process
Reference: p. 361

15. A primigravida 28-year-old client is noted to have Rh negative blood and her
husband is noted to be Rh positive. The nurse should prepare to administer
RhoGAM after which diagnostic procedure?
A. Contraction test
B. Nonstress test
C. Biophysical profile
D. Amniocentesis

Answer: D

Rationale: Amniocentesis is a procedure requiring a needle to enter into the


amniotic sac. There is a risk of mixing of the fetal and maternal blood which could
result in blood incompatibility. A contraction test, a nonstress test, and biophysical
profile are not invasive, so there would be no indication for Rho(D) immune globulin
to be administered.
Question format: Multiple Choice
Chapter 17: Pregnancy at Risk: Pregnancy-Related Complications
Cognitive Level: Apply
Client Needs: Physiological Integrity: Reduction of Risk Potential
Integrated Process: Nursing Process
Reference: p. 358

16. A 25-year-old client at 22 weeks' gestation is noted to have proteinuria and


dependent edema on her routine prenatal visit. Which additional assessment should
the nurse prioritize and convey to the RN or health care provider?
A. initial BP 120/80mm Hg; current BP 130/88 mm Hg
B. initial BP 100/70 mm Hg; current BP 140/90 mm Hg
C. initial BP 140/85 mm Hg; current BP 130/80 mm Hg
D. initial BP 110/60 mm Hg; current BP 112/86 mm Hg

Answer: B

Rationale: A proteinuria of trace to 1+ and a rise in blood pressure to above 140/90


mm Hg is a concern the client may be developing preeclampsia. The blood
pressures noted in the other options are not indicative of developing preeclampsia.
The edema would not necessarily be indicative of preeclampsia; however, edema of
the face and hands would be a concerning sign for severe preeclampsia.
Question format: Multiple Choice
Chapter 17: Pregnancy at Risk: Pregnancy-Related Complications
Cognitive Level: Apply
Client Needs: Physiological Integrity: Physiological Adaptation
Integrated Process: Nursing Process
Reference: p. 371-372

17. A client at 37 weeks' gestation presents to the emergency department with a


BP 150/108 mm Hg, 1+ pedal edema, 1+ proteinuria, and normal deep tendon
reflexes. Which assessment should the nurse prioritize as the client is
administered magnesium sulfate IV?
A. urine protein
B. ability to sleep
C. hemoglobin
D. respiratory rate

Answer: D

Rationale: A therapeutic level of magnesium is 4 to 8 mg/dl (1.65 to 3.29 mmol/L).


If magnesium toxicity occurs, one sign in the client will be a decrease in the
respiratory rate and a potential respiratory arrest. Respiratory rate will be
monitored when on this medication. The client's hemoglobin and ability to sleep are
not factors for ongoing assessments for the client on magnesium sulfate. Urinary
output is measured hourly on the preeclamptic client receiving magnesium sulfate,
but urine protein is not an ongoing assessment.
Question format: Multiple Choice
Chapter 17: Pregnancy at Risk: Pregnancy-Related Complications
Cognitive Level: Apply
Client Needs: Physiological Integrity: Pharmacological and Parenteral Therapies
Integrated Process: Nursing Process
Reference: p. 375

18. A primipara at 36 weeks' gestation is being monitored in the prenatal clinic for
risk of preeclampsia. Which sign or symptom should the nurse prioritize?
A. A systolic blood pressure increase of 10 mm Hg
B. Weight gain of 1.2 lb (0.54 kg) during the past 1 week
C. A dipstick value of 2+ for protein
D. Pedal edema

Answer: C

Rationale: The increasing amount of protein in the urine is a concern the


preeclampsia may be progressing to severe preeclampsia. The woman needs
further assessment by the health care provider. Dependent edema may be seen in
a majority of pregnant women and is not an indicator of progression from
preeclampsia to eclampsia. Weight gain is no longer considered an indicator for the
progression of preeclampsia. A systolic blood pressure increase is not the highest
priority concern for the nurse, since there is no indication what the baseline blood
pressure was.
Question format: Multiple Choice
Chapter 17: Pregnancy at Risk: Pregnancy-Related Complications
Cognitive Level: Apply
Client Needs: Physiological Integrity: Reduction of Risk Potential
Integrated Process: Nursing Process
Reference: p. 371-372

19. The nurse is teaching a client who is diagnosed with preeclampsia how to
monitor her condition. The nurse determines the client needs more instruction after
making which statement?
A. "If I have changes in my vision, I will lie down and rest."
B. "I will weigh myself every morning after voiding before breakfast."
C. "I will count my baby's movements after each meal."
D. "If I have a severe headache, I'll call the clinic."

Answer: A

Rationale: Changes in the visual field may indicate the client has moved from
preeclampsia to severe preeclampsia and is at risk for developing a seizure due to
changes in cerebral blood flow. The client would require immediate assessment and
intervention. Gaining weight is not necessarily a sign of worsening preeclampsia.
The other choices are instructions which the client may be given to follow.
Question format: Multiple Choice
Chapter 17: Pregnancy at Risk: Pregnancy-Related Complications
Cognitive Level: Analyze
Client Needs: Physiological Integrity: Physiological Adaptation
Integrated Process: Teaching/Learning
Reference: p. 371-372

20. A pregnant woman at 38 weeks' gestation is receiving care for preeclampsia


and suddenly complains of sharp abdominal pain. Which action should the nurse
prioritize if the nurse notes a firm, distended, and painful abdomen and dark red
vaginal bleeding?
A. Implement a tocodynamometer.
B. Obtain a full set of vital signs.
C. Place on the fetal heart monitor.
D. Dipstick the urine for protein.

Answer: B

Rationale: The initial assessment is to determine if the client is going into shock
from hemorrhaging. The abdominal pain and dark red bleeding indicate a possible
abruption of the placenta related to the preeclampsia. The health care provider
must be notified ASAP. Assessing the status of the fetus would be next. Assessing
for contractions and the urine for protein are not priority assessments for the nurse.
Question format: Multiple Choice
Chapter 17: Pregnancy at Risk: Pregnancy-Related Complications
Cognitive Level: Analyze
Client Needs: Physiological Integrity: Reduction of Risk Potential
Integrated Process: Nursing Process
Reference: p. 371-372

21. The nurse is caring for a client who has remained in stable condition at 37
weeks' gestation. The client's condition suddenly changes. Which assessment
change should the nurse prioritize?
A. Vaginal bleeding and no pain
B. Uterine contractions with vaginal mucus
C. Fundal height and fetal heart rate
D. Size and contour of the abdomen

Answer: A

Rationale: Placenta previa includes bright red and painless vaginal bleeding, which
is different from the dark red bleeding of placental abruption (abruptio placenta)
accompanied by severe pain. This differentiates the two conditions. Uterine
contractions with vaginal mucus may be indications of the start of labor with the
mucus plug being discharged. The fetal heart rate, fundal height, and contour of the
abdomen are normal components that are assessed during the labor process.
Question format: Multiple Choice
Chapter 17: Pregnancy at Risk: Pregnancy-Related Complications
Cognitive Level: Apply
Client Needs: Physiological Integrity: Physiological Adaptation
Integrated Process: Nursing Process
Reference: p. 366-367
22. The nurse is preparing a woman for discharge after a birth and notes the
mother's record indicates Rh negative and rubella titer is positive. Which nursing
intervention will the nurse prioritize?
A. Administer Rho(D) immune globulin to the mother.
B. Administer rubella vaccine to the mother before discharge.
C. Assess the mother for isoimmunization.
D. Assess the Rh of the baby.

Answer: D

Rationale: The cord blood should be assessed to determine the infant's Rh type. If
it is negative, there is no need for any further treatment or concern. However, if it
is Rh positive the mother needs to be assessed for possible administration of
Rho(D) immune globulin. The criteria for giving Rho(D) immune globulin are as
follows:
• The woman must be Rho(D) negative
• The woman must not have anti-D antibodies (must not be sensitized
• The infant must be Rho(D) positive (fetus cord blood is checked after birth)
• A direct Coombs test (a test for antibodies performed on cord blood at
delivery) must be weakly reactive or negativ
This all needs to be completed within the first 72 hours of birth, so the mother can
receive Rho(D) immune globulin within the proper time frame.
Question format: Multiple Choice
Chapter 17: Pregnancy at Risk: Pregnancy-Related Complications
Cognitive Level: Apply
Client Needs: Physiological Integrity: Reduction of Risk Potential
Integrated Process: Nursing Process
Reference: p. 358-359

23. The nurse is comforting and listening to a young couple who just suffered a
spontaneous abortion (miscarriage). When asked why this happened, which reason
should the nurse share as a common cause?
A. Maternal smoking
B. Lack of prenatal care
C. Chromosomal abnormality
D. The age of the mother

Answer: C

Rationale: The most common cause for the loss of a fetus in the first trimester is
associated with a genetic defect or chromosomal abnormality. There is nothing that
can be done and the mother should feel no fault. The nurse needs to encourage the
parents to speak with a health care provider for further information and questions
related to genetic testing. Early pregnancy loss is not associated with maternal
smoking, lack of prenatal care, or the age of the mother.
Question format: Multiple Choice
Chapter 17: Pregnancy at Risk: Pregnancy-Related Complications
Cognitive Level: Understand
Client Needs: Psychosocial Integrity
Integrated Process: Caring
Reference: p. 361

24. A nurse is preparing a nursing care plan for a client who is admitted at 22
weeks' gestation with advanced cervical dilation (dilatation) to 5 cm, cervical
insufficiency, and a visible amniotic sac at the cervical opening. Which primary
goal should the nurse prioritize at this point?
A. Give birth vaginally
B. Bed rest to maintain pregnancy as long as possible
C. Notification of social support for loss of pregnancy
D. Education on causes of cervical insufficiency for the future

Answer: B

Rationale: At 22 weeks' gestation, the fetus is not viable. The woman would be
placed on total bed rest with every attempt made to halt any further progression of
dilation (dilatation) of the cervix. The nurse would not want this fetus to be born
vaginally at this stage of gestation. It is not the nurse's responsibility to notify the
client's social support of a possible loss of the pregnancy. It is not appropriate at
this time to educate the mother on causes of cervical insufficiency for future
pregnancies.
Question format: Multiple Choice
Chapter 17: Pregnancy at Risk: Pregnancy-Related Complications
Cognitive Level: Apply
Client Needs: Safe, Effective Care Environment: Management of Care
Integrated Process: Nursing Process
Reference: p. 364

25. A client at 27 weeks' gestation is admitted to the obstetric unit after reporting
headaches and edema of her hands. Review of the prenatal notes reveals blood
pressure consistently above 136/90 mm Hg. The nurse anticipates the health care
provider will prescribe magnesium sulfate to accomplish which primary goal?
A. Decrease blood pressure
B. Decrease protein in urine
C. Prevent maternal seizures
D. Reverse edema

Answer: C

Rationale: The primary therapy goal for any client with preeclampsia is to prevent
maternal seizures. Use of magnesium sulfate is the drug therapy of choice for
severe preeclampsia and is only used to manage and attempt to prevent
progression to eclampsia. Magnesium sulfate therapy does not have as a primary
goal of decreasing blood pressure, decreasing protein in the urine, or reversing
edema.
Question format: Multiple Choice
Chapter 17: Pregnancy at Risk: Pregnancy-Related Complications
Cognitive Level: Understand
Client Needs: Physiological Integrity: Pharmacological and Parenteral Therapies
Integrated Process: Nursing Process
Reference: p. 375

26. The following hourly assessments are obtained by the nurse on a client with
preeclampsia receiving magnesium sulfate: 97.3oF (36.2oC), HR 88, RR 12
breaths/min, BP 148/110 mm Hg. What other priority physical assessments by the
nurse should be implemented to assess for potential toxicity?
A. Lung sounds
B. Oxygen saturation
C. Reflexes
D. Magnesium sulfate level

Answer: C

Rationale: Reflex assessment is part of the standard assessment for clients on


magnesium sulfate. The first change when developing magnesium toxicity may be a
decrease in reflex activity. The health care provider needs to be notified
immediately. A change in lung sounds and oxygen saturation are not indicative of
magnesium sulfate toxicity. Hourly blood draws to gain information on the
magnesium sulfate level are not indicated.
Question format: Multiple Choice
Chapter 17: Pregnancy at Risk: Pregnancy-Related Complications
Cognitive Level: Analyze
Client Needs: Physiological Integrity: Pharmacological and Parenteral Therapies
Integrated Process: Nursing Process
Reference: p. 375

27. The nurse is appraising the medical record of a pregnant client who is resting in
a darkened room and receiving oxytocin and magnesium sulfate. The nurse will
continue to monitor this client for progression to which condition?
A. gestational hypertension
B. eclampsia
C. severe preeclampsia
D. mild preeclampsia

Answer: B

Rationale: This woman is in severe preeclampsia and must be monitored for


progression to eclampsia. The administration of magnesium sulfate is to relax the
skeletal muscles and raise the threshold for a seizure. The administration of
oxytocin is to stimulate uterine contractions to hasten birth. The client has already
progressed from mild preeclampsia to severe preeclampsia, and the nurse need to
follow measures to prevent advancement of the disease process. Although
preeclampsia results in a high blood pressure, the scenario described does not
indicate a client with hypertension.
Question format: Multiple Choice
Chapter 17: Pregnancy at Risk: Pregnancy-Related Complications
Cognitive Level: Analyze
Client Needs: Physiological Integrity: Reduction of Risk Potential
Integrated Process: Nursing Process
Reference: p. 371-372

28. A woman at 28 weeks' gestation has been hospitalized with moderate bleeding
that is now stabilizing. The nurse performs a routine assessment and notes the
client sleeping, lying on the back, and electronic fetal heart rate (FHR) monitor
showing gradually increasing baseline with late decelerations. Which action will the
nurse perform first?
A. Administer oxygen to the client.
B. Notify the health care provider.
C. Reposition the client to left side.
D. Increase the rate of IV fluids.

Answer: C

Rationale: The fetus is showing signs of fetal distress. The immediate treatment is
putting the client in a side-lying position to ensure adequate perfusion to the fetus.
After placing the client on the side, the nurse should re-assess the FHR and
determine if oxygen, IV fluids, and calling the health care provider are needed.
Question format: Multiple Choice
Chapter 17: Pregnancy at Risk: Pregnancy-Related Complications
Cognitive Level: Apply
Client Needs: Physiological Integrity: Reduction of Risk Potential
Integrated Process: Nursing Process
Reference: p. 369
Chapter 18
1. Which statement describes why hypertonic contractions tend to become very
painful?
A. More than one contraction may begin at the same time, as receptor points in the
myometrium act independently of each other.
B. The myometrium becomes sensitive from the lack of relaxation and anoxia of
uterine cells.
C. The number of uterine contractions is very low or infrequent.
D. There is an increase in the length of labor because so many contractions are
needed to achieve cervical dilation (dilatation).

Answer: B

Rationale: Hypertonic contractions cause uterine cell anoxia, which is painful.


Question format: Multiple Choice
Chapter 18: Labor at Risk
Cognitive Level: Understand
Client Needs: Physiological Integrity: Physiological Adaptation
Integrated Process: Nursing Process
Reference: p. 385

2. The nurse is assessing a multipara woman who presents to the hospital after
approximately 2 hours of labor and notes the fetus is in a transverse lie. After
notifying the RN and primary care provider, which action should the LPN prioritize?
A. Include a set of piper forceps when the table is prepped.
B. Apply pressure to the woman's lower back with a fisted hand.
C. Assist with nitrazine and fern tests.
D. Prepare to assist with external version.

Answer: D

Rationale: Transverse lie is a fetal malposition and is a cause for labor dystocia. The
fetus would need to be turned to the occipital position using external version or be
born via cesarean birth. Piper forceps are used in the birth of a fetus that is in the
breech position. Nitrazine and fern tests are done to assess if amniotic fluid is
leaking from the sac into the vagina. Counterpressure applied to the lower back
with a fisted hand sometimes helps the woman to cope with the "back labor" that is
characteristic of occiput posterior (OP) positioning.
Question format: Multiple Choice
Chapter 18: Labor at Risk
Cognitive Level: Apply
Client Needs: Health Promotion and Maintenance
Integrated Process: Nursing Process
Reference: p. 385
3. The nurse is monitoring a primipara who has been receiving oxytocin and is now
in hypertonic labor. If the nurse notes the fetal heart rate has suddenly dropped,
which action should the nurse prioritize?
A. Decrease the oxytocin drip rate.
B. Turn the client on the left side.
C. Administer a tocolytic medication.
D. Assist with McRoberts maneuver.

Answer: A

Rationale: The client will be receiving oxytocin to induce the labor. An adverse
reaction would be a hypertonic uterus, in which the uterine contractions are
manifested with increased frequency and intensity. When the infant is showing
signs of distress, the nurse should first either stop or decrease the oxytocin drip
rate, as per the primary care provider's orders. If this does not stop the
contractions, then a tocolytic should be administered to stop the uterus from
contracting and improve the fetal status. McRoberts maneuver is used when there
is shoulder dystocia. Changing positions does not provide an immediate impact.
Question format: Multiple Choice
Chapter 18: Labor at Risk
Cognitive Level: Apply
Client Needs: Physiological Integrity: Reduction of Risk Potential
Integrated Process: Nursing Process
Reference: p. 386

4. A client at 36 weeks' gestation presents to the OB unit reporting continuous,


heavy vaginal discharge and pelvic pressure. Assessment reveals no signs of labor
and positive nitrazine test. The nurse prepares for which nursing intervention after
admitting the client?
A. administering erythromycin IV
B. performing daily pelvic exams
C. administering IM corticosteroids
D. administering oxytocin

Answer: A

Rationale: With preterm PROM and no signs of labor, the woman would be treated
with IV antibiotics (usually ampicillin and erythromycin) for 48 hours, followed by
oral antibiotics for 5 days to decrease the risk of infection. Labor would not
automatically be induced at this time in a client who is 36 weeks' gestation. IM
corticosteroids are given between 24 and 34 weeks' gestation to reduce the risk of
neonatal respiratory distress syndrome, intraventricular hemorrhage, and
necrotizing enterocolitis. Daily pelvic exams are contraindicated as the client will be
at pelvic rest.
Question format: Multiple Choice
Chapter 18: Labor at Risk
Cognitive Level: Apply
Client Needs: Health Promotion and Maintenance
Integrated Process: Nursing Process
Reference: p. 388

5. A multigravida client at 31 weeks' gestation is admitted with confirmed preterm


labor. As the nurse continues to monitor the client now receiving magnesium
sulfate, which assessment findings will the nurse prioritize and report immediately
to the RN or health care provider?
A. Low potassium or elevated glucose, tachycardia, chest pain
B. Respiratory depression, hypotension, absent tendon reflexes
C. Severe lower back pain, leg cramps, sweating
D. Pain in the abdomen, shoulder, or back

Answer: B

Rationale: Magnesium sulfate is a smooth muscle relaxant and can cause


vasodilation and results in respiratory depression and severe hypotension at toxic
levels. The other options are incorrect indications of magnesium sulfate toxicity.
Question format: Multiple Choice
Chapter 18: Labor at Risk
Cognitive Level: Apply
Client Needs: Physiological Integrity: Pharmacological and Parenteral Therapies
Integrated Process: Nursing Process
Reference: p. 390

6. A 39-year-old multigravida with diabetes presents at 32 weeks' gestation


reporting she has not felt movement of her fetus. Assessment reveals the fetus has
died. The nurse shares with the mother that the institution takes pictures after the
birth and asks if she would like one. What is the best response if the mother angrily
says no and starts crying?
A. Apologize and tell her that the photos will be destroyed immediately.
B. Console her with the fact that she has other children.
C. Tell her that the hospital will keep the photos for her in case she changes her
mind.
D. Tell her that once she gets over her shock and grief, she will probably be happy
to have the photos.

Answer: C

Rationale: Emotional care of the woman is complex, especially one who has
suffered the loss of a child. The woman will need time to move through the stages
of grief and the responses of grief vary from person to person. The mother may
request the items later and they should be stored or kept for a year after the birth.
There is no need to apologize to the client. It would be inappropriate to console her
with the fact that she has other children. It negates her feelings and is not
supportive of the woman at this time.
Question format: Multiple Choice
Chapter 18: Labor at Risk
Cognitive Level: Analyze
Client Needs: Psychosocial Integrity
Integrated Process: Caring
Reference: p. 394

7. The nurse is assisting with a G2P1, 24-year-old client who has experienced an
uneventful pregnancy and is now progressing well through labor. Which action
should be prioritized after noting the fetal head has retracted into the vagina after
emerging?
A. Use McRoberts maneuver.
B. Use Zavanelli maneuver.
C. Apply pressure to the fundus.
D. Attempt to push in one of the fetus's shoulders.

Answer: A

Rationale: McRoberts maneuver intervention is used with a large baby who may
have shoulder dystocia and requires assistance. The legs are sharply flexed by a
support person or nurse, and the movement will help to open the pelvis to the
widest diameter possible. Zavanelli maneuver is performed when the practitioner
pushes the fetal head back in the birth canal and performs an emergency cesarean
birth. Fundal pressure is contraindicated with shoulder dystocia. It is outside the
scope of practice for the LPN to attempt birth of the fetus by pushing one of the
fetus' shoulders in a clockwise or counterclockwise motion.
Question format: Multiple Choice
Chapter 18: Labor at Risk
Cognitive Level: Apply
Client Needs: Health Promotion and Maintenance
Integrated Process: Nursing Process
Reference: p. 395

8. A woman at 39 weeks' gestation is brought to the emergency department in


labor following blunt trauma from an vehicle accident. The labor has been
progressing well after the epidural when suddenly the woman reports severe pain in
her back and shoulders. Which potential situation should the nurse suspect?
A. Fractured ribs
B. Placental abruption (abruptio placentae)
C. Uterine rupture
D. Dystocia

Answer: C

Rationale: A uterine rupture should be suspected in a pregnant woman who has


experienced blunt trauma to the abdomen and then complains of severe pain in her
back and shoulder. Uterine rupture occurs when the uterus tears open, leaving the
fetus and other uterine contents exposed to the peritoneal cavity. Traumatic
rupture can occur in connection with a blunt trauma. Abrupt change in the fetal
heart rate pattern is often the most significant sign associated with uterine rupture.
Other signs are reports of pain in the abdomen, shoulder, or back in a laboring
woman who had previous good pain relief from epidural anesthesia. The scenario
presented does not indicate fractured ribs from the accident. Placental abruption
(abruptio placentae) presents with pain; dark red vaginal bleeding; a rigid, board-
like abdomen; hypertonic labor; and fetal distress. Pain has a sudden onset and is
constant, and the uterus may not relax well between contractions. Dystocia is the
abnormally slow progression of labor, which is not indicated.
Question format: Multiple Choice
Chapter 18: Labor at Risk
Cognitive Level: Analyze
Client Needs: Physiological Integrity: Reduction of Risk Potential
Integrated Process: Nursing Process
Reference: p. 397

9. At 31 weeks' gestation, a 37-year-old woman with a history of preterm birth


reports cramps, vaginal pain, and low, dull backache accompanied by vaginal
discharge and bleeding. Assessment reveals cervix 2.1 cm long; fetal fibronectin in
cervical secretions, and cervix dilated 3 to 4 cm. Which interactions should the
nurse prepare to assist with?
A. bed rest and hydration at home
B. hospitalization, tocolytic, and corticosteroids
C. an emergency cesarean birth
D. careful monitoring of fetal movement (kick) counts

Answer: B

Rationale: At 31 weeks' gestation, the goal would be to maintain the pregnancy as


long as possible if the mother and fetus are tolerating the continuation of the
pregnancy. Stopping the contractions and placing the client in the hospital allows
for monitoring in a safe place if the woman continues and gives birth.
Administration of corticosteroids may help to develop the lungs and prepare for
early preterm birth. Sending the woman home is contraindicated in the scenario
described. An emergency cesarean birth is not indicated at this time. Monitoring
fetal movement (kick) counts is typically done with a postterm pregnancy.
Question format: Multiple Choice
Chapter 18: Labor at Risk
Cognitive Level: Apply
Client Needs: Safe, Effective Care Environment: Management of Care
Integrated Process: Nursing Process
Reference: p. 391

10. The nurse is admitting a woman at 32 weeks' gestation in preterm labor. The
nurse should question the order for magnesium sulfate after noting which
assessment finding?
A. cervical dilation (dilatation) of 5 cm
B. strong, regular contractions
C. fetus in a breech presentation
D. a spontaneous abortion (miscarriage) in an earlier pregnancy
Answer: A

Rationale: If cervical dilation (dilatation) has already progressed too far, labor
cannot be halted. Tocolytics are usually not administered if cervical dilation is
greater than 3 to 4 cm. A breech presentation of the fetus would not require the
nurse to question the administration of a tocolytic agent, nor is a spontaneous
abortion (miscarriage) in an earlier pregnancy.
Question format: Multiple Choice
Chapter 18: Labor at Risk
Cognitive Level: Analyze
Client Needs: Physiological Integrity: Pharmacological and Parenteral Therapies
Integrated Process: Nursing Process
Reference: p. 389

11. The nurse is caring for a primipara with PROM who appears extremely anxious
and reveals that she is scared her birthing process will be extremely painful
because it will be "dry". Which is the best response from the nurse?
A. "This is true but you can receive pain medication to help relieve this."
B. "No birth is ever really dry, because amniotic fluid continues to be
manufactured."
C. "Don't think so far ahead; concentrate on the problem at hand."
D. "Although the birth will be dry, it won't be painful."

Answer: B

Rationale: The nurse should let the client know that amniotic fluid is continuously
formed; no birth is ever dry. The client will be given medication for pain as well as
given the option to receive an epidural at the appropriate time. The nurse should
not negate the woman's concerns by telling her to not worry about it. This would be
a good time for informal teaching about the process and letting the woman know
the options available to assist her through this process.
Question format: Multiple Choice
Chapter 18: Labor at Risk
Cognitive Level: Apply
Client Needs: Psychosocial Integrity
Integrated Process: Caring
Reference: p. 388

12. A G2P1 woman is in labor attempting a VBAC, when she suddenly complains of
light-headedness and dizziness. An increase in pulse and decrease in blood pressure
is noted as a change from the vital signs obtained 15 minutes prior. The nurse
should investigate further for additional signs or symptoms of which complication?
A. Uterine rupture
B. Hypertonic uterus
C. Placenta previa
D. Umbilical cord compression

Answer: A
Rationale: The client with any prior history of uterus surgery is at increased risk for
a uterine rupture. A falling blood pressure and increasing pulse is a sign of
hemorrhage, and in this client a uterine rupture needs to be a first consideration.
The scenario does not indicate a hypertonic uterus, a placenta previa, or umbilical
cord compression.
Question format: Multiple Choice
Chapter 18: Labor at Risk
Cognitive Level: Apply
Client Needs: Physiological Integrity: Physiological Adaptation
Integrated Process: Nursing Process
Reference: p. 397

13. A woman who had preterm labor and preterm PROM successfully halted has
reached week 36 of pregnancy and is doing well on home care. Which of the
following nursing diagnoses should the nurse prioritize for this client?
A. Risk for fetal infection related to early rupture of membranes
B. Hopelessness related to potential loss of pregnancy
C. Anticipatory grieving related to high probability for fetal death from placental
dysfunction
D. Powerlessness related to inability to sustain pregnancy

Answer: A

Rationale: Once membranes have ruptured, the seal to the fetus is broken and
microorganisms may infect the uterus or fetus. There is no indication in the
scenario that a potential loss of pregnancy is evident, nor is there an indication of
anticipatory grieving. As described in this scenario there is not a high probability of
fetal death from placental dysfunction. A woman at 36 weeks' gestation would not
be feeling powerless because of an inability to sustain pregnancy.
Question format: Multiple Choice
Chapter 18: Labor at Risk
Cognitive Level: Apply
Client Needs: Safe, Effective Care Environment: Management of Care
Integrated Process: Nursing Process
Reference: p. 391

14. At 33 weeks' gestation, a woman experiencing preterm PROM is being prepared


to discharge home after successful treatment with magnesium sulfate. The nurse
should prioritize which instruction to the woman and her partner?
A. Continue taking nifedipine until you reach term.
B. Perform kick counts after each meal.
C. Complete the 10-day oral antibiotic course.
D. Use a barrier during sexual intercourse.

Answer: B
Rationale: When a client is being monitored at home, the well-being of the fetus
must be assessed as well. The best method for assessing the fetus is with the kick
count method, which should be done after each meal. Nifedipine is an oral tocolytic
that would not be utilized at this time. Oral antibiotics are given for 7 days, not 10.
The woman will be on pelvic rest with no sexual intercourse at this time, as it may
trigger uterine contractions leading to premature birth.
Question format: Multiple Choice
Chapter 18: Labor at Risk
Cognitive Level: Apply
Client Needs: Physiological Integrity: Reduction of Risk Potential
Client Needs: Physiological Integrity: Physiological Adaptation
Integrated Process: Nursing Process
Reference: p. 388

15. The nurse notes the fetal heart rate has slowed in a woman in labor at 8 cm
dilation (dilatation). Assessment reveals a prolapsed umbilical cord. Which action
should the nurse prioritize?
A. Turn client to her left side.
B. Place client in a knee-chest position.
C. Use fingers to press upward on the presenting part.
D. Prep for immediate cesarean delivery.

Answer: C

Rationale: If the woman presents with a visible prolapse of the cord, quickly place
her in bed and gently palpate the cord for pulsations to verify fetal viability. Then
use fingers to press upward on the presenting part. Continue to hold the presenting
part of the cord until delivery of the infant. If you discover the condition and are
unable to call for help, place the client in knee-chest position, call for help, and then
continue to intervene as previously described. Keeping the pressure of the fetus off
the cord improves fetal circulation. Replacing the cord could knot it; allowing it to
dry would constrict cord blood vessels. Turning the woman to the left side is not the
intervention of choice. Another nurse will be helping prepare this client for
immediate cesarean delivery.
Question format: Multiple Choice
Chapter 18: Labor at Risk
Cognitive Level: Apply
Client Needs: Physiological Integrity: Physiological Adaptation
Integrated Process: Nursing Process
Reference: p. 397

16. The nurse is monitoring a woman who is receiving IV oxytocin to assist with
uterine irritability. Which action should the nurse prioritize if the woman's
contractions are determined to be 80 seconds in length after 1 hour of
administration of the oxytocin?
A. Slow the infusion to under 10 gtts per minute.
B. Increase the flow rate of the main line infusion.
C. Discontinue the oxytocin infusion.
D. Continue to monitor contraction duration every 2 hours.

Answer: C

Rationale: If uterine contractions lengthen beyond 70 seconds, there is apt to be an


interference with fetal circulation. Discontinuing the infusion allows contractions to
shorten in length and allow fetal nourishment. The nurse would not increase the
flow rate of the main line infusion or slow the infusion without the health care
provider's prescription. Uterine contractions are monitored continuously.
Question format: Multiple Choice
Chapter 18: Labor at Risk
Cognitive Level: Apply
Client Needs: Physiological Integrity: Pharmacological and Parenteral Therapies
Integrated Process: Nursing Process
Reference: p. 385

17. A multipara woman at a birthing center is becoming very discouraged that her
labor is taking so long. She is confused when her nurse-midwife indicates she has
developed dystocia and needs to be transferred to a more advanced facility. What is
the best response from the nurse to answer the woman's questions?
A. "Your difficult labor needs treatment we cannot provide here."
B. "We don't have the medication you need."
C. "Your blood pressure is getting too high and needs treatment we can't provide
here."
D. "We need to consult with a surgeon in case a cesarean delivery is necessary."

Answer: A

Rationale: Birthing centers are designed to handle uncomplicated births; they have
agreements with more complex facilities to accept their clients if the situation
changes and more advanced treatment is needed. This client is showing signs of
dystocia, which is a general term used to describe difficult or abnormal labor. The
client needs to be in a more advanced medical facility that can provide the
medication needed to address uterine and/or blood pressure complications. More
importantly, this client may require a cesarean birth that would require a surgeon
and operating room facilities to safely deliver the fetus.
Question format: Multiple Choice
Chapter 18: Labor at Risk
Cognitive Level: Apply
Client Needs: Health Promotion and Maintenance
Integrated Process: Caring
Reference: p. 385

18. The nurse assesses that a fetus is in an occiput posterior position. The nurse
predicts the client will experience which situation related to this assessment?
A. Shorter dilation (dilatation) stage of labor
B. Experience of additional back pain
C. Need to have the baby manually rotated
D. Necessity for vacuum extraction for birth

Answer: B

Rationale: Most women whose fetus is in a posterior position experience back pain
while in labor. Pressure against the back by a support person often reduces this
type of pain. An occiput posterior position does not make for a shorter (dilation)
dilatation stage of labor. OP position does not indicate the need to have the baby
manually rotated, nor does it indicate a necessity for a vacuum extraction birth.
Question format: Multiple Choice
Chapter 18: Labor at Risk
Cognitive Level: Apply
Client Needs: Health Promotion and Maintenance
Integrated Process: Nursing Process
Reference: p. 385

19. A client in preterm labor is receiving magnesium sulfate IV and appears to be


responding well. Which finding on assessment should the nurse prioritize?
A. depressed deep tendon reflexes
B. tachypnea
C. bradycardia
D. elevated blood glucose

Answer: A

Rationale: The nurse should assess the woman at least once hourly and report any
dyspnea (not tachypnea), tachycardia (not bradycardia), productive cough,
adventitious breath sounds, and absent or decreased deep tendon reflexes in a
client receiving magnesium sulfate; these are all signs of possible magnesium
toxicity. Elevated blood glucose is a potential adverse reaction if the woman is
receiving terbutaline.
Question format: Multiple Choice
Chapter 18: Labor at Risk
Cognitive Level: Apply
Client Needs: Physiological Integrity: Pharmacological and Parenteral Therapies
Integrated Process: Nursing Process
Reference: p. 391

20. The nurse is assessing a multipara client at 28 weeks' gestation who may be
experiencing labor. Which findings should the nurse prioritize?
A. positive ferning
B. irregular mild contractions every 10 to 15 minutes
C. positive fetal fibronectin
D. cervical length of 2.7 cm

Answer: C
Rationale: Fetal fibronectin is a positive indicator of preterm labor, either active or
imminent. The mother can be dilated and effacing for weeks prior to delivery, as
long as there are no changes. Ferning is a positive sign of amniotic fluid leaking.
Irregular contractions do not indicate true labor. A cervical length of less than 2.5
cm at 20 to 26 weeks' gestation is associated with preterm labor.
Question format: Multiple Choice
Chapter 18: Labor at Risk
Cognitive Level: Apply
Client Needs: Health Promotion and Maintenance
Integrated Process: Nursing Process
Reference: p. 390

21. The nurse is preparing discharge instructions for a client at 32 weeks' gestation
who was admitted for PROM. What is the best response from the nurse when the
client asks when she can have intercourse with her husband again?
A. "You will not be able to have intercourse again until 6 weeks after you give
birth."
B. "The need to keep the infant safe should be of more concern than when to have
sex."
C. "That is a question to ask your health care provider; at this point you are on
pelvic rest to try and stop any further labor."
D. "Intercourse has nothing to do with preterm labor; you can have sex with your
husband."

Answer: C

Rationale: The client needs to be on pelvic rest until the health care provider says
otherwise. The intercourse can cause excitability in the uterus and encourage
cervical softening and should be avoided unless the provider says it is safe. The
nurse should be professional in all communications and not talk down to the client.
Question format: Multiple Choice
Chapter 18: Labor at Risk
Cognitive Level: Apply
Client Needs: Physiological Integrity: Reduction of Risk Potential
Integrated Process: Teaching/Learning
Reference: p. 388

22. A woman at 41 weeks' gestation is progressing well in labor; however, the


nurse notes the amniotic fluid is greenish in color. When questioned by the client
for the reason for this, which explanation should the nurse provide?
A. "Amniotic fluid is normally green."
B. "This is meconium-stained fluid from the baby."
C. "You have an infection and need antibiotics."
D. "Green might be a yeast infection and we need to culture the discharge."

Answer: B
Rationale: Green-tinted amniotic fluid is most often a sign of the infant having a
bowel movement in the uterus, called meconium-stained fluid. This is more typical
in a postdate pregnancy. Green-stained amniotic fluid is not a normal color for
amniotic fluid. However, it does not mean the mother has an infection and needs
antibiotics, nor does it does mean there might be a yeast infection present or
indicate the need for a culture of the fluid.
Question format: Multiple Choice
Chapter 18: Labor at Risk
Cognitive Level: Apply
Client Needs: Physiological Integrity: Physiological Adaptation
Integrated Process: Teaching/Learning
Reference: p. 393

23. The nurse is admitting a client at 23 weeks' gestation in preparation for


induction and delivery after it was determined the fetus had died secondary to
trauma. When asked by the client to explain what went wrong, the nurse can point
out which potential cause for this loss?
A. genetic abnormality
B. premature rupture of membranes
C. preeclampsia
D. placental abruption

Answer: D

Rationale: The most common cause of fetal death after a trauma is placental
abruption (abruptio placentae), where the placenta separates from the uterus, and
the fetus is not able to survive. Genetic abnormalities typically cause spontaneous
abortion (miscarriage) in the first trimester. Trauma does not cause preeclampsia
(which is related to various issues in the mother) nor does trauma usually cause
PROM.
Question format: Multiple Choice
Chapter 18: Labor at Risk
Cognitive Level: Apply
Client Needs: Physiological Integrity: Physiological Adaptation
Integrated Process: Nursing Process
Reference: p. 394

24. A G3P2 woman at 39 weeks' gestation presents highly agitated, reporting


something "came out" when her membranes just ruptured. Which action should the
nurse prioritize after noting the umbilical cord is hanging out of the vagina?
A. Put the client in bed immediately, call for help, and lift the presenting part of the
fetus off the cord.
B. With the client in lithotomy position, hold her legs and sharply flex them toward
her shoulders.
C. Place the client in Trendelenburg position and gently attempt to reinsert the
cord.
D. Contact the health care provider and prepare the client for an emergent vaginal
birth.
Answer: A

Rationale: The nurse must put the woman in a bed immediately, while calling for
help, and holding the presenting part of the fetus off the cord to ensure its safety.
Umbilical cord prolapse occurs when the umbilical cord slips down in front of the
presenting part, which can result in the presenting part compressing the cord,
cutting off oxygen and nutrients to the baby, and the baby is at risk of death. This
is an emergency. When a prolapsed cord is evident the nurse does not put the
woman in lithotomy position, and cannot attempt to reinsert the cord. A vaginal
birth is contraindicated in this situation.
Question format: Multiple Choice
Chapter 18: Labor at Risk
Cognitive Level: Apply
Client Needs: Physiological Integrity: Physiological Adaptation
Integrated Process: Nursing Process
Reference: p. 397

25. A client with preterm labor is receiving terbutaline therapy. Which assessment
will the nurse take?
A. deep tendon reflexes
B. respiratory status
C. serum potassium levels
D. fluid loss

Answer: C

Rationale: If a client s receiving terbutaline, it is important to monitor the client's


serum potassium and glucose levels. Low potassium or elevated glucose levels
should be reported to the RN and health care provider. Monitoring deep tendon
reflexes and respiratory status are critical assessments for a client receiving
magnesium sulfate. Monitoring fluid status is an important nursing function for the
client with preterm labor. However, the client is at risk for fluid overload, not fluid
loss.
Question format: Multiple Choice
Chapter 18: Labor at Risk
Cognitive Level: Apply
Client Needs: Physiological Integrity: Pharmacological and Parenteral Therapies
Integrated Process: Nursing Process
Reference: p. 391
Chapter 19
1. The nurse is caring for a mother within the first four hours after a cesarean birth.
Which nursing intervention would be most appropriate to prevent thrombophlebitis
in the mother?
A. Roll a bath blanket or towel and place it firmly behind the knees.
B. Limit oral intake of fluids for the first 24 hours to prevent nausea.
C. Assist client in performing leg exercises every 2 hours.
D. Ambulate the client as soon as her vital signs are stable.

Answer: D

Rationale: The best prevention for thrombophlebitis is ambulation as soon as


possible after recovery. Ambulation requires blood movement throughout the
cardiovascular system, decreasing thrombophlebitis risks. Placing a bath blanket
behind the knees interrupts circulation and could cause a thrombus. Fluids are
encouraged not limited. Leg exercises may put strain on the abdominal incision.
Question format: Multiple Choice
Chapter 19: Postpartum Woman at Risk
Cognitive Level: Apply
Client Needs: Health Promotion and Maintenance
Integrated Process: Nursing Process
Reference: p. 418

2. A client who delivered approximately 18 hours ago suddenly reports pelvic pain
that is unrelieved by comfort measures and medication. Assessment reveals HR
110, BP 96/68 mm Hg, firm fundus, and dark red, flowing lochia in moderate
amounts with no pooling. The most recent hematocrit is 31.9% and hemoglobin
10.5 g/dl. Which complication should the nurse prioritize?
A. retained placental fragments
B. deep vein thrombosis
C. lacerations in the uterus
D. deep pelvic hematoma

Answer: D

Rationale: The assessment data indicate blood loss in the body, and the lack of
active bleeding is indicative of a hematoma. Retained placental fragments are
characterized by late postpartum bleeding. Along with an abrupt onset of bleeding,
the woman's uterus is not well contracted. The woman with DVT may have no
symptoms. If the client does exhibit signs, these typically include swelling and calf
pain or tenderness in the affected leg. The area may be warm, tender, and red.
Lacerations can occur as small tears or cuts in the perineal tissue, vaginal sidewall,
or cervix. The normal range of hematocrit for women is 34.9% to 44.5% and
hemoglobin is 12.0 to 15.5 g/dl.
Question format: Multiple Choice
Chapter 19: Postpartum Woman at Risk
Cognitive Level: Analyze
Client Needs: Physiological Integrity: Physiological Adaptation
Integrated Process: Nursing Process
Reference: p. 405

3. A G4P4 client is recovering from dystocia for which oxytocin was administered to
assist with the contractions. On assessment 24 hours later, the nurse notes
moderate to heavy lochia with numerous large clots and the uterus in the midline,
above the umbilicus, and boggy. Which action should the nurse prioritize?
A. Ensure that her bladder is empty.
B. Initiate fundal massage.
C. Draw blood for hemoglobin and hematocrit STAT.
D. Encourage breastfeeding to stimulate uterine contractions.

Answer: A

Rationale: A boggy fundus with active bleeding and clots the day after birth is
indicative of uterine atony. The first step is to ensure the woman has an empty
bladder; otherwise, the uterus will not remain contracted once the massage stops.
A full bladder can also impede uterine contraction, which will predispose her to
hemorrhage. If the bladder is full, the uterus would be displaced to the side of the
umbilicus instead of in the midline. The nurse should then prepare to initiate fundal
massage by placing one hand over the symphysis pubis while the other hand gently
palpates and massages the uterus to encourage contractions. The use of firm,
steady, gentle fundal pressure will help express any clots that may have collected in
the uterus. Obtaining blood work for H&H or encouraging breastfeeding would not
be the first choice for handling this situation. The client is at risk of hemorrhaging
and this needs to be addressed quickly.
Question format: Multiple Choice
Chapter 19: Postpartum Woman at Risk
Cognitive Level: Apply
Client Needs: Physiological Integrity: Physiological Adaptation
Integrated Process: Nursing Process
Reference: p. 405

4. Initial measures to stop a client's bleeding have not proved successful, and she is
being transferred to the ICU. Her family is frightened by the IV lines and the nasal
cannula. The client's brother suddenly says to her partner, "This is all your fault!"
What is the best response by the nurse?
A. Leave the room quietly; this is a family matter.
B. Draw the brother aside and tell him that if he cannot control himself, he will have
to leave.
C. Explain the client's care, focus on signs of improvement, and acknowledge this is
a difficult time.
D. Tell them that the RN will be notified and will explain the client's treatment to
them.
Answer: C

Rationale: The nurse's first responsibility is to the client. She needs to be aware of
the interaction and focus the family on the client, explaining the cares and
acknowledging the difficult time for all involved. Leaving the room is not an
appropriate action. The nurse would only ask the client's brother to leave if he could
not be redirected and continued to cause a disruption. The nurse would not have
the RN explain the client's treatment.
Question format: Multiple Choice
Chapter 19: Postpartum Woman at Risk
Cognitive Level: Apply
Client Needs: Safe, Effective Care Environment: Management of Care
Integrated Process: Caring
Reference: p. 407

5. The nurse is caring for several postpartum clients and notes various warning
signs that are concerning. Which client should the nurse suspect is developing
endometritis?
A. A woman with diabetes, vaginal birth, HR 110, temperature 101.7°F (38.7°C) on
the third postpartum day. The next day, appears ill; temperature now 102.9°F
(39.3°C); WBC 31,500/mm3; negative blood cultures.
B. A woman with a history of infection and smoking, temperature 101°F (38.3°C)
on the fourth postpartum day; reports severe perineal pain; edges of the
episiotomy have separated.
C. An obese woman with temperature 100.4°F (38°C) at 12 hours after birth; lochia
is moderate; negative vaginal cultures.
D. A woman with PROM before birth; reports severe burning with urination, malaise
and severe temperature spikes on the 7th postpartum day. WBC is 21,850/mm3;
temperature 101°F (38.3°C); skin pale and clammy.

Answer: A

Rationale: Endometritis is an infection of the endometrium of the uterus. Clinical


manifestations include a fever of 100.4°F (38°C) or higher, usually between the
2nd and 10th day after delivery; tachycardia, chills, anorexia, and general malaise;
client may also report abdominal cramping and pain. Reports of severe perineal
pain and signs of fever and separation of the episiotomy edges would be suspicious
for a wound infection. An elevated temperature of up to 100.4°F (38°C) within the
first 24 hours is a normal response to the birthing process. Reports of severe
burning on urination accompanied by fever and malaise would be suspicious of a
UTI.
Question format: Multiple Choice
Chapter 19: Postpartum Woman at Risk
Cognitive Level: Analyze
Client Needs: Physiological Integrity: Physiological Adaptation
Integrated Process: Nursing Process
Reference: p. 410
6. The nurse is caring for a woman who delivered via a cesarean birth
approximately 16 hours earlier. Which assessment finding should the nurse
prioritize?
A. gradually decreasing temperature and pulse rate
B. uterine height at umbilicus
C. steadily decreasing volume of urine
D. excessive diaphoresis

Answer: C

Rationale: Decreasing amounts of urine indicate hypovolemic complications and


need to be further assessed by the RN and/or health care provider. This may be an
indication the client is hemorrhaging and is in danger of going into shock and must
be evaluated immediately. A gradually decreasing temperature and pulse rate and
excessive diaphoresis are normal findings at this stage and would not need
prioritized attention. The uterine height would not be a priority at this time.
Question format: Multiple Choice
Chapter 19: Postpartum Woman at Risk
Cognitive Level: Apply
Client Needs: Physiological Integrity: Physiological Adaptation
Integrated Process: Nursing Process
Reference: p. 404

7. The nurse is preparing discharge instructions for a client who has developed
endometritis after a cesarean birth. As the client is to be discharged on antibiotic
therapy, which instruction should the nurse prioritize?
A. Handwashing
B. Complete the antibiotic course
C. Proper perineal care
D. Get plenty of sleep

Answer: A

Rationale: Handwashing is the best defense against the spread of infections. The
client is at a higher risk of developing further infections due to her current
situation; handwashing before and after using the restroom and doing perineal care
will help prevent an infection from occurring. It will also be important for the
woman to wash her hands to ensure the infection is not passed to her infant or
other family members. The other options of completing the antibiotics, completing
proper perineal care, and getting plenty of rest are also important but not a
priority.
Question format: Multiple Choice
Chapter 19: Postpartum Woman at Risk
Cognitive Level: Apply
Client Needs: Physiological Integrity: Pharmacological and Parenteral Therapies
Integrated Process: Teaching/Learning
Reference: p. 412
8. The nurse is explaining the discharge instructions to a client who has developed
postpartum cystitis. The client indicates she is not drinking a glass of fluid every
hour because it hurts too much when she urinates. What is the best response from
the nurse?
A. Advise her to take acetaminophen to ease symptoms.
B. Ask primary care provider to prescribe an analgesic.
C. Instruct to use a sitz bath while voiding.
D. Teach that adequate hydration helps clear the infection quicker.

Answer: D

Rationale: Adequate hydration is necessary to dilute the bacterial concentration in


the urine and aid in clearing the organisms from the urinary tract. Encourage the
woman to drink at least 3000 ml of fluid a day, suggesting she drink one glass per
hour. Drinking fluid will make the urine acidic, deterring organism growth. The
other choices are also options but address the symptoms and not the root cause.
The goal should be to rid the body of the infection, not concentrate on
counteracting the results of the infection.
Question format: Multiple Choice
Chapter 19: Postpartum Woman at Risk
Cognitive Level: Apply
Client Needs: Physiological Integrity: Reduction of Risk Potential
Integrated Process: Teaching/Learning
Reference: p. 415

9. The nurse is caring for a client who has been diagnosed with a deep vein
thrombosis. Which assessment finding should the nurse prioritize and report
immediately?
A. Calf pain
B. Pyrexia
C. Edema
D. Dyspnea

Answer: D

Rationale: A DVT is often suspected when an individual with an increased risk


develops calf pain, pyrexia, and edema in one lower extremity. After the individual
has been positively diagnosed with a DVT, any signs of dyspnea should be suspect
of possible pulmonary embolism and should be handled as an emergency. The RN
and/or primary care provider should be notified immediately so emergent care can
be started, as this is often fatal.
Question format: Multiple Choice
Chapter 19: Postpartum Woman at Risk
Cognitive Level: Apply
Client Needs: Physiological Integrity: Physiological Adaptation
Integrated Process: Nursing Process
Reference: p. 418
10. The father of a 2-week-old infant presents to the clinic with his disheveled wife
for a postpartum visit. He reports his wife is acting differently, is extremely
talkative and energetic, sleeping only 1 or 2 hours at a time (if at all), not eating,
and appears to be totally neglecting the infant. The nurse should suspect the client
is exhibiting signs and symptoms of which disorder?
A. Postpartum blues
B. Postpartum depression
C. Postpartum psychosis
D. Maladjustment

Answer: C

Rationale: Postpartum psychosis in a client can present with extreme mood


changes and odd behavior. Her sudden change in behavior from normal, along with
a lack of self-care and care for the infant, are signs of psychosis and need to be
assessed by a provider as soon as possible. Postpartum depression affects the
woman's ability to function; however, her perception of reality remains intact.
Postpartum blues is a transitory phase of sadness and crying common among
postpartum women.
Question format: Multiple Choice
Chapter 19: Postpartum Woman at Risk
Cognitive Level: Apply
Client Needs: Psychosocial Integrity
Integrated Process: Nursing Process
Reference: p. 422

11. The nursing instructor is leading a discussion exploring the various conditions
that can result in postpartum hemorrhage. The instructor determines the session is
successful when the students correctly choose which condition is most frequently
the cause of postpartum hemorrhage?
A. Hematoma
B. Uterine atony
C. Perineal lacerations
D. Disseminated intravascular coagulation

Answer: B

Rationale: Early postpartum hemorrhage usually results from one of the following
conditions: uterine atony, lacerations, or hematoma. Most cases of early
postpartum hemorrhage result from uterine atony, which is due to the uterine
muscles remaining relaxed and not contracting as they should. Disseminated
intravascular coagulation is a complication which can occur with excessive
postpartum hemorrhage.
Question format: Multiple Choice
Chapter 19: Postpartum Woman at Risk
Cognitive Level: Apply
Client Needs: Physiological Integrity: Physiological Adaptation
Integrated Process: Teaching/Learning
Reference: p. 404

12. The nurse is assessing a client who is 14 hours postpartum and notes very
heavy lochia flow with large clots. Which action should the nurse prioritize?
A. Assess her blood pressure.
B. Palpate her fundus.
C. Have her turn to her left side.
D. Assess her perineum.

Answer: B

Rationale: The nurse should assess the status of the uterus by palpating the fundus
and determining its condition. If it is boggy, the nurse would then initiate fundal
massage to help it contract and encourage the passage of the lochia and any
potential clots that may be in the uterus. Assessing the blood pressure and
assessing her perineum would follow if indicated. It would be best if the woman is
in the semi-Fowler position to allow gravity to help the lochia to drain from the
uterus. The nurse would also ensure the bladder was not distended.
Question format: Multiple Choice
Chapter 19: Postpartum Woman at Risk
Cognitive Level: Apply
Client Needs: Physiological Integrity: Reduction of Risk Potential
Integrated Process: Nursing Process
Reference: p. 405

13. The nurse is caring for a postpartum woman who is diagnosed with
endometritis. Which position should the nurse encourage the client to maintain?
A. Flat in bed
B. On her left side
C. Trendelenburg
D. Semi-Fowler

Answer: D

Rationale: A semi-Fowler position encourages lochia to drain so it will not become


stagnant and cause further infection. Placing the woman flat in bed, on her left
side, or in the Trendelenburg position would not accomplish this goal and could
result in the infection spreading to other parts of the body.
Question format: Multiple Choice
Chapter 19: Postpartum Woman at Risk
Cognitive Level: Apply
Client Needs: Physiological Integrity: Reduction of Risk Potential
Integrated Process: Teaching/Learning
Reference: p. 411

14. The nurse observes an ambulating postpartum woman limping and avoiding
putting pressure on her right leg. Which assessments should the nurse prioritize in
this client?
A. Bend the knee and palpate the calf for pain.
B. Ask the client to raise the foot and draw a circle.
C. Blanch a toe, and count the seconds it takes to color again.
D. Assess for warmth, erythema, and pedal edema.

Answer: D

Rationale: This client is demonstrating potential symptoms of DVT, but is avoiding


putting pressure on the leg and limping when ambulating. DVT manifestations are
caused by inflammation and obstruction of venous return and can be assessed by
the presence of calf swelling, warmth, erythema, tenderness, and pedal edema. The
client would not need to bend the knee to assess for pain in the calf. Asking the
client to raise her toe and draw a circle is assessing reflexes, and blanching a toe is
assessing capillary refill (which may be affected by the DVT but is not indicative of
a DVT).
Question format: Multiple Choice
Chapter 19: Postpartum Woman at Risk
Cognitive Level: Apply
Client Needs: Physiological Integrity: Reduction of Risk Potential
Integrated Process: Nursing Process
Reference: p. 418-419

15. The nurse assesses the client who is 1 hour postpartum and discovers a heavy,
steady trickle of bright red blood from the vagina in the presence of a firm fundus.
Which potential cause should the nurse question and report to the RN or primary
care provider?
A. Uterine atony
B. Laceration
C. Perineal hematoma
D. Infection of the uterus

Answer: B

Rationale: A steady trickle of blood with a firm uterus is more likely to occur from a
laceration rather than from the uterine atony. This type of bleeding is usually bright
red in color rather than the dark red color of lochia. A perineal hematoma presents
as a bulging, swollen mass on the perineum. Uterine infection typically presents
with a foul smelling discharge.
Question format: Multiple Choice
Chapter 19: Postpartum Woman at Risk
Cognitive Level: Apply
Client Needs: Physiological Integrity: Physiological Adaptation
Integrated Process: Nursing Process
Reference: p. 408

16. The nurse is monitoring a client who is 5 hours postpartum and notes her
perineal pad has become saturated in approximately 15 minutes. Which action
should the nurse prioritize?
A. Initiate Ringer's lactate infusion.
B. Assess the woman's vital signs.
C. Call the woman's health care provider.
D. Assess the woman's fundus.

Answer: D

Rationale: The nurse should prioritize assessing the uterine fundus to eliminate it as
a source of the bleeding. Assessing the vital signs would be the next step,
especially if the massage is ineffective, to determine if the client is becoming
unstable. The nurse would then alert the RN or health care provider about the
increased bleeding and/or unstable vital signs. The LPN would not initiate an IV
infusion without an order from the health care provider but should be prepared to
do so, if it is ordered.
Question format: Multiple Choice
Chapter 19: Postpartum Woman at Risk
Cognitive Level: Apply
Client Needs: Physiological Integrity: Reduction of Risk Potential
Integrated Process: Nursing Process
Reference: p. 407

17. The nurse is monitoring the woman who is 1 hour postpartum and notes on
assessment the uterine fundus is boggy, to the right, and approximately 2 cm
above the umbilicus. The nurse would conclude this is most likely related to which
potential complication?
A. Urinary infection
B. Excessive bleeding
C. A ruptured bladder
D. Bladder distention

Answer: D

Rationale: The displacement of the uterus to one side is suggestive of bladder


distention. The bladder should be emptied and then fundal massage instituted to
encourage the uterus to contract and stop the excessive bleeding. If the uterus was
in the midline, then this would be related solely to uterine bleeding. It's important
to ensure the bladder is empty before starting the fundal massage to ensure the
uterus will stay contracted. A urinary infection would be noted to cause burning on
urination. A ruptured bladder would be indicative of hematuria as well as pelvic
pain.
Question format: Multiple Choice
Chapter 19: Postpartum Woman at Risk
Cognitive Level: Apply
Client Needs: Physiological Integrity: Physiological Adaptation
Integrated Process: Nursing Process
Reference: p. 407
18. A postpartum client is recovering from the birth and emergent repair of a
cervical laceration. Which sign on assessment should the nurse prioritize and
report to the health care provider?
A. Warm and flushed skin
B. Weak and rapid pulse
C. Elevated blood pressure
D. Decreased respiratory rate

Answer: B

Rationale: Excessive hemorrhage puts the client at risk for hypovolemic shock.
Signs of impending shock include a weak and rapid pulse, decreased blood
pressure, tachypnea, and cool and clammy skin. These findings should be reported
immediately to the health care provider so that proper intervention for the client
may be instituted.
Question format: Multiple Choice
Chapter 19: Postpartum Woman at Risk
Cognitive Level: Apply
Client Needs: Physiological Integrity: Reduction of Risk Potential
Integrated Process: Nursing Process
Reference: p. 404

19. The LPN has reported that uterine massage is ineffective on a client. The nurse
anticipates the health care provider will prescribe which medication to address this
issue?
A. Ibuprofen
B. Oxytocin
C. Penicillin
D. Digoxin

Answer: B

Rationale: Oxytocin is the drug used first for uterine atony. Other medications
which may be ordered include ergonovine, methylergonovine, carboprost, and
misoprostol. Ibuprofen, penicillin, or digoxin would have no effect on uterine atony.
Question format: Multiple Choice
Chapter 19: Postpartum Woman at Risk
Cognitive Level: Apply
Client Needs: Physiological Integrity: Pharmacological and Parenteral Therapies
Integrated Process: Nursing Process
Reference: p. 406

20. The nurse is monitoring several postpartum women for potential complications
related to the birthing process. Which assessment should a nurse prioritize on an
hourly basis?
A. Complete blood count
B. Vital signs
C. Pad count
D. Urine volume excreted

Answer: C

Rationale: The way to monitor for bleeding every hour is to assess pads and
percentage of the pad saturated by blood in the previous hour. It would not be
necessary to do a complete blood count every hour, nor hourly urines. Vital signs
are not typically taken every hour.
Question format: Multiple Choice
Chapter 19: Postpartum Woman at Risk
Cognitive Level: Apply
Client Needs: Health Promotion and Maintenance
Integrated Process: Nursing Process
Reference: p. 407

21. The nurse is assessing a client 48 hours postpartum and notes on assessment:
temperature 101.2oF (38.4oC), HR 82, RR 18, BP 125/78 mm Hg. The nurse should
suspect the vital signs indicate which potential situation?
A. Dehydration
B. Normal vital signs
C. Infection
D. Shock

Answer: C

Rationale: Temperatures elevated above 100.4° F (38° C) 24 hours after birth are
indicative of possible infection. All but the temperature for this client are within
normal limits, so they are not indicative of shock or dehydration.
Question format: Multiple Choice
Chapter 19: Postpartum Woman at Risk
Cognitive Level: Apply
Client Needs: Physiological Integrity: Reduction of Risk Potential
Integrated Process: Nursing Process
Reference: p. 410

22. A woman presents to the clinic at 1-month postpartum and reports her left
breast has a painful, reddened area. On assessment, the nurse discovers a localized
red and warm area. The nurse predicts the client has developed which disorder?
A. Breast yeast
B. Mastitis
C. Plugged milk duct
D. Engorgement

Answer: B

Rationale: Mastitis usually occurs 2 to 3 weeks after birth and is noted to be


unilateral. Assessment should reveal a localized reddened area that is warm and
painful to palpation. The scenario described is not indicative of a plugged milk duct
or engorgement. Yeast is not recognized to cause mastitis.
Question format: Multiple Choice
Chapter 19: Postpartum Woman at Risk
Cognitive Level: Apply
Client Needs: Physiological Integrity: Physiological Adaptation
Integrated Process: Nursing Process
Reference: p. 413

23. The nurse is preparing discharge instructions for a postpartum woman who has
developed DVT after a long and difficult birthing process. The nurse will include
instruction on which medication for this client?
A. NSAIDS
B. Anticoagulants
C. Opioid analgesics
D. Beta-blockers

Answer: B

Rationale: The nurse should instruct the client on the anticoagulant, which will be
prescribed due to the DVT. The client may be advised to use NSAIDs for pain
control. Opioid analgesics would not be appropriate, especially if the client is
breastfeeding her infant. Beta-blockers would not be appropriate for this situation.
Question format: Multiple Choice
Chapter 19: Postpartum Woman at Risk
Cognitive Level: Apply
Client Needs: Physiological Integrity: Pharmacological and Parenteral Therapies
Integrated Process: Teaching/Learning
Reference: p. 418

24. A client who underwent a cesarean birth suddenly complains of shortness of


breath and pain approximately 2 hours after delivery. Which action should the
nurse prioritize?
A. Ensure the head of the bed is at 45 degrees.
B. Apply oxygen.
C. Obtain arterial blood gas.
D. Increase IV fluid rate.

Answer: A

Rationale: The immediate response by the nurse is to ensure the head of the bed is
at 45 degrees. Then the LPN must notify the RN and/or health care provider.
Applying oxygen, obtaining an arterial blood gas specimen, or increasing the IV
fluid rate will be done on orders from the health care provider.
Question format: Multiple Choice
Chapter 19: Postpartum Woman at Risk
Cognitive Level: Apply
Client Needs: Physiological Integrity: Physiological Adaptation
Integrated Process: Nursing Process
Reference: p. 420

25. The nurse is assessing a postpartum client at a 6-week well-care check and
notes questionable behavior on assessment. Which behaviors should the nurse
prioritize and report to the RN or health care provider?
A. Tearful during appointment
B. Talkative and asking questions
C. Restless and agitated, concerned with self and not the infant
D. States being tired and happy at same time

Answer: C

Rationale: When a woman presents with restlessness, irritability and concern only
for self needs and not the infant's needs, further evaluation for possible postpartum
psychosis should be a priority. The other choices would be considered normal
reactions for a postpartum woman.
Question format: Multiple Choice
Chapter 19: Postpartum Woman at Risk
Cognitive Level: Apply
Client Needs: Psychosocial Integrity
Integrated Process: Nursing Process
Reference: p. 422

26. The nurse is interacting with a young mother and her 2-week-old infant. Which
behavior by the mother should the nurse prioritize and report to the RN or health
care provider?
A. Talking to the infant and rocking the infant
B. Not responding to the infant crying
C. Discussing her birth with another new mom
D. Breast-feeding the infant in public

Answer: B

Rationale: When a mother is not engaged with the infant and is demonstrating
signs of not providing care or responding to the infant, there is a concern about
malattachment. This needs to be reported to the health care provider for follow-up.
The other options are normal activities for a new mother who is 2 weeks
postpartum.
Question format: Multiple Choice
Chapter 19: Postpartum Woman at Risk
Cognitive Level: Apply
Client Needs: Psychosocial Integrity
Integrated Process: Nursing Process
Reference: p. 421

27. A client presents to the clinic with her 3-week-old infant reporting general flu-
like symptoms and a painful right breast. Assessment reveals temperature 101°8F
(38.8°C) and the right breast nipple with a movable mass that is red and warm.
The client is diagnosed with mastitis. Which instruction should the nurse prioritize
for this client?
A. Complete the full course of antibiotic prescribed, even if you begin to feel better.
B. Use NSAIDs, warm showers, and warm compresses to relieve discomfort.
C. Breastfeed or otherwise empty your breasts at least every 3 hours.
D. Increase your fluid intake to ensure that you will continue to produce adequate
milk.

Answer: A

Rationale: Mastitis is an infection of the breast tissue with common reports of


general flu-like symptoms that occur suddenly, along with tenderness, pain, and
heaviness in the breast. Inspection reveals erythema and edema in an area
localized to one breast, commonly in a pie-shaped wedge. The area is warm and
moves or compresses on palpation. Nursing care focuses on supporting continued
breastfeeding, preventing milk stasis and administering antibiotics for a full 10 to
14 days. The woman should empty her breasts every 1.5 to 2 hours to help prevent
milk stasis and the spread of the mastitis. The use of analgesics, warm showers,
and warm compresses to relieve discomfort may be encouraged; increasing her
fluid intake will keep the mother well-hydrated and able to produce an adequate
milk supply. However, these actions would not be considered the most important
aspects of self-care for this client at this time.
Question format: Multiple Choice
Chapter 19: Postpartum Woman at Risk
Cognitive Level: Analyze
Client Needs: Physiological Integrity: Pharmacological and Parenteral Therapies
Integrated Process: Teaching/Learning
Reference: p. 413
Chapter 20
1. When assisting in the delivery of a postterm baby, as soon as the baby's head is
delivered, the nurse is most likely to be called upon to help the physician perform
what action?
A. administer oxygen
B. perform a gastric lavage
C. suction the baby's mouth and nose
D. start IV glucose

Answer: C

Rationale: Often the postterm infant has expelled meconium in utero. At birth, the
meconium may be aspirated into the lungs, obstructing the respiratory passages
and irritating the lungs. This may lead to pneumonia. To reduce the chances of
meconium aspiration, upon delivery of the postterm newborn's head and just before
the baby takes his first breath, the physician or clinical nurse will suction the
infant's mouth and nose and also check for respiratory problems related to
meconium aspiration. After delivery, gastric lavage also may be performed to
remove any meconium swallowed and to prevent aspiration of vomitus. The
stressed postterm newborn will not tolerate the labor and delivery process too well;
supplemental oxygen therapy might be ordered for respiratory distress. Typically
postmature newborns are ravenous eaters at birth. With this in mind, if the
newborn is free from respiratory distress, the practical nurse can offer feedings at 1
or 2 hours of age, being observant for potential aspiration and possible asphyxia.
Intravenous glucose infusions may be ordered to stabilize the newborn's glucose
level.
Question format: Multiple Choice
Chapter 20: The Newborn at Risk: Gestational and Acquired Disorders
Cognitive Level: Apply
Client Needs: Health Promotion and Maintenance
Integrated Process: Nursing Process
Reference: p. 444

2. Four weeks before the birth of a client's already large child, the health care
provider has told the client that if the baby gets bigger and the baby's lungs are
ready, a cesarean birth is preferred. The woman asks the nurse what the downside
is to having a cesarean rather than a vaginal birth. What is an appropriate response
by the nurse?
A. "If the health care provider has recommended the procedure, it's likely that the
benefits outweigh the risks."
B. "The procedure isn't risky for the baby, but your healing takes longer, and you'll
have a scar."
C. "As the baby passes through the birth canal some of the excess fluid is expelled
from the lungs; if that doesn't happen there's a higher risk of respiratory distress."
D. "Some women don't have any problem giving birth to large babies. You might
want to get a second opinion."

Answer: C

Rationale: Transient tachypnea of the newborn (TTN) involves the development of


mild respiratory distress in a newborn. TTN results from a delay in absorption of
fetal lung fluid after birth. As the fetus passes through the birth canal during birth,
some of the fluid is expelled as the thoracic area is compressed. TTN is commonly
seen in newborns born by cesarean birth. It typically occurs after birth with the
greatest degree of distress occurring approximately 36 hours after birth. TTN
commonly disappears spontaneously around the third day.
Question format: Multiple Choice
Chapter 20: The Newborn at Risk: Gestational and Acquired Disorders
Cognitive Level: Apply
Client Needs: Physiological Integrity: Reduction of Risk Potential
Integrated Process: Caring
Reference: p. 444

3. The nurse is accepting a new mother and her term infant into the unit after
delivery and notes the newborn is documented as low-birth-weight. How much does
the nurse expect the newborn to weigh?
A. 1450 grams
B. 2000 grams
C. 2550 grams
D. 2950 grams

Answer: B

Rationale: There are three classifications based on weight which may be used to
classify newborns. Low-birth-weight (LBW) newborns are those who weigh from
2,499 grams to 1,500 grams; very-low-birth-weight (VLBW) newborns weigh from
1,499 grams to 1,000 grams; and extremely-low-birth-weight (ELBW) newborns
weigh less than 1,000 grams.
Question format: Multiple Choice
Chapter 20: The Newborn at Risk: Gestational and Acquired Disorders
Cognitive Level: Understand
Client Needs: Health Promotion and Maintenance
Integrated Process: Nursing Process
Reference: p. 428

4. The nurse is evaluating the neonate for gestational age. Which assessment
finding will the nurse note when determining the infant is post-term?
A. A scarf sign shows resistance and the elbow is unable to reach midline.
B. Breast buds are 4.5 mm and have a raised areola.
C. Flexed positions show good muscle tone.
D. Ear cartilage is thick and the pinna is stiff.
Answer: D

Rationale: In the post-term newborn, the ear cartilage is thick and the pinna is
stiff; in the term newborn, cartilage is present within the pinna and the ear shows
the ability for natural recoil when folded. In both post-term and term newborns, a
scarf sign test shows resistance and the elbow is unable to reach midline. A post-
term newborn will have full areolas with 5 to 10 mm breast buds and will be
capable of full flexion of arms and legs.
Question format: Multiple Choice
Chapter 20: The Newborn at Risk: Gestational and Acquired Disorders
Cognitive Level: Apply
Client Needs: Health Promotion and Maintenance
Integrated Process: Nursing Process
Reference: p. 430

5. The nurse is weighing and measuring a term newborn. Which assessment


findings would indicate that this newborn is suffering from asymmetrical growth
restriction?
A. looks wasted and has poor skin turgor
B. is pale with loose, dry skin
C. has cracked and leathery skin
D. has very thin skin and has multiple visible veins

Answer: B

Rationale: Asymmetrically growth-restricted newborns typically appear thin and


pale with loose dry skin. The symmetrically growth-restricted newborn typically
appears wasted with poor skin turgor. Post-term babies have skin that is leathery,
cracked, and wrinkled. Preterm babies have skin that is very thin with little
subcutaneous fat and easily visible veins.
Question format: Multiple Choice
Chapter 20: The Newborn at Risk: Gestational and Acquired Disorders
Cognitive Level: Apply
Client Needs: Health Promotion and Maintenance
Integrated Process: Nursing Process
Reference: p. 430

6. The nurse is caring for a pregnant woman who is struggling with controlling
gestational diabetes mellitus. What effect does the nurse predict this situation may
have on the fetus?
A. have a serious birth defect
B. grow to an unusually large size
C. suffer from symmetrical intrauterine growth restriction
D. suffer from asymmetrical intrauterine growth restriction

Answer: B
Rationale: Maternal diabetes is the most widely known contributing factor to large-
for-gestational-age newborns. LGA babies are frequently born to mothers with
diabetes and poor glucose control. Continued high blood glucose levels in the
mother lead to an increase in insulin production in the fetus. Increased insulin
levels act as a fetal growth hormone causing macrosomia, an unusually large
newborn with a birth weight of greater than 4,500 grams (9 pounds, 14 ounces).
The incidence of birth defects in newborns born of a mother with gestational
diabetes is not greatly increased. IUGR is not a typical outcome of uncontrolled
gestational diabetes. It is more likely that the newborn will be large-for-gestational-
age.
Question format: Multiple Choice
Chapter 20: The Newborn at Risk: Gestational and Acquired Disorders
Cognitive Level: Understand
Client Needs: Physiological Integrity: Physiological Adaptation
Integrated Process: Nursing Process
Reference: p. 432

7. The nurse is preparing a presentation for a health fair presenting the risks which
can lead to sudden infant death syndrome (SIDS). Which factors would the nurse
include as increasing the risk for SIDS?
A. a low-birth-weight baby girl, March birth, middle-class, 25-year-old G2P2
nonsmoker
B. an average weight baby boy, January birth, a poor, 16-year-old G1P1 nonsmoker
C. a low-birth-weight baby boy, November birth, wealthy, educated, 19-year-old
G1P1 smoker
D. An average weight baby boy, April birth, poor, 27 year old G2P2 nonsmoker

Answer: C

Rationale: No single cause of SIDS has been identified but there are contributing
factors. Infants who die of SIDS are usually 2 to 4 months old, although some
deaths have occurred during the first or second week of life. Few infants older than
6 months of age die of SIDS. It is a greater threat to low-birth-weight infants than
to term infants. It occurs more often in winter and affects more male infants than
female infants as well as more infants from minority and lower socioeconomic
groups. Infants born to mothers younger than 20 years of age, infants who are not
a firstborn child, and infants whose mothers smoked during pregnancy also have
been found to be at greater risk.
Question format: Multiple Choice
Chapter 20: The Newborn at Risk: Gestational and Acquired Disorders
Cognitive Level: Apply
Client Needs: Health Promotion and Maintenance
Integrated Process: Teaching/Learning
Reference: p. 445

8. The nurse is caring for a 22-hour-old neonate male and notes on assessment at
the beginning of the shift: Apgar score of 9, nursing without difficulty, and
appeared healthy. As the nurse's shift goes on, subsequent assessment reveals his
sclera and skin have begun to take on a yellow hue. The nurse would report this as
a possible indication of what condition?
A. heroin withdrawal
B. hypoglycemia
C. hypoxia
D. hemolytic disease

Answer: D

Rationale: Any infant admitted to the newborn nursery should be examined for
jaundice during the first 36 hours or more. Early development of jaundice (within
the first 24 hours) is a probable indication of hemolytic disease. Heroin withdrawal
symptoms commonly include tremors, restlessness, hyperactivity, disorganized or
hyperactive reflexes, increased muscle tone, sneezing, tachypnea, vomiting,
diarrhea, disturbed sleep patterns, and a shrill high-pitched cry. The hypoglycemic
newborn's blood glucose would be low, and a newborn with hypoxia would show
signs of respiratory distress.
Question format: Multiple Choice
Chapter 20: The Newborn at Risk: Gestational and Acquired Disorders
Cognitive Level: Apply
Client Needs: Physiological Integrity: Physiological Adaptation
Integrated Process: Nursing Process
Reference: p. 447

9. The nurse is assessing a toddler at a well-child visit and notes the following:
small in stature, appears mildly developmentally delayed; short eyelid folds; and
the nose is flat. Which advice should the nurse prioritize to the mother in response
to her questions about having another baby?
A. "It's a good idea to stop drinking alcohol 3 months before trying to get
pregnant."
B. "It's important to add iron and vitamin B supplements to your diet."
C. "It would be good to stop smoking before getting pregnant."
D. "It's important to keep insulin levels controlled during pregnancy."

Answer: A

Rationale: Alcohol is one of the many teratogenic substances that cross the
placenta to the fetus. Fetal alcohol spectrum disorder is often apparent in newborns
of mothers with chronic alcoholism and sometimes appears in newborns whose
mothers consume low-to-moderate amounts of alcohol. No amount of alcohol is
believed to be safe, and women should stop drinking at least 3 months before they
plan to become pregnant. The ability of the mother's liver to detoxify the alcohol is
apparently of greater importance than the actual amount consumed. Fetal alcohol
spectrum disorder is characterized by low birth weight, smaller height and head
circumference, short palpebral fissures (eyelid folds), reduced ocular growth, and a
flattened nasal bridge. These newborns are prone to respiratory difficulties,
hypoglycemia, hypocalcemia, and hyperbilirubinemia. Their growth continues to be
slow, and their mental development is delayed despite expert care and nutrition.
Smoking is related to respiratory issues. Proper nutrition and glucose control are
also important but do not result in fetal alcohol spectrum disorder.
Question format: Multiple Choice
Chapter 20: The Newborn at Risk: Gestational and Acquired Disorders
Cognitive Level: Apply
Client Needs: Health Promotion and Maintenance
Integrated Process: Teaching/Learning
Reference: p. 449

10. The nurse is assessing a newborn and notes that the size of the infant will
necessitate classification as large for gestational age. When questioned by the
mother as to what this means, the nurse should point out the infant is at which
percentile?
A. 86th percentile
B. 88th percentile
C. 90th percentile
D. 92nd percentile

Answer: D

Rationale: The large-for-gestational-age (LGA) infant is above the 90th percentile


for gestational age using the weight, length, and head circumference as a guideline.
Appropriate-for-gestational-age infants are in the 10th percentile to 90th percentile.
Infants below than 10th percentile are considered small-for-gestational-age.
Question format: Multiple Choice
Chapter 20: The Newborn at Risk: Gestational and Acquired Disorders
Cognitive Level: Apply
Client Needs: Health Promotion and Maintenance
Integrated Process: Nursing Process
Reference: p. 428

11. The nursing instructor is conducting a discussion centered on the various


methods used to describe an infant. The instructor determines the session is
successful when the students correctly choose which as an indication of gestational
age?
A. the weight, height, and length of the newborn at birth
B. the length of time between fertilization of the egg and birth
C. the age of the newborn who is born before 40 weeks
D. the newborn according to their birth assessment

Answer: B

Rationale: Gestational age is the length of time between fertilization of the egg and
birth. Size classifications consider the newborn's weight, length, and head
circumference and include SGA, AGA, and LGA. Infants born between 20 and 37
weeks' gestation are considered preterm. Infants born between 38 and 42 weeks'
gestation are considered term, but this does not determine gestational age. Birth
assessment can help establish gestational age based on the findings; however, it is
not what defines gestational age.
Question format: Multiple Choice
Chapter 20: The Newborn at Risk: Gestational and Acquired Disorders
Cognitive Level: Analyze
Client Needs: Health Promotion and Maintenance
Integrated Process: Teaching/Learning
Reference: p. 428

12. The nurse is assessing a newborn to establish a gestational age. Which factors
will the nurse prioritize when assessing the newborn's neuromuscular maturity?
Select all that apply.
A. skin
B. posture
C. arm recoil
D. heel to ear
E. plantar creases
F. scarf sign

Answer: B, C, D, F

Rationale: The Ballard scoring system is a common gestational age assessment tool
used in assessing the newborn. Posture, arm recoil, heel to ear, and scarf sign are
the categories used to assess neuromuscular maturity. In addition, the square
window and popliteal angle are categories used in this assessment tool for
neuromuscular maturity. The skin and plantar creases are categories used by the
Ballard scoring system to assess physical maturity of the infant.
Question format: Multiple Select
Chapter 20: The Newborn at Risk: Gestational and Acquired Disorders
Cognitive Level: Apply
Client Needs: Health Promotion and Maintenance
Integrated Process: Nursing Process
Reference: p. 428

13. The nurse has completed an assessment on a newborn and documents a score
of 17 for the physical maturity in the records. Which elements has the nurse
prioritized for this assessment? Select all that apply.
A. skin
B. posture
C. breast buds
D. square window
E. plantar creases
F. lanugo

Answer: A, C, E, F

Rationale: The Ballard scoring system assessment tool is used to determine


gestational age of an infant. The skin, lanugo, plantar creases, breast buds, and
ears are the categories which measure the physical maturity of an infant. Posture
and square window are categories used in the neuromuscular maturity assessment
of the Ballard scoring system.
Question format: Multiple Select
Chapter 20: The Newborn at Risk: Gestational and Acquired Disorders
Cognitive Level: Apply
Client Needs: Health Promotion and Maintenance
Integrated Process: Nursing Process
Reference: p. 428

14. The nurse is caring for a new infant and notes on assessment the newborn is
small for gestational age and also has indications of intrauterine growth restriction.
Which assessments should the nurse prioritize about the mother as a potential
cause for the infant's condition?
A. previous smoking history
B. blood glucose levels
C. number of normal pregnancies
D. use of food stamp program during pregnancy

Answer: B

Rationale: Uncontrolled maternal diabetes can be a contributing factor for the infant
with intrauterine growth restriction. Smoking during pregnancy could be a
contributing factor, but being a previous smoker would not affect this pregnancy.
Inadequate maternal nutrition is a contributing factor, but because this mother was
on a food stamp program she was more likely to have had adequate nutrition
during pregnancy. Previous pregnancies with a history of IUGR or other poor
pregnancy outcomes would be a possible contributing factor, but not normal
pregnancies.
Question format: Multiple Choice
Chapter 20: The Newborn at Risk: Gestational and Acquired Disorders
Cognitive Level: Apply
Client Needs: Physiological Integrity: Reduction of Risk Potential
Integrated Process: Nursing Process
Reference: p. 4361

15. A nurse is caring for a newborn who is determined to be small-for-gestational-


age with intrauterine growth restriction. Which finding would lead the nurse to also
question if this infant has asymmetric growth restriction?
A. The head is large in comparison with the body.
B. The head and body parts are in proportion.
C. The length falls below the 10th percentile.
D. The length is above the 10th percentile.

Answer: A

Rationale: The asymmetrically growth-restricted newborn's head is large in


comparison with the body. When weight, length, and head circumference are
plotted on a standard growth chart, one or two of the measurements falls below the
10th percentile. These newborns typically have normal measurements for head
circumference and length. Although the infant's length may fall below the 10th
percentile, this is not common. When the growth pattern is symmetrical, both head
and body parts are in proportion but are below normal size for gestational age.
Weight, length, and head circumference fall below the 10th percentile when plotted
on a standard growth chart.
Question format: Multiple Choice
Chapter 20: The Newborn at Risk: Gestational and Acquired Disorders
Cognitive Level: Apply
Client Needs: Health Promotion and Maintenance
Integrated Process: Nursing Process
Reference: p. 430

16. During a clinical conference, a group of nursing students are discussing a


newborn that is large-for-gestational-age. The instructor determines the students
have successfully differentiated the potential cause after choosing which
contributing maternal factor?
A. being 30 pounds overweight before getting pregnant
B. mother is 5 feet 3 inches (1.6 m) tall and the father is 5 feet 10 inches (1.78 m)
tall
C. a blood glucose of 100 several times during the pregnancy
D. being a 19-year-old G2P2

Answer: A

Rationale: Obesity is a strong contributing factor in infants who are large-for-


gestational-age. Parents of large stature have an increased tendency to reproduce
an LGA newborn. Maternal diabetes, particularly if it is poorly controlled, is the
strongest known contributing factor, but a glucose level of 100 would not indicate
the mother was diabetic. Multiparous women have a higher incidence of large-for-
gestational-age infants.
Question format: Multiple Choice
Chapter 20: The Newborn at Risk: Gestational and Acquired Disorders
Cognitive Level: Apply
Client Needs: Physiological Integrity: Physiological Adaptation
Integrated Process: Nursing Process
Reference: p. 432

17. The nurse is preparing a presentation for a health fair that will illustrate various
factors that contribute to preterm births. Which contributing factor should the nurse
prioritize?
A. tocolytics used to relax the uterus
B. corticosteroids that are used to enhance lung maturity
C. fertility treatments that are resulting in multiple births
D. Antibiotics used to treat prenatal infections

Answer: C
Rationale: The main cause of an increase in the incidence of preterm birth is
advances in fertility treatments resulting in multiple births. Multiple births are often
preterm because of polyhydramnios (excessive amniotic fluid), a larger than
average intrauterine mass, and/or early cervical dilation (dilatation). Tocolytics help
to decrease the number of preterm births and corticosteroids are used if preterm
birth is anticipated. Although intrauterine infections can contribute to preterm
births, the use of antibiotics is less likely to cause a preterm birth.
Question format: Multiple Choice
Chapter 20: The Newborn at Risk: Gestational and Acquired Disorders
Cognitive Level: Apply
Client Needs: Physiological Integrity: Physiological Adaptation
Integrated Process: Nursing Process
Reference: p. 433

18. The nurse is assessing the newborn male of a teen mother who was afraid to
seek appropriate prenatal care. Which assessment finding should lead the nurse to
question if this infant is preterm?
A. smooth skin
B. lanugo on the back and shoulders
C. descended testicles
D. subcutaneous fat on the extremities

Answer: B

Rationale: The preterm infant has lanugo over the extremities, back, and shoulders.
Compared with the term infant, the preterm infant has translucent and usually
wrinkled skin, little muscle or subcutaneous fat, and the testes are undescended in
the male.
Question format: Multiple Choice
Chapter 20: The Newborn at Risk: Gestational and Acquired Disorders
Cognitive Level: Apply
Client Needs: Health Promotion and Maintenance
Integrated Process: Nursing Process
Reference: p. 430

19. The nursing instructor is leading a discussion with a group of nursing students
who are analyzing the preterm infant's physiologic immaturity and the associated
difficulties the newborn and family must deal with. The instructor determines the
session is successful when the students correctly choose which body system that
presents with the most critical concerns related to this immaturity?
A. the genitourinary system
B. the musculoskeletal system
C. the endocrine system
D. the respiratory system

Answer: D
Rationale: Immaturity causes difficulties involving all body systems, the most
critical of which is the respiratory system, the last to fully develop. Typically,
respirations are shallow, rapid, and irregular with periods of apnea (absence of
breathing) that lasts for at least 20 seconds or that causes cyanosis and/or
bradycardia. Although difficulties may occur in the other systems, the most critical
is the respiratory system.
Question format: Multiple Choice
Chapter 20: The Newborn at Risk: Gestational and Acquired Disorders
Cognitive Level: Analyze
Client Needs: Physiological Integrity: Physiological Adaptation
Integrated Process: Teaching/Learning
Reference: p. 433

20. The nurse cares for preterm infants and assesses them for potential
complications to provide adequate countermeasures to prevent father
complications. Which complication should the nurse prioritize and initiate proper
measures to protect the newborn?
A. decreased muscle tone
B. loss of body heat
C. excess antibodies acquired from the mother
D. increased caloric intake

Answer: B

Rationale: Thermoregulation and maintaining fluid and electrolyte balance are


major problems for the preterm newborn because he or she loses heat and fluids
more quickly and has fewer compensatory mechanisms than does the term
newborn. Although the preterm newborn does have decreased muscle tone, the
loss of body heat is a more critical concern. The preterm newborn does not receive
enough antibodies from the mother and cannot produce them, and subsequently
has a decrease rather than an excess. The preterm newborn has high caloric needs
but has a small stomach and a digestive system that may be unprepared to receive
and digest food; therefore, the newborn often has a decreased caloric intake.
Question format: Multiple Choice
Chapter 20: The Newborn at Risk: Gestational and Acquired Disorders
Cognitive Level: Analyze
Client Needs: Physiological Integrity: Physiological Adaptation
Integrated Process: Nursing Process
Reference: p. 433

21. The nurse is teaching new parents about their premature newborn who was
born with respiratory distress syndrome (RDS). The nurse determines the teaching
session is successful when the parents correctly choose which explanation as being
the cause of their newborn's condition?
A. The lungs are hyperextended due to increased load of work.
B. The infant has inherited allergies from the mother.
C. The lungs are immature and deficient in surfactant.
D. The mother has a history of asthma which interfered in lung development.
Answer: C

Rationale: In RDS, the premature infant's lungs are deficient in surfactant and thus
collapse after each breath, greatly increasing the work of breathing. The other
choices do not contribute to the immaturity of the infant's lungs and may not have
any effect on the infant's lungs at all. The lungs will more likely be hypoextended
instead of hyperextended. Inherited allergies and asthma should not effect the lack
of surfactant.
Question format: Multiple Choice
Chapter 20: The Newborn at Risk: Gestational and Acquired Disorders
Cognitive Level: Analyze
Client Needs: Health Promotion and Maintenance
Integrated Process: Teaching/Learning
Reference: p. 435

22. The nurse assesses preterm infants as they come for routine well-baby
checkups. The nurse will carefully assess the infant's vision to assess for which
potential complication related to their birth?
A. cataracts
B. amblyopia
C. nystagmus
D. retinopathy

Answer: D

Rationale: Retinopathy of prematurity (ROP) is a form of retinopathy (degenerative


disease of the retina) commonly associated with the preterm newborn. Cataracts is
common among older adults and results from the lens becoming opaque.
Amblyopia, or lazy eye, is not related to gestational age and is assessed when the
child is a toddler. Nystagmus is also not related to gestational age but to neurologic
dysfunction of the eye.
Question format: Multiple Choice
Chapter 20: The Newborn at Risk: Gestational and Acquired Disorders
Cognitive Level: Understand
Client Needs: Health Promotion and Maintenance
Integrated Process: Nursing Process
Reference: p. 435

23. The nurse is preparing a nursing care plan for a preterm infant in the newborn
nursery. Which nursing diagnoses could the nurse determine to be appropriate for
this infant? Select all that apply.
A. Ineffective breathing pattern
B. Ineffective thermoregulation
C. Risk for fluid volume excess
D. Risk for imbalanced nutrition: more than body requirements
E. Risk for impaired skin integrity
Answer: A, B, E

Rationale: Nursing diagnoses for a preterm newborn could include the following:
Ineffective breathing pattern related to an immature respiratory system; Ineffective
thermoregulation related to immaturity and transition to extrauterine life; and Risk
for impaired skin integrity related to prematurity and exposure to phototherapy
light. Additional diagnoses could include the following: Risk for ineffective infant
feeding pattern related to an inability to suck instead of imbalanced nutrition, more
than body requirements; and Risk for deficit fluid volume instead of Risk for fluid
volume excess.
Question format: Multiple Select
Chapter 20: The Newborn at Risk: Gestational and Acquired Disorders
Cognitive Level: Apply
Client Needs: Safe, Effective Care Environment: Management of Care
Integrated Process: Nursing Process
Reference: p. 436

24. The nurse is admitting a newborn into the nursery and notes the newborn was
born at 42 weeks' gestation. Which action should the nurse prioritize when caring
for this infant?
A. Monitor the newborn's blood glucose levels.
B. Wrap the newborn lightly in a single blanket.
C. Ensure feedings are offered every 4 hours.
D. Anticipate a blood transfusion will be required.

Answer: A

Rationale: The infant is a post-term infant because of the gestational age. In the
last weeks of gestation, the infant relies on glycogen for nutrition. This depletes the
liver glycogen stores and may lead to neonatal hypoglycemia, which will require
monitoring of serial blood glucose levels. The newborn may require intravenous
glucose infusions to stabilize the glucose level. Post-term newborns can have
thermoregulation concerns, so a thermoneutral environment such as a radiant heat
warmer or isolette may be needed. Use measures to minimize heat loss. Post-term
newborns are ravenous eaters at birth and may be offered feedings at 1 or 2 hours
of age and as desired after that. Although exchange transfusion may be done to
prevent or treat hyperviscosity, this is not a common concern so the glucose levels
are a higher priority.
Question format: Multiple Choice
Chapter 20: The Newborn at Risk: Gestational and Acquired Disorders
Cognitive Level: Apply
Client Needs: Health Promotion and Maintenance
Integrated Process: Nursing Process
Reference: p. 444

25. The nursing instructor is conducting a session with a group of nursing students
researching potential respiratory difficulties in newborns. The instructor determines
the session is successful after the students correctly choose which contributing
factor for transient tachypnea of the newborn?
A. usually occurs with maternal history of hypertension
B. associated with fetal distress during labor
C. often seen with advanced gestational age
D. often seen with cesarean births

Answer: D

Rationale: Transient tachypnea of the newborn (TTN) commonly occurs in newborns


born via cesarean birth. The newborn does not experience the compression of the
thoracic cavity that occurs with passage through the birth canal, so he or she
retains some fluid in the lungs that usually squeezes out as the thoracic area is
compressed during a vaginal birth. Maternal hypertension (which can advance into
preeclampsia or eclampsia); fetal distress; and advanced gestational age can all be
contributing factors for the decision of the health care provider to perform a
cesarean, but they are not recognized as a direct cause for TTN.
Question format: Multiple Choice
Chapter 20: The Newborn at Risk: Gestational and Acquired Disorders
Cognitive Level: Apply
Client Needs: Physiological Integrity: Physiological Adaptation
Integrated Process: Teaching/Learning
Reference: p. 444

26. The nurse is examining the morning laboratory results of the newborns of
mothers with diabetes. Which report finding should the nurse prioritize?
A. hypocalcemia
B. hypermagnesemia
C. hypobilirubinemia
D. hyperkalemia

Answer: A

Rationale: The newborn of the mother with diabetes is at risk for hypoglycemia,
hypocalcemia, hypomagnesemia (not hypermagnesemia), polycythemia with
hyperviscosity, and hyperbilirubinemia (not hypobilirubinemia). Potassium concerns
(hyperkalemia or hypokalemia) are not a risk for these newborns.
Question format: Multiple Choice
Chapter 20: The Newborn at Risk: Gestational and Acquired Disorders
Cognitive Level: Apply
Client Needs: Physiological Integrity: Reduction of Risk Potential
Integrated Process: Nursing Process
Reference: p. 448

27. The nurse is admitting to the nursery a newborn of a mother who continued to
drink alcohol during her pregnancy. Which finding does the nurse predict to
encounter on the newborn's assessment?
A. above average birth weight
B. large head circumference
C. lethargic and sleepy
D. hyperactive and irritable

Answer: D

Rationale: The newborn that is withdrawing from alcohol typically is hyperactive,


irritable, has trouble sleeping, and may have tremors or seizures. Characteristics of
fetal alcohol spectrum disorder include low birth weight, and small height and head
circumference. This newborn is prone to respiratory difficulties, hypoglycemia,
hypocalcemia, and hyperbilirubinemia.
Question format: Multiple Choice
Chapter 20: The Newborn at Risk: Gestational and Acquired Disorders
Cognitive Level: Apply
Client Needs: Physiological Integrity: Physiological Adaptation
Integrated Process: Nursing Process
Reference: p. 449

28. The nurse is caring for a newborn who is receiving phototherapy for hemolytic
disease. As the nurse explains the procedure to the mother, which instruction
should the nurse prioritize?
A. Leave the light off for 1 hour six times a day.
B. Dress the newborn in a lightweight gown at night.
C. Turn the newborn every 3 to 4 hours.
D. Remove and change the eye patches every hour.

Answer: C

Rationale: The nurse should turn the newborn every 2 hours to rotate the area of
exposure. Do not turn off the lights except to feed and to change the diaper. The
infant is nude to maximize the skin surface area exposed to the light. Remove the
patches every 4 hours to cleanse the eyes and examine for irritation, inflammation,
and/or dryness. Clean and change the patches daily.
Question format: Multiple Choice
Chapter 20: The Newborn at Risk: Gestational and Acquired Disorders
Cognitive Level: Apply
Client Needs: Physiological Integrity: Physiological Adaptation
Integrated Process: Teaching/Learning
Reference: p. 446
Chapter 21
1. The nurse is preparing pre- and postoperative instructions for a family whose 6-
month-old infant is scheduled for an initial surgical repair of a cleft lip and palate.
Which activity should the nurse prioritize in the instructions to the caregivers of
this infant?
A. Feed the infant with a drinking straw.
B. Feed the infant with a rubber covered spoon.
C. Train the infant to drink from a glass or cup.
D. Let the infant become accustomed to being in elbow restraints.

Answer: D

Rationale: The thumb, although comforting, may quickly undo the repair or cause
undesirable scarring along the suture line. The infant's ultimate happiness and well-
being must take precedence over immediate satisfaction. Acclimating the infant to
elbow restraints gradually before admission is helpful. For an infant who has had a
palate repair, no nipples, spoons, or straws are permitted; only a drinking glass or
a cup is recommended.
Question format: Multiple Choice
Chapter 21: The Newborn at Risk: Congenital Disorders
Cognitive Level: Apply
Client Needs: Physiological Integrity: Reduction of Risk Potential
Integrated Process: Teaching/Learning
Reference: p. 473

2. The parents of a newborn are struggling with the news that their infant has spina
bifida. Which technique should the nurse prioritize teaching to the parents that will
help increase the infant's comfort and development?
A. Diaper the baby safely.
B. Hold the baby during feeding.
C. Cuddle the baby in a chest-to-chest position.
D. Clean and moisturize the myelomeningocele sac.

Answer: C

Rationale: The family of a newborn with such a major anomaly is in a state of shock
on first learning of the problems. Be especially sensitive to their needs and
emotions. Encourage family members to express their feelings and emotions as
openly as possible. If possible, encourage the family members to cuddle or touch
the newborn using proper precautions for the safety of the defect. With the
permission of the physician, the newborn may be held in a chest-to-chest position
to provide closer contact. The preoperative goals for care of the newborn with
myelomeningocele include preventing infection, maintaining skin integrity,
preventing trauma related to disuse, increasing family coping skills, education
about the condition, and support. Diapering is not advisable with a low defect.
Question format: Multiple Choice
Chapter 21: The Newborn at Risk: Congenital Disorders
Cognitive Level: Apply
Client Needs: Health Promotion and Maintenance
Client Needs: Physiological Integrity: Basic Care and Comfort
Integrated Process: Teaching/Learning
Reference: p. 458-459

3. A nurse is providing postoperative care to an infant who had a ventriculoarterial


shunt placed. Approximately 8 hours after surgery, the nurse notes on assessment
shrill crying and projective vomiting. Which response should the nurse prioritize at
this time?
A. Assess surgical site for signs of infection.
B. Increase the flow of IV fluids and maintain NPO status.
C. Assess and administer pain medication.
D. Notify the primary care provider immediately.

Answer: D

Rationale: The projectile vomiting should raise suspicions of increasing intracranial


pressure and requires emergent intervention, so the nurse should notify the
primary care provider immediately. Symptoms of increased intracranial pressure
(ICP) may also include irritability, restlessness, personality change, high-pitched
cry, ataxia, failure to thrive, seizures, severe headache, changes in level of
consciousness, and papilledema. At least every 2 to 4 hours, the nurse should
monitor the newborn's level of consciousness, check the pupils for equality and
reaction, monitor the neurologic status, and observe for a shrill cry, lethargy, or
irritability. Edema and localized redness at the surgical site are potential indications
of an infection. Assessing for pain and administering pain medication in this
situation can result in the symptoms being masked and the infant could die.
Increasing the fluid rate could contribute to the increased volume of fluids in the
brain and would exacerbate the situation.
Question format: Multiple Choice
Chapter 21: The Newborn at Risk: Congenital Disorders
Cognitive Level: Apply
Client Needs: Physiological Integrity: Reduction of Risk Potential
Integrated Process: Nursing Process
Reference: p. 459

4. The nurse is assessing a 4-year-old male born with a heart condition who is
brought in for a routine well-child visit by his parents. The parents report he is very
curious, active, and very social; however, they often see him take breaks in his play
by squatting for a few minutes or sitting on the sidelines at which time they insist
he take a nap. Assessment reveals a child small for his age, mildly cyanotic, and
tires easily. What is the best response to the parents when they ask the nurse for
suggestions on how to encourage their son to take the naps they insist on but he
doesn't want to take?
A. "It's important to limit Stevie's physical exertion, so it is good to bring him inside
when you think he needs to rest."
B. "Children are often aware of their limitations, and because he has shown that he
knows when he needs to take a break he should be encouraged to control his own
activity level."
C. "He might do well with a scheduled 15-minute break for every 30 minutes of
active play."
D. "He might not need to nap, but it would be sensible to call him in so that you can
check his pulse if he squats or takes a rest."

Answer: B

Rationale: Children are rather sensible about finding their own limitations and
usually limit their activities to their capacity if they are not made unduly
apprehensive. A child with congenital heart disease should be allowed to lead as
normal a life as possible. Families are naturally apprehensive and find it difficult not
to overprotect the child. They often increase the child's anxiety and cause fear in
the child about participating in normal activities. The child may assume a squatting
position, which reduces the return flow to the heart, thus temporarily reducing the
workload of the heart. The other choices take the control away from the child and
can increase his or her anxiety level, thereby increasing the work of the heart.
Question format: Multiple Choice
Chapter 21: The Newborn at Risk: Congenital Disorders
Cognitive Level: Apply
Client Needs: Health Promotion and Maintenance
Integrated Process: Teaching/Learning
Reference: p. 467

5. A 2-year old child has gone home following successful hip dysplasia surgery in a
spica cast. Her caregiver calls 2 days later to report the child has been vomiting
after eating, but has no fever. Which response should the nurse prioritize in
response to this caregiver?
A. "If the child develops a fever or isn't able to hold anything down over the next
24 hours she should come back to the hospital."
B. "The vomiting is a possible sign of infection, and the child should be brought
back in."
C. "The child may be reacting to the analgesics. Observe the child for the next 8
hours and see what happens."
D. If there is no fever and the child wants to eat, the cast may be too tight; she
should be brought back in for recasting."

Answer: D

Rationale: For a child in a spica cast, vomiting after a feeding may be an indication
that the cast is too tight over the stomach. The cast may have to be removed and
reapplied. Although vomiting may be a sign the child is ill or is reacting to
analgesics, the child needs to be seen by the care provider to rule out complications
relating to the cast, rather than to wait for further symptoms to develop.
Question format: Multiple Choice
Chapter 21: The Newborn at Risk: Congenital Disorders
Cognitive Level: Apply
Client Needs: Physiological Integrity: Physiological Adaptation
Integrated Process: Nursing Process
Reference: p. 482

6. The nurse is working with an adult female who has PKU and desires to become
pregnant. The nurse notes on her assessment her current serum phenylalanine
level is 10 mg/dl. Which instruction should the nurse prioritize for this client?
A. "Think carefully about the decision. The child might be intellectually disabled
since your PKU is inherited, especially if your levels stay high.
B. "It will be best if you cut back on meat, fish, and dairy products before you
become pregnant to get your serum phenylalanine level down under 8 mg."
C. "It will be best if you cut back on vegetables and fruit before you become
pregnant to get your serum phenylalanine level down under 8 mg."
D. "The baby won't be able to breastfeed. You know breastfeeding is really the best
way to care for a newborn."

Answer: B

Rationale: If a woman with PKU decides to have a child and is not following a diet
low in phenylalanine, she should return to following the dietary treatment for at
least 3 months before becoming pregnant. A low-phenylalanine diet is a very
restricted one: foods to be omitted are breads, meat, fish, dairy products, nuts, and
legumes. The diet is continued through the pregnancy to help prevent the child
from being born with an intellectual disability. Routine blood testing is done to
maintain the serum phenylalanine level at 2 to 8 mg/dl. A formula low in
phenylalanine should be started as soon as the condition is detected; Lofenalac and
Phenyl-free are low-phenylalanine formulas. Best results are obtained if the special
formula is started before the newborn is 3 weeks of age.
Question format: Multiple Choice
Chapter 21: The Newborn at Risk: Congenital Disorders
Cognitive Level: Apply
Client Needs: Physiological Integrity: Basic Care and Comfort
Integrated Process: Teaching/Learning
Reference: p. 483

7. A nurse is providing information for a pregnant woman who has just discovered
that the fetus she's carrying is likely to have Down syndrome. Which statement by
the nurse is most accurate regarding the possible concerns for a child with Down
syndrome?
A. They all suffer from severe intellectual disabilities.
B. They have a higher risk of developing leukemia than those in the general
population.
C. They are cognitively impaired and often threaten the safety of others in their
homes and neighborhoods.
D. They have so many needs and demands on the family that the family structure
rarely holds up.

Answer: B

Rationale: People with Down syndrome have a greater susceptibility to leukemia


than that of the general population. Mental status is usually within the moderate to
severe range of intellectual disability, with most children being moderately disabled.
A cognitively impaired child who is undisciplined or improperly supervised may
threaten the safety of others in the home and the neighborhood, and caring for the
child may demand so much sacrifice from other family members that the family
structure may be significantly affected. However, with consistent care, patience,
and guidelines, families of children with Down syndrome often find joy and pleasure
in the gentle and loving nature of the child.
Question format: Multiple Choice
Chapter 21: The Newborn at Risk: Congenital Disorders
Cognitive Level: Apply
Client Needs: Physiological Integrity: Physiological Adaptation
Integrated Process: Teaching/Learning
Reference: p. 485

8. A caregiver brings a 13-year-old male for a pre-high school checkup and reports
he has spent lots of time in the principal's office or serving detention during junior
high, and questions if he is too immature to be in high school. The nurse's
assessment reveals evident breasts, little underarm or chest hair, and a high-
pitched voice. Which condition should the nurse suspect and discuss with the
primary care provider?
A. Turner syndrome
B. Klinefelter syndrome
C. ambiguous genitalia
D. hypothyroidism

Answer: B

Rationale: The presence of an extra X chromosome causes Klinefelter syndrome,


most commonly seen in males. Characteristics are not often evident until puberty,
when the child does not develop secondary sex characteristics. The testes are
usually small and do not produce mature sperm. Increased breast size and a risk of
developing breast cancer are frequently seen. Boys with Klinefelter syndrome often
have normal intelligence but frequently have behavior problems, show signs of
immaturity and insecurity, and have difficulty with memory and processing.
Characteristics of Turner syndrome include short stature, low set ears, a broad-
based neck that appears webbed and short, a low-set hairline at the nape of the
neck, broad chest, an increased angle of the arms, and edema of the hands and
feet. Females are most often affected by Turner syndrome and do not develop
secondary sex characteristics with the exception of pubic hair. If it is not possible to
determine by observation at birth if the child is a male or female, the condition is
called ambiguous genitalia. In hypothyroidism, the thyroid hormone must be
replaced as soon as the diagnosis is made. Unless therapy is started in early
infancy, intellectual disability and slow growth occur. The later that therapy is
started, the more severe the intellectual disability.
Question format: Multiple Choice
Chapter 21: The Newborn at Risk: Congenital Disorders
Cognitive Level: Apply
Client Needs: Physiological Integrity: Physiological Adaptation
Integrated Process: Nursing Process
Reference: p. 486

9. The nursing instructor is teaching a session on techniques that the nursing


students can use to properly address concerns of parents with children who are
born with a cleft lip and palate. The instructor determines the session is successful
when the students correctly choose which nursing intervention as being the most
effective in these situations?
A. Help the child to understand his or her limitations.
B. Keep the family informed about new and effective treatments.
C. Model good medical practices for the child's family.
D. Use reflective listening with nonjudgmental support.

Answer: D

Rationale: Families are naturally apprehensive and find it difficult not to overprotect
a child who is ill. They often increase the child's anxiety and cause fear in the child
about participating in normal activities. Children are rather sensible about finding
their own limitations and usually limit their activities to their capacity if they are not
made unduly apprehensive. Some families can adjust well and provide guidance
and security for the sick child. Others may become confused and frightened and
show hostility, disinterest, or neglect; these families need guidance and counseling.
The nurse has a great responsibility to support the family. The nurse's primary goal
is to reduce anxiety in the child and family. This goal may be accomplished through
open communication and ongoing contact.
Question format: Multiple Choice
Chapter 21: The Newborn at Risk: Congenital Disorders
Cognitive Level: Apply
Client Needs: Psychosocial Integrity: Psychosocial Integrity
Integrated Process: Caring
Reference: p. 469-470

10. A nursing instructor is preparing a discussion which will illustrate the different
forms of spina bifida. The instructor determines the session is successful after the
students correctly choose which form as being spina bifida with myelomeningocele?
A. The spinal cord, meninges, and nerve roots protrude out the lower back.
B. There's a cystic sac containing the spinal meninges protruding out the back.
C. There is only soft-tissue inflammation without protrusion.
D. There is a bony defect that occurs without soft-tissue involvement.

Answer: A
Rationale: In spina bifida with myelomeningocele, there is a protrusion of the spinal
cord and the meninges, with nerve roots embedded in the wall of the cyst. When
part of the spinal meninges protrudes through the bony defect and forms a cystic
sac, the condition is termed spina bifida with meningocele. A bony defect that
occurs without soft-tissue involvement is called spina bifida occulta. No pressure is
placed on the cyst.
Question format: Multiple Choice
Chapter 21: The Newborn at Risk: Congenital Disorders
Cognitive Level: Analyze
Client Needs: Physiological Integrity: Physiological Adaptation
Integrated Process: Teaching/Learning
Reference: p. 456-457

11. The nurse is preparing to teach the young parents of a newborn who is
diagnosed with spinal bifida occulta how to care for their newborn. Which
information should the nurse prioritize when explaining this defect?
A. There is protrusion of the spinal cord and meninges, with nerve roots embedded.
B. The spinal meninges protrude through the bony defect and form a cystic sac.
C. There is no protrusion of the spinal cord, only soft-tissue inflammation occurs.
D. There is a bony defect that occurs without soft-tissue involvement.

Answer: D

Rationale: A bony defect that occurs without soft-tissue involvement is called spina
bifida occulta. In spina bifida with myelomeningocele, there is a protrusion of the
spinal cord and the meninges, with nerve roots embedded in the wall of the cyst.
When part of the spinal meninges protrudes through the bony defect and forms a
cystic sac, the condition is termed spina bifida with meningocele.
Question format: Multiple Choice
Chapter 21: The Newborn at Risk: Congenital Disorders
Cognitive Level: Apply
Client Needs: Physiological Integrity: Physiological Adaptation
Integrated Process: Teaching/Learning
Reference: p. 456-457

12. The community health nurse is preparing a presentation which will illustrate the
various forms of spina bifida for a health fair. Which explanation should the nurse
use to explain spina bifida with meningocele?
A. There is protrusion of the spinal cord and meninges, with nerve roots embedded.
B. The spinal meninges protrude through the bony defect and form a cystic sac.
C. There is no protrusion of the spinal cord, only soft-tissue inflammation occurs.
D. There is a bony defect that occurs without soft-tissue involvement.

Answer: B

Rationale: When part of the spinal meninges protrudes through the bony defect and
forms a cystic sac, the condition is termed spina bifida with meningocele. In spina
bifida with myelomeningocele, there is a protrusion of the spinal cord and the
meninges, with nerve roots embedded in the wall of the cyst. A bony defect that
occurs without soft-tissue involvement is called spina bifida occulta.
Question format: Multiple Choice
Chapter 21: The Newborn at Risk: Congenital Disorders
Cognitive Level: Apply
Client Needs: Physiological Integrity: Physiological Adaptation
Integrated Process: Teaching/Learning
Reference: p. 456-458

13. The nurse is preparing a nursing care plan for an infant who was born with
spina bifida with myelomeningocele. Which nursing goal should the nurse prioritize
for this child?
A. Reducing family anxiety
B. Preventing infection
C. Providing caregiver teaching
D. Promoting comfort measures

Answer: B

Rationale: The highest priority nursing goal is preventing infection because of the
vulnerability of the myelomeningocele sac. Promoting comfort is important but not
as high a priority because the child does not usually have severe pain with this
diagnosis. Reducing anxiety and teaching are lower priorities; physical is a higher
priority than psychosocial.
Question format: Multiple Choice
Chapter 21: The Newborn at Risk: Congenital Disorders
Cognitive Level: Apply
Client Needs: Safe, Effective Care Environment: Management of Care
Integrated Process: Nursing Process
Reference: p. 457

14. A nursing instructor is leading a group discussion on congenital hydrocephalus.


The instructor determines the session is successful after the students correctly
choose which factor that determines the noncommunicating type?
A. Decreased production of cerebrospinal fluid
B. Obstruction that keeps CSF from passing between the ventricles and the spinal
cord
C. Opening between the ventricles and the spinal cord that usually closes at birth
D. Defective absorption of cerebrospinal fluid

Answer: B

Rationale: In the noncommunicating type of congenital hydrocephalus, an


obstruction occurs, and CSF is not able to pass between the ventricles and the
spinal cord. The blockage causes increased pressure on the brain or spinal cord. In
the communicating type of hydrocephalus, no obstruction of the free flow of CSF
exists between the ventricles and the spinal theca; rather, the condition is caused
by defective absorption of CSF. There is no concern of decreased production of CSF
and no opening between the ventricles and spinal cord in hydrocephalus.
Question format: Multiple Choice
Chapter 21: The Newborn at Risk: Congenital Disorders
Cognitive Level: Analyze
Client Needs: Physiological Integrity: Physiological Adaptation
Integrated Process: Teaching/Learning
Reference: p. 459

15. A newborn is diagnosed with the communicating type of congenital


hydrocephalus. Which explanation should the nurse prioritize when preparing a
teaching session for the parents?
A. There is a decreased production of cerebrospinal fluid.
B. There is an obstruction that keeps cerebrospinal fluid from passing between the
ventricles and the spinal cord.
C. There is an opening between the ventricles and the spinal cord that usually
closes at birth.
D. There is defective absorption of cerebrospinal fluid.

Answer: D

Rationale: In the communicating type of hydrocephalus, no obstruction of the free


flow of cerebral spinal fluid (CSF) exists between the ventricles and the spinal
theca; rather, the condition is caused by defective absorption of CSF. In the
noncommunicating type of congenital hydrocephalus, an obstruction occurs and
CSF is not able to pass between the ventricles and the spinal cord. The blockage
causes increased pressure on the brain or spinal cord. There is no concern of
decreased production of CSF and no opening between the ventricles and spinal cord
in hydrocephalus.
Question format: Multiple Choice
Chapter 21: The Newborn at Risk: Congenital Disorders
Cognitive Level: Apply
Client Needs: Physiological Integrity: Physiological Adaptation
Integrated Process: Teaching/Learning
Reference: p. 459

16. The nurse is monitoring a newborn who exhibited a large head at birth and is
exhibiting an increasing head growth on continued assessment. Which additional
findings on assessment should lead the nurse to suspect hydrocephalus in this
infant?
A. The scalp is dull and becoming dark red.
B. Eyes appear to be pushed downward.
C. Hands short with curved fingers.
D. Neck area is thickened and strong.

Answer: B
Rationale: In hydrocephalus, the eyes appear to be pushed downward slightly with
the sclera visible above the iris, the so-called setting sun sign. The fontanels
(fontanelles) are bulging and tense, with the suture lines separating and the spaces
palpable through the scalp. The scalp also becomes shiny and its veins dilate. In
Down syndrome the hands are short with an incurved finger. In congenital
hypothyroidism, the neck is short and thick.
Question format: Multiple Choice
Chapter 21: The Newborn at Risk: Congenital Disorders
Cognitive Level: Apply
Client Needs: Physiological Integrity: Physiological Adaptation
Integrated Process: Nursing Process
Reference: p. 459-460

17. The nurse is preparing to assess an infant who is diagnosed with a ventricular
septal defect. Which assessment finding should the nurse be prepared to
document?
A. fatigue and dyspnea
B. delayed growth and development
C. loud, harsh murmur
D. bounding pulse

Answer: C

Rationale: Children with ventricular septal defects have a characteristic loud, harsh
murmur. Fatigue and dyspnea, delayed growth and development, and a bounding
pulse are seen in the child with patent ductus arteriosus.
Question format: Multiple Choice
Chapter 21: The Newborn at Risk: Congenital Disorders
Cognitive Level: Apply
Client Needs: Physiological Integrity: Physiological Adaptation
Integrated Process: Nursing Process
Reference: p. 464

18. The nurse is working with a group of parents of children who have congenital
heart disorders. Which statement made by the parents should the nurse prioritize
for further assessment?
A. "She gets so tired when she is eating."
B. "They say he has a heart murmur but it may go away."
C. "His chest measurement is the same as his head."
D. "When I move her legs up toward her chest I hear a click."

Answer: A

Rationale: Newborns with cardiac anomalies that may be developing congestive


heart failure have a history of being poor eaters, tiring easily from the effort to
suck, and failing to grow or thrive normally. A murmur is not a sign of congestive
heart failure. At times during infancy the chest and head would measure the same
without this being a concern. With a congenital dislocation of the hip, an audible
click may be heard.
Question format: Multiple Choice
Chapter 21: The Newborn at Risk: Congenital Disorders
Cognitive Level: Apply
Client Needs: Physiological Integrity: Physiological Adaptation
Integrated Process: Nursing Process
Reference: p. 464

19. The nurse is providing care for a child following a cardiac catheterization. Which
nursing action should the nurse prioritize during the first 12 hours after the
procedure?
A. keeping the head of the bed elevated 45 degrees
B. observing the site and extremity
C. changing the dressing at least every 3 hours
D. monitor vital signs every 4 hours

Answer: B

Rationale: During the first 12 hours after the procedure the site needs to be
monitored and the extremity observed for pulses, edema, skin temperature and
color, and any other signs of poor circulation as well as infection. The child should
be flat in bed and the pressure dressing snug and left in place until the day after
the procedure. The vital signs should be monitored closely.
Question format: Multiple Choice
Chapter 21: The Newborn at Risk: Congenital Disorders
Cognitive Level: Apply
Client Needs: Physiological Integrity: Reduction of Risk Potential
Integrated Process: Nursing Process
Reference: p. 468

20. The nurse is caring for a preterm infant and notes frothing and excessive
drooling. Which additional assessment finding should the nurse prioritize and
report immediately?
A. Bright red blood from the mouth
B. Bradycardia
C. Vomiting
D. Severe cyanosis

Answer: D

Rationale: Esophageal atresia is the absence of a normal esophageal passage from


the pharynx to the stomach, which results in fluids and mucus spilling into the
lungs. This can lead to respiratory distress manifested by excessive drooling and
periods of respiratory distress with choking and cyanosis. No feeding should be
given until the newborn has been examined. If feeding is attempted, the newborn
chokes, coughs, and regurgitates; becomes deeply cyanotic; and appears to be in
severe respiratory distress. Bright red blood from the mouth is not an indication of
esophageal atresia. Bradycardia and vomiting are not signs of respiratory distress.
Question format: Multiple Choice
Chapter 21: The Newborn at Risk: Congenital Disorders
Cognitive Level: Apply
Client Needs: Physiological Integrity: Physiological Adaptation
Integrated Process: Nursing Process
Reference: p. 475

21. The nurse is caring for a newborn diagnosed with a diaphragmatic hernia. The
nurse will prepare to teach the parents concerning which potential treatment
modality?
A. immediate surgery to correct
B. will likely not need surgery
C. will likely have surgery by age 1
D. will require several surgeries during the first 2 years of life

Answer: A

Rationale: In a congenital hernia of the diaphragm, some of the abdominal organs


are displaced into the left chest through an opening in the diaphragm. The heart is
pushed toward the right, and the left lung is compressed. Rapid, labored
respirations and cyanosis are present on the first day of life, and breathing
becomes increasingly difficult. Surgery to repair a diaphragmatic hernia is essential
and may be performed as an emergency procedure. The outcome of surgical repair
depends on the degree of pulmonary development. Further treatment depends on
the degree of herniation.
Question format: Multiple Choice
Chapter 21: The Newborn at Risk: Congenital Disorders
Cognitive Level: Apply
Client Needs: Physiological Integrity: Physiological Adaptation
Integrated Process: Nursing Process
Reference: p. 476

22. The nurse is teaching the caregivers of an infant diagnosed with hypospadias
how to properly care for the infant. The nurse determines the session is successful
when the caregivers make which statement?
A. "At least he won't have to have surgery until he is almost ready to start school."
B. "Being able to most likely correct this in one stage rather than several is
reassuring."
C. "It is upsetting to me that he is in pain when he urinates."
D. "We hadn't decided about circumcision, but he will have to be circumcised before
they do the surgery."

Answer: B

Rationale: Surgical repair is often accomplished in one stage and is often done as
outpatient surgery. Surgical repair is desirable between the ages of 6 and 18
months, before body image and castration anxiety become problems. Urination is
not affected, but the boy cannot void while standing in the normal male fashion.
These newborns should not be circumcised because the foreskin is used in the
repair.
Question format: Multiple Choice
Chapter 21: The Newborn at Risk: Congenital Disorders
Cognitive Level: Analyze
Client Needs: Physiological Integrity: Physiological Adaptation
Integrated Process: Teaching/Learning
Reference: p. 478

23. The nurse is answering questions from the parents of a newborn diagnosed with
clubfoot (congenital talipes equinovarus). When asked by the parents which
treatment will be used, what would the nurse predict?
A. Putting the child in Bryant's traction
B. Doing passive range of motion
C. Placing the child in special shoes
D. Application of a splint or cast

Answer: D

Rationale: Clubfoot (congenital talipes equinovarus) is the most common congenital


foot deformity. Treatment is started during the neonatal period, and correction
usually is accomplished by manipulation and bandaging or by application of a
plastic splint or cast. The child is not put in Bryant's traction. Passive range of
motion is used for positional deformities, and although special shoes may be used
later in treatment, the treatment for the newborn is casting.
Question format: Multiple Choice
Chapter 21: The Newborn at Risk: Congenital Disorders
Cognitive Level: Apply
Client Needs: Physiological Integrity: Physiological Adaptation
Integrated Process: Teaching/Learning
Reference: p. 479

24. The nurse is assessing a newborn and suspects developmental dysplasia of the
hip (DDH). For which sign is the nurse prioritizing in this potential diagnosis?
A. Symmetry of the gluteal skin folds
B. Limited abduction of the affected hip
C. Lengthening of the femur
D. Bilateral adduction of the legs

Answer: B

Rationale: The infant with DDH usually has limited abduction of the affected hip.
The infant has asymmetry of the gluteal skin folds and shortening of the femur.
Adduction is not a concern.
Question format: Multiple Choice
Chapter 21: The Newborn at Risk: Congenital Disorders
Cognitive Level: Apply
Client Needs: Physiological Integrity: Physiological Adaptation
Integrated Process: Nursing Process
Reference: p. 480

25. The nurse is caring for an infant in a hip spica cast. Which nursing intervention
would the nurse prioritize to promote skin integrity?
A. Place the child on a soft mattress following surgery.
B. Use powder on the perineal area to avoid irritation.
C. Give daily sponge baths and clean around the edges of the cast.
D. Offer pacifiers to encourage the infant to suck.

Answer: C

Rationale: Daily sponge baths are important and must include close attention to the
skin under the straps of the device or around the edge of the cast. Place the infant
on a firm mattress and support position changes with firm pillows. Avoid using
powders and lotions. Sucking is important but not related to skin integrity.
Question format: Multiple Choice
Chapter 21: The Newborn at Risk: Congenital Disorders
Cognitive Level: Apply
Client Needs: Physiological Integrity: Basic Care and Comfort
Integrated Process: Nursing Process
Reference: p. 481

26. The nurse is caring for a newborn with congenital hip dysplasia. Which nursing
diagnoses would the nurse prioritize for this infant after the application of a hip
spica cast? Select all that apply.
A. Risk for aspiration
B. Risk for imbalanced nutrition
C. Risk for fluid volume excess
D. Risk for delayed growth and development
E. Risk for impaired skin integrity

Answer: D, E

Rationale: Nursing diagnosis for this newborn would include Risk for delayed growth
and development, and Risk for impaired skin integrity. Risk for aspiration,
imbalanced nutrition, and fluid volume excess are not related to congenital hip
dysplasia.
Question format: Multiple Select
Chapter 21: The Newborn at Risk: Congenital Disorders
Cognitive Level: Apply
Client Needs: Safe, Effective Care Environment: Management of Care
Integrated Process: Nursing Process
Reference: p. 481
27. The parents of an infant diagnosed with phenylketonuria are not sure they
agree with the diagnosis and proposed treatment. The nurse should point out that
this condition can result in which additional condition if left untreated?
A. intellectual disability
B. congenital heart defects
C. increased intracranial pressure
D. strangulated intestine

Answer: A

Rationale: Phenylketonuria (PKU) is a recessive hereditary defect of metabolism


that, if untreated, causes severe intellectual disability. The nurse should provide the
parents with information describing the pros and cons of the treatment. This
disorder is not related to congenital heart defects, increased intracranial pressure,
or to a strangulated intestine.
Question format: Multiple Choice
Chapter 21: The Newborn at Risk: Congenital Disorders
Cognitive Level: Apply
Client Needs: Physiological Integrity: Physiological Adaptation
Integrated Process: Teaching/Learning
Reference: p. 483

28. The nurse is monitoring a new mother changing her newborn's diaper and
notices a musty smell to the infant's urine. Which condition should the nurse
prioritize in further assessments to rule out?
A. Galactosemia
B. Congenital hypothyroidism
C. Phenylketonuria
D. Turner syndrome

Answer: C

Rationale: There is a characteristic musty smell to the urine in the child with
phenylketonuria. None of the other disorders affect the urine or the smell of the
urine.
Question format: Multiple Choice
Chapter 21: The Newborn at Risk: Congenital Disorders
Cognitive Level: Apply
Client Needs: Physiological Integrity: Reduction of Risk Potential
Integrated Process: Nursing Process
Reference: p. 483

29. The nurse is assessing a group of infants and notes one of the infants has
chronic constipation and an enlarged abdomen. The nurse would determine this
infant is showing indications of which condition?
A. galactosemia
B. congenital hypothyroidism
C. phenylketonuria
D. Turner syndrome

Answer: B

Rationale: Two common features seen in the infant with congenital hypothyroidism
are chronic constipation and abdomen enlargement caused by poor muscle tone.
Infants with galactosemia will present with vomiting and diarrhea resulting in
dehydration, weight loss, and jaundice. Infants with phenylketonuria show
progressive mental deficiency, frequent vomiting, aggressive and hyperactive traits,
as well as a musty urine smell. Infants with Turner syndrome present with short
stature, low set ears, broad-based neck, broad chest, increased angle of the arms,
and edema of the hands and feet.
Question format: Multiple Choice
Chapter 21: The Newborn at Risk: Congenital Disorders
Cognitive Level: Apply
Client Needs: Physiological Integrity: Physiological Adaptation
Integrated Process: Nursing Process
Reference: p. 484

30. The nurse caring for a newborn notes a distended abdomen approximately 24
hours after birth. Which action should the nurse take after review of the medical
record reveals an apparent healthy newborn at birth but no documentation of a
bowel movement?
A. Attempt to take a rectal temperature.
B. Inform the health care provider immediately.
C. Schedule radiography to diagnose the problem.
D. Inform the parents that the newborn might need surgery.

Answer: B

Rationale: If the newborn does not pass a stool within the first 24 hours, the health
care provider should be notified. In some newborns, a shallow opening may occur
in the anus with the rectum ending in a blind pouch some distance higher. Thus,
being able to pass a thermometer into the rectum does not guarantee that the
rectoanal canal is normal. More reliable presumptive evidence is obtained by
watching carefully for the first meconium stool. Abdominal distention also occurs.
Definitive diagnosis is made by radiographic studies which would be ordered by the
primary care provider. The nurse would not discuss the potential of surgery with
the parents. That is beyond the scope of the nurse and would be discussed by the
surgeon after the diagnostic studies are completed.
Question format: Multiple Choice
Chapter 21: The Newborn at Risk: Congenital Disorders
Cognitive Level: Apply
Client Needs: Physiological Integrity: Reduction of Risk Potential
Integrated Process: Nursing Process
Reference: p. 475
Chapter 22
1. The nurse is caring for a 4-year-old who is upset that the nurse is leaving at the
end of a shift. Which statement by the nurse will most help the child understand
why the nurse is leaving?
A. "My shift is over and now you will have another nurse."
B. "I need to go out to my car and drive home."
C. "I have a 4-year-old son and I need to go play with him."
D. "The hospital is not paying me to stay here any longer."

Answer: C

Rationale: Preschool thinking is strongly influenced by role fantasy, or how a child


would like something to turn out according to his or her understanding. Because
this 5-year-old is a child, he/she can understand the nurse returning home after
work to her children. While the other options are true statements, the best option is
a statement to which the child can relate.
Question format: Multiple Choice
Chapter 22: Principles of Growth and Development
Cognitive Level: Apply
Client Needs: Health Promotion and Maintenance
Integrated Process: Communication and Documentation
Reference: p. 502

2. The nursing instructor is conducting a class on the development of an infant. The


instructor determines the class is successful when the students correctly illustrate
which as the proximodistal development in an infant?
A. Kick and to wave the arms, then learns to grasp the foot and pull it to the mouth
B. Roll over, then learns to follow an object with the eyes
C. Walk for several steps, then learns to run
D. Imitate speech-like sounds, then learns to speak a language

Answer: A

Rationale: Proximodistal growth starts in the center of the body and progresses
toward the periphery; learning progresses from the simple to the complex. Kicking
and waving require only gross motor skills originating close to the trunk that move
entire limbs, whereas grasping the foot and guiding it to the mouth each require
fine control of the extremities. The other examples show other normal growth and
development, but not the progression from proximal/gross to distal/fine
movements.
Question format: Multiple Choice
Chapter 22: Principles of Growth and Development
Cognitive Level: Apply
Client Needs: Health Promotion and Maintenance
Integrated Process: Teaching/Learning
Reference: p. 493

3. A homeless mother arrives in the emergency department with her 5-year-old son
who appears to have the mumps. During assessment, the nurse discovers the child
is behind on his immunization schedule. Which information should the nurse
prioritize for this family?
A. a list of primary care physicians who are taking new clients
B. a chart depicting the recommended immunization schedules for infants and
children
C. a list of organizations that assist families to get food, shelter, and medical
benefits
D. A list of organizations that provide referral and counseling for partner abuse,
drug and alcohol addiction, and mental illness

Answer: C

Rationale: A list of organizations that will help them to get food, shelter, and
medical benefits will likely be most useful because it offers practical, down-to-earth
information. She is unlikely to be able to afford a primary care physician, and their
living situation is too unstable to guarantee regular visits. Knowing about
recommended vaccination schedules would likewise not be useful if the family does
not have access to regular care. Although some homeless families have problems
with partner abuse, drug and alcohol addiction, or mental illness, you cannot
assume that this family is one of them.
Question format: Multiple Choice
Chapter 22: Principles of Growth and Development
Cognitive Level: Apply
Client Needs: Safe, Effective Care Environment: Management of Care
Integrated Process: Nursing Process
Reference: p. 496

4. The daughter of a divorced couple is hospitalized for injuries received in an


automobile accident. The custodial father is at work when the mother arrives
extremely agitated and demanding to see her daughter immediately. Which action
will be most appropriate for the nurse to take at this time?
A. Tell the mother what room the child is in.
B. Check to see who is allowed to have contact with the child.
C. Ask the child if she would like to see her mother.
D. Ask the mother if she is allowed to see the child.

Answer: B

Rationale: The instructions from the custodial parent or courts should be honored.
The nurse should check to find out if the mother is allowed access to the child.
Asking the child may cause problems with one or both parents and could inflict fear
or guilt on the child. In addition, even when a parent has been hurtful to a child,
the child will often desire to have contact with that parent. Asking the mother if she
has access to the child is unlikely to produce a truthful answer if that answer is
"no."
Question format: Multiple Choice
Chapter 22: Principles of Growth and Development
Cognitive Level: Apply
Client Needs: Safe, Effective Care Environment: Management of Care
Integrated Process: Nursing Process
Reference: p. 497

5. A 14-year-old boy presents to the emergency department alone with facial


lacerations and a fractured left arm. Assessment reveals blue hair, several body
piercings, homemade tattoos, scars on his arms that appear to be track marks,
evasive answers when questioned about where he lives, and responses indicating
that he has no parents when contact information is requested. In addition to being
treated for the fracture and lacerations, what additional care would be the highest
priority for this child?
A. Provide information for organizations which provide shelter.
B. Attempt to find the parents and contact them.
C. Ensure the child's immunizations are up-to-date.
D. An HIV test and counseling would be done.

Answer: D

Rationale: This child's age, appearance, and evidence of drug abuse—together with
his evasiveness about his living arrangements—indicate that he is likely a runaway.
He is at high risk for HIV infection both from his drug abuse and because he is
vulnerable to sexual predators. It is not within the scope of the nurse's
responsibilities to try to find the parents, especially if the child is from out-of-state.
There is no reliable way to ensure the information provided by the child could be
verified to ensure the proper immunizations would be given. Providing the child
with the information about shelters is not the priority and the child may reject the
information, depending on his situation.
Question format: Multiple Choice
Chapter 22: Principles of Growth and Development
Cognitive Level: Apply
Client Needs: Psychosocial Integrity
Integrated Process: Nursing Process
Reference: p. 496

6. A single woman brings her 10-year-old daughter in for the first well-child visit at
the clinic. The nurse discovers they recently relocated to that area due to the
mother's job, the child is now attending a "tough" school and she is home, scared
and alone, for several hours after school. Which action should the nurse prioritize
to assist this family?
A. Discuss concerns about anxiety and depression.
B. Provide information for preventing drug and alcohol use among teens.
C. Provide a list of local after-school programs.
D. Provide a list of emergency contact numbers.
Answer: C

Rationale: A list of local after-school programs will provide this latchkey child with
some safe, constructive alternatives to being home alone. A list of emergency
contact numbers is important for a child in this situation but her mother should
compile the list to ensure it has the correct individuals for the child to contact.
Addressing anxiety, depression, and drug and alcohol abuse would be topics for
later sessions. The priority for the nurse at this time is assisting with providing a
safe environment for the child.
Question format: Multiple Choice
Chapter 22: Principles of Growth and Development
Cognitive Level: Apply
Client Needs: Psychosocial Integrity
Integrated Process: Nursing Process
Reference: p. 498

7. The nurse is assessing a 5-year-old boy in the presence of his parents and notes
they are concerned he is learning bad habits by "playing with himself." They ask the
nurse for the best punishment to address this issue. Which theorist should the
nurse reference when answering their question to help the parents address this
issue and ensure they are providing the child with the best attitude toward his
body and sexuality?
A. Carl Jung
B. Erik Erikson
C. Lawrence Kohlberg
D. Arnold Gesell

Answer: B

Rationale: Both Sigmund Freud and Erik Erikson identified the parents' reactions to
the child's discovery of his or her own body as crucial to the child's later attitudes
toward the body and sexuality. Jung believed that what happens to a child is not so
critical to the child's development as are the child's responses to these events, so
the parents' reaction is less important than how the child responds to it. Kohlberg
was concerned with the development of moral reasoning in children. Arnold Gesell
developed a series of developmental landmarks noting that the child progresses
through a series of "easy" and "difficult" phases.
Question format: Multiple Choice
Chapter 22: Principles of Growth and Development
Cognitive Level: Apply
Client Needs: Health Promotion and Maintenance
Integrated Process: Caring
Reference: p. 502

8. The parents of a 6-year-old boy express frustration and worry that he doesn't
seem to be developing a sense of responsibility. For example, he recently drew on
the walls of his room and was told he had to clean the walls by dinnertime.
Although he agreed and was shown how to do it and was provided cleaning
materials, he had not even started when the deadline arrived. What aspect of child
development should the nurse prioritize to explain to the parents how best to
handle similar situations?
A. The boy is not yet able to see a moral reason to comply with his mother's
demand.
B. The boy has not yet developed a desire to please his parents.
C. The boy has no interest yet in performing tasks independently.
D. The boy has not yet developed a sense of time.

Answer: D

Rationale: At 6, this boy is old enough to perform tasks independently and to want
to be good and please his parents. Morally, he is becoming aware of the feelings of
others and likely wants to live up to his parents' expectations. He accepted the
punishment and was willing to perform it. However, his concept of time may not be
very well developed yet, so a deadline is unlikely to be meaningful to him.
Question format: Multiple Choice
Chapter 22: Principles of Growth and Development
Cognitive Level: Apply
Client Needs: Health Promotion and Maintenance
Integrated Process: Nursing Process
Reference: p. 502

9. The mother of a 3-year-old girl is asking for suggestions for feeding a child who
constantly complains there is not enough food on her plate while still not eating the
foods provided to her. Which suggestion should the nurse prioritize for this
mother?
A. Use a smaller plate with the same portions of food.
B. Use a larger plate with the same portions of food.
C. Scold the child for her poor behavior.
D. Promise the child more food after she eats what she has.

Answer: A

Rationale: The same portion of food on a smaller plate will cover more of the
surface area of the plate and will thus look to her like more food. By contrast,
putting the same portion on a larger plate will make the portion look even smaller.
Because she cannot yet see things from another's point of view, your feelings about
giving her more will not be important to her. Because her sense of time is not
developed yet, a promise of more later is likely to be meaningless to her.
Question format: Multiple Choice
Chapter 22: Principles of Growth and Development
Cognitive Level: Apply
Client Needs: Health Promotion and Maintenance
Integrated Process: Nursing Process
Reference: p. 503
10. The nurse is admitting a 10-year-old child the night before surgery, and the
child is very concerned about what will happen. Which action should the nurse
prioritize in caring for this child to minimize the possible trauma related to this
hospitalization?
A. Sedate the child and allow unlimited TV.
B. Briefly explain the procedure and tell the child not to worry.
C. Encourage the child to ask questions and express fears.
D. Joke with the child and arrange a visit from a clown.

Answer: C

Rationale: The child will naturally be frightened and apprehensive; expecting a child
to mask or deny these feelings will only make the child feel worse. Distraction with
humor or entertainment is unlikely to succeed. Enabling the child to express fears
and answering the child's questions and concerns is most likely to help the child
weather the experience.
Question format: Multiple Choice
Chapter 22: Principles of Growth and Development
Cognitive Level: Apply
Client Needs: Psychosocial Integrity
Integrated Process: Nursing Process
Reference: p. 501

11. A 5-year-old child is brought to the emergency room with an open chin
laceration following a bike accident. The child appears extremely frightened and
asks if he will bleed to death. Which nursing action should the nurse prioritize to
best assist this child?
A. Seat him where he can watch TV with his parents.
B. Tell him that big boys like him must be brave.
C. Reassure him that he will not run out of blood.
D. Allow him to sit on his parent's lap until time to suture the laceration.

Answer: D

Rationale: According to Piaget, a 5-year-old is in the preoperational stage. The


child's thinking is based on intuition and is not logical. Allowing him to sit with his
mother is comforting. Encouraging TV watching, denying his feelings, or joking that
he will not run out of blood is not likely to distract or calm him.
Question format: Multiple Choice
Chapter 22: Principles of Growth and Development
Cognitive Level: Apply
Client Needs: Health Promotion and Maintenance
Integrated Process: Nursing Process
Reference: p. 507

12. The nursing instructor is explaining the various aspects of growth and
development to a group of nursing students. The instructor determines the session
is successful when the students point out which example best illustrates the
concept of development?
A. the 5-month-old whose head circumference and chest measurement are the
same
B. the 3-year-old who has been toilet trained for 6 months
C. the 5-year-old who is 2 inches taller than she was 4 months ago
D. The 10-year-old who weighs the same amount as his 8-year-old brother

Answer: B

Rationale: Development is the progressive change in the child toward maturity.


Toilet training in the child shows evidence the child is moving toward maturity. The
other choices have to do with physical increase in the body's size and appearance,
which relates to the concept of growth.
Question format: Multiple Choice
Chapter 22: Principles of Growth and Development
Cognitive Level: Apply
Client Needs: Health Promotion and Maintenance
Integrated Process: Teaching/Learning
Reference: p. 493

13. The nursing student is preparing a presentation that will illustrate the various
stages of growth and development starting with a newborn. Which example should
the student point out will best illustrate cephalocaudal growth pattern?
A. grasping the foot and pulling it to the mouth
B. controlling the head and neck before being able to control the arms and legs
C. walking for several steps, stopping, and then running for several steps
D. using the fingers to pick up an object and put it back down

Answer: B

Rationale: Cephalocaudal development starts with the head and moves downward.
The child is able to control the head and neck before being able to control the arms
and legs. Grasping the foot and pulling it to the mouth and using the fingers to pick
up objects shows development of fine motor skills, but not cephalocaudal
development. Walking and running show development of gross motor skills.
Question format: Multiple Choice
Chapter 22: Principles of Growth and Development
Cognitive Level: Apply
Client Needs: Health Promotion and Maintenance
Integrated Process: Teaching/Learning
Reference: p. 493

14. The nursing instructor is leading a discussion on the theories of Sigmund Freud
with a group of nursing students. The instructor determines the session is
successful when the students correctly choose which example as indicating a child
is in the anal stage of development?
A. finds satisfaction through making noises and crying
B. recognizes the physical differences between boys and girls
C. has a desire to mold and control the environment
D. beginning to develop a sense of moral responsibility

Answer: C

Rationale: During the anal stage, according to Freud, elimination is one of the
child's first experiences of creativity. It represents the beginnings of the desire to
mold and control the environment. Finding satisfaction through making noise and
crying occurs in the oral stage, in which satisfaction comes through the mouth.
Recognizing the physical differences between boys and girls is in the phallic stage,
where the child's interest moves to the genital area as a source of pride and
curiosity. Beginning to develop a sense of moral responsibility occurs during the
latency stage, when the child's sense of moral responsibility (the superego)
develops, based on what has been taught through the parents' words and actions.
Question format: Multiple Choice
Chapter 22: Principles of Growth and Development
Cognitive Level: Analyze
Client Needs: Health Promotion and Maintenance
Integrated Process: Teaching/Learning
Reference: p. 500

15. The nurse is assessing a young client at a well-visit and notes a normal
progression in growth and development. Which assessment finding would lead the
nurse to determine this client is in the stage Sigmund Freud referred to as the
latency stage?
A. finds satisfaction through making noises and crying
B. can recognize the physical differences between boys and girls
C. has a desire to mold and control the environment
D. beginning to develop a sense of moral responsibility

Answer: D

Rationale: During the latency stage, the child's sense of moral responsibility (the
superego) develops, based on what has been taught through the parents' words
and actions. In the oral stage, satisfaction come through the mouth. In the phallic
stage, the child's interest moves to the genital area as a source of pride and
curiosity and they begin to recognize the differences between the sexes. In the anal
stage, according to Freud, elimination is one of the child's first experiences of
creativity; it represents the beginnings of the desire to mold and control the
environment.
Question format: Multiple Choice
Chapter 22: Principles of Growth and Development
Cognitive Level: Apply
Client Needs: Health Promotion and Maintenance
Integrated Process: Nursing Process
Reference: p. 501
16. A nurse at a pediatric clinic is assessing a young client at a well-child visit. After
the assessment, the nurse determines the child is in Erikson's autonomy stage
based on which finding?
A. learning to trust that needs are met when they cry
B. will perform tasks by themselves, even when it takes a long time
C. They want to know the rules and enjoy being good and getting approval.
D. will interact with others and like to compete

Answer: B

Rationale: Autonomy occurs when the child feels in control and learns to perform
tasks independently. Getting their needs met when crying describes trust that is
developed in the infant. The preschool-aged child wants to know the rules; he or
she enjoys being good and getting approval when developing initiative. Interacting
and competing with others occurs with the development of industry seen in the
school-aged child.
Question format: Multiple Choice
Chapter 22: Principles of Growth and Development
Cognitive Level: Apply
Client Needs: Health Promotion and Maintenance
Integrated Process: Nursing Process
Reference: p. 502

17. The nurse is caring for a young client who was admitted to the hospital. The
nurse concludes this client is in Erikson's developmental stage of industry based on
which action by the child?
A. has learned to trust their needs are met when they cry
B. insists on performing tasks by themselves, even if it takes a long time
C. wants to know what they can and cannot do and enjoys being good and getting
approval
D. is seeking interaction and competition with others on the unit

Answer: D

Rationale: The development of industry is seen in the school-age child when they
interact and compete with others, seeking achievement. When needs are met after
crying, this describes trust that is developed in the infant. Performing tasks by
themselves, no matter how long it takes, relates to autonomy; this occurs when the
child feels in control and learns to perform tasks independently. Knowing the rules
and enjoying being good and getting approval is noted in the preschool-aged child
as they develop initiative.
Question format: Multiple Choice
Chapter 22: Principles of Growth and Development
Cognitive Level: Apply
Client Needs: Health Promotion and Maintenance
Integrated Process: Nursing Process
Reference: p. 502
18. The nursing instructor is preparing a class discussion covering the theories of
Erikson and the growth and development of infants and children. Which example
would best illustrate the phase referred to as initiative by Erikson?
A. learning to trust when needs are met after crying
B. performing a task by oneself, even if it takes a long time
C. knowing the rules, enjoys being good, being independent, and receiving approval
D. interacting and competing with others

Answer: C

Rationale: The development of initiative is noted in the preschool-age child as they


develop a conscience and accepts punishment for doing wrong. This child wants to
know the rules and enjoys being good and independent and likes getting approval
from the adults in their life. The infant learns that crying will get their needs met
which describes trust that is developed. The toddler who performs tasks by
themselves even when it takes a long time begins in the autonomy stage. This
occurs when the child feels in control and learns to perform tasks independently.
The child interacting and competing with others describes industry seen in the
school age child.
Question format: Multiple Choice
Chapter 22: Principles of Growth and Development
Cognitive Level: Apply
Client Needs: Health Promotion and Maintenance
Integrated Process: Teaching/Learning
Reference: p. 502

19. The mother of an 8-year-old is concerned her child is developing too slowly
when compared to a cousin. Which activities, based on Piaget's theories, would the
nurse predict this child should be active in?
A. The child can classify and organize information about their environment.
B. The child has developed an understanding of cause and effect.
C. The child can understand abstract concepts described only in words or symbols.
D. The child has no concept of quantity; if it looks like more, it is more.

Answer: A

Rationale: According to Piaget, the concrete operations phase should occur at the
ages of 7 to 11 years. The child develops the ability to begin problem-solving in a
concrete, systematic way during this stage. Demonstrating an understanding of
cause and effect occurs at the ages of 0 to 2 years and occurs in the sensorimotor
phase. Between the ages of 12 to 15 years, the formal operations phase occurs and
involves understanding abstract concepts, which are described only in words or
symbols. The preoperational phase is noted at ages 2 to 7 years and this child has
no concept of quantity; if it looks like more, it is more.
Question format: Multiple Choice
Chapter 22: Principles of Growth and Development
Cognitive Level: Apply
Client Needs: Health Promotion and Maintenance
Integrated Process: Nursing Process
Reference: p. 503

20. The nursing instructor is leading a discussion on the theories of growth and
development as described by Piaget. The instructor determines the session is
successful when the students correctly choose which action as an example of the
sensorimotor phase?
A. classifying and organizing information about the environment
B. developing an understanding of cause and effect
C. understanding abstract concepts described only in words or symbols
D. having no concept of quantity; if it looks like more, it is more

Answer: B

Rationale: In the sensorimotor phase, an understanding of cause and effect


develops. When random arm motions strike the string of bells stretched across the
crib, the newborn hears the sound made and eventually can manipulate the arms
deliberately to make the bells ring. Classifying and organizing information about the
environment occurs in the concrete operations phase. Understanding abstract
concepts described only in words or symbols occurs in the formal operations phase.
When it looks like more and the child believes that it is, this is due to no concept of
quantity; it occurs in the preoperational phase.
Question format: Multiple Choice
Chapter 22: Principles of Growth and Development
Cognitive Level: Apply
Client Needs: Health Promotion and Maintenance
Integrated Process: Nursing Process
Reference: p. 503

21. The nurse is assessing a 10-year-old child who has come to the clinic with a
caregiver and three other children. Which question would be most appropriate for
the nurse to ask to clarify the family structure?
A. "Do all of these children have the same father?"
B. "Who lives in your home?"
C. "Are you the grandmother of any of these children?"
D. "I don't see that you are wearing a wedding ring. Are you married?"

Answer: B

Rationale: It is more appropriate to ask open-ended questions to encourage the


client to provide information. It is important to identify the family caregivers and
recognize the family structure. To avoid embarrassment, ask direct and clear
questions about other family members. The other responses are inappropriate and
may be seen as judgmental. They are also closed-end questions and will not allow
for the needed information to be shared.
Question format: Multiple Choice
Chapter 22: Principles of Growth and Development
Cognitive Level: Apply
Client Needs: Psychosocial Integrity
Integrated Process: Nursing Process
Reference: p. 507

22. The school nurse is aware that providing proper care involves cooperation from
the adolescent. Which technique will the nurse prioritize to develop open
communication with the adolescents?
A. Ask the parents questions first and then ask the adolescent the same question.
B. Focus the interview on the problems rather than on the adolescent themselves.
C. Show interest in the adolescent and develop a rapport before asking questions.
D. Avoid asking questions about their friends, hobbies, and school activities.

Answer: C

Rationale: Adolescents respond positively to individuals who show an interest in


them and make an effort to connect with them. Focus the interview on the
adolescent rather than the problem. Sharing about friends, hobbies, and activities is
a way to connect with the adolescent. Parents may need to be interviewed as well,
but the adolescent is the first and primary focus.
Question format: Multiple Choice
Chapter 22: Principles of Growth and Development
Cognitive Level: Apply
Client Needs: Health Promotion and Maintenance
Integrated Process: Communication and Documentation
Reference: p. 507

23. An elementary school nurse is leading a group session discussing nutrition with
a group of students and their caregivers. The nurse should provide additional
teaching if which statement is made by one of the participants?
A. "Eating too much could affect my child's ability to do some activities."
B. "If my child has good nutrition she won't get childhood illnesses."
C. "I ate healthy as a child and that has a lot to do with the way I eat now."
D. "My children are at various ages and they all have different daily requirements."

Answer: B

Rationale: Although good nutrition is important in the child's ability to resist


infection and diseases, adequate nutrition alone will not prevent childhood illnesses,
so further teaching is needed with this response. The other responses are correct
and would not need further follow-up or clarification by the nurse.
Question format: Multiple Choice
Chapter 22: Principles of Growth and Development
Cognitive Level: Apply
Client Needs: Health Promotion and Maintenance
Integrated Process: Teaching/Learning
Reference: p. 495
24. The nurse is developing a care plan for several clients who were seen in the
clinic that day. Which situation will the nurse conclude is most likely to experience
challenges in the growth and development of the children?
A. A family lives with the parents of the mother who is working two jobs.
B. Both parents work during the day and the child comes home after school to an
empty house.
C. Children attend school while the father works and then sleep in their car at night.
D. A divorced mother shares an apartment with her best friend and her children.

Answer: C

Rationale: Homelessness creates many stresses for the family, including affecting
the growth and development of the children in the family. Job loss, loss of housing,
drug addiction, insufficient income, domestic turmoil and higher rates of abuse,
drug use, and mental health problems are often seen in homeless families. Living
with an extended family, being a latchkey kid, and children of divorce also can
affect growth and development, but homelessness is the most likely of these
choices to affect the family and child.
Question format: Multiple Choice
Chapter 22: Principles of Growth and Development
Cognitive Level: Apply
Client Needs: Health Promotion and Maintenance
Integrated Process: Caring
Reference: p. 496

25. The nurse is completing the growth chart of a 2-month-old infant at the well-
child visit. When asked by the mother what purpose this chart serves, which
response would be most appropriate by the nurse?
A. These charts give us a good indication of the child's nutritional status.
B. By plotting height and weight, we get an indication of what the adult
measurements will be.
C. Comparing your child with others of the same age and sex gives us statistics
about height and weight.
D. We are comparing the child's current measurements with the child's previous
measurements.

Answer: D

Rationale: A growth chart is used for comparison only and is used to plot and
monitor a child's growth through the years. Although a weight loss or failure to gain
weight might indicate a nutritional issue, this is not the way growth charts are
used. Growth charts do not indicate adult measurements. Statistics may be
gathered from growth charts but that is not the primary reason they are used.
Question format: Multiple Choice
Chapter 22: Principles of Growth and Development
Cognitive Level: Apply
Client Needs: Health Promotion and Maintenance
Integrated Process: Nursing Process
Reference: p. 493-494
Chapter 23
1. The caregivers of an infant state that their child cries when her mother leaves for
even a short amount of time. What might the nurse suggest as a way to console
the infant and develop a sense of security when the child's primary caregiver is out
of sight?
A. Pick the child up as soon as she begins to cry.
B. Play peek-a-boo with the child when happy.
C. Slowly increase the amount of time allowed to cry before being picked up.
D. Give her dolls and stuffed animals so she learns to distract herself.

Answer: B

Rationale: For the infant, self-assurance is necessary to confirm that objects and
people do not cease to exist when out of sight. This is a learning experience on
which the infant's entire attitude toward life depends. The ancient game of "peek-a-
boo" is a universal example of this learning technique. It is also one of the joys of
infancy as the child affirms the ability to control the disappearance and
reappearance of self. In the same manner by which the infant affirms self-
existence, she learns to confirm the existence of others, even when they are
temporarily out of sight.
Question format: Multiple Choice
Chapter 23: Growth and Development of the Infant: 28 Days to 1 Year
Cognitive Level: Apply
Client Needs: Health Promotion and Maintenance
Integrated Process: Teaching/Learning
Reference: p. 518

2. A first-time mother calls the pediatrician's office to ask the nurse about her
baby's tooth eruption. The baby is 8 months old and still does not have any teeth.
What information can the nurse share with this mother that would correctly respond
to her anxiety about her baby's dentition?
A. A baby's first teeth should erupt by 8 to 10 months of age and are the two lower
front teeth.
B. Tooth eruption is often genetically based, with some families having babies with
early tooth eruption, while others have late tooth eruption.
C. If the baby does not have any teeth come in by next month, the mother needs
to bring him back for x-rays.
D. Look for the baby to start running a fever and develop a stuffy nose and that will
indicate his teeth are coming in.

Answer: B

Rationale: Teeth eruption normally begins around age 6 to 8 months but infants'
teething patterns vary greatly between children. Genetics plays a big role in both
the timing and the actual pattern of eruption.
Question format: Multiple Choice
Chapter 23: Growth and Development of the Infant: 28 Days to 1 Year
Cognitive Level: Apply
Client Needs: Health Promotion and Maintenance
Integrated Process: Teaching/Learning
Reference: p. 514

3. A breastfeeding mother asks the nurse about when she can begin feeding her 5-
month-old infant some solids and vitamins. Which information provided by the
nurse would most accurately address this mother's concerns?
A. You can begin feeding the infant fruits and vegetables now followed by iron-
enriched cereal to ensure that he gets enough iron.
B. The first food offered to an infant is iron-enriched rice cereal and can be started
now. Additionally, the infant needs to receive vitamin D and iron.
C. If you give him one or two bottles of juice each day, he should get all the
vitamins he needs. You can begin fruits and cereal in 1 month.
D. At 6 months, you need to quit breastfeeding because he is not getting enough
iron or vitamin C and D and that should help him transition to solids better.

Answer: B

Rationale: Breastfeeding is the best method for feeding infants, according to the
Academy of Pediatrics Committee on Nutrition. However, by 4 to 6 months of age,
breastfed infants need supplements of iron and vitamin D. Solids are introduced at
the same age and parents should begin with iron-fortified rice cereal. The infant
may also have diluted fruit juice in small portions.
Question format: Multiple Choice
Chapter 23: Growth and Development of the Infant: 28 Days to 1 Year
Cognitive Level: Apply
Client Needs: Health Promotion and Maintenance
Integrated Process: Teaching/Learning
Reference: p. 518

4. The nurse is assessing a 1-year-old at the well-child annual visit and notes the
child is meeting the growth parameters. After noting the birth weight was 8 pounds
(3.6 kg) and length was 20 inches (50.8 cm) long, which measurements reflecting
height/weight would the nurse expect to document for this visit?
A. 16 lb (7.2 kg) and 26 inches (65 cm)
B. 20 lb (9.1 kg) and 28 inches (70 cm)
C. 24 pounds (10.8 kg) and 30 inches (75 cm)
D. 28 pounds (12.7 kg) and 32 inches (80 cm)

Answer: C

Rationale: By 1 year of age, the infant should have tripled his or her birth weight
and grown 10 to 12 inches (25 to 30 cm). If this infant was 8 pounds (3.6 kg) at
birth, at 1 year, this child should weigh 24 pounds (8 x 3 = 24) and grown to 30 to
32 inches (20 + 10 to 12 = 30 to 32 inches). Most of the growing occurs during the
first 6 months with the infant's birth weight doubling and height increasing about 6
inches (15 cm). Growth slows slightly during the second 6 months but is still rapid.
Question format: Multiple Choice
Chapter 23: Growth and Development of the Infant: 28 Days to 1 Year
Cognitive Level: Apply
Client Needs: Health Promotion and Maintenance
Integrated Process: Nursing Process
Reference: p. 513

5. The nurse is assessing an 6-month-old infant at a well-baby visit and is


answering questions from the new mother. Which response should the nurse
prioritize when addressing the mother's question concerning what the infant
should be learning at this point in life?
A. Trust
B. Feel anger
C. Love
D. Fear

Answer: A

Rationale: Erikson identifies various developmental stages which all children


accomplish as they grow and develop into adults. The primary psychosocial
developmental task for the infant is learning to trust. This task creates the
foundation for the developmental tasks of the next stages of the child's life. If the
infant does not receive food, love, attention, and comfort, the infant learns to
mistrust the environment and those who are responsible for caring for the child.
Learning to feel anger, love, and fear come at later times in development.
Question format: Multiple Choice
Chapter 23: Growth and Development of the Infant: 28 Days to 1 Year
Cognitive Level: Understand
Client Needs: Health Promotion and Maintenance
Integrated Process: Nursing Process
Reference: p. 513

6. The nurse is reviewing the diet of an 8-month-old infant with the mother who
reveals she has been using evaporated milk to make the formula. Which additional
ingredient should the nurse ensure she is including in the formula?
A. Vitamin D
B. Vitamin E
C. Iron
D. Calcium

Answer: C

Rationale: Infants who are fed home-prepared formulas (based on evaporated milk)
need supplemental vitamin C and iron. Evaporated milk has adequate amounts of
vitamin D, which is unaffected by heat used in the preparation of formula. Calcium
and vitamin E would not be a concern in this infant's formula.
Question format: Multiple Choice
Chapter 23: Growth and Development of the Infant: 28 Days to 1 Year
Cognitive Level: Apply
Client Needs: Health Promotion and Maintenance
Integrated Process: Nursing Process
Reference: p. 518

7. A frustrated mother with a 9-month-old baby comes to the clinic because her son
is refusing all solid food. When talking with this mother, the nurse discovers the
mother has struggled with a weight problem all her life, which she attributes to
being forced to eat even when she was full. Not wanting to treat her child the same
way, each time her son pushes food away with his tongue she believes that he
doesn't want it. Which statement would be most appropriate for the nurse to say to
this mother?
A. "The baby might be allergic to the particular foods you offered, so try different
kinds of food."
B. "Because your baby is a fussy eater, have more than one food available at each
feeding so he can choose a food he likes."
C. "The baby might not be ready for solid food, so wait a month or so and try
again."
D. "The baby needs to learn how to swallow, so catch the food and offer it again
until the baby learns this."

Answer: D

Rationale: The infant knows only one way to take food: namely to thrust the tongue
forward as if to suck. This is called the extrusion (protrusion) reflex and has the
effect of pushing solid food out of the infant's mouth. The process of transferring
food from the front of the mouth to the throat for swallowing is a complicated skill
that must be learned. If the food is pushed out, the caregiver must catch it and
offer it again. The baby soon learns to manipulate the tongue and comes to enjoy
this novel way of eating.
Question format: Multiple Choice
Chapter 23: Growth and Development of the Infant: 28 Days to 1 Year
Cognitive Level: Apply
Client Needs: Health Promotion and Maintenance
Integrated Process: Teaching/Learning
Reference: p. 519

8. The nurse is preparing to teach a class to a group of young parents with infants
the basics of introducing solid foods into the diet. Which factor about the food
should the nurse point out the infants respond to best when introducing solid foods
into the diet?
A. well-heated
B. thickened
C. an interesting texture
D. bland
Answer: D

Rationale: Babies like their food smooth, thin, lukewarm, and bland. If the food is
too hot, thick, or an unusual texture, the infant may refuse to eat it.
Question format: Multiple Choice
Chapter 23: Growth and Development of the Infant: 28 Days to 1 Year
Cognitive Level: Apply
Client Needs: Health Promotion and Maintenance
Integrated Process: Teaching/Learning
Reference: p. 519

9. An 8-month-old child is diagnosed with a second ear infection and the father is
concerned the infections are being caused by something he is doing or by
something in the child's development. Which question should the nurse prioritize
to collect more information to answer this father's questions?
A. "Does the baby go to bed with a bottle of formula each night?"
B. "Does the baby's teething appear to be teething?"
C. "Are the baby's scalp and hair shampooed often?"
D. "Are you supplementing the baby's home-prepared formula with vitamin C?"

Answer: A

Rationale: In addition to nursing bottle caries, liquid from milk, formula, or juice
can pool in the mouth and flow into the eustachian tube, causing otitis media if an
infant falls asleep with a bottle. Teething and shampooing the scalp and hair would
not be a factor in the development of ear infections. Vitamin C is a needed
supplement for the infant who is fed home-prepared formula; it does not cause ear
infections.
Question format: Multiple Choice
Chapter 23: Growth and Development of the Infant: 28 Days to 1 Year
Cognitive Level: Apply
Client Needs: Health Promotion and Maintenance
Integrated Process: Nursing Process
Reference: p. 522

10. The nurse takes a call from a concerned mother whose infant received routine
immunizations the day before and now has a temperature of 101oF (38.3oC), is
fussy, and pulling at the injection site. The mother wants to know what she should
do. Which is the best response from the nurse to this mother?
A. "You need to bring the baby to the emergency department to be sure he is not
having an allergic reaction."
B. "All babies have similar reactions but you should call back if he is still fussy in 24
hours."
C. "This is a common reaction. Give your child acetaminophen, cuddle him, and
apply a cool compress to the injection site."
D. "You can give your child ice-cold fluids and cover the injection site so that he
doesn't scratch the site and get it infected."
Answer: C

Rationale: Adverse reactions vary with the type of immunization but usually are
minor in nature. The most common adverse reaction is a low-grade fever within the
first 24 to 48 hours and possibly a local reaction such as tenderness, redness, and
swelling at the injection site. The child may be fussy and eat less than usual. These
reactions are treated symptomatically with acetaminophen for the fever and cool
compresses applied to the injection site. The child is encouraged to drink fluids but
not necessarily ice-cold fluids. Holding and cuddling are comforting to the child.
These reactions may last longer than 24 hours and should subside. These are not
signs of an allergic reaction. There is no need to cover the site.
Question format: Multiple Choice
Chapter 23: Growth and Development of the Infant: 28 Days to 1 Year
Cognitive Level: Apply
Client Needs: Physiological Integrity: Pharmacological and Parenteral Therapies
Integrated Process: Nursing Process
Reference: p. 523

11. A 6-month-old arrives for a well-baby visit with a case of diaper rash that the
parent believes is normal for infants. The parent reports changing the baby's diaper
when he wakes up and before naps or bedtime. Which frequency should the nurse
point out will better help heal and prevent future incidences of diaper rash?
A. every hour
B. every 1 to 2 hours
C. every 2 to 4 hours
D. every 3 to 5 hours

Answer: C

Rationale: To prevent diaper rash, soiled diapers should be changed frequently.


Check every 2 to 4 hours while the infant is awake to see if the diaper is soiled.
Waking the baby to change the diaper is not necessary.
Question format: Multiple Choice
Chapter 23: Growth and Development of the Infant: 28 Days to 1 Year
Cognitive Level: Apply
Client Needs: Health Promotion and Maintenance
Integrated Process: Teaching/Learning
Reference: p. 524

12. A new mother reports that she is exhausted and that the little sleep she gets is
determined by her baby. Which suggestion should the nurse prioritize to help the
mother establish healthy sleeping patterns in her infant?
A. Put the baby to bed at various times of the evening.
B. Let the baby cry during the night and she will eventually fall back to sleep.
C. Use the crib for sleeping only, not for play activities.
D. Wake the baby from afternoon naps after 1 hour to ensure she is sleepy by
nighttime.
Answer: C

Rationale: A consistent bedtime routine is usually helpful in establishing healthy


sleeping patterns and in preventing sleep problems. Using the crib for sleeping only
helps the child associate the bed with sleep. Depriving the baby of sleep during the
afternoon or evening will make the baby over-tired and less able to establish a
healthy sleeping pattern. While letting a baby cry for a while is acceptable, this
does not promote consistency in the baby's sleeping pattern.
Question format: Multiple Choice
Chapter 23: Growth and Development of the Infant: 28 Days to 1 Year
Cognitive Level: Apply
Client Needs: Health Promotion and Maintenance
Integrated Process: Teaching/Learning
Reference: p. 526

13. The pediatric nurse is meeting with a group of young mothers of newborn
infants who are questioning the "soft spot" on the head and when it will close.
Which fact should the nurse point out to the mothers?
A. It can be expected to close between months 2 and 3 of life.
B. It will close and the bones hardened by the time the child is 1 year old.
C. It may increase slightly in size the first few months of life.
D. It decreases in size rapidly during the first month of life.

Answer: C

Rationale: The anterior fontanel (fontanelle) may increase slightly in size during the
first few months of life. After 6 months it begins to decrease in size, closing
between months 12 and 18. The posterior fontanel is usually closed by month 2 or
3 of life. The sutures between the cranial bones do not ossify until later childhood.
Question format: Multiple Choice
Chapter 23: Growth and Development of the Infant: 28 Days to 1 Year
Cognitive Level: Understand
Client Needs: Health Promotion and Maintenance
Integrated Process: Teaching/Learning
Reference: p. 513

14. The nurse is assessing a 12-week-old infant in the clinic at a well-baby visit.
Which assessment finding does the nurse predict to assess in this healthy infant?
A. Able to sit up and roll over
B. Smiles at significant others
C. Grasps objects and brings them to the mouth
D. Bears weight on legs when held in standing position

Answer: B

Rationale: By 12 weeks of age the infant smiles at their mother and significant
others. The other choices are seen in the infant who is about 20 weeks of age.
Question format: Multiple Choice
Chapter 23: Growth and Development of the Infant: 28 Days to 1 Year
Cognitive Level: Apply
Client Needs: Health Promotion and Maintenance
Integrated Process: Nursing Process
Reference: p. 516

15. The mother of an 8-month-old infant appears frustrated and shares with the
nurse she can't leave the room without her infant son crying. Which is the best
response from the nurse to help this mother?
A. "He knows you are in the next room and he is attempting to get your attention."
B. "By ignoring his crying you will help him develop patience."
C. "It is important for him to be with you, so you should take him with you."
D. "If he cannot see you, he thinks you are gone and that is frightening."

Answer: D

Rationale: An infant cannot apply abstract reasoning but understands only through
the five senses. As the infant matures enough to recognize the mother or primary
caregiver, the infant becomes fearful when this person disappears. To the infant,
out of sight means out of existence, and the infant cannot tolerate this. For the
infant, self-assurance is necessary to confirm that objects and people do not cease
to exist when out of sight. As the infant grows and develops, they will grow out of
this stage.
Question format: Multiple Choice
Chapter 23: Growth and Development of the Infant: 28 Days to 1 Year
Cognitive Level: Apply
Client Needs: Health Promotion and Maintenance
Integrated Process: Nursing Process
Reference: p. 518

16. The nurse is meeting with a group of new mothers of infants and is leading the
discussion related to weaning. Which suggestion should the nurse prioritize to this
group of mothers concerning weaning their infant?
A. "It is best to just pick a day and stop breast or bottle feeding and wean the
infant to a cup."
B. "If the infant is eating from a dish and drinking from a cup, they no longer need
a bottle or breastfeeding."
C. "It is important to let the infant set the pace for weaning, no matter what age
they are."
D. "The infant who is interested in watching others drink from a cup is ready to be
weaned."

Answer: C

Rationale: Weaning must be attempted gradually, letting the infant set the pace.
The abrupt removal of a main source of satisfaction (sucking) may be detrimental
to normal development. At the age of 5 or 6 months, the infant who has watched
others drink from a cup may be ready to try a sip when it is offered but is seldom
ready at this point to be weaned.
Question format: Multiple Choice
Chapter 23: Growth and Development of the Infant: 28 Days to 1 Year
Cognitive Level: Apply
Client Needs: Health Promotion and Maintenance
Integrated Process: Teaching/Learning
Reference: p. 522

17. The mother of an infant is confused after being told by her dentist not to let her
baby go to bed with a bottle of milk. She does not agree with that suggestion.
Which response should the nurse prioritize when addressing this situation with the
mother?
A. "Giving a bottle of milk when the infant goes to bed can lead to obesity."
B. "Bottles given at bedtime can cause erosion of the enamel on the teeth."
C. "Giving your baby a pacifier at bedtime will satisfy the need to suck."
D. "You could occasionally give your baby a bottle of water at bedtime."

Answer: B

Rationale: The sugar from formula or sweetened juice coats the infant's teeth for
long periods and causes erosion of the enamel on the deciduous teeth. While giving
water in a bottle at bedtime is acceptable and a pacifier will satisfy the sucking
need, the most appropriate response is to warn of possible enamel erosion. Giving
a bottle at bedtime is not a factor that leads to obesity.
Question format: Multiple Choice
Chapter 23: Growth and Development of the Infant: 28 Days to 1 Year
Cognitive Level: Apply
Client Needs: Health Promotion and Maintenance
Integrated Process: Teaching/Learning
Reference: p. 522

18. The nurse is conducting a class at the local health department office for new
mothers of infants who are in the various stages of weaning. Which alternative food
should the nurse prioritize when questioned by a mother whose infant is refusing
to drink milk from a cup to ensure the infant is consuming enough calcium?
A. cottage cheese
B. pudding
C. rice cereal
D. bananas

Answer: A

Rationale: Cottage cheese is a good source of calcium and is nutritious. Although


pudding is a source of calcium, it is not the best choice because of the sugar in the
pudding. Rice cereal may be fortified with iron and bananas are a good source of
potassium.
Question format: Multiple Choice
Chapter 23: Growth and Development of the Infant: 28 Days to 1 Year
Cognitive Level: Apply
Client Needs: Health Promotion and Maintenance
Integrated Process: Nursing Process
Reference: p. 522

19. The mother has presented to the pediatric clinic with her 1-year-old infant for
the next round of immunizations. The nurse would question administering the
immunizations if which situation is noted on assessment?
A. The infant has had a runny nose for several days.
B. The infant is being treated for a case of eczema.
C. The infant was given an antipyretic last week.
D. The infant has a rash in the perineal area.

Answer: B

Rationale: An immunization should be postponed if the child has an acute febrile


condition or a condition causing immunosuppression, or if he or she is receiving
corticosteroids, radiation, or antimetabolites. Corticosteroid treatment may be used
for the infant with eczema. Immunizations need not be postponed if the child has a
cold, runny nose, diaper rash, or had antipyretics.
Question format: Multiple Choice
Chapter 23: Growth and Development of the Infant: 28 Days to 1 Year
Cognitive Level: Apply
Client Needs: Physiological Integrity: Pharmacological and Parenteral Therapies
Integrated Process: Nursing Process
Reference: p. 523

20. The nurse is conducting a class for new parents of infants. The nurse
determines the session is successful when the parents correctly choose which
instruction concerning bathing their infant?
A. "When I bathe my baby, I shampoo his hair each time as well."
B. "Giving my baby a bath every day during cold weather will help her stay warm."
C. "My husband uses plain water on our baby's face but I use mild soap."
D. "Now that my baby is bigger, she seems to enjoy having powder applied after
her bath."

Answer: A

Rationale: Regular shampooing is important to prevent seborrheic dermatitis


(cradle cap). The infant does not need to be bathed every day; plain water should
be used on the infant's face (not soap), and powder should not be used after the
bath because it can cause irritation and possible respiratory problems when inhaled
by the infant.
Question format: Multiple Choice
Chapter 23: Growth and Development of the Infant: 28 Days to 1 Year
Cognitive Level: Apply
Client Needs: Health Promotion and Maintenance
Integrated Process: Teaching/Learning
Reference: p. 524

21. The nurse is meeting with a group of older siblings of infants to discuss various
aspects of infant care.The group will be helping the parents with infant care. Which
instruction should the nurse prioritize with this group?
A. The infant should be dressed more warmly than older children and caregivers.
B. The infant should wear hard-soled shoes in order to protect their feet from
injury.
C. The infant sleeps 10 to 12 hours at night and can take two to three naps during
the day.
D. The infant should be sound asleep before being put into the crib for sleeping.

Answer: C

Rationale: Most infants sleep 10 to 12 hours at night and take two to three naps
during the day. By being put to bed while awake and allowed to fall asleep, the
infant learns good sleeping habits. The infant should be dressed in the same
amount of clothing the adult finds comfortable. Hard-soled shoes are not needed by
infants and may hamper the development of the foot.
Question format: Multiple Choice
Chapter 23: Growth and Development of the Infant: 28 Days to 1 Year
Cognitive Level: Understand
Client Needs: Health Promotion and Maintenance
Integrated Process: Teaching/Learning
Reference: p. 526

22. The nurse is conducting a training session for families who are to receive
assistance from the Women, Infants, and Children (WIC) supplemental food
program. The nurse determines the session is successful after the participants
correctly choose which food items they will be able to receive through the program?
Select all that apply.
A. milk
B. dry beans
C. peanuts
D. juice
E. bread
F. eggs

Answer: A, B, D, E, F

Rationale: The foods prescribed by the WIC program include iron-fortified infant
formula and cereal, milk, dry beans, peanut butter, cheese, juice, whole wheat
bread, fruits and vegetables, canned fish, and eggs. Peanuts are not included.
Question format: Multiple Select
Chapter 23: Growth and Development of the Infant: 28 Days to 1 Year
Cognitive Level: Apply
Client Needs: Health Promotion and Maintenance
Integrated Process: Nursing Process
Reference: p. 520

23. A group of nursing students are preparing a presentation illustrating basic


safety measures which can be utilized for infants. Which measures should the
students prioritize in their presentation? Select all that apply.
A. Bottle should only be propped for infants 8 months or older.
B. Crib and playpen bars should be no more than 2 3/8 inches apart.
C. Only small pillows should be used in cribs.
D. Car seats should be placed in back seats.
E. A safe temperature for hot water heaters in households with infants is 120°F
(48.9°C).

Answer: B, D, E

Rationale: Crib and playpen bars should be no more than 2 3/8 inches apart so the
infant can be safe from getting body parts caught between the bars. Car seats are
placed in the back seat and manufacturers' instructions are followed regarding
forward or backward facing depending on the age and size of the child. Water
heaters should be set no higher then 120°F (48.8°C) to prevent potential burns.
Bottles should never be propped and pillows are not placed in cribs of infants.
Question format: Multiple Select
Chapter 23: Growth and Development of the Infant: 28 Days to 1 Year
Cognitive Level: Apply
Client Needs: Safe, Effective Care Environment: Safety and Infection Control
Integrated Process: Teaching/Learning
Reference: p. 526

24. The pediatric nurse is careful to monitor and assess the growth and
development of all clients. Which characteristic should the nurse prepare to assess
in the infants?
A. Has an increased attention span and can be interested in an activity for a long
length of time.
B. Takes in new information at a rapid rate and asks "why" and "how".
C. Insists they can "do it" and the next moment they revert to being dependent.
D. Grows and develops skills more rapidly than at any other time in their life.

Answer: D

Rationale: The infant grows and develops skills more rapidly than he or she ever
will again. The toddler insists he or she can do things one minute and then becomes
dependent the next minute. The preschool age child soaks in information and asks
"why" and "how" over and over. The school-age child has a longer attention span
and can become absorbed in a craft or activity for several hours.
Question format: Multiple Choice
Chapter 23: Growth and Development of the Infant: 28 Days to 1 Year
Cognitive Level: Understand
Client Needs: Health Promotion and Maintenance
Integrated Process: Nursing Process
Reference: p. 512

25. The nurse is assessing the motor skills of an infant who is 9 months old. Which
motor skills should the nurse prepare to assess in an infant achieving normal
growth and development milestones?
A. startles then cries when hearing a loud noise
B. is eating cereal using the thumb and index finger
C. moves from a prone to sitting to standing position
D. can stand and walk around furniture

Answer: B

Rationale: The pincer grasp, use of the thumb and index finger, is a fine motor skill
that develops by 32 weeks of age. The Moro or "startle" reflex disappears by 10 to
12 weeks; the child can change from prone to sitting to a standing position and
walks around 40 weeks to 1 year of age.
Question format: Multiple Choice
Chapter 23: Growth and Development of the Infant: 28 Days to 1 Year
Cognitive Level: Apply
Client Needs: Health Promotion and Maintenance
Integrated Process: Nursing Process
Reference: p. 517

26. The nurse is preparing a presentation for a health fair illustrating the major
milestones of infants as they grow and develop. Which fact should the nurse point
out when illustrating an infant's teeth?
A. The first tooth usually erupts by 6 months.
B. The upper incisors are most often the first teeth to erupt.
C. Fluoride should not be used on a child's teeth before 4 or 5 years of age.
D. Swollen or inflamed gums during teething indicate a serious concern.

Answer: A

Rationale: The first deciduous teeth, usually the lower central incisors, erupt by 6
months of age, but may be anytime between 4 to 8 months. Swollen or inflamed
gums during teething is common. A cold teething ring can help soothe the baby's
discomfort. The American Dental Association recommends administration of fluoride
to infants and children in areas where the fluoride content of drinking water is
inadequate or absent.
Question format: Multiple Choice
Chapter 23: Growth and Development of the Infant: 28 Days to 1 Year
Cognitive Level: Understand
Client Needs: Health Promotion and Maintenance
Integrated Process: Nursing Process
Reference: p. 514
Chapter 24
1. Every time a toddler's mother asks her a question, the response is "No!"
regardless of what the mother asks the child. What would the nurse recommend to
the mother to help alleviate this behavior?
A. Don't ask the child any questions; just tell her what to do each day.
B. Allow the child choices when asking questions, such as "Do you want to wear the
red or the blue top?"
C. Ask the child to please quit saying no to everything asked of her.
D. Respond negatively back to the child each time the child responds in a negative
fashion.

Answer: B

Rationale: Negativism is commonly seen in toddlers and is an expression of


independence, not defiance. The best way to reduce the number of negative
responses is to ask fewer questions and be sure that any question asked allows the
toddler a choice of behavior or response. Berating the child or telling the toddler
what to do all the time will only make matters worse.
Question format: Multiple Choice
Chapter 24: Growth and Development of the Toddler: 1 to 3 Years
Cognitive Level: Apply
Client Needs: Psychosocial Integrity
Integrated Process: Teaching/Learning
Reference: p. 532

2. A mother brings her 2-year-old child to the pediatrician's office, voicing concerns
about her toddler's growth over the last year. According to the child's records, the
toddler has gained 6 pounds (2.7 kg ) and grown 2.5 in (6.25 cm) since the child's
last visit a year ago. How should the nurse respond to this mother's concerns?
A. Tell her that her child's growth is less than is expected and gather a nutritional
history on the child.
B. Inform the mother that her toddler's growth is within normal limits and there is
nothing to be worried about.
C. Ask the mother if there are other small people in her family.
D. Tell the mother that she needs to return to the pediatrician's office in 3 months
to re-weigh the child and measure his height for any changes.

Answer: B

Rationale: Normally, a toddler's growth is 5 to 10 pounds per year and about 3


inches in height. This child falls within the recommended parameters of growth and
the mother has nothing to be worried about.
Question format: Multiple Choice
Chapter 24: Growth and Development of the Toddler: 1 to 3 Years
Cognitive Level: Apply
Client Needs: Health Promotion and Maintenance
Integrated Process: Nursing Process
Reference: p. 531-532

3. A first-time parent asks the nurse what toys would be appropriate for her son's
second birthday next month. What recommendations would be nurse make?
A. dress-up clothing
B. play lawn mower
C. water gun
D. colorful rattle

Answer: B

Rationale: Toys for toddlers should include imitative toys, toys that encourage fine
and gross motor development, and toys that involve socialization. A play lawn
mower meets these criteria by being something the child can push and imitate the
parent's activities. A 2-year-old is too old for a rattle and dress-up clothing and a
water gun are too advanced.
Question format: Multiple Choice
Chapter 24: Growth and Development of the Toddler: 1 to 3 Years
Cognitive Level: Apply
Client Needs: Health Promotion and Maintenance
Integrated Process: Teaching/Learning
Reference: p. 533

4. A nurse is presenting a class on discipline for a group of parents of toddlers.


What information would be important for the nurse to teach this group? Select all
that apply.
A. Toddlers cannot learn self-control until at least 3 to 4 years of age.
B. Consistency in the rules is important so the child understands what is expected.
C. If a child does something wrong, the parent must address the behavior
immediately so the child understands what they did wrong.
D. Even at this young age, children need boundaries.
E. If a child hits or bites another child, the parents should scold them, saying such
things as "You are very naughty for biting Rachel."

Answer: B, C, D

Rationale: Discipline for toddlers must have consistency and correct timing. Parents
need to come to a consensus on how to discipline their child and do so consistently
and in a unified fashion. Also, the toddler needs to receive negative feedback for
negative behavior as soon as the infraction occurs so the child understands what
they did wrong. Parents should never label the child as bad, just their behavior.
Every child needs boundaries—it is just that every family's boundaries may vary.
Discipline begins early in life and toddlers can learn self-control.
Question format: Multiple Select
Chapter 24: Growth and Development of the Toddler: 1 to 3 Years
Cognitive Level: Apply
Client Needs: Health Promotion and Maintenance
Integrated Process: Teaching/Learning
Reference: p. 533-534

5. A mother calls the pediatrician's office upset because her 2-year-old son has
begun acting out now that the new baby is home. He wants to have a bottle like the
newborn and has begun to have accidents in his pants. Which statement by the
nurse would best address this problem?
A. "Often, the first child is jealous of the new baby. Just ignore his acting out and
he will stop."
B. "You need to scold him for wetting his pants and have him change his underwear
himself."
C. "Offer to let him drink some formula in a cup. He will see that being a baby is
not so much fun. "
D. "Set aside time to spend one-on-one with your older child and make him
understand that he is still loved and very special."

Answer: D

Rationale: When parents have a second child, the first-born is often jealous and
feels pushed aside by the new baby. Parents need to make time for the first child
and set aside one-on-one time with them, showing them that they are still loved
and that there is enough time and love for both children. Ignoring the jealous
behavior will not help the situation, nor will scolding him for his accidents.
Regression is commonly seen with the arrival of a new baby. Offering him formula
in a cup does not address the underlying problem of jealousy and feeling neglected.
Question format: Multiple Choice
Chapter 24: Growth and Development of the Toddler: 1 to 3 Years
Cognitive Level: Apply
Client Needs: Psychosocial Integrity
Integrated Process: Teaching/Learning
Reference: p. 534

6. A 2-year-old child is reported to be a "picky eater" by his father. What pointers


could the nurse provide the parent to help minimize stress related to mealtime?
Select all that apply.
A. Offer small portions of 1 to 2 teaspoons to the child.
B. Food jags needs to be addressed and not given in to.
C. Mealtime for a toddler should not exceed 20 minutes.
D. Do not use sweets as a reward for cleaning their plate.
E. Feed the toddler before the rest of the family.

Answer: A, C, D

Rationale: Mealtime with toddlers should not be stressful for families if they follow
some simple feeding tips. Food jags are common and are not harmful. Portion
control of one to two teaspoons is less overwhelming for the toddler, making them
more likely to eat all their food. Limiting mealtime to 20 minutes prevents
excessive dawdling. Toddlers should never be rewarded with sweets or desserts for
good eating habits. Toddlers need the socialization of mealtime with the family and
should eat with the family.
Question format: Multiple Select
Chapter 24: Growth and Development of the Toddler: 1 to 3 Years
Cognitive Level: Apply
Client Needs: Health Promotion and Maintenance
Integrated Process: Teaching/Learning
Reference: p. 535

7. A nurse is reviewing important concepts related to bathing and tub baths with a
parent of a toddler. Which statement by the parent indicates the need for additional
teaching?
A. "I can let my toddler have toys in the tub during bath time."
B. "I should never leave the toddler alone in the tub at any time."
C. "It's okay to let my toddler take bubble baths and play in the bubbles."
D. "I will give my toddler a bath every day at the same time."

Answer: C

Rationale: Bubble baths are not recommended for children, especially girls, due to
the increased incidence of urinary tract infections associated with the use of bubble
bath soap. All other behaviors are appropriate.
Question format: Multiple Choice
Chapter 24: Growth and Development of the Toddler: 1 to 3 Years
Cognitive Level: Analyze
Client Needs: Health Promotion and Maintenance
Integrated Process: Teaching/Learning
Reference: p. 536

8. A toddler is hospitalized and the nurse wants to make the transition from home
to hospital as easy as possible for him. Which action by the nurse would be most
beneficial to assist the toddler in adapting to the hospital?
A. Tell the child what is expected of him to help with compliance.
B. Instruct the parents to allow the nurse to do everything for the child to aid in
attachment.
C. Follow the child's home routines as much as possible while in the hospital.
D. Allow the child to dictate when and what they want to do and adhere to their
requests.

Answer: C

Rationale: Ritualism is used by toddlers to help them adjust to new situations and
provide security for them. By adhering to the home routines regarding such things
as bedtime rituals or how they get dressed, the child is much more comfortable and
more likely to positively adjust to the hospitalization. Telling the toddler what to do,
preventing the parents from performing care tasks, and giving control over to the
child for decisions all would be counterproductive to adjusting to hospitalization.
Question format: Multiple Choice
Chapter 24: Growth and Development of the Toddler: 1 to 3 Years
Cognitive Level: Apply
Client Needs: Psychosocial Integrity
Integrated Process: Nursing Process
Reference: p. 542

9. Parents are beginning to potty train their 2-year-old child and seek advice from
the nurse on how to be successful in this endeavor. Which statement by the parents
indicates that further teaching is needed?
A. "We will place him on the potty for 5 minutes for each session."
B. "I bought him 'big boy' underwear for him to use instead of diapers."
C. "He wants to accompany me to the bathroom but I prefer to go alone."
D. "I will wait until he is off the toilet before flushing it."

Answer: C

Rationale: Allowing a toddler to observe a parent or older sibling going to the


bathroom serves as a positive role model and helps the child understand what he or
she is to do there.
Question format: Multiple Choice
Chapter 24: Growth and Development of the Toddler: 1 to 3 Years
Cognitive Level: Apply
Client Needs: Health Promotion and Maintenance
Integrated Process: Teaching/Learning
Reference: p. 538

10. A mother tells the nurse that her toddler does not want to go to bed at night
and keeps getting back up when she is put to bed. What recommendations would
the nurse make to this mother to foster sleep in the toddler?
A. Place the child in her bed, tell her goodnight, and then lock the door.
B. Read the child a book in bed and take time to calm the child down before turning
out the lights.
C. Instruct the toddler to stay in bed or she will have her favorite stuffed animal
taken away from her.
D. If she does not want to go to sleep at her normal bedtime, let her stay up for a
little while longer.

Answer: B

Rationale: Bedtime rituals are commonly used to prepare children for going to bed.
By slowing down, reading a book in bed with the lights down, it calms the child and
transitions them to a state of sleepiness, when they can then go to sleep. Parents
are never advised to lock the child in the room, threaten them with taking away a
favorite toy or stuffed animal, or allow the child to stall and prolong their bedtime.
Parents must be firm but kind when enforcing bedtimes.
Question format: Multiple Choice
Chapter 24: Growth and Development of the Toddler: 1 to 3 Years
Cognitive Level: Apply
Client Needs: Health Promotion and Maintenance
Integrated Process: Teaching/Learning
Reference: p. 539

11. Which statement by a parent would best prepare the toddler for the parent's
return if the parent must leave the hospital?
A. "I will be back in the morning. Mommy loves you."
B. "I will be back after you eat your dinner and SpongeBob goes off."
C. "I will be back later this afternoon."
D. "I will come back when it is time for your bath tonight."

Answer: B

Rationale: Toddlers have no real concept of time and equate time to events in their
lives. So, the best way for the mother to tell the child when she will be back at the
hospital is to relate her arrival to events such as the child's dinner and TV programs
the child likes.
Question format: Multiple Choice
Chapter 24: Growth and Development of the Toddler: 1 to 3 Years
Cognitive Level: Apply
Client Needs: Psychosocial Integrity
Integrated Process: Teaching/Learning
Reference: p. 542

12. The nurse is preparing to assess a 2-year-old at a well-child visit and notes the
child was 22 lbs (9.98 kg) and 24 in (60.96 cm) tall at 1 year old. The nurse
determines the child is following a normal pattern of growth after obtaining which
set of current measurements?
A. 27 lbs (12.27 kg) and 26 in (66.04 cm) tall
B. 32 lbs (14.54 kg) and 27 in (68.58 cm) tall
C. 38 lbs (17.27 kg) and 32 in (81.28 cm) tall
D. 40 lbs (18.18 kg) and 33 in (83.82 cm) tall

Answer: B

Rationale: Toddlerhood is a time of slowed growth and rapid development. Each


year the toddler gains 4.5 to 6.0 lb (2. to 2.7 kg) and about 3 in (7.62 cm). A child
weighing 22 lbs (9.98 kg) at age 1 year would be expected to now weigh 31 to 34
lbs (14.1 to 15.45 kg). A height of 24 in at age 1 year (60.96 cm) should now be
approximately 27 in (68.58 cm).
Question format: Multiple Choice
Chapter 24: Growth and Development of the Toddler: 1 to 3 Years
Cognitive Level: Apply
Client Needs: Health Promotion and Maintenance
Integrated Process: Nursing Process
Reference: p. 531-532
13. A nursing instructor is leading a class discussion exploring the various aspects
of Erikson's theories of the developmental tasks of toddlers. The instructor
determines the session is successful when the students correctly choose which task
as a priority for toddlers?
A. Learning to trust
B. Learning to speak
C. Learning to act on one's own
D. Learning to understand and respond to discipline

Answer: C

Rationale: Erikson's psychosocial developmental task for toddlers is to achieve


autonomy (independence) while overcoming doubt and shame. Erikson's
psychosocial developmental task for infants is to develop a sense of trust. Learning
to speak and to understand and respond to discipline are not developmental tasks
according to Erikson.
Question format: Multiple Choice
Chapter 24: Growth and Development of the Toddler: 1 to 3 Years
Cognitive Level: Apply
Client Needs: Psychosocial Integrity
Integrated Process: Teaching/Learning
Reference: p. 532

14. A grandmother who is the primary caregiver of a 2-year-old is expressing


concern about how to best handle the temper tantrums that can occur two or three
times a day, often in public places. She explains she spanked her own children for
this but she is worried this is not the best way to handle the situation. Which
response from the nurse will best address this concern?
A. "Spanking is controversial but sometimes necessary, so use it if it works."
B. "Warn the child that they will be punished when they are back at home then
follow through with the punishment."
C. "Remain calm, pick the child up, and move to a quiet and neutral place until the
child gains self-control; don't give in to the child's demands."
D. "Remind the child that they are in a public place and ask the child to respect
those around them; reward the child if the child responds by calming themself."

Answer: C

Rationale: Remaining calm is a must. It is not easy to handle a small child who
drops to the floor screaming and kicking in rage in the middle of the supermarket or
the sidewalk, nor are comments from onlookers at all helpful. The best a caregiver
can do is pick up the out-of-control child as calmly as possible and carry him or her
to a quiet, neutral place to regain self-control. Reasoning, scolding, or punishing
during a tantrum is useless. Do not yield the point or give in to the child's whim.
That would tell the child that to get whatever one wants, a person need only throw
oneself on the floor and scream. The child would have to learn painfully later in life
that people cannot be controlled in this manner. Spanking or other physical
punishment usually does not work well because the child is merely taught that
hitting or other physical violence is acceptable and a child who is spanked
frequently becomes immune to it.
Question format: Multiple Choice
Chapter 24: Growth and Development of the Toddler: 1 to 3 Years
Cognitive Level: Apply
Client Needs: Health Promotion and Maintenance
Integrated Process: Teaching/Learning
Reference: p. 533

15. The caregivers of a 2-year-old are concerned the child is not learning how to
share and play well with other children. While acknowledging their concern and
devotion, the nurse should point out which activity would be best for this child's
developmental level?
A. Mowing the lawn with a toy lawnmower
B. Looking at large print magazines
C. Sharing finger paints and painting with the caregiver
D. Throwing a baseball-sized ball

Answer: A

Rationale: Toddlers enjoy talking on a play telephone. They like pots, pans, and
toys such as brooms, dishes, and lawnmowers that help them imitate the adults in
their environment and promote socialization. Toys that involve the toddler's new
gross motor skills, such as push-pull toys, rocking horses, large blocks, and balls
are popular. Fine motor skills are developed by use of thick crayons, modeling clay,
finger paints, wooden puzzles with large pieces, toys with pieces that fit into shaped
holes, and cloth books. The toddler will not be interested in sharing toys until the
later stage of toddlerhood; adults should not make an issue of sharing at this early
stage.
Question format: Multiple Choice
Chapter 24: Growth and Development of the Toddler: 1 to 3 Years
Cognitive Level: Apply
Client Needs: Health Promotion and Maintenance
Integrated Process: Nursing Process
Reference: p. 533

16. The nurse is preparing a presentation for a health fair which will illustrate
various ways to help introduce siblings to a new member of the family. Which
suggestion should the nurse prioritize to help older siblings, especially toddlers,
understand the change in the family dynamics?
A. Plan time for the secondary caregiver to focus on the toddler while the primary
caregiver focuses on the infant.
B. Plan time for the primary caregiver to focus on the toddler while the secondary
caregiver focuses on the infant.
C. Have a grandparent or another special adult in the child's life take the toddler on
an errand or a special visit.
D. Move the toddler to a new bedroom with a "grown-up-bed."
Answer: B

Rationale: The secondary caregiver can occasionally take over the care of the new
baby while the mother or other primary caregiver devotes herself to the toddler.
The primary caregiver might also plan special times with the toddler when the new
infant is sleeping and the caregiver has no interruptions. This approach helps the
toddler feel special. Moving the older child to a larger bed lets the toddler take pride
in being "grown up" now, but it should be done some time before the new baby
appears. While acknowledging that time with another adult can be a special time,
the main concern is for the toddler to understand they are not being replaced by
the newest member of the family.
Question format: Multiple Choice
Chapter 24: Growth and Development of the Toddler: 1 to 3 Years
Cognitive Level: Apply
Client Needs: Psychosocial Integrity
Integrated Process: Teaching/Learning
Reference: p. 534

17. The nurse is conducting an assessment on a 1-year-old healthy child and


commends the parents for maintaining routine visits. When questioned by the
parents as to the recommended schedule for future visits, which schedule should
the nurse recommend?
A. quarterly until year 2, then every 6 months until the child enters school
B. at 15 months, 18 months, then every 6 months until the child enters school
C. at 15 months, 24 months, then annually until the child enters school
D. at 15 months and at least annually thereafter

Answer: D

Rationale: The child who has been seen routinely up to 1 year old should be seen
next at 15 months for immunization boosters and at least annually thereafter. If the
child has not maintained the routine schedule for immunizations and other
assessments, then the schedule will depend on which immunization boosters and/or
assessments are deemed appropriate.
Question format: Multiple Choice
Chapter 24: Growth and Development of the Toddler: 1 to 3 Years
Cognitive Level: Apply
Client Needs: Health Promotion and Maintenance
Integrated Process: Nursing Process
Reference: p. 536

18. The mother of a 15-month-old son is returning to work and wants to place her
son in the day care close to work; however, they will only accept potty-trained
children. Which response from the nurse will best address this situation in
answering the mother's questions of how best to potty train her son?
A. "Encourage your son to watch his older siblings use the toilet."
B. "Get your son a potty chair and have him sit on it for a few minutes each day."
C. "Each time you change his diaper, tell your son how important and fun it is to
use the potty chair."
D. "Wait a few more months until your son has more muscle control and shows
signs that he's ready to be potty trained."

Answer: D

Rationale: To be able to cooperate in toilet training, the child's anal and urethral
sphincter muscles must have developed to the stage where the child can control
them. Control of the anal sphincter usually develops first. The child also must be
able to postpone the urge to defecate or urinate until reaching the toilet or potty
and must be able to signal the need before the event. In addition, before toilet
training can occur, the child must have a desire to please the caregiver by holding
feces and urine rather than satisfying his/her own immediate need for gratification.
This level of maturation seldom takes place before the age of 18 to 24 months.
Question format: Multiple Choice
Chapter 24: Growth and Development of the Toddler: 1 to 3 Years
Cognitive Level: Apply
Client Needs: Health Promotion and Maintenance
Integrated Process: Teaching/Learning
Reference: p. 538

19. The pediatric nurse is presenting basic safety tips at a local health fair for
families. The nurse should point out the majority of hospital visits for toddlers can
be prevented by exercising which precaution?
A. properly use car seats
B. safely store all chemical substances
C. use plastic protectors in all electrical outlets
D. close supervision during bath time

Answer: B

Rationale: Poisoning is still the most common medical emergency in children with
the highest incidence between the ages of 1 to 4 years. Even with precautionary
labeling and "child-resistant" packaging of medication and household cleaners,
children display amazing ingenuity in opening bottles and packages that catch their
curiosity. Medications such as acetaminophen, salicylates (aspirin), laxatives,
sedatives, tranquilizers, analgesics, antihistamines, cold medicines, and birth-
control pills are commonly associated with poisoning and also need to properly be
stored out of reach of the toddler. The proper use of car seats, preventing access to
electrical outlets, and bath time supervision are also noted to be the cause of
medical emergencies. However, poisoning remains the number one reason.
Question format: Multiple Choice
Chapter 24: Growth and Development of the Toddler: 1 to 3 Years
Cognitive Level: Apply
Client Needs: Safe, Effective Care Environment: Safety and Infection Control
Integrated Process: Teaching/Learning
Reference: p. 540-541
20. A single mother with three young children is reluctant to leave her crying and
upset 16-month-old daughter overnight in the hospital but needs to go home to
care for the other children. Which suggestion from the nurse will best address the
fears and concerns of both the child and mother?
A. Remind them staying in the hospital now will help the child get well quicker and
be home soon, but the other children should not be alone.
B. Distract the child with a special blanket, stuffed animal, or other "lovey" from
home while the mother quietly slips out.
C. Encourage the mother to give the child a personal item of the mother's to hold
on to until she returns and to tell the child a specific time she will return, such as
"when breakfast comes in the morning."
D. Tell both the mother and child that the child will be carefully guarded and won't
be in as much danger as she might be if she were home exploring her environment.

Answer: C

Rationale: When the family caregiver must leave the toddler, it may be helpful for
the adult to give the child some personal item to keep until the adult returns. The
caregiver can tell the child he or she will return "when the cartoons come on TV" or
"when your lunch comes." These are concrete times that the toddler will probably
understand. The toddler is too young to understand that staying is important for
her recovery. Distracting the child while the mother leaves may increase the child's
anxiety when she realizes her mother is gone. Although the child will be watched
closely in the hospital setting, toddlers explore their environment wherever they
are.
Question format: Multiple Choice
Chapter 24: Growth and Development of the Toddler: 1 to 3 Years
Cognitive Level: Apply
Client Needs: Psychosocial Integrity
Integrated Process: Caring
Reference: p. 542

21. The nurse is caring for a toddler in the pediatric unit and notes the child is
responding according to expected developmental stages. Which characteristic will
the nurse predict this toddler to exhibit while in the hospital?
A. will be interested in an activity for a long period of time
B. learns new things quickly by asking questions
C. insists on doing a new skill and then asking for help
D. will go home knowing how to do more things

Answer: C

Rationale: The toddler insists they can do things one minute and then becomes
dependent the next minute. The preschool-age child soaks in information and asks
"why" and "how" over and over. The school-age child has a longer attention span
and can become absorbed in a craft or activity for several hours. The infant grows
and develops skills more rapidly than he or she ever will again.
Question format: Multiple Choice
Chapter 24: Growth and Development of the Toddler: 1 to 3 Years
Cognitive Level: Apply
Client Needs: Psychosocial Integrity
Integrated Process: Nursing Process
Reference: p. 542-543

22. The nurse is presenting an in-service training to a group of pediatric nurses on


the topic of play. The nurse determines the session is successful when the group
correctly chooses which example as best displaying toddlers playing?
A. Playing apart from others without being part of a group
B. Playing together in an activity without organization
C. Playing in an organized group with each other
D. Playing independently and are side by side

Answer: D

Rationale: Parallel play occurs when the toddler plays alongside other children but
not with them. During cooperative play children play in an organized group with
each other, as in team sports. Associative play occurs when children play together
and are engaged in a similar activity but without organization, rules, or a leader—
and each child does what she or he wishes. Solitary independent play means
playing apart from others without making an effort to be part of the group or group
activity.
Question format: Multiple Choice
Chapter 24: Growth and Development of the Toddler: 1 to 3 Years
Cognitive Level: Analyze
Client Needs: Health Promotion and Maintenance
Integrated Process: Teaching/Learning
Reference: p. 533-534

23. The nurse is supervising a play group of children on the unit. The nurse expects
the toddlers will most likely be involved in which activity?
A. Pretending to be mommies and daddies in the play house
B. Playing with the plastic vacuum cleaner and pushing it around the room
C. Painting pictures in the art corner of the room
D. Watching a movie with other children their age

Answer: B

Rationale: Playtime for the toddler involves imitation of the people around them
such as adults, siblings, and other children. Push-pull toys allow them to use their
developing gross motor skills. Preschool children have imitative play, pretending to
be the mommy, the daddy, a policeman, a cowboy, or other familiar characters.
The school-age child enjoys group activities and making things, such as drawings,
paintings, and craft projects. The adolescent enjoys activities they can participate in
with their peers.
Question format: Multiple Choice
Chapter 24: Growth and Development of the Toddler: 1 to 3 Years
Cognitive Level: Apply
Client Needs: Health Promotion and Maintenance
Integrated Process: Nursing Process
Reference: p. 533

24. A 15-month-old toddler is brought to the pediatrician's office for a well-child


checkup and the nurse is reviewing health promotion with the family. What
recommendation(s) would the nurse make to the family to help ensure the child's
health? Select all that apply.
A. Begin dental examinations at age 1 year.
B. Provide foods rich in calories and carbohydrates for the child.
C. Keep all medications locked up and out of reach for the child.
D. Turn the hot water heater temperature down to 120℉ (48.9℉).
E. Never allow the child to ride in the car unless secured in a car seat.

Answer: C, D, E

Rationale: Health promotion begins by properly educating the parents on how to


care for their child. Dental visits begin at age 1 year. Foods need to be nutritious
but not highly seasoned or disproportionately high in carbohydrates or calories. All
medications and toxic substances need to be locked up and out of reach of the
toddler. Hot water temperatures should be lowered to 120℉ (48.9℉) and the child
must be secured in a federally approved child safety seat prior to starting the car.
Question format: Multiple Select
Chapter 24: Growth and Development of the Toddler: 1 to 3 Years
Cognitive Level: Apply
Client Needs: Health Promotion and Maintenance
Integrated Process: Teaching/Learning
Reference: p. 536

25. A 2-year-old child is shopping with her mother when she suddenly falls to the
ground and begins to scream, "I want it!" over and over regarding a bag of candy.
What would the nurse recommend to the mother to deal with this behavior? Select
all that apply.
A. Reason with the toddler and explain that the candy is not nutritious for her.
B. Remain calm and ignore the tantrum.
C. Acknowledge the behavior by giving into the toddler's demands and buying the
candy.
D. Pick the toddler up and take her to the restroom for a spanking.
E. Stay nearby to ensure the child's safety without giving in to the child's desires.

Answer: B, E

Rationale: Temper tantrums in toddlers are very common as they try to control
their environment and the caregiver's environment. They become frustrated at their
inability to do so or to verbalize their desires. If a toddler has a temper tantrum,
the best thing for the parent to do is ignore them and protect them from harm.
Parents cannot reason with a toddler—they lack the ability to understand or the
desire to change their behavior. Never give in to their demands; they will only learn
that if you scream loud enough, they get their way. However, spanking is not
recommended. The child has just lost control and needs time to regain self-control.
Question format: Multiple Select
Chapter 24: Growth and Development of the Toddler: 1 to 3 Years
Cognitive Level: Apply
Client Needs: Psychosocial Integrity
Integrated Process: Teaching/Learning
Reference: p. 533
Chapter 25
1. The nurse is assessing a 3-year-old at a routine well-child visit. Which
assessment should the nurse prioritize in the vision check?
A. visual acuity
B. eye coordination
C. depth perception
D. color perception

Answer: B

Rationale: During the preschool years, the child's vision should be checked to
screen for amblyopia (dimness of vision from disuse of the eye, or "lazy eye"). This
can result from the eyes not coordinating eye movements and working together to
use both eyes to see. Usually by age 6 the child has achieved 20/20 vision, but
mature depth perception may not occur in some children until 8 to 10 years of age.
Question format: Multiple Choice
Chapter 25: Growth and Development of the Preschool Child: 3 to 6 Years
Cognitive Level: Understand
Client Needs: Health Promotion and Maintenance
Integrated Process: Nursing Process
Reference: p. 548

2. The pediatric nurse is discussing the daily activities of a 4-year-old with the
caregiver to assess growth and development status. The nurse would document
that the child has reached the initiative stage of development if the caregiver
indicates the child participates in which activity?
A. tries to sweep up spilled cereal but cries when can't do well
B. broke a dish but blamed it on a friend the caregivers don't know
C. refuses to hold anyone's hand while crossing the street
D. gets upset when a babysitter is in charge, but will do what is asked by the
babysitter

Answer: A

Rationale: According to Erikson, the developmental task of the preschool age is


initiative versus guilt. Preschoolers often try to find ways to do things to help, but
they may feel guilty if scolded when they fail because of inexperience or lack of
skill. Family caregivers need to remember that preschoolers are developing
initiative and a sense of guilt. They want to be good and follow instructions; they
feel bad when they do not, even if they are not physically punished. Imaginary
playmates are common in the preschool child, but blaming the incident on the
imaginary playmate or refusing to hold the mother's hand does not show initiative.
A younger child would commonly show anxiety when being left with the caregiver.
Question format: Multiple Choice
Chapter 25: Growth and Development of the Preschool Child: 3 to 6 Years
Cognitive Level: Apply
Client Needs: Health Promotion and Maintenance
Integrated Process: Nursing Process
Reference: p. 553-554

3. During a well-child visit, the caregiver expresses concern that the 3-year-old
child often stutters when speaking. Which response should the nurse prioritize to
best assist this family?
A. "Stuttering is common in young children because they are not physically capable
of forming all the sounds."
B. "Stuttering is usually indicative of a hearing loss."
C. "Difficulties with speaking generally indicate that the adults in the child's life are
not reading to the child enough."
D. "Children of this age may stutter while they search for just the right word."

Answer: D

Rationale: Between ages 3 and 5, language development is generally rapid. Most 3-


year-old children can construct simple sentences, but their speech has many
hesitations and repetitions as they search for the right word or try to make the
right sound. Stuttering can develop during this period but usually disappears within
3 to 6 months. Physical capability, hearing loss, or lack of being read to are not
reasons stuttering occurs.
Question format: Multiple Choice
Chapter 25: Growth and Development of the Preschool Child: 3 to 6 Years
Cognitive Level: Apply
Client Needs: Health Promotion and Maintenance
Integrated Process: Nursing Process
Reference: p. 549

4. The parents of 5-year-old boy are concerned about the how a recent motorcycle
accident to his father will affect the child. Although the father has fully recovered,
the child is very concerned if the father is away longer than expected; the child is
not as talkative but appears withdrawn and quiet. The nurse should point out the
child's behavior is likely related to which factor?
A. The child is afraid of losing his father and trying show how much he loves him.
B. The boy is afraid of being hurt himself and thinks being "especially good" will
protect him from accidents.
C. The boy believes he caused the accident by telling his father he "hoped he
crashed" when the boy couldn't go along.
D. The child is imitating the adults' behavior and just trying to be nice to everyone.

Answer: C

Rationale: Preschoolers have learned to think about something without actually


seeing it: to visualize or imagine. This normal development, sometimes called
magical thinking, makes it difficult for them to separate fantasy from reality.
Preschoolers believe that words or thoughts can make things real, and this belief
can have either positive or negative results. The child needs reassurance that the
accident was not his fault. The other choices do not demonstrate the "magical
thinking" that preschoolers tend to demonstrate later in life.
Question format: Multiple Choice
Chapter 25: Growth and Development of the Preschool Child: 3 to 6 Years
Cognitive Level: Apply
Client Needs: Psychosocial Integrity
Integrated Process: Nursing Process
Reference: p. 550

5. The nurse is meeting with a group of young parents to discuss nutrition and their
preschooler. Which response should the nurse prioritize when asked if using
desserts as a reward for good behavior is an appropriate idea?
A. The child may only behave on days when dessert is something that is liked.
B. The child will learn to choose sweets over nutritious food.
C. This will result in the child being overweight.
D. The child can use food to manipulate others' behavior.

Answer: D

Rationale: Food should never be used as a reward or bribe; otherwise, the child will
continue to use food as a means to manipulate the environment and the behavior
of others. Because of the high fat and sugar content of most desserts, the
nutritional value of desserts is usually less than a balanced diet of healthier foods
and could add to the concern of obesity in some children. A child might prefer
sweets over nutritious foods and behave differently if dessert were a reward, but
these choices are not the most appropriate answer.
Question format: Multiple Choice
Chapter 25: Growth and Development of the Preschool Child: 3 to 6 Years
Cognitive Level: Apply
Client Needs: Health Promotion and Maintenance
Integrated Process: Nursing Process
Reference: p. 553-554

6. While the nurse is taking a blood pressure on a 4-year-old, the child states that
the blood-pressure cuff is too tight and angrily says, "That hurt, you big poo-poo
head." What is the most appropriate response by the nurse?
A. Scold the child for the insult while apologizing for hurting her, and loosen the
cuff.
B. Calmly explain that you don't mean to hurt her, loosen the cuff, and tell her that
is isn't nice to call you names.
C. Explain that the cuff will only hurt for minute and ask the child's caregiver to
please tell the child not to speak to you that way.
D. Ask the child's caregiver to please hold the child on their lap until she calms
down.

Answer: B
Rationale: Four- and 5-year-olds delight in using "naughty" words or swearing.
Bathroom words become favorites and taunts, such as "you're a big doo-doo," bring
heady excitement to them. Caregivers may become concerned by this turn of
events, but the child simply may be trying out words to test their impact. By using
a calm, matter-of-fact response when a preschooler uses naughty or swear words,
some of the power of using that type of language will be defused. The child learns
that this is not language to use in the company of others.
Question format: Multiple Choice
Chapter 25: Growth and Development of the Preschool Child: 3 to 6 Years
Cognitive Level: Apply
Client Needs: Psychosocial Integrity
Integrated Process: Nursing Process
Reference: p. 549

7. A first-time father calls the pediatric nurse stating he is concerned that his 4-
year-old daughter still wets the bed almost every night. Remembering his own
experience of being punished for wetting the bed at 4 years old, he is not sure
punishment is the best approach to address this. Which nursing instruction is the
most appropriate?
A. "Disciplining is not likely to be effective, but if the child keeps wetting the bed it
may be necessary."
B. "Bedwetting is not uncommon in young children. Try to calmly change the bed
without showing your frustration."
C. "Setting rules is a parent's job to help the child have acceptable social behavior,
so take away a privilege each time she wets the bed."
D. "Nightly bedwetting up to age 12 is developmentally typical, so you will need to
practice patience with your daughter."

Answer: B

Rationale: Occasional bedwetting is not uncommon for young preschoolers and is


not a concern unless it continues past the age of 7. When the child does have an
accident, treating it in a matter-of-fact way and providing the child with clean, dry
clothing is best. The child should not be disciplined or made to feel he or she is
socially unacceptable when bedwetting occurs.
Question format: Multiple Choice
Chapter 25: Growth and Development of the Preschool Child: 3 to 6 Years
Cognitive Level: Apply
Client Needs: Health Promotion and Maintenance
Integrated Process: Nursing Process
Reference: p. 555

8. The nurse is preparing a safety presentation for a health fair for families. Which
instruction should the nurse prioritize when illustrating car safety and the family?
A. "Stop the car any time the preschooler unbuckles the restraints."
B. "Explain that wearing a seat belt is a law and the police officer will give a ticket if
the seat belt is not buckled."
C. "Set a good example. Wear your own seat belt every time you drive."
D. "Reward the child with candy or some other treat each time the child keeps the
seat belt on."

Answer: C

Rationale: A preschooler wants to please, and if the caregiver consistently wears


the seatbelt, that will become the standard for riding in the car. All states have laws
that define safety seat and restraint requirements for children. Adults must teach
and reinforce these rules. One primary responsibility of adults is always to wear
seat belts themselves and to make certain that the child always is in a safety seat
or has a seat belt on when in a motor vehicle. A child can also be calmly taught that
the vehicle "won't go" unless everyone in it is properly restrained. The child should
be taught respect of rules and laws, but making threats or giving rewards is not
appropriate.
Question format: Multiple Choice
Chapter 25: Growth and Development of the Preschool Child: 3 to 6 Years
Cognitive Level: Apply
Client Needs: Safe, Effective Care Environment: Safety and Infection Control
Integrated Process: Teaching/Learning
Reference: p. 555-556

9. The nurse is caring for a 5-year-old who has been hospitalized after an episode
of asthma. As the nurse prepares to teach the child how to use the nebulizer, which
action should the nurse prioritize?
A. Allow the child to touch and play with the nebulizer for a few minutes before the
treatment.
B. Show the child how to use the nebulizer and tell the child how much easier it is
to breathe afterward.
C. Explain that the child will feel better after the treatment and allow the child to
ask questions.
D. Use a poster or brochure to illustrate to the child how the machine works.

Answer: A

Rationale: Children are often less anxious about procedures if they are allowed to
handle equipment beforehand— and perhaps "use" it on a doll or another toy. Play
is an effective way to let children act out their anxieties and to learn what to expect
from the hospital situation. Explaining how the treatment will help him or her feel
better, using posters, and encouraging the child to ask questions would be
appropriate for the older child, not the preschool-aged child.
Question format: Multiple Choice
Chapter 25: Growth and Development of the Preschool Child: 3 to 6 Years
Cognitive Level: Apply
Client Needs: Psychosocial Integrity
Integrated Process: Nursing Process
Reference: p. 557
10. The nurse is caring for several clients on the pediatric unit. When interacting
with the preschool-age child, which action does the nurse predict will occur?
A. Increased attention span and can be interested in an activity for a long length of
time
B. Takes in new information at a rapid rate and asks "why" and "how"
C. Insists doing something and the next moment reverts to being dependent
D. Grows and develops skills more rapidly than at any other time in their life

Answer: B

Rationale: The preschool-aged child soaks in information and asks "why" and "how"
over and over. The school-aged child has a longer attention span and can become
absorbed in a craft or activity for several hours. The toddler insists he or she can do
things one minute and then becomes dependent the next minute. The infant grows
and develops skills more rapidly than he or she ever will again.
Question format: Multiple Choice
Chapter 25: Growth and Development of the Preschool Child: 3 to 6 Years
Cognitive Level: Apply
Client Needs: Health Promotion and Maintenance
Integrated Process: Nursing Process
Reference: p. 548

11. The nurse is presenting an in-service on the types of playing that children may
engage in. The nurse determines the session is successful when the attending
nurses correctly choose which example as representing cooperative play?
A. Playing apart from others without being part of a group.
B. Playing together in an activity without organization.
C. Playing in an organized group with each other.
D. Playing independently and are side-by-side.

Answer: C

Rationale: During cooperative play, children play in an organized group with each
other as in team sports. Solitary independent play means playing apart from others
without making an effort to be part of the group or group activity. Associative play
occurs when children play together and are engaged in a similar activity but without
organization, rules, or a leader, and each child does what she or he wishes. Parallel
play occurs when the toddler plays alongside other children but not with them.
Question format: Multiple Choice
Chapter 25: Growth and Development of the Preschool Child: 3 to 6 Years
Cognitive Level: Apply
Client Needs: Health Promotion and Maintenance
Integrated Process: Teaching/Learning
Reference: p. 551

12. The nurse is monitoring children playing in the unit's playroom. The nurse notes
that some children are involved in associative play by which actions?
A. drawing pictures in the art area
B. pushing toy cars around on a large rug with roads
C. playing a board game with each other
D. several children engrossed in their own tool-and-bench set

Answer: B

Rationale: Associative play occurs when children play together and are engaged in
a similar activity, but without organization, rules, or a leader; each child does
whatthey wish, such as pushing toy cars around on a rug that has roads. Solitary
independent play means playing apart from others without making an effort to be
part of the group or group activity, such as drawing a picture in the art area. During
cooperative play, children play in an organized group with each other as in team
sports or a board game. Parallel play occurs when the toddler plays alongside other
children but not with them, such as each having their own tool-and-bench set to
play with the hammers and other tools.
Question format: Multiple Choice
Chapter 25: Growth and Development of the Preschool Child: 3 to 6 Years
Cognitive Level: Apply
Client Needs: Health Promotion and Maintenance
Integrated Process: Nursing Process
Reference: p. 551

13. The parents of a toddler are concerned their child is not developing correctly
and are questioning the nurse concerning the child's lack of effort to join other
children in a group activity. Which response should the nurse prioritize in
answering the parents?
A. "This is normal for this age group. It's referred to as solitary independent play."
B. "You should try to get your child involved in a local Boys and Girls club to
encourage more interaction."
C. "Perhaps getting your child interested in sports will improve their other play
habits."
D. "Your child is involved with others, just indirectly. See how they sit next to the
other children and play with the same toys?"

Answer: A

Rationale: Solitary independent play means playing apart from others without
making an effort to be part of the group or group activity, which is normal for this
age group. Associative play occurs when children play together and are engaged in
a similar activity but without organization, rules, or a leader, and each child does
what she or he wishes. During cooperative play, children play in an organized group
with each other as in team sports. Parallel play occurs when the toddler plays
alongside other children but not with them. As the child continues to mature, they
will be more prepared and willing to join with others to play alongside in organized
or unorganized situations.
Question format: Multiple Choice
Chapter 25: Growth and Development of the Preschool Child: 3 to 6 Years
Cognitive Level: Apply
Client Needs: Health Promotion and Maintenance
Integrated Process: Communication and Documentation
Reference: p. 551

14. The nursing instructor is illustrating the various types of play. The instructor
determines the class is successful when the students correctly choose which
example as best representing onlooker play?
A. playing apart from others without being part of a group
B. acting out a troubling situation
C. playing in an organized group with each other
D. observing without participating

Answer: D

Rationale: Onlooker play occurs when there is observation without participation,


such as watching television. Solitary independent play means playing apart from
others without making an effort to be part of the group or group activity. Dramatic
play allows a child to act out a troubling situation. During cooperative play, children
play in an organized group with each other as in team sports.
Question format: Multiple Choice
Chapter 25: Growth and Development of the Preschool Child: 3 to 6 Years
Cognitive Level: Apply
Client Needs: Health Promotion and Maintenance
Integrated Process: Teaching/Learning
Reference: p. 551

15. A nurse is preparing a presentation for a health fair discussing various aspects
of preschoolers. Which example should the nurse use to best illustrate dramatic
play?
A. Playing apart from others without being part of a group
B. Acting out a troubling or stressful situation
C. Playing a video game with several other children
D. Watching television or videos

Answer: B

Rationale: Dramatic play allows a child to act out a troubling or stressful situation.
Solitary independent play means playing apart from others without making an
effort to be part of the group or group activity. During cooperative play, children
play in an organized group with each other as in team sports or video games.
Onlooker play occurs when there is observation without participation, such as
watching television or videos.
Question format: Multiple Choice
Chapter 25: Growth and Development of the Preschool Child: 3 to 6 Years
Cognitive Level: Apply
Client Needs: Health Promotion and Maintenance
Integrated Process: Teaching/Learning
Reference: p. 551
16. The nurse has brought a group of preschoolers to the playroom to play. Which
activity would the nurse predict the children to become involved in?
A. Pretending to be mommies and daddies in the playhouse
B. Playing a board game
C. Painting pictures in the art corner of the room
D. Watching a movie with other children their age

Answer: A

Rationale: Preschool children have imitative play, pretending to be the mommy, the
daddy, a policeman, a cowboy, or other familiar characters. The school-aged child
enjoys group activities, such as board games, and making things, such as drawings,
paintings, and craft projects. The adolescent enjoys activities he or she can
participate in with their peers.
Question format: Multiple Choice
Chapter 25: Growth and Development of the Preschool Child: 3 to 6 Years
Cognitive Level: Apply
Client Needs: Health Promotion and Maintenance
Integrated Process: Nursing Process
Reference: p. 551-552

17. When collecting data on a preschool-aged child during a well-child visit, the
nurse discovers the child has gained 12 lb (5.4 kg) and grown 2.5 inches (6.3 cm)
in the last year. The nurse interprets these findings to indicate which situation?
A. Weight and height are within expected patterns of growth.
B. Weight falls within an expected range and height is less than what would be
expected.
C. Weight is above an expected range and height is within an expected range.
D. Weight is below an expected range and height is above an expected range.

Answer: C

Rationale: The preschool period is one of slow growth. The child gains about 4 to 5
lb each year (1.4 to 2.3 kg) and grows about 2.5 inches (6.3 cm). The child's
weight is above the expected gain and the height is what would be expected.
Question format: Multiple Choice
Chapter 25: Growth and Development of the Preschool Child: 3 to 6 Years
Cognitive Level: Apply
Client Needs: Health Promotion and Maintenance
Integrated Process: Nursing Process
Reference: p. 548

18. The nurse is assessing a 3-year-old at a well-child visit and the child appears to
be progressing well. Which activity will the nurse ask the child to attempt to
appropriately assess the fine motor skills of this preschooler?
A. Use scissors.
B. Button clothes.
C. Tie shoelaces.
D. Print a few letters.

Answer: B

Rationale: The 3-year-old should be able to button his clothes and use a pencil or
crayon. By the age of 4 to 5, the child should be able to use scissors, tie shoelaces,
and print his first name.
Question format: Multiple Choice
Chapter 25: Growth and Development of the Preschool Child: 3 to 6 Years
Cognitive Level: Apply
Client Needs: Health Promotion and Maintenance
Integrated Process: Nursing Process
Reference: p. 549

19. The parent of a 4-year-old is expressing concern that this child is not talking as
much—or as well—as her other children did at that age. Which question should the
nurse prioritize when assessing this preschooler for this concern?
A. "How often do you or a family member read to your child?"
B. "Has your child had their hearing tested?"
C. "Does your child have opportunities to have conversations with other people?"
D. "Do you praise and give your child encouragement when the child tries to talk
with you?"

Answer: B

Rationale: Delays or other difficulties in language development may result from


hearing impairment or other physical problems. Although reading to the child,
having conversations with family members and other people, and praising and
encouraging the child's efforts to communicate help the child develop language
skills, most importantly a hearing concern would need to be assessed and treated.
Question format: Multiple Choice
Chapter 25: Growth and Development of the Preschool Child: 3 to 6 Years
Cognitive Level: Apply
Client Needs: Health Promotion and Maintenance
Integrated Process: Nursing Process
Reference: p. 549

20. The father of a 4-year-old is concerned his child is not telling the truth and
blaming others for things that have happened. Which response should the nurse
prioritize after the father shares that the child is blaming someone named
"Andrew" for a broken tool, and they have no idea who this is?
A. "You should punish your son because no child should be telling lies at this age."
B. "You need to show your child the broken tool since at this age they must see
something in order to understand."
C. "Your son may have a friend named Andrew, but it could be an imaginary
friend."
D. "You should watch this type of behavior closely since most children this age tell
the truth."

Answer: C

Rationale: The preschool-aged child may have imaginary playmates who are very
real to them. The imaginary friend often has the characteristics that the child might
wish for. Sometimes the child blames the imaginary friend for breaking a toy or
engaging in another act for which the child does not want to take responsibility. The
child should not be punished because the child is not intentionally telling a lie. At
this age the child can think about things without actually seeing them.
Question format: Multiple Choice
Chapter 25: Growth and Development of the Preschool Child: 3 to 6 Years
Cognitive Level: Apply
Client Needs: Health Promotion and Maintenance
Integrated Process: Nursing Process
Reference: p. 550-551

21. The pediatric nurse is meeting with a group of preschoolers' family members to
discuss various health topics. The nurse determines the sexual development session
is successful after overhearing which comment by one of the participants?
A. "When I find my son masturbating, I will tell him that is unacceptable."
B. "I will tell my daughter that she will have time to explore her body as she gets
older."
C. "I will encourage my son to ask his father any questions that are sexual in
nature."
D. "I feel better knowing that her curiosity is normal."

Answer: D

Rationale: The child's sexual curiosity is a normal, natural part of total curiosity
about oneself and the world. Exploration of the genitalia is natural for the
preschooler. It is one way the child learns to perceive the body as a possible source
of pleasure and is the beginning of the acceptance of sex as natural and
pleasurable. Caregivers can be reassured that this is not uncommon behavior.
Question format: Multiple Choice
Chapter 25: Growth and Development of the Preschool Child: 3 to 6 Years
Cognitive Level: Analyze
Client Needs: Health Promotion and Maintenance
Integrated Process: Teaching/Learning
Reference: p. 551

22. The nurse is presenting nutritional information at a community health fair.


Which suggestion should the nurse prioritize when illustrating proper nutrition for
preschoolers?
A. Need three big meals a day due to rapid growth
B. Need extra calcium for proper muscle growth
C. Snacks throughout the day help the child meet nutritional requirements
D. Should drink at least 4 cups of milk each day

Answer: C

Rationale: The preschool period is not a time of rapid growth, so children do not
need large quantities of food. Protein needs are high to provide for muscle growth.
Portions are smaller than adult-sized portions, so the child may need to have meals
supplemented with nutritious snacks. The preschool child needs 2 to 3 cups of milk
each day.
Question format: Multiple Choice
Chapter 25: Growth and Development of the Preschool Child: 3 to 6 Years
Cognitive Level: Apply
Client Needs: Health Promotion and Maintenance
Integrated Process: Teaching/Learning
Reference: p. 553

23. The mother of a 4-year-old is concerned her child is not eating well. In
addressing the concerns of this mother, which foods should the nurse point out are
high in protein? Select all that apply.
A. cheese and crackers
B. cookies and juice
C. whole grain granola with yogurt
D. strawberries and bananas
E. turkey sandwich

Answer: A, C, E

Rationale: Meat, poultry, fish, milk products, and eggs are good sources of protein.
Whole wheat grains, nuts, peanut butter, and legumes are also good sources of
protein but need to be supplemented by some animal protein, such as meat, eggs,
milk, cheese, cottage cheese, or yogurt. Cookies are usually high in sugar and not
the best for supplying good nutrition. Fruit juices and fruit are good sources of
vitamin C.
Question format: Multiple Select
Chapter 25: Growth and Development of the Preschool Child: 3 to 6 Years
Cognitive Level: Apply
Client Needs: Health Promotion and Maintenance
Integrated Process: Nursing Process
Reference: p. 554

24. The nurse is assessing a 4-year-old on a routine well-child visit. When


assessing the gross motor skills of this preschooler, which activity will the nurse
predict the child to be able to successfully accomplish?
A. Hop on one foot
B. Walk backwards with heel to toe
C. Ride a bicycle
D. Jump rope
Answer: A

Rationale: The 4-year-old should be able to hop on one foot and can control
movements of the hands. By the age of 5 the child can walk backwards heel to toe,
throw and catch a ball well, and jump rope.
Question format: Multiple Choice
Chapter 25: Growth and Development of the Preschool Child: 3 to 6 Years
Cognitive Level: Apply
Client Needs: Health Promotion and Maintenance
Integrated Process: Nursing Process
Reference: p. 549

25. The nurse is observing 4-year-old twins during their well-child appointment.
Which observation by the nurse would indicate the children are demonstrating
cooperative play?
A. They competitively stack blocks and keep score of who stacks the highest
number of blocks.
B. They play with dollhouse furniture, each with different room furnishings.
C. They are building a house for their dolls with blocks in the room.
D. They sit on the floor, near each other, and look at different picture books.

Answer: C

Rationale: Cooperative play is when children work toward a common goal together,
with a distinct theme. The 4-year-old twins demonstrate cooperative play when
they build a house with blocks. Associative play occurs when children play together
and are engaged in similar activity, but without organization, rules, or a leader, and
when each child does what she or he wishes, such as playing with the dollhouse
furniture separately. In parallel play, children play alongside each other but
independently, such as looking at different picture books. Competing for the most
blocks stacked would be an example of competitive play.
Question format: Multiple Choice
Chapter 25: Growth and Development of the Preschool Child: 3 to 6 Years
Cognitive Level: Apply
Client Needs: Health Promotion and Maintenance
Integrated Process: Nursing Process
Reference: p. 551
Chapter 26
1. The 6-year-old at a well-child office visit tells the nurse, "I can't play on teams
because I am not as good at doing things as my big sister is." What suggestion
should the nurse point out to the caregiver that will help increase the child's
feelings of self-confidence?
A. Ask the child's older siblings to compliment her more often.
B. Praise the child for trying even when she can't complete the same activities as
the older sibling.
C. Set up some playdates that include projects or activities in which the child can
be successful.
D. Start a rock collection with the child to show her that she can have fun doing
activities that don't involve other children.

Answer: C

Rationale: As they reach school age, children show an increasing interest in group
activities and in making things. Children of this age work at many activities that
involve motor, cognitive, and social skills. Success in these activities provides the
child with self-confidence and a feeling of competence, but excessive or unrealistic
goals set by a teacher or caregiver will defeat a child and possibly lead to the child
feeling inferior rather than self-confident. Erikson's developmental task for this age
group is industry vs. inferiority. Children who are unsuccessful in completing
activities during this stage, whether from physical, social, or cognitive
disadvantages, develop a feeling of inferiority.
Question format: Multiple Choice
Chapter 26: Growth and Development of the School-Aged Child: 6 to 10 Years
Cognitive Level: Apply
Client Needs: Psychosocial Integrity
Integrated Process: Nursing Process
Reference: p. 562

2. The nurse is preparing a variety of projects for the pediatric clients on the unit to
work on in the playroom. In deciding on projects, the nurse determines the 8-year-
old will be best suited to work on which activity?
A. stack blocks in a tower
B. build a sandcastle with a water-filled moat
C. form vases from blocks of clay
D. put together a model plane

Answer: C

Rationale: At about age 7, the child enters the concrete operational stage (as
identified by Piaget). The skills of conservation (the ability to recognize that a
change in shape does not necessarily mean a change in amount or mass) are
significant in this stage. This begins with the conservation of numbers (when the
child understands that the number of cookies does not change even though they
may be rearranged) and the conservation of mass (when the child can see that an
amount of cookie dough is the same whether in ball form or flattened for baking).
This is followed by conservation of weight, in which the child recognizes that a
pound is a pound, regardless of whether plastic or bricks are weighed. Conservation
of volume (for instance, understanding that a cup of water is the same amount
regardless of the shape of the container) does not come until late in the concrete
operational stage at about 11 or 12 years of age.
Question format: Multiple Choice
Chapter 26: Growth and Development of the School-Aged Child: 6 to 10 Years
Cognitive Level: Apply
Client Needs: Health Promotion and Maintenance
Integrated Process: Nursing Process
Reference: p. 563

3. An 8-year-old boy's foster mother is concerned about three recent cavities found
in his permanent teeth and reports the child eats a nutritional diet, doesn't eat junk
food, and the town water supply is fluoridated. Which suggestion should the nurse
prioritize to this mother in regard to the child's dental health?
A. Encourage the child to abstain from eating sugary snacks at school.
B. Accept that the child is genetically predisposed to having more cavities than
most children.
C. Ensure that the child brushes his teeth after each meal and snacks.
D. Have the child's teeth professionally cleaned every 3 months.

Answer: C

Rationale: Proper dental hygiene includes a routine inspection and conscientious


brushing after meals. A well-balanced diet with plenty of calcium and phosphorus
and minimal sugar is important to healthy teeth. Foods containing sugar should be
eaten only at mealtimes and should be followed immediately by proper brushing.
The school-aged child should visit the dentist at least twice a year for a cleaning
and application of fluoride.
Question format: Multiple Choice
Chapter 26: Growth and Development of the School-Aged Child: 6 to 10 Years
Cognitive Level: Apply
Client Needs: Health Promotion and Maintenance
Integrated Process: Teaching/Learning
Reference: p. 567-568

4. The school nurse is preparing to conduct routine health screenings of the


elementary school students. Which screening will the nurse prioritize for students
ages 6 to 8?
A. signs of scoliosis
B. vision and hearing
C. review immunization records
D. nutritional needs
Answer: B

Rationale: Vision and hearing screening are often conducted by the school nurse,
who then alerts the caregivers if there is a need for further evaluation from the
health care provider. Most states have immunization requirements that must be
met when the child enters school. Signs of scoliosis are usually evaluated at about
the age of 10 to 11 years. There are no specific tests that the school nurse would
use to evaluate nutrition; however, the nurse would be monitoring all students who
come to the nurse's office for potential nutritional situations as deemed
appropriate.
Question format: Multiple Choice
Chapter 26: Growth and Development of the School-Aged Child: 6 to 10 Years
Cognitive Level: Apply
Client Needs: Health Promotion and Maintenance
Integrated Process: Nursing Process
Reference: p. 567

5. The 11-year-old arrives in the emergency department presenting with the


following: nosebleed with no apparent trauma, disoriented, confused, difficulty
walking, nausea, and coughing. The nurse should question the child concerning
which potential activity?
A. falling on the playground at school
B. experimenting with an inhalant
C. eating a food he is allergic to
D. exposure to a respiratory disease

Answer: B

Rationale: Children may experiment with inhalants (substances whose volatile


vapors can be abused) because they are readily available and may seem no more
threatening than an innocent prank. Inhalants classified as deliriants contain
chemicals that give off fumes that can produce symptoms of confusion,
disorientation, excitement, and hallucinations. The fumes are mind-altering when
inhaled. The child initially may experience temporary intoxication, giddiness,
nausea, coughing, nosebleed, fatigue, lack of coordination, or loss of appetite.
These symptoms do not indicate the child fell on the playground, has eaten a food
containing an allergen, or that he has been exposed to a respiratory disease.
Question format: Multiple Choice
Chapter 26: Growth and Development of the School-Aged Child: 6 to 10 Years
Cognitive Level: Understand
Client Needs: Physiological Integrity: Reduction of Risk Potential
Integrated Process: Nursing Process
Reference: p. 569

6. The mother of a 6-year-old is asking the nurse how to handle the child's lying
and fabricated stories when confronted with questionable actions. Which response
would be most appropriate by the nurse?
A. "Your child could be in serious trouble in school if he continues to tell lies."
B. "The child should have privileges taken away for several days each time he tells
a lie."
C. "Is there any possibility he is telling the truth and you just don't know it is the
truth?"
D. "Children this age sometimes can't distinguish between fantasy and reality."

Answer: D

Rationale: Children in the age group 6 to 7 years often engage in magical thinking.
They may still believe in the tooth fairy, Santa Claus, monsters under the bed, and
other imaginary characters. These keen imaginations may also conjure up fears—
especially at night—about remote, fanciful, or imaginary events. If a child of this
age has trouble distinguishing fantasy from reality, it may incline them to lie to
escape punishment or to boost self-confidence. The other choices do not consider
this child's stage of development or give the mother the most appropriate
information for the situation.
Question format: Multiple Choice
Chapter 26: Growth and Development of the School-Aged Child: 6 to 10 Years
Cognitive Level: Apply
Client Needs: Health Promotion and Maintenance
Integrated Process: Communication and Documentation
Reference: p. 564-565

7. The nurse is assessing a 9-year-old girl during her well-child checkup and notes
the child weighs 86 lb (39 kg). After noting she weighed 9.5 lb (4.30 kg) at birth
and her growth has been within normal patterns, the nurse determines this girl is
within which category?
A. slightly underweight
B. slightly overweight
C. significantly overweight
D. significantly underweight

Answer: B

Rationale: By age 7, a child weighs about seven times as much as at birth, so at 7,


Jasmine should have weighed approximately 66.5 lb (30.16 kg). Because average
annual weight gain is about 5 to 6 lb (2.26 to 2.72 kg), by the time she is 9,
Jasmine should weigh approximately 76.5 to 80.5 lb (34.69 to 36.51 kg) . The
child's current weight puts her 5.5 to 9.5 lbs overweight (2.49 to 4.30 kg).
Question format: Multiple Choice
Chapter 26: Growth and Development of the School-Aged Child: 6 to 10 Years
Cognitive Level: Apply
Client Needs: Health Promotion and Maintenance
Integrated Process: Nursing Process
Reference: p. 564

8. The school nurse is meeting with a 10-year-boy who is concerned about his
weight. He reports he doesn't eat much candy but loves fruit, pasta, potatoes, and
bread. Which suggestion should the nurse prioritize to help him maintain a healthy
weight?
A. Encourage portion control at each meal
B. Change to a very low-fat and no-carbohydrate diet.
C. Encourage activities that will increase his physical activity.
D. Encourage the child to not worry about weight until he is older.

Answer: C

Rationale: Encouraging daily physical activity and following the dietary standards
(such as ChooseMyPlate guidelines) will help the child meet necessary nutritional
guidelines. Following popular fad diets or using weight-loss supplements must be
avoided because they do not supply adequate nutrients for the growing child. The
child is aware of the weight problem, but it would not be beneficial to just ignore it
because the child may develop harmful eating habits such as bingeing.
Question format: Multiple Choice
Chapter 26: Growth and Development of the School-Aged Child: 6 to 10 Years
Cognitive Level: Apply
Client Needs: Health Promotion and Maintenance
Integrated Process: Nursing Process
Reference: p. 566

9. The nurse is addressing a caregiver's concerns regarding adequate sleep for an


11-year-old child who gets up at 6:30 a.m. each morning. The nurse should point
out which time as the most appropriate bedtime for this child?
A. 7:30 p.m.
B. 8:00 p.m.
C. 9:00 p.m.
D. 10:00 p.m.

Answer: C

Rationale: Exercise and sufficient rest are important for school-age children. The
school-aged child needs 10 to 12 hours of sleep per night. The 6-year-old needs 12
hours of sleep wherease an 11-year-old child needs closer to 10 hours of sleep per
night, which would mean going to bed at 8:30 to 9:00 p.m. The older adolescent
could go to bed slightly later and get adequate sleep.
Question format: Multiple Choice
Chapter 26: Growth and Development of the School-Aged Child: 6 to 10 Years
Cognitive Level: Apply
Client Needs: Health Promotion and Maintenance
Integrated Process: Teaching/Learning
Reference: p. 567-568

10. The nurse is admitting a 10-year-old for surgery. What action should the nurse
prioritize when caring for this child?
A. Offer to help with bathing.
B. Answer questions regarding pain.
C. Encourage family caregivers to stay with the child.
D. Avoid prolonged discussions about the child's anxiety.

Answer: B

Rationale: School-age children need privacy more than younger children do and
may not want to have physical contact with adults; this wish should be respected.
These attitudes should be recognized and handled in a way that ensures as much
privacy as possible. Children's questions, including those about pain, should be
answered truthfully. An opportunity to verbalize anxieties will help a child deal with
them. Family caregivers may feel guilty about the child's need for hospitalization
and, as a result, may overindulge the child. The child may regress in response to
this, but this regression should not be encouraged.
Question format: Multiple Choice
Chapter 26: Growth and Development of the School-Aged Child: 6 to 10 Years
Cognitive Level: Apply
Client Needs: Psychosocial Integrity
Integrated Process: Nursing Process
Reference: p. 570-571

11. The nurse is preparing a care plan for a school-age child to address the child's
hospitalization. Which factor should the nurse incorporate into this plan?
A. increased attention span and interest in an activity for a long length of time
B. can take in new information at a rapid rate and asks "why" and "how" a lot
C. insists he or she can "do it" and then reverts to being dependent
D. Grows and develops skills more rapidly than at any other time in his or her life

Answer: A

Rationale: The school-aged child has a longer attention span and can become
absorbed in a craft or activity for several hours. The preschool-aged child soaks in
information and asks "why and "how" over and over. The toddler insists he or she
can do things one minute and then becomes dependent the next minute. The infant
grows and develops skills more rapidly than he or she ever will again.
Question format: Multiple Choice
Chapter 26: Growth and Development of the School-Aged Child: 6 to 10 Years
Cognitive Level: Apply
Client Needs: Psychosocial Integrity
Integrated Process: Nursing Process
Reference: p. 562

12. The nurse is monitoring a playgroup of children on the pediatric unit. The nurse
predicts the school-age child will most likely be participating in which activity?
A. pretending to be mommies and daddies in the playhouse
B. playing with the plastic vacuum cleaner, pushing it around the room
C. painting pictures in the art corner of the room
D. watching a movie with other children their age
Answer: C

Rationale: The school-aged child enjoys group activities and making things, such as
drawings, paintings, and craft projects. Preschool children enjoy imitative play,
pretending to be the mommy, the daddy, a police officer, a cowboy, or other
familiar characters. Playtime for the toddler involves imitation of the people around
them, such as adults, siblings, and other children. Push-pull toys allow them to use
their developing gross motor skills. The adolescent enjoys activities he or she can
participate in with peers.
Question format: Multiple Choice
Chapter 26: Growth and Development of the School-Aged Child: 6 to 10 Years
Cognitive Level: Apply
Client Needs: Health Promotion and Maintenance
Integrated Process: Nursing Process
Reference: p. 565

13. A 7-year-old seen in the clinic for a routine well-child visit is noted on
assessment to have gained 5 lb (2.26 kg) and grown 5 in (12.70 cm) over the past
year. The nurse determines this child is within which parameters?
A. Weight and height are within expected patterns of growth.
B. Weight is within expected range and height is less than expected.
C. Weight is above expected range and the height is within an expected range.
D. Weight is within expected range and height is above expected range.

Answer: D

Rationale: Between the ages of 6 and 10 years, average annual weight gain is
about 5 to 6 lb (2 to 3 kg). By age 7, the child weighs about seven times as much
as at birth. Annual height increase is about 2.5 inches (6 cm). For this child, the
weight increase of 5 lbs (2.2 kg) is within expected range and the increase of 5 in.
(12.5 cm) is double what would be expected for this age.
Question format: Multiple Choice
Chapter 26: Growth and Development of the School-Aged Child: 6 to 10 Years
Cognitive Level: Apply
Client Needs: Health Promotion and Maintenance
Integrated Process: Nursing Process
Reference: p. 562

14. The nurse is preparing an in-service training session on Erikson's theories for
the pediatric nurses. Which task should the nurse prioritize as an example for the
school-aged child?
A. insists on doing things their way instead of their caregivers
B. will work on a picture until it is finished
C. wants to tie their own shoes no matter how long it takes
D. understands the need for discipline

Answer: B
Rationale: The developmental task for the school-aged child is to develop a sense
of industry, and completing activities builds that feeling of confidence. This can
include working with arts and crafts. The adolescent developing his or her own
personality is trying to break from their caregivers' methods. Erikson's psychosocial
developmental task for toddlers is to achieve autonomy (independence) and do
things on their own. This can include tasks such as tying shoelaces—no matter how
long it may take. Understanding the need for discipline is not one of Erikson's
theories.
Question format: Multiple Choice
Chapter 26: Growth and Development of the School-Aged Child: 6 to 10 Years
Cognitive Level: Apply
Client Needs: Health Promotion and Maintenance
Integrated Process: Nursing Process
Reference: p. 572

15. The school nurse is assessing a 6-year-old child's dentition. Which assessment
should the nurse prioritize for this child?
A. has first permanent molars
B. has at least six permanent teeth
C. has two cuspid or canine teeth
D. has lost all deciduous teeth

Answer: A

Rationale: At about age 6, the child starts to lose the deciduous teeth, usually
beginning with the lower incisors. They will continue to lose the deciduous teeth
while they are school-aged. The first permanent molars usually erupt about the age
of 6. The child would not have six permanent teeth until between 7 and 8 years of
age. The upper two cuspid or canine teeth erupt about 9 or 10 years or age. The
lower cuspid teeth erupt about 11 or 12 years of age.
Question format: Multiple Choice
Chapter 26: Growth and Development of the School-Aged Child: 6 to 10 Years
Cognitive Level: Apply
Client Needs: Health Promotion and Maintenance
Integrated Process: Nursing Process
Reference: p. 562

16. The nursing instructor is conducting a class discussion exploring the normal
dentition progression of the school-aged child. The instructor determines the
session is successful when the students correctly choose which factor as most likely
occurring in 10-year-olds?
A. All four central incisors have erupted.
B. The child has only six permanent teeth.
C. Two of the cuspid teeth have erupted.
D. There are eight molars.

Answer: C
Rationale: The upper two cuspid or canine teeth erupt about 9 or 10 years or age.
The lower cuspid teeth erupt about 11 or 12 years of age. The central incisors erupt
between 6 and 8 years of age. The child will have approximately 16 permanent
teeth by the age of 10. At 10 years of age, the child will have four molars.
Question format: Multiple Choice
Chapter 26: Growth and Development of the School-Aged Child: 6 to 10 Years
Cognitive Level: Apply
Client Needs: Health Promotion and Maintenance
Integrated Process: Nursing Process
Reference: p. 562

17. A 6-year-old male has come to the clinic for a routine well-child visit. The
nurse, after noting in the medical record that this child has followed basic growth
and development standards, anticipates documenting which common assessment
finding?
A. flat abdomen
B. swayed back
C. long legs
D. slender body

Answer: B

Rationale: The 6-year-old's silhouette is characterized by a flatter but still


protruding abdomen and lordosis (swayback). By the time the child has reached the
age of 10 years, the spine is straighter, the abdomen flatter, and the body
generally more slender and long-legged.
Question format: Multiple Choice
Chapter 26: Growth and Development of the School-Aged Child: 6 to 10 Years
Cognitive Level: Apply
Client Needs: Health Promotion and Maintenance
Integrated Process: Nursing Process
Reference: p. 563

18. The nurse is working with a group of 8-year-olds who are learning about the
concept of conservation of numbers. Which activity will help teach this concept to
these school-aged children?
A. Stacking blocks in a tower and counting how many can be stacked without falling
B. Rearranging a group of coins first into a circle, then a triangle and then a square
C. Forming vases from blocks of clay that are of various weights
D. Measuring the weight of a handful of popcorn and then the weight of a handful of
pennies

Answer: B

Rationale: In understanding the concept of conservation of numbers, the child


understands that the number of objects does not change even though they may be
rearranged. Conservation of weight can be accomplished by weighing different
objects. Conversation of mass is demonstrated by forming vases out of clay.
Question format: Multiple Choice
Chapter 26: Growth and Development of the School-Aged Child: 6 to 10 Years
Cognitive Level: Apply
Client Needs: Health Promotion and Maintenance
Integrated Process: Nursing Process
Reference: p. 563

19. The nursing students working with a group of 8-year-olds have determined they
will illustrate the concept of conservation of weight. Which activity would best help
these children grasp this concept?
A. weighing and measuring each child and comparing the number of pounds to the
number of inches for each person
B. using a balance scale to weigh a variety of objects and counting how many
pennies must be used to balance the scales
C. weighing a pound of oranges and a pound of bricks and counting how many of
each make a pound
D. measuring the weight of a handful of popcorn and then the weight of a handful
of pennies

Answer: C

Rationale: In learning about conservation of weight, the child recognizes that a


pound is a pound, regardless of whether oranges or bricks are weighed. Comparing
pounds and inches, noting the number of pennies needed to balance the weight of
objects, and measuring the weight of handfuls of popcorn and pennies does not
illustrate conservation of weight.
Question format: Multiple Choice
Chapter 26: Growth and Development of the School-Aged Child: 6 to 10 Years
Cognitive Level: Apply
Client Needs: Health Promotion and Maintenance
Integrated Process: Nursing Process
Reference: p. 563

20. The nurse is preparing a presentation for a community health fair on the topic
of helping school-aged children develop the self-confidence they will need to mature
into responsible adults. Which suggestion should the nurse prioritize to caregivers
to enable them to help their children in this area?
A. Encourage the child to cheer on their friends at games.
B. Caregivers should maintain consistent rules and expectations.
C. Ensure that the child has the best teacher possible.
D. Encourage older siblings to include the child in their activities.

Answer: B

Rationale: The school-age child needs consistent rules, positive attention, and clear
expectations in order to develop self-confidence. Although peers are important,
caregivers have more influence over the school-age child relating to building
confidence. Reactions from teachers and trying to keep up with older siblings may
lead to lack of self-confidence.
Question format: Multiple Choice
Chapter 26: Growth and Development of the School-Aged Child: 6 to 10 Years
Cognitive Level: Apply
Client Needs: Health Promotion and Maintenance
Integrated Process: Teaching/Learning
Reference: p. 564

21. The nurse notes a school-aged child in the pediatric unit playroom is busy
sorting through the Legos and putting them in certain groupings. The nurse
determines this child is illustrating which developmental concept?
A. conservation of numbers
B. decentration
C. reversibility
D. hierarchical arrangement

Answer: D

Rationale: The ability to group objects into a grouping by some common system is
known as hierarchical arrangement. Conservation of numbers is when the child
understands that the number of cookies does not change even though they may be
rearranged. Decentration is being able to see several aspects of a problem at the
same time and to understand the relation of various parts to the whole situation.
Reversibility is the ability to think in either direction.
Question format: Multiple Choice
Chapter 26: Growth and Development of the School-Aged Child: 6 to 10 Years
Cognitive Level: Apply
Client Needs: Health Promotion and Maintenance
Integrated Process: Nursing Process
Reference: p. 565-566

22. The caregiver of a 6-year-old is concerned the child is not getting proper
nutrition because on some days, the child will only eat one type of food. Which
response should the nurse prioritize for this caregiver?
A. "It is important that each time she eats, she has a variety of foods."
B. "Food jags are common in this age. This probably won't last long."
C. "Try having her eat with the family and she will want what others are eating."
D. "She should be discouraged from having food likes or dislikes. Have her eat
everything."

Answer: B

Rationale: Food "jags" (the desire for only one kind of food for a while) are typical
of the 6-year-old child. This stage soon passes and is unimportant if the child
generally gets the necessary nutrients. A variety of foods is important but not each
time the child eats. Eating with the family will encourage her to eat what others are
eating, but it is not the most appropriate answer. Allowing the child to express food
dislikes and permitting refusal of a disliked food item is usually the best way to
handle the school-aged child.
Question format: Multiple Choice
Chapter 26: Growth and Development of the School-Aged Child: 6 to 10 Years
Cognitive Level: Apply
Client Needs: Health Promotion and Maintenance
Integrated Process: Nursing Process
Reference: p. 566-567

23. The nurse is presenting information about school-aged children at a community


event. Which statement from the group should the nurse prioritize to address with
further teaching and more information?
A. "Sometimes we have to be firm, but our children wash their hands before
eating."
B. "I make sure they have good teeth by giving them calcium and phosphorus."
C. "Food is so expensive, we always make our children eat everything on their
plates."
D. "Even if the weather is cool and cloudy, our children play outside every day."

Answer: C

Rationale: Obesity can be an issue in the school-aged child, especially if they are
urged to clean their plates even if they have more food than they want or need on
the plate. The parents should be encouraged to use smaller plates for the children
so they will still appear to have a full plate but smaller portions. Firm guidance and
direction is important with the school-aged child. Calcium and phosphorus are
important to healthy teeth. Exercise each day is important, especially outdoor
exercise.
Question format: Multiple Choice
Chapter 26: Growth and Development of the School-Aged Child: 6 to 10 Years
Cognitive Level: Analyze
Client Needs: Health Promotion and Maintenance
Integrated Process: Nursing Process
Reference: p. 566-567

24. The school nurse is discussing obesity with a group of caregivers of school-aged
children. Which statement by the caregivers best illustrates that they are prepared
to help their child prevent obesity?
A. "I always cook foods that are low in fat."
B. "We eat fast foods only on weekends because we are too busy to cook."
C. "Neither my husband nor I have ever had any concerns with weight."
D. "I keep lots of snacks on hand because my child eats all day long."

Answer: A

Rationale: Limiting dietary fat intake to no more than 35 percent of total calories
helps control the child's weight. Fast foods tend to have high fat and calorie content
and contribute to obesity. Some children may have a genetic tendency to obesity,
but environment and a sedentary lifestyle are a concern related to obesity.
Caregivers must carefully supervise children's snacking habits to be sure that
snacks are nutritious and not too frequent since this can lead to obesity.
Question format: Multiple Choice
Chapter 26: Growth and Development of the School-Aged Child: 6 to 10 Years
Cognitive Level: Apply
Client Needs: Health Promotion and Maintenance
Integrated Process: Teaching/Learning
Reference: p. 566-567

25. The nursing instructor is leading a discussion on school-aged children. The


instructor determines the session is successful when the students correctly choose
which factor as being a priority for the school-aged child?
A. Needs 10 to 12 hours of sleep per night
B. Should brush their teeth at bedtime
C. Have a routine physical exam every 6 months
D. Be screened for scoliosis once a year

Answer: A

Rationale: The school-aged child needs 10 to 12 hours of sleep per night. They
need to brush their teeth after every meal and at bedtime. A routine physical exam
once a year is all that is necessary. Children are screened around the age of 10 or
11 for scoliosis.
Question format: Multiple Choice
Chapter 26: Growth and Development of the School-Aged Child: 6 to 10 Years
Cognitive Level: Apply
Client Needs: Health Promotion and Maintenance
Integrated Process: Teaching/Learning
Reference: p. 567
Chapter 27
1. The nurse is measuring the height of a 13-year-old girl who was 56 in (142.24
cm) tall at age 9. The nurse predicts the girl will be approximately how tall if
following the normal pattern of growth?
A. 58 in (147.32 cm)
B. 62 in (157.48 cm)
C. 68 in (172.72 cm)
D. 76 in (193.04 cm)

Answer: C

Rationale: Preadolescence begins in the female between the ages of 9 and 11 years
and is marked by a growth spurt that lasts for about 18 months. Girls grow about 3
inches each year until menarche, after which growth slows considerably. The four
years between age 9 and 13 would mean the girl should have grown approximately
12 inches, making her now 68 in (172.72 cm) tall (56 + 12 = 68 in).
Question format: Multiple Choice
Chapter 27: Growth and Development of the Adolescent: 11 to 18 Years
Cognitive Level: Analyze
Client Needs: Health Promotion and Maintenance
Integrated Process: Nursing Process
Reference: p. 575

2. The parents of a 16-year-old male are worried about recent changes in his
behavior, ignoring his schoolwork and sports, and spending almost all of his free
time interacting with his girlfriend. Which suggestion should the nurse point out
would best address this situation?
A. He is not developmentally mature enough to make healthy choices about the
ways in which he spends his time, so it would be helpful if they would make a
schedule for him that includes about a half-hour per day to talk with his girlfriend.
B. He has developed his own identity by now; being able to establish close
relationships with girls is important preparation for all of his adult relationships.
They should honor his need to be with, or talk to, his girlfriend as long as he has
completed his schoolwork for the day.
C. He is not developmentally mature enough to have an intimate relationship with
one girl; they should encourage him to spend time with groups of friends rather
than time alone with his girlfriend.
D. He has chosen a girl who is overly dependent on him. They should talk to him
about making sure he meets his own needs, including doing the schoolwork he
enjoys, in any relationship.

Answer: B

Rationale: When identity has been established, generally between the ages of 16
and 18 years, adolescents seek intimate relationships, usually with members of the
opposite sex. Intimacy, which is mutual sharing of one's deepest feelings with
another person, is impossible unless both persons have established a sense of trust
and a sense of identity. Intimate relationships are a preparation for long-term
relationships, and people who fail to achieve intimacy may develop feelings of
isolation and experience chronic difficulty in communicating with others.
Question format: Multiple Choice
Chapter 27: Growth and Development of the Adolescent: 11 to 18 Years
Cognitive Level: Apply
Client Needs: Psychosocial Integrity
Integrated Process: Nursing Process
Reference: p. 577-578

3. The father of a 15-year-old daughter is concerned she is not getting adequate


nutrition to play high school basketball. Her games are on Friday nights. Which
suggestion should the nurse point out will best suit the needs of this adolescent?
A. Pasta with a small amount of meat sauce and two slices of bread for dinner on
Wednesday and Thursday evenings and again at 2 p.m. on Friday.
B. Boiled eggs with bacon or ham and a glass of orange juice for breakfast on
Thursday and Friday mornings along with some sliced turkey and a salad at noon
on Friday.
C. Three daily meals that include choices from each of the food groups with an
additional serving of fruit and several extra glasses of water on Friday.
D. Three daily meals that include choices from each of the food groups; Friday's
lunch eaten around 2 p.m. with a small amount of fat and a somewhat larger than
usual portion of complex carbohydrates.

Answer: D

Rationale: A meal that is low in fat and high in complex carbohydrates, eaten 3 to 4
hours before an event, is appropriate for the teen athlete. Carbohydrate-loading,
which some practice during the week before an athletic event, increases the muscle
glycogen level to 2 to 3 times normal and may hinder heart function. The other
suggested menus would not provide the additional muscle glycogen needed for
optimal functioning.
Question format: Multiple Choice
Chapter 27: Growth and Development of the Adolescent: 11 to 18 Years
Cognitive Level: Apply
Client Needs: Health Promotion and Maintenance
Integrated Process: Nursing Process
Reference: p. 579-580

4. The parent of an adolescent has called to talk to the pediatric nurse over concern
that her daughter is not getting proper nutrition now that she has started following
a vegetarian diet. Which response should the nurse prioritize for this parent?
A. Vegetarian diets can be healthy; ensure she includes whole-grain products,
legumes, nuts, seeds, and soy dairy substitutes.
B. Lacto-ovo vegetarian diets are acceptable for teens but, until the body's growth
cycle is complete, she needs some egg and dairy products for good health.
C. Vegetarian diets can be healthy for teens but should be supplemented with
protein drinks and megavitamins.
D. A vegetarian diet is unhealthy for growing bodies, and teens are too immature to
make a healthy decision.

Answer: A

Rationale: If planned with care, vegetarian diets can provide all needed nutrients.
All vegetarians should include whole-grain products, legumes, nuts, seeds, and
fortified soy substitutes. If the individual is following the lactovegetarian approach,
then low-fat dairy products are also appropriate. The lacto-ovo vegetarian diet
includes eggs and dairy products but excludes meat, poultry, and fish. The
vegetarian diet does not need to be supplemented with protein drinks and
megavitamins if the food choices are carefully chosen to provide adequate nutrition.
Question format: Multiple Choice
Chapter 27: Growth and Development of the Adolescent: 11 to 18 Years
Cognitive Level: Apply
Client Needs: Health Promotion and Maintenance
Integrated Process: Nursing Process
Reference: p. 579-580

5. The nurse is assessing an adolescent in the clinic at a routine well-visit. Which


question should the nurse prioritize to encourage the adolescent to share detailed
information?
A. "Do you like members of the opposite sex?"
B. "Do you go out on dates frequently?"
C. "Are you sexually active?"
D. "Do you have oral, anal, or vaginal sex?"

Answer: D

Rationale: The nurse should ask open-ended questions which will encourage
continued conversation versus closed-ended questions which can be answered with
just a simple "yes" or "no". It's important for the nurse to encourage the adolescent
to give direct answers. The interviewer must be alert to verbal and nonverbal clues.
Being specific about the type of sex, versus just sex, will alert the nurse to closely
evaluate specific areas of the body which might have signs of STIs.
Question format: Multiple Choice
Chapter 27: Growth and Development of the Adolescent: 11 to 18 Years
Cognitive Level: Apply
Client Needs: Health Promotion and Maintenance
Integrated Process: Nursing Process
Reference: p. 580

6. The nurse is assessing a teenage client and notes his lower front teeth are
slightly crossed over. The nurse points out to his caregiver that he should see an
orthodontist about this to prevent which potential situation?
A. Leaving even slightly crooked teeth will lead to more cavities later in life.
B. Even slight malocclusions make chewing and jaw function less efficient.
C. This could have a lasting effect on his future to include everything from dating to
hiring.
D. Uncorrected malocclusions lead to infection and ultimately tooth loss.

Answer: B

Rationale: Dental malocclusion (improper alignment of the teeth) is a common


condition that affects the way the teeth and jaws function. Correction of the
malocclusion with dental braces improves chewing ability and appearance. Crooked
teeth do not lead to more cavities, nor do they lead to infection and tooth loss.
While appearance and acceptance in society is important to the adolescent, that is
not the most important reason for orthodontic care for the adolescent.
Question format: Multiple Choice
Chapter 27: Growth and Development of the Adolescent: 11 to 18 Years
Cognitive Level: Apply
Client Needs: Health Promotion and Maintenance
Integrated Process: Nursing Process
Reference: p. 581-582

7. A 15-year-old client tells the nurse he has been having wet dreams and is
ashamed and afraid he will get into trouble because he believes his parents think he
is too young to understand or know about sex. To which statement would be the
most appropriate for the nurse to respond?
A. "Since your parents feel you are not ready to talk about sex, you need to honor
their wishes and not bother them with this."
B. "Don't worry, it is perfectly normal for 15-year-olds to have the kinds of
fantasies that lead to wet dreams. You should tell your parents about this."
C. "Wet dreams are not the result of anything you are doing but are simply the
body's way of ridding itself of excess semen."
D. "You don't need to feel ashamed. I will speak to your parents about the
importance of allowing you to experiment with and feel good about your sexuality."

Answer: C

Rationale: Boys who are unprepared for nocturnal emissions may feel guilty,
believing that they have caused these "wet dreams" by sexual fantasies or
masturbation. They need to understand that this is a normal occurrence and is
simply the body's method of getting rid of surplus semen. The other suggestions do
not address the situation in a professional manner.
Question format: Multiple Choice
Chapter 27: Growth and Development of the Adolescent: 11 to 18 Years
Cognitive Level: Apply
Client Needs: Health Promotion and Maintenance
Integrated Process: Caring
Reference: p. 582
8. The school nurse is conducting a health class with a group of high school
students on the topic of sex and sexuality issues. The nurse determines the session
is successful when the students correctly choose which aspect as most important?
A. "It is important for adolescents to abstain from sexual activity."
B. "Males, as well as their partners, are responsible for avoiding an unwanted
pregnancy."
C. "Adolescents need to know how to use condoms correctly to avoid sexually
transmitted infections."
D. "Girls should carry their own condoms and use them to avoid pregnancy."

Answer: C

Rationale: The priority is to prevent STIs. Adolescents need to understand that STIs
can be acquired via oral sex as well as vaginal sex. They also need to understand
that using contraceptive pills will not prevent STIs. They need contraceptive
information to prevent pregnancy but more importantly, they need straightforward
information about using condoms to protect them against HIV and other infections.
Both male and female adolescents need this information, and girls should be
advised to carry their own condoms if they believe there is any possibility of having
sexual intercourse.
Question format: Multiple Choice
Chapter 27: Growth and Development of the Adolescent: 11 to 18 Years
Cognitive Level: Analyze
Client Needs: Health Promotion and Maintenance
Integrated Process: Teaching/Learning
Reference: p. 585

9. The nurse is preparing to participate in a community discussion on the needs of


the adolescents in the local school. The nurse should point out which goal is the
primary concern for these young individuals as the committee makes plans?
A. Teens are busy developing their own personal identity.
B. They want to successfully complete activities.
C. Each child is learning to do things on his or her own.
D. They understand and respond to discipline.

Answer: A

Rationale: According to Erikson, the central task of adolescence is to develop


unique personality and identity. The developmental task for the school-age child is
to develop a sense of industry, and completing activities builds that feeling of
confidence. Erikson's psychosocial developmental task for toddlers is to achieve
autonomy (independence) and do things on their own. Learning to speak and to
understand and respond to discipline are not developmental tasks, according to
Erikson.
Question format: Multiple Choice
Chapter 27: Growth and Development of the Adolescent: 11 to 18 Years
Cognitive Level: Apply
Client Needs: Health Promotion and Maintenance
Integrated Process: Teaching/Learning
Reference: p. 575

10. The nursing instructor is leading a class discussion on the various aspects of
adolescents. The instructor determines the class is successful after the students
correctly choose which milestone as the beginning of adolescence?
A. The deciduous teeth falling out
B. The beginning of puberty
C. The child becoming self-supported
D. The fine motor skills fully developed

Answer: B

Rationale: Early adolescence (preadolescence, pubescence) begins with a dramatic


growth spurt that signals the advent of puberty or reproductive maturity. The
deciduous teeth are all out around the age of 10 or 11. Children usually are not
self-supported. Fine motor skills continue to develop through adolescence.
Question format: Multiple Choice
Chapter 27: Growth and Development of the Adolescent: 11 to 18 Years
Cognitive Level: Analyze
Client Needs: Health Promotion and Maintenance
Integrated Process: Teaching/Learning
Reference: p. 582

11. The nurse is assessing a 16-year-old girl at an annual well-clinic visit and notes
the girl started menses at 13 years of age and grew 1 inch (2.5 cm) over the past
year. When questioned by the young lady if this is normal, which answer should the
nurse prioritize?
A. "Your height is less than expected."
B. "You're taller than what would be expected."
C. "Your weight is more revealing than your height."
D. "You're following expected patterns of growth."

Answer: D

Rationale: Girls grow about 3 inches (7.6 cm) each year until menarche, after which
growth slows considerably. This height increase would be within expected patterns
of growth. Height is not dependent on weight.
Question format: Multiple Choice
Chapter 27: Growth and Development of the Adolescent: 11 to 18 Years
Cognitive Level: Apply
Client Needs: Health Promotion and Maintenance
Integrated Process: Nursing Process
Reference: p. 575

12. The school nurse is meeting with a group of 11-year-old girls to discuss
expected puberty changes in their bodies. When one of the girls states, "I just feel
like my whole body is changing and I don't know why" what should the nurse point
out to this group?
A. "You will feel better about yourself as you get older."
B. "You have lots of hormone changes going on right now."
C. "You may feel like you are changing, but you still look the same."
D. "Your other friends are feeling like this too."

Answer: B

Rationale: Preadolescents need information about their changing bodies and


feelings. Sex education that includes information about the hormonal changes that
are occurring or will occur is necessary to help them through this developmental
stage. Although adjustments will occur as the child gets older and friends are
important, these responses are not the most direct and appropriate.
Question format: Multiple Choice
Chapter 27: Growth and Development of the Adolescent: 11 to 18 Years
Cognitive Level: Apply
Client Needs: Health Promotion and Maintenance
Integrated Process: Communication and Documentation
Reference: p. 575

13. A male nurse is meeting with a group of 12-year-old boys to discuss expected
bodily changes. After one of the boy's says, "My older brother told me my bed
might be wet and that means I had a wet dream. Is that true?" What is the best
response from the nurse?
A. "When you are thinking about people you find sexually attractive or dating those
people you might have a wet dream."
B. "It is not common to wet the bed or urinate when you have a wet dream."
C. "Having wet dreams indicates that your body is going through a process of
maturing."
D. "It will be several years before you will start having wet dreams."

Answer: C

Rationale: In boys, the appearance of nocturnal emissions ("wet dreams") is often


used as the indication that the preadolescent period has ended and that the
adolescent is maturing into an adult. Nocturnal emissions usually occur at about the
age of 11 in boys. Wet dreams are not associated with urination. Boys at this age
start thinking of relationships with people they find sexually attractive, but this is
not the best response.
Question format: Multiple Choice
Chapter 27: Growth and Development of the Adolescent: 11 to 18 Years
Cognitive Level: Apply
Client Needs: Health Promotion and Maintenance
Integrated Process: Teaching/Learning
Reference: p. 575-576
14. The nurse is preparing a presentation for a local health fair depicting the
differences in maturity between preadolescents. Which differing factor should the
nurse prioritize in the presentation?
A. Boys grow at a slower, steadier rate than do girls.
B. Girls grow at a slower, steadier rate than do boys.
C. Boys grow at a rapid, sporadic rate.
D. Boys and girls grow at the same rate.

Answer: A

Rationale: Preadolescent boys grow generally at a slower, steadier rate than do


girls. Girls grow more rapidly during preadolescence and then their growth rate
slows dramatically after menarche.
Question format: Multiple Choice
Chapter 27: Growth and Development of the Adolescent: 11 to 18 Years
Cognitive Level: Apply
Client Needs: Health Promotion and Maintenance
Integrated Process: Teaching/Learning
Reference: p. 575

15. The nurse is meeting with a group of caregivers of adolescents and discussing
sex and sexuality, including how to discuss these issues with their children. Which
comment should the nurse prioritize with this group of caregivers?
A. Teenagers spend so much time with their peers, and that is usually how they
find out about sex.
B. Being honest and straightforward with teenagers will encourage them to ask
about subjects like sexuality.
C. Most schools have excellent programs to teach adolescents about sex and
sexuality.
D. Internet resources, movies, and television have the most accurate and current
information for your adolescent to learn about sexuality issues.

Answer: B

Rationale: The most important aspect of discussions about sexuality with


adolescents is giving honest, straightforward answers in an atmosphere of caring
concern. Children whose need for information is not met through family, school, or
community programs will get the information—often inaccurately—from peers,
movies, television, or other media.
Question format: Multiple Choice
Chapter 27: Growth and Development of the Adolescent: 11 to 18 Years
Cognitive Level: Apply
Client Needs: Health Promotion and Maintenance
Integrated Process: Nursing Process
Reference: p. 576

16. A 17-year-old female is meeting with the nurse for an annual well-visit and is
asking the nurse questions about how to know when one is in love. The nurse
should point out which factor to help decide if both individuals have reached a
mutual agreement and are ready for an intimate relationship?
A. A sense of trust and identity
B. An ability to be autonomous
C. A willingness to take initiative
D. An understanding of socialization and of isolation

Answer: A

Rationale: In order to be intimate or to share one's deepest feelings with another


person, it is impossible unless both persons have established a sense of trust and a
sense of identity. Being autonomous or taking initiative are not aspects that lead
toward intimate relationships. Socialization and isolation are not relevant to the
establishment of intimate relationships.
Question format: Multiple Choice
Chapter 27: Growth and Development of the Adolescent: 11 to 18 Years
Cognitive Level: Apply
Client Needs: Health Promotion and Maintenance
Integrated Process: Nursing Process
Reference: p. 578

17. The nurse is meeting with a group of caregivers of adolescents. Which example
should the nurse point out is most effective for the caregiver to support the
adolescent?
A. Leave pamphlets about topics such as drugs and alcohol in their room so they
can read them.
B. Let them choose their hairstyle, even though it may not look the best for them.
C. Discourage spending too much time with school friends since we know they can
be a negative influence.
D. Our house rules are stricter than their friends but everyone follows the same
rules in our home.

Answer: B

Rationale: The adolescent whose family caregivers make it difficult to conform are
adding another stress to an already emotion-laden period. By allowing the
adolescent to follow trends and fads in clothing choices, hairstyles, and music, the
caregiver decreases the stress for the child. Information about drugs and alcohol is
important to share, but these topics would be better discussed with the child. It is
important the adolescent spend time with peers.
Question format: Multiple Choice
Chapter 27: Growth and Development of the Adolescent: 11 to 18 Years
Cognitive Level: Apply
Client Needs: Health Promotion and Maintenance
Integrated Process: Teaching/Learning
Reference: p. 577
18. The school nurse is preparing for a career fair at the local high school. Which
factor should the nurse point out when presenting various medical career options to
the students?
A. The adolescent should seriously consider the parents' choice for the best career.
B. The career choices may be limited after high school depending on location.
C. Many individuals have already chosen what they want to do.
D. The individual's sex no longer decides which career options are available.

Answer: D

Rationale: Sex role stereotypes have been shattered in most careers and
professions. More women are becoming lawyers, physicians, plumbers, and
carpenters; more men are entering nursing or choosing to become stay-at-home
fathers while their wives earn the primary family income. The adolescent should
consider what the parents are saying, but the final choice is theirs. With the use of
the Internet, travel to other areas, and a variety of school options, there are ways
for the students to go to a school in a different area. There are also numerous
resources that will help the student seek higher education. It just may mean they
have to work a little harder to achieve their goal. There are many career choices for
the adolescent to choose from.
Question format: Multiple Choice
Chapter 27: Growth and Development of the Adolescent: 11 to 18 Years
Cognitive Level: Apply
Client Needs: Health Promotion and Maintenance
Integrated Process: Caring
Reference: p. 577

19. The nurse is leading a discussion in a health class at the high school on
adolescent relationships. The nurse determines the session is successful when the
students correctly choose which type of relationship as involving a member of the
opposite sex?
A. gay or lesbian relationship
B. unhealthy relationship
C. heterosexual (straight) relationship
D. bisexual relationship

Answer: C

Rationale: Heterosexual (straight) relationships are formed between members of


the opposite sex. Relationships formed with members of the same sex are gay or
lesbian relationships. Bisexual relationships are those in which the individual has
both same-sex and opposite-sex relationships. Any relationship can be an
unhealthy relationship.
Question format: Multiple Choice
Chapter 27: Growth and Development of the Adolescent: 11 to 18 Years
Cognitive Level: Analyze
Client Needs: Health Promotion and Maintenance
Integrated Process: Caring
Reference: p. 578

20. The nurse is meeting with a group of adolescent athletes to discuss their
nutritional needs. The nurse should encourage the adolescents to include which
foods in the diet to increase iron intake? Select all that apply.
A. dried fruits
B. peanuts
C. milk shakes
D. grapes
E. hard-boiled eggs
F. cheese sticks

Answer: A, B, E

Rationale: Dried fruits, peanuts, and eggs (especially the egg yolk) are high in iron.
Milk shakes, grapes, and cheese sticks are low or have little iron content.
Question format: Multiple Select
Chapter 27: Growth and Development of the Adolescent: 11 to 18 Years
Cognitive Level: Apply
Client Needs: Health Promotion and Maintenance
Integrated Process: Teaching/Learning
Reference: p. 580

21. The school nurse is teaching a health class on nutrition with some adolescents.
Which comment by a student should the nurse prioritize and provide more
teaching?
A. "I hate eating with my family. I would much rather snack all day."
B. "I had lunch with my boyfriend before his game and all he ate were
carbohydrates."
C. "I am feeling so fat. I think I need to exercise a few extra hours today."
D. "I don't really like it, but I have been drinking lots of skim milk."

Answer: C

Rationale: Even after the body has become emaciated, the adolescent may believe
that he or she is still fat and, therefore, unattractive. This condition is called
anorexia nervosa and is a serious health concern related to adolescents and their
nutritional status. Even if the adolescent doesn't eat with the family, he or she can
still consume needed nutrients. Although carbohydrate-loading is a concern, this
statement does not indicate the adolescent girl was following this eating pattern.
Skim milk has adequate nutrients.
Question format: Multiple Choice
Chapter 27: Growth and Development of the Adolescent: 11 to 18 Years
Cognitive Level: Apply
Client Needs: Health Promotion and Maintenance
Integrated Process: Teaching/Learning
Reference: p. 579-580
22. The nurse is teaching a nutritional class to a group of adolescents who are
lactose-intolerant. Which food should the nurse point out will help these
adolescents consume the calcium they need for proper nutrition?
A. chocolate pudding
B. nonfat milk
C. soybean curd
D. cottage cheese

Answer: C

Rationale: For a lactose-intolerant client, the nurse can recommend other products
high in calcium, such as tofu (soybean curd), soybeans, and greens, to increase
calcium intake. Chocolate pudding, nonfat milk, and cottage cheese are all high in
lactose.
Question format: Multiple Choice
Chapter 27: Growth and Development of the Adolescent: 11 to 18 Years
Cognitive Level: Apply
Client Needs: Health Promotion and Maintenance
Integrated Process: Nursing Process
Reference: p. 579-580

23. The nutritionist is presenting information about vegetarian diets to a group of


nursing students. The nutritionist determines the session is successful when the
students correctly choose which factor concerning the semi-vegetarian diet?
A. "The diet excludes red meat and possibly poultry."
B. "The diet excludes red meat, poultry, and fish."
C. "The diet excludes meat, poultry, fish, and eggs."
D. "The diet excludes dairy products, meat, poultry, fish, and eggs."

Answer: A

Rationale: The semi-vegetarian diet includes dairy products, eggs, and fish and
excludes red meat and possibly poultry. The lacto-ovo-vegetarian diet includes eggs
and dairy products but excludes meat, poultry, and fish. The lacto-vegetarian diet
includes dairy products and excludes meat, fish, poultry, and eggs. The vegan diet
excludes all food of animal origin, including dairy products, eggs, fish, meat, and
poultry.
Question format: Multiple Choice
Chapter 27: Growth and Development of the Adolescent: 11 to 18 Years
Cognitive Level: Apply
Client Needs: Health Promotion and Maintenance
Integrated Process: Nursing Process
Reference: p. 579-580

24. The school nurse is presenting information to a group of high school students
who are asking about vegetarian diets. The nurse determines that the group needs
more information when they identify which foods as being included in a vegan diet?
A. red meat and possibly poultry
B. red meat, poultry, and fish
C. meat, poultry, fish, and eggs
D. dairy products, meat, poultry, fish, and eggs

Answer: D

Rationale: The vegan diet excludes all food of animal origin, including dairy
products, eggs, fish, meat, and poultry. The semi-vegetarian diet includes dairy
products, eggs, and fish and excludes red meat and possibly poultry. The lacto-ovo-
vegetarian diet includes eggs and dairy products but excludes meat, poultry, and
fish. The lacto-vegetarian diet includes dairy products and excludes meat, fish,
poultry, and eggs.
Question format: Multiple Choice
Chapter 27: Growth and Development of the Adolescent: 11 to 18 Years
Cognitive Level: Analyze
Client Needs: Physiological Integrity: Basic Care and Comfort
Integrated Process: Teaching/Learning
Reference: p. 579-580

25. A group of nursing students are preparing sample menus for the lacto-ovo-
vegetarian diet. The instructor determines the presentation is successful after
noting the students avoided the use of which foods in planning the menus?
A. red meat and possibly poultry
B. red meat, poultry, and fish
C. meat, poultry, fish, and eggs
D. dairy products, meat, poultry, fish, and eggs

Answer: B

Rationale: The lacto-ovo-vegetarian diet includes eggs and dairy products but
excludes meat, poultry, and fish. The semi-vegetarian diet includes dairy products,
eggs, and fish and excludes red meat and possibly poultry. The lactovegetarian diet
includes dairy products and excludes meat, fish, poultry, and eggs. The vegan diet
excludes all food of animal origin, including dairy products, eggs, fish, meat, and
poultry.
Question format: Multiple Choice
Chapter 27: Growth and Development of the Adolescent: 11 to 18 Years
Cognitive Level: Analyze
Client Needs: Physiological Integrity: Basic Care and Comfort
Integrated Process: Nursing Process
Reference: p. 579-580

26. A male nurse is meeting with a group of high school boys to discuss various
health topics. After the session on testicular self-exam, the nurse determines the
session is successful when one of the students responds with which comment?
A. "It sounds like we will need to know how to do this when we are in our 20s, so
we might as well learn now."
B. "My uncle had testicular cancer after he got married and had my cousin."
C. "I am almost 15 now, so that means I could possibly get this disease."
D. "Men my grandfather's age will probably die if they don't do these exams."

Answer: C

Rationale: Testicular cancer is one of the most common cancers in young men
between the ages of 15 and 34. Teaching the boys how and when to perform
testicular self-examination is an important health maintenance activity. It is easier
to treat testicular cancer if it is caught early and the best way to catch it is routine
self-exams. A family history of testicular cancer increases the chances of the
younger family members developing the cancer. The self-exam should be continued
on a routine basis at least until the individual is 35 years old.
Question format: Multiple Choice
Chapter 27: Growth and Development of the Adolescent: 11 to 18 Years
Cognitive Level: Analyze
Client Needs: Health Promotion and Maintenance
Integrated Process: Nursing Process
Reference: p. 584

27. The nurse in an emergency department is assessing an 18-year-old who is


reporting a gap in memory for the previous night. The client reports feeling dizzy
and disoriented after drinking only one drink. The client woke up in a friend's bed
but has no recollection of getting there. What action should the nurse prioritize for
this client?
A. Call social services to counsel the client about blackouts related to alcohol use
disorder.
B. Call the neurology department so they can see the client to rule out a
neurological impairment.
C. Have the client call a caregiver to get permission to treat and counsel the client.
D. Complete a rape kit to rule out possible sexual abuse while the client was
blacked out.

Answer: D

Rationale: In recent years the use of rohypnol, also known as the "date rape drug,"
has become a concern for the adolescent. Rohypnol is not sold legally in the United
States but is brought in from countries where it is sold legally. The drug, especially
in combination with alcohol, causes memory loss, blackouts, and an inability to
resist sexual attacks. Often the drug is secretly slipped into a person's drink. The
drug has no taste or odor, but within a few minutes after ingesting the drug, the
person feels dizzy, disoriented, and nauseated, and then rapidly passes out. After
several hours, the person awakens and has no memory of what happened while
under the influence of the drug. Waking up in someone's bed should raise
suspicions that the individual was raped. Completing a rape kit will provide the
authorities with the evidence they will need to further pursue the case.
Question format: Multiple Choice
Chapter 27: Growth and Development of the Adolescent: 11 to 18 Years
Cognitive Level: Analyze
Client Needs: Physiological Integrity: Reduction of Risk Potential
Integrated Process: Nursing Process
Reference: p. 585
Chapter 28
1. The nurse is interviewing the mother of a child who is at the local clinic. When
asked why she brought her toddler in today, she replies that the child has been
running a fever and coughing a lot since last weekend. This information would be
noted in the chart as what data?
A. heath history
B. present health concern
C. chief complaint
D. biographical history

Answer: C

Rationale: The reason for a child's visit to a health care facility on a specific day is
called the chief complaint. A health history is information encompassing the
maternal and pregnancy history as well as the child's health occurrences up to this
time. The present health concern is further information about the reason for the
visit, such as the quality of the cough, frequency of the cough, and how high the
fever was. A biographical history of a child would be the child's name, address, and
other contact information.
Question format: Multiple Choice
Chapter 28: Assessment of the Child (Data Collection)
Cognitive Level: Apply
Client Needs: Health Promotion and Maintenance
Integrated Process: Nursing Process
Reference: p. 594

2. The nurse asks the parents of a child about the family health history. The father
asks the nurse why she needs his information. The nurse would explain that the
family health history is gathered for what reason?
A. The number of family members that have a certain health problem will help the
nurse know if the child will have the same problem.
B. Identifying risk factors in families decreases the child's risk of developing the
same conditions or health problems.
C. The nurse needs to know everything about a family to take care of the child.
D. By establishing family behavior, the nurse forces the parents to alter their care
of their child and make them healthier.

Answer: B

Rationale: Collecting a complete family health history helps the nurse learn if there
are certain behaviors or risk factors for the family; this will hopefully educate the
family in how to improve both their health and the child's health, as well as reduce
the incidence of diseases and chronic conditions.
Question format: Multiple Choice
Chapter 28: Assessment of the Child (Data Collection)
Cognitive Level: Understand
Client Needs: Health Promotion and Maintenance
Integrated Process: Nursing Process
Reference: p. 594

3. As part of an admission to the pediatric floor, the nurse gathers a lifestyle


history. What components would be included in this part of the client history?
Select all that apply.
A. the child's grade in school
B. habits such as thumb sucking or nail biting
C. the child's developmental milestones
D. client allergies and medications
E. custody of the child if parents are divorced

Answer: A, B, E

Rationale: A lifestyle history includes data related to the school history, social
history, personal history and nutritional history. Asking about what grade the child
is in, his or her habits, and who has primary custody in the case of divorced parents
are all part of a lifestyle history. Developmental milestones and client allergies and
medications fall under other history categories.
Question format: Multiple Select
Chapter 28: Assessment of the Child (Data Collection)
Cognitive Level: Apply
Client Needs: Health Promotion and Maintenance
Integrated Process: Nursing Process
Reference: p. 597

4. The nurse is observing an infant and notes that the infant is restless, wide-awake
but quiet. The mother reports that her baby often is awake for hours at a time and
is very fidgety. How would the nurse respond to her observation?
A. The nurse should report the infant's behavior to her supervisor for further
investigation.
B. Since the infant is often awake for long periods, the nurse needs to talk to the
doctor about prescribing some medication to calm the child.
C. The nurse should do a more in-depth investigation to determine why the infant is
awake so much and is restless.
D. Because the mother reports that the infant's behavior is normal, there is no need
for further observation.

Answer: D

Rationale: Often, the nurse observes what she would consider concerning behavior
in an infant or child but, when talking with the parent, determines that the child's
behavior is normal for them. If this happens, there is no need for further
evaluation. The nurse would simply document the findings.
Question format: Multiple Choice
Chapter 28: Assessment of the Child (Data Collection)
Cognitive Level: Apply
Client Needs: Health Promotion and Maintenance
Integrated Process: Nursing Process
Reference: p. 598

5. An 8-month-old infant in being held by her mother and the nurse needs to obtain
the infant's vital signs. Which approach would most likely ensure accurate
readings?
A. Place the infant on the examination table, take the temperature rectally and
listen to the heart rate and note respirations.
B. Allow the mother to continue to hold the infant, listen to the child's heart rate,
count respirations by the abdominal rise, then take an axillary temperature.
C. Have the mother hold the infant in her lap lying down, take a rectal temperature,
and listen to heart and breath sounds from there.
D. Lay the infant on the examination table, have the mother help hold the infant
still, and take an axillary temperature and count both heart rate and respiratory
rate by auscultation.

Answer: B

Rationale: The best method for obtaining vital signs on an infant is to keep the
infant calm and begin with listening to heart and breath sounds, followed by
temperature. Respiratory rate is easily obtained by watching an infant's abdomen
rise and fall since infants are abdominal breathers. Keeping the infant calm is best
accomplished by allowing the parent to hold the client.
Question format: Multiple Choice
Chapter 28: Assessment of the Child (Data Collection)
Cognitive Level: Apply
Client Needs: Health Promotion and Maintenance
Integrated Process: Nursing Process
Reference: p. 600-601

6. Which physical assessment data would the nurse find concerning and would
warrant reporting to the primary care provider?
A. Head circumference is 2 inches less than the chest circumference in a 5-year-old
child.
B. A school-aged child has a heart rate of 90.
C. a blood pressure of 128/80 in a preschool-aged child
D. an infant who has a closed posterior fontanel (fontanelle) at age 4 months

Answer: C

Rationale: The normal blood pressure for a preschooler is 89-112 systolic and 46-
72 diastolic, so a blood pressure of 128/80 is abnormal and needs reporting to the
physician. The chest circumference for a child is 2 to 3 inches greater than the head
circumference. The normal heart rate for a school-aged child is 60-110. The
posterior fontanel (fontanelle) closes around 2 to 3 months of age in infants.
Question format: Multiple Choice
Chapter 28: Assessment of the Child (Data Collection)
Cognitive Level: Analyze
Client Needs: Health Promotion and Maintenance
Integrated Process: Nursing Process
Reference: p. 604

7. A mother observes the office nurse charting her son's height and weight on a
growth chart and asks the nurse the purpose of plotting this information for her
child. The nurse would reply with which explanation?
A. The height and weight of each client is plotted on a growth chart at each visit to
note how the child is growing and compare the growth to the norm.
B. The doctor wants to see how fast he is growing over the last year.
C. Each child grows at a certain rate and the doctor wants to see her child's rate of
growth.
D. Boys and girls grow differently and the doctor wants to compare her son's
growth to her daughter's growth at the same age.

Answer: A

Rationale: Plotting a child's height and weight are very important to determine if a
child is growing normally or is too large or too small for their age. This is
determined by plotting these measurements on a standardized growth chart. This
data gives the health care provider a good clue as to the child's health status.
Question format: Multiple Choice
Chapter 28: Assessment of the Child (Data Collection)
Cognitive Level: Apply
Client Needs: Health Promotion and Maintenance
Integrated Process: Nursing Process
Reference: p. 598-600

8. The caregiver of a 3-year-old boy presents at the receptionist desk and reports
that the child is nauseated. In interviewing the child's caregiver, which question
should the nurse prioritize when starting the assessment?
A. "Why did you decide to bring your son to the clinic today?"
B. "How often does your son complain of being nauseated?"
C. "Does anyone else in the family have the same symptoms?"
D. "Has your son had anything to eat that he might be allergic to?"

Answer: A

Rationale: To provide the best care for the child, it is important to get the most
complete explanation of what brought the child to the health care setting. The
nurse may not have heard the entire conversation and asking an open-ended
question will allow the caregiver to provide the nurse with all the information
necessary to provide the best care. The other questions could be asked depending
on the answers given by the caregiver.
Question format: Multiple Choice
Chapter 28: Assessment of the Child (Data Collection)
Cognitive Level: Apply
Client Needs: Health Promotion and Maintenance
Integrated Process: Nursing Process
Reference: p. 594

9. A 14-year-old boy has come to his primary care physician's office for a routine
well-child visit with his parent. Which statement by the parent should the nurse
prioritize for further investigation after noting the father has a history of alcohol
use disorder?
A. "I wish there was a blood test for alcohol use disorder. I know my son is at risk."
B. "Our next door neighbor is older than my son, and he drinks when they hang out
together."
C. "I think I know how my son feels about drinking. He has had substance use
disorder education in school."
D. "Sometimes my son asks me questions about his father's low tolerance for
alcohol."

Answer: B

Rationale: Some diseases and conditions are seen across families, and this is
important in prevention as well as detection for the child. The caregiver can usually
provide information regarding family health history. The nurse should use this
information to do preventive teaching with the child and family. Early adolescence
is a time when experimental use of substances, especially alcohol and tobacco,
might be seen. It would be important to assess the use of substances and follow up
regarding the behaviors of the adolescent.
Question format: Multiple Choice
Chapter 28: Assessment of the Child (Data Collection)
Cognitive Level: Apply
Client Needs: Psychosocial Integrity
Integrated Process: Nursing Process
Reference: p. 594

10. The nurse is assisting with the physical exam on a 2-year-old child. The nurse
predicts the order of the exam will be in which sequence?
A. Head and neck; eyes; ears; nose, mouth; then the back and extremities
B. Back and extremities; eyes, ears, nose, mouth; then the head and neck
C. Back and extremities; head and neck; then the eyes, ears, nose, and mouth
D. Eyes, ears, nose, mouth, back, and extremities; then the head and neck

Answer: C

Rationale: Data are collected by examination of the body systems. Often the exam
for a child is not done in a head-to-toe manner, as is done with adults, but rather in
an order that takes the child's age and developmental needs into consideration.
Aspects of the exam that might be more traumatic or uncomfortable for the child,
such as examining the nose and mouth, are done last.
Question format: Multiple Choice
Chapter 28: Assessment of the Child (Data Collection)
Cognitive Level: Apply
Client Needs: Health Promotion and Maintenance
Integrated Process: Nursing Process
Reference: p. 607

11. The nurse is assessing a 5-month-old infant at a well-child visit. In measuring


and weighing the child, the nurse will prioritize which action?
A. Wipe the scale with an alcohol-based cleanser.
B. Cover the scale with a clean piece of paper or paper towel.
C. Straighten the child's legs and hold the knees flat.
D. Hold one hand within 1 inch (2.54 cm) of the child.

Answer: D

Rationale: Safety is always the priority. The nurse must have a hand near to be
able to prevent the child from any type of fall or injury. The infant is weighed nude
while lying on an infant scale; when the infant is big enough to sit, he or she can
also be weighed while sitting. The scale is covered with a fresh paper towel or clean
sheet of paper as a means of infection control. To measure an infant, place the
child flat, with the knees held flat, on an examining table. Measure the child's
height by straightening the child's body and measuring from the top of the head to
the bottom of the foot. The measurement can be done by making marks on the
paper table covering and then measuring between the marks.
Question format: Multiple Choice
Chapter 28: Assessment of the Child (Data Collection)
Cognitive Level: Apply
Client Needs: Safe, Effective Care Environment: Safety and Infection Control
Integrated Process: Nursing Process
Reference: p. 598

12. The nurse is caring for a child in the emergency department who is on a cardiac
monitor. Which nursing action should the nurse prioritize?
A. Clean the skin with alcohol before placing the electrodes.
B. Confirm the alarms are set with maximum and minimum settings.
C. Check the site and skin condition every couple of hours.
D. Check to be sure that the electrodes are secure when the alarm sounds.

Answer: B

Rationale: Cardiac monitors are used to detect changes in cardiac function. The
highest priority would be to ensure the alarms are set with maximum and minimum
settings and that the alarm is turned on. Many of these monitors have a visual
display of the cardiac actions. Electrodes must be placed properly to obtain
accurate readings of the cardiac system. The skin is cleaned to remove dirt, lotions,
and powder before the electrodes are applied. The electrode sites must be checked
every two hours to detect any skin redness or irritation and to determine that the
electrodes are secure.
Question format: Multiple Choice
Chapter 28: Assessment of the Child (Data Collection)
Cognitive Level: Apply
Client Needs: Physiological Integrity: Physiological Adaptation
Integrated Process: Nursing Process
Reference: p. 603

13. The nurse is admitting a child to the intensive care unit. The child arrives with a
cardiac monitor, pulse oximeter, and an IV infusion. As the nurse begins collecting
data on the child, which nursing interventions should the nurse prioritize?
A. Verify that the alarms on the monitor are still properly set.
B. Assess the neurologic function using the Glasgow coma scale.
C. Change the probe on the pulse oximeter.
D. Check the apical pulse rate using a pediatric stethoscope.

Answer: A

Rationale: At the beginning of each shift and after transport of the client, the nurse
must check that alarms are accurately set and have not been inadvertently
changed. This is true for all types of monitors. The neurologic status will most likely
be checked, as well as the apical pulse, but they are not priorities. The probe on the
pulse oximeter is changed if needed, but not routinely and not as a priority.
Question format: Multiple Choice
Chapter 28: Assessment of the Child (Data Collection)
Cognitive Level: Apply
Client Needs: Safe, Effective Care Environment: Safety and Infection Control
Integrated Process: Nursing Process
Reference: p. 603

14. The nurse is caring for a child admitted with a head injury and is conducting an
ongoing assessment. The nurse prepares to utilize the Glasgow coma scale to
achieve which priority nursing intervention?
A. Look back at the client's neurologic status the last time the nurse cared for this
client.
B. Create reports based on using a consistent tool.
C. Have a comparison of the child's status from one check to another.
D. Report to the oncoming nurse any significant changes.

Answer: C

Rationale: The nurse often is responsible for using neurologic assessment tools to
monitor a child's neurologic status after the initial neurologic exam. The nurse uses
a neurologic assessment tool such as the Glasgow coma scale. The use of a
standard scale for monitoring allows for the comparison of results from one time to
another and from one examiner to another. If a child is hospitalized with a
neurologic concern, the neurologic status is monitored closely and a neurologic
assessment tool is used every 1 or 2 hours to observe for significant changes.
Question format: Multiple Choice
Chapter 28: Assessment of the Child (Data Collection)
Cognitive Level: Apply
Client Needs: Physiological Integrity: Reduction of Risk Potential
Integrated Process: Nursing Process
Reference: p. 607

15. The nursing student is preparing to explain the appropriate steps for assessing
an infant. The instructor determines the student's presentation is successful after
illustrating which location as appropriate for obtaining an apical pulse?
A. Between the sternum and the left nipple
B. Above the sternum, slightly to the right
C. Below the ribs about one half of an inch
D. Above the clavicle on the left side

Answer: A

Rationale: When taking an infant's apical pulse, the stethoscope is placed between
the child's left nipple and sternum. The other locations will not assist with localizing
over the apex of the heart.
Question format: Multiple Choice
Chapter 28: Assessment of the Child (Data Collection)
Cognitive Level: Analyze
Client Needs: Health Promotion and Maintenance
Integrated Process: Teaching/Learning
Reference: p. 602

16. The nursing instructor is monitoring the nursing students as they role-play
conducting assessments on children and their caregivers. The instructor determines
the session is successful after witnessing the students collect the necessary
subjective data during which portion of the assessment process?
A. Interviewing the child's caregiver
B. Weighing and measuring the child
C. Reinforcing teaching with the child's caregivers
D. Taking the child's vital signs

Answer: A

Rationale: Information spoken by the child or family is called subjective data.


Interviewing the family caregiver and child allows you to collect information that
can be used to develop a plan of care for the child. Collecting objective data would
include weighing and measuring the child and taking the child's vital signs.
Reinforcing teaching would involve client education to ensure the caregivers are
aware of the treatments and/or further care.
Question format: Multiple Choice
Chapter 28: Assessment of the Child (Data Collection)
Cognitive Level: Analyze
Client Needs: Health Promotion and Maintenance
Integrated Process: Teaching/Learning
Reference: p. 593

17. A toddler is brought to the pediatric clinic by the caregiver because the child
"doesn't feel well." As the nurse interviews the caregiver about why the client is
there, which goal is the nurse prioritizing at this point?
A. Obtaining the health history
B. Interviewing the client
C. Obtaining biographical data
D. Determining the chief complaint

Answer: D

Rationale: The reason for a visit to a health care setting is called the chief
complaint. In a well-child setting, this reason might be a routine check or
immunizations, or an illness or other condition. The nurse will obtain this
information by interviewing the caregiver. Obtaining the health history and
biographical data will also be completed during the assessment process.
Question format: Multiple Choice
Chapter 28: Assessment of the Child (Data Collection)
Cognitive Level: Apply
Client Needs: Health Promotion and Maintenance
Integrated Process: Nursing Process
Reference: p. 594

18. The nursing instructor is conducting a clinical session on the proper techniques
for assessing a child's head circumference. The instructor should point out which
factor concerning this assessment?
A. Measure the head circumference routinely on children up to the age of 6 years.
B. Place the tape measure around the head just above the eyebrows.
C. Expect the head circumference and the chest circumference measurements to be
equal up to the age of 6 years.
D. Place the tape measure around the head with the tape touching just below the
eyes.

Answer: B

Rationale: The head circumference is measured routinely in children to the age of 2


or 3 years, or in any child with a neurologic concern. Place a paper or plastic tape
measure around the largest part of the head just above the eyebrows and around
the most prominent part of the back of the head. During childhood the chest
exceeds the head circumference by 2 to 3 inches.
Question format: Multiple Choice
Chapter 28: Assessment of the Child (Data Collection)
Cognitive Level: Analyze
Client Needs: Health Promotion and Maintenance
Integrated Process: Teaching/Learning
Reference: p. 600
19. The nurse is explaining to a group of nursing students the proper technique for
obtaining an accurate temperature on a child. The instructor determines the session
is successful when the students correctly choose which factor as related to taking a
temperature?
A. "A rectal temperature is usually 0.5° to 1.0° lower than the oral measurement."
B. "Tympanic temperatures should not be taken on a child who is sleeping."
C. "An axillary temperature usually measures 0.5° to 1.0° higher than the oral
measurement."
D. "Rectal temperatures should not be taken on a child with diarrhea."

Answer: D

Rationale: A rectal temperature should not be taken in the newborn because of the
danger of irritation to the rectal mucosa or in children who have had rectal surgery
or who have diarrhea. A rectal temperature is usually 0.5° to 1.0° higher than the
oral temperature and the axillary temperature is usually 0.5° to 1.0° lower than the
oral temperature. It is easier to obtain a tympanic temperature in a sleeping child
as the temperature can be obtained without walking them up.
Question format: Multiple Choice
Chapter 28: Assessment of the Child (Data Collection)
Cognitive Level: Analyze
Client Needs: Physiological Integrity: Reduction of Risk Potential
Integrated Process: Teaching/Learning
Reference: p. 601

20. The caregiver of an infant keeps removing the pulse oximetry sensor claiming it
is too tight and hurting her baby. Which response should the nurse prioritize in
addressing this situation?
A. Explain that pulse oximetry measures the oxygen saturation of arterial
hemoglobin.
B. Place the probe of the oximeter on the child's chest and secure it with tape.
C. Ensure the oximeter probe site is checked every 8 hours for possible reactions.
D. Explain that pulse oximetry is done to detect respiratory retractions.

Answer: A

Rationale: Pulse oximetry measures the oxygen saturation of arterial hemoglobin.


The probe of the oximetry unit can be placed on the finger, toe, or clipped on the
earlobe. In an infant, the foot or toe is often used. In certain situations the probe is
left in place to continually monitor the oxygen saturation. Check the site every 2
hours to ensure that the probe is secure and tissue perfusion is adequate. Change
the site at least every 4 hours to prevent skin irritation. The probe cannot be taped
to the chest wall and still collect the desired information. Respiratory retractions are
determined by direct inspection of the chest wall.
Question format: Multiple Choice
Chapter 28: Assessment of the Child (Data Collection)
Cognitive Level: Apply
Client Needs: Physiological Integrity: Reduction of Risk Potential
Integrated Process: Nursing Process
Reference: p. 603

21. The nurse in the pediatric unit is using the Glasgow coma scale to monitor a
child admitted with a neurologic disorder. The nurse will prioritize the continued
use of the Glasgow coma scale based on which reason?
A. measures the verbal and motor responses of the child
B. used one or two times a day to note changes in the neurologic status
C. used to compare the child's results with other children of the same age
D. monitors the child's vision by using a bright flashlight

Answer: A

Rationale: The Glasgow coma scale is used to monitor a child's neurologic status
after the initial neurologic exam. Motor, verbal, and papillary responses are
monitored. The use of a standard scale for monitoring permits the comparison of
results from one time to another and from one examiner to another. Use the tool
every 1 or 2 hours to observe for significant changes. This scale is not used to
compare children's results nor is it used to monitor vision.
Question format: Multiple Choice
Chapter 28: Assessment of the Child (Data Collection)
Cognitive Level: Apply
Client Needs: Physiological Integrity: Reduction of Risk Potential
Integrated Process: Nursing Process
Reference: p. 607

22. A new client arrives for a first visit to the pediatric clinic. The nurse will
prioritize which step in this appointment?
A. recording the health history
B. interviewing the caregiver
C. obtaining biographical data
D. determining the chief complaint

Answer: C

Rationale: The nurse begins a client history by obtaining the biographical data first.
The nurse should collect and record identifying information about the child,
including the child's name, address, and phone number, as well as information
about the caregiver. This information will assist the nurse in ensuring the right
information is documented with the right client. The other information will then be
collected. All this information will be obtained by interviewing the caregiver, as well
as possibly using a questionnaire.
Question format: Multiple Choice
Chapter 28: Assessment of the Child (Data Collection)
Cognitive Level: Apply
Client Needs: Health Promotion and Maintenance
Integrated Process: Nursing Process
Reference: p. 594
23. The nurse is collecting data on a 3-year-old child whose caregiver is present.
Which action should the nurse take?
A. Instruct the caregiver to fill out the paperwork outside the room while the nurse
is measuring the child's height and weight.
B. Escort the child to another room to take the child's vital signs.
C. Perform a head-to-toe assessment starting with the eyes, ears, and nose.
D. Show the caregiver the process for collecting a urine specimen and ask the
caregiver to help the child.

Answer: D

Rationale: If comfortable with helping, the caregiver may help with the data
collection. For example, the caregiver might help take a young child's temperature
and obtain a urine specimen. Arrangements should be made so that the caregiver
also may be present, if possible, for tests or examinations that need to be
performed. Often the examination for a child does not proceed in a head-to-toe
manner as in adults but rather in an order that takes the child's age and
developmental needs into consideration. Examining the nose and mouth may be
uncomfortable and traumatic for the child; the nurse should save these for last. The
paperwork should be completed before the physical assessment begins. It will be
less traumatic for the child if as many aspects of the assessment can be done in
one location.
Question format: Multiple Choice
Chapter 28: Assessment of the Child (Data Collection)
Cognitive Level: Apply
Client Needs: Health Promotion and Maintenance
Integrated Process: Nursing Process
Reference: p. 598

24. The nurse is assisting with the physical examination on a sleeping10-month-old


infant being held by the parent against the parent's shoulder. In what sequence
would the nurse complete the assessment?
A. head and neck; eyes, ears, nose, mouth; then the back and extremities
B. back and extremities; eyes, ears, nose, mouth; then the head and neck
C. back and extremities; head and neck; then the ears, nose, mouth, and eyes
D. eyes, ears, nose, mouth; back and extremities; then the head and neck

Answer: C

Rationale: Data are collected by examination of the body systems. Often the exam
for an infant is not done in a head-to-toe manner, as is done with adults, but rather
in an order that takes the infant's age and developmental needs into consideration.
Because the infant is asleep and held against the parent's shoulder, the nurse
would begin by assessing the infant's back and extremities. The infant's eyes would
be inspected last to allow the infant to be most comfortable until the end of the
assessment. Aspects of the examination that might be more traumatic or
uncomfortable for the infant are completed last.
Question format: Multiple Choice
Chapter 28: Assessment of the Child (Data Collection)
Cognitive Level: Apply
Client Needs: Health Promotion and Maintenance
Integrated Process: Nursing Process
Reference: p. 598

25. A 2-year-old child with a 3-day history of diarrhea is brought to the urgent care
clinic by the caregiver. The nurse determines the child's axillary temperature is
within the normal range based on which finding?
A. 99.3°F (37.4°C)
B. 96.6°F (35.8°C)
C. 100.3°F (38.4°C)
D. 99.8°F (37.9°C)

Answer: B

Rationale: Axillary temperatures are taken on newborns, on infants and children


with diarrhea, and in other cases when a rectal temperature is contraindicated. An
axillary temperature usually measures 0.5 to 1.0 degrees lower than the oral
measurement. A normal oral temperature range is 97.6°F to 99.3°F (36.4°C to
37.4°C). A rectal temperature is usually 0.5° to 1° higher than the oral
measurement.
Question format: Multiple Choice
Chapter 28: Assessment of the Child (Data Collection)
Cognitive Level: Apply
Client Needs: Physiological Integrity: Reduction of Risk Potential
Integrated Process: Nursing Process
Reference: p. 601
Chapter 29
1. The nurse is doing the preoperative preparation for a 6-year-old who is having
surgery. What is most important for the nurse to document in the child's preop
chart?
A. the name and phone number of the individuals who will be in the waiting room
during the surgical procedure
B. the number of loose teeth the child has and which teeth are missing
C. the number of times the child has voided in the past 8 hours and if the child has
had a stool
D. the type and amount of pain medication the child has been given in the past 24
hours

Answer: B

Rationale: Loose teeth are also a potential hazard for the child having a surgical
procedure done and should be counted and recorded according to hospital policy.
Question format: Multiple Choice
Chapter 29: Care of the Hospitalized Child
Cognitive Level: Understand
Client Needs: Safe, Effective Care Environment: Management of Care
Integrated Process: Nursing Process
Reference: p. 622

2. The nurse administering the preoperative medications to the child going to


surgery would anticipate which of the following related to giving preoperative
medications?
A. A sedative and an analgesic-atropine mixture to be sent to the operating room
with the child.
B. A sedative to be given one-and-a-half to two hours before surgery and an
analgesic-atropine mixture given just before the child leaves for the operating
room.
C. A sedative and an analgesic-atropine mixture to be given just before the child
goes to sleep the night before the surgery.
D. A sedative to be given three to four hours before surgery and an analgesic-
atropine mixture given one to two hours before the child leaves for the operating
room.

Answer: B

Rationale: Preoperative medications usually are given in two stages: a sedative is


administered about one-and-a-half to two hours before surgery, and an analgesic-
atropine mixture may be administered immediately before the client leaves for the
operating room.
Question format: Multiple Choice
Chapter 29: Care of the Hospitalized Child
Cognitive Level: Understand
Client Needs: Physiological Integrity: Reduction of Risk Potential
Integrated Process: Nursing Process
Reference: p. 622

3. What actions can a pediatric hospital staff take to lessen the stress of
hospitalization for a child? Select all that apply.
A. Have the parents stay with the client at all times.
B. Allow children to wear their own clothing.
C. Perform any treatments in the procedure room.
D. Select foods for the child that they normally like.
E. Involve Child Life in teaching about upcoming surgeries.

Answer: B, C, E

Rationale: By allowing a child to have some control over their lives while
hospitalized, their transition to the hospital is smoother and less traumatic. Wearing
their own clothing makes the hospital feel more like home. Performing procedures
in another room outside the child's hospital room maintains the feeling of safety in
their own room. Child Life is an integral part of educating children about upcoming
surgeries and procedures and is designed to lessen the stress the children are
experiencing.
Question format: Multiple Select
Chapter 29: Care of the Hospitalized Child
Cognitive Level: Apply
Client Needs: Psychosocial Integrity
Integrated Process: Nursing Process
Reference: p. 619

4. The nurse is reviewing a client's chart and notes that the child is in droplet
precautions for pneumonia. What protection would she need to take in caring for
the child?
A. gown, handwashing and gloves
B. handwashing and gloves
C. gown, gloves, and mask
D. mask and handwashing

Answer: C

Rationale: For droplet precautions, the nurse needs to wear a mask, gown, and
gloves for client care since the transmission is airborne and is spread through
contact with secretions from the mouth, nose, and lungs when a client sneezes or
coughs.
Question format: Multiple Choice
Chapter 29: Care of the Hospitalized Child
Cognitive Level: Apply
Client Needs: Safe, Effective Care Environment: Safety and Infection Control
Integrated Process: Nursing Process
Reference: p. 616

5. The nurse has been assigned to care for a child who is on transmission-based
precautions. This nurse has not cared for this child before. Which action would be
the best way to help the child feel comfortable with the nurse?
A. Ask the parent to introduce the new nurse.
B. Let the child see the nurse's face before the mask is put on.
C. Read to the child for a few minutes before starting care.
D. Remind the child that her caregivers will be in to visit soon.

Answer: B

Rationale: If masks or gloves are part of the necessary precautions, the child may
experience even greater feelings of isolation. Before putting on the mask, the nurse
should allow the child to see his or her face; this process will help the child easily
identify the nurse. Being introduced by the parents may be effective but is not
likely to be possible. Reading to the child, or explaining that the caregiver will visit
soon, are appropriate actions but are not the best ways to help the child feel less
isolated and more comfortable with the nurse in the isolation setting.
Question format: Multiple Choice
Chapter 29: Care of the Hospitalized Child
Cognitive Level: Apply
Client Needs: Psychosocial Integrity
Integrated Process: Nursing Process
Reference: p. 617

6. A 5-year-old boy is brought to the emergency room with a possible broken arm.
His 18-year-old sister, who is also his primary caregiver, has come with him. The
boy is relatively calm, but his sister is so upset she is nearly hysterical. The nurse
notes that as her behaviors show more anxiety, the boy is getting more upset and
his anxiety is also increasing. What initial action would be best for the nurse to
take in this situation?
A. While attending to the child, reassure the sister and suggest interventions she
can help with.
B. Ask his sister to calm down so that she can help her brother remain calm as well.
C. Reassure the child that he is going to be fine and that his sister is just nervous.
D. Ask the sister if she prefers to leave the exam room.

Answer: A

Rationale: In an emergency, physical needs assume priority over emotional needs.


The emergency itself is frightening to the child and the family and the need for
treatment is urgent. Even though a caregiver tries to act calm and composed, the
child often may sense the anxiety. If the hospital is still a great unknown, it will
only add to the child's fear and panic. It is best if the caregiver can be present to
reassure the child.
Question format: Multiple Choice
Chapter 29: Care of the Hospitalized Child
Cognitive Level: Apply
Client Needs: Psychosocial Integrity
Integrated Process: Nursing Process
Reference: p. 619-620

7. The nurse is providing care for a 4-year-old being treated in the emergency
department. Which intervention demonstrates a nurse's understanding of the basic
principle of emergency care when addressing the child's need for a sense of
security?
A. The parents are asked to keep the child distracted until the admission process is
conducted.
B. The nurse conducts the assessment while gently reassuring both the child and
the parents.
C. The nurse keeps the parents informed so they may remain calm and composed,
thus minimizing the child's anxiety.
D. The child is sedated so that his or her anxiety level can be managed effectively.

Answer: B

Rationale: In an emergency, physical needs assume priority over emotional needs.


By addressing the child's need for an assessment while attempting to acknowledge
the existing emotional needs, the nurse is providing quality care. Sedation would be
used only as a last resort. The other options can affect the child's sense of security,
but the factor most affecting feelings of security is that emotional needs cannot be
adequately responded to in an emergency situation.
Question format: Multiple Choice
Chapter 29: Care of the Hospitalized Child
Cognitive Level: Apply
Client Needs: Psychosocial Integrity
Integrated Process: Caring
Reference: p. 619-620

8. The nurse is caring for a 14-year-old child who is scheduled to have surgery the
next day. What would be the most appropriate intervention for the nurse to
implement to help prepare this child for surgery?
A. Allow the child to use the equipment that will be seen in the operating room,
such as masks, gowns, and gloves.
B. Explain to the caregivers while they are in the room with the child the reason the
child will be kept NPO.
C. Show the child models of the body that include the organs or parts of the body
the child will be having surgery on.
D. Give the child a book to read which describes the people who are involved in the
preoperative, surgical, and postoperative areas.

Answer: C

Rationale: The older child or adolescent may have a greater interest in the surgery
itself, what is wrong and why, how the repair is done, and the expected
postoperative results. Models of a child's internal organs or individual organs, such
as a heart, are useful for demonstration, or the client may be involved in making a
drawing showing the process.
Question format: Multiple Choice
Chapter 29: Care of the Hospitalized Child
Cognitive Level: Apply
Client Needs: Physiological Integrity: Reduction of Risk Potential
Integrated Process: Teaching/Learning
Reference: p. 621-622

9. The nurse at a pediatric clinic is preparing a 5-year-old child for admission to the
hospital for surgery. Which response should the nurse prioritize when asked by
the child what happens at the hospital?
A. "Remember? It's where your mom went to get your baby brother."
B. "People go there when they have a bad accident and get fixed."
C. "Some people go here to have babies, or when they're sick or hurt, so they can
try to get better."
D. "It's like a big doctor's office with lots of people and beds."

Answer: C

Rationale: Young children need to know that the hospital is more than a place
where "mommies go to get babies." It is also important to both avoid fostering the
view of the hospital as a place where people go to die and to avoid the impression
that everyone who comes in will definitely get well. Comparing the hospital to a
doctor's office minimizes the seriousness and breadth of services of the hospital.
Question format: Multiple Choice
Chapter 29: Care of the Hospitalized Child
Cognitive Level: Apply
Client Needs: Psychosocial Integrity
Integrated Process: Teaching/Learning
Reference: p. 613-614

10. The grandmother of a 1-year-old with cancer comes to visit the child in the
PICU. She sits close to the bed but doesn't touch the child. Which action by the
nurse would be most appropriate?
A. Respect their privacy and leave the room.
B. Pick up the child so that the grandmother sees he can be touched.
C. Ask the grandmother if she would like to talk to the child.
D. Encourage the grandmother to rock the child in a rocking chair.

Answer: D

Rationale: The nurse should promote the relationship between the family caregiver
and the child as much as possible. The caregiver should be encouraged to touch
and talk to the child. If possible, the caregiver may hold and rock the child; if not,
he or she can comfort the child by caressing and stroking. The nurse can often be a
support to the family by staying in the room rather than leaving them alone.
Question format: Multiple Choice
Chapter 29: Care of the Hospitalized Child
Cognitive Level: Apply
Client Needs: Psychosocial Integrity
Integrated Process: Nursing Process
Reference: p. 614-615

11. A 3-year-old client has been hospitalized for 1 week with her mother rooming-
in; however, the mother has gone home to tend to other family responsibilities for
a few days. After being inconsolable for the first 24 hours after the mother's
departure, the nurse notes the child is now lying quietly in bed sucking her thumb.
Which response should the nurse prioritize in this situation?
A. "Are you feeling sad? Your mom didn't want to leave, but she will be back after
two more breakfasts."
B. "I'm glad you're feeling better. You must be tired from all that crying."
C. "Do you miss your mom? Your sister missed her, too, so she had to go home to
visit for a few days."
D. "Should I read a story to you or would you like to play a game?"

Answer: A

Rationale: Three characteristic, consecutive stages of response to separation have


been identified: protest, despair, and denial. During the protest stage, the young
child cries, often refuses to be comforted by others, and constantly seeks the
primary caregiver. When the caregiver does not appear, the child enters the despair
stage and becomes apathetic and listless. Health care personnel often interpret this
as a sign that the child is accepting the situation, but this is not the case; rather,
the child has given up. She needs a chance to talk about her feelings, and facts
should be framed in a way she can understand. Young children have no concept of
time, so separation from their primary caregivers is especially difficult for them to
understand.
Question format: Multiple Choice
Chapter 29: Care of the Hospitalized Child
Cognitive Level: Apply
Client Needs: Psychosocial Integrity
Integrated Process: Caring
Reference: p. 618

12. The mother of a 4-year-old returns to the hospital after being away for 3 days.
She is anxious and excited to be back; however, the toddler turns his back to her
and scoots away as she attempts to pick him up. Which response should the nurse
prioritize in this situation?
A. "He is probably just tired from the many tests run while you were away."
B. "His distrust is abnormal. It may or may not go away in a day or two; we will
just have to wait and see."
C. "His distrust is normal and may have lingering effects, but you should touch and
soothe him as much as possible."
D. "Now that he has seen that you have returned he will feel better, but you should
leave the room for a few minutes to help him feel in control."

Answer: C

Rationale: Three characteristic, consecutive stages of response to separation have


been identified: protest, despair, and denial. In the denial stage, the child begins
taking interest in his or her surroundings and appears to accept the situation.
However, the damage is revealed when the caregivers do visit: the child may turn
away from them, showing distrust and rejection. It may take a long time before the
child accepts them again; even then remnants of the damage may linger. The child
may always have a memory of being abandoned at the hospital.
Question format: Multiple Choice
Chapter 29: Care of the Hospitalized Child
Cognitive Level: Apply
Client Needs: Psychosocial Integrity
Integrated Process: Nursing Process
Reference: p. 618

13. In a preadmission visit, a 9-year-old girl says she is afraid her tonsillectomy will
hurt too much. Which statement by the nurse would be best to help the child feel
more comfortable and less afraid?
A. "I'm sure you are worried about the pain, but lots of people have tonsillectomies
and they're fine after a few days."
B. "There's nothing to worry about. You will forget about the pain because you can
have all the ice cream and pudding you want."
C. "It won't hurt too much, so there is nothing to worry about. Besides, you need to
be brave for the younger ones."
D. "I understand that you're worried. It will hurt for only a few days; we'll give you
something to help stop the pain when you need it."

Answer: D

Rationale: In a successful preadmission visit, children's feelings are explored and


validated. The children are told that some things will hurt but that doctors and
nurses will do everything they can to make the pain go away. Honesty must be
central to any discussion of this kind. The other responses do not take seriously the
child's concerns and fears.
Question format: Multiple Choice
Chapter 29: Care of the Hospitalized Child
Cognitive Level: Apply
Client Needs: Psychosocial Integrity
Integrated Process: Caring
Reference: p. 619

14. A few days after discharge, the parent of an 8-year-old calls the pediatric clinic,
expressing concern about the child's behavior now that she is home. The child has
been treating her siblings badly and using inappropriate language. Which
suggestion should the nurse prioritize to this caregiver as an appropriate way to
handle this situation?
A. "Coming home is a difficult adjustment. Warn your daughter that you expect her
to begin to behave better over the next few weeks."
B. "Respond to her behavior in a firm, loving, consistent way."
C. "Children often feel guilty for the attention they've taken away from their siblings
and act out as a way of earning the attention."
D. "Tell her you don't like her behavior and have her stay in her room until she can
be nicer to her siblings."

Answer: B

Rationale: The return home may be a difficult period of adjustment for the entire
family. The older child may demonstrate anger or jealousy of siblings. The family
may be advised to encourage positive behavior and to avoid making the child the
center of attention because of the illness. Discipline should be firm, loving, and
consistent. The child may express feelings verbally or in play activities. The family
may be reassured that this is not unusual.
Question format: Multiple Choice
Chapter 29: Care of the Hospitalized Child
Cognitive Level: Apply
Client Needs: Psychosocial Integrity
Integrated Process: Teaching/Learning
Reference: p. 620

15. The nurse is caring for an adolescent just transferred from PACU to the
inpatient unit. As the nurse helps the child settle into the unit, which nursing
intervention should the nurse prioritize?
A. Turn the child every 2 hours.
B. Create an IV flow sheet.
C. Administer ice chips.
D. Monitor for any change in vital signs.

Answer: D

Rationale: After the child has been returned to his or her room, nursing care
focuses on careful observation of the vital signs for any signs or symptoms of
complications: shock, hemorrhage, or respiratory distress. An IV flow sheet is
begun that documents the type of fluid administered, the amount of fluid to be
absorbed, the rate of flow, any additive medications, the site, and the site's
appearance and condition. The physician should be notified if anuria (absence of
urine) persists longer than 6 hours. Postoperative orders may or may not provide
for ice chips or clear liquids to prevent dehydration; these may be administered
with a spoon or in a small medicine cup. Frequent repositioning is necessary to
prevent skin breakdown, orthostatic pneumonia, and decreased circulation.
Coughing, deep breathing, and position changes are performed at least every 2
hours.
Question format: Multiple Choice
Chapter 29: Care of the Hospitalized Child
Cognitive Level: Apply
Client Needs: Physiological Integrity: Physiological Adaptation
Integrated Process: Nursing Process
Reference: p. 621

16. A 10-year-old boy who had an appendectomy had expressed worry that
following the procedure he would have lots of pain. Two days after the procedure
the child is claiming he is having no pain. Which nursing intervention should the
nurse prioritize when assessing this child?
A. Tell him to let you know if he begins to feel pain.
B. Explain to his caregiver that his pain level shows he is getting better quickly.
C. Observe him for physical signs which might indicate pain.
D. Ask him to show you his pain level using the color pain scale.

Answer: C

Rationale: Nursing judgment is in order. Some children may try to hide pain
because they fear an injection or because they are afraid that admitting to pain will
increase the time they have to stay in the hospital. To use the color scale, a child
younger than 7 is given crayons ranging from yellow to red or black. Yellow
represents no pain; the darkest color (or red) represents the most pain. The child
selects the color that represents the amount of pain he or she feels. The most
appropriate pain scale to use with this child would be the 1 to 10 (with 10 being the
worst pain) or the faces scale.
Question format: Multiple Choice
Chapter 29: Care of the Hospitalized Child
Cognitive Level: Apply
Client Needs: Physiological Integrity: Basic Care and Comfort
Integrated Process: Nursing Process
Reference: p. 623

17. The nurse is meeting with a group of families to assist them in dealing with the
hospitalization of their child. Which comment by a family member should alert the
nurse to assist the family in coping with the situation?
A. "We have really good insurance—it covered everything the last time she was in
the hospital."
B. "When my sister was in the hospital before, the nurse let me get up on her bed
while she read me a story."
C. "Sometimes I wonder if the reason she is sick is because I have so many
responsibilities at work and at home."
D. "My husband was so relieved when he heard that after the next surgery our son
will probably not need to have any more, and will be fine."

Answer: C

Rationale: The child's family suffers stress for a number of reasons. In this
situation, the caregiver felt guilt about the illness. The cause of the illness, its
treatment, guilt about the illness, past experiences of illness and hospitalization,
disruption in family life, the threat to the child's long-term health, cultural or
religious influences, coping methods within the family, and financial impact of the
hospitalization all may affect how the family responds to the child's illness.
Although some of these are concerns of the family and not specifically the child,
they nevertheless influence how the child feels.
Question format: Multiple Choice
Chapter 29: Care of the Hospitalized Child
Cognitive Level: Apply
Client Needs: Psychosocial Integrity
Integrated Process: Teaching/Learning
Reference: p. 613

18. A team of nurses has been chosen to devise a program to help educate children
and their families about the hospital and being hospitalized. Which activity should
the nurses plan to best achieve this goal?
A. Ask the school nurse to do a presentation to groups of schoolchildren about what
it is like to be in the hospital.
B. Offer guided tours to an organized group such as Boy Scouts or Girl Scouts.
C. Plan an open house and invite the families of the community to come tour the
hospital.
D. Work with the local library to provide resources (books, pamphlets, or DVDs)
explaining the admission process and hospitalization, which the families can check
out and view together.

Answer: C

Rationale: The first choice should be working with the family and caregivers with
the open house. The family and caregivers have the most important role in
educating children about hospitals and hospitalization. The child trusts and feels
safer most often with the caregiver. During a tour of the hospital, a room is set
aside where children can handle equipment, try out call bells, try on masks and
gowns, have their blood pressure taken to feel the squeeze of the blood pressure
cuff, and see a hospital pediatric bed and compare it with their bed at home. The
caregivers can reinforce what the child has seen in the home setting. Not all
children are involved in an organized group such as Boy Scouts or Girl Scouts. The
library could be a potential source; however, with many homes now having internet
access, the same materials may be viewed online. This can reinforce what the child
saw at the open house.
Question format: Multiple Choice
Chapter 29: Care of the Hospitalized Child
Cognitive Level: Apply
Client Needs: Health Promotion and Maintenance
Integrated Process: Teaching/Learning
Reference: p. 613-614

19. The pediatric unit in the hospital is designed to make the child feel comfortable
and secure. Which action will best accomplish this goal?
A. The hallways are painted with bright colors and characters familiar to children.
B. Procedures and treatments are done in a room other than the child's hospital
room.
C. Staff members wear clothes that are a variety of colors, designs and patterns.
D. A playroom stocked with toys and activities for a variety of ages is readily
available.

Answer: B

Rationale: Treatments are performed in a treatment room, not in the child's room.
Using a separate room to perform procedures promotes the concept that the child's
bed is a safe place. All treatments, with no exceptions, should be performed in the
treatment room to reassure the child. The other options may make the pediatric
unit appear more pleasing to the child, but performing treatments in an area
separate from the bed will be the priority.
Question format: Multiple Choice
Chapter 29: Care of the Hospitalized Child
Cognitive Level: Apply
Client Needs: Psychosocial Integrity
Integrated Process: Nursing Process
Reference: p. 614

20. The nurse working in the pediatric unit of the hospital is always monitoring for
safety issues on the unit. The nurse determines the greatest concern related to
safety of the hospitalized child is related to which situation?
A. The child is out of the home environment he or she is used to, without safety
locks on doors and cabinets and things being placed in safe storage.
B. The caregivers are in a high-stress situation and are unable to concentrate and
pay attention to what the child is doing at all times.
C. Taking a child in and out of bed, doing frequent checks and procedures, and
caregivers sitting at the side of the bed all bring on an opportunity for the side rails
to be left down.
D. A variety of nursing staff and other hospital staff are in and out of the child's
room during hospitalization, and close observation of the child by the staff is
difficult.

Answer: C

Rationale: Hospital staff must encourage family members to keep crib sides/bed
rails up when not directly caring for an infant or child. One unguarded moment may
mean that the infant falls out of a crib or the child slides off the bed. Depending on
the age of the child, the use of safety locks and safe storage would also be
important at home. Although keeping track of the child at every waking moment
can be challenging, it would eliminate the need to maintain a safe environment. The
variety of nursing staff in and out of the room would enable closer observation due
to the many interactions; however, the priority here is to ensure the side rails are
up so the child cannot fall out of bed.
Question format: Multiple Choice
Chapter 29: Care of the Hospitalized Child
Cognitive Level: Apply
Client Needs: Safe, Effective Care Environment: Safety and Infection Control
Integrated Process: Nursing Process
Reference: p. 615

21. The nursing instructor is discussing the various modes of transmission for
pathogens. The instructor determines the session is successful when the students
correctly point out that microorganisms transmitted by food, water, or blood are
spread via which mechanism?
A. contact
B. vehicle
C. airborne
D. vector

Answer: B

Rationale: Microorganisms are spread by contact (direct, indirect, or droplet),


vehicle (food, water, blood, or contaminated products), air (dust particles in the
air), or vector (mosquitoes, vermin) means of transmission. Each type of
microorganism is transmitted in a specific way, so precautions are tailored to
prevent the spread of specific microorganisms.
Question format: Multiple Choice
Chapter 29: Care of the Hospitalized Child
Cognitive Level: Analyze
Client Needs: Safe, Effective Care Environment: Safety and Infection Control
Integrated Process: Teaching/Learning
Reference: p. 615

22. The nurse is preparing a presentation depicting the various ways pathogens are
spread around. The nurse will illustrate that pathogens passed around via insects is
referred to as which mechanism?
A. contact
B. vehicle
C. airborne
D. vector

Answer: D

Rationale: Microorganisms are spread by contact (direct, indirect, or droplet),


vehicle (food, water, blood, or contaminated products), air (dust particles in the
air), or vector (mosquitoes, vermin) means of transmission. Each type of
microorganism is transmitted in a specific way, so precautions are tailored to
prevent the spread of specific microorganisms.
Question format: Multiple Choice
Chapter 29: Care of the Hospitalized Child
Cognitive Level: Apply
Client Needs: Safe, Effective Care Environment: Safety and Infection Control
Integrated Process: Teaching/Learning
Reference: p. 615

23. The nurse must follow certain precautions in order to protect everyone from the
spread of disease. Which action will the nurse prioritize when utilizing
transmission-based precautions?
A. Precautions are followed if a client has a disease that can be easily spread.
B. Precautions apply when coming in contact with blood.
C. Precautions are followed if a child is immunosuppressed to protect them from
other children.
D. Precautions apply to contact with stool and urine.

Answer: A

Rationale: In children who are documented or suspected of having highly


transmissible pathogens, health care providers must follow transmission-based
precautions. Standard precautions blend the primary characteristics of universal
precautions and body substance isolation. Standard precautions apply to blood; all
body fluids, secretions, and excretions (except sweat); nonintact skin; and mucous
membranes. Standard precautions are intended to reduce the risk of transmission
of microorganisms from recognized or unrecognized sources of infection in
hospitals.
Question format: Multiple Choice
Chapter 29: Care of the Hospitalized Child
Cognitive Level: Apply
Client Needs: Safe, Effective Care Environment: Safety and Infection Control
Integrated Process: Nursing Process
Reference: p. 615

24. The nurse is preparing discharge teaching for a child and the caregivers after a
week of hospitalization. Which activity should the nurse prioritize in this plan?
A. Begin the teaching sessions just prior to the child leaving the facility.
B. Provide written instructions which can be taken home to read and follow.
C. Ensure the caregiver has assistance to handle the complex treatments at home.
D. Plan to review information and procedures with the family caregivers before
discharge.

Answer: D

Rationale: Shortly before the child is discharged from the hospital, it is important to
review information and procedures with the family caregivers. The nurse should not
wait until the client is almost out the door before starting discharge training.
Depending on the training, it can begin shortly after admission. Not all families will
need home health care to assist with transitioning from the hospital to the home
setting. Written instructions may not always be appropriate if the family cannot
read the literature or if it is not in their native language. The best option is to spend
time with the family, demonstrating what needs to be done and observing them
performing the various treatments.
Question format: Multiple Choice
Chapter 29: Care of the Hospitalized Child
Cognitive Level: Apply
Client Needs: Safe, Effective Care Environment: Management of Care
Integrated Process: Nursing Process
Reference: p. 620

25. The nurse is caring for a child who is recovering from a surgical procedure. The
nurse will contact the primary care provider if anuria is noted within which time
frame?
A. within the first 30 minutes after surgery
B. within the first hour after leaving PACU
C. within the first 2 hours following surgery
D. within the first 6 hours following surgery

Answer: D

Rationale: The first voiding is an important milestone in postoperative progress


because it indicates the adequacy of blood flow and urinary output. The nurse
should notify the primary care provider if a child is still experiencing anuria within
the first 6 hours after surgery.
Question format: Multiple Choice
Chapter 29: Care of the Hospitalized Child
Cognitive Level: Understand
Client Needs: Physiological Integrity: Physiological Adaptation
Integrated Process: Nursing Process
Reference: p. 623

26. The nurse is preparing a child for surgery and is demonstrating and evaluating
the child for using a patient-controlled analgesia device after surgery. The nurse
would determine which client would be most appropriate to use this device?
A. a 3-year-old who has a caregiver at the bedside at all times
B. a 6-year-old who is able to use the call light to call the nurse when needed
C. a 7-year-old who has told the nurse about using the control button
D. a 10-year-old with a history of behavior issues who has been compliant with care

Answer: C

Rationale: Patient-controlled analgesia may be used for children 7 years of age or


older who have no cognitive impairment and undergo a careful evaluation. The child
with a history of behavior issues may not be appropriate to use the PCA; however,
further evaluation would be needed to determine this.
Question format: Multiple Choice
Chapter 29: Care of the Hospitalized Child
Cognitive Level: Analyze
Client Needs: Physiological Integrity: Pharmacological and Parenteral Therapies
Integrated Process: Nursing Process
Reference: p. 623
27. The nurse is working with a child-life specialist to assist a young preadolescent
who is preparing for treatment for cancer. Which technique will the nurse and
specialist prioritize to assist this child in better understanding what will be
happening in the treatment of the cancer?
A. play therapy
B. therapeutic play
C. onlooker play
D. cooperative play

Answer: B

Rationale: Therapeutic play is a play technique used to help the child better
understand what will be happening to him or her in a specific situation. For
instance, the child who will be having an IV started before surgery might be given
the materials and encouraged to "start" an IV on a stuffed animal or doll. By
observing the child, you can often note concerns, fears, and anxieties the child
might express. Therapeutic play helps the child express feelings, fears, and
concerns. The other types of play will not accomplish this goal.
Question format: Multiple Choice
Chapter 29: Care of the Hospitalized Child
Cognitive Level: Apply
Client Needs: Psychosocial Integrity
Integrated Process: Nursing Process
Reference: p. 625-626
Chapter 30
1. The nurse is caring for a 5-year-old hospitalized child who needs to have blood
drawn for the third time. When the lab technician arrives, the child says to the
nurse, "You can't take my blood, my mama isn't here." The child's caregiver is not
in the hospital and is not expected to return for several hours. Which action taken
by the nurse would best meet the needs of the child while supporting medical
treatment?
A. Check with the floor administrator to see if the blood draw can wait until the
caregiver returns.
B. Recognize the statement is a stalling technique, reassure the child, and continue
with the procedure.
C. Tell the child that if he will let the phlebotomist take his blood without putting up
a fuss, he will get a lollipop.
D. Remind him that he has said that before and that he knows that the blood must
be taken.

Answer: B

Rationale: During any painful procedure, children must be allowed to cry if


necessary, should always be listened to, and have their questions answered. It
takes maturity and experience on the nurse's part to know exactly which questions
are stalling techniques and which call for firmness and action. Children need
someone to take charge in a kind, firm manner that tells them the decision is not in
their hands. They are too young to take this responsibility upon themselves.
Because the pain caused by the procedure has nothing to do with the child's
behavior, if a reward is given, it should not be a reward for being brave or good or
mature; it is simply a part of the entire treatment. The reward will mitigate the
unpleasant part of the experience.
Question format: Multiple Choice
Chapter 30: Procedures and Treatments
Cognitive Level: Apply
Client Needs: Psychosocial Integrity
Integrated Process: Caring
Reference: p. 632

2. Tess is a 5-year-old client who must receive an IV infusion of antibiotics. She is


anxious, resistant, and wiggly. To keep her safe during the time the IV is in place,
the nurse would choose which method to restrain her?
A. Restrain her with a mummy restraint and loosen and rewrap it every 3 hours.
B. Restrain her on a papoose board and release her as soon as the IV is in place.
C. Use a clove-hitch restraint to keep her arm still and loosen it every 2 hours.
D. Allow her caregiver to hold her during the time the IV is in place.

Answer: C
Rationale: Restraints often are needed to protect a child from injury during a
procedure or an examination, or to ensure the infant's or child's safety and comfort.
Clove-hitch restraints are used to secure an arm or leg, most often when a child is
receiving an IV infusion. The site should be checked and loosened at least every 2
hours. A mummy restraint can be used for an infant or small child during a
procedure. This device is a snug wrap that is effective when performing a scalp
venipuncture, inserting a nasogastric tube, or performing other procedures that
involve only the head or neck. Papoose boards are used with toddlers or
preschoolers but are not as appropriate as a clove-hitch restraint for this procedure.
Question format: Multiple Choice
Chapter 30: Procedures and Treatments
Cognitive Level: Apply
Client Needs: Physiological Integrity: Reduction of Risk Potential
Integrated Process: Nursing Process
Reference: p. 633

3. The nurse is teaching a couple who are first-time parents. They are being
discharged with their newborn. The mother tells the nurse she has heard about
sudden infant death syndrome and is worried about that happening to her baby. In
teaching about the prevention of SIDS, the nurse teaches the new parents to:
A. let the newborn sleep on his stomach in their bed.
B. prop the baby on his side or put him to sleep on his back.
C. lay him on his stomach with a small pillow under his abdomen.
D. prop him up with a pillow so his head is at a slightly elevated angle.

Answer: B

Rationale: Infants should be positioned on their backs or supported on their sides


for sleeping. The nurse working with family caregivers teaches and reinforces this
information. These positions seem to have decreased the incidence of sudden infant
death syndrome (formerly called "crib death"). Railings or sides will help prevent
falls.
Question format: Multiple Choice
Chapter 30: Procedures and Treatments
Cognitive Level: Understand
Client Needs: Health Promotion and Maintenance
Integrated Process: Teaching/Learning
Reference: p. 635

4. The nurse is teaching a group of first-time parents who are being discharged
with their newborns. One parent asks the nurse what to do if the child has a
temperature. After conducting teaching regarding how to care for a child with an
elevated temperature, the caregivers make the following statements. Which
statement would indicate a need for further teaching?
A. "A rectal temperature above 102.5 ℉ (39.1 ℉) should be lowered."
B. "Giving extra fluids is the way I have always heard to lower a temperature."
C. "I don't plan to give my child medications, but the pediatrician might tell us to
give our child acetaminophen every 4 to 6 hours if she has a fever."
D. "If my child starts to shiver I will know that what I am doing is working and that
her fever will soon come down."

Answer: D

Rationale: "Normal" body temperature varies from 97.6 ℉ (36.4 ℃) orally to 100.3
℉ (37.9 ℃) rectally. The body temperature generally should be maintained below
101℉ (38.3 ℃) orally or 102℉ (38.9 ℃) rectally, although the health care facility or
practitioner may set lower limits. Methods used to reduce fever include maintaining
hydration by encouraging fluids and administering acetaminophen. Excess
coverings should be removed from the child with a fever to permit additional
cooling through evaporation. Changing to lightweight clothes, removing clothes,
lowering the room temperature, or applying cool compresses to the forehead may
help to lower the temperature. If a child begins to shiver, whatever is being done to
lower the temperature should be stopped. Shivering indicates the child is chilling,
which will cause the body temperature to increase again. Because of their
ineffectiveness in reducing fever and the discomfort they cause, tepid sponge baths
are no longer recommended for reducing fever.
Question format: Multiple Choice
Chapter 30: Procedures and Treatments
Cognitive Level: Analyze
Client Needs: Physiological Integrity: Reduction of Risk Potential
Integrated Process: Teaching/Learning
Reference: p. 636

5. The nurse is assigned to care for a child diagnosed with a chronic illness. The
child has just been admitted but has been on the unit many times before. From the
report the admitting nurse gives, the child is sicker than the last time she was
admitted. In planning the child's care, the nurse notes that the provider has
ordered a nasogastric gavage feeding, but the nurse remembers that even the last
time the child was on the unit, she was unable to tolerate the nasogastric feedings.
The most appropriate nursing action would be for the nurse to:
A. begin the nasogastric gavage feeding to see if the child can tolerate it.
B. begin an orogastric gavage in hopes the child can handle the feeding.
C. ask the nursing supervisor to decide which type of feeding to give.
D. talk with the health care provider and request further instruction and orders.

Answer: D

Rationale: Sometimes infants or children who have had surgery or have a chronic
or serious condition are unable to take adequate food and fluid by mouth and must
receive nourishment by means of nasogastric or orogastric gavage feedings.
Gavage feedings provide nourishment directly through a tube passed into the
stomach. This procedure is particularly appropriate in infants but also may be used
in the older child. If gavage feedings are not well tolerated, the nurse should report
it and await alternate orders from the provider.
Question format: Multiple Choice
Chapter 30: Procedures and Treatments
Cognitive Level: Understand
Client Needs: Safe, Effective Care Environment: Management of Care
Integrated Process: Nursing Process
Reference: p. 636-637

6. The nurse is conducting teaching with the caregivers of a child who is being
discharged from the pediatric unit. The care provider has recommended the child
have moist heat applications at home. In conducting teaching with this caregiver,
the nurse will teach the caregiver to use which of the following to provide the moist
heat?
A. an electric heating pad
B. towels dampened and heated in the microwave
C. a hot water bottle
D. towels dampened with hot water

Answer: D

Rationale: Towels can be used to provide moist heat. They should not be warmed in
the microwave because the microwave may unevenly heat the towels, which in turn
may burn the child. Dry heat may be applied by means of an electric heating pad, a
K-pad (a unit that circulates warm water through plastic-enclosed tubing), or a hot-
water bottle. Many children, however, have been burned because of the improper
use of hot-water bottles; therefore, these devices are not recommended.
Question format: Multiple Choice
Chapter 30: Procedures and Treatments
Cognitive Level: Apply
Client Needs: Physiological Integrity: Reduction of Risk Potential
Integrated Process: Teaching/Learning
Reference: p. 641

7. The nurse is caring for a child with an ileostomy. What nursing intervention will
be included in this child's plan of care?
A. Clean the outside of the collection device.
B. Check for leakage around the stoma.
C. Leave the ileostomy open to the air.
D. Apply a sterile dressing around the stoma.

Answer: B

Rationale: An ileostomy is made by bringing a part of the small intestine through


the abdominal wall to create an outlet for fecal material. The drainage from the
ileostomy contains digestive enzymes, so the stoma must be fitted with a collection
device to prevent skin irritation and breakdown. A colostomy is a similar opening in
the colon that allows fecal material to be eliminated. A new colostomy may be left
open to the air; alternatively a bag, pouch, or other appliance may be used to
collect the stool. A urostomy may be created to help in the elimination of urine.
Ostomy bags should be checked for leakage, emptied frequently, and changed
when needed.
Question format: Multiple Choice
Chapter 30: Procedures and Treatments
Cognitive Level: Apply
Client Needs: Physiological Integrity: Physiological Adaptation
Integrated Process: Nursing Process
Reference: p. 642

8. A urine specimen has been ordered for a 1-year-old girl. Which method would be
the best way for the nurse to obtain this urine specimen?
A. Clean the child's genital area thoroughly and when she has urinated, squeeze the
urine from her diaper into a specimen cup.
B. Give the child some water or juice, leave off her diaper, ask the caregiver to call
you when the child needs to void, and obtain the specimen in a sterile container.
C. Place a sterile cotton ball into the child's diaper; after the child has urinated,
squeeze the urine from the cotton ball into a sterile container to be sent to the lab.
D. Get down on the child's level and speak to her explaining that you need her to
tell you when she needs to use the bathroom and when she does, obtain the
specimen.

Answer: C

Rationale: If a urine specimen is needed for diagnostic purposes, a cotton ball can
be placed in the diaper of an infant; the urine squeezed from the cotton ball can be
collected and used for many urine tests. Because toddlers and young children
cannot usually void on command, they should be offered fluids 15 to 20 minutes
before the urine specimen is needed. In preparation for collecting a urine specimen,
the infant or child is positioned so that the genitalia are exposed and the area can
be cleansed. In the female patient, the labia majora are cleansed front to back
using one cotton pad for each wipe. The labia minora are then exposed and
cleansed in the same fashion. The area is rinsed with a cotton ball saturated with
sterile water.
Question format: Multiple Choice
Chapter 30: Procedures and Treatments
Cognitive Level: Apply
Client Needs: Physiological Integrity: Reduction of Risk Potential
Integrated Process: Nursing Process
Reference: p. 642

9. The nurse is helping assist the care provider in doing a lumbar puncture on a 9-
month-old infant. Which method will be the safest and most appropriate method of
restraining the child?
A. Hold the child snugly against the chest, pass one arm below the child's legs; with
the other arm, hold the child's hands.
B. Wrap the child in a mummy restraint, then place your hands over the infant's
hands tightly on top of the restraint.
C. Show the caregiver how to hold the child in a seated position securely on the
caregiver's lap.
D. Use a jacket restraint to keep the child snugly held to a bed or gurney.
Answer: A

Rationale: When analysis of cerebrospinal fluid is necessary, a lumbar puncture is


performed. During this procedure, the nurse must restrain the client until the
procedure is completed. Grasp the child's hands with the hand that you have
passed under the child's lower extremities and hold the child snugly against your
chest. This position enlarges the intervertebral spaces for easier access with the
aspiration needle. A mummy board can be used for procedures that involve only the
head and neck. Safety belts are used if a child is being transported from one place
to another.
Question format: Multiple Choice
Chapter 30: Procedures and Treatments
Cognitive Level: Apply
Client Needs: Physiological Integrity: Reduction of Risk Potential
Integrated Process: Nursing Process
Reference: p. 644

10. The nurse is helping the care provider with a scalp venipuncture on an infant.
Which of the following restraints would be most appropriate for this procedure?
A. mummy restraint
B. clove-hitch restraint
C. elbow restraint
D. jacket restraint

Answer: A

Rationale: Mummy restraints are snug wraps used to restrain an infant or small
child during a procedure. This device is effective when performing a scalp
venipuncture, inserting a nasogastric tube, or performing other procedures that
involve only the head or neck.
Question format: Multiple Choice
Chapter 30: Procedures and Treatments
Cognitive Level: Apply
Client Needs: Safe, Effective Care Environment: Safety and Infection Control
Integrated Process: Nursing Process
Reference: p. 633

11. The nurse is caring for a preschool-aged child who is used to being able to get
out of bed at any time. Which of the following restraints would be the most
appropriate for the nurse to ensure the child remains in the bed?
A. mummy restraint
B. clove-hitch restraint
C. elbow restraint
D. jacket restraint

Answer: D
Rationale: Jacket restraints are used to secure the child from climbing out of bed or
a chair or to keep the child in a horizontal position.
Question format: Multiple Choice
Chapter 30: Procedures and Treatments
Cognitive Level: Apply
Client Needs: Safe, Effective Care Environment: Safety and Infection Control
Integrated Process: Nursing Process
Reference: p. 634

12. The nurse is caring for a child with an elevated temperature. Which of the
following nursing interventions would be the highest priority in caring for this
child?
A. The nurse should dress the child in lightweight clothing.
B. The nurse should monitor the child for seizure activity.
C. The nurse should encourage oral fluids at frequent intervals.
D. The nurse should keep the temperature of the child's room cool.

Answer: B

Rationale: The highest priority is to monitor the child for any seizure activity. The
other interventions are appropriate but not as high a priority as monitoring for
seizures.
Question format: Multiple Choice
Chapter 30: Procedures and Treatments
Cognitive Level: Apply
Client Needs: Physiological Integrity: Reduction of Risk Potential
Integrated Process: Nursing Process
Reference: p. 635

13. The nurse is administering a gavage feeding through a nasogastric feeding


tube. Which nursing intervention is the highest priority?
A. The nurse positions the child in a sitting position.
B. The nurse verifies the position of the feeding tube.
C. The nurse replaces stomach content that has been aspirated.
D. The nurse documents how the child tolerated the feeding.

Answer: B

Rationale: Verifying the position of the tube to ensure that the tube is in the
stomach by aspirating stomach contents is the highest priority. This is a top priority
because of the danger of aspiration if the tube is not in the stomach but rather in
the esophagus or the lung.
Question format: Multiple Choice
Chapter 30: Procedures and Treatments
Cognitive Level: Apply
Client Needs: Physiological Integrity: Basic Care and Comfort
Integrated Process: Nursing Process
Reference: p. 637
14. After the provider has written a prescription for the use of heat therapy, the
nurse will apply heat using a K-pad or heating pad as ordered. Which of the
following is most accurate regarding the use of heat therapy?
A. Heat is a vasoconstrictor and decreases circulation.
B. Heat may be used to prevent swelling.
C. Heat should be used 1 hour at a time and then removed.
D. Heat causes muscle relaxation and decreases pain.

Answer: D

Rationale: The local application of heat increases circulation by vasodilatation and


promotes muscle relaxation, thereby relieving pain and congestion.
Question format: Multiple Choice
Chapter 30: Procedures and Treatments
Cognitive Level: Apply
Client Needs: Physiological Integrity: Basic Care and Comfort
Integrated Process: Nursing Process
Reference: p. 641

15. As part of the admitting process for a child on the pediatric unit, the nurse
instills a small amount of saline into the nose, aspirates, and then places the
solution into a sterile specimen container. What is the most likely reason this will
be done?
A. to diagnose an infection
B. to clear the nasal passages
C. to remove a foreign body
D. to detect the presence of old blood

Answer: A

Rationale: Specimens from the nose and throat are used to help diagnose infection.
Question format: Multiple Choice
Chapter 30: Procedures and Treatments
Cognitive Level: Understand
Client Needs: Physiological Integrity: Reduction of Risk Potential
Integrated Process: Nursing Process
Reference: p. 642

16. A toddler is being discharged home with oxygen by nasal cannula. In educating
the parents about oxygen safety, which statement by the father indicates that
further teaching is needed?
A. "We will keep all flammable materials out of his room."
B. "They showed us how to wash the equipment to keep down bacteria."
C. "I will use only cotton blankets in his bed for sleeping."
D. "We will let him play with his Leap Frog tablet to keep him entertained."

Answer: D
Rationale: When a client is on oxygen, all electric or battery-powered toys need to
be removed from the bed to avoid the possibility of sparking and a fire.
Question format: Multiple Choice
Chapter 30: Procedures and Treatments
Cognitive Level: Apply
Client Needs: Physiological Integrity: Pharmacological and Parenteral Therapies
Integrated Process: Nursing Process
Reference: p. 640

17. Under which conditions would the nurse be justified in restraining a child in the
hospital? Select all that apply.
A. The child is a toddler and continues to attempt to crawl out of the crib.
B. The mother has to go home for a short time and there is no one to sit in the
room with the child.
C. The parents request that their child be restrained.
D. The child had a new gastrostomy tube placed today.
E. The preschool-aged client needs to have an IV started.

Answer: A, D

Rationale: The nurse only uses restraints when it is necessary for safety reasons,
and when no other option is available. Restraints must be ordered by the physician
and are not applied per parent request. If a young client is attempting to climb over
the top of the crib railings, a jacket restraint or a crib top restraint are needed to
keep the child from falling out. Also, with new gastrostomy tube insertions, the
nurse needs to prevent the child from pulling it out, so restraints are necessary.
The nurse cannot use restraints in place of a caregiver for procedures.
Question format: Multiple Select
Chapter 30: Procedures and Treatments
Cognitive Level: Understand
Client Needs: Safe, Effective Care Environment: Safety and Infection Control
Integrated Process: Nursing Process
Reference: p. 633

18. A young client has a temperature of 102 ℃ (38.9 ℃). In addition to the
scheduled antipyretic the child received, the nurse is treating the fever with
nonpharmacologic methods by removing the blanket that covers the child. What is
the rationale for this action?
A. The blanket adds heat to the child.
B. Removing excess coverings allows for evaporation, which aids in cooling the
child.
C. The blanket soaks up the sweat, making the child warmer.
D. Covering the child with a blanket leads to shivering, which will only generate
more heat.

Answer: B
Rationale: Removing covering from a child when he or she is febrile aids in
additional cooling by allowing evaporative heat loss. The rationale is not that the
blanket warms the child further, or that it will soak up sweat. Nor will covering the
child lead to shivering. Shivering occurs when the child is chilled by some
intervention; shivering serves as a sign that the cooling procedure needs to be
stopped.
Question format: Multiple Choice
Chapter 30: Procedures and Treatments
Cognitive Level: Apply
Client Needs: Physiological Integrity: Physiological Adaptation
Integrated Process: Nursing Process
Reference: p. 635-636

19. A child on oxygen reports a "sore nose" and the nurse assesses that the child is
experiencing dry nasal passages. What action can the nurse take to help relieve the
child's discomfort?
A. Turn the oxygen flow rate down to reduce the amount of air passing through the
child's nose.
B. Apply a petroleum-based lubricant such as Vaseline on and around the nose to
ease the discomfort.
C. Place lotion on a cotton swab and gently place some inside each nostril to
moisturize the nasal passages.
D. Use a water-based gel or spray for lubrication of the dry nasal passages.

Answer: D

Rationale: Any time a client is on oxygen, he or she needs to be instructed to use a


non-petroleum or water-based product for dry nasal passages. Petroleum products
greatly increase the risk of catching fire since they are so flammable. The nurse
cannot reduce the amount of needed oxygen for the client by turning down the flow
rate nor should the nurse insert lotion into the nasal passages.
Question format: Multiple Choice
Chapter 30: Procedures and Treatments
Cognitive Level: Apply
Client Needs: Physiological Integrity: Physiological Adaptation
Integrated Process: Nursing Process
Reference: p. 639

20. A 5-year-old client is scheduled to have an influenza injection before being


discharged from the hospital. When the nurse explains what she has to do, the
client begins to cry and asks the nurse if she can have the shot after her movie is
over in 2 hours. Which response by the nurse would be the best choice for the
client?
A. Agree to postpone the injection until after the movie is done.
B. Tell the child that you need to give her the injection now.
C. Negotiate with the child to give the injection in 1 hour.
D. Offer to contact the pediatrician's office to have it given at the next visit.
Answer: B

Rationale: Children often try to postpone frightening or painful procedures by


stalling. The nurse needs to take responsibility for making the decisions of care and
when and what needs to be done. In this case, the best response would be to tell
the client that you need to give the injection now and get it over with. Postponing
events that the child dreads only heightens their anxiety.
Question format: Multiple Choice
Chapter 30: Procedures and Treatments
Cognitive Level: Apply
Client Needs: Physiological Integrity: Pharmacological and Parenteral Therapies
Integrated Process: Nursing Process
Reference: p. 632

21. A child is undergoing a painful procedure and is upset. Which statement by the
nurse would be the best approach in dealing with the child?
A. "If you hold still and be quiet, I will give you a popsicle."
B. "You were brave and good, so you get a sucker."
C. "I know that this hurts some but you are being so strong. It is OK to cry."
D. "Please don't bite or kick me; that would be very naughty."

Answer: C

Rationale: Children should be given the right to cry and be verbally praised for
cooperating. Pediatric clients should not routinely be rewarded for acting
appropriately during a procedure or for being brave or good, but if they are given a
small reward such as a sticker or small toy afterward, the child's memory of the
experience is more positive. A nurse never tells a child to be quiet during a painful
procedure nor tells the child that he/she is naughty for acting out in pain.
Question format: Multiple Choice
Chapter 30: Procedures and Treatments
Cognitive Level: Apply
Client Needs: Psychosocial Integrity
Integrated Process: Caring
Reference: p. 632

22. The charge nurse is explaining the use of a papoose restraint to a newly hired
nurse in the emergency department. Which client would the charge nurse
recommend using the papoose board to restrain?
A. a school-aged child having stitches on the arm
B. a toddler who is having a nasogastric tube placed
C. an infant who needs a suppository
D. a preschool-age child needing his fractured leg set

Answer: B

Rationale: The papoose board is used for the restraint of toddlers and preschool-
age children who need to have procedures performed on their head or neck, such
as a venipuncture, passage of a nasogastric tube, or stitches for a scalp laceration.
Older children do not need restraining in such a manner and infants receiving rectal
medications are never restrained. When a child is having a fracture set, he or she is
not restrained.
Question format: Multiple Choice
Chapter 30: Procedures and Treatments
Cognitive Level: Apply
Client Needs: Safe, Effective Care Environment: Safety and Infection Control
Integrated Process: Teaching/Learning
Reference: p. 633

23. An 8-month old infant has a colostomy placed following abdominal surgery for
removal of a section of bowel. The stoma is 2/3 full, draining liquid stool and the
bag appears inflated. What actions would the nurse take in caring for this client?
Select all that apply.
A. Remove the stoma bag and discard it every day.
B. Empty the bag and record the output.
C. Examine the skin around the stoma site for redness or irritation.
D. Look at the infant's intake to determine if any foods could be causing gas.
E. Remove the stoma bag and allow the stoma to remain open to air for 1 to 2
hours.

Answer: B, C, D

Rationale: In caring for an infant with a colostomy, the nurse empties the contents
of the bag on a regular basis, rinses it out, closes it again, but does not discard it
each time it is emptied. The nurse also inspects the skin around the stoma for any
redness or skin breakdown. Since the bag was inflated initially, the nurse should
review the infant's intake to note if the infant is consuming any gas-causing foods
and recommend limiting them. The bag is never left off for any length of time due
to the constant stooling pattern of the infant.
Question format: Multiple Select
Chapter 30: Procedures and Treatments
Cognitive Level: Apply
Client Needs: Physiological Integrity: Physiological Adaptation
Integrated Process: Nursing Process
Reference: p. 642

24. A nurse is discussing post-procedure interventions with new pediatric nurses.


Which statement addresses the most immediate safety action required?
A. "Remove all equipment related to the procedure from the child's environment."
B. "Handle all contaminated linens in accordance with the facility policies."
C. "Assess to ensure that the side rails are up and the bed is lowered to the floor."
D. "Document the procedure and the response of the child as soon as the procedure
is completed."

Answer: C
Rationale: When the procedure is completed, leave the child in a safe position with
side rails raised and bed lowered. For the older child, place the call light within
reach. While the other options are safety related, the child's immediate safety
relates to the side rails and bed position.
Question format: Multiple Choice
Chapter 30: Procedures and Treatments
Cognitive Level: Apply
Client Needs: Safe, Effective Care Environment: Safety and Infection Control
Integrated Process: Nursing Process
Reference: p. 632

25. The nurse demonstrates appropriate knowledge when making what statement
related to gavage feedings of a child?
A. "A gastrostomy tube is a tube passed into the stomach through the nose or
mouth."
B. "An enteral tube is surgically inserted through the abdominal wall into the
stomach."
C. "A bolus feeding is given continuously at a slow rate of speed. "
D. "For long-term gavage feedings, a gastrostomy button may be inserted."

Answer: A

Rationale: Gavage feeding, also called enteral tube feeding, provides nourishment
directly through a tube passed into the stomach through the nose or mouth. These
feedings may be given intermittently as a bolus feeding or may be given
continuously at a slower rate over a longer period of time. If feedings will be given
continuously, a gastrostomy tube, surgically inserted through the abdominal wall
into the stomach, may be considered. For long-term gastrostomy feedings, a
gastrostomy button may be inserted. Some advantages of buttons are that they are
more desirable cosmetically, are simple to care for, and cause less skin irritation.
Question format: Multiple Choice
Chapter 30: Procedures and Treatments
Cognitive Level: Understand
Client Needs: Physiological Integrity: Basic Care and Comfort
Integrated Process: Nursing Process
Reference: p. 636

26. The nurse is presenting a post-conference seminar to a group of nursing


students on the topic of nasal/oral suctioning of a child. Which statement made by
a student demonstrates a need for further instruction on the procedure?
A. "The purpose of this form of suctioning is to removes secretions from the nose
and mouth."
B. "Sterile normal saline drops are used to loosen the dried secretions prior to nasal
suctioning."
C. "Such suturing is only done with a bulb syringe."
D. "If possible, the child should be asked to cough before suctioning."

Answer: C
Rationale: Excess secretions in the nose or mouth can obstruct the infant's or
child's airway and decrease respiratory function. Coughing often clears the airway,
but when the infant or child is unable to remove secretions, they are removed by
suctioning. A bulb syringe can remove secretions from the nose and mouth but
nasotracheal suctioning with a sterile suction catheter may be needed if secretions
cannot be removed by the bulb syringe. Sterile normal saline drops may be used to
loosen dried nasal secretions.
Question format: Multiple Choice
Chapter 30: Procedures and Treatments
Cognitive Level: Analyze
Client Needs: Physiological Integrity: Physiological Adaptation
Integrated Process: Teaching/Learning
Reference: p. 640
Chapter 31
1. The nurse at a camp for children with asthma is teaching these children about
the medications they are taking and how to properly take them. The nurse
recognizes that many medications used on a daily basis for the treatment of asthma
are given by which method?
A. Directly into the vein
B. Through a gastrostomy tube
C. Using a nebulizer
D. Sprinkled onto the food

Answer: C

Rationale: Many of these drugs used in the treatment of asthma can be given either
by a nebulizer (tube attached to a wall unit or cylinder that delivers moist air via a
face mask) or a metered-dose inhaler ([MDI], which is a hand-held plastic device
that delivers a premeasured dose). Emergency medications are given
intravenously. Most children do not have a gastrostomy tube, and medications
sprinkled on foods are given with cystic fibrosis.
Question format: Multiple Choice
Chapter 31: Medication Administration and Intravenous Therapy
Cognitive Level: Apply
Client Needs: Physiological Integrity: Pharmacological and Parenteral Therapies
Integrated Process: Nursing Process
Reference: p. 751

2. In caring for the child with asthma, the nurse recognizes that bronchodilator
medications are administered to children with asthma for which reason?
A. Relief of acute symptoms
B. Management of chronic pain
C. To stabilize the cell membranes
D. Prevention of mild symptoms

Answer: A

Rationale: Bronchodilators are used for quick relief of acute exacerbations of


asthma symptoms. Mast cell stabilizers help to stabilize the cell membrane by
preventing mast cells from releasing the chemical mediators that cause
bronchospasm and mucous membrane inflammation. Leukotriene inhibitors are
given by mouth along with other asthma medications for long-term control and
prevention of mild, persistent asthma. Bronchodilators are not effective for pain.
Question format: Multiple Choice
Chapter 31: Medication Administration and Intravenous Therapy
Cognitive Level: Apply
Client Needs: Physiological Integrity: Pharmacological and Parenteral Therapies
Integrated Process: Nursing Process
Reference: p. 751

3. The nurse has administered a medication to a child. When documenting the


dosage given, the nurse realizes that an incorrect dose has been administered. The
nurse has taken the child's vital signs, and they are within normal range. The child
does not appear to be in any distress and is acting normally. Which of the following
would be the next action for the nurse to take?
A. Give the correct dosage the next time the medication is given.
B. Monitor the child for any change in vital signs or behavior.
C. Follow the facility policy regarding medication errors.
D. Notify the next nurse who cares for this child that a dosage error has been
made.

Answer: C

Rationale: Medication errors can occur because nurses are human and not perfect.
To admit an error is often difficult, especially if there has been carelessness
concerning the rules. A person may be strongly tempted to adopt a "wait and see"
attitude, which is the gravest error of all. Nurses must accept responsibility for their
own actions. Serious consequences for the child may be avoided if a mistake is
disclosed promptly. Facility policies regarding medication errors should be followed.
Question format: Multiple Choice
Chapter 31: Medication Administration and Intravenous Therapy
Cognitive Level: Apply
Client Needs: Safe, Effective Care Environment: Safety and Infection Control
Integrated Process: Nursing Process
Reference: p. 650

4. The nurse is caring for a child who weighs 75 lb. The medication ordered for the
child has a therapeutic dosage range of 33 mg/kg per day to 48 mg/kg per day.
The medication ordered is to be given 4 times per day. Which dosages would be
appropriate for the nurse to administer to this child in one dose?
A. 28.0 mg per dose
B. 40.8 mg per dose
C. 250 mg per dose
D. 375 mg per dose

Answer: D

Rationale: One kilogram equals 2.2 lb.; therefore, a child weighing 75 lb weighs 34
kg. The low dose of this medication would be 34 X 33 = 1122 divided by 4 times a
day equals 280.5 mg per dose. The high dose of this medication would be 34 X 48
= 1632 divided by 4 times a day equals 408 mg per dose. Therefore, a dose of 375
mg per dose would be appropriate.
Question format: Multiple Choice
Chapter 31: Medication Administration and Intravenous Therapy
Cognitive Level: Analyze
Client Needs: Physiological Integrity: Pharmacological and Parenteral Therapies
Integrated Process: Nursing Process
Reference: p. 651

5. The nurse is caring for a child who weighs 31 kg. A medication is ordered for this
child with a dosage range of 20 to 40 mg per kg of body weight per dose. Which
dosage would be appropriate for the nurse to administer to this child in one dose?
A. 12.4 mg per dose
B. 62.0 mg per dose
C. 124.0 mg per dose
D. 1,000 mg per dose

Answer: D

Rationale: If a dosage range of 20 to 40 mg per kg of body weight is a safe dosage


range, and a child weighs 31 kg, the low dose of this medication would be 31 X 20
= 620. The high dose of this medication would be 31 X 40 = 1,240. Therefore, a
dose of 1,000 mg per dose would be appropriate.
Question format: Multiple Choice
Chapter 31: Medication Administration and Intravenous Therapy
Cognitive Level: Analyze
Client Needs: Physiological Integrity: Pharmacological and Parenteral Therapies
Integrated Process: Nursing Process
Reference: p. 651-653

6. Using the West nomogram scale, the nurse needs to calculate the safe dosage of
a medication for a child. The child is 50 inches tall and weighs 76 lb. The normal
dosage of the medication for an adult is 300 mg. Which of the following is the
correct way to use the West nomogram scale?
A. Locate the child's height and weight on the scale, multiply those two numbers,
and divide the adult dosage by the resulting number.
B. Use the scale to locate the child's height and weight. Use a straight edge to align
these numbers with the scale indicating the surface area, divide that by the
average adult body surface area, and multiply the resulting number by the adult
dose.
C. Locate the child's height and weight on the nomogram. Use a straight edge to
align these numbers with the scale indicating percentage of adult dosage and
multiply the adult dosage by this number.
D. Use the "shortcut" scale because this child is average. This will show the nurse
the percentage of the adult dosage appropriate for this child. Use this percentage to
calculate the dosage.

Answer: B

Rationale: The West nomogram, commonly used to calculate body surface area
(BSA), is a graph with several scales arranged so that when two values are known,
the third can be plotted by drawing a line with a straight edge. The child's weight is
marked on the right scale, the height on the left scale. Use a straight edge to draw
a line between the two marks. The point where the lines cross the column labeled
SA (surface area) is the child's BSA expressed in square meters. (A fourth scale on
the nomogram is a "shortcut" scale that can be used for children of average
proportions.) The average adult BSA is 1.73 square meters; thus, the way to
calculate the appropriate dosage for a child is to divide the child's BSA by 1.73 and
multiply the resulting number by the adult dose. Here, 1.15 divided by 1.73 equals
0.66. The result, 0.66, when multiplied by the adult dose (here, 300 mg), equals
198. Therefore, the correct dose for this child would be 198 mg.
Question format: Multiple Choice
Chapter 31: Medication Administration and Intravenous Therapy
Cognitive Level: Analyze
Client Needs: Physiological Integrity: Pharmacological and Parenteral Therapies
Integrated Process: Nursing Process
Reference: p. 651-653

7. The nurse is administering an oral medication that comes in an elixir form. When
giving the medication, the child begins to choke and sputter. The nurse confers with
the pharmacist about alternate ways to give the medication to this child. The
pharmacist might recommend that the nurse should:
A. dilute the elixir so that the medication is not as hard to swallow.
B. ask the care provider to change the order to give a drug that comes in a
suspension.
C. mix the medication with food so the medication will be easy to swallow.
D. give the medication in a glass of orange juice.

Answer: A

Rationale: Elixirs contain alcohol and are apt to cause choking unless they are
diluted. Syrups and suspensions are thick and may need dilution to ensure that the
child gets the full dose. Always check with the pharmacist before diluting any
medication. Do not use orange juice to give medication unless specifically ordered
to do so; otherwise, the child may always associate the taste of orange juice with
the unpleasant medicine. It is best not to mix medication in food in case the child
refuses to eat the food.
Question format: Multiple Choice
Chapter 31: Medication Administration and Intravenous Therapy
Cognitive Level: Understand
Client Needs: Physiological Integrity: Pharmacological and Parenteral Therapies
Integrated Process: Nursing Process
Reference: p. 654

8. The nurse has an order to administer a rectal suppository to a 3-month-old child.


In addition to lubricating the suppository, which intervention will help assure
appropriate administration of the medication?
A. Placing the child on the abdomen for insertion of the suppository
B. Pre-warming the suppository in the hand for 10 to 20 seconds
C. Using the index finger to insert the suppository into the rectum
D. Holding the buttocks tightly together for 1 to 2 minutes after insertion
Answer: D

Rationale: For the administration of rectal medications, the child is placed in a side-
lying position and the nurse must wear gloves or a finger cot. The suppository is
lubricated, then inserted into the rectum, followed by a finger, which is inserted up
to the first knuckle joint. The little finger should be used for insertion in infants.
After the insertion of the suppository, the buttocks must be held tightly together for
1 to 2 minutes until the child's urge to expel the suppository passes. The
suppository would begin to soften making it difficult to insert if warmed prior to
insertion.
Question format: Multiple Choice
Chapter 31: Medication Administration and Intravenous Therapy
Cognitive Level: Apply
Client Needs: Physiological Integrity: Pharmacological and Parenteral Therapies
Integrated Process: Nursing Process
Reference: p. 656

9. The nurse has been caring for a 12-year-old boy during his 5-day hospitalization.
The child's IV has infiltrated, and the care provider is getting ready to change the
intravenous line site. Which statement made by the nurse would be appropriate in
supporting the child?
A. "I will be back after your IV is in place."
B. "Would you like me to stay with you or are you OK alone?"
C. "The client is left-handed and likes to draw; an IV site in his right arm would be
best."
D. "The nurses on the unit know the client well, so maybe a nurse could start the
IV."

Answer: C

Rationale: The staff nurse may serve as the child's advocate when the care provider
comes to start an infusion. The staff nurse who has cared for the child has the
child's confidence and knows the child's preferences.
Question format: Multiple Choice
Chapter 31: Medication Administration and Intravenous Therapy
Cognitive Level: Apply
Client Needs: Safe, Effective Care Environment: Management of Care
Integrated Process: Nursing Process
Reference: p. 661

10. An order for an intramuscular medication has been ordered for a 4-year-old.
The nurse finds the child in the playroom with his caregiver. Which action by the
nurse would be the best procedure to follow when administering this medication?
A. Move him to a spot in the room where no other children are playing and give the
injection. Reward him with a lollipop or sticker if he cooperates. Complete all
required documentation.
B. Take him back to his bed to give him his injection. Let him go back to the
playroom if he doesn't cry. Complete all required documentation.
C. Take him to a separate treatment room to give him his injection. Praise his
cooperation even if he cries. Take him back to the playroom. Complete all required
documentation.
D. Ask the caregiver to let you know when he is finished playing and give him his
injection in a treatment room at that time. Document the time you actually give the
injection.

Answer: C

Rationale: Whenever possible, give injections and do treatments in the treatment


room. Keep the bed and playroom "safe" places for the child. Recording the
administration of the medication, especially PRN medications, is critical to avoiding
potential errors in medication administration. Documentation must be completed
after the administration of any medication.
Question format: Multiple Choice
Chapter 31: Medication Administration and Intravenous Therapy
Cognitive Level: Apply
Client Needs: Physiological Integrity: Pharmacological and Parenteral Therapies
Integrated Process: Nursing Process
Reference: p. 656

11. The nurse is administering a PRN pain medication to a child. What is the
highest priority for the nurse in this situation?
A. The nurse double-checks the medication calculation with another nurse.
B. The nurse checks the last time the medication was given.
C. The nurse asks the child to explain the pain the child is having.
D. The nurse documents the effect of the medication within 1 hour of
administration.

Answer: B

Rationale: When giving a PRN medication, always check the last time it was given
and clarify how much has been given during the past 24 hours. The other choices
are important but checking when and how much the child has had are the priorities.
Question format: Multiple Choice
Chapter 31: Medication Administration and Intravenous Therapy
Cognitive Level: Apply
Client Needs: Physiological Integrity: Pharmacological and Parenteral Therapies
Integrated Process: Nursing Process
Reference: p. 650

12. In understanding fluid and electrolyte balance, it is important for the nurse to
recognize that fluid contained within the body cells is known as:
A. intracellular
B. interstitial
C. extracellular
D. intravascular
Answer: A

Rationale: Intracellular fluid is contained within the body cells.


Question format: Multiple Choice
Chapter 31: Medication Administration and Intravenous Therapy
Cognitive Level: Remember
Client Needs: Physiological Integrity: Physiological Adaptation
Integrated Process: Nursing Process
Reference: p. 659

13. After the RN has initiated intravenous (IV) therapy on the child in the pediatric
unit, the newly licensed LPN is caring for the child. Which of the following is the LPN
responsible for related to the IV therapy? Select all that apply.
A. The LPN monitors the IV site for redness.
B. The LPN monitors the IV site for pain.
C. The LPN administers IV medications.
D. The LPN monitors the IV flow rate.
E. The LPN changes the IV site as needed.
F. The LPN monitors the site for swelling

Answer: A, B, D, F

Rationale: The child and the IV infusion require monitoring as frequently as every
hour. Check the IV site to see that it is intact and observe for redness, pain,
induration (hardness), flow rate, moisture at the site, and swelling. Documentation
is sometimes done on an IV flow sheet that lists the flow rate, the amount in the
bottle, the amount in the burette, the amount infused, and the condition of the site.
Although in some states the LPN can be specially trained to do IV therapy, a newly
licensed LPN is responsible for monitoring the child undergoing IV therapy but not
licensed to start IV therapy or to administer IV medications.
Question format: Multiple Select
Chapter 31: Medication Administration and Intravenous Therapy
Cognitive Level: Apply
Client Needs: Physiological Integrity: Pharmacological and Parenteral Therapies
Integrated Process: Nursing Process
Reference: p. 662

14. The nurse has brought a 3-year-old's oral medications into the room for
administration. Upon approaching the child, the nurse said, "I have your
medication. Would you rather have me hand it to you or Mommy?" In critiquing the
nurse's actions, which is most accurate?
A. The nurse's behavior is correct. Children are afraid of the nurse.
B. The nurse's behavior is correct. The nurse provided the child a choice between
two acceptable options with the outcome of taking the medication.
C. The nurse's behavior is incorrect. The nurse should have been firm in expecting
the child to take the medication.
D. The nurse's behavior is incorrect. The mother did not prepare the medication
and should not have administered the medication.
Answer: B

Rationale: The nurse is correct to offer a choice to the preschooler and then for the
mother to administer the medication, if chosen. This meets the developmental level
of autonomy. The nurse prepared the medication and the medication remained with
the nurse until handing it to the mother, who handed it to the child. The nurse
witnessed the medication administration and documents it. The nurse firmly
requires the medication to be taken but found a way for the child to take it that was
acceptable to the child and accomplished the goal.
Question format: Multiple Choice
Chapter 31: Medication Administration and Intravenous Therapy
Cognitive Level: Apply
Client Needs Pn: Physiological Integrity: Pharmacological Therapies
Client Needs: Physiological Integrity: Pharmacological and Parenteral Therapies
Integrated Process: Nursing Process
Reference: p. 652

15. The nurse is determining a pediatric dosage of medication using the West
nomogram for estimating body surface area (BSA). Which two known factors are on
the left and the right scales?
A. Use the height and milligrams of medication.
B. Use the weight and milliliters of medication.
C. Use the client's height and weight.
D. Use the milligrams and milliliters of medication.

Answer: C

Rationale: The West nomogram for estimating body surface area uses the
dimensions of the client to form a line that passes through a point determined as
the body surface area. Following that determination, the particulars of the
medication prescribed are used.
Question format: Multiple Choice
Chapter 31: Medication Administration and Intravenous Therapy
Cognitive Level: Understand
Client Needs Pn: Physiological Integrity: Pharmacological Therapies
Client Needs: Physiological Integrity: Pharmacological and Parenteral Therapies
Integrated Process: Nursing Process
Reference: p. 651-653

16. The nurse is preparing an injection of ceftriaxone sodium 500 mg IM for a 9-


month-old client. When assessing the best site for injection, which landmarks are
determined in preparation for injection?
A. trochanter and knee joint
B. anterior superior iliac spine, iliac crest, and greater trochanter
C. acromion process and armpit
D. posterior superior iliac spine, greater trochanter, and gluteus maximus
Answer: A

Rationale: When locating the trochanter and knee joint as landmarks, the nurse is
using the vastus lateralis site—the main injection site for infants. The ventrogluteal
site uses the anterior superior iliac spine, iliac crest, and greater trochanter. The
deltoid site uses the acromion process and armpit. The dorsogluteal site, which is
not recommended for the infant, uses the posterior superior iliac spine, greater
trochanter, and gluteus maximus.
Question format: Multiple Choice
Chapter 31: Medication Administration and Intravenous Therapy
Cognitive Level: Apply
Client Needs Pn: Physiological Integrity: Pharmacological Therapies
Client Needs: Physiological Integrity: Pharmacological and Parenteral Therapies
Integrated Process: Nursing Process
Reference: p. 657

17. The nurse is preparing an intravenous solution of D5 ¼ NS @ 20 ml/hour for a


6-month-old client. Which safety device will be added to protect against fluid
overload?
A. a secondary line
B. a volume control device
C. a syringe pump
D. pediatric IV tubing

Answer: B

Rationale: A volume control device (also called buretrol) is designed to deliver only
the amount of medication placed in the chamber. The nurse places no more than 2
hours of fluid within the device. Should the pump fail, the client will receive only the
fluid amount in the chamber, thus preventing fluid overload.
Question format: Multiple Choice
Chapter 31: Medication Administration and Intravenous Therapy
Cognitive Level: Apply
Client Needs Pn: Physiological Integrity: Pharmacological Therapies
Client Needs: Physiological Integrity: Pharmacological and Parenteral Therapies
Integrated Process: Nursing Process
Reference: p. 662

18. What is a fundamental rationale for the administration of intravenous therapy?


Select all that apply.
A. It is minimally invasive.
B. to maintain fluid and electrolyte balance
C. to have access to the intravenous route for testing
D. It makes medication administration easier for the nurse.
E. Medications and nutrients can be infused through the line.

Answer: B, C, E
Rationale: Intravenous therapy is commonly administered to maintain fluid and
electrolyte balance, to administer medications for therapeutic reasons and
diagnostic testing, and for a primary medication administration and nutrient
administration site. It is an invasive procedure that requires the advanced skill of
the nurse. The rationale for intravenous therapy is not about convenience for the
nurse.
Question format: Multiple Select
Chapter 31: Medication Administration and Intravenous Therapy
Cognitive Level: Understand
Client Needs Pn: Physiological Integrity: Pharmacological Therapies
Client Needs: Physiological Integrity: Pharmacological and Parenteral Therapies
Integrated Process: Nursing Process
Reference: p. 659

19. The nurse is preparing an intravenous infusion for a 3-month-old client with
dehydration secondary to gastroenteritis. Which condition or scenario puts this
infant at greatest risk for a higher volume of fluid loss?
A. the decreased body surface area (BSA) to the body fluid volume
B. greater fluid exchange caused by increased metabolic activity
C. only taking sips of fluids when the infant does not feel well
D. experiencing bouts of nausea throughout the day

Answer: B

Rationale: Infants are at high risk for fluid volume deficit due to greater fluid
exchange caused by increased metabolic activity. The infant is in a period of rapid
growth. The infant also has an increased BSA relative to the body fluid volume.
Because of this, larger quantities of fluid are lost through the skin. Taking sips of
fluids places the infant at risk but not the highest risk. The child can be nauseated
without fluid loss from vomiting.
Question format: Multiple Choice
Chapter 31: Medication Administration and Intravenous Therapy
Cognitive Level: Apply
Client Needs Pn: Physiological Integrity: Pharmacological Therapies
Client Needs: Physiological Integrity: Physiological Adaptation
Integrated Process: Nursing Process
Reference: p. 660

20. Which client does the nurse identify as in a state of alkalosis from
hyperventilation?
A. the client with a pH of 6.
B. the client with a pH of 7.3
C. the client with a pH of 7.45
D. the client with a pH of 7.55

Answer: D
Rationale: It is important to monitor acid–base balance in children as it can change
rapidly. The acidity of a solution is determined by hydrogen ions. Acid fluids are
typically lower than 7 and can occur in conditions such as diabetes, kidney failure,
and diarrhea. Normal pH is 7.35 to 7.45 and common with internal body fluids.
Alkalosis is pH >7.45 and exceeds normal limits in clients with hyperventilation.
Question format: Multiple Choice
Chapter 31: Medication Administration and Intravenous Therapy
Cognitive Level: Apply
Client Needs Pn: Physiological Integrity: Pharmacological Therapies
Client Needs: Physiological Integrity: Physiological Adaptation
Integrated Process: Nursing Process
Reference: p. 660

21. The nurse is caring for a pediatric client who requires vitamins, minerals, lipids
and amino acids through the circulatory system instead of the gastrointestinal tract.
Which type of intravenous therapy is anticipated for long-term therapy?
A. peripheral venous catheter infusions
B. intermittent fluids via a syringe pump
C. central venous catheter infusions
D. use of a short line catheter and volume control chamber

Answer: C

Rationale: Total parenteral nutrition (TPN) is the administration of dextrose, lipids,


amino acids, electrolytes, vitamins, minerals and trace elements through the
circulatory system. It is anticipated that long-term therapy will be completed via a
central venous access device. Peripheral devices and short catheters are for short-
term therapy. Syringe pumps are for small amounts of fluid over a period of time.
The volume control chamber prevents fluid overload from pump malfunction.
Question format: Multiple Choice
Chapter 31: Medication Administration and Intravenous Therapy
Cognitive Level: Apply
Client Needs Pn: Physiological Integrity: Pharmacological Therapies
Client Needs: Physiological Integrity: Pharmacological and Parenteral Therapies
Integrated Process: Nursing Process
Reference: p. 663

22. The caregivers of a 1-year-old tell the nurse they are frustrated because the
medication their child requires daily comes in pill form, which they have been
unable to get him to swallow. After clarifying with the pharmacist, what would be
appropriate advice for the nurse to give these caregivers to help them make sure
the child gets the full dose of medication each day?
A. Grind the tablet to a powder and give it to the child by spoon in the pureed peas
he loves to eat.
B. Dissolve the tablet in a small amount of water sweetened with corn syrup;
delivered by a syringe.
C. Hold him firmly on the caregiver's lap and insert the tablet beneath his tongue;
hold his mouth firmly shut for a minute or so until his urge to expel the tablet
passes.
D. Break the tablet into smaller pieces and put them beneath his tongue.
Remember that babies need to learn how to swallow, so if he thrusts the pill piece
out of his mouth, simply push it back in.

Answer: B

Rationale: It usually is best to give medicine in solution form to a small child.


Tablets, if used, must be dissolved in water. Do not use orange juice for a solvent
unless specifically ordered to do so; otherwise, the child may always associate the
taste of orange juice with the unpleasant medicine. If the medicine is bitter, corn
syrup may disguise the taste. The child may develop a dislike for corn syrup, but
that is not as problematic as would be developing a dislike for orange juice.
Medications should not be given in food because if the child does not consume the
entire amount of food, the dosage of medication will not be accurate. In addition, if
medication is given with food, the child may eventually associate the bad taste of
the medication with food and may refuse to eat that food. There is little excuse for
restraining a small child and forcing a medication down the child's throat. The child
can always have the last word and bring it up again. Also there is a danger of
aspiration. Of even greater importance are the antagonism and helplessness that
build up in the child subjected to such a procedure. A child's sense of dignity must
be respected as much as that of an adult.
Question format: Multiple Choice
Chapter 31: Medication Administration and Intravenous Therapy
Cognitive Level: Apply
Client Needs: Physiological Integrity: Pharmacological and Parenteral Therapies
Integrated Process: Nursing Process
Reference: p. 654

23. The nurse is administering oral medications to an 11-year-old child. Which


intervention should the nurse implement to encourage the child to be medication
adherent?
A. The nurse administers the oral medication using a syringe or dropper.
B. The nurse disguises the medication in a small amount of corn syrup.
C. The nurse involves the caregiver in the routine of administering the medications.
D. The nurse encourages the child to develop a chart of times when medications are
due.

Answer: D

Rationale: The child of this age is beginning to want more independence. They can
tell time and want to be a participant in their care and in the daily schedule, such
as in medication administration and other daily activities. Special interventions
related to delivery of the medication (syringe and flavoring) are not required of a
child this age. Involving the caregiver doesn't necessarily encourage adherence.
Question format: Multiple Choice
Chapter 31: Medication Administration and Intravenous Therapy
Cognitive Level: Apply
Client Needs: Physiological Integrity: Pharmacological and Parenteral Therapies
Integrated Process: Nursing Process
Reference: p. 655

24. The nurse is administering an intramuscular injection of an antibiotic to a 3-


month-old infant. Which would be the best site for the nurse to give this
medication?
A. vastus lateralis muscle
B. dorsogluteal muscle
C. deltoid muscle
D. ventrogluteal muscle

Answer: A

Rationale: The muscle preferred for intramuscular injections in the infant less than
7 months of age is the vastus lateralis, located on the thigh. The ventrogluteal and
deltoid are used in older children to adults. The dorsogluteal is no longer considered
suitable for an intramuscular injection due to the risk of injury to the sciatic nerve.
Question format: Multiple Choice
Chapter 31: Medication Administration and Intravenous Therapy
Cognitive Level: Understand
Client Needs: Physiological Integrity: Pharmacological and Parenteral Therapies
Integrated Process: Nursing Process
Reference: p. 657-658

25. The nurse is administering otic medication to a 22-month-old with a diagnosis


of otitis media. Which nursing action ensures that the medication is distributed
appropriately?
A. Place in a side-lying position
B. Pull the pinna down and back
C. Place a cotton ball in the outer ear
D. Pull the pinna up and back

Answer: B

Rationale: By making sure that the ear canal is straight for the medication to
progress to the tympanic membrane, the medication is distributed appropriately. It
is also appropriate to place the child in a side-lying position. Some nurses place a
cotton ball in the ear but that does not impact distribution of the medication. A child
over 3 years of age needs the pinna pulled up and back.
Question format: Multiple Choice
Chapter 31: Medication Administration and Intravenous Therapy
Cognitive Level: Understand
Client Needs: Physiological Integrity: Pharmacological and Parenteral Therapies
Integrated Process: Nursing Process
Reference: p. 656
26. The pediatric nurse recognizes that what statement is true regarding
medications administered via the intravenous route?
A. The medication is injected into the fatty tissue between the skin and the muscle.
B. Giving medications through the intravenous route is less traumatic than multiple
injections.
C. The medications are absorbed more slowly by being given intravenously.
D. Administering medications intravenously is safer than by other routes.

Answer: B

Rationale: Delivering medications intravenously is actually less traumatic than


administering multiple injections. An injection into the fatty tissue between the skin
and the muscle is a subcutaneous injection. Medication absorption is quickest via an
IV route. When performed properly, all routes of medication administration are
safe.
Question format: Multiple Choice
Chapter 31: Medication Administration and Intravenous Therapy
Cognitive Level: Understand
Client Needs: Physiological Integrity: Pharmacological and Parenteral Therapies
Integrated Process: Nursing Process
Reference: p. 659

27. A nurse has just performed the calculation to check the dosage of
amoxicillin/potassium clavulanate prescribed for a child. The calculation indicates
that the dosage is within the safe dosage range. Which action is appropriate for the
nurse to do next?
A. Call the health care provider to confirm the dosage is correct.
B. Have another staff nurse confirm the calculation.
C. Prepare to administer the medication.
D. Call the pharmacy to confirm the dose.

Answer: B

Rationale: After computing any dosage, the nurse should always have the
computation checked by another staff person qualified to give medication or by
someone in the department who is delegated for this purpose. Errors are easy to
make and easy to overlook. A second person should do the computation separately;
then both results should be compared. There is no need to call the health care
provider or pharmacy unless the calculation determined that there was a problem
with the dose. The dosage calculation needs to be confirmed by a second person
before the medication is administered.
Question format: Multiple Choice
Chapter 31: Medication Administration and Intravenous Therapy
Cognitive Level: Apply
Client Needs: Physiological Integrity: Pharmacological and Parenteral Therapies
Integrated Process: Nursing Process
Reference: p. 650-653
Chapter 32
1. The nurse caring for chronically ill Jared notices that the relationship between his
parents is quite strained. His father has been doing everything for him, while his
mother has been expecting him to have the same capabilities as his healthy twin.
Which statement would accurately describe this situation?
A. They are both in denial about the child's condition and will not be helpful to their
child unless they seek counseling.
B. Both parents are demonstrating normal responses, but they will be most helpful
to Jared if they can find a middle ground.
C. Jared's mother needs to recognize and support Jared's more limited capabilities.
D. Jared's father needs to let Jared do everything he is possibly able to do himself
so that he doesn't fall behind developmentally.

Answer: B

Rationale: Caregivers who respond with gradual acceptance take a common-sense


approach to the child's condition. They help the child to set realistic goals for self-
care and independence and encourage the child to achieve social and physical skills
within his or her capability. Caregivers responding with overprotection try to protect
the child at all costs: they hover – which prevents the child from learning new
skills, they fail to use discipline, and they use any means to prevent the child from
experiencing any frustration. Caregivers in denial behave as though the condition
does not exist and they encourage the child to overcompensate for any disabilities.
Question format: Multiple Choice
Chapter 32: The Child with a Chronic Health Problem
Cognitive Level: Apply
Client Needs: Psychosocial Integrity
Integrated Process: Caring
Reference: p. 669-670

2. A pediatric nurse is caring for a group of 4-year-old clients with various


diagnoses. The child being treated for which diagnosis will most need help
understanding what is happening to him or her?
A. burns sustained in a fireworks accident
B. broken bones sustained in a fall
C. lung and spleen damage sustained in a car accident
D. a rash suspected of being measles

Answer: C

Rationale: The perception of illness is subject to magical thinking for children until
they are 7 or 8. They have more difficulty understanding illness when "what's
wrong" is inside rather than outside.
Question format: Multiple Choice
Chapter 32: The Child with a Chronic Health Problem
Cognitive Level: Apply
Client Needs: Psychosocial Integrity
Integrated Process: Caring
Reference: p. 670

3. The caregivers of 8-year-old Alicia, diagnosed with a chronic illness, are talking
with the nurse about how they plan to care for their child at home. In discussing
the dietary aspects of her treatment, the caregivers tell the nurse that the entire
family is going to follow her diet. In addition, time will be spent each day focusing
on the child's growth and development, and tasks will be given to each of her
siblings to help in the care of the ill child as well as to help the home run smoothly
while the caregivers focus on Alicia. Which statement would be most appropriate
for the nurse to respond to the caregivers?
A. "That's wonderful. She is lucky to have caregivers who are so willing to make
whatever sacrifices necessary in caring for her."
B. "That sounds really effective. Is it necessary for all the family members to follow
the same diet?"
C. "It sounds like you have a good plan. It is really great that her siblings will be
picking up extra tasks as a way to support their sister."
D. "You are very devoted. Siblings often feel guilty or jealous; be sure that time is
planned to spend with those children as well."

Answer: D

Rationale: By overemphasizing the ill child's needs, some family members


unknowingly create feelings of guilt in the healthy children. It is important for the
caregivers to spend time with and do special activities with all of the children in the
family. Explaining one sibling's chronic illness to the other siblings helps them to
better understand the situation.
Question format: Multiple Choice
Chapter 32: The Child with a Chronic Health Problem
Cognitive Level: Apply
Client Needs: Psychosocial Integrity
Integrated Process: Caring
Reference: p. 671

4. The nurse is working with a 12-year-old who is hospitalized with a chronic


illness. Which action by the nurse might help the chronically ill preteen thrive while
hospitalized?
A. Encourage the client to wear his or her own clothes, talk to friends on the phone,
and interact with other clients who have similar illnesses.
B. Make all treatment and care decisions; the preteen is too young to have any
responsibility for his or her own care.
C. Create a clear list of behavioral rules to give the client when he or she arrives.
D. Encourage the client to keep his or her limitations foremost in mind when trying
a new skill or task at which his or her peers have begun to excel.

Answer: A
Rationale: Family caregivers may become overprotective, which can prevent an ill
child from exhibiting growth and development appropriate for his or her age and
disability. Encourage the child to wear regular clothes, rather than stay in pajamas,
to reduce feelings of being an invalid. Help the caregivers recognize the child's
potential and set realistic growth and development goals. Set age-appropriate limits
and enforce appropriate discipline. Accomplish this gradually by displaying a kind
and caring attitude. Give the child choices within the limits of treatments and other
aspects of required care. Introducing the child to other children with the same or a
similar condition can help dispel feelings that he or she is the only person with such
a condition. An older child or adolescent can benefit from social interaction with
peers with and without disabilities.
Question format: Multiple Choice
Chapter 32: The Child with a Chronic Health Problem
Cognitive Level: Apply
Client Needs: Psychosocial Integrity
Integrated Process: Caring
Reference: p. 672

5. The parents of a newly admitted client on the pediatric unit are angry and upset
about their child's diagnosis. The child has been diagnosed with a serious chronic
illness. One parent is weeping, asking, "Are they sure?" The other is pounding his
fist into his hand, saying over and over, "It's not fair. I love him too much. It's not
fair." Which of the following would be an appropriate response for the nurse in this
situation?
A. Stay in the room but say nothing.
B. Leave them alone for a while until they calm down.
C. Tell them that most children do fine for years with the illness and explain that
they will adjust to the child's special needs.
D. Acknowledge their sadness and their anger and let them know that the staff is
available for their support.

Answer: D

Rationale: Denial is usually the first reaction that family caregivers have to the
diagnosis. This is a time when they say, "How could this be?" or "Why my child?"
Let them express their emotions and respond in a nonjudgmental way. Staying with
them and offering quiet, accepting support may be helpful. Statements such as "It
will seem better in time" are inappropriate. Acknowledge the caregivers' feelings as
acceptable and reasonable.
Question format: Multiple Choice
Chapter 32: The Child with a Chronic Health Problem
Cognitive Level: Apply
Client Needs: Psychosocial Integrity
Integrated Process: Caring
Reference: p. 674
6. The primary caregiver of Eddy, a 10-year-old child, is his single grandfather.
Over several months of inpatient and outpatient treatment, the nurse notices that
the grandfather seems increasingly tired and stressed. Which statement would be
most appropriate for the nurse to suggest to this grandfather as a possible course
of action for himself and the child?
A. "When Eddy is feeling well enough, you and Eddy should go on a short vacation."
B. "You would probably benefit from some counseling from the hospital's clinical
social worker."
C. "It is important for you to take a break from caring for Eddy. I can give you
information about the hospital's respite care program."
D. "Going out with friends a few times a week might be beneficial in helping you
cope with Eddy's condition."

Answer: C

Rationale: A child who requires constant or frequent attention often can be


exhausting for the family caregivers. The family with no close extended family and
few close friends may find getting away for rest and relaxation, even for an
evening, almost impossible. Help the family caregivers find resources for respite
care. Any caregiver, no matter how devoted, needs to have a break from everyday
cares and concerns.
Question format: Multiple Choice
Chapter 32: The Child with a Chronic Health Problem
Cognitive Level: Apply
Client Needs: Psychosocial Integrity
Integrated Process: Caring
Reference: p. 673

7. The nurse who regularly cares for chronically ill children will be most effective in
helping the family adjust to the child's condition when they:
A. have had experience with a chronically ill child in his or her own family.
B. understand the full scope of the medical issues involved with each illness.
C. understand the medication issues and the psychosocial implications of the illness
on the child.
D. are aware of the implications of the disease and the resources available in the
community for the child and family.

Answer: D

Rationale: When a family member has a chronic illness, the entire family is affected
in many ways. Chronic illness may affect the child's physical, psychosocial, and
cognitive development. Because nurses are usually involved from the early stages
of diagnosis, and because the child and family have ongoing and long-term needs,
the nurse can play a vital role in helping the family adjust to the condition. It is
important for the nurse to know the resources available for the child and family.
Question format: Multiple Choice
Chapter 32: The Child with a Chronic Health Problem
Cognitive Level: Understand
Client Needs: Safe, Effective Care Environment: Management of Care
Integrated Process: Caring
Reference: p. 674

8. Which is the best determination of the family's ability to manage a chronic


condition?
A. financial stability
B. outcome of condition
C. family's coping ability
D. knowledge base

Answer: C

Rationale: The effects of a chronic condition affect a family in a variety of ways.


How families cope depends on their support of each other. Families who have
strong support systems usually are better able to meet the challenges of work,
family and caring for a child with a chronic illness. Financial stability is nice to have
but not the most important factor. Outcome of condition and knowledge base
affects how the family feels and reacts but is not the best determination of the
ability to manage the chronic condition.
Question format: Multiple Choice
Chapter 32: The Child with a Chronic Health Problem
Cognitive Level: Apply
Client Needs: Psychosocial Integrity
Integrated Process: Nursing Process
Reference: p. 669

9. The nurse is caring for a mother and 12-year-old who is newly diagnosed with a
chronic pediatric condition. When the nurse is instructing on the injection technique
needed for care, the mother rises and takes the equipment stating, "I will care for
him." Which behavior is recognized?
A. overprotection
B. dominance
C. age-appropriate parenting
D. ensuring safe and appropriate care

Answer: A

Rationale: The nurse must consider the ability of the client to manage his/her care.
A 12-year-old should be able to manage care and is encouraged to do so.
Caregivers responding with overprotection try to protect the client at all times. They
hover, prevent the child from learning new skills, and use any means to prevent the
client from experiencing any frustration. Dominance is a controlling behavior. To
assisting her child to be independent in an age-appropriate activity and to ensure
safe care, the mother must be knowledgeable and oversee the injection process.
Question format: Multiple Choice
Chapter 32: The Child with a Chronic Health Problem
Cognitive Level: Understand
Client Needs: Physiological Integrity: Physiological Adaptation
Integrated Process: Teaching/Learning
Reference: p. 669

10. The nurse is volunteering at a respite care weekend camp for children with a
chronic condition. This provides the caregivers with:
A. financial strain due to the costliness of the programs.
B. time to enjoy family activities that the child cannot participate in.
C. a chance for rest and refreshment.
D. the opportunity to complete chores at home.

Answer: C

Rationale: Respite care affords caregivers of chronically ill children a period of rest
and refreshment. Respite care assists single caregivers and married parents who
need to reconnect with others or reduce stress. The camps are typically affordable
due to working with associations and having volunteers. It is not usual to send
children away and then have a family activity. This would make the child feel
excluded. It is nice to get "caught up" on activities but this is not the focus of
respite care.
Question format: Multiple Choice
Chapter 32: The Child with a Chronic Health Problem
Cognitive Level: Apply
Client Needs: Health Promotion and Maintenance
Integrated Process: Nursing Process
Reference: p. 670

11. Which action by the nurse is the helpful if the parents of a child with a chronic
condition are experiencing financial difficulty? Select all that apply.
A. Provide information on state financial assistance.
B. Arrange a lunch tray to be sent for the parents.
C. Volunteer in a fundraiser for outstanding debts.
D. Ask the health care provider to decrease compensation costs.
E. Offer personal financial assistance to the family.

Answer: A, B, C

Rationale: The nurse has many opportunities to assist clients and their families;
however, the nurse must remember that this client is one of many. The nurse can
provide information on local contacts for state assistance, provide tangible
assistance immediately by arranging a lunch tray, and, if the nurse is able,
volunteer in a fundraiser. It is not appropriate for the nurse to ask the health care
provider to decrease his/her compensation or to offer personal financial assistance.
Question format: Multiple Select
Chapter 32: The Child with a Chronic Health Problem
Cognitive Level: Understand
Client Needs: Safe, Effective Care Environment: Management of Care
Integrated Process: Caring
Reference: p. 674

12. Which does the nurse identify as the best indicator of how a child will react to
the diagnosis of a chronic condition?
A. how the family reacts to the news
B. how the health care provider explains the condition
C. how much the child understands the condition
D. how much medical intervention is needed

Answer: A

Rationale: The child looks to the parent for guidance in areas they do not
understand; thus, how a parent reacts to the chronic condition influences how the
child reacts—not how the health care provider explains the condition or how much
the child understands or even understanding the medical intervention needed.
Question format: Multiple Choice
Chapter 32: The Child with a Chronic Health Problem
Cognitive Level: Apply
Client Needs: Psychosocial Integrity
Integrated Process: Nursing Process
Reference: p. 670

13. Which scenario does the nurse anticipate for the life of an adolescent with a
chronic condition who has deteriorating function?
A. feelings of being special due to decline
B. independence in medication administration
C. decrease in high school grades
D. decline in involvement with peer activities

Answer: D

Rationale: Peers can be very supportive of the client with deteriorating function;
however, at this stage of development, the adolescent is typically activity involved
in sports and school activities. The adolescent with a chronic condition may not
physically be able to be included in such activities, thus there is a decline in
involvement. Special accommodations need to be made for the client but these
typically do not make the client feel special. With a decrease in function, an
independent adolescent may need assistance in medication administration. The
adolescent with a chronic condition will focus on what he/she is able to do, which
often includes focusing on schoolwork. Also, most conditions do not include a
decrease in cognitive function.
Question format: Multiple Choice
Chapter 32: The Child with a Chronic Health Problem
Cognitive Level: Apply
Client Needs: Psychosocial Integrity
Integrated Process: Nursing Process
Reference: p. 672
14. The nurse is interacting with a family of 3 (parent, school-aged child with a
chronic illness, and preschool sibling) in a specialty clinic office. The parent is
struggling with the sibling being disruptive when preparing the child with a chronic
illness for the exam. Which nursing action is most helpful?
A. Remove the sibling from the room to a secure playroom area.
B. Support the parent by telling the sibling that the behavior is unacceptable.
C. Offer stickers and coloring books for the sibling to play with.
D. Encourage the sibling to assist in the preparation for the exam.

Answer: C

Rationale: Some degree of sibling rivalry is found in all families. Anger, resentment
and jealousy can occur due to the amount of time and attention that a child with a
chronic condition requires. Having an age-appropriate activity, given to the child by
the health care provider, provides positive attention to that child and allows focus
on the exam. It is uncomfortable for a preschooler to be removed from the parent.
The nurse must acknowledge the behavior but not tell the sibling that the behavior
is unacceptable. The child may not be interested in assisting in preparation for the
exam.
Question format: Multiple Choice
Chapter 32: The Child with a Chronic Health Problem
Cognitive Level: Apply
Client Needs: Psychosocial Integrity
Integrated Process: Nursing Process
Reference: p. 671

15. The nurse is completing a family assessment. Which behavior best highlights a
positive interaction between the younger child with a chronic condition and an older
sibling?
A. The older sibling allows the child to interact on the bus with friends.
B. The older sibling checks on the child before he attends basketball practice.
C. The older sibling organizes an outdoor game that all can participate in.
D. The older sibling acknowledges the child's struggles when talking with friends.

Answer: C

Rationale: All interactions are supportive of the child with a chronic illness, but the
best interaction is organizing an outdoor game which all can be involved in. This
promotes acceptance of the child with a chronic condition and interaction with
others. It also promotes sibling bonding and good psychosocial interaction.
Question format: Multiple Choice
Chapter 32: The Child with a Chronic Health Problem
Cognitive Level: Apply
Client Needs: Psychosocial Integrity
Integrated Process: Nursing Process
Reference: p. 671
16. Which role of the nurse is the highest priority when a child is newly diagnosed
with a chronic condition?
A. manager of diagnostic testing
B. facilitator of coping skills
C. educator of changes in lifestyle
D. coordinator of health care appointments

Answer: B

Rationale: The highest priority is to help the child and family cope with the news of
a newly diagnosed chronic illness. It is important to encourage the family to
express their feelings and reactions to the situation. The nurse is to support by
listening and formulating appropriate nursing interventions.
Question format: Multiple Choice
Chapter 32: The Child with a Chronic Health Problem
Cognitive Level: Apply
Client Needs: Psychosocial Integrity
Integrated Process: Nursing Process
Reference: p. 671-672

17. The nurse is caring for a child with a chronic condition who has advanced skills
and can perform activities of daily living independently. Which nursing goal is
progressing?
A. The client will meet developmental milestones.
B. The client will adjust to THE ROLE OF THE NURSEonic condition.
C. The client will work with parents to accomplish needs.
D. The client will improve his/her self-esteem.

Answer: B

Rationale: Major goals for the child are to accomplish growth and develop
milestones, perform self-care tasks, decrease anxiety and improve social
interaction. The nursing goal in progress is that the client is learning to adjust to
living with a chronic condition by being able to complete tasks such as performing
activities of daily living. The task of performing activities of daily living is not
aligned with the outcomes of meeting developmental milestones, working with
parents, or improving self-esteem.
Question format: Multiple Choice
Chapter 32: The Child with a Chronic Health Problem
Cognitive Level: Apply
Client Needs: Health Promotion and Maintenance
Integrated Process:
Reference: p. 672

18. When caring for a preschooler with a chronic condition, which nursing action
encourages client cooperation with daily tasks?
A. Explain tasks thoroughly.
B. Give the child a choice in participation.
C. Provide information with pictures.
D. Integrate play into tasks.

Answer: D

Rationale: To obtain cooperation from a preschooler, it is best to incorporate play


into the task. If the preschooler believes that the task is fun, he/she will participate
willingly. The preschooler is unable to understand the need for medical tasks, so
thorough explanations may scare the child. Some tasks must be completed, taking
away the choice in participation. Pictures of the tasks can be helpful and used for
client understanding.
Question format: Multiple Choice
Chapter 32: The Child with a Chronic Health Problem
Cognitive Level: Apply
Client Needs: Health Promotion and Maintenance
Integrated Process: Nursing Process
Reference: p. 673

19. The nurse is assigned four clients in the pediatric unit. Which child with a
chronic condition does the nurse anticipate having the nursing diagnosis, Impaired
physical mobility?
A. the child with cystic fibrosis
B. the child with sickle cell anemia
C. the child with muscular dystrophy
D. the child with hemophilia

Answer: C

Rationale: The child with muscular dystrophy experiences progressive muscle


weakness and loss of muscle mass. Eventually, the child will have an impaired
physical mobility. The child with cystic fibrosis would have a respiratory nursing
diagnosis. The child with sickle cell anemia would have a pain nursing diagnosis.
The child with hemophilia would have a fluid volume deficiency nursing diagnosis.
Question format: Multiple Choice
Chapter 32: The Child with a Chronic Health Problem
Cognitive Level: Apply
Client Needs: Safe, Effective Care Environment: Management of Care
Integrated Process: Nursing Process
Reference: p. 668-669

20. Which issue would a nurse likely identify as related to caregiving of a child with
a chronic illness?
A. anxiety
B. depression
C. potential social isolation
D. situational low self-esteem

Answer: C
Rationale: Social isolation is a common occurrence because of the demands of
parenting a child with a chronic condition. Having a child with a chronic condition
requires constant attention, which can be exhausting. All caregivers need to have a
break. Anxiety, depression and low self-esteem are not necessarily related to
caregiving.
Question format: Multiple Choice
Chapter 32: The Child with a Chronic Health Problem
Cognitive Level: Apply
Client Needs: Psychosocial Integrity
Integrated Process: Nursing Process
Reference: p. 673

21. The health care provider has advised respite care to assist the family of a child
with a chronic condition. Which would be most appropriate for the nurse to
facilitate?
A. an enterostomal therapist to visit the home
B. a home health aide scheduled in the morning
C. age- and activity-appropriate weekend camps
D. nutritional support from a registered dietitian

Answer: C

Rationale: When the health care provider has advised respite care, it is most helpful
for the nurse to facilitate an appropriate camp where the child can be well cared for
and the parents are able to have some rest. Camps provide activities for
socialization for the children as well as an opportunity for the parents to participate
in normal activities.
Question format: Multiple Choice
Chapter 32: The Child with a Chronic Health Problem
Cognitive Level: Apply
Client Needs: Psychosocial Integrity
Integrated Process: Nursing Process
Reference: p. 673-674

22. The parents and their child with a chronic illness have been seen by the nurse
in the pediatrician's office since birth. Which situation is appropriate for instruction
by the nurse?
A. diagnosis of secondary problems
B. growth and development changes
C. recent diagnostic testing and lab work
D. consents for upcoming surgery

Answer: B

Rationale: Nursing instruction for a pediatric client is often focused on growth and
development changes; that is no different in the child with a chronic condition.
Parents need to allow the child to develop and meet the milestones within the
child's capabilities. The other options are for the health care provider.
Question format: Multiple Choice
Chapter 32: The Child with a Chronic Health Problem
Cognitive Level: Apply
Client Needs: Physiological Integrity: Physiological Adaptation
Integrated Process: Teaching/Learning
Reference: p. 672

23. Which medical diagnosis does the nurse understand most often begins with an
acute injury and then turns into a chronic condition?
A. a fractured arm
B. low back pain
C. pneumonia
D. cerebral palsy

Answer: B

Rationale: Low back pain can begin with an acute episode (e.g., an accident) and
then become a chronic condition due to damage or degeneration. A fractured arm,
if fixed properly, is a self-limiting condition. Pneumonia is an acute condition that is
resolved with antibiotics. Cerebral palsy is a chronic condition from the diagnosis.
Question format: Multiple Choice
Chapter 32: The Child with a Chronic Health Problem
Cognitive Level: Apply
Client Needs: Physiological Integrity: Physiological Adaptation
Integrated Process: Nursing Process
Reference: p. 668-669

24. The nurse is working with a group of chronically ill school-aged children and
their caregivers. Which strategy would be most effective in helping the children
reach their self-care treatment goals?
A. Make a chart of self-care goals the child should do successfully; add a sticker to
the chart each time a goal is reached.
B. The caregiver and child co-write a contract that identifies what reward will be
earned when certain self-care treatment goals are mastered.
C. Frequently reinforce for the child why self-care goals are so important to their
overall health.
D. Encourage the child to socialize with healthy peers every day so that he or she is
motivated to become as independent as those peers.

Answer: B

Rationale: Use a chart or other visual aid with listed tasks as a tool to help the
younger child reach a desired goal. Stickers can also be used to record the younger
child's progress. School-age children often respond well to contracts: for example, a
special privilege or another incentive may be awarded when a set number of
stickers is earned. While the remaining options are appropriate actions, they are
not as likely to encourage self-care as effectively as the reward system.
Question format: Multiple Choice
Chapter 32: The Child with a Chronic Health Problem
Cognitive Level: Apply
Client Needs: Psychosocial Integrity
Integrated Process: Nursing Process
Reference: p. 673

25. An 8-year-old diagnosed with a chronic illness is returning home after an


extended hospital stay. While the child tires easily, she is mobile and active when
rested. Keeping in mind that the child is particularly interested in engaging in arts
and crafts, which intervention should the nurse suggest to meet the child's
development and socialization needs?
A. Finding a weekly children's group where there is a focus on arts and crafts
projects and a willingness to address her energy needs.
B. Finding an art teacher who can come to the home, after the child's afternoon
rest, to help nurture the child's interest in art.
C. Taking the child to the local library at least monthly to check out children's craft
books to support the interest in arts and crafts.
D. Identifying for the caregiver the tools and supplies the child will need to best
support her interest in arts and crafts when her energy level permits.

Answer: C

Rationale: The chronically ill child may feel isolated from peers. Ask the child about
his or her interests; these may give some clues about suitable after-school
activities that will increase the child's interactions with peers. Make suggestions and
confer with family caregivers to ensure that proposed plans are carried out.
Question format: Multiple Choice
Chapter 32: The Child with a Chronic Health Problem
Cognitive Level: Apply
Client Needs: Psychosocial Integrity
Integrated Process: Nursing Process
Reference: p. 672

26. During a well-child checkup, a child shares being embarrassed when she is seen
with her 7-year-old sister who wears an arm prosthesis after the limb was
amputated. Which statement by the nurse most appropriately addresses the child's
concerns?
A. "Your sister didn't want to lose her arm; you shouldn't feel embarrassed."
B. "That must be confusing, but it's important for your sister that you support her."
C. "That must be hard. I know you love your sister; it's normal for you to feel a
little embarrassed."
D. "Your sister probably feels more embarrassed than you do."

Answer: C
Rationale: Siblings may feel stigma (embarrassment or shame) because of a
brother or sister with a chronic illness, especially if the ill child has a physical
disfigurement or apparent cognitive deficit. It is appropriate to acknowledge these
feelings. The other options either minimize the child's concern or focus on the
sibling's needs rather than the child's feelings of embarrassment.
Question format: Multiple Choice
Chapter 32: The Child with a Chronic Health Problem
Cognitive Level: Apply
Client Needs: Psychosocial Integrity
Integrated Process: Nursing Process
Reference: p. 671
Chapter 33
1. A group of nurses is discussing child abuse (child maltreatment) and one nurse
suggests that inadequate parenting skills may lead to child maltreatment. The
nurse recognizes that sometimes child maltreatment occurs because the parent:
A. is responding appropriately to the child's behavior.
B. believes the child is intentionally trying to make him or her angry.
C. may have unrealistic expectations of the child.
D. works hard to provide for the family and feels unappreciated.

Answer: C

Rationale: Commonly, abusive parents have inadequate parenting skills; if they


have unrealistic expectations of the child, they may not respond appropriately to
the child's behavior.
Question format: Multiple Choice
Chapter 33: Abuse in the Family and Community
Cognitive Level: Apply
Client Needs: Psychosocial Integrity
Integrated Process: Teaching/Learning
Reference: p. 680

2. A caregiver who works in the hospital brings his 9-year-old son to the emergency
room with a spiral fracture of the tibia. The caregiver reports that the injury
occurred when the boy's 5-year-old sister hit him with wooden bat. The injury is
inconsistent with an impact and with the sister's strength. Which of the following
would be appropriate for the nurse to do in this situation?
A. Tell the caregiver that the story is not plausible and ask what really happened.
B. Leave the treatment area and call the police.
C. Leave the treatment area and call the social services department in the hospital.
D. Let the hospital administrator know so that the hospital can take any necessary
action.

Answer: C

Rationale: Abusive parents can be found at all socioeconomic levels. When a child is
brought to a physician or hospital with abuse injuries, family caregivers may
attribute the injury to an action of a sibling. Whenever the child's symptoms do not
match the injury the caregiver describes, be alert for possible abuse. State laws
require health care personnel to report suspected child abuse (child maltreatment).
This requirement overrides the concern for confidentiality. Laws have been enacted
that protect the nurse who reports suspected child abuse from reprisal by a
caregiver (e.g., being sued for slander) even if it is found that the child's situation
is not a result of abuse. If the nurse does not report suspected child abuse, the
penalty can be the loss of the nursing license. Do not accuse the caregiver before a
complete investigation takes place.
Question format: Multiple Choice
Chapter 33: Abuse in the Family and Community
Cognitive Level: Apply
Client Needs: Safe, Effective Care Environment: Management of Care
Integrated Process: Communication and Documentation
Reference: p. 680

3. Which situation should the nurse explore for possible emotional abuse of the
child by the caregiver?
A. The caregiver of a 13-year-old reports, "Amy is really frustrating me. I've been
late for work every day this week because she hasn't been ready to leave for school
on time."
B. The caregiver of a 16-year-old reports, "I couldn't believe that dinner wasn't
ready when I got home from work last night. Jack knew it was his turn to cook."
C. The caregiver of a 12-year-old reports, "I'm sorry we missed Jamal's
appointment last week, but he was supposed to remind me to come."
D. The caregiver of a 15-year-old reports, "Laia promised to earn the money to care
for the dog if we got him, but she keeps turning down baby-sitting jobs to go out
with her friends, and I have to add dog food to my already impossible budget."

Answer: C

Rationale: Injury from emotional abuse can be just as serious and lasting as that
from physical abuse, but it is much more difficult to identify. Several types of
emotional abuse can occur, including role reversal in which the child must take on
the role of parenting the parent and is blamed for the parent's problems.
Question format: Multiple Choice
Chapter 33: Abuse in the Family and Community
Cognitive Level: Understand
Client Needs: Psychosocial Integrity
Integrated Process: Nursing Process
Reference: p. 682

4. The nurse is assisting with a physical exam on a child who has been admitted
with a diagnosis of possible child abuse (child maltreatment). Which finding might
alert the nurse to this possibility that the child may have been abused?
A. The child has a fractured bone.
B. The child has bruises on the knees and elbows.
C. The child is hyperactive and angry.
D. The child has a burn that has not been treated.

Answer: D

Rationale: Burns are a common type of injury seen in the abused child. Although
burns may be accidental in young children, certain types of burns are highly
suspicious. Cigarette burns, or burns from immersion of a hand in hot liquid, from a
hot register (as evidenced by the grid pattern), from a steam iron, or from a curling
iron are common abuse injuries. Caregivers have been known to immerse the
buttocks of a child in hot water if they thought the child was uncooperative in toilet
training. Children often injure themselves and may have fractured bones or bruises
on knees and elbows that are not from child abuse. Hyperactivity and anger are not
physical signs of child abuse; they are emotional signs.
Question format: Multiple Choice
Chapter 33: Abuse in the Family and Community
Cognitive Level: Apply
Client Needs: Psychosocial Integrity
Integrated Process: Nursing Process
Reference: p. 680

5. The nurse is interviewing the caregiver of a 5-year-old child who has been
admitted with bruises on the abdomen and thighs as well as additional bruises in
various stages of healing. Which statement made by the caregiver might alert the
health care team to the possibility of child abuse (child maltreatment)?
A. "His brother just plays too rough with him."
B. "My child goes to day care after school."
C. "He just learned to ride his bicycle."
D. "When he is in trouble I make him go to his room."

Answer: A

Rationale: When a child is brought to a physician or hospital because of physical


injuries, family caregivers may attribute the injury to some action of the child's that
is not in keeping with the child's age or level of development. The caregiver may
also attribute the injury to an action of a sibling. When the child's symptoms do not
match the injury that the caregiver describes, be alert for possible abuse. The other
statements might be normal situations for a 5-year-old and would not necessarily
alert the nurse to a possibility of child abuse.
Question format: Multiple Choice
Chapter 33: Abuse in the Family and Community
Cognitive Level: Apply
Client Needs: Psychosocial Integrity
Integrated Process: Nursing Process
Reference: p. 680

6. In caring for a child who has been admitted after being sexually abused, which
intervention would be included in the child's plan of care?
A. Observe for signs of anxiety.
B. Weigh on the same scale each day.
C. Encourage frequent family visits.
D. Test the urine for glucose upon admission.

Answer: A

Rationale: The child who has been sexually abused may exhibit anxiety, acts of
aggression or hostility, or have physical complaints regarding various aches and
pains, gastrointestinal upsets, changes in bowel and bladder habits (including
enuresis), or nightmares. In cases of incest it is important that the perpetrator not
be allowed to visit the child. Weighing the child and testing urine for glucose are not
indicated for a child who has been sexually abused.
Question format: Multiple Choice
Chapter 33: Abuse in the Family and Community
Cognitive Level: Apply
Client Needs: Safe, Effective Care Environment: Management of Care
Integrated Process: Nursing Process
Reference: p. 684

7. The nurse is caring for a small child who has been admitted with a diagnosis of
abusive head trauma (shaken baby syndrome). Which condition or concern often
occurs with shaken baby syndrome?
A. edema to the arms and legs
B. loss of vision and intellectual disability
C. chronic infection and scarring
D. reports of illness and nightmares

Answer: B

Rationale: When the child is shaken, a whiplash-type injury occurs to the neck. In
addition, the child may have edema to the brain stem and retinal or brain
hemorrhages. Loss of vision, intellectual disability, or even death may occur in
children diagnosed with abusive head trauma (shaken baby syndrome). Reports of
illness and nightmares may occur with children who are emotionally abused.
Peripheral edema, infection, and scarring are not concerns noted in children
diagnosed with abusive head trauma (shaken baby syndrome).
Question format: Multiple Choice
Chapter 33: Abuse in the Family and Community
Cognitive Level: Apply
Client Needs: Physiological Integrity: Physiological Adaptation
Integrated Process: Nursing Process
Reference: p. 681

8. A group of pediatric nurses is discussing the diagnosis of Munchausen syndrome


by proxy. Which statement most accurately describes this disorder?
A. "It is seen when one person fabricates or induces illness in another to get
attention."
B. "The disorder occurs when a child is shaken by the arms or shoulders in a
repetitive manner."
C. "Children with this diagnosis often appear worried or fearful and have
nightmares."
D. "It happens when there is sexual contact between a child and a caregiver."

Answer: A

Rationale: In caregiver-fabricated illness (formerly Munchausen syndrome by


proxy), one person either fabricates or induces illness in another to get attention.
When a caregiver has this syndrome, he or she frequently brings the child to a
health care facility and reports symptoms of illness when the child is actually well.
Abusive head trauma (shaken baby syndrome) occurs when a child is shaken by the
arms or shoulders in a repetitive manner. Children who are emotionally abused
often appear worried and fearful, and have nightmares. When there is sexual
contact between a child and a caregiver, it is classified as sexual abuse.
Question format: Multiple Choice
Chapter 33: Abuse in the Family and Community
Cognitive Level: Apply
Client Needs: Physiological Integrity: Physiological Adaptation
Integrated Process: Nursing Process
Reference: p. 681

9. The nurse is assessing an infant diagnosed with abusive head trauma (shaken
baby syndrome). Clinical manifestations of shaken baby syndrome affect which
body system the most?
A. musculoskeletal system
B. cardiovascular system
C. respiratory system
D. nervous system

Answer: D

Rationale: The nervous system of the infant is most affected by abusive head
trauma (shaken baby syndrome). When a caregiver shakes the infant, the brain is
jarred in the skull. Clinical manifestations affecting the nervous system include
irritability, lethargy, vomiting, and seizures. Bruising may be noted on the skin.
Question format: Multiple Choice
Chapter 33: Abuse in the Family and Community
Cognitive Level: Understand
Client Needs: Psychosocial Integrity
Integrated Process: Nursing Process
Reference: p. 681

10. The health care provider is unsure of physical symptoms and diagnosis of a
pediatric client. The health care provider is prescribing diagnostic testing to
determine possible poisoning by the parent. When the parent asks why laboratory
work is being drawn, which response is best?
A. "Since we have not found the cause of the illness yet, the health care provider
has ordered more blood tests."
B. "We are drawing laboratory work to rule out poison in the child's system."
C. "I am not really sure at this time. I just saw it on the prescription sheet."
D. "It is hospital policy to draw laboratory work daily to monitor the client status."

Answer: A

Rationale: Informing the parent of the laboratory work prescribed is appropriate.


This response is telling the truth but does not alarm the parent. Telling the parent
that the tests are to rule out poison in the child's system could anger and alarm the
parent. The other options either lie to the parent or do not answer the question.
Question format: Multiple Choice
Chapter 33: Abuse in the Family and Community
Cognitive Level: Apply
Client Needs: Psychosocial Integrity
Integrated Process: Communication and Documentation
Reference: p. 684

11. The nurse on a pediatric unit is caring for a client who has been physically and
emotionally abused. Which adjustment in care is most appropriate?
A. Only draw laboratory work in the treatment room.
B. Allow the nonabusive caregiver to remain with the client.
C. Assign consistent nursing staff to the client when possible.
D. Provide play and distraction techniques to relieve stress.

Answer: C

Rationale: All of the options are appropriate when caring for a client who has been
physically and emotionally abused. The adjustment in care is assigning consistent
nursing staff. This aids in establishing therapeutic communication and trust. All
other options are standards of care on pediatric units.
Question format: Multiple Choice
Chapter 33: Abuse in the Family and Community
Cognitive Level: Apply
Client Needs: Psychosocial Integrity
Integrated Process: Caring
Reference: p. 684

12. The school nurse observes an elementary school child arriving in the winter in
shorts and a short sleeved shirt with no coat. The nurse would consider this as
which type of abuse?
A. physical abuse
B. emotional abuse
C. sexual abuse
D. neglect

Answer: D

Rationale: Not providing for the child's safety in winter conditions is an example of
neglect. Neglect is comprised of the failure to provide adequate hygiene, health
care, education, nutrition, love, nurturing, and supervision. Physical abuse includes
physical harm. Emotional abuse includes demeaning verbal threats. Sexual abuse
includes inappropriate contact between a child and caregiver.
Question format: Multiple Choice
Chapter 33: Abuse in the Family and Community
Cognitive Level: Apply
Client Needs: Psychosocial Integrity
Integrated Process: Nursing Process
Reference: p. 685

13. Which statement made by a participant at a workshop on sexual abuse needs


further clarification from the group leader?
A. "The difference between sexual abuse and sexual assault is the perpetrator."
B. "Sexual abuse by a person of trust is the most damaging type."
C. "Mostly low-income people commit sexual violence against children."
D. "In cases of incest, the child rarely tells anyone else."

Answer: C

Rationale: Clarification is needed because the statement regarding socioeconomic


status is incorrect. Sexual abuse of children exists in all age groups and cultures.
There is no socioeconomic, racial, religious, or ethnic boundaries. All of the other
statements are true and need no clarification.
Question format: Multiple Choice
Chapter 33: Abuse in the Family and Community
Cognitive Level: Analyze
Client Needs: Psychosocial Integrity
Integrated Process: Teaching/Learning
Reference: p. 683

14. A toddler is brought to the emergency department with burns on the feet and
buttocks. The toddler is screaming in pain. The caregiver states that she did not
know the bath water was so hot. What is the initial action completed by the nurse?
A. assessment of the toddler
B. notifying a health care provider
C. completing injury documentation
D. calling law enforcement

Answer: B

Rationale: When a toddler is screaming in pain, the health care provider is notified
at once. The nurse can assess the client with the health care provider. Verbal
orders can be obtained and implemented. The documentation is completed after
caring for the toddler. Law enforcement will be notified if needed. Most health care
facilities have security on staff.
Question format: Multiple Choice
Chapter 33: Abuse in the Family and Community
Cognitive Level: Apply
Client Needs: Safe, Effective Care Environment: Management of Care
Integrated Process: Nursing Process
Reference: p. 684

15. The nurse is caring for a young child whose parents have been charged with
physical abuse. Which child reaction is noted as a result of the parental behavior?
A. despondent and does not respond to questions
B. fretful and shrinks away when approached
C. aggressive and kicks and fights during the assessment
D. cries frequently and stares at the nurse

Answer: B

Rationale: Behaviors of the young child are intuitive. The child is fretful and shrinks
away when approached. This is related to the conduct of the parents as they would
punish the child. A young child may take time to trust the nurse. The nurse may
provide crayons and paper for drawing to get the child to open up about fears and
feelings. Otherwise, it may be difficult to communicate. Aggression can be common,
including temper tantrums. Typically the child will not make eye contact with the
nurse.
Question format: Multiple Choice
Chapter 33: Abuse in the Family and Community
Cognitive Level: Apply
Client Needs: Psychosocial Integrity
Integrated Process: Nursing Process
Reference: p. 684

16. The nurse is caring for a 4-year-old child with bruises on various areas of the
body. Which parental statement does the nurse note as most suspicious?
A. "The child plays aggressively with the siblings in the house."
B. "The child plays tackle football at the local school. He has advanced skills."
C. "The child bruises easily but was checked for a bleeding disorder."
D. "The child is clumsy when running and playing with older siblings."

Answer: B

Rationale: The most suspicious statement is related to playing tackle football at 4


years of age. Tackle football does not begin until the child is in school, not
preschool. This would make the nurse concerned about the accuracy of the
information. Education needs to be completed if the bruising is obtained from
playing with siblings or taking part in various activities outside of the child's
developmental level. The health care provider needs to be alerted to the various
bleeding, particularly if previous testing for a bleeding disorder is within the medical
record.
Question format: Multiple Choice
Chapter 33: Abuse in the Family and Community
Cognitive Level: Apply
Client Needs: Psychosocial Integrity
Integrated Process: Nursing Process
Reference: p. 680

17. Which type of abuse is reported most quickly to the police?


A. sexual assault
B. incest
C. neglect
D. emotional abuse

Answer: A

Rationale: Sexual assault is when sexual contact is by someone other than a


caregiver. When the caregivers become aware, the assault is quickly reported.
Incest is rarely reported quickly as the child does not tell another person what is
happening. Neglect and emotional abuse are more difficult to document and not as
easy to differentiate.
Question format: Multiple Choice
Chapter 33: Abuse in the Family and Community
Cognitive Level: Remember
Client Needs: Psychosocial Integrity
Integrated Process: Nursing Process
Reference: p. 683

18. Which is the primary physical sign of a child who the nurse suspects as being
emotionally abused?
A. poor hygiene
B. fatigue
C. delays in physical development
D. inadequate medical care

Answer: C

Rationale: Failure to thrive and delays in physical development are the primary
physical signs that the child is being emotionally abused. The nurse is correct to
assess the interaction between the caregiver and child. Poor hygiene, fatigue and
inadequate medical care indicate neglect.
Question format: Multiple Choice
Chapter 33: Abuse in the Family and Community
Cognitive Level: Understand
Client Needs: Psychosocial Integrity
Integrated Process: Nursing Process
Reference: p. 685

19. Which physical symptoms confirm that a child has been sexually abused or
assaulted? Select all that apply.
A. stained and bloody underclothing
B. The child states that someone touched the breasts.
C. inappropriate vaginal opening measurements
D. suspicious actions of the caregiver
E. sperm on body or clothes

Answer: A, C, E

Rationale: For confirmation that the child has been sexually abused or assaulted,
there would be evidence such as stained and bloody underclothing, inappropriate
vaginal opening indicating intercourse, and the physical presence of sperm.
Suspicion of abuse includes the child's statement and suspicious actions.
Question format: Multiple Select
Chapter 33: Abuse in the Family and Community
Cognitive Level: Apply
Client Needs: Psychosocial Integrity
Integrated Process: Nursing Process
Reference: p. 685

20. Which is essential in working with caregivers who have been affected by abuse?
A. knowing the legal system
B. developing a therapeutic relationship
C. providing details on social services in the community
D. arranging family meetings to discuss issues

Answer: B

Rationale: The first step in assisting caregivers and families affected by abuse is
developing a therapeutic relationship. This nonjudgmental relationship is essential
in assisting the family to make decisions and, without it, the family will discount
what the nurse is saying. Providing information on the legal system and social
services in the community is enabled after the therapeutic relationship has begun.
The family is ultimately responsible for arranging family meetings but it can be
suggested by the nurse.
Question format: Multiple Choice
Chapter 33: Abuse in the Family and Community
Cognitive Level: Apply
Client Needs: Psychosocial Integrity
Integrated Process: Nursing Process
Reference: p. 685-686

21. The nurse is admitting a pediatric client for a common pediatric surgical
procedure. When completing the domestic violence screening, which statements
require clarification? Select all that apply.
A. "My Dad yells a lot but never means it and doesn't hit anyone."
B. "We stay out of Mom's way when she is in a bad mood."
C. "I see my Dad drive by our new house to see who is here."
D. "I love my parents even though they are divorced."
E. "My Mom will hit my Dad when she is drunk but it doesn't hurt."

Answer: A, C, E

Rationale: There are many types of domestic abuse such as physical violence,
threats, emotional abuse, harassment, and stalking. Using this criteria of threats,
the nurse requires clarification when the child states that Dad yells a lot and doesn't
mean it. Driving by the new house could be harassment or stalking. The Mom
hitting the Dad but not hurting him is physical abuse. The other statements do not
imply domestic violence.
Question format: Multiple Select
Chapter 33: Abuse in the Family and Community
Cognitive Level: Analyze
Client Needs: Psychosocial Integrity
Integrated Process: Nursing Process
Reference: p. 686

22. Which adolescent behavior is most common if the parents have substance use
problems?
A. staying away from the house
B. taking on adult responsibilities
C. being clingy to teachers at school
D. telling everyone of the parental issues

Answer: B

Rationale: It is very common for the adolescent to take on parental roles in the
home hoping that the parents will change their behavior. These children do not
want negative attention to come to the family; thus, they keep to themselves and
do not tell others of the difficulty at home.
Question format: Multiple Choice
Chapter 33: Abuse in the Family and Community
Cognitive Level: Apply
Client Needs: Psychosocial Integrity
Integrated Process: Nursing Process
Reference: p. 688

23. The nurse has identified the outcome of improving parenting and coping skills in
a caregiver who has shown past poor judgement in caring for the children. Which
action promotes a positive image of his or her abilities?
A. educating on positive parenting skills
B. providing feedback on how to better cope with stressful situations
C. providing a compliment when a caregiver does something well
D. initiating a reward system when the parent accomplishes a goal

Answer: C

Rationale: It is best to compliment a caregiver on doing something well. This


improves self-esteem and makes the parent feel worthy of parenting. Educating
and providing feedback is the action of the nurse. The caregiver must have a desire
to improve for the family and not for rewards.
Question format: Multiple Choice
Chapter 33: Abuse in the Family and Community
Cognitive Level: Apply
Client Needs: Psychosocial Integrity
Integrated Process: Communication and Documentation
Reference: p. 685-686
24. A mother brings a 23-month-old child for a well-child check. She apologizes,
stating, "It's my allergies, not a cold, that is causing me to sniffle so much." The
mother reports that the toddler seems "slower at doing things than my older
children did." The nurse notices that the child sits in the mother's lap without fear
or hesitation and responds to the her questions, but fails to looks at the mother's
face when being spoken to. These behaviors are supportive of what additional
assessment?
A. autistic behaviors demonstrated by the child
B. attention deficit hyperactive disorder (ADHD) characteristics demonstrated by
the child
C. indications of physical abuse of the child by the caregiver
D. evidence of cocaine abuse by the caregiver affecting the child

Answer: D

Rationale: Developmental delays occur in young children of substance abusers.


Infants of cocaine abusers avoid the caregiver's gaze, which contributes further to
bonding delays. The children at greatest risk of becoming substance abusers are
those who have families in which alcohol or drug abuse has been present; who
suffer from abuse, neglect, or loss; or who have no close relationships as a result of
a dysfunctional family.
Question format: Multiple Choice
Chapter 33: Abuse in the Family and Community
Cognitive Level: Apply
Client Needs: Psychosocial Integrity
Integrated Process: Nursing Process
Reference: p. 687-688

25. A child is brought to the emergency department for evaluation. The caregiver
states, "The child was running and tripped over the scooter on the sidewalk and
landed on the left arm." The nurse assesses and records the findings (above). The
nurse suspects child abuse (child mistreatment) and notifies the proper authorities.
Which finding(s) will the nurse use to support this suspicion? Select all that apply.
A. spiral fracture
B. bruises on the knees
C. bruises on the thighs
D. abrasion on the left palm
E. bruise on the upper back

Answer: A, C, E

Rationale: Bruises that occur accidentally to a child usually appear over bony areas
such as the knees, elbows, shin, and forehead. Those that have been inflicted in an
abusive manner may be found in soft tissue such as the buttocks, genitalia, thighs,
back of the knees, upper back, and nose and eyes. In this case, findings suggesting
child abuse (child mistreatment) include the spiral fracture and bruises on the
thighs and upper back. The bruises on the knee and hand abrasion are likely
accidental.
Question format: Multiple Select
Chapter 33: Abuse in the Family and Community
Cognitive Level: Apply
Client Needs: Psychosocial Integrity
Integrated Process: Nursing Process
Reference: p. 680
Chapter 34
1. The nurse is working with a group of caregivers who have dying children. Which
statement made by one of the caregivers indicates that this caregiver is in the
depression stage of anticipatory grief?
A. "It is going to be hard for me when I can't hug my daughter anymore."
B. "I've got to go into my office for a meeting. I'm sure my son will be fine while
I'm gone."
C. "I used to love playing board games with my son. Now that just makes me feel
sad."
D. "I'll be able to handle my child's dying if she can just live until her next
birthday."

Answer: C

Rationale: Caring for a family that is facing the death of their child calls on all of the
nurse's personal and professional skills. It includes offering sensitive, gentle,
physical care and comfort measures for the child, as well as continuing emotional
support for the child, the family caregivers, and the siblings. The diagnosis of a
fatal illness initiates the grieving process in the child and the family: denial and
isolation (she'll be fine without me), anger or bargaining (if only), depression
(things that usually give pleasure no longer do) and acute grief, and finally,
acceptance (I will be sad).
Question format: Multiple Choice
Chapter 34: The Dying Child
Cognitive Level: Apply
Client Needs: Psychosocial Integrity
Integrated Process: Caring
Reference: p. 696-698

2. A 3-year-old who has been attending preschool has been diagnosed with
leukemia. The caregivers of this child ask the nurse what they can do to help their
child feel secure. Which recommendation could the nurse make to these caregivers
that would be helpful in making the child feel secure?
A. "Let your child continue to attend preschool as much as possible."
B. "Keep your child at home and spend as much one-on-one time with her as
possible."
C. "Keep your child out of school but invite some friends over for play dates."
D. "Plan special outings with just the family during the time the child would
normally have been at school."

Answer: A

Rationale: Maintaining routine as much as possible helps to give the ill or dying
toddler a greater sense of security.
Question format: Multiple Choice
Chapter 34: The Dying Child
Cognitive Level: Understand
Client Needs: Psychosocial Integrity
Integrated Process: Nursing Process
Reference: p. 694

3. The nurse is caring for a child who has been a client in the pediatric unit for
several weeks. The child has a terminal illness and is dying. The caregivers and the
nurse are in the room when the child takes the last breath and dies. Which action
by the nurse would be considered appropriate at this time?
A. The nurse walks out of the room and calls the nursing supervisor and chaplain.
B. The nurse tells the family that they share in the family's sadness and cries with
them.
C. The nurse maintains their professional demeanor and tells the caregivers that a
staff person will be in to take the body soon.
D. The nurse consoles the family members by hugging or rubbing their backs and
then moves to a corner of the room to cry.

Answer: B

Rationale: When death comes, it is perfectly appropriate to share the family's grief,
crying with them and then giving them privacy to express their sorrow. The nurse
can stay with the family for awhile, remaining quietly supportive with an attitude of
a comforting listener. An appropriate comment may be, "I am so sorry" or "This is a
very sad time." The nurse needs to keep the focus on the family's grief and what
the nurse can do to support them.
Question format: Multiple Choice
Chapter 34: The Dying Child
Cognitive Level: Apply
Client Needs: Psychosocial Integrity
Integrated Process: Caring
Reference: p. 698

4. In interacting for several days with a dying child, the nurse realizes that the child
has not mentioned death. There are some dead flowers in a vase on the child's
windowsill. The child looks at the flowers and says, "Did they die?" Which statement
would be most appropriate for the nurse to make to give the child an opportunity
to open up and talk about dying?
A. "These flowers don't seem to be alive anymore. I'll take them away."
B. "These flowers are dead now. What do you think it would be like to die?"
C. "I'm going to throw these flowers in the trash, okay?"
D. "Well, these flowers aren't very cheerful anymore. I'll get rid of them."

Answer: B

Rationale: Although the dying child may be unable to understand death, the
emotions of family caregivers and others alert the child that something is
threatening his or her secure world. Dealing with the child's anxieties with openness
and honesty restores the child's trust and comfort. The nurse can talk about death
regarding less emotional situations such as dead flowers, trees, insects, or birds.
Question format: Multiple Choice
Chapter 34: The Dying Child
Cognitive Level: Apply
Client Needs: Psychosocial Integrity
Integrated Process: Caring
Reference: p. 693

5. The nurse has recently started caring for a dying child. The nurse has not yet
had a chance to discuss the family's beliefs with the child's caregivers. In the
middle of the night, the child awakens and is frightened. She asks the nurse what
the nurse's religious beliefs are about death. Which response would be appropriate?
A. "I believe that good people, like you, go to heaven."
B. "I don't have any religion but I don't think you should be scared of dying."
C. "I like to keep my beliefs to myself. Are you having trouble sleeping?"
D. "Most religions seem to believe that spirits live on after the body dies. What are
you thinking about?"

Answer: D

Rationale: Nighttime is especially frightening for children because they often think
they will die at night. Provide company and comfort. Be alert for periods of
wakefulness when the child may need someone to talk to. Be honest and
straightforward; avoid injecting your beliefs into the conversation.
Question format: Multiple Choice
Chapter 34: The Dying Child
Cognitive Level: Apply
Client Needs: Psychosocial Integrity
Integrated Process: Caring
Reference: p. 703

6. The father of a 10-year-old who has recently been diagnosed with a terminal
illness tells the nurse that his family had been working together for the previous
year trying to get the 10-year-old and his 6-year-old sister to play better together
and to share with each other. Which statement would be most appropriate for the
nurse to tell the father regarding children and the diagnosis of an illness?
A. "The younger child might believe that her anger at her brother caused his illness;
she will need reassurance that she does not have that power."
B. "The older child might believe that his sister's anger caused his illness; he will
need reassurance that his sister does not have that power."
C. "It will be important that the children learn to play well together while the older
child is alive so that the younger child will have positive memories of the older
child."
D. "Both children are likely to be more cooperative and pleasant with each other as
they deal with this crisis."

Answer: A
Rationale: When a child dies, young siblings who are still prone to magical thinking
may feel guilty, particularly if a strong degree of rivalry existed before the illness.
These children need continued reassurance that they did not cause nor help to
cause their sibling's death. Reactions to the illness and its accompanying stresses
can cause classroom problems for school-aged siblings; these may be incorrectly
labeled as learning disabilities or behavioral disorders unless school personnel are
aware of the family situation.
Question format: Multiple Choice
Chapter 34: The Dying Child
Cognitive Level: Apply
Client Needs: Psychosocial Integrity
Integrated Process: Nursing Process
Reference: p. 700

7. A young nurse has had no experiences of death in her own life. As a staff nurse,
she finds herself dealing with young children who are terminally ill. She feels
extremely uncomfortable facing these children and their caregivers. What is the
best approach for the nurse to take in caring for these families?
A. Focus on the physical needs of the client; the nurse's job is to care for those
aspects of the child's needs.
B. Ask another nurse to perform those tasks that involve discussions with the
clients and their caregivers.
C. Ask her supervisor to allow her to attend a seminar on dealing with the
terminally ill and to recommend books and articles that will help her understand
death better.
D. Recognize that the bad thing is not happening to her, and face her responsibility
to care for all of the needs of the client.

Answer: C

Rationale: Health care workers often are uncomfortable with dying clients, so they
avoid them and are afraid that the clients will ask questions that they cannot or
should not answer. These caregivers signal by their behavior that the client should
avoid the fact of his or her impending death and should keep up a show of bravery.
In effect, they are asking the client to meet their needs instead of trying to meet
the client's needs. A workshop, conference, or seminar in which one's own feelings
about life and death are explored is useful in preparing the nurse to care for the
dying child and family.
Question format: Multiple Choice
Chapter 34: The Dying Child
Cognitive Level: Understand
Client Needs: Safe, Effective Care Environment: Management of Care
Integrated Process: Nursing Process
Reference: p. 693
8. A child who is dying becomes restless and fidgety, then becomes calm and
peaceful. The caregivers feel hopeful that the child seems "better." Which
statement would be the best for the nurse to tell the caregivers about this pattern?
A. "This pattern is part of dying; the child's death is imminent."
B. "This pattern is not unusual in a dying child and might happen over and over
again."
C. "This pattern suggests that the child is feeling more comfortable and has less
distress from the illness."
D. "This pattern is consistent with medication cycles; the fidgeting is caused by
pain, and the calmness is a sign that the pain has been controlled with medicine."

Answer: A

Rationale: Just before death, the child who has remained conscious may go through
a period of restlessness, followed by a period of peace and calm. The nurse and
family members should be aware of these reactions and know that death is near.
Question format: Multiple Choice
Chapter 34: The Dying Child
Cognitive Level: Apply
Client Needs: Psychosocial Integrity
Integrated Process: Caring
Reference: p. 699-700

9. The caregivers of a dying 9-year-old boy are in his hospital room. Death is near
and the child has drifted in and out of consciousness for several days. The
caregivers are at two different stages of anticipatory grief and are arguing in
whispers in the room. The best response in this situation would be for the nurse to:
A. recognize that their differences are normal and leave them to their argument.
B. interrupt their conversation, acknowledge that their feelings are valid, and let
them know that the child can hear them and might be upset by their discord.
C. interrupt their conversation, acknowledge that their feelings are valid, and ask
them to leave the room.
D. call a chaplain to come and speak with them.

Answer: B

Rationale: Even though the dying child may have a decreased level of
consciousness, his or her hearing remains intact. Family members at the bedside as
well as health care personnel may need to be reminded to avoid saying anything
that would not be said if the child were fully conscious. Discourage whispered
conversations in the room. Emotions and fears must be acknowledged, and
caregivers should be reassured that their reactions are normal. The support of a
member of the clergy may be helpful during this time. Help family members contact
their own spiritual counselor or offer to contact the hospital chaplain if the family
desires.
Question format: Multiple Choice
Chapter 34: The Dying Child
Cognitive Level: Apply
Client Needs: Psychosocial Integrity
Integrated Process: Caring
Reference: p. 699-700

10. A single parent of a dying child has told the nurse she would like to take care of
her daughter at home. What would be best for the nurse to do to help the mother
reach a decision that will be good for both the mother and the child? The nurse
should:
A. Put the mother in touch with the department of a hospital that can help her sort
out the availability of home nursing care, insurance coverage, and respite care.
B. Remind the mother that the number of staff people caring for the child in the
hospital is unlikely to be duplicated at home, making the hospital a better choice.
C. Validate her courageous idea and say that the staff at the hospital will always be
available to answer her questions.
D. Suggest that she look into a hospice residence, which is often a good alternative
to caring for the child alone at home or having the child remain in the hospital.

Answer: A

Rationale: The home can be a more loving and caring environment but this choice
must be considered carefully in terms of financial, health care, and caregiver health
concerns. This is an extremely difficult decision for a family. Family members need
support and guidance from health care personnel while they are trying to make the
decision, after the decision is made, and even after the child dies.
Question format: Multiple Choice
Chapter 34: The Dying Child
Cognitive Level: Apply
Client Needs: Safe, Effective Care Environment: Management of Care
Integrated Process: Nursing Process
Reference: p. 701

11. When mentoring a graduate practical nurse (PN), which behavior would the
registered nurse (RN) address if demonstrated by the PN in the care of a dying
child?
A. agreeing to pray with the client and family
B. checking on the client and family once every shift
C. assessing comfort measures needed by the client every hour
D. providing bedding for the family to remain close to the client

Answer: B

Rationale: Graduate nurses frequently have a difficult time interacting with dying
clients and their families. When unsure of care or uncomfortable with the situation,
nurses tend to avoid the client and family. Checking on the client and family just
one time every shift demonstrates practicing avoidance behavior. Taking care
measures to ensure client comfort and support for the family requires frequent
assessment and nursing care. Praying with the client and family when asked is
often welcomed and demonstrates spiritual support. Providing comfort measures for
the dying child is a priority nursing action. Caring for the family by providing
opportunities for rest while remaining close to the client encourages family support
for the client.
Question format: Multiple Choice
Chapter 34: The Dying Child
Cognitive Level: Analyze
Client Needs: Safe, Effective Care Environment: Management of Care
Integrated Process: Nursing Process
Reference: p. 693

12. A toddler's older sibling has died. Which nursing response best addresses a
toddler's concern about, "Where has my brother gone?"
A. "Your brother is gone but is in a special place. We can think about him and
remember the fun times."
B. "Your brother has fallen asleep and is safe where he has gone. We look forward
to seeing him again someday."
C. "We do not know or understand where your brother has gone as we have never
gone there. Many people say it is a wonderful place."
D. "It is best to ask your Mommy where he is. She will explain it to you."

Answer: A

Rationale: Questions are best answered simply and honestly when explaining the
death of a loved one to a toddler. The toddler must know that the brother is not
returning, but we can remember him. Also, many toddlers want to know that the
brother is not sick anymore or hurting. The nurse would not relate death to
sleeping, because the toddler may become afraid to fall asleep. Stating that we do
not understand where the brother is can also be scary for the toddler, because he
may think that the brother is lost. Deflecting the toddler's question is not helpful to
the toddler and is a block to communication.
Question format: Multiple Choice
Chapter 34: The Dying Child
Cognitive Level: Apply
Client Needs: Psychosocial Integrity
Integrated Process: Caring
Reference: p. 693

13. Which action by the nurse providing end-of-life care to a preschool-aged client
best promotes comfort?
A. positioning a favorite toy by the client's hand
B. allowing a pet to visit and sit on the client's lap
C. providing a warm bath and back massage before bed
D. arranging for a bed to allow the parent to lie with the child

Answer: D

Rationale: The main fear of preschoolers is to be separated from the parents.


Allowing the parents to lie closely with the child soothes the child and promotes
comfort, especially at the end of life. The other options are good client-specific
nursing interventions.
Question format: Multiple Choice
Chapter 34: The Dying Child
Cognitive Level: Apply
Client Needs: Physiological Integrity: Basic Care and Comfort
Integrated Process: Nursing Process
Reference: p. 694

14. The nurse is providing client-centered support to an adolescent diagnosed with


a terminal illness. Keeping the client's developmental stage in mind, which nursing
intervention will best meet the client's needs?
A. rearranging the medication schedule so the client can see a favorite sports team
B. changing the client's room so the client can watch the busy outdoors
C. limiting visits from friends at school to the client's birthday
D. encouraging siblings to participate with client in hospital activities

Answer: A

Rationale: The nurse must consider the developmental stage of the client.
Adolescents try to live their lives to the fullest while they are able. Adolescents
understand their impending death. For the nurse, the best action is to assist, if
possible, participation in activities important to the client. Having the client watch
the world from the inside is not considering his/her desire to participate in
activities. Peers are very important to the adolescent and should be allowed to visit
more than on the client's birthday. Hospital activities are not selected by the client.
Question format: Multiple Choice
Chapter 34: The Dying Child
Cognitive Level: Apply
Client Needs: Psychosocial Integrity
Integrated Process: Nursing Process
Reference: p. 695

15. When considering a school-aged child, the nurse should anticipate which factor
as playing an influential role in the development of the concept of death?
A. social media
B. video games
C. family
D. friends

Answer: C

Rationale: Past experiences with death affect the client's attitude and interpretation
of death. Family members are role models who have the most influence on the
child. Friends and social media have strong influences on the adolescent. Video
games are not a typical influence.
Question format: Multiple Choice
Chapter 34: The Dying Child
Cognitive Level: Apply
Client Needs: Psychosocial Integrity
Integrated Process: Nursing Process
Reference: p. 694

16. When caring for a child who is dying, which statement by the child leads the
nurse to believe that the topic of death needs to be discussed further?
A. "I am going to a better place."
B. "I do not want to leave my parents."
C. "The bogeyman is going to come and fly me away."
D. "I want to make a picture so my sister remembers me."

Answer: C

Rationale: Misconceptions by the child, such as the bogeyman statement, often


indicate that no one has prepared the child for death, thus leaving the child to
imagine what may happen. The other statements are common for a child who is
understands that he or she has a terminal diagnosis.
Question format: Multiple Choice
Chapter 34: The Dying Child
Cognitive Level: Analyze
Client Needs: Psychosocial Integrity
Integrated Process: Nursing Process
Reference: p. 695

17. The parents of a terminally ill child ask the nurse how to have a conversation
about illness. Which is the most appropriate concept for the nurse to base the
response?
A. Be direct, using age-appropriate language the child can understand.
B. Use metaphors that the child has heard from past stories.
C. Relate to a favorite family member who has died.
D. Use broad terms and general statements.

Answer: A

Rationale: Children know when they are dying and want to be able to understand
what is happening. Being truthful and direct evokes trust and security. Always use
age-appropriate language to enhance therapeutic communication. Using metaphors
is commonly not realistic and thus can be confusing to the child. Relating to a
family member is not about the client. Using broad terms does not thoroughly
answer the child's questions and may be a block to communication.
Question format: Multiple Choice
Chapter 34: The Dying Child
Cognitive Level: Apply
Client Needs: Psychosocial Integrity
Integrated Process: Nursing Process
Reference: p. 696
18. Which technique will best foster the initial communication between the nurse
and a school-aged child recently told of a cancer diagnosis?
A. Provide supplies for the child to draw a picture.
B. Begin a conversation about the diagnosis with the parent.
C. Sit quietly at the bedside until the child is ready to communicate.
D. Offer age-appropriate board games, movies, or video games.

Answer: A

Rationale: Drawing, poetry, or writing lyrics are ways that a school-aged child can
express his or her emotions. Upon viewing, the nurse is able to comment, which
opens communication. Talking with the parent excludes communication with the
child. Sitting quietly is ineffective if the child does not know how to verbalize
emotions. Offering board games can be fun and divert the child's attention from the
cancer, but this does not help the child communicate feelings.
Question format: Multiple Choice
Chapter 34: The Dying Child
Cognitive Level: Understand
Client Needs: Psychosocial Integrity
Integrated Process: Nursing Process
Reference: p. 699-700

19. A graduate nurse has provided care for a pediatric client who has died. Which
statement to the family would require further discussion between the graduate
nurse and the registered nurse (RN) mentor?
A. "It is so hard to see you cry."
B. "This reminds me of the time my aunt died."
C. "I am very sorry of your loss. He was a great kid."
D. "Please let me know if there is something I can do for you."

Answer: B

Rationale: At the time when the client has died, the focus is on the family and their
grief. The nurse would not mention personal experiences. Acknowledging family
feelings, validating the life of the child, and letting the family know that nursing
staff are there for them are appropriate nursing statements.
Question format: Multiple Choice
Chapter 34: The Dying Child
Cognitive Level: Analyze
Client Needs: Safe, Effective Care Environment: Management of Care
Integrated Process: Caring
Reference: p. 698

20. The emergency room nurse is caring for a family who lost an adolescent in a
violent car accident. Which initial nursing action best helps the family's grieving
process?
A. notifying the family's religious leader of the tragedy
B. cleaning and making the deceased presentable to the family
C. calling the funeral home to make arrangements for the body transfer
D. presenting the family with the deceased's personal belongings

Answer: B

Rationale: The death of a child unexpectedly produces acute grief. Even if the child
has a traumatic death with disfigurement, the family must be given the opportunity
to see the adolescent to help with closure. The nurse can best aid in the grieving
process by cleaning and preparing the body for viewing. The other options are
appropriate measures for the nurse to complete, but not most helpful with the
grieving process.
Question format: Multiple Choice
Chapter 34: The Dying Child
Cognitive Level: Apply
Client Needs: Psychosocial Integrity
Integrated Process: Caring
Reference: p. 699

21. The nurse is discussing the principles of hospice care to the family of a
terminally ill pediatric client. Which parental statement would the nurse clarify as a
misconception?
A. "My son will be assessed for pain relief."
B. "We can participate in our religious practices."
C. "My son will still receive aggressive treatments for a cure."
D. "My son will have holistic nursing care meeting his needs."

Answer: C

Rationale: The client who is in hospice care will not receive heroic lifesaving
measures. Families must understand that hospice services emphasize sensitive,
humane care for the dying. Hospice care provides pain relief and holistic care
through the dying process. Families are encouraged to practice their faith.
Question format: Multiple Choice
Chapter 34: The Dying Child
Cognitive Level: Analyze
Client Needs: Psychosocial Integrity
Integrated Process: Teaching/Learning
Reference: p. 700

22. When evaluating a terminally ill client for pain relief following medication
administration, which nursing assessments would be documented to support that
the medication was effective? Select all that apply.
A. Heart rate and respirations are within normal limits.
B. The client is lying in a flexed position on the bed.
C. The client is quietly sleeping on the parent's lap.
D. The child is withdrawing from the environment.
E. The client verbalizes a 1 on the analog pain scale.
Answer: A, C, E

Rationale: When the nurse is assessing the client for pain relief following medication
administration, vital signs within normal limits support that pain relief has occurred.
Blood pressure and heart and respiration rates are elevated in clients who are in
pain. Typically clients who are able to relax and sleep have pain relief. On the
analog pain scale, a low score of 0 to 1 documents pain relief. Clients who are
flexed or withdrawn display signs of unrelieved pain.
Question format: Multiple Select
Chapter 34: The Dying Child
Cognitive Level: Analyze
Client Needs: Physiological Integrity: Pharmacological and Parenteral Therapies
Integrated Process: Nursing Process
Reference: p. 702

23. Which factor associated with the caring of a dying pediatric client is most often
cited by nurses as presenting the greatest degree of difficulty?
A. realizing that the clients have not lived much of their life
B. caring for such families takes a high emotional toll
C. being constantly reminded of their personal mortality
D. addressing cultural difference in beliefs and practices

Answer: C

Rationale: Death reminds all people of their own mortality, thus is uncomfortable
and must be personally confronted. Having a personal impact is most difficult when
caring for clients who are dying. The other options are correct statements, but not
as frequently noted.
Question format: Multiple Choice
Chapter 34: The Dying Child
Cognitive Level: Understand
Client Needs: Psychosocial Integrity
Integrated Process: Nursing Process
Reference: p. 693

24. The nurse is attempting to include family in the care of a hospitalized,


terminally ill adolescent. Which interventions can be appropriately delegated to
family members? Select all that apply.
A. Provide mouth swabs for the family to moisten lips.
B. Discuss the client's status with care options at the beside.
C. Provide pillows to facilitate the repositioning for comfort.
D. Keep a blood pressure cuff and stethoscope available at bedside.
E. Supply sufficient disposable pads to be placed under the client as needed.

Answer: A, C, E

Rationale: The family is able to provide comfort measures to the dying child. This
provides the family with a sense of control and the feeling that they are able to care
for the child in the final days. The family is able to moisten the lips, position the
client for comfort, or change pads under or around the child. Discussing the client's
status is inappropriate at the bedside because the child may be able to hear the
conversation. Nursing measures, such as taking blood pressure, are assigned to the
nurse.
Question format: Multiple Select
Chapter 34: The Dying Child
Cognitive Level: Apply
Client Needs: Psychosocial Integrity
Integrated Process: Nursing Process
Reference: p. 703

25. Which communication strategies are appropriate when discussing the topic of
death and dying with a terminally ill school-aged child? Select all that apply.
A. Use the terms death and dying in the conversation.
B. Listen to the child and accept the child's feelings.
C. Tell the client that you know how hard this is.
D. Consider the developmental level of the child.
E. Say "I do not know" when unsure of an answer.
F. Allow the health care provider to answer all questions.

Answer: A, B, D, E

Rationale: The nurse should encourage communication with the client and observe
for clues that the client is willing to express feelings. Once communication is
established, the nurse uses appropriate terms in conversation and listens and
accepts the client's feelings. Nurses should always say "I do not know" when unsure
of an answer. The nurse must consider the developmental age of the child when
interacting with a child. Telling the client that you know it is difficult is a cliché and
inappropriate. Nurses, as well as health care providers, may answer the client's
questions.
Question format: Multiple Select
Chapter 34: The Dying Child
Cognitive Level: Apply
Client Needs: Psychosocial Integrity
Integrated Process: Communication and Documentation
Reference: p. 695
Chapter 35
1. When caring for an infant who is hospitalized with Haemophilus influenzae
meningitis, an important nursing intervention for the child would be for the nurse
to:
A. restrain the child before and during a seizure.
B. place the child in a side-lying position and keep the position using pillows.
C. monitor intake and output and increase fluid intake every 4 hours.
D. check the child's neurologic status every 2 hours.

Answer: D

Rationale: The nursing interventions for the child with meningitis are related to the
goals for this child, which include monitoring for complications related to neurologic
compromise, preventing aspiration, keeping the child safe from injury during a
seizure, and monitoring fluid balance. During a seizure, stay with the child, protect
the child from injury, but do not restrain him or her. To prevent aspiration, position
the child in a side-lying position, watch for and remove excessive mucus as much
as possible, and use suction sparingly. Every 2 hours, observe the child for seizure
activity, vital signs, neurologic changes, and change in level of consciousness. The
child is placed on fluid restrictions if he or she has decreased urinary output,
hyponatremia, increased weight, nausea, and irritability.
Question format: Multiple Choice
Chapter 35: The Child with a Sensory/Neurologic Disorder
Cognitive Level: Apply
Client Needs: Safe, Effective Care Environment: Management of Care
Integrated Process: Nursing Process
Reference: p. 724

2. The caregiver of a 2-year-old calls the clinic concerned that her child may have
pushed paper into her ears, and she asks the nurse what to do. The mother found
the child pushing on her ears with torn paper on the floor in front of her. What
would be the appropriate response by the nurse?
A. "Wait for a day or two to see if the child's ears seem irritated. If they do, bring
her in."
B. "Sterilize a pair of narrow tweezers. While someone else holds the child's head
still, carefully insert the tweezers and remove the paper if there is any."
C. "The child should be seen by a care provider. Don't put anything in her ear and
bring her in right away."
D. "Wash her ear out using warm water and an ear syringe then the paper will flush
out with the water."

Answer: C

Rationale: Irrigation of the ear may remove small objects except paper, which
becomes impacted as it absorbs moisture. The physician generally uses small
forceps to remove objects not dislodged by irrigation. The caregiver should not
insert anything into the child's ear.
Question format: Multiple Choice
Chapter 35: The Child with a Sensory/Neurologic Disorder
Cognitive Level: Apply
Client Needs: Physiological Integrity: Reduction of Risk Potential
Integrated Process: Nursing Process
Reference: p. 717

3. The nurse caring for a deaf child is preparing to give the child an intramuscular
injection in the child's upper arm. The child's attention is focused on the caregiver
on the opposite side of the bed. Before administering the injection, it would be
most important for the nurse to do which of the following?
A. Pat the child gently on the shoulder to get his attention and point to her own arm
so he knows she is giving him the injection.
B. Ask his caregiver to point out that the nurse is there.
C. Move around the bed and make eye contact, then show the child the syringe and
indicate that the shot is going to be given.
D. Ask the caregiver to keep the child's attention focused on her.

Answer: C

Rationale: When speaking to the deaf child, stand or sit face-to-face on the child's
level. Be certain that a deaf child can see you before you touch him or her.
Demonstrate each procedure before it is performed, showing the child the
equipment or pictures of the equipment to be used.
Question format: Multiple Choice
Chapter 35: The Child with a Sensory/Neurologic Disorder
Cognitive Level: Apply
Client Needs: Physiological Integrity: Basic Care and Comfort
Integrated Process: Nursing Process
Reference: p. 716

4. A 7-year-old client has been complaining of headache, coughing, and an aching


chest. The care provider makes a diagnosis of a viral infection. The child's mother
tells the nurse that when she first said she had a headache, the child's father gave
her half of an adult aspirin. The mother has heard of Reye syndrome and asks the
nurse if her child could get this. Which statement would be the best response by
the nurse?
A. "This might or might not be a problem. Watch your daughter for signs of
lethargy, unusual irritability, confusion, or vomiting. If you notice any of these,
bring her to the emergency room immediately so she can be checked for Reye
syndrome."
B. "This is a serious problem. Aspirin is likely to cause Reye syndrome, and she
should be admitted to the hospital for observation as a precaution."
C. "This is unlikely to be a problem. Half an aspirin is not enough to cause harm.
Reye syndrome generally only develops from prolonged use of aspirin in connection
with a virus."
D. "This might or might not be a problem. Watch your daughter for signs of nasal
discharge, sneezing, itching of the nose, or dark circles under the eyes. If you
notice any of these, bring her to the emergency room immediately so she can be
checked for Reye syndrome."

Answer: A

Rationale: Reye syndrome usually occurs after a viral illness, particularly after an
upper respiratory infection or varicella (chickenpox). Administration of aspirin
during the viral illness has been implicated as a contributing factor. As a result, the
American Academy of Pediatrics recommends that aspirin or aspirin compounds not
be given to children with viral infections. The symptoms appear within 3 to 5 days
after the initial illness: The child is recuperating unremarkably when symptoms of
severe vomiting, irritability, lethargy, and confusion occur. Immediate intervention
is needed to prevent serious insult to the brain, including respiratory arrest.
Question format: Multiple Choice
Chapter 35: The Child with a Sensory/Neurologic Disorder
Cognitive Level: Apply
Client Needs: Physiological Integrity: Physiological Adaptation
Integrated Process: Nursing Process
Reference: p. 717

5. The caregivers of a child who has had difficulty learning to walk notice that when
the child attempts to pull himself up to stand, he can't seem to get his legs
uncrossed and beside each other. When he is in a standing position, he stays up on
his toes. This is different from what they saw with their older children and they are
concerned. Further diagnostic tests indicate the child has cerebral palsy. When
instructing on the types of cerebral palsy, which type will the nurse present?
A. ataxic cerebral palsy
B. athetoid cerebral palsy
C. rigidity cerebral palsy
D. spastic cerebral palsy

Answer: D

Rationale: Among other things, spastic cerebral palsy is characterized by scissoring


caused by severe hip adduction. When scissoring is present, the child's legs are
crossed and the toes are pointed down. When standing, the child is on his or her
toes. It is difficult for this child to walk on the heels or run. Ataxic cerebral palsy is
essentially a lack of coordination caused by disturbances in the kinesthetic and
balance senses; it is characterized by an awkward and wide-based gait. Athetoid
cerebral palsy is marked by involuntary, uncoordinated motion with varying degrees
of muscle tension. Children with this disorder are constantly in motion; the whole
body is in a state of slow, writhing muscle contractions whenever voluntary
movement is attempted. Rigidity cerebral palsy is uncommon and is characterized
by rigid postures and lack of active movement.
Question format: Multiple Choice
Chapter 35: The Child with a Sensory/Neurologic Disorder
Cognitive Level: Apply
Client Needs: Physiological Integrity: Physiological Adaptation
Integrated Process: Nursing Process
Reference: p. 727

6. The nurse is discussing discipline issues with a group of caregivers of preschool-


aged children who have a cognitive impairment. One father tells the group that
after he tells his child to stop doing something, the child just continues. Parents in
the group make the following statements. Which statement indicates an
understanding of disciplining the cognitively impaired child?
A. "We hold a family meeting to explain to our other children that she gets away
with things they will not get away with because of her impairment."
B. "We wait until a behavior happens a second time and immediately put our child
in time-out."
C. "We always take away a privilege our child has been looking forward to, such as
going to a movie next weekend."
D. "My wife and I just ignore the behavior and if there is a mess, clean up the
mess. He doesn't understand that he has an impairment."

Answer: B

Rationale: If discipline is needed, be certain it follows the misdeed immediately so


that the cause-and-effect relationship is clear. Obedience is an important part of
discipline, especially for the child with faulty reasoning ability, but the objectives of
discipline should be much broader than simply obedience. The child needs to know
what to expect and finds security and support in routines and consistency. Use
kindness, love, understanding, and physical comforting as a major part of
discipline.
Question format: Multiple Choice
Chapter 35: The Child with a Sensory/Neurologic Disorder
Cognitive Level: Apply
Client Needs: Health Promotion and Maintenance
Integrated Process: Teaching/Learning
Reference: p. 731

7. The mother of a child newly diagnosed with an intellectual disability tells the
nurse that her partner disagrees with the diagnosis and believes that the child is
perfectly normal. The mother shares with the nurse that she finds this reaction
frustrating and confusing. Which action by the nurse would be appropriate in
supporting this mother?
A. Reassure the mother that her partner's reaction is a normal stage in the grieving
process.
B. Offer to speak with the partner to explain how the diagnosis was reached.
C. Suggest that the couple get a second opinion about the child's condition.
D. Recommend that the couple consider placing the child in foster care until they
adjust to the diagnosis.

Answer: A
Rationale: The family's first reaction to learning that the child may have cognitive
impairment is grief because this is not the perfect child of their dreams. A parent
may feel shame, assuming that he or she cannot produce a perfect child. Some
rejection of the child is almost inevitable at least in the initial stages, but this must
be worked through for the family to cope.
Question format: Multiple Choice
Chapter 35: The Child with a Sensory/Neurologic Disorder
Cognitive Level: Apply
Client Needs: Psychosocial Integrity
Integrated Process: Caring
Reference: p. 733

8. What information is most correct regarding the nervous system of the child?
A. The child's nervous system is fully developed at birth.
B. As the child grows, the gross and fine motor skills increase.
C. The child has underdeveloped fine motor skills and well-developed gross motor
skills.
D. The child has underdeveloped gross motor skills and well-developed fine motor
skills.

Answer: B

Rationale: As the child grows, the quality of the nerve impulses sent through the
nervous system develops and matures. As these nerve impulses become more
mature, the child's gross and fine motor skills increase in complexity. The child
becomes more coordinated and able to develop motor skills.
Question format: Multiple Choice
Chapter 35: The Child with a Sensory/Neurologic Disorder
Cognitive Level: Remember
Client Needs: Physiological Integrity: Physiological Adaptation
Integrated Process: Nursing Process
Reference: p. 709

9. The nurse recognizes that if the infant is following normal development, the
infant will be able to focus and follow an object with the eyes by what age?
A. 7 days of age
B. 21 days of age
C. 1 month of age
D. 2 months of age

Answer: D

Rationale: Newborns are born nearsighted. They prefer the human face to other
objects. At 1 month they can recognize by site the people they know. By 2 months
of age, the infant can focus and follow an object with the eyes. Binocularity
develops at 6 months and color vision follows at 7 months.
Question format: Multiple Choice
Chapter 35: The Child with a Sensory/Neurologic Disorder
Cognitive Level: Remember
Client Needs: Health Promotion and Maintenance
Integrated Process: Nursing Process
Reference: p. 709

10. The nurse is discussing the ears and hearing in a child with a group of peers.
Which statement is most accurate related to this topic?
A. The eustachian tube in the infant is straighter and wider than in the adult.
B. The infant usually responds to sounds around the age of 6 months.
C. The eardrum is located between the middle and the internal ear.
D. Most children do not have acutely developed hearing until the age of 5 years.

Answer: A

Rationale: In infants and young children, the eustachian tube is straighter, shorter,
and wider than in the older child and adult. Hearing in children is acute, and the
infant will respond to sounds within the first month of life.
Question format: Multiple Choice
Chapter 35: The Child with a Sensory/Neurologic Disorder
Cognitive Level: Remember
Client Needs: Health Promotion and Maintenance
Integrated Process: Nursing Process
Reference: p. 716

11. The nurse is teaching a group of parents about eyes and eye concerns. The
nurse tells these caregivers about a condition that occurs when unequal curvatures
in the cornea bend the light rays in different directions and this causes images to be
blurred. The condition the nurse is referring to is:
A. Refraction
B. Myopia
C. Astigmatism
D. Hyperopia

Answer: C

Rationale: Astigmatism is caused by unequal curvatures in the cornea that bend the
light rays in different directions and produce a blurred image. Refraction is the way
light rays bend as they pass through the lens to the retina. Myopia is
nearsightedness; hyperopia is farsightedness.
Question format: Multiple Choice
Chapter 35: The Child with a Sensory/Neurologic Disorder
Cognitive Level: Understand
Client Needs: Physiological Integrity: Physiological Adaptation
Integrated Process: Teaching/Learning
Reference: p. 710
12. The caregiver of a 2-year-old child tells the nurse, "They told me my daughter
has an eye disorder called hyperopia." Which statement made by the mother
indicates she has an understanding of this child's current condition?
A. "She can see better close up than at a distance."
B. "Now I know why when she is working on puzzles she says her eye is sleepy."
C. "She has to have glasses right away."
D. "At least by the time she gets married maybe she can have contacts."

Answer: B

Rationale: Hyperopia is farsightedness in which a person sees objects better at a


distance than close up. Considerable eye fatigue may result from efforts at
accommodation for close work. It is common in young children and often persists
into the first grade or even later. Usually correction is not needed in a preschooler.
Question format: Multiple Choice
Chapter 35: The Child with a Sensory/Neurologic Disorder
Cognitive Level: Apply
Client Needs: Physiological Integrity: Physiological Adaptation
Integrated Process: Teaching/Learning
Reference: p. 709

13. The nurse is caring for an 8-year-old hospitalized child who is visually impaired.
Which nursing intervention would be the highest priority in helping this child
reduce anxiety related to hospitalization?
A. The nurse takes the child to the playroom and lets her touch the toys.
B. The nurse talks to the child when entering and leaving the room.
C. The nurse encourages the child to call her friends on the telephone.
D. The nurse stays with the child during meals and helps her eat.

Answer: B

Rationale: The highest priority is for the nurse to identify herself to the child when
entering or leaving the room. This helps reduce fear and anxiety and the possibility
of being startled by an unexpected touch. It is important for the child to have
activities related to normal growth and development, such as touching and playing
with the toys in the playroom, talking on the telephone, and having the nurse
spend time with the child, but these are not the highest priorities.
Question format: Multiple Choice
Chapter 35: The Child with a Sensory/Neurologic Disorder
Cognitive Level: Apply
Client Needs: Physiological Integrity: Basic Care and Comfort
Integrated Process: Caring
Reference: p. 712

14. The nurse is observing a child following an eye injury. Which symptom should
alert the nurse to the possibility that the child's uninjured eye may be exhibiting
signs of an inflammatory reaction?
A. swelling or dryness
B. brightness of vision
C. discharge of purulent drainage
D. an intolerance to light

Answer: D

Rationale: Sympathetic ophthalmia, an inflammatory reaction of the uninjured eye,


may follow perforation wounds of the eye. Sympathetic ophthalmia often includes
photophobia (intolerance to light), lacrimation (secretion of tears), pain, and some
dimness of vision. Symptoms of eye infections include swelling discharge of
purulent material.
Question format: Multiple Choice
Chapter 35: The Child with a Sensory/Neurologic Disorder
Cognitive Level: Apply
Client Needs: Physiological Integrity: Physiological Adaptation
Integrated Process: Nursing Process
Reference: p. 713

15. The nurse is teaching an in-service program to a group of nurses on the topic of
children diagnosed with otitis media. The nurses in the group make the following
statements. Which statement is most accurate related to the diagnosis of otitis
media?
A. "When you look at the ear of the infant, pull the ear up and back."
B. "They usually are fussy, but they rarely have a fever."
C. "After the eardrum ruptures, the child's pain is relieved."
D. "Antibiotics don't have much effect, so they aren't used for treatment."

Answer: C

Rationale: Spontaneous rupture of the eardrum may occur, in which case there will
be purulent drainage, and the pain caused by the pressure buildup in the ear will be
relieved. The ear in the infant is examined with an otoscope by pulling the ear down
and back to straighten the ear canal. In the older child, the ear is pulled up and
back. Symptoms may include fever, irritability, and hearing impairment. Antibiotics
are used during the period of infection and for several days after to prevent
mastoiditis or chronic infection.
Question format: Multiple Choice
Chapter 35: The Child with a Sensory/Neurologic Disorder
Cognitive Level: Analyze
Client Needs: Physiological Integrity: Physiological Adaptation
Integrated Process: Teaching/Learning
Reference: p. 716

16. In children with otitis media, a procedure known as a myringotomy may be


performed. Which statement is most accurate regarding this procedure?
A. During this procedure, small tubes are inserted into the tympanic membrane.
B. This procedure is performed as soon as otitis media is diagnosed.
C. A small incision is made in the earlobe during this procedure.
D. The purpose of this procedure is to decrease or stop the drainage.

Answer: A

Rationale: Myringotomy (incision of the eardrum) may be performed to establish


drainage and to insert tiny tubes into the tympanic membrane to facilitate drainage.
The procedure is done for children with chronic otitis media, not as soon as the
child is diagnosed.
Question format: Multiple Choice
Chapter 35: The Child with a Sensory/Neurologic Disorder
Cognitive Level: Understand
Client Needs: Physiological Integrity: Reduction of Risk Potential
Integrated Process: Nursing Process
Reference: p. 716

17. The nurse is collecting data on an 18-month-old child admitted with a diagnosis
of possible seizures. When interviewing the caregivers, which questions would be
most important for the nurse to ask?
A. "Is your child up to date on his immunizations?"
B. "Has anyone in your family been sick recently"
C. "What type of activities was your child doing today?"
D. "Have you checked your child's temperature?"

Answer: D

Rationale: In children between the ages of 6 months and 3 years, febrile seizures
(seizures resulting from fever) are the most common. Febrile seizures usually occur
in the form of a generalized seizure early in the course of a fever. Although an
illness in another member of the family might indicate the possibility of an infection
or other illness, this would not be the priority question to ask. If the child has had
an immunization recently, the child could develop a fever, but asking if the
immunizations are up-to-date is not asking about recent immunizations. The child's
daily activities would not likely promote seizure activity.
Question format: Multiple Choice
Chapter 35: The Child with a Sensory/Neurologic Disorder
Cognitive Level: Apply
Client Needs: Physiological Integrity: Reduction of Risk Potential
Integrated Process: Nursing Process
Reference: p. 718

18. The nurse is caring for a child admitted with focal onset impaired awareness
seizure (complex partial seizure). Which clinical manifestation would likely have
been noted in the child with this diagnosis?
A. The child had jerking movements and then the extremities stiffened.
B. The child had shaking movements on one side of the body.
C. The child was rubbing the hands and smacking the lips.
D. The child was dizzy and had decreased coordination.
Answer: C

Rationale: With the focal onset impaired awareness seizure, formerly called
complex partial seizure, the child is confused or their awareness is affected during
the seizure. The seizure begins in a small area of the brain and changes or alters
consciousness. These seizures can have motor and non-motor symptoms. They
cause memory loss and staring and nonpurposeful movements, such as hand
rubbing, lip smacking, arm dropping, and swallowing. In the tonic phase of tonic–
clonic seizures, the child's muscles contract, the child may fall, and the child's
extremities may stiffen. During the prodromal period of the tonic–clonic seizure, the
child might have a lack of coordination.Focal onset aware seizures (formerly called
simple partial seizures) can either have motor or sensory symptoms. A focal onset
motor seizure causes a localized motor activity such as shaking of an arm, leg, or
other body part.
Question format: Multiple Choice
Chapter 35: The Child with a Sensory/Neurologic Disorder
Cognitive Level: Understand
Client Needs: Physiological Integrity: Physiological Adaptation
Integrated Process: Nursing Process
Reference: p. 718

19. The nurse is collecting data from the caregivers of a child admitted with
seizures. Which statement indicates the child most likely had an absence seizure?
A. "His arms had jerking movements in his legs and face."
B. "He was just staring into space and was totally unaware."
C. "He kept smacking his lips and rubbing his hands."
D. "He usually is very coordinated, but he couldn't even walk without falling."

Answer: B

Rationale: Absence seizures rarely last longer than 20 seconds. The child loses
awareness and stares straight ahead but does not fall. Myoclonic seizures are
characterized by a sudden jerking of a muscle or group of muscles, often in the
arms or legs, without loss of consciousness.. Focal onset impaired awareness
seizures (formerly called complex partial seizure) cause nonpurposeful movements,
such as hand rubbing and lip smacking. During the prodromal period of the tonic-
clonic seizure, the child might have a lack of coordination.
Question format: Multiple Choice
Chapter 35: The Child with a Sensory/Neurologic Disorder
Cognitive Level: Understand
Client Needs: Physiological Integrity: Physiological Adaptation
Integrated Process: Nursing Process
Reference: p. 719

20. The nurse is caring for a child admitted with focal onset motor seizures (simple
partial motor seizures). Which clinical manifestation would likely have been noted in
the child with this diagnosis?
A. The child had jerking movements in the legs and facial muscles.
B. The child had shaking movements on one side of the body.
C. The child was rubbing the hands and smacking the lips.
D. The child was dizzy and had decreased coordination.

Answer: B

Rationale: Focal onset aware seizures (formerly called simple partial seizures) can
either have motor or sensory symptoms. A focal onset motor seizure causes a
localized motor activity such as shaking of an arm, leg, or other body part. A focal
onset sensory seizure may include sensory symptoms called an aura, which signals
an impending attack. Myoclonic seizures are characterized by a sudden jerking of a
muscle or group of muscles, often in the arms or legs, without loss of
consciousness. Complex partial seizures may cause nonpurposeful movements,
such as hand rubbing and lip smacking. During the prodromal period of the tonic-
clonic seizure, the child might have a lack of coordination.
Question format: Multiple Choice
Chapter 35: The Child with a Sensory/Neurologic Disorder
Cognitive Level: Understand
Client Needs: Physiological Integrity: Physiological Adaptation
Integrated Process: Nursing Process
Reference: p. 718

21. The nurse is interviewing the caregivers of a child brought to the emergency
unit. The caregiver states, "She has a history of seizures but this time it lasted
more than 30 minutes and she just keeps having them." The most accurate
description of this child's condition would be:
A. The child is in status epilepticus.
B. The child is having generalized seizures.
C. The child's history indicates she has infantile seizures.
D. The child may begin to have absence seizures every day.

Answer: A

Rationale: Status epilepticus is the term used to describe a seizure that lasts longer
than 30 minutes, or a series of seizures in which the child does not return to his or
her previous normal level of consciousness. The child likely is having generalized
seizures, but the most accurate description of what is happening is status
epilepticus. With infantile spasms, muscle contractions are sudden, brief,
symmetrical, and accompanied by rolling eyes. With absence seizures the child
loses awareness and stares straight ahead but does not fall.
Question format: Multiple Choice
Chapter 35: The Child with a Sensory/Neurologic Disorder
Cognitive Level: Understand
Client Needs: Physiological Integrity: Reduction of Risk Potential
Integrated Process: Nursing Process
Reference: p. 719
22. The caregiver of a child who has a history of seizures reports that her child was
complaining of being dizzy, and the caregiver noted the child seemed drowsy and
was clumsy. The nurse recognizes that the description of the child's behavior was
most likely which stage of a tonic-clonic seizure?
A. aura
B. clonic
C. postictal
D. prodromal

Answer: D

Rationale: During the prodromal period of a seizure, the child might be drowsy,
dizzy, or have a lack of coordination. The aura is a warning immediately before the
seizure and may relate to sensory signs. There are generalized jerking muscle
movements in the clonic phase. During the postictal period, the child may sleep
soundly and may have a period of confusion or stupor.
Question format: Multiple Choice
Chapter 35: The Child with a Sensory/Neurologic Disorder
Cognitive Level: Apply
Client Needs: Physiological Integrity: Physiological Adaptation
Integrated Process: Nursing Process
Reference: p. 719

23. The nurse caring for a 3-year-old child with a history of seizures observes the
child having a seizure. What information should the nurse document concerning the
event? Select all that apply.
A. Time the seizure started
B. Factors present before seizure started
C. Persons in attendance during seizure
D. Number of seizures child has had in the last 48 hours
E. Eye position and movement
F. Incontinence of urine or stool

Answer: A, B, E, F

Rationale: Following a seizure, the nurse documents the following: time the seizure
started; what the child was doing when the seizure began; any factor present just
before the seizure (bright light, noise); part of the body where seizure activity
began; movement and parts of the body involved; any cyanosis; eye position and
movement; incontinence of urine or stool; time seizure ended; and child's activity
after the seizure. Who was with the child or the number of seizures the child has
had are not relevant to document regarding observation of this seizure.
Question format: Multiple Select
Chapter 35: The Child with a Sensory/Neurologic Disorder
Cognitive Level: Apply
Client Needs: Physiological Integrity: Physiological Adaptation
Integrated Process: Communication and Documentation
Reference: p. 722
24. The nurse is preparing discharge education for the caregivers of a child with a
seizure disorder. Which goal of treatment is priority for this client?
A. The caregivers will be prepared to care for the child at home.
B. The child will have an understanding of the disorder.
C. The family will understand seizure precautions.
D. The child will remain free from injury during a seizure.

Answer: D

Rationale: Keeping the child free from injury is the priority goal. The other choices
are important, but keeping the child safe is higher than preparing for home care or
knowledge deficit concerns. The physical concerns are always priority over the
psychological concerns when caring for clients.
Question format: Multiple Choice
Chapter 35: The Child with a Sensory/Neurologic Disorder
Cognitive Level: Analyze
Client Needs: Safe, Effective Care Environment: Management of Care
Integrated Process: Nursing Process
Reference: p. 722

25. A symptom often seen in the child diagnosed with Haemophilus influenza
meningitis occurs when the child has a stiff neck. This symptom is referred to as
which of the following?
A. opisthotonos
B. nuchal rigidity
C. encephalopathy
D. purpuric rash

Answer: B

Rationale: Nuchal rigidity (stiff neck) is a symptom seen in Haemophilus influenzae


meningitis that may progress to opisthotonos (arching of the back). Encephalopathy
is an abnormal condition of the brain tissues. Bleeding under the skin causes a
purpuric rash.
Question format: Multiple Choice
Chapter 35: The Child with a Sensory/Neurologic Disorder
Cognitive Level: Remember
Client Needs: Physiological Integrity: Physiological Adaptation
Integrated Process: Nursing Process
Reference: p. 723

26. In caring for the child with meningitis, the nurse recognizes that which nursing
diagnosis would be most important to include in this child's plan of care?
A. Delayed growth and development related to physical restrictions
B. Risk for injury related to seizure activity
C. Risk for acute pain related to surgical procedure
D. Ineffective airway clearance related to history of seizures
Answer: B

Rationale: The child's risk for injury would be an appropriate nursing diagnosis.
Surgery is not indicated for the child with meningitis, and if the child has a history
of seizures, it would specifically impact airway clearance. Growth and development
issues are a concern but not likely delayed due to this diagnosis.
Question format: Multiple Choice
Chapter 35: The Child with a Sensory/Neurologic Disorder
Cognitive Level: Understand
Client Needs: Safe, Effective Care Environment: Management of Care
Integrated Process: Nursing Process
Reference: p. 724

27. Haemophilus influenzae meningitis is usually spread by which method of


transmission?
A. intravenous
B. fecal
C. droplet
D. contact

Answer: C

Rationale: Transmission of H. influenzae meningitis is by means of droplet infection


from an infected person; other forms of meningitis are contracted by an invasion of
the meninges via the bloodstream from an infection elsewhere. Fecal and contact
transmission is not how H. influenzae meningitis is spread or transmitted.
Question format: Multiple Choice
Chapter 35: The Child with a Sensory/Neurologic Disorder
Cognitive Level: Remember
Client Needs: Safe, Effective Care Environment: Safety and Infection Control
Integrated Process: Nursing Process
Reference: p. 723

28. The nurse is observing a group of children diagnosed with various types of
cerebral palsy. One of the children has an awkward and wide-based gait. The nurse
recognizes this characteristic as common in which type of cerebral palsy?
A. ataxic cerebral palsy
B. athetoid cerebral palsy
C. rigidity cerebral palsy
D. spastic cerebral palsy

Answer: A

Rationale: Ataxic cerebral palsy is essentially a lack of coordination caused by


disturbances in the kinesthetic and balance senses; it is characterized by an
awkward and wide-based gait. Athetoid cerebral palsy is marked by involuntary,
uncoordinated motion with varying degrees of muscle tension. Children with this
disorder are constantly in motion; the whole body is in a state of slow, writhing
muscle contractions whenever voluntary movement is attempted. Rigidity cerebral
palsy is uncommon and is characterized by rigid postures and lack of active
movement. Spastic cerebral palsy is characterized by scissoring caused by severe
hip adduction.
Question format: Multiple Choice
Chapter 35: The Child with a Sensory/Neurologic Disorder
Cognitive Level: Understand
Client Needs: Physiological Integrity: Physiological Adaptation
Integrated Process: Nursing Process
Reference: p. 727

29. The nurse is reinforcing teaching with the caregivers of a child who has had a
head injury and is being discharged. Which statement made by the caregiver
indicates an accurate understanding of the follow-up care for this child?
A. "If he falls asleep, we will wake him up every 15 minutes."
B. "We can give him acetaminophen for a headache, but no aspirin."
C. "Even if the flashlight bothers him, we will check his eyes."
D. "If he vomits again, we will bring him back immediately."

Answer: C

Rationale: The child's pupils are checked for reaction to light every 4 hours for 48
hours. If the child falls asleep, he or she should be awakened every 1 to 2 hours to
determine that the level of consciousness has not changed. No analgesics or
sedatives should be administered during this period of observation. The caregiver
should observe the child for at least 6 hours for vomiting or a change in the child's
level of consciousness. The caregiver should notify the health care provider
immediately if the child vomits more than three times, but if the child vomits once,
returning to the care provider immediately is not needed.
Question format: Multiple Choice
Chapter 35: The Child with a Sensory/Neurologic Disorder
Cognitive Level: Apply
Client Needs: Physiological Integrity: Reduction of Risk Potential
Integrated Process: Teaching/Learning
Reference: p. 735

30. A child is home with the caregivers following a treatment for a head injury. The
caregiver should contact the care provider if the child makes which statement?
A. "You look funny. Well, both of you do. I see two of you."
B. "My stomach is upset. I feel like I might throw up."
C. "I am glad that my headache is getting better."
D. "It will be nice when you will let me take a long nap. I am sleepy."

Answer: A

Rationale: The caregiver should notify the health care provider immediately if the
child vomits more than three times, has pupillary changes, has double or blurred
vision, has a change in level of consciousness, acts strange or confused, has trouble
walking, or has a headache that becomes more severe or wakes him or her from
sleep. These instructions should be provided in written form to the caregiver. Just
feeling nauseated is not a reason to notify the provider.
Question format: Multiple Choice
Chapter 35: The Child with a Sensory/Neurologic Disorder
Cognitive Level: Apply
Client Needs: Physiological Integrity: Physiological Adaptation
Integrated Process: Teaching/Learning
Reference: p. 735

31. Which of these age groups has the highest actual rate of death from drowning?
A. infants
B. toddlers
C. preschool children
D. school-aged children

Answer: B

Rationale: Toddlers and older adolescents have the highest actual rate of death
from drowning.
Question format: Multiple Choice
Chapter 35: The Child with a Sensory/Neurologic Disorder
Cognitive Level: Remember
Client Needs: Safe, Effective Care Environment: Safety and Infection Control
Integrated Process: Nursing Process
Reference: p. 735

32. The nurse is talking with the mother of a 4-year-old boy who will soon be going
to a pre-kindergarten program. The child has had the Snellen vision test done at
home, and he was unable to distinguish the pictures at the distance that would
indicate his vision is normal. The child's mother asks the nurse if he will need
glasses. Which statement made by the nurse would be most appropriate regarding
the child's vision?
A. "He might be suffering from hyperopia and probably will need glasses now."
B. "A child's vision is not completely developed by this age. Your child might
outgrow this nearsightedness."
C. "He is likely to have a slight astigmatism, which almost always needs to be
corrected by glasses."
D. "His vision problem will get in the way of his learning, so he will probably have to
have glasses before he starts school."

Answer: B

Rationale: Visual acuity of children gradually increases from birth, when the visual
acuity is usually between 20/100 and 20/400, until about 5 years of age, when
most children have 20/20 vision. Hyperopia (farsightedness) is a refractive
condition in which the person can see objects better at a distance than close up.
Astigmatism is caused by unequal curvatures in the cornea that bend the light rays
in different directions.
Question format: Multiple Choice
Chapter 35: The Child with a Sensory/Neurologic Disorder
Cognitive Level: Apply
Client Needs: Health Promotion and Maintenance
Integrated Process: Nursing Process
Reference: p. 709

33. The health care provider orders amoxicillin 35 mg/kg/day in three divided doses
for a child with otitis media. The child weighs 44 lb and the medication is available
in a suspension of 50 mg/ml. What is the total daily dosage in ml for this child?
A. 0.4 ml
B. 1.4 ml
C. 14.0 ml
D. 140 ml

Answer: C

Rationale: The child weighs 20 kg. The total daily dose would be 700 mg of
medication or a total of 14 ml of medication for the total daily dosage.
Question format: Multiple Choice
Chapter 35: The Child with a Sensory/Neurologic Disorder
Cognitive Level: Analyze
Client Needs: Physiological Integrity: Pharmacological and Parenteral Therapies
Integrated Process: Nursing Process
Reference: p. 716

34. When caring for a child who has a history of seizures, which nursing
interventions would be appropriate? Select all that apply.
A. The nurse pads the crib or side rails before a seizure.
B. The nurse positions the child on the side during a seizure.
C. The nurse places a washcloth in the mouth to prevent injury during seizure.
D. The nurse stays with the child and calls for help when a seizure begins.
E. The nurse has oxygen available to use during a seizure.
F. The nurse teaches the caregivers regarding seizure precautions.

Answer: A, B, D, E, F

Rationale: The nurse should pad the crib sides and keep sharp or hard items out of
the crib. The nurse should also position the child to one side to prevent aspiration
of saliva or vomitus and have oxygen and suction equipment readily available for
emergency use. The nurse should teach family caregivers seizure precautions so
they can handle a seizure that occurs at home. The nurse should not put anything
in the child's mouth; doing so could cause injury to the child or to the nurse. It is
important for the nurse to promptly inform other members of the care team when a
child is experiencing seizure activity, but leaving the bedside to do so would be
unsafe.
Question format: Multiple Select
Chapter 35: The Child with a Sensory/Neurologic Disorder
Cognitive Level: Apply
Client Needs: Physiological Integrity: Reduction of Risk Potential
Integrated Process: Nursing Process
Reference: p. 722

35. The nurse asks the caregivers of a child diagnosed with cerebral palsy whether
the child "seems to be in a state of constant motion." The question is designed to
identify what form of cerebral palsy?
A. ataxic
B. athetoid
C. rigidity
D. spastic

Answer: B

Rationale: Athetoid cerebral palsy is marked by involuntary, uncoordinated motion


with varying degrees of muscle tension. Children with this disorder are constantly in
motion; the whole body is in a state of slow, writhing muscle contractions whenever
voluntary movement is attempted. Ataxic cerebral palsy is essentially a lack of
coordination caused by disturbances in the kinesthetic and balance senses; it is
characterized by an awkward and wide-based gait. Rigidity cerebral palsy is
uncommon and is characterized by rigid postures and lack of active movement.
Spastic cerebral palsy is characterized by scissoring caused by severe hip
adduction. When scissoring is present, the child's legs are crossed and the toes are
pointed down. When standing, the child is on her or his toes. It is difficult for this
child to walk on the heels or run.
Question format: Multiple Choice
Chapter 35: The Child with a Sensory/Neurologic Disorder
Cognitive Level: Apply
Client Needs: Physiological Integrity: Physiological Adaptation
Integrated Process: Nursing Process
Reference: p. 727

36. The nurse is providing education to the caregivers of a child recently diagnosed
with tonic-clonic seizure disorder. What instructions should the nurse provide
related to the tonic stage of this type of seizure?
A. Be prepared for the child to report being dizzy.
B. Ask the child whether he or she is experiencing any unusual sensory sensations.
C. Monitor the child's breathing closely.
D. Be prepared for the child to be temporarily confused.

Answer: C

Rationale: Tonic–clonic seizures consist of four stages: the prodromal period, when
the child may be drowsy or dizzy; the aura, which is sensory event that serves as a
warning immediately before the seizure; the tonic–clonic movements, when the
child's muscles contract, the child may fall, and the child's extremities may stiffen.
The contraction of respiratory muscles during the tonic phase may cause the child
to become cyanotic and appear briefly to have respiratory arrest. During the
postictal period, the child may sleep soundly and may have a period of confusion or
stupor.
Question format: Multiple Choice
Chapter 35: The Child with a Sensory/Neurologic Disorder
Cognitive Level: Apply
Client Needs: Physiological Integrity: Physiological Adaptation
Integrated Process: Nursing Process
Reference: p. 719

37. The nurse cares for a 7-year-old child with new-onset seizure disorder. Which
prescription will the nurse anticipate for this client?
A. frequent temperature assessment
B. use of anticonvulsant medications
C. ketogenic diet
D. vagus nerve stimulation

Answer: B

Rationale: Complete control of seizures can be achieved for most people through
the use of anticonvulsant drug therapy. These medications are typically used first
as treatment for seizure disorders. Frequent temperature assessment would only be
useful in febrile seizures. Ketogenic diets (high in fat, low in carbohydrates, and
adequate in protein) cause the child to have high levels of ketones, which help to
reduce seizure activity. Diet is generally used when medications cannot control a
child's seizure activity. Stimulating the left vagus nerve intermittently with electrical
pulses may reduce seizure frequency. This requires surgically implanting a
stimulator under the skin and is approved for children 12 and older.
Question format: Multiple Choice
Chapter 35: The Child with a Sensory/Neurologic Disorder
Cognitive Level: Understand
Client Needs: Physiological Integrity: Pharmacological and Parenteral Therapies
Integrated Process: Nursing Process
Reference: p. 720
Chapter 36
1. A female nurse of childbearing ages caring for a 2-year-old child diagnosed with
bacterial pneumonia. The child has been placed in a mist tent. In caring for the
child, it is important for the nurse to:
A. monitor the child regularly for signs of cyanosis.
B. avoid contact with the mist.
C. use contact transmission precautions.
D. check for hyperthermia related to enclosure in the tent.

Answer: A

Rationale: In some treatment of bacterial pneumonia a croupette or mist tent is


used. When a child is in a mist tent, the nurse must be constantly observed for
cyanosis. Studies how that children have become cyanotic in mist tents and have
suffer subsequent arrest, due to the lack of visibility while in the tent. Treatments
for bacterial pneumonia do not carry risk for teratogenicity. Ribavirin is an antiviral
drug that may be used to treat certain children with respiratory syncytial virus. It is
administered as an inhalant by hood, mask, or tent and has a high risk for
teratogenicity. In treating a client with bacterial pneumonia, the client may need to
be placed on infection control precautions according to the policy of the health care
facility; in most facilities, these will be droplet precautions. Although the nurse
should look for hyperthermia related to the infection process, there is no risk for
hyperthermia related to the child being enclosed in the tent.
Question format: Multiple Choice
Chapter 36: The Child with a Respiratory Disorder
Cognitive Level: Apply
Client Needs: Physiological Integrity: Reduction of Risk Potential
Integrated Process: Nursing Process
0 p. 757

2. The caregivers of a child who was diagnosed with cystic fibrosis 5 months ago
report that they have been following all of the suggested guidelines for nutrition,
fluid intake, and exercise, but the child has been having bouts of constipation and
diarrhea. The nurse tells the caregiver to increase the amount of which substance in
the child's diet?
A. Iodized salt
B. Saturated fat
C. Pancreatic enzymes
D. Calories from protein

Answer: C

Rationale: Adequate nutrition helps the child resist infections. Pancreatic enzymes
must be administered with all meals and snacks. If the child has bouts of diarrhea
or constipation, the dosage of enzymes may need to be adjusted. The child's diet
should be high in carbohydrates and protein with no restriction of fats. The child
may need 1.5 to 2 times the normal caloric intake to promote growth. Low-fat
products can be selected if desired. The child also may require additional salt in the
diet. Increased caloric intake compensates for impaired absorption.
Question format: Multiple Choice
Chapter 36: The Child with a Respiratory Disorder
Cognitive Level: Understand
Client Needs: Physiological Integrity: Pharmacological and Parenteral Therapies
Integrated Process: Nursing Process
Reference: p. 760

3. The nurse is teaching the caregivers of a child with cystic fibrosis. What is most
important for the nurse to teach this family?
A. Be sure the child exercises daily.
B. Watch out for signs that family members are overly stressed.
C. Avoid overprotecting the child.
D. Encourage everyone in the family to use good handwashing techniques.

Answer: D

Rationale: The child with cystic fibrosis has low resistance, especially to respiratory
infections. For this reason, take care to protect the child from any exposure to
infectious organisms. Good handwashing techniques should be practiced by the
whole family; teach the child and family the importance of this first line of defense.
Practice and teach other good hygiene habits.
Question format: Multiple Choice
Chapter 36: The Child with a Respiratory Disorder
Cognitive Level: Understand
Client Needs: Safe, Effective Care Environment: Safety and Infection Control
Integrated Process: Teaching/Learning
Reference: p. 764

4. The nurse is doing discharge teaching for a child who has had a tonsillectomy.
The nurse tells the client and family that the child should have plenty of fluids. In
addition, the nurse would explain to the child's caregiver that the child may:
A. have a painful earache around the third day postoperatively, but the earache will
be gone by the fourth day.
B. vomit dark, old blood, but the caregiver should call the clinic if the child has
bleeding between the fifth and seventh days postoperatively.
C. be given ice cream and milk the first postoperative day because these foods
make swallowing easier.
D. have severe throat pain for up to 2 weeks postoperatively; this is not a concern.

Answer: B

Rationale: Bleeding is most often a concern within the first 24 hours following
surgery and between the fifth to seventh days postoperatively. Bright, red-flecked
emesis or oozing indicates fresh bleeding. If at any time following the surgery there
is bright red bleeding, frequent swallowing, or restlessness, the care provider
should be notified. A mild earache may be expected around the third day.
Encourage fluid intake but avoid irritating liquids such as orange juice. Be aware
that milk and ice cream products tend to cling to the surgical site and make
swallowing more difficult; thus they are poor choices despite the old tradition of
offering ice cream after a tonsillectomy.
Question format: Multiple Choice
Chapter 36: The Child with a Respiratory Disorder
Cognitive Level: Apply
Client Needs: Physiological Integrity: Reduction of Risk Potential
Integrated Process: Teaching/Learning
Reference: p. 746

5. The caregivers of an 8-year-old bring their child to the pediatrician and report
that the child has not had breathing problems before, but since taking up lacrosse
the child has been coughing and wheezing at the end of every practice and game.
Their friend's child has often been hospitalized for asthma; they are concerned that
their child has a similar illness. The nurse knows that because the problems seem
to be directly related to exercise, it is likely that the child will be able to be treated
with:
A. decreased activity and increased fluids.
B. corticosteroids and leukotriene inhibitors.
C. removal of allergens in the home and school.
D. a bronchodilator and mast cell stabilizers.

Answer: D

Rationale: Mast cell stabilizers are used to help decrease wheezing and exercise-
induced asthma attacks. A bronchodilator often is given to open up the airways just
before the mast cell stabilizer is used. Corticosteroids are anti-inflammatory drugs
used to control severe or chronic cases of asthma. Leukotriene inhibitors are given
by mouth along with other asthma medications for long-term control and
prevention of mild, persistent asthma.
Question format: Multiple Choice
Chapter 36: The Child with a Respiratory Disorder
Cognitive Level: Apply
Client Needs: Physiological Integrity: Pharmacological and Parenteral Therapies
Integrated Process: Nursing Process
Reference: p. 749

6. The nurse is doing an in-service training with nurses working with families who
may be in situations that create high-risk health situations for their children. The
nurse explains that children of caregivers with which situation should be tested
annually for tuberculosis?
A. a caregiver with a diagnosis of HIV
B. a caregiver who has been serving in the military in the Middle East
C. a caregiver whose family is homeless
D. a caregiver who immigrated from Africa before her child was born
Answer: C

Rationale: Tuberculosis screening is recommended annually for children in high-risk


situations or communities, including the following: a family in which there is an
active case of TB; Native American children; and children who recently immigrated
from Central or South America, the Caribbean, Africa, Asia, or the Middle East.
Other high-risk children are those infected with HIV, those who are homeless or live
in overcrowded conditions, and those immunosuppressed from any cause.
Question format: Multiple Choice
Chapter 36: The Child with a Respiratory Disorder
Cognitive Level: Understand
Client Needs: Health Promotion and Maintenance
Integrated Process: Nursing Process
Reference: p. 766

7. The caregiver of a 6-month-old boy calls the nurse concerned about her child.
The child has been irritable, fussy, and is sneezing. The child's temperature is 100oF
(37.8oC). The nurse suspects that the cause of the symptoms is:
A. pneumonia.
B. a pollen-based allergy.
C. cystic fibrosis.
D. a common cold.

Answer: D

Rationale: The child with a common cold sneezes and becomes irritable, fussy and
restless. The congested nasal passages can interfere with nursing, increasing the
infant's irritability. Because an older child can mouth breathe, nasal congestion is
not as great a concern as it is in the infant. The infant usually develops fever early
in the course of the infection, often as high as 102°F to 104°F (38.9°C to 40°C),
The child with an allergy woulld have more symptoms and will not likely have an
elevated temperature The infant with pneumonia will most likely have a rapid and
abrupt temperature increase, possibly to 103°F to 105°F (39.4°C to 40.6°C) and
marked respiratory distress with obvious air hunger, flaring of the nostrils,
circumoral (around the mouth) cyanosis, and chest retractions. The child with cystic
fibrosis will have a hard, nonproductive chronic cough, a barrel chest, and clubbing
of fingers. The abdomen becomes distended, and body muscles become flabby.
Question format: Multiple Choice
Chapter 36: The Child with a Respiratory Disorder
Cognitive Level: Apply
Client Needs: Physiological Integrity: Physiological Adaptation
Integrated Process: Nursing Process
Reference: p. 743-744

8. The nurse is bottle feeding an infant diagnosed with pneumonia. An important


action for the nurse to take is clearing the infant's nose, and then the nurse should
feed the infant using:
A. a large-holed nipple so that the infant doesn't have to work too hard.
B. the smallest-holed nipple available so that the infant can exercise the muscles.
C. a nipple that is small enough so that the baby doesn't choke, but not so small
that they have to work too hard to eat.
D. a nipple specially designed to fit an infant's mouth.

Answer: C

Rationale: For the infant with pneumonia, use the smallest nipple so that he or she
does not choke, but also does not have to work too hard. A large-holed nipple
would allow the fluids to flow too quickly and likely choke the infant. Working too
hard with a small-holed nipple would wear the infant out and require the use of
excessive energy. There is no need for a special nipple.
Question format: Multiple Choice
Chapter 36: The Child with a Respiratory Disorder
Cognitive Level: Apply
Client Needs: Physiological Integrity: Basic Care and Comfort
Integrated Process: Nursing Process
Reference: p. 757

9. What statement is the most accurate regarding the structure and function of the
newborn's respiratory system?
A. The diameter of the child's trachea is the same as that of adults.
B. Most infants are nasal breathers rather than mouth breathers.
C. The respiratory tract in the child is fully developed by age 2.
D. Infants and young children have smaller tongues in proportion to their mouths.

Answer: B

Rationale: Newborns are obligatory nose breathers until at least 4 weeks of age.
The diameter of the infant and child's trachea is about the size of the child's little
finger. The respiratory tract grows and changes until the child is about 12 years of
age. During the first 5 years of life, infants and young children have larger tongues
in proportion to their mouths.
Question format: Multiple Choice
Chapter 36: The Child with a Respiratory Disorder
Cognitive Level: Remember
Client Needs: Physiological Integrity: Physiological Adaptation
Integrated Process: Nursing Process
Reference: p. 743

10. The nurse is working with a group of caregivers of small children discussing
various disorders seen in children. One of the caregivers makes the statement that
her children always seem to have a common cold. After discussing this condition,
the caregivers make the following statements. Which statement indicates the most
accurate understanding of a complication related to the common cold?
A. "When my 6-month-old gets a cold, I know her temperature will be high."
B. "At least with a cold they only have diarrhea for a few days."
C. "Next time he has a cold, I will watch closely to see if my 1-year-old pulls at his
ears."
D. "Last week my son came home from school with a cold and I gave him Tylenol
for the pain."

Answer: C

Rationale: The most common complication of a common cold is otitis media. If the
symptoms persist for several days, the child must be seen by a physician to rule
out complications such as otitis media. Most young children do not have a fever
with a common cold. The child may have mild diarrhea caused by mucus drainage
into the digestive system, but this is not a major complication seen with the
condition. The school-age child can be given acetaminophen, but pain is not a
complication associated with the common cold.
Question format: Multiple Choice
Chapter 36: The Child with a Respiratory Disorder
Cognitive Level: Analyze
Client Needs: Physiological Integrity: Physiological Adaptation
Integrated Process: Teaching/Learning
Reference: p. 744

11. The caregivers of a child report that their child had a cold and complained of a
sore throat. When interviewed further they report that the child has a high fever, is
very anxious, and is breathing by sitting up and leaning forward with the mouth
open and the tongue out. The nurse recognizes these symptoms as those seen with
which disorder?
A. Spasmodic laryngitis
B. Tonsillitis
C. Laryngotracheobronchitis
D. Epiglottitis

Answer: D

Rationale: The child with epiglottitis may have had a mild upper respiratory
infection before the development of a sore throat, and then became anxious and
prefers to breathe by sitting up and leaning forward with the mouth open and the
tongue out. The child with tonsillitis may have a fever, sore throat, difficulty
swallowing, hypertrophied tonsils, and erythema of the soft palate. Exudate may be
visible on the tonsils. The child with acute laryngotracheobronchitis develops
hoarseness and a barking cough with a fever, cyanosis, heart failure; acute
respiratory embarrassment can also result.
Question format: Multiple Choice
Chapter 36: The Child with a Respiratory Disorder
Cognitive Level: Apply
Client Needs: Physiological Integrity: Physiological Adaptation
Integrated Process: Nursing Process
Reference: p. 748
12. The nurse is assigned four pediatric clients with diagnoses of acute bronchiolitis
and acute interstitial pneumonia. When discussing the history of the disorder, the
nurse anticipates which for the age of onset?
A. "It is most often seen in premature newborns right after they are born."
B. "Most children who have this are about 6 months old."
C. "It usually occurs when children start school, around the age of 5."
D. "Most children with this disorder are in their teens."

Answer: B

Rationale: Acute bronchiolitis (acute interstitial pneumonia) is most common during


the first 6 months of life and is rarely seen after the age of 2 years.
Question format: Multiple Choice
Chapter 36: The Child with a Respiratory Disorder
Cognitive Level: Apply
Client Needs: Physiological Integrity: Physiological Adaptation
Integrated Process: Nursing Process
Reference: p. 748

13. The nurse is working with a group of caregivers of children diagnosed with
asthma. Which statement made by a caregiver is most accurate regarding the
triggers that may cause an asthma attack?
A. "My neighbor told me that asthma attacks are caused by hot weather."
B. "I always thought that a lack of exercise caused my child's asthma."
C. "My sister and her family love animals, and when we go to their house my
daughter always has an asthma attack."
D. "One person told me that asthma is caused by using antibiotics for infection."

Answer: C

Rationale: Asthma may be a response to certain foods, or may be triggered by


exercise or exposure to cold weather. Irritants such as wood-burning stoves,
cigarette smoke, dust, pet dander, and foods such as chocolate, milk, eggs, nuts,
and grains may also aggravate the condition. Additionally, infections such as
bronchitis and upper respiratory infection can provoke asthma attacks. Using
antibiotics to treat infections does not cause an asthma attack.
Question format: Multiple Choice
Chapter 36: The Child with a Respiratory Disorder
Cognitive Level: Apply
Client Needs: Physiological Integrity: Physiological Adaptation
Integrated Process: Nursing Process
Reference: p. 749

14. The nurse is caring for a child admitted with asthma. Which clinical
manifestations would likely have been noted in the child with this diagnosis?
A. Elevated temperature
B. Wheezing
C. Circumoral cyanosis
D. Clubbed fingers

Answer: B

Rationale: Symptoms of asthma include dry hacking cough, wheezing (the sound of
expired air being pushed through obstructed bronchioles), and difficulty breathing.
Elevated temperature is not usually seen. Circumoral cyanosis is seen with a
diagnosis of pneumonia, and clubbing of the fingers is seen in cystic fibrosis.
Question format: Multiple Choice
Chapter 36: The Child with a Respiratory Disorder
Cognitive Level: Apply
Client Needs: Physiological Integrity: Physiological Adaptation
Integrated Process: Nursing Process
Reference: p. 749

15. The nurse is caring for a child who has been admitted with a diagnosis of
asthma. What laboratory/diagnostic tool would likely have been used for this child?
A. Purified protein derivative test
B. Sweat sodium chloride test
C. Blood culture and sensitivity
D. Pulmonary functions test

Answer: D

Rationale: Pulmonary function tests are valuable diagnostic tools for the child with
asthma and indicate the amount of obstruction in the bronchial airways, especially
in the smallest airways of the lungs. Purified protein derivative tests are used to
detect TB. Sweat sodium chloride tests are used for determining the diagnosis of
cystic fibrosis. Blood culture and sensitivity is done to determine the causative
agent as well as the anti-infective needed to treat an infection.
Question format: Multiple Choice
Chapter 36: The Child with a Respiratory Disorder
Cognitive Level: Remember
Client Needs: Physiological Integrity: Reduction of Risk Potential
Integrated Process: Nursing Process
Reference: p. 750

16. The nurse is caring for a child who has been admitted with a possible diagnosis
of cystic fibrosis. Which laboratory/diagnostic tools would most likely be used to
help determine the diagnosis of this child?
A. Purified protein derivative test
B. Sweat sodium chloride test
C. Blood culture and sensitivity
D. Pulmonary functions test

Answer: B
Rationale: Sweat sodium chloride tests are used for determining the diagnosis of
cystic fibrosis. Purified protein derivative tests are used to detect TB. Blood culture
and sensitivity is done to determine the causative agent as well as the anti-infective
needed to treat an infection. Pulmonary function tests are diagnostic tools for the
child with asthma and indicate the amount of obstruction in the bronchial airways,
especially in the smallest airways of the lungs.
Question format: Multiple Choice
Chapter 36: The Child with a Respiratory Disorder
Cognitive Level: Remember
Client Needs: Physiological Integrity: Reduction of Risk Potential
Integrated Process: Nursing Process
Reference: p. 760

17. The nurse is caring for a child who has been admitted with a possible diagnosis
of tuberculosis. Which laboratory/diagnostic tools would most likely be used to help
diagnose this child?
A. Purified protein derivative test
B. Sweat sodium chloride test
C. Blood culture and sensitivity
D. Pulmonary functions test

Answer: A

Rationale: Purified protein derivative tests are used to detect TB. Sweat sodium
chloride tests are used for determining the diagnosis of cystic fibrosis. Blood culture
and sensitivity is done to determine the causative agent as well as the anti-infective
needed to treat an infection. Pulmonary function tests are diagnostic tools for the
child with asthma and indicate the amount of obstruction in the bronchial airways,
especially in the smallest airways of the lungs.
Question format: Multiple Choice
Chapter 36: The Child with a Respiratory Disorder
Cognitive Level: Remember
Client Needs: Physiological Integrity: Reduction of Risk Potential
Integrated Process: Nursing Process
Reference: p. 766

18. The LPN is working with the RN to develop a plan of care for a child with
asthma. Which would be appropriate goals of treatment for this child? Select all
that apply.
A. preventing symptoms
B. decreasing activity levels
C. preventing recurrence
D. decreasing fluid intake
E. relieving pain

Answer: A, C
Rationale: The goals of asthma treatment includes preventing symptoms,
maintaining near-normal lung function and activity levels, preventing recurring
exacerbations and hospitalizations, and providing the best medication treatment
with the fewest adverse effects. Pain is not a major concern for the child with
asthma.
Question format: Multiple Select
Chapter 36: The Child with a Respiratory Disorder
Cognitive Level: Apply
Client Needs: Safe, Effective Care Environment: Management of Care
Integrated Process: Nursing Process
Reference: p. 751

19. The nurse is reinforcing teaching with a group of caregivers of children


diagnosed with asthma. Which statement best indicates an understanding of the
management and treatment for this diagnosis?
A. "We have taken the carpet out of our house and let my mom take our dog."
B. "He knows how and even when he needs to use his peak flow meter."
C. "Even the babysitter helps us keep up the diary with her symptoms."
D. "The medications she takes are all in one place, ready for her to take at any
time."

Answer: A

Rationale: Families must make every effort to eliminate any possible allergens from
the home. Prevention is the most important aspect in the treatment of asthma.
Learning how to use a peak flow meter, using a peak flow and symptom diary, and
having the medications available are important aspects of treatment, but
prevention is the best.
Question format: Multiple Choice
Chapter 36: The Child with a Respiratory Disorder
Cognitive Level: Analyze
Client Needs: Physiological Integrity: Physiological Adaptation
Integrated Process: Teaching/Learning
Reference: p. 750

20. The nurse at a camp for children with asthma is teaching these children about
the medications they are taking and how to properly take them. The nurse
recognizes that many medications used on a daily basis for the treatment of asthma
are given by which method?
A. Directly into the vein
B. Through a gastrostomy tube
C. Using a nebulizer
D. Sprinkled onto the food

Answer: C

Rationale: Many of these drugs used in the treatment of asthma can be given either
by a nebulizer (tube attached to a wall unit or cylinder that delivers moist air via a
face mask) or a metered-dose inhaler ([MDI], which is a hand-held plastic device
that delivers a premeasured dose). Emergency medications are given
intravenously. Most children do not have a gastrostomy tube, and medications
sprinkled on foods are given with cystic fibrosis.
Question format: Multiple Choice
Chapter 36: The Child with a Respiratory Disorder
Cognitive Level: Apply
Client Needs: Physiological Integrity: Pharmacological and Parenteral Therapies
Integrated Process: Nursing Process
Reference: p. 751

21. The nurse is reinforcing teaching about medications with the parents of a 2-
year-old who has cystic fibrosis. The nurse suggests that pancreatic enzymes may
be given by which method?
A. Directly into the vein
B. Through a gastrostomy tube
C. Using a nebulizer
D. Sprinkled onto the food

Answer: D

Rationale: Pancreatic enzymes are used in the treatment of cystic fibrosis and are
given by opening the capsule and sprinkling the medication on the child's food. If
the child with cystic fibrosis has an infection, IV medications may be given, but this
is not on a daily basis. Most children do not have a gastrostomy tube. Many of
these drugs used in the treatment of asthma can be given either by a nebulizer
(tube attached to a wall unit or cylinder that delivers moist air via a face mask) or a
metered-dose inhaler [MDI], which is a hand-held plastic device that delivers a
premeasured dose.
Question format: Multiple Choice
Chapter 36: The Child with a Respiratory Disorder
Cognitive Level: Apply
Client Needs: Physiological Integrity: Pharmacological and Parenteral Therapies
Integrated Process: Teaching/Learning
Reference: p. 760

22. During a class for caregivers of children with asthma, a caregiver asks the
nurse the following question when medications are being discussed. "They told me
about a plastic device my child can hold in his a hand which will give him a
premeasured and exact amount of his corticosteroid." The nurse recognizes that the
caregiver is most likely referring to which device?
A. Medication cup
B. Metered-dose inhaler
C. Nebulizer
D. Needleless syringe

Answer: B
Rationale: In the treatment of asthma, corticosteroids are most often delivered by
metered-dose inhaler ([MDI], which is a hand-held plastic device that delivers a
premeasured dose). The medication cup and needleless syringe may deliver PO
medications, but most often corticosteroids are not given PO in the treatment of
asthma, and those would not be premeasured and an exact dosage like a metered-
dose inhaler would be. Corticosteroids are not administered by nebulizer.
Question format: Multiple Choice
Chapter 36: The Child with a Respiratory Disorder
Cognitive Level: Apply
Client Needs: Physiological Integrity: Pharmacological and Parenteral Therapies
Integrated Process: Nursing Process
Reference: p. 751

23. In caring for the child with asthma, the nurse recognizes that bronchodilator
medications are administered to children with asthma for which reason?
A. Relief of acute symptoms
B. Management of chronic pain
C. To stabilize the cell membranes
D. Prevention of mild symptoms

Answer: A

Rationale: Bronchodilators are used for quick relief of acute exacerbations of


asthma symptoms. Mast cell stabilizers help to stabilize the cell membrane by
preventing mast cells from releasing the chemical mediators that cause
bronchospasm and mucous membrane inflammation. Leukotriene inhibitors are
given by mouth along with other asthma medications for long-term control and
prevention of mild, persistent asthma. Bronchodilators are not effective for pain.
Question format: Multiple Choice
Chapter 36: The Child with a Respiratory Disorder
Cognitive Level: Apply
Client Needs: Physiological Integrity: Pharmacological and Parenteral Therapies
Integrated Process: Nursing Process
Reference: p. 751

24. In caring for the child with asthma, the nurse recognizes that which nursing
diagnosis would be the highest priority in this child's plan of care?
A. Delayed growth and development related to physical restrictions
B. Risk for fluid volume excess related to medications
C. Risk for infection related to anatomic structures of involved body system
D. Ineffective airway clearance related to the diagnosis

Answer: D

Rationale: The highest priority for the child with asthma is to keep the airway clear
because of the bronchospasms and increased pulmonary secretions the child may
have. The child is more likely to have deficient fluid volume related to tachypnea
and diaphoresis. Infections can occur, but they are less of a concern than the
airway clearance. Growth and development issues can occur because the child may
have to limit activities, but these issues are not the priority.
Question format: Multiple Choice
Chapter 36: The Child with a Respiratory Disorder
Cognitive Level: Apply
Client Needs: Safe, Effective Care Environment: Management of Care
Integrated Process: Nursing Process
Reference: p. 753

25. The nurse caring for the child with asthma weighs the child daily. What is the
most important reason for doing a daily weight on this child?
A. To determine medication dosages
B. To monitor the child's growth pattern
C. To ensure that the child's food intake is adequate
D. To determine fluid losses

Answer: D

Rationale: During an acute asthma attack the child may lose a great quantity of
fluid through the respiratory tract and may have poor oral intake because of
coughing and vomiting. Theophylline administration also has a diuretic effect, which
compounds the problem. Weigh the child daily to help determine fluid losses. The
child's weight is used to determine medication dosages, to ensure that the child is
appropriately gaining weight and growing, and that the intake is adequate.
However, the most important reason for a daily weight is to determine fluid loss.
Question format: Multiple Choice
Chapter 36: The Child with a Respiratory Disorder
Cognitive Level: Apply
Client Needs: Physiological Integrity: Physiological Adaptation
Integrated Process: Nursing Process
Reference: p. 753

26. A group of nurses is reviewing the diagnosis of cystic fibrosis. With regard to
the effect of this disease on the body, which parts of the body (besides the lungs)
are most affected by this disease?
A. Brain and spinal cord
B. Pancreas and liver
C. Heart and blood vessels
D. Kidney and bladder

Answer: B

Rationale: The major organs affected are the lungs, pancreas, and liver. The brain,
spinal cord, heart, blood vessels, kidney and bladder are not the most affected
organs.
Question format: Multiple Choice
Chapter 36: The Child with a Respiratory Disorder
Cognitive Level: Remember
Client Needs: Physiological Integrity: Physiological Adaptation
Integrated Process: Nursing Process
Reference: p. 759

27. The student nurse is collecting data on a child diagnosed with cystic fibrosis and
notes the child has a barrel chest and clubbing of the fingers. In explaining this
manifestation of the disease, the staff nurse explains the cause of this symptom to
be:
A. impaired digestive activity.
B. high sodium chloride concentration in the sweat.
C. chronic lack of oxygen.
D. decreased respiratory capacity.

Answer: C

Rationale: In the child with cystic fibrosis the development of a barrel chest and
clubbing of fingers indicate chronic lack of oxygen. Impaired digestive activity may
occur due to a lack of pancreatic enzymes. The high sodium concentration makes
the child taste salty, but is not related to the barrel chest and clubbing of the
fingers. Respiratory issues are a concern, but the barrel chest and clubbing of the
fingers are not because of the child's respiratory capacity.
Question format: Multiple Choice
Chapter 36: The Child with a Respiratory Disorder
Cognitive Level: Understand
Client Needs: Physiological Integrity: Physiological Adaptation
Integrated Process: Teaching/Learning
Reference: p. 759

28. The nurse is reinforcing teaching with the family caregivers of a child diagnosed
with tuberculosis who is being treated with the drug rifampin. Which statement
made by the caregivers indicates an understanding of this medication?
A. "My son will have to take this medication the rest of his life."
B. "While she is taking this medication, I won't worry if her tears look orange."
C. "This medication may cause slight bleeding when she urinates."
D. "He will not be able to attend school for the first few months that he is on this
medication."

Answer: B

Rationale: Rifampin is tolerated well by children, but causes body fluids such as
urine, sweat, tears, and feces to turn orange-red. Drug therapy is continued for 9
to 18 months. After drug therapy has begun, the child or adolescent may return to
school and normal activities. Although the urine may be orange-red, this does not
indicate bleeding. If bleeding with urination presents, then it should be reported
and followed up on.
Question format: Multiple Choice
Chapter 36: The Child with a Respiratory Disorder
Cognitive Level: Analyze
Client Needs: Physiological Integrity: Pharmacological and Parenteral Therapies
Integrated Process: Teaching/Learning
Reference: p. 766

29. The nurse is collecting data on a child with a diagnosis of tonsillitis. Which
clinical manifestation would likely have been noted in the child with this diagnosis?
A. bark-like cough
B. hoarseness
C. erythema of the pharynx
D. inability to make audible voice sounds

Answer: C

Rationale: The child with tonsillitis may have a fever of 101°F (38.4°C) or higher, a
sore throat, often with dysphagia (difficulty swallowing), hypertrophied tonsils, and
erythema of the pharynx. The child with spasmodic laryngitis has a bark-like cough,
hoarseness, and an inability to make audible voice sounds.
Question format: Multiple Choice
Chapter 36: The Child with a Respiratory Disorder
Cognitive Level: Understand
Client Needs: Physiological Integrity: Physiological Adaptation
Integrated Process: Nursing Process
Reference: p. 744

30. When caring for a child with acute bronchiolitis, which nursing interventions
should be included in the plan of care? Select all that apply.
A. Encourage fluids.
B. Administer oxygen.
C. Suction the nose.
D. Administer antibiotics.
E. Follow contact precautions.
F. Encourage activity.

Answer: A, B, C, E

Rationale: The child is treated with nasal suctioning, rest, and increased fluids.
Humidified oxygen may be administered via a nasal cannula, face mask, or an
oxyhood (a clear plastic hood placed over the infant's head) if needed. Antibiotics
are not prescribed because the causative organism is a virus. IV fluids often are
administered to ensure an adequate intake and to permit the infant to rest. The
hospitalized child is placed on contact transmission precautions to prevent the
spread of infection.
Question format: Multiple Select
Chapter 36: The Child with a Respiratory Disorder
Cognitive Level: Apply
Client Needs: Physiological Integrity: Physiological Adaptation
Integrated Process: Nursing Process
Reference: p. 748
31. The nurse makes the statement that if an older child inhales a foreign body, the
inhaled object is more likely to be drawn into the right bronchus rather than the
left. What is the basis for this statement?
A. The left bronchus is shorter and wider than the right.
B. The right bronchus is shorter and wider than the left.
C. Both bronchi are the same size, but the left is more vertical than the right.
D. The left bronchus is more vertical than the right.

Answer: B

Rationale: In children older than 2 years, the right bronchus is shorter, wider, and
more vertical than the left. If the child inhales a foreign body, it is more likely to be
drawn into the right bronchus rather than the left.
Question format: Multiple Choice
Chapter 36: The Child with a Respiratory Disorder
Cognitive Level: Understand
Client Needs: Health Promotion and Maintenance
Integrated Process: Nursing Process
Reference: p. 743

32. When assessing a child for the probable cause of acute bronchiolitis, the nurse
focuses on which factor?
A. Bacterial infections
B. Environmental allergies
C. Prenatal complications
D. Viral infections

Answer: D

Rationale: Acute bronchiolitis is caused by a viral not bacterial infection. Neither


allergies nor prenatal complications contribute to the development of this disorder.
Question format: Multiple Choice
Chapter 36: The Child with a Respiratory Disorder
Cognitive Level: Understand
Client Needs: Physiological Integrity: Physiological Adaptation
Integrated Process: Nursing Process
Reference: p. 743

33. The caregivers bring a 2-year-old child to the emergency care center after
becoming concerned when the child woke up in the middle of the night with a bark-
like cough and labored breathing. Which assessment question related to the child's
condition earlier in the day would the nurse ask to rule out the possibility of
spasmodic laryngitis?
A. "Did the child have a runny nose?"
B. "Was there any reason to think the child had difficulty swallowing?"
C. "Did you notice the child drooling?"
D. "Did the child breathe through the mouth rather than the nose?"
Answer: A

Rationale: The child with spasmodic laryngitis often presents with a runny nose
prior to the respiratory event. The child with spasmodic laryngitis often awakens
after a few hours of sleep with a bark-like cough and increasing respiratory
difficulty. The child with epiglottitis may have a sore throat, often with dysphagia
(difficulty swallowing and resulting drooling) and demonstrates mouth breathing
with the tongue protruding out of the mouth.
Question format: Multiple Choice
Chapter 36: The Child with a Respiratory Disorder
Cognitive Level: Apply
Client Needs: Physiological Integrity: Reduction of Risk Potential
Integrated Process: Nursing Process
Reference: p. 747

34. The caregiver of a child diagnosed with allergic rhinitis asks the nurse working
in the allergy clinic why her child frequently pushes his nose upward and backward.
Which response accurately explains this action?
A. The child is attempting to draw attention to his/her nose.
B. The child has a nasal discharge and he is trying to keep his nose from running.
C. The child is concerned that he is going to sneeze and is trying to stop the
sneeze.
D. The child is trying to relieve the itching and open the air passages.

Answer: D

Rationale: The "allergic salute" is an action in which the child pushes his or her
nose upward and backward to relieve itching and open the air passages in the nose.
The child with allergic rhinitis may have a nasal discharge and frequently sneeze,
but this action does not have any relevance to these symptoms.
Question format: Multiple Choice
Chapter 36: The Child with a Respiratory Disorder
Cognitive Level: Apply
Client Needs: Physiological Integrity: Physiological Adaptation
Integrated Process: Nursing Process
Reference: p. 744

35. A 4-week-old infant is diagnosed with acute bronchiolitis. The parent states, "I
do not know how the baby got this!" How should the nurse respond?
A. "Do you have allergies in the family?"
B. "Do any family members have history of asthma?"
C. "Do you have air conditioning in your house?"
D. "Has your infant been around any crowds?"

Answer: D
Rationale: Acute bronchiolitis is caused by a viral infection, most often, respiratory
syncytial virus. Viruses are often spread between groups of people in close contact.
Hereditary and environmental complications do not relate to this disorder.
Question format: Multiple Choice
Chapter 36: The Child with a Respiratory Disorder
Cognitive Level: Apply
Client Needs: Physiological Integrity: Physiological Adaptation
Integrated Process: Nursing Process
Reference: p. 743

36. A 2-year-old child has had a common cold for 4 days. The caregiver calls the
nurse in the emergency department at 2 a.m. on a cold winter night to say that the
child has awakened with a barking cough and an elevated temperature; the child
seems blue around the mouth. The nurse would appropriately recommend what
action to the caregiver?
A. "Put a cool mist humidifier or vaporizer in the room to see if that relieves the
cough. Call back if there is no relief in an hour."
B. "Bundle the child up and take the child out into the cold for a few minutes. Call
back if the exposure to the cold air does not provide relief."
C. "Bring the child to the emergency room immediately."
D. "Turn on all of the hot water taps in the bathroom and close the door. Take the
child into the steam-filled room for 15 minutes. If there is no relief, bring the child
to the emergency room."

Answer: C

Rationale: Acute laryngotracheobronchitis generally occurs after an upper


respiratory infection with fairly mild rhinitis and pharyngitis. The child develops
hoarseness and a barking cough with a fever that may reach 104° F (40° C) to
105° F (40.6° C). As the disease progresses, marked laryngeal edema occurs and
the child's breathing becomes difficult; the pulse is rapid and cyanosis may appear.
Heart failure and acute respiratory distress can result. The child needs to be treated
immediately. Humidified air is helpful in reducing laryngospasm; humidifiers may
be used in the child's bedroom to provide high humidity. Cool humidifiers are
recommended, but vaporizers also may be used. Taking the child into the bathroom
and opening the hot water taps with the door closed is a quick method for providing
moist air, if the water runs hot enough. Sometimes the spasm is relieved by
exposure to cold air: for instance, when the child is taken out into the night to go to
the emergency department or to see the physician.
Question format: Multiple Choice
Chapter 36: The Child with a Respiratory Disorder
Cognitive Level: Apply
Client Needs: Physiological Integrity: Physiological Adaptation
Integrated Process: Nursing Process
Reference: p. 747
Chapter 37
1. A nurse admits an infant with a possible diagnosis of congestive heart failure.
Which signs or symptoms would the infant most likely be exhibiting?
A. Feeding problems
B. Bradypnea
C. Bradycardia
D. Yellowish color

Answer: A

Rationale: The indications of CHF vary in children of different ages. Signs in the
infant may be hard to detect because they are subtle, but in infants, feeding
problems are often seen. In infants and older children, tachycardia is one of the
first signs of CHF. In a child with CHF, tachypnea would be seen, not bradypnea.
The heart beats faster in an attempt to increase blood flow. Failure to gain weight,
weakness, and an enlarged liver and heart are other possible indicators of CHF but
are not as common as tachycardia and may take longer to develop.
Question format: Multiple Choice
Chapter 37: The Child with a Cardiovascular/Hematologic Disorder
Cognitive Level: Understand
Client Needs: Physiological Integrity: Reduction of Risk Potential
Integrated Process: Nursing Process
Reference: p. 772

2. In discussing the causes of iron-deficiency anemia in children with a group of


nurses, the following statements are made. Which of these statements is a
misconception related to iron-deficiency anemia?
A. "A family's economic problems are often a cause of malnutrition."
B. "Milk is a perfect food, and babies should be able to have all the milk they want."
C. "Caregivers sometimes don't understand the importance of iron and proper
nutrition."
D. "Children have a hard time getting enough iron from food during their first few
years."

Answer: B

Rationale: Babies with an inordinate fondness for milk can take in an astonishing
amount and, with their appetites satisfied, may show little interest in solid foods.
These babies are prime candidates for iron-deficiency anemia. Many children with
iron-deficiency anemia, however, are undernourished because of the family's
economic problems. A caregiver's knowledge deficit about nutrition is often present.
Because only 10 percent of dietary iron is absorbed, a diet containing 8 to 10 mg of
iron is needed for good health. During the first years of life, obtaining this quantity
of iron from food is often difficult for a child. If the diet is inadequate, anemia
quickly results.
Question format: Multiple Choice
Chapter 37: The Child with a Cardiovascular/Hematologic Disorder
Cognitive Level: Analyze
Client Needs: Physiological Integrity: Physiological Adaptation
Integrated Process: Teaching/Learning
Reference: p. 778

3. The nurse is caring for a child diagnosed with hemophilia. The child is crying
because of the pain. Which nursing action would be appropriate for the nurse to do
to relieve the child's pain?
A. Give aspirin as ordered.
B. Immobilize the affected extremity.
C. Distract the child with an age-appropriate activity.
D. Have the child perform range-of-motion exercises.

Answer: B

Rationale: The involved extremity should be promptly immobilized to prevent


contractures of soft tissues, to decrease bleeding, and to relieve pain. Aspirin and
drugs containing aspirin should not be given to children with hemophilia because it
may prolong bleeding. (Ibuprofen, however, has been proven safe for these
children.) While diversionary activities can help the child deal with pain, they will
not relieve it. Range-of-motion exercises should not be done during or right after an
acute episode because stretching of the joint capsule may cause additional
bleeding.
Question format: Multiple Choice
Chapter 37: The Child with a Cardiovascular/Hematologic Disorder
Cognitive Level: Apply
Client Needs: Physiological Integrity: Physiological Adaptation
Integrated Process: Nursing Process
Reference: p. 785

4. A nurse admits a child with a diagnosis of acute lymphoblastic leukemia (ALL).


During the each phase of treatment, which would most likely be seen periodically?
A. a relapse
B. a complete remission
C. a bone marrow transplant
D. methotrexate injection into cerebrospinal fluid

Answer: D

Rationale: Intensive chemotherapy is initially divided into three phases: induction


therapy—geared to achieving a complete remission with no leukemia cells;
consolidation Therapy—given to prevent the regrowth of leukemic cells; and
maintenance Therapy—maintaining the remission. Given periodically throughout
each of the phases of treatment is CNS Preventive Therapy—preventing invasion of
the central nervous system by leukemia cells. During CNS Preventive Therapy,
intrathecal administration (drugs injected into the cerebrospinal fluid by lumbar
puncture) of methotrexate is used to eradicate leukemia cells in the central nervous
system.
Question format: Multiple Choice
Chapter 37: The Child with a Cardiovascular/Hematologic Disorder
Cognitive Level: Apply
Client Needs: Physiological Integrity: Pharmacological and Parenteral Therapies
Integrated Process: Nursing Process
Reference: p. 788

5. In developing a plan of care for the child diagnosed with rheumatic fever, the
nursing intervention that takes highest priority for this child is to:
A. position the child to relieve joint pain.
B. monitor the C-reactive protein and erythrocyte sedimentation rate (ESR) levels.
C. provide age-appropriate diversional activities.
D. promote rest periods and bed rest.

Answer: D

Rationale: As long as the rheumatic process is active, progressive heart damage is


possible. To prevent heart damage, bed rest is essential to reduce the heart's
workload. Laboratory tests for ESR and C-reactive protein can be used to evaluate
disease activity and guide treatment, but they do not improve the child's health.
The child's comfort is important, so it is essential to relieve joint pain and prevent
injury with padded bed rails. But these measures are less important than rest when
it comes to preventing long-term complications such as residual heart disease.
Question format: Multiple Choice
Chapter 37: The Child with a Cardiovascular/Hematologic Disorder
Cognitive Level: Apply
Client Needs: Safe, Effective Care Environment: Management of Care
Integrated Process: Nursing Process
Reference: p. 775

6. The caregiver of a child with sickle cell disease asks the nurse how much fluid
her child should have each day after the child goes home. In response to the
caregiver's question, the nurse would explain that for the child with sickle cell
disease, it is best that the child have:
A. 300 to 800 ml of fluid per day.
B. 1,000 to 1,200 ml of fluid per day.
C. 1,500 to 2,000 ml of fluid per day.
D. 2,500 to 3,200 ml of fluid per day.

Answer: C

Rationale: Prevention of crises is the goal between episodes. Adequate hydration is


vital; fluid intake of 1,500 to 2,000 ml daily is desirable for a child weighing 20 kg
and should be increased to 3,000 ml during the crisis.
Question format: Multiple Choice
Chapter 37: The Child with a Cardiovascular/Hematologic Disorder
Cognitive Level: Apply
Client Needs: Physiological Integrity: Physiological Adaptation
Integrated Process: Nursing Process
Reference: p. 780

7. The nurse is preparing a child for discharge following a sickle cell crisis. Which
statement by the mother indicates a need for further teaching?
A. "She loves popsicles, so I'll let her have them as a snack or for dessert."
B. "I bought the medication to give to her when she says she is in pain."
C. "She has been down, but playing in soccer camp will cheer her up."
D. "I put her legs up on pillows when her knees start to hurt."

Answer: C

Rationale: Following a sickle cell crisis, the child should avoid extremely strenuous
activities that may cause oxygen depletion. Fluids are encouraged, pain
management will be needed, and the child's legs may be elevated to relieve
discomfort, so these are all statements that indicate an understanding of caring for
the child who has had a sickle cell crisis.
Question format: Multiple Choice
Chapter 37: The Child with a Cardiovascular/Hematologic Disorder
Cognitive Level: Analyze
Client Needs: Physiological Integrity: Reduction of Risk Potential
Integrated Process: Teaching/Learning
Reference: p. 780

8. The nurse is talking with a 9-year-old child diagnosed with acute leukemia who
will soon begin chemotherapy. The child expresses worry that when her hair falls
out friends won't like her or want to play with her anymore. Which response by the
nurse would be best?
A. Reassure the child that her hair will grow back in 3 to 6 months.
B. Tell the child that having chemotherapy is the only way she'll get better.
C. Talk with her and her family about wearing a wig, cap, or scarf.
D. Distract the child with a book or educational computer games.

Answer: C

Rationale: The child undergoing chemotherapy may want to wear a wig, especially
when returning to school. Encourage the family to choose the wig before
chemotherapy is started so that it matches the child's hair and the child has time to
get used to it. A cap or scarf often is appealing to a child, particularly if it carries a
special meaning for him or her. The hair will most likely grow back, chemotherapy
is necessary, and distraction can decrease the anxiety, but these are not the best
responses for this child.
Question format: Multiple Choice
Chapter 37: The Child with a Cardiovascular/Hematologic Disorder
Cognitive Level: Apply
Client Needs: Psychosocial Integrity
Integrated Process: Caring
Reference: p. 790

9. A toddler who is beginning to walk has fallen and hit his head on the corner of a
low table. The caregiver has been unable to stop the bleeding and brings the child
to the pediatric clinic. The nurse is gathering data during the admission process and
notes several bruises and swollen joints. A diagnosis of hemophilia is confirmed.
This child most likely has a deficiency of which blood factor?
A. Factor V
B. Factor VIII
C. Factor X
D. Factor XIII

Answer: B

Rationale: The most common types of hemophilia are factor VIII deficiency and
factor IX deficiency, which are inherited as sex-linked recessive traits, with
transmission to male offspring by carrier females.
Question format: Multiple Choice
Chapter 37: The Child with a Cardiovascular/Hematologic Disorder
Cognitive Level: Remember
Client Needs: Physiological Integrity: Physiological Adaptation
Integrated Process: Nursing Process
Reference: p. 783

10. A group of nurses is reviewing the cardiovascular system and its function.
Which statement by one of the nurses demonstrates an understanding of a child's
cardiovascular system?
A. "At birth, the infant's right and left ventricle are about the same size."
B. "Between the ages of 5 and 6, the child's left ventricle grows to about two times
the size of the right."
C. "The heart rate of the child decreases whenever the child experiences a fever."
D. "The child's heart doesn't mature and function like an adult's until between 8 and
10 years of age."

Answer: A

Rationale: At birth, both the right and left ventricles are about the same size, but
by a few months of age, the left ventricle is about two times the size of the right. If
the infant has a fever, respiratory distress, or any increased need for oxygen, the
pulse rate goes up to increase the cardiac output. Although the size of the heart is
smaller, by the time the child is 5 years old, the heart has matured, developed, and
functions just as the adult's heart.
Question format: Multiple Choice
Chapter 37: The Child with a Cardiovascular/Hematologic Disorder
Cognitive Level: Apply
Client Needs: Physiological Integrity: Physiological Adaptation
Integrated Process: Nursing Process
Reference: p. 771

11. The nurse is caring for a child with congestive heart failure and is administering
the drug digoxin. At the beginning of this drug therapy, the process of digitalization
is done for which reason?
A. To decrease the pain to a tolerable level
B. To increase the heart rate
C. To establish a maintenance dose of the drug
D. To build the blood levels to a therapeutic level

Answer: D

Rationale: The use of large doses of digoxin at the beginning of therapy,


administered to build up the blood levels of the drug to a therapeutic level, is
known as digitalization. A maintenance dose is given, usually daily, after
digitalization. Digoxin is used to improve the cardiac efficiency by slowing the heart
rate and strengthening the cardiac contractility. Digoxin is not indicated for relief of
pain.
Question format: Multiple Choice
Chapter 37: The Child with a Cardiovascular/Hematologic Disorder
Cognitive Level: Understand
Client Needs: Physiological Integrity: Pharmacological and Parenteral Therapies
Integrated Process: Nursing Process
Reference: p. 772

12. The nurse is teaching an in-service program to a group of nurses on the topic of
children diagnosed with rheumatic fever. The nurses in the group make the
following statements. Which statement is most accurate regarding the diagnosis of
rheumatic fever?
A. "This disorder is caused by genetic factors."
B. "Children who have this diagnosis may have had strep throat."
C. "Being up-to-date on immunizations is the best way to prevent this disorder."
D. "The onset and progression of this disorder is rapid."

Answer: B

Rationale: Rheumatic fever is precipitated by a streptococcal infection, such as


strep throat, tonsillitis, scarlet fever, or pharyngitis, which may be undiagnosed or
untreated. Rheumatic fever is a chronic disease of childhood, affecting the
connective tissue of the heart, joints, lungs, and brain. There is no immunization to
prevent rheumatic fever. The onset of rheumatic fever is often slow and subtle.
Question format: Multiple Choice
Chapter 37: The Child with a Cardiovascular/Hematologic Disorder
Cognitive Level: Understand
Client Needs: Physiological Integrity: Physiological Adaptation
Integrated Process: Teaching/Learning
Reference: p. 774
13. The nurse is caring for a child with rheumatic fever who has polyarthritis. Which
lab result would the nurse most anticipate with this child's diagnosis and
symptoms?
A. increased clotting time
B. decreased white blood cell count (WBC)
C. increased erythrocyte sedimentation rate (ESR)
D. decreased leukocyte count

Answer: C

Rationale: In a child with rheumatic fever who has polyarthritis, the erythrocyte
sedimentation rate (ESR) is increased. Although the clotting time may be altered
with the use of aspirin, this is not anticipated with the treatment. The white blood
cell and leukocyte counts would be elevated.
Question format: Multiple Choice
Chapter 37: The Child with a Cardiovascular/Hematologic Disorder
Cognitive Level: Apply
Client Needs: Physiological Integrity: Physiological Adaptation
Integrated Process: Nursing Process
Reference: p. 774

14. The nurse is reinforcing teaching with the caregivers of a child with rheumatic
fever. Which statement made by the caregivers best indicates an understanding of
the treatment regimen?
A. "She will be on a high sodium diet until the symptoms go away."
B. "Keeping her involved in her regular sports activities will be important."
C. "She'll be so happy about watching a movie while on bed rest."
D. "If she cuts herself, we know to keep that part of her body elevated above the
heart."

Answer: C

Rationale: The chief concern in caring for a child with rheumatic fever is the
prevention of residual heart disease. Bed rest is important and the length of bed
rest is determined by the degree of carditis present. Residual heart disease is
treated in accordance with its severity and its type with digitalis, restricted
activities, diuretics, and a low-sodium diet as indicated. Bleeding is not a concern
with rheumatic fever.
Question format: Multiple Choice
Chapter 37: The Child with a Cardiovascular/Hematologic Disorder
Cognitive Level: Analyze
Client Needs: Physiological Integrity: Physiological Adaptation
Integrated Process: Teaching/Learning
Reference: p. 775

15. In caring for the child with rheumatic fever, the nurse recognizes that which
nursing diagnosis would be most important to include in this child's plan of care?
A. Delayed growth and development
B. Risk for aspiration
C. Risk for acute pain
D. Disturbed body image

Answer: C

Rationale: Acute pain related to joint pain when extremities are touched or moved
is a concern for the child with rheumatic fever. Diversional activities are important,
but growth and development is not likely to be delayed. Chorea may be frustrating
to the child but body image is not altered. Respiratory issues are not noted with
rheumatic fever.
Question format: Multiple Choice
Chapter 37: The Child with a Cardiovascular/Hematologic Disorder
Cognitive Level: Apply
Client Needs: Physiological Integrity: Physiological Adaptation
Integrated Process: Nursing Process
Reference: p. 776

16. The nurse is collecting data from a caregiver, and the caregiver states that the
child has had a "strawberry-colored tongue." The nurse recognizes this as a
manifestation of which disorder?
A. congestive heart failure
B. rheumatic fever
C. Kawasaki disease
D. hemophilia

Answer: C

Rationale: A strawberry-colored tongue is a symptom seen in the child with


Kawasaki disease. This symptom is not seen in congestive heart failure, rheumatic
fever, nor hemophilia.
Question format: Multiple Choice
Chapter 37: The Child with a Cardiovascular/Hematologic Disorder
Cognitive Level: Understand
Client Needs: Physiological Integrity: Physiological Adaptation
Integrated Process: Nursing Process
Reference: p. 778

17. The nurse is discussing the stages of Kawasaki disease with a group of peers.
The nurse states that the child is at the greatest risk for having an aneurysm
during:
A. the acute stage.
B. the subacute stage.
C. the convalescent stage.
D. the chronic stage.

Answer: B
Rationale: Kawasaki disease occurs in three stages: Acute-high fever that does not
respond to antibiotics or antipyretics; child is irritable. Subacute-fever resolves,
irritability continues; greatest risk for aneurysms. Convalescent-symptoms are
gone; phase continues until lab values are normal and child's energy, appetite, and
temperament have returned. The chronic stage is not one of the identified stages of
the disorder.
Question format: Multiple Choice
Chapter 37: The Child with a Cardiovascular/Hematologic Disorder
Cognitive Level: Understand
Client Needs: Physiological Integrity: Physiological Adaptation
Integrated Process: Nursing Process
Reference: p. 778

18. The most serious concern for the child diagnosed with Kawasaki disease is the
effect on which of the following body organs?
A. brain
B. lungs
C. heart
D. kidneys

Answer: C

Rationale: In the child with Kawasaki disease, inflammation of the arteries, veins,
and capillaries occurs, and this inflammation can lead to serious cardiac concerns.
The other organs are not at as great a risk for complications in the child with
Kawasaki disease.
Question format: Multiple Choice
Chapter 37: The Child with a Cardiovascular/Hematologic Disorder
Cognitive Level: Remember
Client Needs: Physiological Integrity: Reduction of Risk Potential
Client Needs: Physiological Integrity: Physiological Adaptation
Integrated Process: Nursing Process
Reference: p. 778

19. For the child with Kawasaki disease who is treated with immunoglobulin, it is
most important to delay the administration of which of the following immunizations
for 3 to 6 months following the treatment?
A. DTaP (diphtheria, tetanus, acellular pertussis)
B. hepatitis B
C. inactivated poliovirus
D. MMR (measles, mumps, rubella)

Answer: D

Rationale: Following treatment of Kawasaki disease with immunoglobulins, live


vaccines such as MMR (measles, mumps, and rubella) should not be given for 3 to
6 months to a child who has been treated with immunoglobulin (IG). The IG
prevents the body from building antibodies, so the vaccine will likely be ineffective
in preventing the disease that it is being given to prevent. Although immunizations,
in general, are not given, in particular the MMR should not be given.
Question format: Multiple Choice
Chapter 37: The Child with a Cardiovascular/Hematologic Disorder
Cognitive Level: Apply
Client Needs: Physiological Integrity: Pharmacological and Parenteral Therapies
Integrated Process: Nursing Process
Reference: p. 778

20. The nurse is reinforcing teaching with a group of caregivers of children


diagnosed with iron-deficiency anemia. One of the caregivers tells the group, "I
give my child ferrous sulfate." Which statement made by the caregivers is correct
regarding giving ferrous sulfate?
A. "When I give my son ferrous sulfate I know he also needs potassium
supplements."
B. "I always give the ferrous sulfate with meals."
C. "My husband gives our daughter orange juice when she takes her ferrous sulfate,
so she gets Vitamin C."
D. "We watch closely for any diarrhea since that usually happens when he takes
ferrous sulfate."

Answer: C

Rationale: When ferrous sulfate is administered, it should be given between meals


with juice (preferably orange juice, because vitamin C aids in iron absorption). For
best results, iron should not be given with meals. Ferrous sulfate can cause
constipation or turn the child's stools black.
Question format: Multiple Choice
Chapter 37: The Child with a Cardiovascular/Hematologic Disorder
Cognitive Level: Analyze
Client Needs: Physiological Integrity: Pharmacological and Parenteral Therapies
Integrated Process: Teaching/Learning
Reference: p. 779

21. The nurse is teaching an in-service program to a group of nurses on the topic of
children diagnosed with sickle cell disease. The nurses in the group make the
following statements. Which statement is most accurate regarding this condition?
A. "The trait or the disease is seen in one generation and skips the next
generation."
B. "If the trait is inherited from both parents the child will have the disease."
C. "The disease is most often seen in individuals of Asian decent."
D. "Males are much more likely to have the disease than females."

Answer: B

Rationale: When the trait is inherited from both parents (homozygous state), the
child has sickle cell disease, and anemia develops. The trait does not skip
generations. The trait occurs most commonly in black clients. Either sex can have
the trait and disease.
Question format: Multiple Choice
Chapter 37: The Child with a Cardiovascular/Hematologic Disorder
Cognitive Level: Apply
Client Needs: Physiological Integrity: Physiological Adaptation
Integrated Process: Teaching/Learning
Reference: p. 779

22. In caring for a child with sickle cell disease, the highest priority goal is:
A. the caregiver's anxiety will be reduced.
B. the child's skin integrity will be maintained.
C. the family will verbalize understanding of the disease crisis.
D. The child's fluid intake will improve.

Answer: D

Rationale: The highest priority goals for this child are maintaining comfort and
relieving pain. The child is prone to dehydration because of the kidneys' inability to
concentrate urine, so increasing fluid intake is the next highest priority. Other goals
include improving physical mobility, maintaining skin integrity, reducing the
caregiver's anxiety, and increasing the caregiver's knowledge about the causes of
crisis episodes — but these goals are not the highest priority.
Question format: Multiple Choice
Chapter 37: The Child with a Cardiovascular/Hematologic Disorder
Cognitive Level: Apply
Client Needs: Safe, Effective Care Environment: Management of Care
Integrated Process: Nursing Process
Reference: p. 781

23. The nurse is collecting data from the caregivers of a child brought to the clinic
setting. The parents tell the nurse that the child's skin seems to be an unusual
color. The nurse notes that the child's skin appears bronze-colored and jaundiced.
This observation alerts the nurse to the likelihood that this child has which
disorder?
A. Hemophilia
B. Sickle cell disease
C. Kawasaki disease
D. Thalassemia

Answer: D

Rationale: In the child with thalassemia the skin may appear bronze-colored or
jaundiced. The child with hemophilia may have bruised areas on the skin. The skin
color in children with sickle cell disease may be pale in color, and with Kawasaki
disease the child may have a rash on the trunk and extremities.
Question format: Multiple Choice
Chapter 37: The Child with a Cardiovascular/Hematologic Disorder
Cognitive Level: Understand
Client Needs: Physiological Integrity: Physiological Adaptation
Integrated Process: Nursing Process
Reference: p. 783

24. The child with thalassemia may be given which classification of medication to
prevent one of the complications frequently seen with the treatment of this
disorder?
A. Vitamin supplements
B. Iron-chelating drugs
C. Factor VIII preparations
D. Potassium supplements

Answer: B

Rationale: Frequent transfusions can lead to complications and additional concerns


for the child, including the possibility of iron overload. For these children, iron-
chelating drugs such as deferoxamine mesylate may be given. Vitamin and
potassium supplements would not be given to treat the iron overload. Factor VIII
preparations are given to the child with hemophilia.
Question format: Multiple Choice
Chapter 37: The Child with a Cardiovascular/Hematologic Disorder
Cognitive Level: Understand
Client Needs: Physiological Integrity: Pharmacological and Parenteral Therapies
Integrated Process: Nursing Process
Reference: p. 783

25. A child diagnosed with hemophilia presents with warm, swollen, painful joints.
Which action will the nurse take first?
A. Document the presence of hemarthrosis in the client's chart
B. Notify the client's primary health care provider
C. Assess the client's urine and stool for blood
D. Prepare to administer factor replacement medication

Answer: D

Rationale: Many clients with hemophilia have repeated episodes of hemarthrosis or


bleeding into the joints, and develop functional impairment of the joints, despite
careful treatment. To assist in limiting impairment, the nurse would prepare to
administer factor replacement medications, such as plasma, recombinant clotting
factor VIII, or a clotting promotor medication. The nurse would document the
finding, notify the health care provider, and assess the client for additional
symptoms after limiting the amount of blood loss.
Question format: Multiple Choice
Chapter 37: The Child with a Cardiovascular/Hematologic Disorder
Cognitive Level: Understand
Client Needs: Physiological Integrity: Physiological Adaptation
Integrated Process: Nursing Process
Reference: p. 785

26. The nurse is caring for a child admitted with suspected leukemia. The nurse has
taken the child's history and performed an assessment. The nurse will plan to
prepare the child for which additional diagnostic test first?
A. urinalysis
B. complete blood cell count (CBC)
C. bone marrow aspiration
D. magnetic resonance imaging (MRI)

Answer: B

Rationale: After obtaining the child's history and symptoms, the nurse would
prepare the child for laboratory blood studies to assess the child's white blood cell
(WBC) count. A complete blood cell (CBC) count will provide data on the child's
WBC level. A bone marrow aspiration would be scheduled based on the results of
the CBC as it is required to confirm the diagnosis of leukemia. A urinalysis is done
for many disorders. In regard to childhood cancers, it is done to assist assessment
for neuroblastoma. MRIs can also be done for many disorders. Brain tumors are
common childhood cancers where MRIs are used.
Question format: Multiple Choice
Chapter 37: The Child with a Cardiovascular/Hematologic Disorder
Cognitive Level: Understand
Client Needs: Physiological Integrity: Reduction of Risk Potential
Integrated Process: Nursing Process
Reference: p. 788

27. Which site is most frequently used to perform a bone marrow aspiration?
A. Humerus
B. Iliac crest
C. Rib cage
D. Femur

Answer: B

Rationale: The preferred site for bone marrow aspiration in children is the iliac
crest. The other sites are not used for a bone marrow aspiration.
Question format: Multiple Choice
Chapter 37: The Child with a Cardiovascular/Hematologic Disorder
Cognitive Level: Remember
Client Needs: Physiological Integrity: Reduction of Risk Potential
Integrated Process: Nursing Process
Reference: p. 788

28. The nurse is caring for a child with leukemia. Which nursing intervention would
be the highest priority for this child?
A. encouraging the child to share feelings
B. grouping nursing care
C. following guidelines for reverse isolation
D. providing age-appropriate activities

Answer: C

Rationale: The child with leukemia is susceptible to infection, especially during


chemotherapy. Infections such as meningitis, septicemia, and pneumonia are the
most common causes of death. To protect the child from infectious organisms,
follow standard guidelines for reverse isolation. Grouping nursing care to provide
rest is important, but not the highest priority. Encouraging the child to share
feelings and providing age-appropriate activities are important, but psychological
issues are a lower priority than physical.
Question format: Multiple Choice
Chapter 37: The Child with a Cardiovascular/Hematologic Disorder
Cognitive Level: Apply
Client Needs: Safe, Effective Care Environment: Safety and Infection Control
Integrated Process: Nursing Process
Reference: p. 789

29. Which nursing interventions would be appropriate for the plan of care designed
for a child diagnosed with hemophilia who is experiencing an episode of active
bleeding? Select all that apply.
A. Performing passive range-of-motion exercises
B. Administer ibuprofen as prescribed for pain
C. Applying splints to extremities
D. Schedule physical therapy
E. Schedule regular active range-of-motion exercises

Answer: B, C, E

Rationale: Passive range-of-motion exercises help prevent the development of joint


contractures. Do not use them, however, after an acute episode because stretching
of the joint capsule may cause bleeding. Encourage the child to do active range-of-
motion exercises because he or she can recognize his or her own pain tolerance.
Many clients who have had repeated episodes of hemarthrosis (bleeding into the
joints) develop functional impairment of the joints, despite careful treatment. Use
splints and devices to position the limb in a functional position. Physical therapy is
helpful after the bleeding episode is under control. Ibuprofen has been proven safe
for the management of related joint pain experienced by these children.
Question format: Multiple Select
Chapter 37: The Child with a Cardiovascular/Hematologic Disorder
Cognitive Level: Apply
Client Needs: Physiological Integrity: Physiological Adaptation
Integrated Process: Nursing Process
Reference: p. 778

30. A group of nursing students is discussing the diagnosis of iron deficiency


anemia. The students demonstrate an understanding of the need for dietary iron
when suggesting the inclusion of what foods into the diet of a 4-year-old diagnosed
with this form of anemia? Select all that apply.
A. Egg yolks
B. Egg whites
C. Raisins
D. Peanut butter
E. Cheese
F. Oatmeal

Answer: A, C, D, F

Rationale: Egg yolks, raisins, peanut butter and oatmeal are food sources high in
iron. Cheese is not as high in iron. Avoid egg whites for young children because of
allergies.
Question format: Multiple Select
Chapter 37: The Child with a Cardiovascular/Hematologic Disorder
Cognitive Level: Analyze
Client Needs: Physiological Integrity: Physiological Adaptation
Integrated Process: Teaching/Learning
Reference: p. 779

31. The nurse is collecting data on a 5-year-old child admitted with the diagnosis of
congestive heart failure. Which clinical manifestation observed during the physical
assessment would be consistent with this diagnosis?
A. Jerking movements of the arms and legs
B. Scissoring of the legs with toes pointed down
C. Failure to gain weight
D. Spooning of the finger nails

Answer: C

Rationale: In infants and older children, one of the first signs of CHF is tachycardia.
Other signs of CHF often seen in the older child include failure to gain weight;
weakness; fatigue; restlessness; irritability; and a pale, mottled, or cyanotic color.
Rapid respirations or tachypnea, dyspnea, and coughing with bloody sputum also
are seen. Edema and enlargement of the liver and heart may be present. Jerking
movements indicate seizure activity. Scissoring of the legs is seen in cerebral palsy,
and spooning of the fingernails is seen in iron-deficiency anemia.
Question format: Multiple Choice
Chapter 37: The Child with a Cardiovascular/Hematologic Disorder
Cognitive Level: Understand
Client Needs: Physiological Integrity: Physiological Adaptation
Integrated Process: Nursing Process
Reference: p. 772

32. The child is prescribed liquid ferrous sulfate. The nurse should encourage the
child to take which action immediately after each dose to best eliminate possible
side effects?
A. Drink a glass of milk
B. Brush his or her teeth
C. Remain in an upright position for at least 15 minutes
D. Not eat or drink for one hour

Answer: B

Rationale: To prevent staining of the teeth, the child should brush the teeth after
administration of iron preparations such as ferrous sulfate. There is no need to
remain upright, drink milk or to refrain from eating or drinking for one hour.
Question format: Multiple Choice
Chapter 37: The Child with a Cardiovascular/Hematologic Disorder
Cognitive Level: Apply
Client Needs: Physiological Integrity: Pharmacological and Parenteral Therapies
Integrated Process: Nursing Process
Reference: p. 779
Chapter 38
1. The nurse has admitted a child with a diagnosis of severe gastroenteritis. To help
prevent the risk of transmitting infection to other clients, the nurse should:
A. wear a mask when handling articles contaminated with feces.
B. follow standard precautions.
C. discourage anyone from visiting.
D. sterilize thermometers between clients.

Answer: B

Rationale: To prevent the spread of possibly infectious organisms to other pediatric


patients, follow standard precautions issued by the Centers for Disease Control and
Prevention. Gloves should be worn when handling items contaminated with feces,
but masks are not necessary. Visitors should be limited to family only. Take the
temperature with a thermometer that is used only for that child.
Question format: Multiple Choice
Chapter 38: The Child with a Gastrointestinal/Endocrine Disorder
Cognitive Level: Understand
Client Needs: Safe, Effective Care Environment: Safety and Infection Control
Integrated Process: Nursing Process
Reference: p. 804

2. The nurse is caring for an infant immediately after pyloromyotomy surgery has
been performed to treat pyloric stenosis. The infant's parents are understandably
anxious about their child. Given the situation, what is the most appropriate way for
the nurse to position the infant during the anesthesia recovery period?
A. Allow the parents to hold their infant
B. Place the infant on the back.
C. Lay the infant on their stomach.
D. Support the infant and place them on their side.

Answer: D

Rationale: Postoperatively, the child should be placed on his side to prevent


aspiration of mucus or vomitus, especially during the anesthesia recovery period.
After fully waking from the surgery, he can be held by a family caregiver in a
position that does not interfere with IV infusions and is comforting to both caregiver
and child.
Question format: Multiple Choice
Chapter 38: The Child with a Gastrointestinal/Endocrine Disorder
Cognitive Level: Apply
Client Needs: Physiological Integrity: Physiological Adaptation
Integrated Process: Nursing Process
Reference: p. 809
3. A nurse notices a child chewing several pieces of something white. The child is
also drooling and crying. A container that looks like an empty pill bottle is on the
floor. The first action by the nurse would be to:
A. call 911 for emergency help.
B. ask the poison control center about an antidote.
C. give the child syrup of ipecac to induce vomiting.
D. remove the substance from the child's mouth.

Answer: D

Rationale: Treatment steps in order of importance for poisoning: Remove the


obvious remnants of the poison. Call 911 for emergency help if the child has
collapsed or stopped breathing. If the child is conscious and alert, call the poison
control center and follow their instructions. Administer the appropriate antidote if
recommended. Administer general supportive and symptomatic care. The American
Academy of Pediatrics (AAP) no longer recommends administering syrup of ipecac
because it hasn't been proven that inducing vomiting prevents poisoning. Because
of the potential for misuse, the AAP also recommends safely disposing of any syrup
of ipecac already in the home. In an emergency care setting, gastric lavage may be
used to empty the stomach of toxic substances.
Question format: Multiple Choice
Chapter 38: The Child with a Gastrointestinal/Endocrine Disorder
Cognitive Level: Apply
Client Needs: Safe, Effective Care Environment: Management of Care
Integrated Process: Nursing Process
Reference: p. 820

4. A young child has been admitted with a diagnosis of enterobiasis (pinworm


infection). This child will most likely have a history of:
A. bedwetting.
B. restlessness.
C. perianal itching.
D. malnutrition.

Answer: C

Rationale: Intense perianal itching is the primary symptom of pinworm infection,


enterobiasis. Young children who cannot clearly verbalize their feelings may be
restless, sleep poorly, or have episodes of bedwetting. Pinworm infestation is as
common as an infection or cold, making a history of malnutrition less likely. Chronic
hookworm infestation can cause malnutrition, however.
Question format: Multiple Choice
Chapter 38: The Child with a Gastrointestinal/Endocrine Disorder
Cognitive Level: Remember
Client Needs: Physiological Integrity: Physiological Adaptation
Integrated Process: Nursing Process
Reference: p. 817
5. The nurse is caring for a 7-year-old diagnosed with pinworms. The nurse talks
with the child's caregiver about proper treatment and prevention of future
infections. Which statement made by the caregiver indicates a need for further
teaching?
A. "I always have to remind him to wash his hands before eating."
B. "We just bought a washer and dryer, and the hot water works well."
C. "Thank goodness my other children and I are not sick too."
D. "He hates having his nails trimmed but I will insist they are kept short."

Answer: C

Rationale: The life cycle of pinworms is 6 to 8 weeks. Clothing, bedding, food, toilet
seats, and other articles become infected, and the infestation spreads to other
members of the family. Pinworm eggs also can float in the air and be inhaled.
Family members may be infected and not realize it. Because pinworms are so easily
transmitted, the nurse should encourage all family members to be treated as well.
Washing hands before eating and after using the toilet, frequent laundering of
bedding and underclothes in hot water, and short, clean fingernails are all ways to
prevent subsequent infections.
Question format: Multiple Choice
Chapter 38: The Child with a Gastrointestinal/Endocrine Disorder
Cognitive Level: Analyze
Client Needs: Physiological Integrity: Physiological Adaptation
Integrated Process: Teaching/Learning
Reference: p. 817

6. The caregivers of a child just diagnosed with diabetes express concern that they
won't remember the different signs and symptoms of hyperglycemia and
hypoglycemia. As a result, they are afraid they won't handle an emergency
correctly. What is the best initial response by the nurse to help ensure the child's
safety?
A. Instruct them to treat the reaction as if it's hypoglycemia, which is more likely.
B. Repeat the signs and symptoms over and over until they seem to understand.
C. Suggest that the child wear an insulin pump for continuous insulin
administration.
D. Give the caregivers educational pamphlets and videos about diabetes.

Answer: A

Rationale: Hypoglycemia is much more likely to occur than hyperglycemia; if there


is any doubt as to whether the child is having a hypoglycemic or hyperglycemic
reaction, it should be treated as hypoglycemia. While the pump may offer
continuous insulin, it does not sense blood glucose level; insulin reactions can still
occur. Careful monitoring of blood glucose is still needed. While repeating signs and
symptoms may be helpful, caregivers of a recently diagnosed child have lots of
information to absorb and the repetition may create more anxiety. Assuming that
the caregivers can read and understand them, written materials and videos may be
helpful but they should not take the place of an initial teaching session with a
nurse.
Question format: Multiple Choice
Chapter 38: The Child with a Gastrointestinal/Endocrine Disorder
Cognitive Level: Apply
Client Needs: Physiological Integrity: Reduction of Risk Potential
Integrated Process: Caring
Reference: p. 830

7. The nurse is working with the mother of a newborn. The mother asks why a baby
needs small feedings at frequent intervals. The nurse explains to the mother that
this is necessary because in the infant:
A. the enzymes secreted by the liver and pancreas are reduced.
B. food moves more slowly through the GI tract.
C. the pylorus has not been fully formed.
D. peristaltic action is absent in the lower portion of the bowel.

Answer: A

Rationale: In the newborn, the enzymes secreted by the liver and pancreas are
reduced. Thus, the infant cannot break down and use complex carbohydrates. As a
result, the newborn diet must be adjusted to allow for this immaturity. By the age
of 4 to 6 months, the needed enzymes are usually sufficient in amount. The smaller
capacity of the infant's stomach and the increased speed at which food moves
through the GI tract requires feeding smaller amounts at more frequent intervals.
In addition, the small capacity of the colon leads to a bowel movement after each
feeding. The pyloric sphincter is formed, but is lax and does not have bearing on
the frequency of feeding.
Question format: Multiple Choice
Chapter 38: The Child with a Gastrointestinal/Endocrine Disorder
Cognitive Level: Apply
Client Needs: Health Promotion and Maintenance
Integrated Process: Teaching/Learning
Reference: p. 796

8. The nurse admits an infant who is nutritionally deprived. The infant is weak and
seems somewhat uninterested in food. In developing the infant's plan of care, how
often will the nurse most likely plan to feed this infant?
A. every hour
B. every 2 or 3 hours
C. every 4 hours
D. on demand

Answer: B

Rationale: For the child who is nutritionally deprived, scheduling feedings every 2 or
3 hours is best because most weak babies can handle frequent, small feedings
better than feedings every 4 hours. Feeding every hour would not give the weak
child an adequate amount of time to rest and sleep between feedings.
Question format: Multiple Choice
Chapter 38: The Child with a Gastrointestinal/Endocrine Disorder
Cognitive Level: Understand
Client Needs: Physiological Integrity: Basic Care and Comfort
Integrated Process: Nursing Process
Reference: p. 799

9. The teeth, tongue, gallbladder, appendix, salivary glands, liver, and pancreas are
referred to as accessory organs, and the purpose of these is:
A. to secrete liquids that help the food to be tasted as a person eats.
B. to aid in the digestive process and to produce substances that aid in the
digestive process.
C. to cushion and protect the digestive organs.
D. to decrease the secretion of acids in the digestive organs.

Answer: B

Rationale: Other organs, called accessory organs, include structures that aid in the
digestive process, as well as glands that secrete substances that further aid in
digestion. These accessory organs include the teeth, tongue, gallbladder, appendix,
salivary glands, liver, and pancreas. These organs do not affect the taste of food.
Cerebrospinal fluid cushions and protects the nerve cells. These organs do not
decrease the secretion of acids.
Question format: Multiple Choice
Chapter 38: The Child with a Gastrointestinal/Endocrine Disorder
Cognitive Level: Remember
Client Needs: Physiological Integrity: Basic Care and Comfort
Integrated Process: Nursing Process
Reference: p. 796

10. Which of the following is most correct regarding the gastrointestinal system of
the child?
A. The child's gastrointestinal system is fully matured when the child is born.
B. The enzymes secreted by the child's liver and pancreas are much greater in
amount than in the adult.
C. The child cannot break down and use complex carbohydrates in the same way
the adult can.
D. The speed with which food passes through the gastrointestinal tract in the child
is much slower than in the adult.

Answer: C

Rationale: In the GI tract of the newborn, the enzymes secreted by the liver and
pancreas are reduced. Thus, the infant cannot break down and use complex
carbohydrates. In the infant, food moves through the GI tract with increased speed.
The GI tract matures and the capacity of the GI tract increases as the child gets
older.
Question format: Multiple Choice
Chapter 38: The Child with a Gastrointestinal/Endocrine Disorder
Cognitive Level: Understand
Client Needs: Physiological Integrity: Basic Care and Comfort
Integrated Process: Nursing Process
Reference: p. 796

11. The school nurse is working with a group of teachers who instruct children who
are nutritionally deprived. As the teachers are talking with the nurse they make the
following statements. Which statement most indicates a problem related to
decreased nutrition?
A. "One of my students is taller than several of the other children in the class."
B. "I am really glad that during this quarter the absence rate in my classroom has
dropped."
C. "Several of the children in my class have such a hard time concentrating."
D. "The grades of the children in my class are higher than in the classroom next to
me."

Answer: C

Rationale: Malnourished children grow at a slower rate, have a higher rate of illness
and infection, and have more difficulty concentrating and achieving in school.
Question format: Multiple Choice
Chapter 38: The Child with a Gastrointestinal/Endocrine Disorder
Cognitive Level: Apply
Client Needs: Health Promotion and Maintenance
Integrated Process: Nursing Process
Reference: p. 797

12. The nurse is teaching an in-service program to a group of nurses on the topic of
children diagnosed with Kwashiorkor. The nurses in the group make the following
statements. Which statement is most accurate related to the diagnosis of
Kwashiorkor?
A. "These children have a severe deficiency of vitamin D."
B. "It is important to increase the intake of protein for these children."
C. "The highest incidence of this disease is seen in children who are adolescents."
D. "The cause of this disease can be treated very simply."

Answer: B

Rationale: Kwashiorkor results from severe deficiency of protein with an adequate


caloric intake. It accounts for most of the malnutrition in the world's children today.
The highest incidence is in children 4 months to 5 years of age. Although strenuous
efforts are being made around the world to prevent this condition, its causes are
complex.
Question format: Multiple Choice
Chapter 38: The Child with a Gastrointestinal/Endocrine Disorder
Cognitive Level: Analyze
Client Needs: Physiological Integrity: Physiological Adaptation
Integrated Process: Teaching/Learning
Reference: p. 797

13. In understanding the disease of marasmus when seen in children, the nurse
recognizes that the disease is caused because of which of the following?
A. deficiency of vitamin C and iron
B. excess of vitamin C and iron
C. deficiency of protein and calories
D. excess of protein and calories

Answer: C

Rationale: Marasmus is a deficiency in calories as well as protein. Scurvy is caused


by inadequate intake of vitamin C, and anemia is caused by lack of iron. Excess
calories add to the concern of obesity in children. Excess vitamin C is excreted, and
it is unusual to have an excess of iron or protein in the diet of children; those
nutrients are more often inadequate in children's diets.
Question format: Multiple Choice
Chapter 38: The Child with a Gastrointestinal/Endocrine Disorder
Cognitive Level: Understand
Client Needs: Physiological Integrity: Physiological Adaptation
Integrated Process: Nursing Process
Reference: p. 797

14. The nurse recognizes that in the disorder referred to as rickets, the child has a
lack of vitamin D. Because of the lack of vitamin D, the absorption of which of the
following is decreased?
A. calcium and phosphorus
B. vitamin C and thiamine
C. riboflavin and niacin
D. iron and potassium

Answer: A

Rationale: Rickets, a disease affecting the growth and calcification of bones, is


caused by a lack of vitamin D. The absorption of calcium and phosphorus is
diminished because of the lack of vitamin D, which is needed to regulate the use of
these minerals. The absorption of the other nutrients is not affected by the lack of
vitamin D.
Question format: Multiple Choice
Chapter 38: The Child with a Gastrointestinal/Endocrine Disorder
Cognitive Level: Understand
Client Needs: Physiological Integrity: Physiological Adaptation
Integrated Process: Nursing Process
Reference: p. 797
15. The incidence of vitamin D deficiency in the United States is less than in many
countries. What is the most likely reason for this?
A. Many children in the U.S. take daily vitamin supplements.
B. The water in many towns and cities in the U.S. has vitamin D added.
C. Some foods in the U.S. have been fortified with vitamin D.
D. The amount of ultraviolet sunlight each day in the U.S. is adequate to provide
needed vitamin D.

Answer: C

Rationale: Whole milk and evaporated milk fortified with 400 U of vitamin D per
quart are available throughout the United States, which decreases the vitamin D
deficiency of children in the U.S. Vitamin D can be administered orally in the form
of fish liver oil or synthetic vitamin, but this is not common for children in the U.S.
Water is not fortified with vitamin D, and some communities in the U.S. do not get
adequate sunshine to meet vitamin D needs.
Question format: Multiple Choice
Chapter 38: The Child with a Gastrointestinal/Endocrine Disorder
Cognitive Level: Understand
Client Needs: Health Promotion and Maintenance
Integrated Process: Nursing Process
Reference: p. 797

16. A group of nursing students is discussing the diagnosis of vitamin C deficiency.


The students have an appropriate understanding of the need for appropriate dietary
intake when they identify which vitamin C rich foods? Select all that apply.
A. strawberries
B. potatoes
C. peas
D. fish sticks
E. cottage cheese
F. bagels

Answer: A, B, C

Rationale: A variety of fresh vegetables and fruits supplies vitamin C for the older
infant and child. Strawberries, potatoes, and peas are high in vitamin C content.
Meat, dairy, and grain foods have little vitamin C content.
Question format: Multiple Select
Chapter 38: The Child with a Gastrointestinal/Endocrine Disorder
Cognitive Level: Apply
Client Needs: Health Promotion and Maintenance
Integrated Process: Nursing Process
Reference: p. 797

17. The nurse is collecting data on a child who has been nutritionally deprived. The
nurse notes that the child is irritable and listless. The foster caregiver reports that
the child says she is not hungry and has been vomiting. It is discovered that the
child has beriberi. This disease is due to a deficiency in which of the following?
A. thiamine
B. vitamin C
C. niacin
D. iron

Answer: A

Rationale: Children whose diets are deficient in thiamine exhibit irritability,


listlessness, loss of appetite, and vomiting. A severe lack of thiamine in the diet
causes beriberi, a disease characterized by cardiac and neurologic symptoms.
Beriberi does not occur when balanced diets that include whole grains are eaten.
Lack of vitamin C causes scurvy, lack of niacin causes pellagra, and lack of iron
causes anemia.
Question format: Multiple Choice
Chapter 38: The Child with a Gastrointestinal/Endocrine Disorder
Cognitive Level: Remember
Client Needs: Physiological Integrity: Physiological Adaptation
Integrated Process: Nursing Process
Reference: p. 798

18. The nurse is discussing the disease known as pellagra. This disease is due to a
deficiency in which of the following?
A. thiamine
B. vitamin C
C. niacin
D. iron

Answer: C

Rationale: Niacin insufficiency in the diet causes a disease known as pellagra, which
presents with GI and neurologic symptoms. A diet deficient in thiamine causes
beriberi. Lack of vitamin C causes scurvy, and lack of iron causes anemia.
Question format: Multiple Choice
Chapter 38: The Child with a Gastrointestinal/Endocrine Disorder
Cognitive Level: Remember
Client Needs: Physiological Integrity: Physiological Adaptation
Integrated Process: Nursing Process
Reference: p. 798

19. The child is diagnosed with hypocalcemia. The nurse should assess the child for
which possible complication?
A. cardiac arrhythmias
B. neurologic deficiencies
C. kidney failure
D. urinary tract disorders
Answer: B

Rationale: Hypocalcemia (insufficient calcium) causes neurologic damage, including


intellectual disability. Calcium is necessary for bone and tooth formation and is also
needed for proper nerve and muscle function. Hypokalemia can cause cardiac
issues. Kidney and urinary disorders are not likely to be caused by insufficiencies in
the diet.
Question format: Multiple Choice
Chapter 38: The Child with a Gastrointestinal/Endocrine Disorder
Cognitive Level: Apply
Client Needs: Physiological Integrity: Physiological Adaptation
Integrated Process: Nursing Process
Reference: p. 798

20. The nurse is collecting data from the caregivers of a child who is suspected of
having a food allergy. Which clinical manifestation would likely have been noted in
this child?
A. restlessness and irritability
B. blinking and twitching of the mouth
C. nasal discharge and sneezing
D. urticaria and pruritus

Answer: D

Rationale: Common symptoms are urticaria (hives), pruritus (itching), stomach


pains, and respiratory symptoms. Some of the symptoms may appear quickly after
the child has eaten the offending food, but other foods may cause a delayed
reaction. Restlessness and irritability may be seen in children with seizure
disorders, blinking, and twitching of the mouth are seen with absence seizures, and
nasal discharge and sneezing are seen with allergic rhinitis.
Question format: Multiple Choice
Chapter 38: The Child with a Gastrointestinal/Endocrine Disorder
Cognitive Level: Understand
Client Needs: Physiological Integrity: Physiological Adaptation
Integrated Process: Nursing Process
Reference: p. 798

21. The nurse is educating the caregivers of a child suspected of having developed
a food allergy. Which foods will the nurse identify for elimination from the child's
diet? Select all that apply.
A. eggs
B. broccoli
C. corn
D. oranges
E. grapes
F. soybeans

Answer: A, C, D, F
Rationale: Among the foods most likely to cause allergic reactions are milk, eggs,
wheat, corn, legumes (including peanuts and soybeans), oranges, strawberries, and
chocolate. Vegetables other than corn are less likely to cause allergies. Citric fruits
are more often the cause of allergies.
Question format: Multiple Select
Chapter 38: The Child with a Gastrointestinal/Endocrine Disorder
Cognitive Level: Apply
Client Needs: Physiological Integrity: Basic Care and Comfort
Integrated Process: Nursing Process
Reference:

22. The nurse is caring for a pediatric client with idiopathic celiac disease. Which
meal will the nurse select for this client?
A. Ham and cheese sandwich, orange slices, chips, and whole milk
B. Whole wheat pasta, meatballs, carrot sticks, apple, and water
C. Baked salmon, potato slices, vanilla ice cream, and apple juice
D. Meatloaf, green beans, peanut butter cookie, and fat-free milk

Answer: C

Rationale: Celiac disease is an autoimmune condition where contact with gluten


causes a reaction from the body's immune system. Clients with celiac disease
should be educated to eat a gluten-free diet to decrease symptoms and limit small
intestine irritation. Gluten is a protein found in wheat, barley, and rye. Most
commercially used flour contains wheat and should be avoided. The nurse would
select foods such as meats/fish (salmon), fruits, vegetables (potatoes), and rice.
Single flavor ice creams, such as chocolate, vanilla, and strawberry are gluten free,
while cookies contain flour and should be avoided. Fruit juices, water, and milk are
all gluten free. Meatloaf may contain oats; however, studies suggest oats are fine
to consume, as long as the oats did not come into contact with wheat during
processing. Reading the label would indicate if wheat was contacted. Sandwich
bread and pastas contain gluten unless special gluten-free products are purchased
or it is homemade. The nurse would not assume those items were gluten-free.
Question format: Multiple Choice
Chapter 38: The Child with a Gastrointestinal/Endocrine Disorder
Cognitive Level: Apply
Client Needs: Physiological Integrity: Basic Care and Comfort
Integrated Process: Nursing Process
Reference: p. 801

23. The nurse is reinforcing dietary teaching with the caregiver of a child diagnosed
with celiac disease. The caregivers make the following statements. Which statement
indicates a need for further teaching regarding the dietary restrictions for the child
with celiac disease?
A. "The soup we eat at our house is all made from scratch."
B. "She loves hot dogs, and we always cut hers up into small pieces."
C. "I have learned to make my own bread with no gluten."
D. "Even though milk and pudding are good for her, we don't give her those foods."

Answer: B

Rationale: Commercially canned creamed soups, cold cuts, frankfurters, and


pudding mixes generally contain wheat products and should not be included in the
diet of the child with celiac syndrome. This caregiver needs further teaching
regarding giving her child hot dogs, even if they are cut into small pieces. The other
choices show an understanding of the dietary restrictions.
Question format: Multiple Choice
Chapter 38: The Child with a Gastrointestinal/Endocrine Disorder
Cognitive Level: Analyze
Client Needs: Physiological Integrity: Reduction of Risk Potential
Integrated Process: Teaching/Learning
Reference: p. 801

24. The nurse is teaching an in-service program to a group of nurses on the topic of
gastrointestinal disorders. The nurses in the group make the following statements.
Which statement is most accurate related to the diagnosis of congenital aganglionic
megacolon?
A. A partial or complete intestinal obstruction occurs.
B. A thickened, elongated muscle causes an obstruction at the end of the stomach.
C. There are recurrent paroxysmal bouts of abdominal pain.
D. In this disorder the sphincter that leads into the stomach is relaxed.

Answer: A

Rationale: Congenital aganglionic megacolon, also called Hirschsprung disease, is


characterized by persistent constipation resulting from partial or complete intestinal
obstruction of mechanical origin. Pyloric stenosis is characterized by hypertrophy of
the circular muscle fibers of the pylorus, which leads to an obstruction at the distal
end of the stomach. Colic consists of recurrent paroxysmal bouts of abdominal pain.
Gastroesophageal reflux (GER) occurs when the sphincter in the lower portion of
the esophagus is relaxed and allows gastric contents to be regurgitated back into
the esophagus.
Question format: Multiple Choice
Chapter 38: The Child with a Gastrointestinal/Endocrine Disorder
Cognitive Level: Apply
Client Needs: Physiological Integrity: Physiological Adaptation
Integrated Process: Teaching/Learning
Reference: p. 810

25. The nurse is teaching an in-service program to a group of nurses on the topic of
gastrointestinal disorders. The nurses in the group make the following statements.
Which statement is most accurately related to the diagnosis of pyloric stenosis?
A. A partial or complete intestinal obstruction occurs.
B. A thickened, elongated muscle causes an obstruction at the end of the stomach.
C. There are recurrent paroxysmal bouts of abdominal pain.
D. In this disorder the sphincter that leads into the stomach is relaxed.

Answer: B

Rationale: Pyloric stenosis is characterized by hypertrophy of the circular muscle


fibers of the pylorus, with a severe narrowing of its lumen. The pylorus is thickened
to as much as twice its size, is elongated, and has a consistency resembling
cartilage. As a result of this obstruction at the distal end of the stomach, the
stomach becomes dilated. Congenital aganglionic megacolon is characterized by
persistent constipation resulting from partial or complete intestinal obstruction of
mechanical origin. Colic consists of recurrent paroxysmal bouts of abdominal pain.
Gastroesophageal reflux (GER) occurs when the sphincter in the lower portion of
the esophagus is relaxed and allows gastric contents to be regurgitated back into
the esophagus.
Question format: Multiple Choice
Chapter 38: The Child with a Gastrointestinal/Endocrine Disorder
Cognitive Level: Apply
Client Needs: Physiological Integrity: Physiological Adaptation
Integrated Process: Teaching/Learning
Reference: p. 807

26. The nurse is teaching an in-service program to a group of nurses on the topic of
gastrointestinal disorders. The nurses in the group make the following statements.
Which statement is most accurately related to the diagnosis of colic?
A. A partial or complete intestinal obstruction occurs.
B. A thickened, elongated muscle causes an obstruction at the end of the stomach.
C. There are recurrent paroxysmal bouts of abdominal pain.
D. In this disorder the sphincter that leads into the stomach is relaxed.

Answer: C

Rationale: Colic is described as episodes of crying in the infant, lasting up to several


hours a day and recurring several times a week for several weeks. These episodes
are often associated with recurrent gastrointestinal disturbances and are fairly
common in young infants. Congenital aganglionic megacolon is characterized by
persistent constipation resulting from partial or complete intestinal obstruction of
mechanical origin. Pyloric stenosis is characterized by hypertrophy of the circular
muscle fibers of the pylorus, which leads to an obstruction at the distal end of the
stomach. Gastroesophageal reflux (GER) occurs when the sphincter in the lower
portion of the esophagus is relaxed and allows gastric contents to be regurgitated
back into the esophagus.
Question format: Multiple Choice
Chapter 38: The Child with a Gastrointestinal/Endocrine Disorder
Cognitive Level: Apply
Client Needs: Physiological Integrity: Physiological Adaptation
Integrated Process: Nursing Process
Reference: p. 802
27. The nurse is caring for a child admitted with pyloric stenosis. Which clinical
manifestation would likely have been noted in the child with this diagnosis?
A. Explosive diarrhea
B. Projectile vomiting
C. Severe abdominal pain
D. Frequent urination

Answer: B

Rationale: During the first weeks of life, the infant with pyloric stenosis often eats
well and gains weight and then starts vomiting occasionally after meals. Within a
few days the vomiting increases in frequency and force, becoming projectile. The
child may have constipation, and peristaltic waves may be seen in the abdomen,
but the child does not appear in severe pain. Urine output is decreased and
urination is infrequent.
Question format: Multiple Choice
Chapter 38: The Child with a Gastrointestinal/Endocrine Disorder
Cognitive Level: Understand
Client Needs: Physiological Integrity: Physiological Adaptation
Integrated Process: Nursing Process
Reference: p. 807

28. The nurse is caring for a child with a diagnosis of pyloric stenosis during the
preoperative phase of the child's treatment. What is the highest priority at this
time?
A. Preparing family for home care
B. Promoting comfort
C. Maintaining skin integrity
D. Improving hydration

Answer: D

Rationale: Preoperatively, the highest priority for the child with pyloric stenosis is to
improve nutrition and hydration. Maintaining mouth and skin integrity, and relieving
family anxiety are important, but these are not the priority. The child will not likely
have intense pain. Preparing the family for home care would be a postoperative
goal.
Question format: Multiple Choice
Chapter 38: The Child with a Gastrointestinal/Endocrine Disorder
Cognitive Level: Apply
Client Needs: Safe, Effective Care Environment: Management of Care
Integrated Process: Nursing Process
Reference: p. 808

29. The nurse is caring for a child admitted with congenital aganglionic megacolon.
Which clinical manifestation would likely have been noted in the child with this
diagnosis?
A. Prolonged bleeding
B. Chronic cough
C. Persistent constipation
D. Irregular breathing

Answer: C

Rationale: Congenital aganglionic megacolon, also called Hirschsprung disease, is


characterized by persistent constipation resulting from partial or complete intestinal
obstruction of mechanical origin. Prolonged bleeding is a manifestation of
hemophilia. A chronic cough is noted in the child with cystic fibrosis. Irregular
breathing occurs in children with seizures.
Question format: Multiple Choice
Chapter 38: The Child with a Gastrointestinal/Endocrine Disorder
Cognitive Level: Apply
Client Needs: Physiological Integrity: Physiological Adaptation
Integrated Process: Nursing Process
Reference: p. 810

30. The nurse is discussing the treatment of congenital aganglionic megacolon with
the caregivers of a child diagnosed with this disorder. Which statement is the best
explanation of the treatment for this diagnosis?
A. "The treatment for the disorder will be a surgical procedure."
B. "Your child will be treated with oral iron preparations to correct the anemia."
C. "We will give enemas until clear and then teach you how to do these at home."
D. "Your child will receive counseling so the underlying concerns will be addressed."

Answer: A

Rationale: Treatment of congenital aganglionic megacolon involves surgery with the


ultimate resection of the aganglionic portion of the bowel. Chronic anemia may be
present, but iron will not correct the disorder. Enemas may be given to initially
achieve bowel elimination, but they will not treat the disorder. Differentiation must
be made between this condition and psychogenic megacolon because of coercive
toileting or other emotional problems. The child with aganglionic megacolon does
not withhold stools or defecate in inappropriate places, and no soiling occurs.
Question format: Multiple Choice
Chapter 38: The Child with a Gastrointestinal/Endocrine Disorder
Cognitive Level: Apply
Client Needs: Physiological Integrity: Reduction of Risk Potential
Integrated Process: Nursing Process
Reference: p. 811

31. The nurse is caring for a child admitted with acute appendicitis. Prior to the
child going to the operating room for emergency surgery, which nursing
intervention would the nurse most likely perform?
A. The nurse gives the child laxatives to evacuate the colon.
B. The nurse encourages the child and family to express their fears.
C. The nurse administers oral fluids to prevent dehydration.
D. The nurse applies a heating pad to the abdomen to manage pain.

Answer: B

Rationale: The child facing an emergency surgery may be extremely frightened and
also may be in considerable pain. The family caregiver may be apprehensive about
impending surgery. Explain to the child and the family what is happening and why,
and encourage them to express their fears. Laxatives and enemas are
contraindicated because they increase peristalsis, which increases the possibility of
rupturing an inflamed appendix. Oral fluids are withheld and the child is NPO before
surgery. A heating pad is contraindicated because of the danger of rupture of the
appendix.
Question format: Multiple Choice
Chapter 38: The Child with a Gastrointestinal/Endocrine Disorder
Cognitive Level: Apply
Client Needs: Psychosocial Integrity
Integrated Process: Nursing Process
Reference: p. 816

32. The treatment for children with a pinworm infection is to administer which
classification of medication?
A. anticoagulants
B. anticonvulsants
C. anthelmintics
D. antipyretics

Answer: C

Rationale: Treatment consists of the use of an anthelmintic (or vermifugal, a


medication that expels intestinal worms). Anticoagulants are used to prevent clot
formation and extension. Anticonvulsants are used for seizure disorders.
Antipyretics are used to treat elevated temperatures.
Question format: Multiple Choice
Chapter 38: The Child with a Gastrointestinal/Endocrine Disorder
Cognitive Level: Remember
Client Needs: Physiological Integrity: Pharmacological and Parenteral Therapies
Integrated Process: Nursing Process
Reference: p. 817

33. The most common source of lead poisoning in children comes from which of the
following sources?
A. paint used in older homes
B. juice stored in glass jars
C. water purchased in plastic jugs
D. toys painted with spray paint

Answer: A
Rationale: Lead poisoning has other causes, but the most common cause has been
the lead in paint, especially paint used on the outside or the inside of older houses.
Other sources of lead are toys painted with lead-containing paint, drinking water
contaminated by lead pipes or copper pipes with lead-soldered joints, and fruit
juices or other food improperly stored in glazed earthenware.
Question format: Multiple Choice
Chapter 38: The Child with a Gastrointestinal/Endocrine Disorder
Cognitive Level: Remember
Client Needs: Safe, Effective Care Environment: Safety and Infection Control
Integrated Process: Nursing Process
Reference: p. 820

34. In caring for a child with lead poisoning, which method of treatment is used to
remove the lead from the child's system?
A. diuretics
B. chelating agents
C. laxatives
D. emetics

Answer: B

Rationale: The use of a chelating agent (an agent that binds with metal) increases
the urinary excretion of lead. Diuretics, laxatives, and emetics are not used in the
treatment of lead poisoning.
Question format: Multiple Choice
Chapter 38: The Child with a Gastrointestinal/Endocrine Disorder
Cognitive Level: Understand
Client Needs: Physiological Integrity: Pharmacological and Parenteral Therapies
Integrated Process: Nursing Process
Reference: p. 821

35. The nurse is interviewing the caregivers of a child admitted with a diagnosis of
type 1 diabetes mellitus. The caregiver states, "She is hungry all the time and eats
everything, but she is losing weight." The caregiver's statement indicates the child
most likely has:
A. Polyuria
B. Pica
C. Polyphagia
D. Polydipsia

Answer: C

Rationale: Symptoms of type 1 diabetes mellitus include polyphagia (increased


hunger and food consumption), polyuria (dramatic increase in urinary output,
probably with enuresis) and polydipsia (increased thirst). Pica is eating nonfood
substances.
Question format: Multiple Choice
Chapter 38: The Child with a Gastrointestinal/Endocrine Disorder
Cognitive Level: Remember
Client Needs: Physiological Integrity: Physiological Adaptation
Integrated Process: Nursing Process
Reference: p. 824

36. The nurse is interviewing the caregivers of a child admitted with a diagnosis of
type 1 diabetes mellitus. The caregiver states, "The teacher tells us that our child
has to use the restroom many more times a day than other students do." The
caregiver's statement indicates the child most likely has:
A. Polyuria
B. Pica
C. Polyphagia
D. Polydipsia

Answer: A

Rationale: Symptoms of type 1 diabetes mellitus include polyuria (dramatic


increase in urinary output, probably with enuresis), polydipsia (increased thirst),
and polyphagia (increased hunger and food consumption). Pica is eating nonfood
substances.
Question format: Multiple Choice
Chapter 38: The Child with a Gastrointestinal/Endocrine Disorder
Cognitive Level: Remember
Client Needs: Physiological Integrity: Physiological Adaptation
Integrated Process: Nursing Process
Reference: p. 824

37. The nurse is caring for a child admitted to the emergency center in diabetic
ketoacidosis. Which clinical manifestations would the nurse most likely note in this
child?
A. Pale and moist skin
B. Drowsiness and fruity odor to breath
C. Hyperactive and restless behavior
D. Slow pulse and elevated blood pressure

Answer: B

Rationale: Diabetic ketoacidosis is characterized by drowsiness, decreased skin


turgor, acetone breath with a fruity smell, and Kussmaul breathing (abnormal
increase in the depth and rate of the respiratory movements). Nausea and vomiting
may occur. If untreated, the child lapses into coma and exhibits dehydration,
electrolyte imbalance, rapid pulse, and subnormal temperature and blood pressure.
Question format: Multiple Choice
Chapter 38: The Child with a Gastrointestinal/Endocrine Disorder
Cognitive Level: Understand
Client Needs: Physiological Integrity: Physiological Adaptation
Integrated Process: Nursing Process
Reference: p. 824
38. When collecting data on a child diagnosed with diabetes mellitus, the nurse
notes that the child has had weight loss and other symptoms of the disease. The
nurse would anticipate which finding in the child's fasting glucose levels?
A. 60 mg/dl
B. 100 mg/dl
C. 140 mg/dl
D. 220 mg/dl

Answer: D

Rationale: A fasting blood sugar result of 200 mg/dL or more almost certainly is
diagnostic for diabetes when other signs, such as polyuria and weight loss despite
polyphagia, are present.
Question format: Multiple Choice
Chapter 38: The Child with a Gastrointestinal/Endocrine Disorder
Cognitive Level: Understand
Client Needs: Physiological Integrity: Reduction of Risk Potential
Integrated Process: Nursing Process
Reference: p. 824

39. The nurse is caring for a 3-year-old diagnosed with diabetes mellitus. The
child's eating patterns are unpredictable. One day the child will eat almost nothing,
the next day the child eats everything on her tray. The nurse recognizes that this
type of insulin would most likely be used in treating this child?
A. Long-acting insulin
B. Regular insulin
C. Rapid-acting insulin
D. Intermediate-acting insulin

Answer: C

Rationale: The introduction of rapid-acting insulin, such as lispro, has greatly


changed insulin administration in children. The onset of action of rapid-acting
insulin is less than 15 minutes. Rapid-acting insulin can even be used after a meal
in children with unpredictable eating habits. Regular, intermediate, and long-acting
insulin all have a longer onset, peak, and duration than rapid-acting insulin, and are
more difficult to regulate in the child with unpredictable eating patterns.
Question format: Multiple Choice
Chapter 38: The Child with a Gastrointestinal/Endocrine Disorder
Cognitive Level: Apply
Client Needs: Physiological Integrity: Pharmacological and Parenteral Therapies
Integrated Process: Nursing Process
Reference: p. 825

40. The school nurse notes that a child diagnosed with diabetes mellitus is
experiencing an insulin reaction and is unable to eat or drink. Which action would
be the most appropriate for the school nurse to take?
A. Request that someone call 911.
B. Administer subcutaneous glucagon.
C. Anticipate that the child will need intravenous glucose.
D. Dissolve a piece of candy in the child's mouth.

Answer: B

Rationale: If the child having an insulin reaction cannot take a sugar source orally,
glucagon should be administered subcutaneously to bring about a prompt increase
in the blood glucose level. This treatment prevents the long delay while waiting for
a physician to administer IV glucose or for an ambulance to reach the child.
Question format: Multiple Choice
Chapter 38: The Child with a Gastrointestinal/Endocrine Disorder
Cognitive Level: Apply
Client Needs: Physiological Integrity: Physiological Adaptation
Integrated Process: Nursing Process
Reference: p. 825

41. The nurse is collecting data for a child who is having a routine checkup. The
caregiver tells the nurse that her child eats things such as laundry starch, clay,
paper, and paint. The nurse recognizes that the child's behavior indicates that the
child likely has which disorder?
A. pica
B. invagination
C. steatorrhea
D. polyuria

Answer: A

Rationale: Pica is the ingestion of nonfood substances, such as laundry starch, clay,
paper, and paint. Invagination is the telescoping of a portion of the bowel.
Steatorrhea (fatty stools) is a condition seen in idiopathic celiac disease, and
polyuria is a dramatic increase in the urinary output.
Question format: Multiple Choice
Chapter 38: The Child with a Gastrointestinal/Endocrine Disorder
Cognitive Level: Remember
Client Needs: Physiological Integrity: Physiological Adaptation
Integrated Process: Nursing Process
Reference: p. 820

42. The nurse is collecting data on a 2-year-old child admitted with a diagnosis of
gastroenteritis. When interviewing the caregivers, which question is most
important for the nurse to ask?
A. "How many times a day does your child urinate?"
B. "How long has your child been toilet trained?"
C. "Tell me about the types of stools your child has been having."
D. "What foods has your child eaten during the last few days?"
Answer: C

Rationale: For the child with gastroenteritis, the interview with the family caregiver
must include specific information about the history of bowel patterns and the onset
of diarrheal stools, with details on the number and type of stools per day. Recent
eating patterns, determining if the child is toilet trained, and how many times a day
the child urinates are important questions, but the highest priority is gathering data
regarding the stools and stool pattern.
Question format: Multiple Choice
Chapter 38: The Child with a Gastrointestinal/Endocrine Disorder
Cognitive Level: Apply
Client Needs: Physiological Integrity: Reduction of Risk Potential
Integrated Process: Nursing Process
Reference: p. 804

43. The nurse is teaching an in-service program to a group of nurses on the topic of
gastrointestinal disorders. The nurses in the group make the following statements.
Which statement is most accurate related to the diagnosis of gastroesophageal
reflux?
A. A partial or complete intestinal obstruction occurs.
B. A thickened, elongated muscle causes an obstruction at the end of the stomach.
C. There are recurrent paroxysmal bouts of abdominal pain.
D. In this disorder the sphincter that leads into the stomach is relaxed.

Answer: D

Rationale: Gastroesophageal reflux (GER) occurs when the sphincter in the lower
portion of the esophagus, which leads into the stomach, is relaxed and allows
gastric contents to be regurgitated back into the esophagus. Congenital aganglionic
megacolon is characterized by persistent constipation resulting from partial or
complete intestinal obstruction of mechanical origin. Colic consists of recurrent
paroxysmal bouts of abdominal pain. Pyloric stenosis is characterized by
hypertrophy of the circular muscle fibers of the pylorus, which leads to an
obstruction at the distal end of the stomach.
Question format: Multiple Choice
Chapter 38: The Child with a Gastrointestinal/Endocrine Disorder
Cognitive Level: Understand
Client Needs: Physiological Integrity: Physiological Adaptation
Integrated Process: Teaching/Learning
Reference: p. 801

44. The nurse is doing teaching with the caregivers of toddler and preschool aged-
children. One of the caregivers tells the group that her child had diarrhea and she
was told that it was caused by giardiasis. Which statement made by the caregiver
indicates the most likely situation in which the child contacted the disorder?
A. "My son spent time with a neighbor who was diagnosed with pinworms."
B. "He attends a day care center four days a week while I am at work."
C. "I won't let his sister take bubble baths but I do let him take one a few times a
week."
D. "My mother is in a nursing home but I always make the kids wash their hands
before we leave her."

Answer: B

Rationale: Giardiasis is caused by the protozoan parasite Giardia lamblia. It is a


common cause of diarrhea and is prevalent in children who attend day care centers
and other types of residential facilities; it may be found in contaminated mountain
streams or pools frequented by diapered infants. Bubble baths can lead to urinary
tract infections, but are not the cause of Giardiasis infestations. It is not related to
either C. Diff or pinworms.
Question format: Multiple Choice
Chapter 38: The Child with a Gastrointestinal/Endocrine Disorder
Cognitive Level: Apply
Client Needs: Safe, Effective Care Environment: Safety and Infection Control
Integrated Process: Nursing Process
Reference: p. 818

45. The nurse is educating the parents of a client newly diagnosed with type 1
diabetes. Which statement by the parents indicates additional teaching is needed?
A. "When our child is sick, we may need to check glucose levels more frequently."
B. "Our child should eat three meals and midafternoon and bedtime snacks each
day."
C. "We and our child need to learn to identify carbohydrate, protein, and fat foods."
D. "Our child should not participate in sports or physical activity."

Answer: D

Rationale: The nurse would provide additional education if the parents state the
child should not participate in sports or physical activity. The child with diabetes
can, and should, be physically active to maintain proper health and facilitate
efficient insulin usage by the body. Glucose levels should be checked more
frequently during times of sickness, as well as assessing the urine for ketones.
Consistency of intake can help prevent complications and maintain near-normal
blood glucose levels. The parents and child should know how to identify foods to
adequately monitor the child's nutritional intake. A dietitian with expertise in
diabetes education should be consulted for referral as needed.
Question format: Multiple Choice
Chapter 38: The Child with a Gastrointestinal/Endocrine Disorder
Cognitive Level: Analyze
Client Needs: Physiological Integrity: Physiological Adaptation
Integrated Process: Teaching/Learning
Reference: p. 828

46. The nurse is teaching a group of caregivers of children diagnosed with diabetes
mellitus. The nurse is explaining insulin shock and the caregivers make the
following statements. Which statement indicates the best understanding of a
reason an insulin reaction might occur?
A. "If my child eats as much as their older brother eats they could have an insulin
reaction."
B. "My child measures their own medication but sometimes doesn't administer the
correct amount."
C. "My child monitors their glucose levels to keep them from going too high."
D. "On the weekends we encourage our child to participate in lots of sports
activities and stay busy so they don't have an insulin reaction."

Answer: B

Rationale: Insulin reaction (insulin shock, hypoglycemia) is caused by insulin


overload, resulting in too-rapid metabolism of the body's glucose. This may be
attributable to a change in the body's requirement, carelessness in diet (such as
failure to eat proper amounts of food), an error in insulin measurement, or
excessive exercise.
Question format: Multiple Choice
Chapter 38: The Child with a Gastrointestinal/Endocrine Disorder
Cognitive Level: Analyze
Client Needs: Physiological Integrity: Physiological Adaptation
Integrated Process: Nursing Process
Reference: p. 825

47. The nurse is doing dietary teaching with the caregivers of a child diagnosed
with idiopathic celiac disease. What food will the nurse recommend as an
appropriate diet choice?
A. Bananas
B. Toast
C. Oatmeal
D. Canned soup

Answer: A

Rationale: People with idiopathic celiac disease will need to avoid gluten in the diet.
Of the foods listed, the nurse should recommend the child eat bananas, which do
not contain gluten and are usually well tolerated. Products that contain wheat, rye,
or oats should be excluded. The nurse should teach the family that gluten is often
hidden in processed or canned food, for example, many canned soups contain
hidden gluten.
Question format: Multiple Choice
Chapter 38: The Child with a Gastrointestinal/Endocrine Disorder
Cognitive Level: Understand
Client Needs: Physiological Integrity: Physiological Adaptation
Integrated Process: Teaching/Learning
Reference: p. 801
48. The parent of a child diagnosed with intussusception asks the nurse why the
child's stool looks so strange. How should the nurse reply?
A. "Your child's stool is mixed with blood and mucus, so it looks very different."
B. "Your child's stool looks like that because the intestine is inflamed."
C. "The stool looks like that from the barium used to reduce the intussusception."
D. "I know it concerns you, but the stool is normal for the problem your child is
having."

Answer: A

Rationale: In the child with intussusception, the stools consist of blood and mucus,
thereby earning the name currant jelly stool. There is a telescoping of the upper
portion of the bowel slipping over the lower portion. The condition occurs more
often in boys than in girls and the highest incidence occurs in infants between the
ages of 4 and 10 months. The infant who previously appeared healthy and happy
suddenly becomes pale, cries out sharply, and draws up the legs in a severe,
colicky spasm of pain. This spasm may last for several minutes, after which the
infant relaxes and appears well until the next episode, which may occur 5, 10, or 20
minutes later. The stool does not have the appearance because of infection or
inflammation. It is because of the stricture caused by the telescoping of the bowel.
Making these statements gives the parent false information. Stating "I know it
concerns you" might be true but the remainder of the sentence telling the parent
the stool is normal is not true and is not reassuring to the parent. Barium or air
may be used to reduce the intussusception, but it's not the reason for the color and
consistency of the stool.
Question format: Multiple Choice
Chapter 38: The Child with a Gastrointestinal/Endocrine Disorder
Cognitive Level: Apply
Client Needs: Physiological Integrity: Physiological Adaptation
Integrated Process: Teaching/Learning
Reference: p. 814

49. The nurse working with the child diagnosed with type 2 diabetes mellitus
recognizes the disorder can be managed by:
A. taking oral hypoglycemic agents.
B. increasing carbohydrates in the diet, especially in the evening.
C. conserving energy with rest periods during the day.
D. decreasing amounts of daily insulin.

Answer: A

Rationale: Oral hypoglycemic agents, such as metformin, are often effective for
controlling blood glucose levels in children diagnosed with type 2 diabetes mellitus.
Insulin may be used for a child with type 2 diabetes if oral hypoglycemic agents
alone are not effective, but "decreasing" the daily insulin would not help treat this
disorder. Lifestyle changes such as increased exercise (not conserving energy by
resting during the day), and limiting large amounts of carbohydrates are important
aspects of treatment for the child.
Question format: Multiple Choice
Chapter 38: The Child with a Gastrointestinal/Endocrine Disorder
Cognitive Level: Apply
Client Needs: Physiological Integrity: Pharmacological and Parenteral Therapies
Integrated Process: Nursing Process
Reference: p. 833

50. The nurse admits a 7-year-old child who reports pain in the lower right
quadrant of the abdomen, nausea, and constipation. An assessment shows that the
child has a fever of 101℉ (38.3℃). Which nursing intervention should the nurse
implement to safely address the child's reported pain?
A. Request a prescription for a laxative.
B. Place a heating pad or hot water bottle on the abdomen.
C. Help the child find a comfortable position.
D. Give the child an analgesic such as acetaminophen.

Answer: C

Rationale: The child's symptoms indicate possible appendicitis. When appendicitis is


suspected, laxatives and enemas are contraindicated because they increase
peristalsis, which increases the possibility of rupturing an inflamed appendix. Heat
to the abdomen is also contraindicated because of the danger of rupture of the
appendix. Medicating with analgesics is inappropriate, because medication may
conceal signs of tenderness that are important for diagnosis. Comfort can be
provided through positioning.
Question format: Multiple Choice
Chapter 38: The Child with a Gastrointestinal/Endocrine Disorder
Cognitive Level: Apply
Client Needs: Physiological Integrity: Basic Care and Comfort
Integrated Process: Nursing Process
Reference: p. 815
Chapter 39
1. The nurse is caring for a child who is being evaluated for a possible
nephroblastoma. Which nursing intervention would be important for this child?
A. Monitor for protein in the urine at each voiding.
B. Protect the child from having the abdomen palpated.
C. Check blood pressure every 2 hours.
D. Measure the child's intake and output every hour.

Answer: B

Rationale: When the child is being evaluated and treated, abdominal palpation
should be avoided because cells may break loose and spread the tumor.
Question format: Multiple Choice
Chapter 39: The Child with a Genitourinary Disorder
Cognitive Level: Understand
Client Needs: Physiological Integrity: Reduction of Risk Potential
Integrated Process: Nursing Process
Reference: p. 846

2. The caregiver of a child with a history of ear infections calls the nurse and
reports that her son has just told her his urine "looks funny." He also has a
headache, and his mother reports that his eyes are puffy. Although he had a fever
2 days ago, his temperature is now down to 100 ℉ (37.8 ℃). The nurse encourages
the mother to have the child seen by the care provider because the nurse suspects
the child may have:
A. a urinary tract infection.
B. lipoid nephrosis (idiopathic nephrotic syndrome).
C. acute glomerulonephritis.
D. rheumatic fever.

Answer: C

Rationale: Acute glomerulonephritis is a condition that appears to be an allergic


reaction to specific infections, most often group A beta-hemolytic streptococcal
infections such as rheumatic fever. Presenting symptoms appear 1 to 3 weeks after
the onset of a streptococcal infection such as strep throat, otitis media, tonsillitis, or
impetigo. Usually the presenting symptom is grossly bloody urine. Periorbital
edema may accompany or precede hematuria. Fever may be 103 ℉ to 104 ℉ (39.4
℃ to 40 ℃) at the onset, but decreases in a few days to about 100 ℉ (37.8 ℃).
Slight headache and malaise are usual, and vomiting may occur.
Question format: Multiple Choice
Chapter 39: The Child with a Genitourinary Disorder
Cognitive Level: Analyze
Client Needs: Physiological Integrity: Physiological Adaptation
Integrated Process: Teaching/Learning
Reference: p. 841

3. A caregiver brings her 7-year-old son to the pediatrician's office, concerned


about the child's bedwetting after being completely toilet trained even at night for
over 2 years. The caregiver further reports that the child has wet the bed every
night since returning home from a 1-week fishing trip. The child refuses to talk
about the bedwetting. The nurse notes the child is shy, skittish, and will not make
eye contact. Further evaluation needs to be done to rule out what possible
explanation for the bedwetting?
A. The child has a urinary tract infection due to not bathing while on the fishing trip.
B. The child is out of the habit of waking himself up during the night to void.
C. The child did not want to go on the fishing trip and is now retaliating against
being made to go.
D. The child has been sexually abused, maybe on the fishing trip.

Answer: D

Rationale: Enuresis may have a physiologic or psychological cause and may indicate
a need for further exploration and treatment. Enuresis in the older child may be an
expression of resentment toward family caregivers or of a desire to regress to an
earlier level of development to receive more care and attention. Emotional stress
can be a precipitating factor. The health care team also needs to consider the
possibility that enuresis can be a symptom of sexual abuse. Bruising, bleeding, or
lacerations on the external genitalia, especially in the child who is extremely shy
and frightened, may be a sign of child abuse (child mistreatment) and should be
further explored.
Question format: Multiple Choice
Chapter 39: The Child with a Genitourinary Disorder
Cognitive Level: Apply
Client Needs: Psychosocial Integrity
Integrated Process: Nursing Process
Reference: p. 840

4. A 12-year-old girl who has not yet reached menarche comes to the pediatrician's
office for her annual well-child check. As the nurse is weighing and measuring her,
the child says emphatically that she does not want to get her period. Which
response would be most appropriate for the nurse to make to this child?
A. "What have you heard about it that makes you worried?"
B. "But it's a good thing, having a period is a part of growing up."
C. "Are you afraid of getting pregnant?"
D. "Do you think it will hurt?"

Answer: A

Rationale: The beginning of menstruation, called menarche, normally occurs


between the ages of 12 and 13 years. For many girls this is a joyous affirmation of
their womanhood, but others may have negative feelings about the event
depending on how they have been prepared for menarche and for their roles as
women. The nurse would need to explore the child's understanding of the
implications of menarche.
Question format: Multiple Choice
Chapter 39: The Child with a Genitourinary Disorder
Cognitive Level: Apply
Client Needs: Health Promotion and Maintenance
Integrated Process: Nursing Process
Reference: p. 846

5. A single male caregiver of a 14-year-old girl accompanies his daughter to her


pre-high school physical. In the course of discussion about how his daughter is
developing, he remarks, "She's terrific most of the time. Of course when she gets
her period, she's miserable and mean, but I tell her that's just what it's like to be a
woman." What would be the most appropriate response by the nurse?
A. "PMS is a problem for a lot of women, but sometimes it's worse in the beginning.
She might outgrow it."
B. "There are nutritional and medical things she can do to lessen the symptoms; I'll
give both of you information about some strategies and we'll track her for a few
months."
C. "That must be hard on you, especially because you are raising her by yourself."
D. "That doesn't make being a woman sound very good. It would probably be
easier for her if you could be more supportive."

Answer: B

Rationale: Women of all ages are subject to the discomfort of premenstrual


syndrome (PMS), but the symptoms may be alarming to the adolescent. Symptoms
include edema (resulting in weight gain), headache, increased anxiety, mild
depression, and mood swings. Generally the discomforts of PMS are minor and can
be relieved by reducing salt intake during the week before menstruation, taking
mild analgesics, and applying local heat. When symptoms are more severe, the
physician may prescribe a mild diuretic to be taken the week before menstruation
to relieve edema; occasionally, oral contraceptive pills are prescribed to prevent
ovulation.
Question format: Multiple Choice
Chapter 39: The Child with a Genitourinary Disorder
Cognitive Level: Apply
Client Needs: Physiological Integrity: Physiological Adaptation
Integrated Process: Teaching/Learning
Reference: p. 846

6. The nurse is reinforcing teaching with the caregiver of 5-year-old twins regarding
urinary tract infections (UTIs). The caregiver is puzzled about why her daughter has
had three urinary tract infections but her son has had none. She reports that their
diets and fluid intake is similar. Which statement would be accurate for the nurse to
tell this mother?
A. "A girl's urethra is much shorter and straighter than a boy's, so it can be
contaminated fairly easily."
B. "Girls tend to urinate less frequently than boys, making them more susceptible
to UTI's."
C. "Girls need more vitamin C than boys to keep their urinary tract healthy, so your
daughter may be deficient in vitamin C."
D. "It is unlikely that your daughter is practicing good cleaning habits after she
voids."

Answer: A

Rationale: Many different bacteria may infect the urinary tract, and intestinal
bacteria, particularly Escherichia coli, account for about 80% of acute episodes. The
female urethra is shorter and straighter than the male urethra, so it is more easily
contaminated with feces.
Question format: Multiple Choice
Chapter 39: The Child with a Genitourinary Disorder
Cognitive Level: Apply
Client Needs: Physiological Integrity: Physiological Adaptation
Integrated Process: Teaching/Learning
Reference: p. 837-838

7. A child who has been diagnosed with minimal change nephrotic syndrome
(MCNS) is being discharged after a 3-week hospitalization. Her edema has been
greatly reduced and her appetite is beginning to return. Her caregivers have
promised to have a family party to celebrate her return. The child has requested
the following foods for the party. Which of these foods would the nurse suggest is
appropriate for this child's diet?
A. banana splits
B. popcorn
C. potato chips
D. orange soda

Answer: A

Rationale: For the child with nephrotic syndrome, the addition of salt is
discouraged, and sometimes the child is put on a low sodium diet. In addition, the
child may be placed on a high protein diet. Popcorn, potato chips, and orange soda
all have higher sodium content than a banana split. The banana split would also
have higher protein content.
Question format: Multiple Choice
Chapter 39: The Child with a Genitourinary Disorder
Cognitive Level: Analyze
Client Needs: Physiological Integrity: Basic Care and Comfort
Integrated Process: Nursing Process
Reference: p. 843

8. The caregiver of a child being treated at home for acute glomerulonephritis calls
the nurse reporting that her daughter has just had a convulsion. The child is resting
comfortably but the caregiver would like to know what to do. The nurse would
instruct the caregiver to take which action?
A. Weigh the child in the same clothes she had been weighed in the day before and
report the two weights to the nurse while the nurse is on the phone.
B. Take the child's blood pressure and report the findings to the nurse while the
nurse is still on the phone.
C. Give the child a diuretic and report back to the nurse in a few hours.
D. Give the child fluids and report back to the nurse in a few hours.

Answer: B

Rationale: Blood pressure should be monitored regularly using the same arm and a
properly fitting cuff. If hypertension develops, a diuretic may help reduce the blood
pressure to normal levels. An antihypertensive drug may be added if the diastolic
pressure is 90 mm Hg or higher. The concern is immediate so reporting the findings
in a few hours could delay needed treatment. The child should be weighed daily in
the same clothes and using the same scale, but the blood pressure is the priority in
this situation.
Question format: Multiple Choice
Chapter 39: The Child with a Genitourinary Disorder
Cognitive Level: Apply
Client Needs: Physiological Integrity: Reduction of Risk Potential
Integrated Process: Teaching/Learning
Reference: p. 841

9. If the newborn is following a normal development process, the child will most
likely void when which amount of urine is in the bladder?
A. 3 ml
B. 6 ml
C. 15 ml
D. 25 ml

Answer: C

Rationale: In the newborn, the bladder empties when about 15 ml of urine is


present.
Question format: Multiple Choice
Chapter 39: The Child with a Genitourinary Disorder
Cognitive Level: Remember
Client Needs: Health Promotion and Maintenance
Integrated Process: Nursing Process
Reference: p. 837-838

10. The location of the kidneys in the child in relationship to the location of the
kidneys in the adult makes which fact a greater likelihood in the child?
A. The adult has less fat to cushion the kidney.
B. The child has a greater risk for trauma to the kidney.
C. The child has more frequent urges to empty the bladder.
D. The adult has a greater chance of retaining fluids than the child.

Answer: B

Rationale: The kidneys in children are located lower in relationship to the ribs than
in adults. This placement and the fact that the child has less of a fat cushion around
the kidneys cause the child to be at greater risk for trauma to the kidneys. The
location of the kidneys does not affect the urges to empty the bladder nor the
retaining of fluids.
Question format: Multiple Choice
Chapter 39: The Child with a Genitourinary Disorder
Cognitive Level: Apply
Client Needs: Health Promotion and Maintenance
Integrated Process: Nursing Process
Reference: p. 837-838

11. If the child follows a normal development process, the child's kidneys will most
likely have reached their full size and function by which age?
A. 5 years of age
B. 8 years of age
C. 12 years of age
D. 20 years of age

Answer: C

Rationale: The kidneys reach their full size and function by the time the child is an
adolescent.
Question format: Multiple Choice
Chapter 39: The Child with a Genitourinary Disorder
Cognitive Level: Remember
Client Needs: Health Promotion and Maintenance
Integrated Process: Nursing Process
Reference: p. 837-838

12. The nurse is discussing urinary tract infections (UTI's) in children with a group
of peers. Which fact is the most accurate regarding urinary tract infection seen in
children?
A. Urinary tract infections are rarely seen after toilet training.
B. The most common age for UTIs in children is 2 to 6 years of age.
C. Males between the ages of 10 to 12 years of age commonly get UTIs.
D. Girls who have gone through puberty most commonly get UTIs.

Answer: B

Rationale: Urinary tract infections (UTIs) are fairly common in the "diaper age," in
infancy, and again between the ages of 2 and 6 years. Older school-aged and
adolescent girls are not as prone to UTIs.
Question format: Multiple Choice
Chapter 39: The Child with a Genitourinary Disorder
Cognitive Level: Understand
Client Needs: Health Promotion and Maintenance
Integrated Process: Nursing Process
Reference: p. 838-839

13. Most urinary tract infections seen in children are caused by:
A. hereditary causes.
B. fungal infections.
C. intestinal bacteria.
D. dietary insufficiencies.

Answer: C

Rationale: Although many different bacteria may infect the urinary tract, intestinal
bacteria, particularly Escherichia coli, account for about 80% of acute episodes.
Hereditary and dietary concerns are not causes of urinary tract infections.
Question format: Multiple Choice
Chapter 39: The Child with a Genitourinary Disorder
Cognitive Level: Remember
Client Needs: Physiological Integrity: Physiological Adaptation
Integrated Process: Nursing Process
Reference: p. 838

14. The nurse is teaching a group of nursing students about genitourinary


conditions. The nurse tells these students about a condition that occurs when there
is an inflammation of the kidney and renal pelvis. The condition the nurse is
referring to is:
A. oliguria.
B. amenorrhea.
C. pyelonephritis.
D. ascites.

Answer: C

Rationale: Pyelonephritis is an inflammation of the kidney and renal pelvis. Oliguria


is a subnormal volume of urine. Amenorrhea is the absence of menstruation.
Ascites is edema in the peritoneal cavity.
Question format: Multiple Choice
Chapter 39: The Child with a Genitourinary Disorder
Cognitive Level: Remember
Client Needs: Physiological Integrity: Physiological Adaptation
Integrated Process: Nursing Process
Reference: p. 838-839

15. The nurse is discussing genitourinary conditions with a group of 16-year-old


girls. One of the girls says she has heard about girls who have stopped taking birth
control pills and now don't have periods. The condition the girl is referring to is:
A. oliguria.
B. amenorrhea.
C. pyelonephritis.
D. ascites.

Answer: B

Rationale: Amenorrhea is the absence of menstruation. Pyelonephritis is an


inflammation of the kidney and renal pelvis. Oliguria is a subnormal volume of
urine. Ascites is edema in the peritoneal cavity.
Question format: Multiple Choice
Chapter 39: The Child with a Genitourinary Disorder
Cognitive Level: Remember
Client Needs: Physiological Integrity: Physiological Adaptation
Integrated Process: Teaching/Learning
Reference: p. 847

16. The first method of choice for obtaining a urine specimen from a 3-year-old
child with a possible urinary tract infection is:
A. performing a suprapubic aspiration.
B. placing a cotton ball in the underwear to catch urine.
C. placing an indwelling urinary catheter.
D. obtaining a clean catch voided urine.

Answer: D

Rationale: In the cooperative, toilet-trained child, a clean midstream urine may be


used successfully to obtain a "clean catch" voided urine. If a culture is needed, the
child may be catheterized, but this is usually avoided if possible. A suprapubic
aspiration also may be done to obtain a sterile specimen. In the toilet-trained child,
using a cotton ball to collect the urine would not be appropriate.
Question format: Multiple Choice
Chapter 39: The Child with a Genitourinary Disorder
Cognitive Level: Apply
Client Needs: Physiological Integrity: Reduction of Risk Potential
Integrated Process: Nursing Process
Reference: p. 839

17. Urinary tract infections are usually successfully treated by what means?
A. Increasing fluids, such as cranberry juice
B. Administering antibiotics
C. Performing bladder irrigations
D. Administering diuretics

Answer: B

Rationale: UTIs may be treated with antibiotics (usually sulfamethoxazole or


ampicillin) at home. Fluids are encouraged, but they do not treat the infection.
Bladder irrigations and diuretics are not used in the treatment of urinary tract
infections.
Question format: Multiple Choice
Chapter 39: The Child with a Genitourinary Disorder
Cognitive Level: Remember
Client Needs: Physiological Integrity: Pharmacological and Parenteral Therapies
Integrated Process: Nursing Process
Reference: p. 838-839

18. The nurse is collecting data on a 2-year-old child admitted with a diagnosis of
urinary tract infection. When interviewing the caregivers, which question would be
most important for the nurse to ask?
A. "Is your child potty trained?"
B. "Has your child complained of pain?"
C. "How often do you bathe your child?"
D. "Do any of your other children have a temperature?"

Answer: B

Rationale: Gather information about the current illness: when the fever started and
its course thus far; signs of pain or discomfort on voiding; recent change in feeding
pattern; presence of vomiting or diarrhea; irritability; lethargy; abdominal pain;
unusual odor to urine; chronic diaper rash; and signs of febrile convulsions. Toilet
training and bathing habits would be of importance, but they are not the most
important to ask. Temperatures in other children in the family would not be related
to this child's current situation.
Question format: Multiple Choice
Chapter 39: The Child with a Genitourinary Disorder
Cognitive Level: Apply
Client Needs: Physiological Integrity: Reduction of Risk Potential
Integrated Process: Nursing Process
Reference: p. 838-839

19. The nurse is collecting data on a 6-year-old child admitted with a possible
urinary tract infection. Which vital sign might indicate the possibility of an infection?
A. pulse rate 135 bpm
B. pulse oximetry 93% on room air
C. respirations 22 per minute
D. blood Pressure 100/70

Answer: A

Rationale: Data to collect regarding the child includes temperature, pulse (be alert
for tachycardia) and respiration rates; normal vital signs for a 6-year-old would be
a pulse rate of 70 to 115 beats per minute, so this rate shows tachycardia. The
other vital signs are all within normal limits for a child this age.
Question format: Multiple Choice
Chapter 39: The Child with a Genitourinary Disorder
Cognitive Level: Apply
Client Needs: Physiological Integrity: Reduction of Risk Potential
Integrated Process: Nursing Process
Reference: p. 839

20. When caring for a child who has a urinary tract infection, which nursing
interventions would be most appropriate. Select all that apply.
A. The nurse encourages the child to void every 30 minutes.
B. The nurse positions the child on a bedpan rather than on the toilet.
C. The nurse observes for signs of pain or burning on urination.
D. The nurse monitors intake and output.
E. The nurse administers pain medications each time the child voids.
F. The nurse administers antipyretics as needed.

Answer: C, D, F

Rationale: Because of pain and burning on urination, the toilet-trained child may try
to hold urine and not void. Encourage the child to void every 3 or 4 hours to
prevent recurrent infection. Observe the child for signs of burning and pain when he
or she is urinating. Monitor and measure urine output. Antipyretic medications may
be ordered to reduce body temperature. A bedpan would not be necessary in most
children with a UTI. Pain medications following each voiding would not be indicated.
Question format: Multiple Select
Chapter 39: The Child with a Genitourinary Disorder
Cognitive Level: Apply
Client Needs: Physiological Integrity: Basic Care and Comfort
Integrated Process: Nursing Process
Reference: p. 839

21. The nurse recognizes that what would be a likely physiologic cause for a child to
have enuresis?
A. Regression to get attention
B. Stress and stressful situations
C. Sexual abuse
D. Sleeping too soundly

Answer: D

Rationale: Physiologic causes may include a small bladder capacity, urinary tract
infection, and lack of awareness of the signal to empty the bladder because of
sleeping too soundly. Psychological causes might include rigorous toilet training,
resentment toward family caregivers, a desire to regress to an earlier level of
development to receive more care and attention, or emotional stress and stressful
situations. Enuresis can be a symptom of sexual abuse.
Question format: Multiple Choice
Chapter 39: The Child with a Genitourinary Disorder
Cognitive Level: Understand
Client Needs: Physiological Integrity: Basic Care and Comfort
Integrated Process: Nursing Process
Reference: p. 840

22. The nurse is teaching an in-service program to a group of colleagues on the


topic of children diagnosed with acute glomerulonephritis. In which age range is the
peak incidence of this disorder noted?
A. 2 to 4 years of age
B. 6 to 7 years of age
C. 12 to 13 years of age
D. 15 to 17 years of age

Answer: B

Rationale: Acute glomerulonephritis has a peak incidence in children 6 to 7 years of


age and occurs twice as often in boys.
Question format: Multiple Choice
Chapter 39: The Child with a Genitourinary Disorder
Cognitive Level: Remember
Client Needs: Health Promotion and Maintenance
Integrated Process: Nursing Process
Reference: p. 841

23. The nurse is collecting data for a child diagnosed with acute glomerulonephritis.
What would the nurse likely find in this child's history?
A. The child has a sibling with the same diagnosis.
B. The child had a congenital heart defect.
C. The child recently had an ear infection.
D. The child is being treated for asthma.

Answer: C

Rationale: In the child with acute glomerulonephritis, presenting symptoms appear


1 to 3 weeks after the onset of a streptococcal infection, such as strep throat, otitis
media, tonsillitis, or impetigo. There is not a family history of the disorder, a history
of congenital concerns or defects, nor asthma in children with acute
glomerulonephritis.
Question format: Multiple Choice
Chapter 39: The Child with a Genitourinary Disorder
Cognitive Level: Apply
Client Needs: Physiological Integrity: Physiological Adaptation
Integrated Process: Nursing Process
Reference: p. 841

24. The nurse is caring for a child admitted with acute glomerulonephritis. Which
clinical manifestation would likely have been noted in the child with this diagnosis?
A. Loose, dark stools
B. Tea-colored urine
C. Strawberry-red tongue
D. Jaundiced skin

Answer: B

Rationale: The presenting symptom in acute glomerulonephritis is grossly bloody


urine. The caregiver may describe the urine as tea- or cola-colored. Periorbital
edema may accompany or precede hematuria. Loose stools are seen in diarrhea. A
strawberry-colored tongue is a symptom seen in the child with Kawasaki disease.
Jaundiced skin is noted in hepatitis.
Question format: Multiple Choice
Chapter 39: The Child with a Genitourinary Disorder
Cognitive Level: Understand
Client Needs: Physiological Integrity: Physiological Adaptation
Integrated Process: Nursing Process
Reference: p. 841

25. A symptom often seen in acute glomerulonephritis is edema. The most


common site the edema is first noted is in which area of the body?
A. Ankles
B. Hands
C. Eyes
D. Sacrum

Answer: C

Rationale: Periorbital edema may accompany or precede hematuria in children with


acute glomerulonephritis. Edema in the ankles, hands, and sacrum are not noted in
acute glomerulonephritis.
Question format: Multiple Choice
Chapter 39: The Child with a Genitourinary Disorder
Cognitive Level: Apply
Client Needs: Physiological Integrity: Physiological Adaptation
Integrated Process: Nursing Process
Reference: p. 841

26. The nurse is collecting data on a 6-year-old child admitted with acute
glomerulonephritis. Which vital sign would the nurse anticipate with this child's
diagnosis?
A. Pulse rate 112 bpm
B. Pulse oximetry 93% on room air
C. Respirations 24 per minute
D. Blood pressure 136/84

Answer: D

Rationale: Hypertension appears in 60% to 70% of clients during the first 4 or 5


days with a diagnosis of acute glomerulonephritis. The pulse of 112 would be a little
high for a child this age, but not a concern with this diagnosis. The other vital signs
are within normal limits for a child of this age.
Question format: Multiple Choice
Chapter 39: The Child with a Genitourinary Disorder
Cognitive Level: Apply
Client Needs: Physiological Integrity: Physiological Adaptation
Integrated Process: Nursing Process
Reference: p. 841

27. When caring for a child who has a diagnosis of acute glomerulonephritis, which
nursing interventions would most likely be included in the child's plan of care?
Select all that apply.
A. The nurse encourages ambulation several times a day.
B. The nurse promotes increased fluid intake.
C. The nurse administers diuretics.
D. The nurse administers antihypertensives.
E. The nurse weighs the child every day using the same scale.
F. The nurse dipsticks the child's urine to test for protein.

Answer: C, D, E, F

Rationale: Fluid intake and urinary output should be carefully monitored and
recorded. Special attention is needed to keep the intake within prescribed limits.
The amount of fluid the child is allowed may be based on output, as well as on
evidence of continued hypertension and oliguria. If hypertension develops, a
diuretic may help reduce the blood pressure to normal levels. An antihypertensive
drug may be added if the diastolic pressure is 90 mm Hg or higher. Weigh the child
daily, at the same time, on the same scale, and in the same clothes. The urine
must be tested regularly for protein and hematuria using dipstick tests. Bed rest
should be maintained until acute symptoms and gross hematuria disappear.
Question format: Multiple Select
Chapter 39: The Child with a Genitourinary Disorder
Cognitive Level: Apply
Client Needs: Physiological Integrity: Physiological Adaptation
Integrated Process: Nursing Process
Reference: p. 841-842

28. The nurse is teaching an in-service program on children diagnosed with


nephrotic syndrome. Which statement made by the nurse accurately reflects
information on the disease process?
A. "These children have such a big appetite and are always hungry."
B. "The child may look chubby, but he is really malnourished."
C. "When you look at the urine of these children it is smoky and bloody."
D. "Their lab work shows a decreased level of cholesterol."

Answer: B
Rationale: In children with nephrotic syndrome, malnutrition may become severe.
The generalized edema masks the loss of body tissue, causing the child to present a
chubby appearance and to double his or her weight. After diuresis, the malnutrition
becomes quite apparent. Anorexia, irritability, and loss of appetite develop.
Hematuria is not usually present, although a few red blood cells may appear in the
urine. There is an increase in the level of cholesterol in the blood.
Question format: Multiple Choice
Chapter 39: The Child with a Genitourinary Disorder
Cognitive Level: Analyze
Client Needs: Physiological Integrity: Physiological Adaptation
Integrated Process: Nursing Process
Reference: p. 842

29. The LPN is working with the RN to develop a plan of care for a child with
nephrotic syndrome. Which of the following would be appropriate goals of
treatment for this child? Select all that apply.
A. conserving energy
B. encouraging a high-salt diet
C. preventing infection
D. restricting protein intake
E. promoting coping

Answer: A, C, E

Rationale: The major goals for the child with nephrotic syndrome are relieving
edema, improving nutritional status, maintaining skin integrity, conserving energy,
and preventing infection. The family goals include learning about the disease and
treatments, as well as learning ways to cope with the child's long-term care. The
child may be on a no-added-salt or low-salt diet, but there are usually no food
restrictions.
Question format: Multiple Select
Chapter 39: The Child with a Genitourinary Disorder
Cognitive Level: Understand
Client Needs: Safe, Effective Care Environment: Management of Care
Integrated Process: Nursing Process
Reference: p. 843

30. The nurse is caring for an 8-year-old child hospitalized with nephrotic
syndrome. Which nursing intervention would be appropriate for this child?
A. Measure the abdominal girth daily.
B. Weigh the child once a week.
C. Test the urine for ketones twice a day.
D. Administer antipyretics as needed.

Answer: A

Rationale: Measure the child's abdomen daily at the level of the umbilicus, and
make certain that all staff personnel measure at the same level. Weigh the child at
the same time every day on the same scale in the same clothing. Test the urine
regularly for albumin and specific gravity. Elevated temperature is not an issue with
nephrotic syndrome.
Question format: Multiple Choice
Chapter 39: The Child with a Genitourinary Disorder
Cognitive Level: Apply
Client Needs: Physiological Integrity: Physiological Adaptation
Integrated Process: Nursing Process
Reference: p. 843

31. A group of nursing students is discussing terminology related to the


genitourinary system during a post-conference setting. One of the students asks
what mittelschmerz is or what it means. A classmate of this student correctly
answers that mittelschmerz is:
A. a symptom of premenstrual syndrome.
B. the beginning of menstruation.
C. a dull, aching abdominal pain at ovulation.
D. a medication given to treat dysmenorrhea.

Answer: C

Rationale: Mittelschmerz is a dull, aching abdominal pain at the time of ovulation


(hence the name, which means "midcycle"). The beginning of menstruation is called
menarche. Symptoms include edema (resulting in weight gain), headache,
increased anxiety, mild depression, and mood swings. Nonsteroidal anti-
inflammatory drugs (NSAIDs) such as ibuprofen (Advil, Motrin) inhibit
prostaglandins and are the treatment of choice for primary dysmenorrhea, which is
painful menstruation.
Question format: Multiple Choice
Chapter 39: The Child with a Genitourinary Disorder
Cognitive Level: Remember
Client Needs: Physiological Integrity: Physiological Adaptation
Integrated Process: Teaching/Learning
Reference: p. 846

32. The nurse is doing a presentation for a group of nursing students about the
topic of menstrual disorders. After discussing the disorder secondary amenorrhea,
the students make the following statements. Which statement made by the nursing
students is the most accurate regarding the cause of secondary amenorrhea?
A. "It is caused from taking birth control pills when a girl is younger than 13 years
old."
B. "This disorder is usually seen after a girl has had a spontaneous abortion
(miscarriage)."
C. "Emotional stress can be a cause of this disorder."
D. "This is what happens if a 16-year-old girl has never had any periods at all."

Answer: C
Rationale: Secondary amenorrhea can be the result of discontinuing contraceptives,
a sign of pregnancy, the result of physical or emotional stress, or a symptom of an
underlying medical condition. A complete physical examination, including
gynecologic screening, is necessary to help determine the cause. Primary
amenorrhea occurs when a girl has had no previous menstruation. A spontaneous
abortion (miscarriage) does not cause secondary amenorrhea.
Question format: Multiple Choice
Chapter 39: The Child with a Genitourinary Disorder
Cognitive Level: Analyze
Client Needs: Physiological Integrity: Physiological Adaptation
Integrated Process: Teaching/Learning
Reference: p. 847

33. A child is hospitalized with nephrotic syndrome. Which measurement is best for
the nurse to determine the child's edema?
A. weight, daily
B. urine output, every shift
C. amount of protein in the urine
D. abdominal circumference

Answer: A

Rationale: The classic sign of nephrotic syndrome is edema. It is usually


generalized, but may be manifested as ascites or be periorbital depending on the
seriousness of the disease. The easiest way to determine edema is by weighing the
child. The child should be weighed on the same scale, at the same time daily, and
with the same amount of clothing. The abdomen would only need to be measured if
ascites was suspected or known. Measuring urine output will not determine edema,
although it should be done to determine if urine is being produced in adequate
amounts. Measuring the amount of protein in the urine will also not determine
edema. The measurement is important to determine the progress of the disease,
however.
Question format: Multiple Choice
Chapter 39: The Child with a Genitourinary Disorder
Cognitive Level: Apply
Client Needs: Physiological Integrity: Physiological Adaptation
Integrated Process: Nursing Process
Reference: p. 842-843

34. A school-aged child has come to the clinic with symptoms of a urinary tract
infection. The child reports dysuria, frequency and hesitancy. What nursing
assessment is most important for the nurse to complete?
A. Assess for bladder distention.
B. Measure the urine output.
C. Monitor the temperature.
D. Assess for flank pain.

Answer: A
Rationale: In school-aged and older children, a urinary tract infection can be
manifested by fever, vomiting, dysuria, frequency, hesitancy, urgency, flank pain
and poor appetite. Because there are dysuria and hesitancy, the bladder may not
empty fully. It is most important for the nurse to palpate the bladder for distention.
Keeping a distended bladder can cause reflux and continue to harbor bacteria. The
urine output can be measured to determine the amount, but the urine also provides
a look at the color for hematuria. The temperature should be measured, and
antipyretics administered if necessary. Pain medication may be prescribed by the
health care provider. The nurse can also recommend warm sitz baths and the use
of a heating pad over the flank area for pain relief.
Question format: Multiple Choice
Chapter 39: The Child with a Genitourinary Disorder
Cognitive Level: Analyze
Client Needs: Physiological Integrity: Reduction of Risk Potential
Integrated Process: Nursing Process
Reference: p. 839
Chapter 40
1. The nurse caring for a child who has been put into a leg cast must be on the alert for
signs of nerve and muscle damage. Which symptom might be an early warning signal that
the child has developed compartment syndrome? The child:
A. cannot plantarflex his foot.
B. feels increasing severe pain.
C. has a weak femoral pulse.
D. has blue-looking nail beds on the toes.

Answer: B

Rationale: Any reports of pain in a child with a new cast or immobilized extremity need to
be explored and monitored closely for the possibility of compartment syndrome.
Question format: Multiple Choice
Chapter 40: The Child with a Musculoskeletal Disorder
Cognitive Level: Apply
Client Needs: Physiological Integrity: Physiological Adaptation
Integrated Process: Nursing Process
Reference: p. 854

2. The caregivers of an 8-year-old child diagnosed with muscular dystrophy are discussing
appropriate activities for their child. The nurse might suggest to these caregivers that they
take which action in regard to their child's activities?
A. Find a Little League team that encourages participation of kids at all disability levels.
B. Find an art class that will allow the child to participate without physical exertion.
C. Modify family activities so they are more sedentary for the child.
D. Teach the child at home so that the child does not feel overly different.

Answer: A

Rationale: The child must be encouraged to be as active as possible to delay muscle atrophy
and contractures. To help keep the child active, physiotherapy, diet to avoid obesity, and
parental encouragement are important. The nurse should advise the family to keep the
child's life as normal as possible.
Question format: Multiple Choice
Chapter 40: The Child with a Musculoskeletal Disorder
Cognitive Level: Apply
Client Needs: Health Promotion and Maintenance
Integrated Process: Caring
Reference: p. 861
3. The nurse is talking with the caregiver of a 13-year-old diagnosed with scoliosis. The
child has come to the clinic to be fitted with a brace to begin her treatment. The child
appears upset and angry and states, "I hate this brace; I hate it already." In an effort to
support this child, which statement would be the most appropriate for the nurse to make
to this child's caregiver?
A. "Remind your child that her spine needs to be corrected in order to keep her whole
musculoskeletal system healthy for a long, long time."
B. "Take your daughter to an oncology floor for a few minutes so she can see children who
are much sicker than she is."
C. "If you can afford it, let your daughter choose an article or two of clothing that she can
wear with the brace that will help her feel that she looks good."
D. "Children her age often withdraw during stressful times; let her have some time alone to
think about the situation and to get used to the brace."

Answer: C

Rationale: Help the child select clothing that blends with current styles but is loose enough
to hide the brace. Self-image and the need to be like others are very important at this age.
Wearing a brace creates a distinct change in body image, especially in the older child or
adolescent, at a time when body consciousness is at an all-time high. The need to wear the
brace and deal with the limitations it involves may cause anger; the change in body image
can cause a grief reaction. Handling these feelings successfully requires understanding
support from the nurse, family, and peers. It is important for the child to have an
opportunity to talk about his or her feelings.
Question format: Multiple Choice
Chapter 40: The Child with a Musculoskeletal Disorder
Cognitive Level: Apply
Client Needs: Psychosocial Integrity
Integrated Process: Caring
Reference: p. 868

4. The nurse is caring for a group of children on the pediatric unit. The nurse should collect
further data and explore the possibility of child abuse (child mistreatment) in which
situation?
A. A 10-year-old with a simple fracture of the femur, which the caregiver reports as having
been caused when the child fell down a set of stairs.
B. A 7-year-old with a spiral fracture of the humerus, which the caregiver reports as having
been caused when the child was hit by a bat swung by a Little League teammate.
C. A 9-year-old with a compound fracture of the tibia, which the caregiver reports as having
been caused when the child attempted a flip on a skateboard.
D. A 6-year-old with a greenstick fracture of the wrist, which the caregiver reports as
having been caused when the child fell while ice-skating.

Answer: B
Rationale: Spiral fractures, which twist around the bone, are frequently associated with
child abuse (child mistreatment) and are caused by a wrenching force. When a broken bone
penetrates the skin, the fracture is called compound, or open. A simple, or closed, fracture
is a single break in the bone without penetration of the skin. In a greenstick fracture, the
bone bends and often just partially breaks.
Question format: Multiple Choice
Chapter 40: The Child with a Musculoskeletal Disorder
Cognitive Level: Analyze
Client Needs: Psychosocial Integrity
Integrated Process: Nursing Process
Reference: p. 853

5. The caregiver of a child with a recent puncture wound on the arm calls the pediatrician's
office reporting that after seeming well at bedtime last night, the child now has a
temperature of 101 ℉ (38.3 ℃), pain at the site of the injury, and is unable to fully bend the
elbow of the arm which had been injured. The nurse recommends the child be brought in to
see the health care provider. What would likely be ordered for this child?
A. an x-ray
B. a muscle biopsy
C. blood work
D. a surgical consult

Answer: C

Rationale: In osteomyelitis, diagnosis is based on laboratory findings of leukocytosis


(15,000 to 25,000 cells or more), an increased ESR, and positive blood cultures.
Radiographic examination does not reveal the process until 5 to 10 days after the onset. A
biopsy is not indicated with these symptoms.
Question format: Multiple Choice
Chapter 40: The Child with a Musculoskeletal Disorder
Cognitive Level: Analyze
Client Needs: Physiological Integrity: Reduction of Risk Potential
Integrated Process: Nursing Process
Reference: p. 859

6. The nurse is caring for a child diagnosed with Legg–Calvé–Perthes disease (LCPD). What
is the most important nursing intervention for the nurse to include in working with this
child and his caregivers?
A. The nurse should provide information when the child or caregiver requests it.
B. The nurse should be a contact person when the child is hospitalized.
C. The nurse should support the caregivers in restricting activity during the treatment.
D. The nurse should help the caregivers to understand and help the child to effectively use
the corrective devices.

Answer: D
Rationale: Nursing care focuses on helping the child and caregivers to manage the
corrective device and on the importance of compliance to promote healing and to avoid
long-term disability.
Question format: Multiple Choice
Chapter 40: The Child with a Musculoskeletal Disorder
Cognitive Level: Understand
Client Needs: Physiological Integrity: Physiological Adaptation
Integrated Process: Nursing Process
Reference: p. 863

7. The nurse admits an 11-year-old boy who reports lower leg pain after he was hit with a
lacrosse stick during a game. The level of pain is not consistent with the description of the
incident. With further questioning, the nurse discovers the child has had pain in that spot
off and on for several months. The child has a biopsy, a bone scan and a bone marrow
aspiration ordered by the care provider. What is the most likely condition the care
provider is looking for?
A. osteosarcoma
B. Ewing sarcoma
C. Legg–Calvé–Perthes syndrome
D. acute osteomyelitis

Answer: B

Rationale: Many times an injury draws attention to the pain at the site of the tumor with
Ewing sarcoma. The pain may be sporadic for a period of time, but continues and becomes
severe enough to keep the child awake at night. Metastasis to the lung and other bones may
already have taken place by the time of diagnosis. A biopsy, bone scan, and bone marrow
aspiration are done to further diagnose the tumor.
Question format: Multiple Choice
Chapter 40: The Child with a Musculoskeletal Disorder
Cognitive Level: Apply
Client Needs: Physiological Integrity: Physiological Adaptation
Integrated Process: Nursing Process
Reference: p. 863

8. The nurse is caring for a child diagnosed 3 months ago with juvenile idiopathic arthritis
(JIA). The caregiver states that the child has recently reported little pain and is not
currently taking aspirin or NSAIDs. The caregiver also tells the nurse that just to be on the
safe side, she is continuing to keep the child from doing physical exercise. The mother
states, "I think we have beaten this disease." In working with this child and the caregiver,
which statement would be best for the nurse to make?
A. "It sounds as though things are going well. Be sure to restart the medication as soon as
there is any flare-up of pain."
B. "That is not what is best. The medications aren't for pain; she needs to take aspirin every
few days."
C. "Let's review some of the instructions. She does need to take an anti-inflammatory every
day."
D. "I'm glad you're feeling optimistic. She should exercise now but give her aspirin before
she does."

Answer: C

Rationale: Teach family caregivers the importance of regular administration of the


medications, even when the child is not experiencing pain. The primary purpose of aspirin
or NSAIDs is not to relieve pain but to decrease joint inflammation.
Question format: Multiple Choice
Chapter 40: The Child with a Musculoskeletal Disorder
Cognitive Level: Apply
Client Needs: Physiological Integrity: Reduction of Risk Potential
Integrated Process: Teaching/Learning
Reference: p. 864

9. The caregiver of a 2-year-old who has a polyurethane resin cast on her arm calls the
clinic to report that her child is crying and says that the cast has sand in it. The caregiver
states that she has had casts herself and knows how badly they can itch. She says she
always used a hanger to scratch but is worried that it will be too sharp for the child. Which
statement would be appropriate for the nurse to make to this caregiver?
A. "Since the child's cast is synthetic, she could soak it with cool water."
B. "A plastic ruler is less likely than a hanger to cut the child's skin."
C. "Nothing should be put into the cast. You can blow cool air into it with a hair dryer."
D. "You could give the child an extra dose of acetaminophen and see if that helps."

Answer: C

Rationale: Children and caregivers should be cautioned not to put anything inside the cast,
no matter how much the casted area itches. Small toys and sticks or stick-like objects
should be kept out of reach until the cast has been removed. Ice packs applied over the cast
may help decrease the itching. Blowing cool air through a cast with a hair dryer set on a
cool temperature or using a fan may help to relieve discomfort under a cast.
Question format: Multiple Choice
Chapter 40: The Child with a Musculoskeletal Disorder
Cognitive Level: Apply
Client Needs: Physiological Integrity: Basic Care and Comfort
Integrated Process: Teaching/Learning
Reference: p. 855

10. In understanding the physiology of the musculoskeletal system, the nurse recognizes
that which of the following are stored in the bones?
A. vitamins
B. minerals
C. spinal fluid
D. cartilage

Answer: B

Rationale: Minerals such as calcium, phosphorus, magnesium, and fluoride are stored in the
bones.
Question format: Multiple Choice
Chapter 40: The Child with a Musculoskeletal Disorder
Cognitive Level: Remember
Client Needs: Health Promotion and Maintenance
Integrated Process: Nursing Process
Reference: p. 852

11. In understanding the function of the musculoskeletal system, the nurse recognizes that
which of the following allows for movement of the body parts?
A. tendons
B. ligaments
C. joints
D. cartilage

Answer: C

Rationale: Bones are attached to each other by connecting links called joints, which allow
for movement of the body parts. Skeletal muscles attach to the bones, with a moveable joint
between them. Tendons and ligaments hold the muscles and bones together. Cartilage is a
type of connective tissue consisting of cells implanted in a gel-like substance, which
gradually calcifies and becomes bone.
Question format: Multiple Choice
Chapter 40: The Child with a Musculoskeletal Disorder
Cognitive Level: Remember
Client Needs: Health Promotion and Maintenance
Integrated Process: Nursing Process
Reference: p. 851

12. In understanding the development of the musculoskeletal system, the nurse recognizes
that what is implanted in a gel-like substance during fetal life?
A. Tendons
B. Ligaments
C. Joints
D. Cartilage

Answer: D
Rationale: During fetal life, tissue called cartilage, which is a type of connective tissue
consisting of cells implanted in a gel-like substance, gradually calcifies and becomes bone.
Question format: Multiple Choice
Chapter 40: The Child with a Musculoskeletal Disorder
Cognitive Level: Remember
Client Needs: Health Promotion and Maintenance
Integrated Process: Nursing Process
Reference: p. 852

13. The musculoskeletal system is made up of various types of bones. Most bone growth is
completed by which age?
A. 5 years of age
B. 10 years of age
C. 14 years of age
D. 20 years of age

Answer: D

Rationale: Bone growth takes place between birth and puberty, with most growth being
complete by 20 years of age.
Question format: Multiple Choice
Chapter 40: The Child with a Musculoskeletal Disorder
Cognitive Level: Remember
Client Needs: Health Promotion and Maintenance
Integrated Process: Nursing Process
Reference: p. 852

14. The nurse is working with a group of caregivers of school-aged children discussing
fractures. The nurse explains that if the fragments of fractured bone are separated, the
fracture is said to be:
A. Incomplete
B. Complete
C. Greenstick
D. Spiral

Answer: B

Rationale: If the fragments of fractured bone are separated, the fracture is said to be
complete. If fragments remain partially joined, the fracture is termed incomplete.
Greenstick fractures are one kind of incomplete fracture, caused by incomplete ossification,
common in children. Spiral fractures twist around the bone.
Question format: Multiple Choice
Chapter 40: The Child with a Musculoskeletal Disorder
Cognitive Level: Remember
Client Needs: Physiological Integrity: Physiological Adaptation
Integrated Process: Teaching/Learning
Reference: p. 852

15. The nurse is educating a group of caregivers about fractures seen in children. One of the
caregivers states, "I have heard that if a bone breaks it can cause permanent damage and
stop the growth of the bone." This statement is accurate if the break occurs in the:
A. humerus.
B. joint.
C. xiphoid process.
D. epiphyseal plate.

Answer: D

Rationale: Fractures in the area of the epiphyseal plate (growth plate) can cause permanent
damage and severely impair growth.
Question format: Multiple Choice
Chapter 40: The Child with a Musculoskeletal Disorder
Cognitive Level: Remember
Client Needs: Physiological Integrity: Physiological Adaptation
Integrated Process: Teaching/Learning
Reference: p. 853

16. The nurse is discussing types of treatment used when working with children who have
orthopedic disorders. Which form of treatment covers the lower part of the body, usually
from the waist down, and either one or both legs while leaving the feet open?
A. Internal fixation device
B. External fixation device
C. Spica cast
D. Stockinette

Answer: C

Rationale: The hip spica cast covers the lower part of the body, usually from the waist
down, and either one or both legs while leaving the feet open. The cast maintains the legs in
a frog-like position. Usually, there is a bar placed between the legs to help support the cast.
Question format: Multiple Choice
Chapter 40: The Child with a Musculoskeletal Disorder
Cognitive Level: Remember
Client Needs: Physiological Integrity: Basic Care and Comfort
Integrated Process: Teaching/Learning
Reference: p. 854

17. The nurse is caring for a child who has just had a plaster cast applied to the arm. The
nurse is correct in performing which action with this child?
A. Handling the cast with open palms when moving the arm.
B. Using only a draw sheet to move the casted arm.
C. Keeping a clove-hitch restraint gently tied on the hand to stabilize the arm.
D. Encouraging the child to move the arm slowly up and down to help the cast dry.

Answer: A

Rationale: A wet plaster cast should be handled only with open palms because fingertips
can cause indentations and result in pressure points. There is no reason the arm should be
restrained or the arm moved to aid in the drying process.
Question format: Multiple Choice
Chapter 40: The Child with a Musculoskeletal Disorder
Cognitive Level: Understand
Client Needs: Physiological Integrity: Basic Care and Comfort
Integrated Process: Nursing Process
Reference: p. 854

18. The caregiver of a child who has had a cast applied to the leg observes the nurse putting
adhesive tape strips around the edge of the cast. The caregiver asks the nurse why she is
doing this. The best response by the nurse would be:
A. "We put these on so the child will not pull the padding from under the cast."
B. "These will help the cast look more attractive so the child won't feel self-conscious."
C. "These make a smooth edge on the cast so the skin is better protected."
D. "In case the child has an accident and misses the bedpan, these can be changed to keep
the area dry."

Answer: C

Rationale: If the cast has no protective edge, it should be petaled with adhesive tape strips.
These help keep the skin protected from the rough edge of the cast. If the cast is near the
genital area, plastic should be taped around the edge to prevent wetting and soiling of the
cast; petaling the cast does not provide protection to keep the cast dry.
Question format: Multiple Choice
Chapter 40: The Child with a Musculoskeletal Disorder
Cognitive Level: Understand
Client Needs: Physiological Integrity: Basic Care and Comfort
Integrated Process: Nursing Process
Reference: p. 854

19. The nurse is doing neurovascular checks on a child who has had a cast applied to treat a
fracture. The nurse observes for diminished or absent sensation and numbness or tingling.
In doing this the nurse is monitoring for which symptom?
A. pain
B. pallor
C. paresthesia
D. paralysis

Answer: C
Rationale: Paresthesia is diminished or absent sensation or numbness or tingling. Pallor is
paleness of color and paralysis is the loss of function.
Question format: Multiple Choice
Chapter 40: The Child with a Musculoskeletal Disorder
Cognitive Level: Understand
Client Needs: Physiological Integrity: Reduction of Risk Potential
Integrated Process: Nursing Process
Reference: p. 855

20. The nurse is reinforcing discharge teaching with the caregivers of a child who is going
home after a cast has been applied. The nurse explains to the caregivers that which issues
should be reported if they occur or are seen related to this child? Select all that apply.
A. Any area on the cast that is warm to the touch
B. A foul odor under the cast
C. Any itching under or around the edges of the cast
D. Drainage from under the cast
E. Any pink color in the fingers or toes of casted extremity
F. Looseness of the cast on the extremity

Answer: A, B, D, F

Rationale: In addition to the five Ps, any foul odor or drainage on or under the cast, "hot
spots" on the cast (areas warm to the touch), looseness or tightness, or any elevation of
temperature must be noted, documented, and reported. Family caregivers should be
instructed to watch carefully for these same danger signals. Itching is common and does
not need to be reported. Pink coloration of fingers and toes would be normal and not a
concern.
Question format: Multiple Select
Chapter 40: The Child with a Musculoskeletal Disorder
Cognitive Level: Apply
Client Needs: Physiological Integrity: Physiological Adaptation
Integrated Process: Teaching/Learning
Reference: p. 855

21. The nurse is reinforcing teaching with the caregivers of a child who has had a cast on
the arm removed after 6 weeks. The caregiver makes the following statements. Which
statement indicates an understanding of the teaching?
A. "As soon as we get home I will scrub his arm really well to remove the loose skin."
B. "He loves to play outside now that the weather is nice; I will have to remind him to use
sunscreen."
C. "We will be careful to not let him put his little sister's baby oil on his arm for a few
weeks."
D. "I usually let him take a tub bath, but we will keep him out of warm water for a couple of
days."

Answer: B
Rationale: After cast removal, sunscreen should be applied to the previously casted area
when the child will have sun exposure. The casted area should be soaked in warm water to
help remove the crusty layer of accumulated skin. Application of oil or lotion may prove
comforting. Family caregivers and the child must be cautioned against scrubbing or
scraping this area because the tender layer of new skin underneath the crust may bleed.
Question format: Multiple Choice
Chapter 40: The Child with a Musculoskeletal Disorder
Cognitive Level: Apply
Client Needs: Physiological Integrity: Basic Care and Comfort
Integrated Process: Nursing Process
Reference: p. 855

22. The type of traction in which a pin, wire, tongs, or other device is surgically inserted
through a bone is:
A. skin traction.
B. skeletal traction.
C. Russell traction.
D. Buck extension traction.

Answer: B

Rationale: Skeletal traction exerts pull directly on skeletal structures by means of a pin,
wire, tongs, or other device surgically inserted through a bone. Skin traction applies pull on
tape, rubber, or a plastic material attached to the skin, which indirectly exerts pull on the
musculoskeletal system. Examples of skin traction are Bryant traction, Buck extension
traction, and Russell traction.
Question format: Multiple Choice
Chapter 40: The Child with a Musculoskeletal Disorder
Cognitive Level: Remember
Client Needs: Physiological Integrity: Basic Care and Comfort
Integrated Process: Nursing Process
Reference: p. 856

23. The nurse is reinforcing teaching with the caregivers of a child who has been placed in
an external fixation device for the treatment of an orthopedic condition. Which statement
made by the caregivers indicates an understanding of the external fixation device?
A. "We will have to get some of the elastic bandages to place around the pins and pin sites."
B. "It will be hard, but we know our child will be in this device for a long time."
C. "He is very sensitive about the way the device looks. I am glad that his clothes will fully
cover it so his friends won't tease him."
D. "If we see any drainage around the pins when we are cleaning them, we won't be
concerned."

Answer: B
Rationale: External fixation devices are sometimes left in place for as long as 1 year. The
pin sites are left open to the air and should be inspected and cleansed every 8 hours. The
child and caregiver should be able to recognize the signs of infection at the pin sites. The
appearance of the pins puncturing the skin and the unusual appearance of the device can
be upsetting to the child.
Question format: Multiple Choice
Chapter 40: The Child with a Musculoskeletal Disorder
Cognitive Level: Analyze
Client Needs: Physiological Integrity: Reduction of Risk Potential
Integrated Process: Teaching/Learning
Reference: p. 859

24. A nursing student tells the staff nurse on the pediatric orthopedic unit that she has
heard of a musculoskeletal disorder in which there is an infection of the bone. Which
disorder does this statement describe?
A. osteomyelitis
B. muscular dystrophy
C. osteosarcoma
D. juvenile idiopathic arthritis

Answer: A

Rationale: Osteomyelitis is an infection of the bone usually caused by Staphylococcus


aureus. Acute osteomyelitis is twice as common in boys and results from a primary
infection.
Question format: Multiple Choice
Chapter 40: The Child with a Musculoskeletal Disorder
Cognitive Level: Remember
Client Needs: Physiological Integrity: Physiological Adaptation
Integrated Process: Nursing Process
Reference: p. 859

25. The nurse is caring for a child with osteomyelitis who has a leg wound. The highest
priority nursing intervention for this child would be for the nurse to:
A. minimize the movement of the leg.
B. follow transmission-based precautions.
C. monitor nutritional intake.
D. encourage the child to avoid weight bearing.

Answer: B

Rationale: All of these interventions are done for the child with osteomyelitis who has a
wound, but the highest priority would be to follow transmission-based precautions to
prevent the spread of infection, especially if the wound is open and draining.
Question format: Multiple Choice
Chapter 40: The Child with a Musculoskeletal Disorder
Cognitive Level: Apply
Client Needs: Safe, Effective Care Environment: Safety and Infection Control
Integrated Process: Nursing Process
Reference: p. 860

26. The child diagnosed with muscular dystrophy uses a method of rising from the floor
which is referred to as which of the following?
A. Legg–Calvé–Perthes
B. Gowers sign
C. Milwaukee brace
D. Boston brace

Answer: B

Rationale: The child cannot rise easily to an upright position from a sitting or squatting
position on the floor; instead, he or she develops Gowers sign, a method where the child
rises from the floor by "walking up" the lower extremities with the hands.
Question format: Multiple Choice
Chapter 40: The Child with a Musculoskeletal Disorder
Cognitive Level: Understand
Client Needs: Physiological Integrity: Physiological Adaptation
Integrated Process: Nursing Process
Reference: p. 860

27. The nurse is caring for a child with a possible diagnosis of muscular dystrophy. The
nurse explains to the parents that which of the following will likely be used to confirm this
child's diagnosis?
A. an x-ray
B. a muscle biopsy
C. complete blood count
D. a surgical consult

Answer: B

Rationale: A decrease in muscle fibers, which is seen in a muscle biopsy, can confirm the
diagnosis of muscular dystrophy.
Question format: Multiple Choice
Chapter 40: The Child with a Musculoskeletal Disorder
Cognitive Level: Understand
Client Needs: Physiological Integrity: Reduction of Risk Potential
Integrated Process: Nursing Process
Reference: p. 861
28. A child diagnosed with muscular dystrophy has recently developed kyphosis. The home
health nurse stresses to the caregivers that the child is at an increased risk for which
injury?
A. falling when attempting to stand
B. decreasing cognitive abilities
C. possibility of respiratory infection
D. excessive weight gain

Answer: C

Rationale: When a child with muscular dystrophy becomes wheelchair-bound, kyphosis


(hunchback) develops and causes a decrease in respiratory function and an increase in the
incidence of infections. Breathing exercises are a daily necessity for these children.
Question format: Multiple Choice
Chapter 40: The Child with a Musculoskeletal Disorder
Cognitive Level: Apply
Client Needs: Physiological Integrity: Physiological Adaptation
Integrated Process: Nursing Process
Reference: p. 862

29. Legg-Calvé-Perthes disease (LCPD) is more likely to be seen in which group?


A. Asian girls
B. Black boys
C. white boys
D. Hispanic girls

Answer: C

Rationale: Legg-Calvé-Perthes disease (LCPD) is an aseptic necrosis of the head of the


femur. It occurs 4 to 5 times more often in boys than in girls and 10 times more often in
whites than in other ethnic groups.
Question format: Multiple Choice
Chapter 40: The Child with a Musculoskeletal Disorder
Cognitive Level: Remember
Client Needs: Physiological Integrity: Physiological Adaptation
Integrated Process: Nursing Process
Reference: p. 862

30. The nurse is caring for a child admitted with juvenile idiopathic arthritis (JIA). Which
clinical manifestation would likely have been noted in the child with this diagnosis?
A. difficulty standing and walking
B. inflammation of the joints
C. poor posture and malformed vertebrae
D. pain in the groin and a limp

Answer: B
Rationale: In the child with juvenile idiopathic arthritis, joint inflammation occurs first; if
untreated, inflammation leads to irreversible changes in joint cartilage, ligaments, and
menisci (the crescent-shaped fibrocartilage in the knee joints), eventually causing complete
immobility.
Question format: Multiple Choice
Chapter 40: The Child with a Musculoskeletal Disorder
Cognitive Level: Understand
Client Needs: Physiological Integrity: Physiological Adaptation
Integrated Process: Nursing Process
Reference: p. 862

31. The caregivers of a child with juvenile idiopathic arthritis are asking the nurse about
exercise for their child. The nurse explains to the caregivers that gentle daily exercise is
needed to prevent which condition from occurring?
A. scoliosis
B. ankylosis
C. kyphosis
D. lordosis

Answer: B

Rationale: In the child with juvenile idiopathic arthritis, joints must be immobilized by
splinting during active disease, but gentle daily exercise is necessary to prevent ankylosis
(immobility of a joint). The nurse should stress to the caregivers the importance of
encouraging the child to perform independent activities of daily living to maintain function
and independence.
Question format: Multiple Choice
Chapter 40: The Child with a Musculoskeletal Disorder
Cognitive Level: Apply
Client Needs: Physiological Integrity: Physiological Adaptation
Integrated Process: Nursing Process
Reference: p. 864

32. The school nurse is doing a presentation to a group of caregivers of children diagnosed
with scoliosis. One of the caregivers asks the nurse about structural scoliosis. Which
condition is involved with the diagnosis of structural scoliosis?
A. poor posture
B. muscle spasm caused by trauma
C. unequal leg length
D. rotated and malformed vertebrae

Answer: D
Rationale: Structural scoliosis involves rotated and malformed vertebrae. Functional
scoliosis can have several causes: poor posture, muscle spasm caused by trauma, or
unequal length of legs.
Question format: Multiple Choice
Chapter 40: The Child with a Musculoskeletal Disorder
Cognitive Level: Remember
Client Needs: Physiological Integrity: Physiological Adaptation
Integrated Process: Teaching/Learning
Reference: p. 864

33. The nurse is discussing treatment for a child diagnosed with scoliosis. Which statement
indicates the parents understand the nurse's education?
A. "Because our child has scoliosis, treatment will include halo traction."
B. "The treatment for our child's scoliosis is anticipated to last between 3 to 4 months."
C. "Because our child is being treated by using braces, the braces will have to be worn
almost all the time."
D. "The most successful treatment for scoliosis is surgery before reaching adult age."

Answer: C

Rationale: The Boston or the thoracolumbosacral orthosis (TLSO) brace is made of plastic
and is customized to fit the child for treatment of scoliosis. The brace should be worn
constantly, except during bathing or swimming, to achieve the greatest benefit. Halo
traction may be used to treat clients with severe scoliosis, but not all clients. Children will
be reassessed every 4 to 6 months to determine the prognosis for continuing brace therapy
and potentially refitting. Bracing may be indicated for months or years. Surgery may be
indicated, depending on the severity and complications resulting from the scoliosis;
however, surgery is not the best option for all clients.
Question format: Multiple Choice
Chapter 40: The Child with a Musculoskeletal Disorder
Cognitive Level: Apply
Client Needs: Physiological Integrity: Reduction of Risk Potential
Integrated Process: Teaching/Learning
Reference: p. 865

34. A type of traction sometimes used in the treatment of the child with scoliosis is called:
A. Russell traction.
B. halo traction.
C. Bryant traction.
D. Dunlop traction.

Answer: B

Rationale: When a child has a severe spinal curvature or cervical instability, a form of
traction known as halo traction may be used to reduce spinal curves and straighten the
spine. Halo traction is achieved by using stainless steel pins inserted into the skull while
counter-traction is applied by using pins inserted into the femur. Weights are increased
gradually to promote correction.
Question format: Multiple Choice
Chapter 40: The Child with a Musculoskeletal Disorder
Cognitive Level: Remember
Client Needs: Physiological Integrity: Basic Care and Comfort
Integrated Process: Nursing Process
Reference: p. 859

35. In caring for a child being treated for scoliosis, the highest priority goal is which of the
following?
A. The child will participate in age-appropriate activities.
B. The child's skin integrity will be maintained.
C. The child will verbalize acceptance of the treatment.
D. The child will remain free of injury.

Answer: D

Rationale: The highest priority goal for the child being treated for scoliosis is preventing
injury. Other important goals, but with less priority, include minimizing the disruption of
activities and maintaining skin integrity and self-image. Goals for the child and caregiver
include accepting the treatment and complying with long-term care.
Question format: Multiple Choice
Chapter 40: The Child with a Musculoskeletal Disorder
Cognitive Level: Apply
Client Needs: Safe, Effective Care Environment: Management of Care
Integrated Process: Nursing Process
Reference: p. 867

36. The nurse caring for a child diagnosed with juvenile idiopathic arthritis (JIA) should
provide medication education to the caregivers for which first-choice medication to
manage the child's pain and joint inflammation?
A. Acetaminophen
B. Naproxen
C. Aspirin
D. Codeine

Answer: B

Rationale: Enteric-coated aspirin had long been the drug of choice for JIA, but because of
the concern of aspirin therapy and Reye syndrome, NSAIDs such as naproxen,
indomethacin, and ibuprofen are being used in the treatment of JIA. Acetaminophen is not
an appropriate substitute because it lacks anti-inflammatory properties. Codeine is not
prescribed for symptoms associated with JIA.
Question format: Multiple Choice
Chapter 40: The Child with a Musculoskeletal Disorder
Cognitive Level: Apply
Client Needs: Physiological Integrity: Pharmacological and Parenteral Therapies
Integrated Process: Nursing Process
Reference: p. 863

37. The nurse is caring for a child admitted with possible Legg–Calvé–Perthes disease.
Which assessment question should the nurse ask the child's caregivers to help support this
diagnosis?
A. "Does your child have difficulty standing or walking?"
B. "Are your child's knees every swollen and red?"
C. "Have you ever been told your child has any malformed vertebrae?"
D. "Does your child report pain in the groin that results in a limp?"

Answer: D

Rationale: Symptoms first noticed in Legg–Calvé–Perthes disease are pain in the hip or
groin and a limp accompanied by muscle spasms and limitation of motion.
Question format: Multiple Choice
Chapter 40: The Child with a Musculoskeletal Disorder
Cognitive Level: Apply
Client Needs: Physiological Integrity: Reduction of Risk Potential
Integrated Process: Nursing Process
Reference: p. 862
Chapter 41
1. An 8-month-old has been diagnosed with infantile eczema. At a follow-up
appointment, the child's caregiver seems exhausted and angry. He explains that he
has done all of the child's care because his wife is repulsed by the child's raw and
uncomfortable appearance. What responses would be appropriate for the nurse to
say to this caregiver?
A. "He will be better soon and your family can get back to normal."
B. "I understand her feelings. It is hard to see a child in pain sometimes."
C. "That's not fair to you; she should get some counseling to learn how to cope with
illness better."
D. "That's not an uncommon reaction, although it's hard on you and on your child."

Answer: D

Rationale: The family caregivers of the child with eczema are often frustrated and
exhausted. Family caregivers may feel apprehensive or repulsed by this unsightly
child. Support them in expressing their feelings and help them view this as a
distressing but temporary skin condition. Although the caregiver can be assured
that most cases of eczema clear up by the age of 2, this does little to relieve the
present situation.
Question format: Multiple Choice
Chapter 41: The Child with an Integumentary Disorder/Communicable Disease
Cognitive Level: Apply
Client Needs: Psychosocial Integrity
Integrated Process: Caring
Reference: p. 880

2. A pediatric client who has been seriously burned is being given IV fluid
replacements. It has been determined that the client will initially need 24 ounces of
replacement fluids. In following a normal burn replacement treatment for this child,
if the treatment is started at 10:00 AM, which of the following would be correct?
The child would have received:
A. 12 ounces of IV fluid replacement by 4:00 PM.
B. 12 ounces of IV fluid replacement by 6:00 PM.
C. 18 ounces of IV fluid replacement by 4:00 PM.
D. 18 ounces of IV fluid replacement by 6:00 PM.

Answer: B

Rationale: Intravenous fluids for maintenance and replacement of lost body fluids
are estimated for the first 24 hours, with half of this calculated requirement given
during the first 8 hours.
Question format: Multiple Choice
Chapter 41: The Child with an Integumentary Disorder/Communicable Disease
Cognitive Level: Analyze
Client Needs: Physiological Integrity: Physiological Adaptation
Integrated Process: Nursing Process
Reference: p. 886

3. The caregiver of a fifth-grade boy calls the pediatrician's office and reports that
her son has been scratching at his groin for a week or so. Despite his need for
privacy, he's finally allowed the caregiver to look at the groin area. The caregiver
describes the area to the nurse saying, "It looks like black pin dots that seem to
have dark tails." The nurse suspects that the child may have which of the following?
A. lice
B. scabies
C. ringworm
D. prickly heat

Answer: B

Rationale: Scabies burrows are visible as dark lines and the mite is seen as a black
dot at the end of the burrow. Lice can be seen as tiny, pearly-white flecks attached
to the hair shafts. They look much like dandruff, but dandruff flakes can be flicked
off easily, whereas the nits are tightly attached and not easily removed. Ringworm
of the scalp begins as a small papule on the scalp which then spreads, leaving scaly
patches of baldness. Ringworm of the body appears as a scaly ring with clearing in
the center. Ringworm of the feet is more commonly known as athlete's foot. It is
evidenced by the scaling or cracking of the skin between the toes.
Question format: Multiple Choice
Chapter 41: The Child with an Integumentary Disorder/Communicable Disease
Cognitive Level: Apply
Client Needs: Physiological Integrity: Reduction of Risk Potential
Integrated Process: Nursing Process
Reference: p. 878

4. The dermatologist treating a 16-year-old girl with a history of severe acne has
ordered a pregnancy test so she can be started on a course of isotretinoin. The
teen's caregiver has said that her daughter is a virgin and she refuses to allow her
to have the required pregnancy testing. What would be the best action for the
nurse to take?
A. Acknowledge the caregiver's discomfort about the pregnancy testing but
encourage her to allow the daughter to have the testing so that she can use the
medication.
B. Speak with the teen alone to ask her if she is sexually active. If she says she is
not sexually active, let the provider know that it is okay to write the prescription.
C. Arrange for the daughter to have the pregnancy testing without the caregiver's
knowledge.
D. Have the caregiver sign a permission form acknowledging that the girl has been
given the drug without pregnancy testing.

Answer: A
Rationale: Isotretinoin is a pregnancy category X drug: It must not be used at all
during pregnancy because of serious risk of fetal abnormalities. To rule out
pregnancy, a urine test is done before beginning treatment. For the sexually active
adolescent girl, an effective form of contraception must be used for a month before
beginning and during isotretinoin therapy. The risk to the fetus, should pregnancy
occur, should be discussed with the girl whether she is sexually active or not.
Question format: Multiple Choice
Chapter 41: The Child with an Integumentary Disorder/Communicable Disease
Cognitive Level: Apply
Client Needs: Physiological Integrity: Pharmacological and Parenteral Therapies
Integrated Process: Nursing Process
Reference: p. 876

5. The nurse admits a 10-year-old who has just eaten lobster and has hives over
much of the body. In collecting data regarding this child, which question should the
nurse ask the caregiver first?
A. "Has the child ever eaten shellfish before now?"
B. "Is your child allergic to peanuts or other foods?"
C. "Does anyone in your family have any food allergies?"
D. "Have you ever given your child antihistamines?"

Answer: A

Rationale: The first time the child comes in contact with an allergen, no reaction
may be evident, but an immune response is stimulated—helper lymphocytes
stimulate B lymphocytes to make the immunoglobulin E (IgE) antibody. The IgE
antibody attaches to mast cells and macrophages. When contacted again, the
allergen attaches to the IgE receptor sites, and a response occurs in which certain
substances, such as histamine, are released; these substances produce the
symptoms known as allergy. Asking the other questions is important, but the first
question the nurse should ask is related to this child and this situation.
Question format: Multiple Choice
Chapter 41: The Child with an Integumentary Disorder/Communicable Disease
Cognitive Level: Apply
Client Needs: Safe, Effective Care Environment: Safety and Infection Control
Integrated Process: Nursing Process
Reference: p. 878

6. The caregiver of a 4-year-old girl who lives in a heavily wooded area calls the
clinic nurse to report that the child has a swollen tick on her arm. What would be
the best action for the caregiver to follow in removing the tick?
A. The caregiver should take the child to a health care facility where the tick can be
removed aseptically.
B. The caregiver should have another adult hold the child still, light a match and let
it burn for 1 second, then blow out the match and quickly hold it on the tick.
C. The caregiver should hold a gauze pad beneath the tick to catch the blood and
carefully crush the tick.
D. The caregiver should use tweezers to carefully remove the tick without crushing
it.

Answer: D

Rationale: If a tick is found on the body, the tick should be carefully removed with
tweezers. To prevent the release of pathogenic organisms, care should be taken not
to crush the tick. A health care provider must be consulted if there is any suspicion
that a deer tick has bitten a child or an adult.
Question format: Multiple Choice
Chapter 41: The Child with an Integumentary Disorder/Communicable Disease
Cognitive Level: Apply
Client Needs: Health Promotion and Maintenance
Integrated Process: Nursing Process
Reference: p. 883

7. An adolescent girl and her caregiver present at the pediatrician's office. The
adolescent reports severe abdominal pain. A diagnosis of pelvic inflammatory
disease (PID) is made. The nurse notes in the child's chart that this is the third time
she has been treated for PID. Which action by the nurse would be most
appropriate?
A. Contact the necessary authorities to report a suspected case of sexual abuse.
B. Take the child to a private room and interview her regarding her sexual history
and partners.
C. Take the caregiver to a private room and tell her that the child's diagnosis can
only come from sexual activity.
D. Talk to the child and caregiver together and explain that the condition is often a
result of a sexually transmitted infection and discuss the importance of safe sex
practices.

Answer: B

Rationale: Adolescents must be made aware of the seriousness of PID, a common


result of a chlamydial infection. Pelvic inflammatory disease can cause sterility in
the female, primarily by causing scarring in the fallopian tubes that prohibits the
passage of the fertilized ovum into the uterus. A tubal pregnancy may be the
consequence of a chlamydial infection. In the male, sterility may result from
epididymitis caused by a chlamydial infection. All sexual partners must be treated.
Question format: Multiple Choice
Chapter 41: The Child with an Integumentary Disorder/Communicable Disease
Cognitive Level: Apply
Client Needs: Psychosocial Integrity
Integrated Process: Nursing Process
Reference: p. 892

8. The nurse instructor is reviewing the integumentary system during a


presentation to a group of student nurses. Which statement made by the instructor
is the most accurate regarding the integumentary system?
A. "The sebaceous and sweat glands are fully functional in the infant."
B. "The largest organ of the body helps regulate body temperature."
C. "The integumentary system is not in place until after the child is born and then
takes many years to mature."
D. "One role of the integumentary system is to distribute oxygen to the body cells."

Answer: B

Rationale: The skin is the largest organ of the body. The skin helps regulate the
body temperature by heating and cooling. The sebaceous and sweat glands are not
fully functional until middle childhood. The major role of the skin is to protect the
organs and structures of the body against bacteria, chemicals, and injury. Excretion
in the form of perspiration is also a function of the skin glands, called the sweat
glands. Sebaceous glands in the skin secrete oils to lubricate the skin and hair. The
integumentary system is in place at birth, but the system is immature. A function of
the respiratory system is to distribute oxygen to body cells.
Question format: Multiple Choice
Chapter 41: The Child with an Integumentary Disorder/Communicable Disease
Cognitive Level: Understand
Client Needs: Physiological Integrity: Physiological Adaptation
Integrated Process: Teaching/Learning
Reference: p. 873

9. The public health nurse is discussing immunizations with a group of caregivers of


infants. One of the mothers asks the nurse why the child will need immunizations.
Which statement would be the most appropriate for the nurse to make to this
mother?
A. "The infant is born with immunity to some diseases, but those immunities
decrease over the first year of life."
B. "The antibodies the fetus gets from the mother are in the placenta, so after birth
they are no longer available to the infant."
C. "The immunities that the infant is born with are not for the same diseases they
will be immunized against."
D. "Infants are unable to develop antibodies to protect them from diseases so they
must be immunized."

Answer: A

Rationale: During fetal life, the mother's antibodies cross the placenta, giving the
fetus a temporary immunity against certain diseases. This immunity is present at
birth and decreases during the first year of life. In the meantime, the infant begins
developing antibodies to fight against pathogens and disease. In addition, during
the first year of life immunizations are started to help the infant develop protection
against certain diseases.
Question format: Multiple Choice
Chapter 41: The Child with an Integumentary Disorder/Communicable Disease
Cognitive Level: Apply
Client Needs: Health Promotion and Maintenance
Integrated Process: Teaching/Learning
Reference: p. 873

10. The nurse is presenting an in-service to a group of nurses who will be working
in a dermatology clinic. They are discussing various disorders related to the
integumentary system. One of the nurses describes a disorder by telling the group
that it is common in children during the summer when it is hot or when the child is
overdressed. Which disorder is the nurse most likely referring to?
A. seborrheic dermatitis
B. miliaria rubra
C. candidiasis
D. impetigo

Answer: B

Rationale: Miliaria rubra, often called prickly heat, is common in children who are
exposed to summer heat or are overdressed. It also may appear in febrile illnesses
and may be mistaken for the rash of one of the communicable diseases.
Question format: Multiple Choice
Chapter 41: The Child with an Integumentary Disorder/Communicable Disease
Cognitive Level: Understand
Client Needs: Physiological Integrity: Physiological Adaptation
Integrated Process: Teaching/Learning
Reference: p. 873

11. When teaching a group of caregivers of infants, the nurse is discussing the topic
of diaper rash. The caregivers in the group make the following statements. Which
statement is the most accurate related to the child with diaper rash.
A. "The formula she drinks sometimes causes her to have a diaper rash."
B. "My child gets diaper rash if I wash her clothes in the same detergent we use for
the family."
C. "Sometimes if I leave the diaper off and let him be in the sunshine he gets
diaper rash."
D. "They told me to use baby powder every time I change her so she won't get
diaper rash."

Answer: B

Rationale: Infants may have a sensitivity to some soaps or disposable diaper


perfumes, and the use of strong laundry detergents without thorough rinsing can
cause diaper rash. Diarrheal stools, prolonged exposure to wet or soiled diapers,
use of plastic or rubber pants, infrequently changed disposable diapers, inadequate
cleansing of the diaper area (especially after bowel movements), and yeast
infections are also causative factors. Exposing the diaper area to the air helps clear
up the dermatitis. The use of baby powder when diapering is discouraged because
caked powder helps create an environment in which organisms thrive.
Question format: Multiple Choice
Chapter 41: The Child with an Integumentary Disorder/Communicable Disease
Cognitive Level: Understand
Client Needs: Health Promotion and Maintenance
Integrated Process: Teaching/Learning
Reference: p. 874

12. The nurse is collecting data on a 4-month-old child brought to the clinic by the
caregiver. The nurse observes the child's mouth and notes a white coating that
looks like milk curds. Which of the following questions would be most important for
the nurse to ask the caregiver?
A. "Did you have a vaginal delivery with this baby?"
B. "Has your child been treated for any type of infection recently?"
C. "Are you breastfeeding or bottle feeding your baby?"
D. "Does everyone in your house wash their hands often?"

Answer: B

Rationale: The symptoms the child has indicate the child most likely has thrush,
which is caused by Candida albicans. Asking about recent infections is most
important because episodes of thrush or diaper rash often appear after antibiotic
therapy. Antibiotic therapy can change the balance of normal intestinal flora,
leading to candidal overgrowth. Newborns can be exposed to a candidiasis vaginal
infection in the mother during delivery, but since the child is not a newborn, this
question would not be the most important. Poor handwashing practices and
inadequate washing of bottles and nipples are contributing factors.
Question format: Multiple Choice
Chapter 41: The Child with an Integumentary Disorder/Communicable Disease
Cognitive Level: Apply
Client Needs: Physiological Integrity: Reduction of Risk Potential
Integrated Process: Nursing Process
Reference: p. 874

13. The nurse is discussing medications to be given to a child who has been
diagnosed with oral candidiasis (thrush). Which medication would most likely be
prescribed for the child?
A. acetaminophen
B. ampicillin
C. aspirin
D. nystatin

Answer: D

Rationale: Application of nystatin to the oral lesions every 6 hours is an effective


treatment for oral candidiasis (thrush). Treatment for diaper rash caused by
Candida albicans is nystatin ointment or cream applied to the affected area.
Question format: Multiple Choice
Chapter 41: The Child with an Integumentary Disorder/Communicable Disease
Cognitive Level: Remember
Client Needs: Physiological Integrity: Pharmacological and Parenteral Therapies
Integrated Process: Nursing Process
Reference: p. 875

14. The nurse is presenting an in-service to a group of nurses who will be working
in a dermatology clinic. One participant asks the nurse about a bacterial skin
infection that she has seen in children that involves honey-colored crusted lesions.
The nurse most likely is referring to:
A. seborrheic dermatitis.
B. miliaria rubra (heat rash).
C. candidiasis.
D. impetigo.

Answer: D

Rationale: Impetigo is a superficial bacterial skin infection. Impetigo in the newborn


is usually bullous (blister-like, fluid filled); in the older child, the lesions are
nonbullous and have a honey-colored, crusted appearance.
Question format: Multiple Choice
Chapter 41: The Child with an Integumentary Disorder/Communicable Disease
Cognitive Level: Remember
Client Needs: Safe, Effective Care Environment: Safety and Infection Control
Client Needs: Physiological Integrity: Physiological Adaptation
Integrated Process: Nursing Process
Reference: p. 875

15. The nurse is caring for an infant who has impetigo and is hospitalized. Which
nursing action is priority?
A. The nurse soaks the skin with warm water.
B. The nurse applies topical antibiotics to the lesions.
C. The nurse follows contact precautions.
D. The nurse applies elbow restraints to the infant.

Answer: C

Rationale: Impetigo is highly contagious and can spread quickly. The nurse should
follow contact (skin and wound) precautions, including wearing a cover gown and
gloves. The nurse will soak the crusts with warm water, apply topical antibiotics,
and apply elbow restraints, but these are not as high a priority as trying to prevent
the spread of the infection by following contact precautions.
Question format: Multiple Choice
Chapter 41: The Child with an Integumentary Disorder/Communicable Disease
Cognitive Level: Understand
Client Needs: Safe, Effective Care Environment: Safety and Infection Control
Integrated Process: Nursing Process
Reference: p. 875
16. The nurse is discussing acne vulgaris with a group of adolescents. The
teenagers make the following statements regarding the topic. Which statement is
the most accurate regarding acne vulgaris?
A. "My mom says I have acne because I eat too much chocolate."
B. "Sometimes I get acne when I use my sister's makeup."
C. "My next door neighbor told me that acne was caused by a fungus."
D. "There is a new immunization that you can get to keep from having acne."

Answer: B

Rationale: Irritation and irritating substances, such as vigorous scrubbing and


cosmetics with a greasy base, can cause acne vulgaris. Increased hormone levels,
hereditary factors, and anaerobic bacteria can cause acne vulgaris as well. Eating
chocolate and fatty foods does not cause acne, but a well-balanced, nutritious diet
does promote healing.
Question format: Multiple Choice
Chapter 41: The Child with an Integumentary Disorder/Communicable Disease
Cognitive Level: Understand
Client Needs: Physiological Integrity: Physiological Adaptation
Integrated Process: Teaching/Learning
Reference: p. 876

17. The nurse is discussing skin disorders with a group of caregivers. Which
caregiver statement indicates an understanding of tinea capitis?
A. "That is an infection that you get under your fingernails."
B. "I always tell my daughter to use her own hairbrush."
C. "My son got that infection when he was at the swimming pool."
D. "My husband had that once and his groin itched so much."

Answer: B

Rationale: Ringworm of the scalp is called tinea capitis or tinea tonsurans. The most
common cause is infection with Microsporum audouinii, which is transmitted from
person to person through combs, towels, hats, barber scissors, or direct contact. A
less common type, Microsporum canis, is transmitted from animal to child.
Question format: Multiple Choice
Chapter 41: The Child with an Integumentary Disorder/Communicable Disease
Cognitive Level: Apply
Client Needs: Safe, Effective Care Environment: Safety and Infection Control
Integrated Process: Teaching/Learning
Reference: p. 876

18. Which of the following infections is often contracted by having contact with an
infected dog or cat?
A. tinea capitis
B. tinea corporis
C. tinea pedis
D. tinea cruris
Answer: B

Rationale: The child usually contracts tinea corporis from contact with an infected
dog or cat.
Question format: Multiple Choice
Chapter 41: The Child with an Integumentary Disorder/Communicable Disease
Cognitive Level: Remember
Client Needs: Physiological Integrity: Physiological Adaptation
Integrated Process: Nursing Process
Reference: p. 877

19. The nurse caring for children with fungal infections most often administers
which medication?
A. benadryl
B. acetaminophen
C. prednisone
D. griseofulvin

Answer: D

Rationale: Griseofulvin, an oral antifungal, is the medication of choice for fungal


infections. In some infections the treatment may be prolonged (3 months or more),
and compliance must be reinforced.
Question format: Multiple Choice
Chapter 41: The Child with an Integumentary Disorder/Communicable Disease
Cognitive Level: Remember
Client Needs: Physiological Integrity: Pharmacological and Parenteral Therapies
Integrated Process: Nursing Process
Reference: p. 877

20. The nurse is reinforcing teaching regarding the treatment for head lice with the
caregiver of a child who has the disorder. Which of the following is most accurate
related to the treatment for the child with an infection of head lice?
A. The treatment will be done every day for about 2 weeks.
B. The treatment will need to be repeated about once per month for a year.
C. The treatment will need to be repeated in about 7 to 9 days.
D. The treatment is done one time and won't need to be repeated.

Answer: C

Rationale: Nonprescription medications are available to treat cases of head lice. A


second treatment is suggested in 7 to 9 days to kill the nits after they have
hatched.
Question format: Multiple Choice
Chapter 41: The Child with an Integumentary Disorder/Communicable Disease
Cognitive Level: Apply
Client Needs: Physiological Integrity: Pharmacological and Parenteral Therapies
Integrated Process: Teaching/Learning
Reference: p. 877

21. The nurse is discussing allergies with a group of caregivers of children who have
food allergies. Which statement made by the caregivers indicates the most
concerning issue faced by the caregivers of children with food allergies?
A. "My daughter loves milk; it is so hard to tell her that she can't drink it."
B. "I read all the labels, but sometimes it is not easy to understand the
ingredients."
C. "When he goes to a friend's house I always have to pack a bag of snacks for
him."
D. "We have had to change all of the things we prepare for meals at our house."

Answer: B

Rationale: If a food allergy is suspected, an elimination diet may help identify the
allergen. Eliminating the food suspected is sometimes difficult because there are
often "hidden" ingredients in food products.
Question format: Multiple Choice
Chapter 41: The Child with an Integumentary Disorder/Communicable Disease
Cognitive Level: Apply
Client Needs: Physiological Integrity: Physiological Adaptation
Integrated Process: Teaching/Learning
Reference: p. 879

22. Which treatment will most likely be the best long-term benefit for the child with
an allergic condition in which it is difficult to remove or avoid the allergen?
A. use of corticosteroids
B. electrical stimulation
C. use of antihistamines
D. hyposensitization

Answer: D

Rationale: When specific allergens have been identified, clients can either avoid
them or, if this is impossible, undergo immunization therapy by injection. This
process is called hyposensitization or immunotherapy. Hyposensitization is
considered a safe procedure with considerable benefit for some children.
Symptomatic relief in allergic reactions can be gained through antihistamine or
steroid therapy, but the best treatment is prevention.
Question format: Multiple Choice
Chapter 41: The Child with an Integumentary Disorder/Communicable Disease
Cognitive Level: Understand
Client Needs: Physiological Integrity: Reduction of Risk Potential
Integrated Process: Nursing Process
Reference: p. 879
23. In working with infants diagnosed with atopic dermatitis, the nurse anticipates
that when these children are older they will likely have a tendency to have which
disorder?
A. hemophilia
B. asthma
C. rheumatoid arthritis
D. otitis media

Answer: B

Rationale: Infants who have atopic dermatitis (infantile eczema) tend to have
allergic rhinitis or asthma later in life.
Question format: Multiple Choice
Chapter 41: The Child with an Integumentary Disorder/Communicable Disease
Cognitive Level: Apply
Client Needs: Physiological Integrity: Physiological Adaptation
Integrated Process: Nursing Process
Reference: p. 879

24. A mother brings her 2-year-old son to the clinic with a bumpy rash. Which
statement by the mother would lead the nurse to suspect that the child may have
atopic dermatitis?
A. "I feel so bad for him because he has asthma, and now he gets this rash."
B. "It started as a flat red rash in between his legs."
C. "We just had him to the barber for his first haircut."
D. "He just started swim lessons at the YMCA."

Answer: A

Rationale: Children who have atopic dermatitis are at increased risk for also
developing allergic rhinitis (hay fever) and/or asthma. Diaper dermatitis does not
usually result in a bumpy rash, but starts as a flat red rash in the convex skin
creases. If the child has recently seen a barber he may have contracted tinea
capitis. Contact in damp areas such as locker rooms or a swimming pool may
indicate tinea pedis.
Question format: Multiple Choice
Chapter 41: The Child with an Integumentary Disorder/Communicable Disease
Cognitive Level: Understand
Client Needs: Physiological Integrity: Physiological Adaptation
Integrated Process: Nursing Process
Reference: p. 879

25. The nurse caring for a child with a skin allergy recognizes that the highest
priority in the treatment for skin allergies is aimed at:
A. regulating skin and body temperature.
B. controlling nausea and vomiting.
C. reducing swelling and relieving itching.
D. managing pain and discomfort.
Answer: C

Rationale: Treatment for skin allergies is aimed at reducing the swelling and
relieving the itching. With swelling in various parts of the body, the child may have
pain, nausea and vomiting, but reducing the swelling would prevent those
symptoms from occurring.
Question format: Multiple Choice
Chapter 41: The Child with an Integumentary Disorder/Communicable Disease
Cognitive Level: Apply
Client Needs: Physiological Integrity: Reduction of Risk Potential
Integrated Process: Nursing Process
Reference: p. 880

26. The camp nurse is caring for a child who was bitten on the leg by a dangerous
spider. The child is being taken to a health care provider. What is the most
appropriate action for the nurse to do with this child?
A. Administer pain medication.
B. Apply ice to the affected area.
C. Splint the leg.
D. Briskly scrub the site.

Answer: B

Rationale: Spider bites can cause serious illness if untreated. Bites of black widow
spiders, brown recluse spiders, and scorpions demand medical attention. Applying
ice to the affected area until medical care is obtained can slow absorption of the
poison.
Question format: Multiple Choice
Chapter 41: The Child with an Integumentary Disorder/Communicable Disease
Cognitive Level: Apply
Client Needs: Physiological Integrity: Physiological Adaptation
Integrated Process: Nursing Process
Reference: p. 883

27. The nurse is collecting data on a child admitted to the burn unit with a partial-
thickness or second-degree burn. What is most accurate regarding this type of
burn?
A. The nerve endings are destroyed.
B. The child will have minimal pain.
C. The child will likely have blisters.
D. There is no destruction of tissue.

Answer: C

Rationale: In a partial-thickness or second-degree burn, the epidermis and


underlying dermis are both injured and devitalized or destroyed. Blistering usually
occurs with an escape of body plasma, but regeneration of the skin occurs from the
remaining viable epithelial cells in the dermis.
Question format: Multiple Choice
Chapter 41: The Child with an Integumentary Disorder/Communicable Disease
Cognitive Level: Apply
Client Needs: Physiological Integrity: Physiological Adaptation
Integrated Process: Nursing Process
Reference: p. 884

28. A school-aged child is brought to the office of the camp nurse with a small,
superficial burn (first-degree burn). Which action by the nurse would be most
appropriate to take first?
A. Apply a topical anesthetic ointment.
B. Administer acetaminophen.
C. Cover the area with a sterile bandage.
D. Apply cold compresses to the area.

Answer: D

Rationale: Cool water is an excellent emergency treatment for burns involving small
areas. The immediate application of cool compresses or cool water to burn areas
appears to inhibit capillary permeability and thus suppress edema, blister
formation, and tissue destruction.
Question format: Multiple Choice
Chapter 41: The Child with an Integumentary Disorder/Communicable Disease
Cognitive Level: Understand
Client Needs: Physiological Integrity: Physiological Adaptation
Integrated Process: Nursing Process
Reference: p. 885

29. The nurse is collecting data on a 4-year-old child admitted to the burn unit. The
nurse is concerned about the possibility of the child going into hypovolemic shock.
Which of the following data would the nurse recognize as an indication that this
may be occurring?
A. The child is complaining of intense pain.
B. The child's blood pressure is 128/86.
C. The child's face is bright red in color.
D. The child's apical pulse is 140 bpm.

Answer: D

Rationale: Hypovolemic shock is the major manifestation in the first 48 hours in


massive burns. Symptoms of shock are low blood pressure, rapid pulse, pallor, and
often considerable apprehension. Intense pain is seldom a major factor.
Question format: Multiple Choice
Chapter 41: The Child with an Integumentary Disorder/Communicable Disease
Cognitive Level: Analyze
Client Needs: Physiological Integrity: Physiological Adaptation
Integrated Process: Nursing Process
Reference: p. 886

30. The nurse admits a child who has sustained a severe burn. The child's
immunizations are up to date. Which immunization would the child most likely be
given at this time?
A. hepatitis A vaccine
B. hepatitis B vaccine
C. tetanus toxoid vaccine
D. Haemophilus influenzae type B vaccine

Answer: C

Rationale: If inoculations are up to date, a booster dose of tetanus toxoid is


required to protect the child from infection introduced into the burn.
Question format: Multiple Choice
Chapter 41: The Child with an Integumentary Disorder/Communicable Disease
Cognitive Level: Apply
Client Needs: Safe, Effective Care Environment: Safety and Infection Control
Integrated Process: Nursing Process
Reference: p. 886

31. The nurse is caring for an adolescent diagnosed with genital herpes. The drug
of choice for treating genital herpes is:
A. griseofluvin
B. ceftriaxone
C. acyclovir
D. penicillin

Answer: C

Rationale: The drug acyclovir is useful in relieving or suppressing the symptoms of


genital herpes.
Question format: Multiple Choice
Chapter 41: The Child with an Integumentary Disorder/Communicable Disease
Cognitive Level: Remember
Client Needs: Physiological Integrity: Pharmacological and Parenteral Therapies
Integrated Process: Nursing Process
Reference: p. 893

32. The nurse is caring for an adolescent diagnosed with syphilis. The drug of
choice for treating syphilis is:
A. griseofluvin
B. ceftriaxone
C. acyclovir
D. penicillin

Answer: D
Rationale: Syphilis responds to one intramuscular injection of penicillin G
benzathine; if the child is sensitive to penicillin, oral doxycycline, tetracycline, or
erythromycin can be administered as alternative treatment.
Question format: Multiple Choice
Chapter 41: The Child with an Integumentary Disorder/Communicable Disease
Cognitive Level: Remember
Client Needs: Physiological Integrity: Pharmacological and Parenteral Therapies
Integrated Process: Nursing Process
Reference: p. 893

33. The nurse is caring for a 16-year-old child with a diagnosis of acquired
immunodeficiency syndrome (AIDS). What treatment goal has the highest priority
for this child?
A. preventing spread of infection
B. promoting comfort
C. maintaining skin integrity
D. improving nutrition

Answer: A

Rationale: Major goals for the child include maintaining the highest level of wellness
possible by preventing infection and the spread of the infection. Because the
adolescent has the belief that nothing can hurt him or her, and because of the
increasing rate of sexual activity in this age group often involving multiple partners,
the highest priority is teaching and preventing the spread of the infection. Other
goals include maintaining skin integrity, minimizing pain, improving nutrition,
alleviating social isolation, and diminishing a feeling of hopelessness. The primary
goal for the family is improving coping skills and helping the teen cope with the
illness.
Question format: Multiple Choice
Chapter 41: The Child with an Integumentary Disorder/Communicable Disease
Cognitive Level: Apply
Client Needs: Safe, Effective Care Environment: Management of Care
Integrated Process: Nursing Process
Reference: p. 893

34. Infectious mononucleosis ("mono") is caused by which of the following?


A. Treponema pallidum
B. Epstein-Barr virus
C. Microsporum canis
D. Streptococcal bacterium

Answer: B

Rationale: Infectious mononucleosis ("mono") is caused by the Epstein-Barr virus,


one of the herpes virus groups. The organism is transmitted through saliva.
Question format: Multiple Choice
Chapter 41: The Child with an Integumentary Disorder/Communicable Disease
Cognitive Level: Remember
Client Needs: Physiological Integrity: Physiological Adaptation
Integrated Process: Nursing Process
Reference: p. 897

35. The nurse is doing an in-service training on clinical manifestations seen in


communicable diseases. Which skin condition best describes erythema?
A. redness of the skin produced by congestion of the capillaries
B. small, circumscribed, solid elevation of the skin
C. discolored skin spot not elevated at the surface
D. small elevation of epidermis filled with a viscous fluid

Answer: A

Rationale: Erythema is redness of the skin produced by congestion of the


capillaries.
Question format: Multiple Choice
Chapter 41: The Child with an Integumentary Disorder/Communicable Disease
Cognitive Level: Remember
Client Needs: Physiological Integrity: Physiological Adaptation
Integrated Process: Nursing Process
Reference: p. 903

36. The nurse is doing an in-service training on clinical manifestations seen in


communicable diseases. Which skin condition best describes pustule?
A. redness of the skin produced by congestion of the capillaries
B. small, circumscribed, solid elevation of the skin
C. discolored skin spot not elevated at the surface
D. small elevation of epidermis filled with a viscous fluid

Answer: D

Rationale: A pustule is a small elevation of epidermis filled with pus.


Question format: Multiple Choice
Chapter 41: The Child with an Integumentary Disorder/Communicable Disease
Cognitive Level: Remember
Client Needs: Physiological Integrity: Physiological Adaptation
Integrated Process: Nursing Process
Reference: p. 903

37. The nurse is doing an in-service training on clinical manifestations seen in


communicable diseases. Which best describes a macule?
A. redness of the skin produced by congestion of the capillaries
B. small, circumscribed, solid elevation of the skin
C. discolored skin spot not elevated at the surface
D. small elevation of epidermis filled with a viscous fluid
Answer: C

Rationale: A macule is a discolored skin spot not elevated above the surface.
Question format: Multiple Choice
Chapter 41: The Child with an Integumentary Disorder/Communicable Disease
Cognitive Level: Remember
Client Needs: Physiological Integrity: Physiological Adaptation
Integrated Process: Nursing Process
Reference: p. 903

38. A nurse is promoting vaccine administration. When instructing on the


physiological changes, which statement best explains what occurs in the child when
vaccines are administered?
A. The child develops a passive immunity.
B. The child becomes a carrier of the disease.
C. The child develops an active immunity.
D. The child becomes a host for the disease.

Answer: C

Rationale: When a vaccine is given, active immunity occurs which then stimulates
the development of antibodies to destroy infective agents without causing the
disease.
Question format: Multiple Choice
Chapter 41: The Child with an Integumentary Disorder/Communicable Disease
Cognitive Level: Apply
Client Needs: Health Promotion and Maintenance
Integrated Process: Nursing Process
Reference: p. 903

39. The nurse is caring for a child hospitalized with pertussis. Which nursing
intervention would be the highest priority for this child?
A. The nurse will administer oxygen.
B. The nurse will encourage bed rest.
C. The nurse will administer antibiotics.
D. The nurse will monitor caloric intake.

Answer: A

Rationale: The major complication of pertussis (whooping cough) is pneumonia and


respiratory complications. Oxygen, bed rest, and monitoring for airway obstruction
are nursing interventions. The highest priority is administering oxygen to maintain
adequate oxygenation of cells.
Question format: Multiple Choice
Chapter 41: The Child with an Integumentary Disorder/Communicable Disease
Cognitive Level: Apply
Client Needs: Safe, Effective Care Environment: Management of Care
Integrated Process: Nursing Process
Reference: p. 899

40. The nurse is caring for a child diagnosed with Haemophilus Influenzae type B.
Which of the following nursing interventions would be the highest priority in the
prevention of complications seen with this disorder?
A. The nurse will monitor for seizure activity.
B. The nurse will elevate the head of the bed.
C. The nurse will administer antibiotics.
D. The nurse will monitor fluid intake.

Answer: C

Rationale: A serious complication of Haemophilus Influenzae type B is meningitis.


Antibiotics are administered to treat the bacterium infection caused by Haemophilus
Influenzae type B.
Question format: Multiple Choice
Chapter 41: The Child with an Integumentary Disorder/Communicable Disease
Cognitive Level: Understand
Client Needs: Physiological Integrity: Pharmacological and Parenteral Therapies
Integrated Process: Nursing Process
Reference: p. 900

41. The school nurse is teaching a group of teachers about various skin disorders
seen in children. The nurse would identify the most obvious symptom of pediculosis
as:
A. scaly rings with clear centers located on the scalp.
B. white flecks that are firmly attached to the hair shafts.
C. small papule on the scalp.
D. scaling or cracking of the skin on the scalp.

Answer: B

Rationale: The nits can be seen as tiny, pearly white flecks attached to the hair
shafts. They look much like dandruff, but dandruff flakes can be flicked off easily,
whereas the nits are tightly attached and not easily removed. Tinea capitis begins
as a small papule on the scalp. Tinea corporis is ringworm that appear as a scaly
ring with clearing in the center. Tinea pedis is evidenced by the scaling or cracking
of the skin.
Question format: Multiple Choice
Chapter 41: The Child with an Integumentary Disorder/Communicable Disease
Cognitive Level: Understand
Client Needs: Physiological Integrity: Physiological Adaptation
Integrated Process: Teaching/Learning
Reference: p. 877

42. A 17-year-old is diagnosed with infectious mononucleosis. The nurse should


discuss which intervention with the teenager's caregiver to best assure an
uncomplicated recovery?
A. admission to the hospital for about 7 days
B. a 10-day course of antibiotics
C. a high-protein, high-fiber, low-fat diet
D. precautions to avoid secondary infections

Answer: D

Rationale: No cure exists for infectious mononucleosis; treatment is based on


symptoms. An analgesic-antipyretic, such as acetaminophen, usually is
recommended for fever and headaches. Fluids and a soft, bland diet are
encouraged to reduce throat irritation. Because the immune system is weakened,
the child must take precautions to avoid secondary infections.
Question format: Multiple Choice
Chapter 41: The Child with an Integumentary Disorder/Communicable Disease
Cognitive Level: Apply
Client Needs: Physiological Integrity: Physiological Adaptation
Integrated Process: Teaching/Learning
Reference: p. 897

43. The nurse is assessing a child who presents with a history of fever, malaise,
fatigue, and headache. The nurse notes a bulls-eye rash on the child's right leg.
Which action will the nurse take?
A. Notify the primary health care provider.
B. Place the child on contact precautions.
C. Obtain an electrocardiography (ECG).
D. Clean the rash with rubbing alcohol.

Answer: A

Rationale: The nurse would suspect the child has Lyme disease and notify the
health care provider for additional testing and potential antibiotic therapy.
Precautions are not indicated for clients with Lyme disease. An ECG would only be
needed if cardiac symptoms were noted. It is recommended to clean the site of the
tick bite with rubbing alcohol when the tick is removed, not at a later time.
Question format: Multiple Choice
Chapter 41: The Child with an Integumentary Disorder/Communicable Disease
Cognitive Level: Apply
Client Needs: Physiological Integrity: Physiological Adaptation
Integrated Process: Nursing Process
Reference: p. 902

44. The nurse is caring for a 9-year-old child with partial-thickness (second-degree)
burns. The client rates the pain at an 8 on a 1 to 10 numerical pain scale. The
nurse notes the client is sitting in the bed playing with toys and smiling. Which
action will the nurse take?
A. Administer pain medication as prescribed.
B. Reassess the client's pain in 30 minutes.
C. Ask the parents to rate the client's pain.
D. Use another pain scale to measure the client's pain.

Answer: A

Rationale: The nurse would administer the client pain medication as prescribed.
Pain is how a client describes and rates it, regardless of outward appearances. The
nurse should not allow the child to continue to feel pain while waiting an additional
30 minutes, asking the parents their opinion, or using a different pain scale.
Question format: Multiple Choice
Chapter 41: The Child with an Integumentary Disorder/Communicable Disease
Cognitive Level: Apply
Client Needs: Physiological Integrity: Basic Care and Comfort
Integrated Process: Nursing Process
Reference: p. 884

45. A 6-year-old child is diagnosed with tinea pedis. Which prescription will the
nurse question?
A. luliconazole cream daily for 2 weeks
B. The child may return to school in 1 week.
C. Cleanse the skin with antibacterial soap.
D. Perform warm foot soaks daily.

Answer: B

Rationale: Tinea pedis refers to a fungal infection that typically begins between the
toes. The nurse would question the child being out of school for a week. While
these infections are highly contagious, children can return to school once treatment
is started. Tinea pedis can be treated with topical or oral antifungals or a
combination of both. Topical agents, such as luliconazole, are used for 1 to 6
weeks, depending on the brand. Antibacterial soaps help reduce the risk of infection
to the affected area. Warm soaks may help soothe painful muscles or joints and can
help drain skin infections, if present.
Question format: Multiple Choice
Chapter 41: The Child with an Integumentary Disorder/Communicable Disease
Cognitive Level: Apply
Client Needs: Safe, Effective Care Environment: Safety and Infection Control
Integrated Process: Nursing Process
Reference: p. 877

46. The nurse is working with the caregiver of a neonate. The caregiver states, "My
first baby had a disorder they called seborrheic dermatitis; what can I do to prevent
this baby from having that?" Which response is the most appropriate?
A. "There is nothing you can do to prevent this disorder; it will go away by the time
your neonate is a toddler."
B. "Be careful to not get any lotion or oil on the neonate's head, and that will help
prevent the disorder."
C. "You should wash your neonate's hair every day to help prevent this disorder."
D. "If you will keep hats and bonnets off your neonate's head, you will not have to
worry about the neonate getting the disorder."

Answer: C

Rationale: Seborrheic dermatitis is commonly known as cradle cap. Daily washing


of the neonate's hair and scalp can help prevent it.
Question format: Multiple Choice
Chapter 41: The Child with an Integumentary Disorder/Communicable Disease
Cognitive Level: Apply
Client Needs: Health Promotion and Maintenance
Integrated Process: Teaching/Learning
Reference: p. 873

47. The nurse is monitoring the urinary output of 3-year-old child admitted with a
severe burn. The child weighs 44 lb (20 kg). Which would be a desirable and
adequate urinary output for this child?
A. 1 to 2 mL/kg/hr
B. 10 to 12 mL/kg/hr
C. 15 to 25 mL/kg/hr
D. 30 to 50 mL/hr

Answer: A

Rationale: Following a burn injury, an output of 1 to 2 mL/kg/hr for children


weighing 30 kg (66 lb) or less, or 30 to 50 mL/hr for those weighing more than 30
kg is desirable. This child weighs 20 kg (44 lb).
Question format: Multiple Choice
Chapter 41: The Child with an Integumentary Disorder/Communicable Disease
Cognitive Level: Apply
Client Needs: Physiological Integrity: Physiological Adaptation
Integrated Process: Nursing Process
Reference: p. 886
Chapter 42
1. The nurse is developing a plan of care for the child hospitalized with nonorganic
failure to thrive (NFTT). The following nursing interventions would be included in
this plan. Which nursing intervention would be the highest priority for this child?
A. The nurse should monitor fluid intake and urinary output.
B. The nurse should monitor bowel elimination.
C. The nurse should lubricate the skin to keep it moisturized and intact.
D. The nurse should teach and support the caregiver.

Answer: D

Rationale: The family relationships of children with NFTT are often so disrupted that
the child has no warm, close relationship with a family caregiver. For some reason,
proper attachment has not occurred. The child cannot accomplish the
developmental task of establishing basic trust. Treatment initially depends almost
entirely on good nursing care. By teaching child care skills, acting as a role model,
and supporting caregiver-child interactions, the nurse can help to both reverse the
child's growth failure as well as begin an improved caregiver-child relationship.
Question format: Multiple Choice
Chapter 42: The Child with a Psychosocial Disorder
Cognitive Level: Understand
Client Needs: Psychosocial Integrity
Integrated Process: Nursing Process
Reference: p. 917

2. The nurse has gathered the following data on four different children. Which child
will likely be further assessed and tested for autism spectrum disorder?
A. a 16-month-old who spends 30 minutes moving empty plastic bottles in and out
of a container
B. an 18-month-old who enjoys spinning until she falls, then announces, "Me fall!"
C. a 14-month-old whose reaction to his caregiver leaving him is to calmly repeat
the statement, "Bye bye, Will. Bye bye, Will."
D. a 20-month-old who frequently walks on tiptoes and smiles at her caregiver
when the caregiver looks at the child

Answer: C

Rationale: Children with autism spectrum disorder do not show the normal fear of
separation from parents that most toddlers exhibit. Echolalia ("parrot speech") is
typical of autistic children; they echo words they have heard, such as a television
commercial, but offer no indication that they understand the words. Although
autistic children are self-centered, their speech indicates that they seem to have no
sense of self, because they never use the pronouns "I" or "me." Children with
autism do not develop a smiling response to others nor an interest in being touched
or cuddled. During their second year, autistic children become completely absorbed
in strange repetitive behaviors such as spinning an object, flipping an electrical
switch on and off, or walking around the room feeling the walls. Their bodily
movements are as follows: rocking, twirling, flapping arms and hands, walking on
tiptoes, twisting and turning fingers. Although infants and toddlers normally are
self-centered, ritualistic, and prone to displays of temper, autistic children show
these characteristics to an extreme degree, coupled with an almost total lack of
response to other people.
Question format: Multiple Choice
Chapter 42: The Child with a Psychosocial Disorder
Cognitive Level: Analyze
Client Needs: Psychosocial Integrity
Integrated Process: Nursing Process
Reference: p. 908

3. When weighing and measuring an adolescent whose ideal body weight is 105
pounds, the nurse would identify the child as obese if the child weighs which of the
following?
A. 116 pounds
B. 121 pounds
C. 126 pounds
D. 131 pounds

Answer: C

Rationale: Obesity generally is defined as an excessive accumulation of fat that


increases body weight by 20 percent or more over ideal weight. Overweight,
although not necessarily signifying obesity, means that a person's weight is more
than average for height and body build.
Question format: Multiple Choice
Chapter 42: The Child with a Psychosocial Disorder
Cognitive Level: Analyze
Client Needs: Health Promotion and Maintenance
Integrated Process: Nursing Process
Reference: p. 921

4. A 16-year-old girl is being seen for a long-overdue checkup. Her caregiver has
come with her. She is calm, pleasant, and in good spirits. The caregiver reports to
the nurse that she is relieved because for the past 6 months the teenager has been
lethargic, angry, and sad. The mother reports that since she got her driver's license
two days earlier, her child's mood has changed dramatically. Rather than resist this
appointment, the girl had simply smiled and said, "It won't matter much, but okay,
I'll be ready in a minute." The nurse recognizes that the child's seeming well-being
and drastic change in behavior should be further investigated to determine if the
child:
A. is planning to commit suicide.
B. has been smoking marijuana.
C. is experimenting with alcohol.
D. is excited that she can drive now.
Answer: A

Rationale: Attempted suicide rarely occurs without warning and usually is preceded
by a long history of emotional problems, difficulty forming relationships, feelings of
rejection, and low self-esteem. Suicidal adolescents may appear suddenly elated
after a long period of acting dejected, and might verbalize their hopelessness with
statements such as "I won't be around much longer," or "After Monday, it won't
matter anyhow." Some deaths reported as accidents, particularly one-car accidents,
are thought to be suicides.
Question format: Multiple Choice
Chapter 42: The Child with a Psychosocial Disorder
Cognitive Level: Analyze
Client Needs: Psychosocial Integrity
Integrated Process: Nursing Process
Reference: p. 911

5. The nurse in the well-child clinic observes that a 5-year-old child in the waiting
room is having trouble using a crayon to color. During the visit, the same child
climbs off the table several times even after the nurse has asked him to stay on the
table. Each time the nurse reminds him he says, "Oh, yeah," and happily climbs
back up. The nurse suspects that the child has:
A. attention deficit hyperactive disorder (ADHD).
B. autism spectrum disorder.
C. failure to thrive.
D. an addicted caregiver.

Answer: A

Rationale: The child with ADHD may have these characteristics: Impulsiveness,
easy distractibility, frequent fidgeting or squirming, difficulty sitting still, problems
following through on instructions despite being able to understand them,
inattentiveness when being spoken to, frequent losing of things, going from one
uncompleted activity to another, difficulty taking turns, frequent excessive talking,
and engaging in dangerous activities without considering the consequences.
Question format: Multiple Choice
Chapter 42: The Child with a Psychosocial Disorder
Cognitive Level: Apply
Client Needs: Psychosocial Integrity
Integrated Process: Nursing Process
Reference: p. 909

6. The nurse is collecting data on an 18-month-old old child with a diagnosis of


autism spectrum disorder (ASD). What clinical manifestation would likely have been
noted in the child with this diagnosis?
A. The child cries and runs to the door when the caregiver leaves the room.
B. The child smiles when the caregiver shows her a stuffed animal.
C. The child sits quietly in the caregiver's lap during the interview.
D. The child does not make eye contact.

Answer: D

Rationale: Children with ASD often display hyperactivity, aggression, temper


tantrums, or self-injury behaviors like head banging or hand-biting. They may resist
cuddling, lack eye contact, be indifferent to touch or affection, and have little
change in facial expression. They do not develop a smiling response to others nor
an interest in being touched or cuddled. In fact, they can react violently to attempts
to hold them. They do not show the normal fear of separation from parents that
most toddlers exhibit. Often they seem not to notice when family caregivers are
present.
Question format: Multiple Choice
Chapter 42: The Child with a Psychosocial Disorder
Cognitive Level: Apply
Client Needs: Psychosocial Integrity
Integrated Process: Nursing Process
Reference: p. 908

7. The nurse is observing a group of 2- and 3-year-olds in a play group. Which


behavior noted in one of the children indicates to the nurse that the child may have
autism spectrum disorder (ASD)?
A. After another child takes a toy, the child cries and stomps his feet.
B. A child playing in the kitchen area pretends to pour a glass of milk and repeats
this over and over.
C. While the other children are eating a snack, the child walks around the room
feeling the walls and ignores the caregiver who offers him a snack.
D. A child flips the light switch off and on until the caregiver asks her to stop and
join the other children in playing.

Answer: C

Rationale: Children with ASD become completely absorbed in strange repetitive


behaviors such as spinning an object, flipping an electrical switch on and off, or
walking around the room feeling the walls. If these movements are interrupted or if
objects in the environment are moved, a violent temper tantrum may result. These
tantrums may include self-destructive acts such as hand biting and head banging.
Although infants and toddlers normally are self-centered, ritualistic, and prone to
displays of temper, autistic children show these characteristics to an extreme
degree coupled with an almost total lack of response to other people.
Question format: Multiple Choice
Chapter 42: The Child with a Psychosocial Disorder
Cognitive Level: Analyze
Client Needs: Psychosocial Integrity
Integrated Process: Nursing Process
Reference: p. 908
8. The nurse is working with a group caregivers of children who are diagnosed with
autism. Which statement made by the parents shows an example of echolalia?
A. "When she watches TV and hears a commercial, she repeats one word from the
commercial but doesn't seem to understand what she is saying."
B. "When we were in a room with a high ceiling, she was so excited when she said
hello and she heard her voice that she said it over and over."
C. "She was at the park and the older kids were playing a game and she was
staring at them as if she was wanted to try to copy what they were doing."
D. "When reading her a story book about owls, her sister kept telling her that when
owls made noises they sounded like an echo. She seemed to understand that
animals sounded different from people."

Answer: A

Rationale: Echolalia ("parrot speech") is typical of autistic children; they echo words
they have heard, such as from a television commercial, but they offer no indication
that they understand the words.
Question format: Multiple Choice
Chapter 42: The Child with a Psychosocial Disorder
Cognitive Level: Apply
Client Needs: Psychosocial Integrity
Integrated Process: Nursing Process
Reference: p. 908

9. The nurse is working with a group of caregivers of school-aged children


diagnosed with attention deficit hyperactivity disorder (ADHD). Which statement
would be most appropriate for the nurse to make to this group of caregivers?
A. "These children study better with quiet background music such as the radio or a
CD."
B. "The medications your child is on may cause a decreased appetite."
C. "These children function best if given a set of instructions and then left to do the
task."
D. "A frequent change in routine will be helpful so the child does not get bored."

Answer: B

Rationale: Learning situations should be structured so that the child has minimal
distractions. Structured, consistent guidance from the caregivers is needed.
Medication is used for some children, and these medications may suppress the
appetite and affect the child's growth. The child should be given only one simple
instruction at a time. Limiting distractions, using consistency, and offering praise for
accomplishments are invaluable.
Question format: Multiple Choice
Chapter 42: The Child with a Psychosocial Disorder
Cognitive Level: Apply
Client Needs: Psychosocial Integrity
Integrated Process: Nursing Process
Reference: p. 909
10. The nurse is working with school-aged children who are having enuresis or
encopresis. What will most likely be the first step in this child's treatment?
A. The child will be given medications.
B. The child will be taken to a therapist.
C. The child will have a complete physical exam.
D. The child will be given a strict daily schedule.

Answer: C

Rationale: The child with enuresis or encopresis may have a physiologic or


psychological cause and may indicate a need for further exploration and treatment.
A complete physical exam and assessment is done first to rule out any physical
cause.
Question format: Multiple Choice
Chapter 42: The Child with a Psychosocial Disorder
Cognitive Level: Apply
Client Needs: Physiological Integrity: Basic Care and Comfort
Integrated Process: Nursing Process
Reference: p. 910

11. The nurse is doing in-service training with nurses who will be working with
children who have concerns with drugs and alcohol. In discussing this topic, one of
the nurses says she has heard that there may be physical and psychological signs
that occur when a drug is no longer being used. Which term best describes what
the nurse is referring to?
A. withdrawal symptoms
B. dependence
C. tolerance
D. substance abuse

Answer: A

Rationale: Withdrawal symptoms, such as vomiting, chills, tremors, and


hallucinations, are the physical and psychological symptoms that occur when the
drug is no longer being used. The magnitude of physical dependence determines
the severity of withdrawal symptoms.
Question format: Multiple Choice
Chapter 42: The Child with a Psychosocial Disorder
Cognitive Level: Remember
Client Needs: Psychosocial Integrity
Integrated Process: Nursing Process
Reference: p. 913

12. The ability of body tissues to endure and adapt to continued or increased use of
a substance, so that the drug user requires larger doses of the drug to produce the
desired effect, is called:
A. withdrawal symptoms.
B. dependence.
C. tolerance.
D. substance abuse.

Answer: C

Rationale: Continued use of an addictive substance can result in tolerance (the


ability of body tissues to endure and adapt to continued or increased use of a
substance); this dynamic means the drug user requires larger doses of the drug to
produce the desired effect.
Question format: Multiple Choice
Chapter 42: The Child with a Psychosocial Disorder
Cognitive Level: Remember
Client Needs: Psychosocial Integrity
Integrated Process: Nursing Process
Reference: p. 913

13. The most common use disorder among children and adolescents is:
A. marijuana.
B. oxycodone.
C. alcohol.
D. ecstasy.

Answer: C

Rationale: Alcohol misuse occurs when a person ingests a quantity sufficient to


cause intoxication. It is also the most common substance use disorder among
children and adolescents.
Question format: Multiple Choice
Chapter 42: The Child with a Psychosocial Disorder
Cognitive Level: Remember
Client Needs: Psychosocial Integrity
Integrated Process: Nursing Process
Reference: p. 914

14. The nurse is discussing the use of alcohol with a group of adolescents. One of
the teens says that she heard your fine motor control could be affected at low
levels of alcohol. Another teen was correct in saying that most states consider a
person under the influence of alcohol if the blood alcohol level is which of the
following?
A. 0.03 %
B. 0.05 %
C. 0.06 %
D. 0.08 %

Answer: D
Rationale: Most states determine charges of driving under the influence using a
standard of 0.1% blood alcohol content. However, many states have lowered the
limit to 0.08%. Many children do not realize that fine motor control and judgment
are affected at even lower levels, and driving ability may be decreased.
Question format: Multiple Choice
Chapter 42: The Child with a Psychosocial Disorder
Cognitive Level: Remember
Client Needs: Psychosocial Integrity
Integrated Process: Teaching/Learning
Reference: p. 915

15. The nurse is working with a group of caregivers of adolescents who have
abused substances. One of the caregivers states, "My son used to do great in
school. However, since he started using drugs he has no motivation, he is never on
time, and he makes the worst decisions, especially when he is driving." These
behaviors indicate that this adolescent was likely using which substance?
A. amphetamines
B. anabolic steroids
C. cocaine
D. marijuana

Answer: D

Rationale: Marijuana affects judgment, sense of time, and motivation. These effects
make driving hazardous and may even cause hallucinations at higher doses.
Question format: Multiple Choice
Chapter 42: The Child with a Psychosocial Disorder
Cognitive Level: Apply
Client Needs: Psychosocial Integrity
Integrated Process: Nursing Process
Reference: p. 915

16. Adolescent athletes sometimes take megadoses of which of the following


substances because they believe it will increase their athletic abilities?
A. amphetamines
B. anabolic steroids
C. cocaine
D. hallucinogens

Answer: B

Rationale: Adolescent athletes take anabolic steroids to build up muscle mass in the
belief that the drug will increase their athletic ability. These athletes take
megadoses of illegally obtained drugs.
Question format: Multiple Choice
Chapter 42: The Child with a Psychosocial Disorder
Cognitive Level: Understand
Client Needs: Psychosocial Integrity
Integrated Process: Nursing Process
Reference: p. 916

17. The nurse is collecting data on an adolescent who is brought to the emergency
center by her caregivers, who suspect that she has been using substances. The
adolescent states that she has not had anything to eat nor any sleep in several
days, but that she is nauseated. The nurse observes that she is diaphoretic,
clenching her teeth, and complaining of blurred vision. Which substance would this
adolescent most likely have been using?
A. cocaine
B. alcohol
C. hallucinogens
D. ecstasy

Answer: D

Rationale: Ecstasy suppresses the need to eat, drink, or sleep. The drug can cause
increased heart rate and blood pressure, muscle tension, teeth clenching, nausea,
blurred vision, chills, and sweating.
Question format: Multiple Choice
Chapter 42: The Child with a Psychosocial Disorder
Cognitive Level: Analyze
Client Needs: Psychosocial Integrity
Integrated Process: Nursing Process
Reference: p. 916

18. The nurse is caring for a 6-month-old infant diagnosed with nonorganic failure
to thrive. Which of the following would the nurse most likely have noted with this
child?
A. The child's mother is attentive, but the father appears uninterested in the child.
B. The child's weight and height are above average according to the growth charts.
C. The child smiles and laughs when the nurse picks up and holds the child.
D. The child plays and sleeps quietly for long periods of time without fussing.

Answer: D

Rationale: Infants with NFTT often are listless, well below average weight and
height, have poor muscle tone and a loss of subcutaneous fat, and are immobile for
long periods of time. They may be unresponsive to (or actually try to avoid)
cuddling and vocalization. Family relationships, particularly the mother-child
relationship, often provide important insights into the problem.
Question format: Multiple Choice
Chapter 42: The Child with a Psychosocial Disorder
Cognitive Level: Apply
Client Needs: Psychosocial Integrity
Integrated Process: Nursing Process
Reference: p. 917
19. The nurse is collecting data for a child diagnosed with nonorganic failure to
thrive, and notes that the child voluntarily vomits after eating. This is referred to
as:
A. rumination.
B. tolerance.
C. self-inflicted starvation.
D. encopresis.

Answer: A

Rationale: A common characteristic noted in the child with nonorganic failure to


thrive is rumination (voluntary regurgitation), perhaps as a means of self-
satisfaction when the desired response is not received from the caregiver.
Question format: Multiple Choice
Chapter 42: The Child with a Psychosocial Disorder
Cognitive Level: Remember
Client Needs: Psychosocial Integrity
Integrated Process: Nursing Process
Reference: p. 917

20. The nurse is doing a presentation on the diagnosis of bulimia to a group of


pediatric nurses. Which clinical manifestation would the nurse teach this group to
observe for in the adolescent with bulimia?
A. dry skin and brittle nails
B. constipation
C. soft hair on their backs and arms
D. hypokalemia

Answer: D

Rationale: In the child with bulimia, hypokalemia may occur, especially if the child
abuses diuretics to prevent weight gain.
Question format: Multiple Choice
Chapter 42: The Child with a Psychosocial Disorder
Cognitive Level: Apply
Client Needs: Psychosocial Integrity
Integrated Process: Nursing Process
Reference: p. 918

21. Which characteristic is commonly noted in the child with anorexia nervosa?
A. The child is impulsive and inattentive when spoken to.
B. The child has rigid study skills and ritualistic behavior.
C. The child has trouble sitting still and is fidgety.
D. The child is inactive and participates in sedentary activities.

Answer: B
Rationale: Anorexic children often are described as successful students who tend to
be perfectionists and are always trying to please parents, teachers, and other
adults. They may make demands on themselves for cleanliness and order in their
environment, or they may engage in rigid schedules for studying and other
ritualistic behavior.
Question format: Multiple Choice
Chapter 42: The Child with a Psychosocial Disorder
Cognitive Level: Understand
Client Needs: Psychosocial Integrity
Integrated Process: Nursing Process
Reference: p. 917

22. The nurse is collecting data on a 16-year-old girl with the diagnosis of bulimia.
What would the nurse most likely note in this child?
A. The child is of normal weight for her height according to the growth charts.
B. The child socializes with friends and shares all her dreams and secrets with
them.
C. The child has a ritualistic program of exercise that she does every day after
school.
D. The child is a perfectionist and tries hard to please her parents and teachers.

Answer: A

Rationale: The typical bulimic person is a white female in late adolescence. Most
often, the bulimic person is of normal weight or is slightly overweight.
Question format: Multiple Choice
Chapter 42: The Child with a Psychosocial Disorder
Cognitive Level: Understand
Client Needs: Psychosocial Integrity
Integrated Process: Nursing Process
Reference: p. 918

23. The nurse is discussing health concerns and issues with a group of caregivers of
adolescents. Which statement made by a caregiver would indicate a need for
follow-up by the nurse?
A. "My 16-year-old spends so much time with his girlfriend, it worries me."
B. "I don't know if I want my 15-year-old son to drive; I know that at that age
teens sometimes don't use good judgement."
C. "Our daughter has never had any health problems, and now she has developed
so many cavities."
D. "My daughter is already 15 and she hasn't ever had a menstrual period."

Answer: C

Rationale: The clues to bulimia nervosa may be few but include dental caries and
erosion from frequent exposure to stomach acid, throat irritation, and endocrine
and electrolyte imbalances.
Question format: Multiple Choice
Chapter 42: The Child with a Psychosocial Disorder
Cognitive Level: Analyze
Client Needs: Psychosocial Integrity
Integrated Process: Nursing Process
Reference: p. 918

24. The nurse is caring for a child with an eating disorder. Which is the priority
treatment for the disorder?
A. Develop a behavior modification plan
B. Correct fluid and electrolyte imbalances
C. Supervise and document food intake
D. Monitor urine for presence of ketones

Answer: B

Rationale: If the child's condition is at a critical stage with fluid and electrolyte
deficiencies, parenteral fluids should be administered immediately to hydrate the
client before additional treatment can be implemented.
Question format: Multiple Choice
Chapter 42: The Child with a Psychosocial Disorder
Cognitive Level: Apply
Client Needs: Physiological Integrity: Physiological Adaptation
Integrated Process: Nursing Process
Reference: p. 919

25. Which disorder is a concern in adulthood as well as in childhood for the person
who is overweight or obese during childhood?
A. anorexia
B. alcoholism
C. diabetes mellitus
D. chronic asthma

Answer: C

Rationale: Type 2 diabetes mellitus, which formerly was seen almost exclusively in
adults and is associated with being overweight, is now being diagnosed in childhood
with long-range health concerns.
Question format: Multiple Choice
Chapter 42: The Child with a Psychosocial Disorder
Cognitive Level: Remember
Client Needs: Physiological Integrity: Reduction of Risk Potential
Integrated Process: Nursing Process
Reference: p. 921

26. The nurse recognizes in working with children that the child who has polyphagia
will likely exhibit which of the following?
A. abuse of several drugs
B. overuse of laxatives
C. compulsive overeating
D. withdrawal symptoms

Answer: C

Rationale: Some children experience polyphagia (compulsive overeating). They lack


control of their food intake, cannot postpone their urge to eat, hide food for later
secret consumption, eat when not hungry or to escape from worries, and expend a
great deal of energy thinking about securing and eating food.
Question format: Multiple Choice
Chapter 42: The Child with a Psychosocial Disorder
Cognitive Level: Understand
Client Needs: Psychosocial Integrity
Integrated Process: Nursing Process
Reference: p. 922

27. The nurse is working with a group of caregivers of adolescents who have
abused substances. One of the caregivers states, "I have heard that some drugs
have the effect of causing an excited feeling and it makes the person more social."
Which drug would most likely have the effect this caregiver is describing?
A. barbiturates
B. anabolic steroids
C. cocaine
D. marijuana

Answer: C

Rationale: The psychological effect of using cocaine is a feeling of euphoria and


increased sociability.
Question format: Multiple Choice
Chapter 42: The Child with a Psychosocial Disorder
Cognitive Level: Apply
Client Needs: Psychosocial Integrity
Integrated Process: Teaching/Learning
Reference: p. 915

28. The nurse is doing an in-service training with nurses who will be working with
children who have concerns with drugs and alcohol. In discussing this topic, one of
the nurses says that sometimes a person has a compulsive need to use a substance
for its satisfying or pleasurable effects. Which term best describes what the nurse
is referring to?
A. withdrawal symptoms
B. dependence
C. tolerance
D. substance abuse

Answer: B
Rationale: The use or abuse of substances can lead to addiction or dependence (a
compulsive need to use a substance for its satisfying or pleasurable effects).
Dependence may be psychological, physical, or both.
Question format: Multiple Choice
Chapter 42: The Child with a Psychosocial Disorder
Cognitive Level: Understand
Client Needs: Psychosocial Integrity
Integrated Process: Teaching/Learning
Reference: p. 913

29. A 10-year-old girl has a pattern of school success, solid peer relationships, and
a healthy family life. The girl's caregiver reports for the past 3 weeks the child has
reported abdominal pain with vomiting and diarrhea, and tells the caregiver she
doesn't feel good just before it's time to go to school. However, after the caregiver
lets her stay home, she is fine by mid-morning. She is also fine on weekends.
Which question should the nurse prioritize to this caregiver?
A. "Have you explored with your child if something is causing her to be afraid to go
to school?"
B. "Does your child have new dental caries or other signs of bulimia?"
C. "Could it be that your child is being abused by another family member or by her
teacher?"
D. "Does your child eat a different breakfast on weekends than the one she eats on
school days?"

Answer: A

Rationale: School phobia may be the child's unconscious reaction to a seemingly


overwhelming problem at school. The parent can unwittingly reinforce school
phobia by permitting the child to stay home. The symptoms of vomiting, diarrhea,
abdominal or other pain, and even a low-grade fever, are genuine and are caused
by anxiety that may approach panic. They disappear with relief of the immediate
anxiety after the child has been given permission to stay home.
Question format: Multiple Choice
Chapter 42: The Child with a Psychosocial Disorder
Cognitive Level: Apply
Client Needs: Psychosocial Integrity
Integrated Process: Caring
Reference: p. 911

30. The nurse is working on forming a contract with a hospitalized adolescent


diagnosed with anorexia nervosa. Which information should the nurse prioritize with
the client when making the contract?
A. Remind the adolescent about the consequences of misbehavior.
B. Stress to the client that he or she is in control of the outcome of the care.
C. Encourage the caregivers to take responsibility for the adolescent.
D. Reward the client after several days of successful behavior.

Answer: B
Rationale: Contract agreements are often recommended for people with eating
disorders. These agreements, which are usually part of a behavioral modification
plan, specify the client's and the staff's responsibilities for the diet, activity
expectations for the client, and other aspects of the client's behavior. The contract
also may spell out specific privileges that can be gained by meeting the contract
goals. This places the client in greater control of the outcome. Some type of reward
should be achieved daily to encourage continued participation in the agreement.
Question format: Multiple Choice
Chapter 42: The Child with a Psychosocial Disorder
Cognitive Level: Apply
Client Needs: Psychosocial Integrity
Integrated Process: Nursing Process
Reference: p. 919

31. The nurse is caring for an adolescent with anorexia nervosa. What intervention
should the nurse include in the care of this client?
A. Encourage the client and the client's family to participate in family therapy.
B. Take daily weights to document weight gain.
C. Encourage at least 30 minutes of exercise daily to help the client feel like he or
she is not gaining weight too fast.
D. Encourage the family to vary their daily routine at home so the client does not
get bored.

Answer: A

Rationale: Both the client and the client's family need to be included in the plan of
care along with therapy sessions to help better treat the adolescent. Clients should
be weighed weekly, not daily. The focus needs to move to better nutrition and a
goal of a 0.5 to 2 lb (0.25 to 1 kg) weekly weight gain. A structured routine of
meals and snacks needs to be established. Daily exercise is not encouraged.
Question format: Multiple Choice
Chapter 42: The Child with a Psychosocial Disorder
Cognitive Level: Apply
Client Needs: Psychosocial Integrity
Integrated Process: Nursing Process
Reference: p. 919

32. A parent expresses concern that the child has an autism spectrum disorder. The
nurse obtains a health history of the symptoms to understand the parent's concern.
Which question would the nurse ask first?
A. "Does your child interact with you when playing?"
B. "Does your child only eat the same food?"
C. "Does your child respond to his or her name?"
D. "Is your child happy when friends come to visit?"

Answer: A
Rationale: Children on the autism spectrum have a very wide range of symptoms.
Children have impaired social interactions and communication. They may have
stereotypical and preservative behaviors. Two indicators a child is on the autism
spectrum is the lack of ability to point at objects by 18 months of age and the lack
of ability to gaze jointly with another. By asking if the child interacts with the
parent when playing, the nurse is seeking to determine these two cardinal signs. All
of the answer choices are correct questions to ask as part of the history because
the child could have a multitude of symptoms. These answer choices help provide
information into the diagnosis.
Question format: Multiple Choice
Chapter 42: The Child with a Psychosocial Disorder
Cognitive Level: Apply
Client Needs: Health Promotion and Maintenance
Integrated Process: Nursing Process
Reference: p. 908

You might also like